Download as pdf or txt
Download as pdf or txt
You are on page 1of 1511

Jaypee Brothers Medical Publishers (P) Ltd

Headquarters
Jaypee Brothers Medical Publishers (P) Ltd
4838/24, Ansari Road, Daryaganj
New Delhi 110 002, India
Phone: +91-11-43574357
Fax: +91-11-43574314
Email: [email protected]
Overseas Office
J.P. Medical Ltd
83 Victoria Street, London
SW1H 0HW (UK)
Phone: +44 20 3170 8910
Fax: +44 (0)20 3008 6180
Email: [email protected]
Website: www.jaypeebrothers.com
Website: www.jaypeedigital.com
© 2020, Gobind Rai Garg and Sparsh Gupta
The views and opinions expressed in this book are solely those of
the original contributor(s)/author(s) and do not necessarily
represent those of editor(s) of the book.
All rights reserved. No part of this publication may be reproduced,
stored or transmitted in any form or by any means, electronic,
mechanical, photocopying, recording or otherwise, without the prior
permission in writing of the publishers.
All brand names and product names used in this book are trade
names, service marks, trademarks or registered trademarks of
their respective owners. The publisher is not associated with any
product or vendor mentioned in this book.
Medical knowledge and practice change constantly. This book is
designed to provide accurate, authoritative information about the
subject matter in question. However, readers are advised to check
the most current information available on procedures included and
check information from the manufacturer of each product to be
administered, to verify the recommended dose, formula, method
and duration of administration, adverse effects and
contraindications. It is the responsibility of the practitioner to take
all appropriate safety precautions. Neither the publisher nor the
author(s)/editor(s) assume any liability for any injury and/or
damage to persons or property arising from or related to use of
material in this book.
This book is sold on the understanding that the publisher is not
engaged in providing professional medical services. If such advice
or services are required, the services of a competent medical
professional should be sought.
Every effort has been made where necessary to contact holders of
copyright to obtain permission to reproduce copyright material. If
any have been inadvertently overlooked, the publisher will be
pleased to make the necessary arrangements at the first
opportunity. The CD/DVD-ROM (if any) provided in the sealed
envelope with this book is complimentary and free of cost. Not
meant for sale.
Inquiries for bulk sales may be solicited at:
[email protected]
Assistant Editors: Mrs Praveen Kumari, Mrs Krishna Gupta

Review of Pathology and Genetics


First Edition: 2009 Second Edition: 2010
Third Edition: 2011 Fourth Edition: 2012
Fifth Edition: 2013 Sixth Edition: 2014
Seventh Edition: 2015 Eighth Edition: 2016
Ninth Edition: 2017 Tenth Edition: 2018
Eleventh Edition: 2019 Twelfth Edition: 2020
ISBN: 978-93-89776-82-9
We acknowledge the support shown by each of our esteemed readers (present
and past) for placing the book at the numero uno position amongst the
pathology books for PGMEE. We are also highly grateful for the wonderful
response to the lecture DVD on immunology. To meet the expectations of
students, we have tried to further improve this twelfth edition.
The book has been fully colored to increase its appeal to the esteemed
readers.
After the launch of Mobile Application “Pharmacology by Dr Gobind Rai
Garg” (that contains video lectures of entire Pharmacology), we received a large
number of requests from students to prepare the similar App in Pathology. To
meet the demand of the students, Dr Sparsh Gupta has launched his Mobile
Application on Pathology “Pathology by Dr Sparsh Gupta”(that contains video
lectures of entire Pathology). It is now available to download.
Dear friends, the apprehension regarding the National Eligibility Cum
Entrance Test (NEET) has now been taken care of as the examination pattern
has not been modified drastically. Cracking the NEET exam and other important
PG examinations require a thorough knowledge and understanding of the
subject. Readers of this book have got an edge over others in the National
Eligibility Cum Entrance Test (NEET) because of the strong theory and
conceptual questions. This along with the key points given under the heading of
various boxes in the chapters has helped many of you to get extremely good
ranks in NEET PG entrance examinations.
It is a humble request from our side that all the chapters (and not only
general pathology) of this book has to be read by the student so as to maximize
the benefit. This is because many additional concepts and questions asked
frequently in the exam have been explained in systemic pathology chapters.
This is one important point which came as a gist differentiating the people
getting a good rank in NEET versus those who did not.
In our constant endeavor to improvise the book, there has been
incorporation of important additions in almost all chapters along with the new
section of most recent questions. Important new topics pertaining to recent
exam questions (like blood transfusion, genetic diagnosis of disorders,
Updated WHO criteria for Plasma cell cancers etc.) have been incorporated
in this twelfth edition. More than 300 latest pattern questions-answers have
been incorporated in this new addition. The question bank of the every chapter
has been subdivided into smaller portions. It will help students to solve MCQs
after reading the theory of a particular topic of a chapter.
In this twelfth edition, we have added a lot of diagrams and flowcharts to
make learning interesting and easier. Several flow- charts linking clinical and
pathological findings have been added in this addition. Several diagrams to
understand concepts of diseases have been added in this edition.
Another salient feature of this edition is the updating of appropriate
authentic references from standard textbooks particularly in regard to the
controversial questions so that reader gets all the relevant information under
one roof. Other salient features of the current edition are:

• The book includes Most Recent Pattern Questions similar to AIIMS


Nov 2019, AIIMS May 2019, AIIMS Nov 2018, AIIMS May 2018, NEET
Pattern 2020, NEET Pattern 2019. Any resemblance to an actual
question is purely coincidental.
• The book has been fully colored so as to enhance its visual appeal for
the readers.
• The book has been thoroughly updated keeping in sync with the
recent exam questions.
• Important new topics pertaining to recent exam questions (like blood
transfusion, genetic diagnosis of disorders, Updated WHO criteria for
Plasma cell cancers etc.) have been incorporated.
• Large numbers of images as per latest All India and AIIMS pattern
have been added with the theory so as to increase the grasp over
image-based questions being asked in different examinations.
• A new feature “Golden points” has been added before the chapter
review so as to facilitate last minute revision prior to the exams.
• Every chapter has the presence of an annexure containing the
information required for the extra edge in the exams.
• Special boxes “Mnemonic”, “Recent exam questions”, “Key points”
and “Concept” have been placed regularly in the text so as to highlight
important facts.
• Author support in the form of Live Sessions, facebook pages and
videos (Available in Mobile App).

We have fully revised the book and corrected the typographical and some
other errors present in the previous editions. We have also added plenty of vital
information under the heading of “Concept”, “Info” and “Subject Links” at
multiple places in almost all the chapters in the book. Further, we have also
expanded some of the old topics.
Preface to the Twelfth Edition
Questions from latest entrance examinations of AIIMS have been added.
Several other questions have been incorporated from PGI, DNB and other state
PG entrance examinations. In some topics, there are contradictions between
different books. In such a situation, we have quoted the text from Harrison’s
Principles of Internal Medicine, 20th edition.
To help the students to understand the subject better, Dr Gobind has
started Ayush Institute of Medical Sciences. Any query regarding the
admission in the same maybe addressed to Dr Gobind at the under mentioned
e-mail ID.
We must admit hereby that despite keeping an eagle’s eye for any
inaccuracy regarding factual information or typographical errors, some mistakes
must have crept in inadvertently. You are requested to communicate these errors
and send your valuable suggestions for the improvement of this book. Your
suggestions, appreciation and criticism are most welcome.

March 2020

Gobind Rai Garg


Sparsh Gupta
E-mail: [email protected][email protected]
Pathology is one of the most difficult and at the same time most important
subject in various postgraduate entrance examinations. As we experienced it
ourselves, most of the students preparing for postgraduate entrance
examinations are in a dilemma, whether to study antegrade or retrograde.
Antegrade study takes a lot of time and due to bulky textbooks, some important
questions are likely to be missed. In a retrograde study, the students are likely to
answer the frequently asked MCQs but new questions are not covered. We have
tried to overcome the shortcomings of both of the methods while keeping the
advantages intact.
In this book, we have given a concise and enriched text in each chapter
followed by MCQs from various postgraduate entrance examinations and other
important questions likely to come. The text provides the advantage of
antegrade study in a short span of time.
After going through the book, it will be easier for the student to solve the
questions of most recent examinations, which are given at the end of the book.
Ninth edition of Robbins is just to strike the Indian market. We have
arranged this edition directly from US and added the references from its text.
Further, important differences between 7th and 8th edition of Robbins have been
mentioned with the relevant questions.
It is very difficult and at times very confusing to remember large number of
pathological features. To make learning easy, several easy to grasp
MNEMONICS have been given throughout the text.
Despite our best efforts, some mistakes might have crept in, which we
request all our readers to kindly bring to our notice.
Your suggestions, appreciation and criticism are most welcome.
Gobind Rai Garg
Sparsh Gupta
E-mail: [email protected]
[email protected]
When emotions are profound, words sometimes are not sufficient to
express our thanks and gratitude. With these few words, we would like to thank
our teachers at University College of Medical Sciences and Guru Teg Bahadur
Hospital, Delhi, for the foundation they helped to lay in shaping our careers.
We are especially thankful to Dr KK Sharma, Ex-Professor and Head,
Department of Pharmacology, UCMS, Delhi who has been a father-figure to
whole of the department.
We would also like to acknowledge the encouragement and guidance of
Prof Sonal Sharma (UCMS), Prof Mrinalini Kotru (UCMS), Prof Sachin Kolte
(VMMC) and Prof Lalita J in the completion of this book.
We feel immense pleasure in conveying our sincere thanks to all the
residents of department of Pharmacology at VMMC, MAMC and ACMS for their
indispensable help and support.
No words can describe the immense contribution of our parents, Ms
Praveen Garg, Ms Ruhee, Mr Nitin Misra, Ms Dhwani Gupta, Mr Rohit Singla
and Mrs Komal Singla, without whose support this book could not have seen the
light of the day.
We would also like to extend our special thanks to Dr Sonal Pruthi (UCMS,
Delhi), Dr Amrita Talwar (ACMS, Delhi),Dr Avinash A, Dr Nikita Mary Baby, Dr
Mamta Nikhurpa, Dr Rihas Mohammed, Dr Anuradha Tiwari (ACMS, Delhi) and
Dr Sandeep Goel (MD Radiodiagnosis, AIIMS).
Although it is impossible to acknowledge the contribution of all individually,
we extend our heartfelt thanks to:
• Dr Nitin Jain, DCH, DNB (Pediatrics, Std), Delhi
• Dr Sushant Verma, MS (General Surgery), MAMC, Delhi
• Dr Kapil Dev Mehta, MD (Pharmacology), UCMS, Delhi
• Dr Saurabh Arya, MD (Pharmacology), UCMS, Delhi
• Dr Deepak Marwah, MD (Pediatrics), MAMC, Delhi
• Dr Shubh Vatsya, MD (Medicine), MAMC, Delhi
• Dr Puneet Dwivedi DA (Std), Hindu Rao Hospital, Delhi
• Dr Sandeep Agnihotri, DVD, Safdarjung Hospital, Delhi
• Dr Harsh Vardhan Gupta MD, Pediatrics (Std), Patiala
• Mr Tarsem Garg, LLB, DM, SBOP
• Dr Pardeep Bansal, MD (Radiodiagnosis), UCMS, Delhi
• Dr Pankaj Bansal, MS (Orthopedics), RML Hospital, Delhi
• Dr Pradeep Goyal, MD (Radiodiagnosis), LHMC, Delhi
• Dr Rakesh Mittal, MS (Surgery), Safdarjung Hospital, Delhi
• Dr Amit Miglani, DM (Gastroenterology), PGI, Chandigarh
• Dr Sachin Gupta DA, DMC (Ludhiana)
• Dr Reenu Gupta DGO BMC (Bangalore)
• Dr Shiv Narayan Goel, MCh (Urology), KEM, Mumbai
• Dr Gaurav Jindal, MD, Radiodiagnosis Resident, Boston, USA
• Dr Saket Kant, MD (Medicine UCMS), DM (Endocrinology, BHU)
• Dr Mukesh Kr Joon, DM (Cardiology), Udaipur, Rajasthan
• Dr Amit Garg, Assistant Professor (Psychiatry), IHBAS, Delhi
• Dr Garima Mahajan, MD (Pathology), UCMS, Delhi
• Dr Ravi Gupta, MD (Psychiatry), Jolly Grants Medical College,
Dehradun, Uttarakhand
• Dr Shashank Mohanty, MD (Medicine), Udaipur, Rajasthan
Review of Pathology and Genetics
• Dr Amit Shersia, MS (Orthopedics), MAMC, Delhi
• Dr Mohit Gupta, DCP, DNB (Pathology), Delhi
• Dr Mayank Dhamija, DCH, DNB (Pediatrics), DNB (Hemato-oncology),
Delhi
Last but not the least, we would like to thank Shri Jitendar P Vij (Group
Chairman), M/s Jaypee Brothers Medical Publishers (P) Ltd, New Delhi, India,
publishers of this book and the entire PGMEE team, for their keen interest,
innovative suggestions and hardwork in bringing out this edition.
March 2020
Gobind Rai Garg
Sparsh Gupta
From the Publisher’s Desk
We request all the readers to provide us their
valuable suggestions/errors (if any) at:
[email protected]
so as to help us in further improvement of this book
in the subsequent edition
• Dr Swapnil Aggarwal, MBBS (LHMC), MD (MAMC)
• Dr Mohit Gupta, DCP, DNB
• Dr Garima Mahajan, MD, DNB (UCMS)
• Dr Amandeep Kaur, MD, (PGI Chandigarh)
• Dr Paritosh Garg, MD (UCMS)
• Dr Ashish K DM (AIIMS)
• Dr Ini Nair, MD Kochi

• Robbin’s Pathological Basis of Diseases, 9th edition


• Harsh Mohan’s Textbook of Pathology, 8th edition
• Harrison’s Principles of Internal Medicine, 20th edition
• Current Medical Diagnosis and Treatment 2020
• Wintrobe’s Clinical Hematology, 13th edition
• Ackerman’s Surgical Pathology, 10th edition
• Sternberg’s Diagnostic Surgical Pathology, 6th edition

SYMBOLS USED IN BOXES ON ‘HIGH YIELD POINTS’

• Key points

• Definition

• Mnemonic

• Concept

• Questions asked in most recent exam


1. For second prof students:
It is preferable to begin reading this book during your second year
MBBS to build your basics right from the beginning.

ò
Read the theory of a chapter from this book and then read the
textbook. You will be able to easily understand the textbook now.

ò
Now read the theory of that chapter once again.

ò
Now solve the MCQ from the book.

ò
Follow this with another reading from the textbook.
• This completes your chapter with one reading and revision.
• While reading the book, either make notes or mark in the book
itself for quick revision. Mark the difficult and important MCQ for
further revision.
• Do this for all the chapters.
• After completing the syllabus, start revising.
• Remember minimum 4-5 readings are required as Pathology is a
volatile subject.
2. For students who have passed second prof (final year students,
interns and post-interns):
• We do not recommend studying textbook now due to paucity of
time. However, text book should be kept as a reference material.
Do revise the images from Robbins along with the footnotes time
and again.
• Do not confuse yourself by studying many books.
• You should spend 10-15 days for Pathology for first reading.
Read the theory of a chapter and solve MCQ of that chapter. While
solving MCQs, solve minimum 50-100 questions at a stretch and only after
this compare the answers.
Re-read the theory of this chapter and now mark the important points
for revision. Remember, you should mark only that much so that the next
reading of book can be finished in one third of the time. Similarly, encircle
or mark the important MCQ for revision.
Do same for all the chapters.
During revision, study only marked portion with encircled MCQ only.
Second reading should be finished in 4-5 days.
Similarly third and fourth revision should be completed in 3 days each.
Give one last revision just before exams in a day or two.
• Remember, Pathology is a very important subject. You can answer
nearly 35-40 questions in NEET from this book as it covers not
only Pathology but many related subjects
Best wishes
Authors

Important Notes of New Chapter Added in Robbins 9th Edition


1. Cell Injury
Most Essential Topics: Necrosis, Apoptosis, Lipofuscin and
Calcification««
Chapter Review
Multiple Choice Questions
Explanations
Most Recent Pattern Questions with Answer
2. Inflammation

Most Essential Topic: Wound heading««


Chapter Review
Multiple Choice Questions
Explanations
Most Recent Pattern Questions with Answer 57

3. Hemodynamics«
Chapter Review
Multiple Choice Questions
Explanations
Most Recent Pattern Questions with Answer
4. Genetics
Most Essential Topics: Pedigree analysis(JIPMER and AIIMS) and non
classical inheritance«««
Chapter Review
Multiple Choice Questions
Explanations
Most Recent Pattern Questions with Answer
5. Neoplasia

Most Essential Topics: Genes and Diagnosis of Tumors«««


Chapter Review
Multiple Choice Questions
Explanations
Most Recent Pattern Questions with Answer
6. Immunity

Most Essential Topics: Amyloidosis and Autoimmune disorders«««


Chapter Review
Multiple Choice Questions
Explanations
Most Recent Pattern Questions with Answer
Review of Pathology and Genetics
7. Anemia and Red Blood Cells
Most Essential Topics: Hemolytic Anemia««
Chapter Review
Multiple Choice Questions
Explanations
Most Recent Pattern Questions with Answer 256
8. White Blood Cells
Most Essential Topics: Leukemia and Lymphoma««
Chapter Review
Multiple Choice Questions
Explanations
Most Recent Pattern Questions with Answer
9. Platelets and Blood Transfusion
Most Essential Topics: Bleeding disorders«« Blood Transfusion«««
Chapter Review
Multiple Choice Questions
Explanations
Most Recent Pattern Questions with Answer
10. Cardiovascular System««
Chapter Review
Multiple Choice Questions
Explanations
Most Recent Pattern Questions with Answer
11. Respiratory System««
Chapter Review
Multiple Choice Questions
Explanations
Most Recent Pattern Questions with Answer
12. Kidney and Urinary Bladder
Most Essential Topics: Kidney Theory and Biopsy findings«««
Chapter Review
Multiple Choice Questions
Explanations
Most Recent Pattern Questions with Answer
13. Gastrointestinal Tract««
Chapter Review
Multiple Choice Questions
Explanations
Most Recent Pattern Questions with Answer
14. Liver««
Chapter Review
Multiple Choice Questions
Explanations
Most Recent Pattern Questions with Answer
Contents
15. Genital System and Breast
Most Essential Topics: Breast (Molecular classification and prognostic
factors)«««
Chapter Review
Multiple Choice Questions
Explanations
Most Recent Pattern Questions with Answer
16. Central Nervous System
Most Essential Topic: Tumors«««
Chapter Review
Multiple Choice Questions
Explanations
Most Recent Pattern Questions with Answer
17. Endocrine System
Most Essential Topics: Thyroid and Diabetes««
Chapter Review
Multiple Choice Questions
Explanations
Most Recent Pattern Questions with Answer
18. Musculoskeletal System
Most Essential Topics: Skeletal Muscle Disorders and Bone««
Chapter Review
Multiple Choice Questions
Explanations
Most Recent Pattern Questions with Answer
19. Miscellaneous«
Chapter Review
Multiple Choice Questions
Explanations
20. Important Stains and Bodies
IMPORTANT NOTES OF NEW
CHAPTER ADDED IN ROBBINS
9TH EDITION
CELL AS A UNIT OF HEALTH AND DISEASE

• Virchow coined the term ‘cellular pathology’


• The human genome contains roughly 3.2 billion DNA base
pairs and only about 2% is used for coding of proteins.
• 80% of the human genome either binds proteins, implying
it is involved in regulating gene expression, or can be
assigned some functional activity, mostly related to the
regulation of gene expression, often in a cell-type specific
fashion.
• The two most common forms of DNA variation in the
human genome are single-nucleotide polymorphisms
(SNPs) and copy number variations (CNVs).
• SNPs are variants at single nucleotide positions and are
almost always biallelic (i.e., only two choices exist at a given
site within the population, such as A or T).
• CNVs are a more recently identified form of genetic variation
consisting of different numbers of large contiguous stretches
of DNA from 1000 base pairs to millions of base pairs.
• Epigenetics is defined as heritable changes in gene
expression that are not caused by alterations in DNA
sequence.
• Nuclear chromatin exists in two basic forms:
1. Cytochemically dense and
transcriptionally inactive heterochromatin
and
2. Cytochemically dispersed and
transcriptionally active euchromatin
• Different histone modifications are generically called as
marks. The modifications include methylation, acetylation, or
phosphorylation of specific amino acid residues on the
histones.
Gene regulation can also be done through noncoding RNAs which
can be of the following subtypes:
a. MicroRNAs (miRNA): The miRNAs are small RNA molecules
22 nucleotides in length which do not encode proteins. They
function primarily to modulate the translation of target mRNAs
into their corresponding proteins.
b. Long noncoding RNAs (lncRNA): RNAs are >200
nucleotides in length. Its example includes XIST, which is
transcribed from the X chromosome and plays an essential
role in physiologic X chromosome inactivation.
MiRNA associated with cancers are called oncomiRs. They act by
increasing the number of cancer causing genes and suppress the
tumor suppressor genes.
The non coding RNAs fall into several classes:
• Piwi-interacting RNAs (piRNAs), the most common type of
small noncoding RNA, which (like miRs) are believed to have
a role in post-transcriptional gene silencing;
• Sno RNAs, which are important in maturation of rRNA and
the assembly of ribosomes; and
• Long intervening noncoding RNAs (lincRNAs), some of
which regulate the activity of chromatin “writers,” the factors
that modify histones and thereby control gene expression
CELLULAR HOUSEKEEPING (as per 9th edition of Robbins)

Organelles Key points

Mitochondria Oxidative phosphorylation


Intermediates for heme synthesis
Intrinsic pathway of Apoptosis (programmed cell death)

Smooth Abundant in gonads and liver


endoplasmic Used for lipoprotein and steroid hormone synthesis,
reticulum Required for converting the hydrophobic compounds like drugs
(SER) into water-soluble molecules
Sequestration of calcium
Organelles Key points

Proteasomes Required for selectively chewing of denatured proteins using


ubiquitin.
Also needed for presentation of peptides in context of the
class I major histocompatibility molecules

Peroxisomes Breakdown of fatty acids

Plasma Phosphatidylinositol serves as scaffold for intracellular proteins


membrane as well as for the generation of intracellular second signals like
proteins diacylglycerol and inositol trisphosphate.
Phosphatidylserine is required for apoptosis (programmed cell
death) and on platelets, it serves as a cofactor in the clotting of
blood.
Glycolipids are important in cell-cell and cell-matrix
interactions, including inflammatory cell recruitment and
sperm-egg interactions.

Lysosomes Most cytosolic enzymes prefer to work at pH 7.4 whereas


lysosomal enzymes function best at pH 5 or less.

Golgi Mannose 6 phosphateQ is the marker


apparatus

• Channel proteins create hydrophilic pores, which,


when open, permit rapid movement of solutes
(usually restricted by size and charge)
• Carrier proteins bind their specific solute and undergo
a series of conformational changes to transfer the
ligand across the membrane; their transport is
relatively slow.
• Exocytosis is the process by which large molecules are
exported from cells. In this process, proteins
synthesized and packaged within the RER and Golgi
apparatus are concentrated in secretory vesicles, which
then fuse with the plasma membrane and expel their
contents.
• Transcytosis is the movement of endocytosed vesicles
between the apical and basolateral compartments of
cells. It is a mechanism for transferring large amounts
of intact proteins across epithelial barriers.
• Potocytosis is literally “cellular sipping.” whereas
pinocytosis is “cellular drinking”
• Endocytosis is the uptake of fluids or macromolecules
by the cell. It could be of the following types:
1. Caveolae-mediated endocytosis: CaveolinQ is the
major structural protein of caveole. Internalization of
caveolae with any bound molecules and associated
extracellular fluid is sometimes called potocytosis—
literally “cellular sipping.”
2. Pinocytosis and receptor mediated endocytosis:
Pinocytosis (“cellular drinking”) describes a fluid-phase
process during which the plasma membrane
invaginates and is pinched off to form a cytoplasmic
vesicle. Receptor-mediated endocytosis is the major
uptake mechanism for certain macromolecules like
transferrin and low-density lipoprotein (LDL).
• Most cytosolic enzymes prefer to work at pH 7.4
whereas lysosomal enzymes function best at pH 5 or
less.

Cytoskeleton
The ability of cells to adopt a particular shape, maintain polarity,
organize the relationship of intracellular organelles, and move
about depends on the intracellular scaffolding of proteins called
the cytoskeleton. The three major classes of cytoskeletal proteins
are:
i. Actin microfilaments are 5- to 9-nm diameter fibrils
formed from the globular protein actin (G-actin), the
most abundant cytosolic protein in cells.
ii. Intermediate filaments are 10-nm diameter fibrils that
impart tensile strength and allow cells to bear
mechanical stress. The examples include:
• Lamin A, B, and C: nuclear lamina of all cells
• Vimentin: mesenchymal cells (fibroblasts, endothelium)
• Desmin: muscle cells, forming the scaffold on which
actin and myosin contract
• Neurofilaments: axons of neurons, imparting strength
and rigidity
• Glial fibrillary acidic protein: glial cells around neurons
• Cytokeratins: 30 different types are present, hence can
be used as cell markers

Clinical significance!
Since they have characteristic tissue-specific patterns of
expression, they are useful for assigning a cell of origin for poorly
differentiated tumors.
iii. Microtubules: these are 25-nm-thick fibrils composed
of non-covalently polymerized dimers of α- and β-
tubulin arrayed in constantly elongating or shrinking
hollow tubes with a defined polarity. Within cells,
microtubules are required to move vesicles, organelles,
or other molecules around cells along microtubules.
There are two varieties of these motor proteins:
kinesins (for anterograde transport) and dyneins (for
retrograde transport).
Mitochondrial function: key points

• Intermembrane space in the mitochondria is the chief


site of ATP synthesis.
• Thermogenin is an inner membrane protein which is
used to generate heat by uncoupling electron transport
chain with ATP generation. It is present in high
concentration in brown fat and is useful to generate
heat by non-shivering thermogenesis.
• Warburg effect: it is the phenomenon in which rapidly
growing cells (both benign and malignant) upregulate
glucose and glutamine uptake and decrease their
production of ATP per glucose molecule. This is
responsible for providing metabolic intermediates
which are useful for cellular growth and maintenance.

Receptors
Cell-surface receptors are generally transmembrane proteins with
extra cellular domains that bind soluble secreted ligands. They
can be of the following types:
1. Ion channels (typically at the synapse between
electrically excitable cells)
2. G protein coupled receptors: activate an associated
GTP-binding regulatory protein
3. Enzymatic receptors: activate an associated enzyme
usually tyrosine kinase
4. Receptors which trigger a proteolytic event or a change
in protein binding or stability that activates a latent
transcription factor. Examples include Notch, Wnt, and
Hedgehog receptors which regulate normal
development.

Transcription factors

• MYC and JUN are the transcription factors that


regulate the expression of genes that are needed for
growth.
• p53 is a transcription factor that triggers the expression
of genes that lead to growth arrest.

Summary of growth factors and the receptors


The major role of growth factors is to stimulate the activity of
genes that are required for cell growth and cell division. They are
also involved in the non-growth activities, including migration,
differentiation, and synthetic capacity. Some important examples
include:
a. Epidermal Growth Factor and Transforming Growth
Factor-α.
The “EGF receptor family” includes four membrane
receptors with intrinsic tyrosine kinase activity. The
examples include EGFR1 involved in lung cancer, head
and neck, breast etc. and the ERBB2 receptor (also
known as HER2) involved in breast cancer
b. Hepatocyte Growth Factor (also known as scatter
factor)
HGF acts as a morphogen in embryonic development,
promotes cell migration and enhances hepatocyte survival.
MET is the receptor for HGF, it has intrinsic tyrosine kinase
activity and is frequently over-expressed or mutated in
tumors, particularly renal and thyroid papillary
carcinomas.
c. Platelet-Derived Growth Factor
PDGF is stored in platelet granules and is released on platelet
activation.
d. Vascular Endothelial Growth Factor
• VEGF-A is the major angiogenic factor (inducing
blood vessel development) after injury and in tumors.
• VEGF-B and PlGF (placental growth factor) are
involved in embryonic vessel development, and VEGF-
C and -D stimulate both angiogenesis and lymphatic
development (lymphangiogenesis).
• In adults, VEGFs are also involved in the maintenance
of normal adult endothelium and not involved in
angiogenesis.
• Hypoxia is the most important inducer of VEGF
production.
• VEGFR-2 is highly expressed in endothelium and is
the most important for angiogenesis.
• Anti-VEGF antibodies are being used for a number of
ophthalmic diseases including “wet” age-related
macular degeneration, the angiogenesis associated
with retinopathy of prematurity; and diabetic macular
edema.
e. Fibroblast Growth Factor (FGF-7)
• FGF-7 is also referred to as keratinocyte growth
factor (KGF).
f. Transforming Growth Factor-β
TGF-β has multiple and often opposing effects depending on the
tissue and concurrent signals. Agents with such multiplicity of
effects are called pleiotropic.
• TGF-β is involved in scar formation after injury. It
also drives fibrosis in lung, liver, and kidneys in
conditions of chronic inflammation.
• TGF-β is an anti-inflammatory cytokine that serves
to limit and terminate inflammatory responses.

Extracellular matrix
• Laminin is the most abundant glycoprotein in
basement membrane
• The major constituents of basement membrane are
amorphous nonfibrillar type IV collagen and laminin.
• Collagens are typically composed of three separate
polypeptide chains braided into a ropelike triple helixQ.
• Hypoxia is the most common cause of cell injury.
• Neurons are the most sensitive cell to hypoxic injury in the
brain.
• Coagulative necrosis is associated with “tombstone
appearance”. It is seen with ischemic injury to all tissues
except central nervous system.
• Caseous necrosis is caused by: TB (most common),
syphilis, fungus (Histoplasmosis, Coccidiodomycosis).
• Best example of coexistence of hypertrophy and hyperplasia
is uterus during pregnancy (gravid uterus).
• Most common metaplasia is squamous metaplasia in the
lungs of smokers.
• Sign of reversible cell injury in alcoholic liver disease:
Cytoplasmic lipid vacuole.
• CD 95 plays a role in apoptosis (extrinsic pathway).
• Mitochondria plays a pivotal role on apoptosis.
• Marker for apoptosis (programmed cell death) is annexin V.
• Most important stimulatory gene for apoptosis is p53
gene and most important inhibitory gene for apoptosis is bcl
family (bcl-2) of genes.
• Keywords associated with apoptosis: caspases, cytochrome
C and embryogenesis.
• ‘Chromatin condensation’ is the hallmark feature of
apoptosis.
• “Step ladder pattern” on gel electrophoresis is a feature of
apoptosis. Stepladder fever is seen in typhoid/enteric fever.
• Intranucleosomal cleavage of DNA is characteristic of
Apoptosis.
• Anticancer drugs (chemotherapeutic agents) can cause:
Both necrosis and apoptosis.
• Important example of apoptotic bodies: Councilman
bodies, civatte bodies, kamino bodies, Tangible bodies.
• Mitochondrial abnormality is seen in Oncocytoma.
• Steatosis means Fatty change due to accumulation of
triglyceride.
• Caspases are involved in: Apoptosis
(organogenesis/morphogenesis).
• Lipofuscin is also known by several other names like
‘lipochrome’, ‘wear and tear’ pigment, pigment of aging
and “indicator of free radical injury”. It gets deposited
mostly in heart and liver.
• The endogenous brown-black pigments include melanin
(present in skin) and homogentisic acid (the black pigment
in patients with alkaptonuria).
• Dystrophic calcification: normal serum calcium levels and
in dead tissues (areas of necrosis).
• Metastatic calcification: increased serum calcium levels
and in living tissues.
• Suprasellar calcification is always a pathological
calcification.
• Oncocytes are seen in: Salivary glands, thyroid, parathyroid,
kidney, lung, pituitary, and pancreas.
• “Lungs” are the commonest site for metastatic calcification.
Other sites include stomach, pulmonary vein, systemic artery
and kidneys.
• Psammoma bodies: meningioma, papillary thyroid
carcinoma, prolactinoma, glucagonoma and serous
cystadenoma of the ovary.
• Gandy gamma body is seen in congestive splenomegaly.
It contains hemosiderin and calcium.
• Oncocytes are formed with modified mitochondria.
• Germ cells have the capacity for self renewal because of
telomerase activation.
• Cancer cells have the phenomenon of ‘telomerase
reactivation’.
• Germ cells have the maximum telomerase activity amongst
all the cells of the body.
• ‘Not’ seen in cell aging: Increased free radical injury,
increased somatic mutation, decreased number of
mitochondria & cells, cross-linkage of collagen shortening of
telomeres, glycosylation of proteins.
• Cell cannibalization required for self survival is called
autophagy. In Alzheimer disease, formation of
autophagosomes is accelerated and in Huntington
disease, mutant huntingtin impairs autophagy.
• Necroptosis is a caspase independent process which
resembles necrosis morphologically and apoptosis
mechanistically as a form of programmed cell death. It is also
called “programmed necrosis”.
• Pyroptosis is a programmed cell death is accompanied by
the release of fever inducing cytokine IL-1. It also involves
caspases 1 and 11.
• Commonest fixative used for light microscopic examination:
10% buffered neutral formalin.
• Commonest fixative used for electron microscopic
examination: glutaraldehyde.
• Fenton’s reaction leads to free radical generation when:
Iron is converted from ferrous to ferric form.
• Haber-Weiss reaction is: Generation of free radical from
H2O2.
• Enzymes that protect against free radical damage:
Superoxide dismutase (SOD), catalase, glutathione
peroxidase.

Pathology is a science dealing with the study of diseases. Four


important components of pathology are etiology (causative factors),
pathogenesis (mechanism or process by which disease develops),
morphology (appearance of cells, tissues or organs) and clinical
features.

CELL INJURY

Disease occurs due to alteration of the functions of tissues or cells at


the microscopic level. The various causes of cell injury include:
1. Hypoxia: It is the most common cause of cell injury. It results
due to decrease in oxygen supply to the cells. Hypoxia may be
caused by

a. Ischemia: Results due to decrease in blood supply. It is the most


common cause of hypoxiaQ
b. Anemia: Results due to decrease in oxygen carrying capacity of blood
c. Cardio-respiratory disease: Results from decreased oxygenation of
blood due to cardiac or respiratory disease.

2. Physical Agents: Cell injury may occur due to radiation


exposure, pressure, burns, frost bite etc.
3. Chemical Agents: Many drugs, poisons and chemicals can result
in cell injury.
4. Infections: Various infectious agents like bacteria, virus, fungus
and parasites etc can cause cell injury.
5. Immunological reactions: These include hypersensitivity
reactions and autoimmune diseases.
6. Genetic causes: Cell injury can also result due to derangement
of the genes.
7. Nutritional imbalance: Cell injury can result due to deficiency of
vitamins, minerals etc.

In response to injury, a cell/tissue can have following


consequences:
• Adaptation: The cell changes its physiological functions in
response to an injurious stimulus.
• Reversible cell injury
• Irreversible cell injury.
1. REVERSIBLE CELL INJURY: As already discussed, hypoxia
is the most common cause of cell injury. Oxygen is an
important requirement of mitochondria for the formation of
ATP; therefore, hypoxia will result in earliest involvement of
mitochondriaQ resulting in decreased formation of ATP. All
cellular processes requiring ATP for normal functioning will be
affected. Important organelles affected are cell membranes
(require ATP for functioning of Na+ - K+ pump), endoplasmic
reticulum (require ATP for protein synthesis) and nucleus.

• Swelling of organelles like endoplasmic reticulum results in decreased


protein synthesis
• Bleb formation results due to outpouching from the cell membrane to
accommodate more water.
• Loss of microvilli
• Formation of myelin figures due to breakdown of membranes of cellular
organelles like endoplasmic reticulum. These are composed of
phospholipidsQ. Myelin figures are intracellular whorls of laminated lipid
material (resembling myelin of nerves). When these are present in
membrane bound structures containing lysosomal enzymes, these are
known as myeloid bodies or myelinoid bodies.

Fig. 1: Hydropic change in kidney.

All the features discussed above are of reversible cell injury


because if the injurious agent is removed at this point, cell
can recover back to its normal state of functioning. However,
if the stimulus continues, then irreversible cell injury ensues.
2. IRREVERSIBLE CELL INJURY: Features of irreversible cell
injury include
– Damage to cell membrane: It results due to continued
influx of water, loss of membrane phospholipids and loss
of protective amino acids (like glycine). Damage to cell
membranes result in massive influx of calcium.
– Calcium influx: Massive influx of Ca2+ results in the
formation of large flocculent mitochondrial densities and
activation of enzymes.
– Nuclear changes: These are the most specific
microscopic features of irreversible cell injury. These
include: *Pyknosis (nuclear condensation),
*Karyorrhexis (fragmentation of the nucleus) and
*Karyolysis (nuclear dissolution).
Fig. 2: Liquefactive necrosis in CNS. ...(All India Image)

Irreversible cell injury may be necrosis or apoptosis


(Programmed cell death)
Fig. 3: Fat necrosis with saponification.

APOPTOSIS

Apoptosis or programmed cell death can be induced by intrinsic or


extrinsic pathway. Normally, growth factors bind to their receptors in
the cells and prevent the release of cytochrome C and SMAC. So,
withdrawal or absence of growth factors can result in release of these
mediators and initiate the intrinsic pathway.

Intrinsic pathway: It is initiated by the release of cytochrome C


and SMAC (second mitochondrial activator of caspases) from the
mitochondrial inter-membrane space. Cytoplasmic cytochrome C
stimulates APAF-1 (apoptosis activating factor -1) leading to
sequential activation of caspase-9 and effector caspases {Caspases-
3 and -7}. On the other hand, the released SMAC binds and blocks
the function of caspases inhibitor IAPs (Inhibitor of Apoptosis
Proteins).
Extrinsic pathway: It is activated by binding of Fas ligand to
CD95 (Fas; member of TNF receptor family) or binding of TRAIL (TNF
related apoptosis inducing ligand) to death receptors DR4 and DR5.
This induces the association of FADD (Fas- associated death domain)
and procaspase-8 to death domain motifs of the receptors resulting in
activation of caspase 8 (in humans caspase 10) which finally activates
caspases- 3 and 7 that are final effector caspases. A cellular
proteins called FLIP, binds to procaspase-8 but can not activate it.
This is important because some viruses produce homologues of FLIP
and protect themselves from Fas mediated apoptosis.

Fig. 4: Apoptotic body.


Mnemonic: Short Story to understand the pathogenesis of apoptosis.
Suppose, a person is working in some institution. If he leaves his job, this will
be equivalent to apoptosis. There are two reasons due to which that person
can leave the work. 1. This person is fired from work (equivalent to extrinsic
pathway through death receptors). 2. Person is not given pay for long time,
so that the person himself gives resignation (equivalent to intrinsic pathway,
due to absence of growth factors). In latter case, before giving resignation,
the person will talk to his colleagues, whether he should leave or not. Some
of them will suggest him to leave (equivalent to pro-apoptotic gene products
like bak, bid etc.) and some of them will stop him and suggest to wait
(equivalent to anti-apoptotic factors like bcl-2, bcl-xL etc.) This regulation has
been discussed below.

REGULATION OF APOPTOSIS

Regulation is primarily by bcl-2 family of genes located on


chromosome 18. Some members of this family like bak, bid, bin, bcl-
xS (to remember, S for stimulate apoptosis) stimulate apoptosis
whereas others like bcl-2, bcl-xL (to remember, L for lower apoptosis)
etc inhibit apoptosis.
Normal cells have bcl-2 and bcl-xL present in the mitochondrial
membrane. They inhibit apoptosis because their protein products
prevent the leakage of mitochondrial cyt ‘c’ into the cytoplasm. When
there is absence of growth factors or hormones, bcl-2 and bcl-xL are
replaced by bax, bin etc. resulting in increased permeability of
mitochondrial membrane. This result in stimulation of intrinsic pathway
of apoptosis (described above in flowchart).
Fig. 5: Syndactyly. ...(All India Image)

EXAMPLES OF APOPTOSIS
Physiological conditions Pathological conditions
1. Endometrial cells (Menstruation) 1. Councilman bodies: Viral
2. Cell removal during embryogenesis hepatitis
(see Figure 5) 2. Gland atrophy following duct
3. Virus infected cells and Neoplastic cells obliteration as in cystic fibrosis
by cytotoxic T cells 3. Graft versus host disease
(GVHD)

Diagnosis of apoptosis: Special topics for PG


1. Chromatin condensation seen by hematoxylin, Feulgen and acridine
orange staining.
2. Estimation of cytochrome ‘c’
3. Estimation of activated caspase
4. Estimation of Annexin V (apoptotic cells express phosphatidylserine
on the outer layer of plasma membrane because of which these cells
are recognized by the dye Annexin VQ. Some cells also express high
concentration of thrombospondin).
5. DNA breakdown at specific sites can be detected by ‘step ladder
pattern’ on gel electrophoresis or TUNEL (TdT mediated d-UTP Nick
End Labelling) technique.
Clinical Significance of apoptosis in cancers

Mutated cells are cleared normally in the body by apoptosis but in cancers,
apoptosis is decreased. Commonly it could be due to mutation in p53 gene
or increased expression of genes like bcl-2. The bcl-2 over expression is
seen with translocation (14;18) preventing the apoptosis of abnormal B
lymphocytes which proliferate then and result in the development of B cell
follicular lymphoma.

Fig. 6: Ladder pattern in apoptosis. ...(AIIMS Image)


Necrosis Apoptosis
• Always pathological • May be physiological or
pathological
• Associated with disruption of cellular • Important for development,
homeostasis (e.g. ischemia, hypoxia & homeostasis & elimination of
poisoning) pathogens & tumor cells
• Affects contiguous (adjacent) group of • Affect single cells
cells
• Cell size is increased • Cell size is shrunken
• Passive • Active
• Causes inflammatory reaction • No inflammatory reaction
• Plasma membrane is disrupted • Plasma membrane is intact
• ‘Smear pattern’ on electrophoresis • Step ladder pattern is seen

Mnemonic: Apoptosis can be considered as suicide whereas necrosis as


murder. Like
• Murder is always done by someone else (i.e. pathological) whereas
suicide can be committed by oneself (physiological) or due to undue
pressure (pathological).
• A person can murder many people (affects group of cells) whereas
suicide can be committed only by oneself (affect single cells).
• The person who is being killed doesn’t need to plan or do anything
(passive) whereas for suicide, lot of planning and effort has to be made
(active process).
• When a person is being killed, he will make a lot of efforts to save himself
and thus may lead to accumulation of other people or police (equivalent
to inflammatory mediators coming there) whereas in suicide no help is
called for (no-inflammation).

Other patterns of irreversible cell injury are Necroptosis and


Pyroptosis
1. Necroptosis

• Necroptosis resembles necrosis morphologically and apoptosis mechanistically


as a form of programmed cell death. It is triggered by ligation of TNFR1, and
viral proteins of RNA and DNA viruses.
• Necroptosis is caspase-independent but dependent on signaling by the RIP1
and RIP3 complex.
• RIP1-RIP3 signaling reduces mitochondrial ATP generation, causes production
of ROS, and permeabilizes lysosomal membranes, thereby causing cellular
swelling and membrane damage as occurs in necrosis.
• The release of cellular contents evokes an inflammatory reaction as in
necrosis.

2. Pyroptosis

Pyroptosis occurs in cells infected by microbes. It involves activation of


caspase-1 which cleaves the precursor form of IL-1 to generate biologically active
IL-1. Caspase-1 along with closely related caspase-11 also lead to death of the
infected cell.

3. ADAPTATION: Cells may show adaptation to injury by various


processes like atrophy, hypertrophy, hyperplasia, metaplasia,
dysplasia etc.

Metaplasia Dysplasia
• Reversible change in which one • Abnormal multiplication of cells
differentiated cell type (epithelial or characterized by change in size,
mesenchymal) is replaced by another shape and loss of cellular
cell type. organization
• Results from “reprogramming” of stem • The basement membrane is
cells that are known to exist in normal intactQ
tissues, or of undifferentiated • Can progress to cancer
mesenchymal cells in connective tissue.

INTRACELLULAR ACCUMULATIONS

Various substances like proteins, lipids, pigments, calcium etc. can


accumulate in cells.
1. Proteins: Proteins are synthesized as polypeptides on
ribosomes. These are then re-arranged into a-helix or b sheets
and folded. Chaperones help in protein folding and transportation
across endoplasmic reticulum and golgi apparatus. Chaperones
thus can be induced by stress (like heat shock proteins; hsp 70
and hsp 90). They also prevent ‘misfolding’ of proteins. However,
if misfolding occurs, chaperones facilitate degradation of damaged
protein via ubiquitin-proteasome complex.

Disorders with protein defects

Defect in transport and secretion of


proteins Misfolded/unfolded proteins
Accumulation of proteins inside cells Initially increase chaperone
concentration, Later, these induce
apoptosis by activating caspases
• a1 – Antitrypsin deficiency Q • Alzheimer’s DiseaseQ
• Cystic fibrosis Q • Huntington’s Disease
• Parkinson’s Disease

2. Lipids:
– Triglycerides: Fatty change in liver, heart and kidney (stained
with Sudan IV or Oil Red O).
– Cholesterol: Atherosclerosis, xanthoma
– Complex lipids: Sphingolipidosis

Fig. 7: Lipofuscin (wear and tear pigment).


3. Endogenous Pigments:

Lipofuscin Melanin Hemosiderin


- Perinuclear, brown - Only naturally - Golden yellow pigment
coloured pigment occurring - Seen at sites of
- Responsible for endogenous black hemorrhage or bruiseQ
brown atrophyQ of pigment derived - Also seen in
liver and heart from tyrosineQ hemochromatosisQ (Iron
- It is derived through - Responsible for overload)
lipid peroxidation of pigmentation of skin
membrane lipids. and hair
- Seen in aging,
protein energy
malnutrition and
cancer cachexia.

Fig. 8: Pearl reaction of hemosiderin. ...(AIIMS Image)


4. Hyaline change: It is any intracellular or extracellular
accumulation that has pink homogenous appearance.
Intracellular Extracellular
- Mallory alcoholic hyaline - Hyaline membrane in newborns
- Russell bodies (seen in multiple - Hyaline arteriosclerosis
myeloma) - Corpora amylacea in prostate, brain,
- Zenker’s hyaline change spinal cord in elderly, old lung infarct

INFO: The deposition of such hyaline like material and the associated
sclerosis is important in diseases affecting the kidneys (glomerulopathies).
5. Calcification: Pathologic calcification is the abnormal tissue
deposition of calcium salts, together with smaller amounts of iron,
magnesium, and other mineral salts. It can be of the following two
types:
Dystrophic Metastatic

- Seen in dead tissuesQ - Seen in living tissues also


- Serum calcium is normalQ - Association with elevated serum
- Seen at sites of necrosisQ Ca2+
- Often causes organ dysfunction - Does not cause clinical dysfunction
- Examples include: - Seen in
R – Rheumatic heart disease (in • HyperparathyroidismQ
cardiac valves) • Renal failureQ
A – Atheromatous plaque • Vitamin D intoxicationQ
T – Tubercular lymph node • SarcoidosisQ
Tumors (MOST for PG) • Milk alkali syndromeQ
• M – Meningioma, Mesothelioma • Multiple myelomaQ
• O – Papillary carcinoma of
• Metastatic tumors to boneQ
Ovary (serous ovarian
- Found in organs which loose acid
cystadenoma)
and have alkaline environment
• S – Papillary carcinoma of
inside them [like lungs (most
Salivary gland
commonly), kidneys, stomach,
• T – Papillary carcinoma of
systemic artery, pulmonary veins
Thyroid
etc]
• Prolactinoma
• Glucagonoma

Note: *Hypercalemia normally is responsible for metastatic calcification but it


also accentuates dystrophic calcification.
Fig. 9: Psammoma bodies in meningioma. ...(All India Image)

REPERFUSION INJURY

It is seen with cerebral or myocardial injury. On re-establishment of


blood flow, there is increased recruitment of white blood cells which
cause inflammation as well as generation of more free radicals.

Cellular Ageing
Features of ageing include decreased oxidative phosphorylation,
decreased synthesis of nucleic acids and proteins, deposition of
lipofuscin, accumulation of glycosylation products and abnormally
folded proteins. The most effective way to prolong life is calories
restriction because of a family of proteins called SIRTUINS. The latter
have histone deacetylase activity and promote expression of genes
whose products increase longevity.

The best-studied mammalian sirtuin is Sirt-1Q which has been shown to


improve glucose tolerance and enhance β cell insulin secretion. It is
implicated in diabetesQ.

• Ends of the chromosomes are known as telomeres. Enzyme


telomerase helps in keeping the length of telomere constant.
Decreased activity of this enzyme is associated with ageing
whereas excessive activity is associated with cancers.
FREE RADICAL INJURY

Free radical injury is caused by the following mechanisms:


1. Oxidative stress/reactive oxygen species (O2–, H2O2, OH)
2. Radiation exposure
3. Drugs (carbon tetrachloride, paracetamol)
4. Metals (iron, copper):

Nitric oxide (NO), an important chemical mediator generated by endothelial


cells, macrophages, neurons, and other cell types can act as a free radical
and can also be converted to highly reactive peroxynitrite anion (ONOO-) as
well as NO2 and NO3.

Mechanism of Free Radical Injury

It can result in lipid peroxidation, DNA breaks and fragmentation of the


proteins. This is associated with formation of more free radicals
thereby making free radical induced injury as an autocatalytic reaction.

Antioxidants
Antioxidants may act by inhibiting the generation of free radials or
scavenging the already present free radicals. These may be divided
into enzymatic and non-enzymatic.

Enzymatic Non-enzymatic
a. Superoxide dismutase a. Vitamin E
b. Catalase b. Sulfhydryl containing compounds: cysteine
c. Glutathione peroxidase and glutathione
c. Serum proteins: Albumin, Ceruloplasmin
and Transferrin

• Catalase is present in peroxisomes and decomposes H2O2 into O2


and H2O. (2 H2O2 → O2 + 2 H2O).
• Superoxide dismutase is found in many cell types and converts
superoxide ions to H2O2. (2 O2- + 2 H→ H2O2 + O2).This group
includes both manganese-superoxide dismutase, which is
localized in mitochondria, and copper-zinc-superoxide. dismutase,
which is found in the cytosol.
• Glutathione peroxidase also protects against injury by catalyzing
free radical breakdown. (H2O2 + 2 GSH → GSSG [glutathione
homodimer] + 2 H2O, or 2 OH + 2 GSH →GSSG + 2 H2O).
CHEMICAL FIXATIVES
• Chemical fixatives are used to preserve tissue from degradation,
and to maintain the structure of the cell and of sub-cellular
components such as cell organelles (e.g., nucleus, endoplasmic
reticulum, mitochondria).
• These fixatives preserve tissues or cells mainly by irreversibly
cross-linking proteins.
• Frozen section is a rapid way to fix and mount histology sections. It
is used in surgical removal of tumors, and allow rapid
determination of margin (that the tumor has been completely
removed). It is done using a refrigeration device called a cryostat.
The frozen tissue is sliced using a microtome, and the frozen
slices are mounted on a glass slide and stained the same way as
other methods.

Commonly Used Stains

Substance Stain

Glycogen Carmine (best), PAS with diastase sensitivity

Lipids Sudan black, Oil Red ‘O’

Amyloid Congo Red, Thioflavin T (for JG apparatus of kidney)


and S

Calcium Von Kossa, Alzarine Red

Hemosiderin Perl’s stain (see Figure 8)


Trichrome CollagenQ appears blue, while smooth muscleQ appears
red.


CELL INJURY, NECROSIS, APOPTOSIS

1. CD 95 is a marker of:
(AIIMS Nov 2012)
(a) Intrinsic pathway of apoptosis
(b) Extrinsic pathway of apoptosis
(c) Necrosis of cell
(d) Cellular adaption
2. Which of the following is the characteristic of irreversible
injury on electron microscopy?
(a) Disruption of ribosomes
(AIIMS May 2012)
(b) Amorphous densities in mitochondria
(c) Swelling of endoplasmic reticulum
(d) Cell swelling
3. Caspases are associated with which of the following?
(a) Hydopic degeneration
(AIIMS May 2010)
(b) Collagen hyalinization
(c) Embryogenesis
(d) Fatty degeneration
4. Caspases are seen in which of the following?
(a) Cell division
(b) Apoptosis
(AI 2010)
(c) Necrosis
(d) Inflammation
5. Light microscopic characteristic feature of apoptosis is:
(a) Intact cell membrane
(AI 2010)
(b) Eosinophilic cytoplasm
(c) Nuclear moulding
(d) Condensation of the nucleus
6. Coagulative necrosis is found in which infection?
(AI 2009, AIIMS May’ 10
(a) TB)
(b) Sarcoidosis
(c) Gangrene
(d) Fungal infection
7. Organelle which plays a pivotal role in apoptosis is:
(a) Cytoplasm
(AI 2011, 09, AIIMS May 2010)
(b) Golgi complex
(c) Mitochondria
(d) Nucleus
8. All of the following statements are true regarding reversible
cell injury, except:
(AI 2005)
(a) Formation of amorphous densities in the mitochondrial
matrix
(b) Diminished generation of adenosine triphosphate (ATP).
(c) Formation of blebs in the plasma membrane.
(d) Detachment of ribosomes from the granular endoplasmic
reticulum.
9. Fibrinoid necrosis may be observed in all of the following,
except:
(AI 2005)
(a) Malignant hypertension
(b) Polyarteritis nodosa
(c) Diabetic glomerulosclerosis
(d) Aschoff’s nodule
10. In apoptosis, Apaf-I is activated by release of which of the
following substances from the mitochondria?
(a) Bcl-2
(b) Bax
(AI 2005)
(c) Bcl-XL
(d) Cytochrome C
11. Which of the following is an anti-apoptotic gene?
(a) C-myc
(b) p 53
(AI 2004)
(c) Bcl-2
(d) Bax
12. Annexin V on non-permeable cell is indicative of:
(a) Apoptosis
(AIIMS May 2009)
(b) Necrosis
(c) Cell entering replication phase
(d) Cell cycle arrest
13. Ultra-structural finding of irreversible injury:
(a) Ribosomal detachment from endoplasmic reticulum
(b) Amorphous densities in mitochondria
(c) Formation of phagolysosomes
(AIIMS Nov 2007)
(d) Cell swelling
14. Caspases are involved in:
(AIIMS Nov 2007)
(a) Necrosis
(b) Apoptosis
(c) Atherosclerosis
(d) Inflammation
15. True about Apoptosis are all except:
(a) Inflammation is present
(AIIMS May 2007)
(b) Chromosomal breakage
(c) Clumping of chromatin
(d) Cell shrinkage
16. The following is an antiapoptotic gene:
(AIIMS Nov 2006
(a) Bax
(b) Bad)
(c) Bcl-X
(d) Bim
17. Cytosolic cytochrome C plays an important function in
(AIIMS Nov 2006:
(a) Apoptosis)
(b) Cell necrosis
(c) Electron transport chain
(d) Cell division
18. Most pathognomic sign of irreversible cell injury:
(AIIMS Nov 2006
(a) Amorphous densities in mitochondria
(b) Swelling of the cell membrane)
(c) Ribosomes detached from endoplasmic reticulum
(d) Clumping of nuclear chromatin
19. Internucleosomal cleavage of DNA is characteristic of
(AIIMS Nov 2005:
(a) Reversible cell injury)
(b) Irreversible cell injury
(c) Necrosis
(d) Apoptosis
20. Programmed cell death is known as:
(AIIMS Nov 2005
(a) Cytolysis
(b) Apoptosis)
(c) Necrosis
(d) Proptosis
21. Ladder pattern of DNA electrophoresis in apoptosis is
caused by the action of the following enzyme:
(AIIMS Nov 2004
(a) Endonuclease)
(b) Transglutaminase
(c) DNAse
(d) Caspase
22. Which finding on electron microscopy indicates irreversible
cell injury?
(AIIMS Nov 2002)
(a) Dilatation of endoplasmic reticulum
(b) Dissociation of ribosomes from rough endoplasmic reticulum
(c) Flocculent densities in the mitochondria
(d) Myelin figures
23. True about apoptosis is all, except:
(AIIMS Nov 2001)
(a) Considerable apoptosis may occur in a tissue before it
becomes apparent in histology
(b) Apoptotic cells appear round mass of the intensely
eosinophilic cytoplasm with dense nuclear chromatin
fragments
(c) Apoptosis of cells induces inflammatory reaction
(d) Macrophages phagocytose the apoptotic cells and degrade
them
24. Morphological changes of apoptosis include:
(PGI Dec 01)
(a) Cytoplasmic blebs
(b) Inflammation
(c) Nuclear fragmentation
(d) Spindle formation
(e) Cell swelling
25. True about apoptosis:
(PGI June 2003)
(a) Migration of Leukocytes
(b) End products are phagocytosed by macrophage
(c) Intranuclear fragmentation of DNA
(d) Activation of caspases
(e) Annexin V is a marker of apoptotic cell

MOST RECENT QUESTIONS

26. Which of the following is the hallmark of programmed cell


death?
(a) Apoptosis
(b) Coagulation necrosis
(c) Fibrinoid necrosis
(d) Liquefaction necrosis
27. Inhibitor of apoptosis is:
(a) p53
(b) Ras
(c) Myc
(d) Bcl-2
28. Apoptosis is associated with all of the following features
except:
(a) Cell shrinkage
(b) Intact cellular contents
(c) Inflammation
(d) Nucleosome size fragmentation of nucleus
29. All of the following are morphological features of apoptosis
except:
(a) Cell shrinkage
(b) Chromatin condensation
(c) Inflammation
(d) Apoptotic bodies
30. Irreversible injury in cell is:
(a) Deposition of Ca++ in mitochondria
(b) Swelling
(c) Mitotic figure
(d) Ribosomal detachment
31. Apoptosis is:
(a) Cell degeneration
(b) Type of cell injury
(c) Cell regeneration
(d) Cell activation
32. Pyogenic infection and brain infarction are associated with:
(a) Coagulative necrosis
(b) Liquefactive necrosis
(c) Caseous necrosis
(d) Fat necrosis
33. In apoptosis initiation:
(a) The death receptors induce apoptosis when they get
engaged by fas ligand system
(b) Cytochrome C inhibits Apoptosis Activating (Apaf-1) Factor
–1
(c) Apoptosis may be initiated by caspase activation
(d) Apoptosis mediated through DNA damage
34. Apoptosis is alternatively called:
(a) Ischemic cell death
(b) Programmed cell death
(c) Post traumtic cell death
(d) All
35. First cellular change in hypoxia:
(a) Decreased oxidative phosphorylation in mitochondria
(b) Cellular swelling
(c) Alteration in cellular membrane permeability
(d) Clumping of nuclear chromatin
36. About apoptosis, true statement is:
(a) Injury due to hypoxia
(b) Inflammatory reaction is present
(c) Councilman bodies are associated with apoptosis
(d) Cell membrane is damaged
37. Fournier’s gangrene is seen in:
(a) Nose
(b) Scrotal skin
(c) Oral cavity
(d) All are true
38. Coagulative necrosis is seen in:
(a) Brain
(b) Breast
(c) Liver
(d) All
39. In cell death, myelin figures are derived from:
(a) Nucleus
(b) Cell membrane
(c) Cytoplasm
(d) Mitochondria
40. Irreversible cell injury is characterised by which of the
following?
(a) Mitochondrial densities
(b) Cellular swelling
(c) Blebs
(d) Myelin figures
41. Coagulative necrosis as a primary event is most often seen
in all except:
(a) Kidneys
(b) CNS
(c) Spleen
(d) Liver
42. Organelle that plays a pivotal role in apoptosis:
(a)Endoplasmic reticulum
(b) Golgi complex
(c) Mitochondria
(d) Nucleus
43. Liquefactive necrosis is seen in:
(a) Heart
(b) Brain
(c) Lung
(d) Spleen
44. In apoptosis, cytochrome C acts through:
(a) Apaf 1
(b) Bcl-2
(c) FADD
(d) TNF
45. Cells most sensitive to hypoxia are:
(a) Myocardial cells
(b) Neurons
(c) Hepatocytes
(d) Renal tubular epithelial cells
46. Intracellular calcification begins in which of the following
organelles?
(a) Mitochondria
(b) Golgi body
(c) Lysososme
(d) Endoplasmic reticulum
47. Which of the following induces apoptosis in a cell?
(AIIMS Nov 2013)
(a) Oleic acid
(b) Isoprenoids
(c) Myristic acid
(d) Glucocorticoids
48. Which of the following is not seen is apoptosis?
(a) Chromatin condensation
(AIIMS Nov 2013)
(b) DNA fragmentation
(c) Inflammation
(d) Cell membrane shrinkage
49. Following gene when mutated protects tumor cells from
Apoptosis:
(a) BCL – 2
(b) BRCA
(c) RB
(d) TGF – β
50. Following is seen in both apoptosis and necrosis:
(a) Both may be physiological
(b) Both may be pathological
(c) Inflammation
(d) Intact cell membrane
51.Agarose gel electrophoresis from DNA of a population of cells
as seen under ultraviolet light is shown below. What is the
correct explanation for the finding is seen in the band
labelled as “C”?
(AIIMS May 2016)

(a) Predominantly necrotic cells


(b) Apoptotic cells
(c) Mixed population of normal and apoptotic cells
(d) A population of viable cells
52. What is the common change in cell death associated with
both apoptosis and necrosis?
(a) Cell shrinkage
(b) Bleb formation
(c) Chromatin condensation
(d) Presence of inflammation
53. Example/s of physiological atrophy is:
(a) Senile atrophy
(b) Disuse atrophy
(c) Post-pregnancy uterine atrophy
(d) All of the above
54. Caseous necrosis is seen in which of the following:
(a) CMV infection
(b) Staphylococcal infection
(c) Treponemal infection
(d) HSV infection
55. Apoptotic bodies are:
(a) Clumped chromatin bodies
(b) Pyknotic nucleus without organelles
(c) Cell membrane bound with organelles
(d) No nucleus with organells
56.Neutrophilic infiltration with fibrinoid necrosis in walls of
vessels is seen in:
(a) Giant cell arteritis
(b) Takayasu arteritis
(c) Churg strauss syndrome
(d) Polyarteritis nodosa
57. Myelin figures are derived from:
(a) Nucleus
(b) Cell membrane
(c) Cytoplasm
(d) Mitochondria
58. Diabetic foot is associated with following typegangrene:
(a) Dry gangrene
(b) Wet gangrene
(c) Gas gangrene
(d) Fourniers gangrene
59. Spread of infection causes?
(a) Fibrinoid necrosis
(b) Fat necrosis
(c) Liquefactive necrosis
(d) Coagulative necrosis
60. Unfolded protein metabolism is associated with:
(a) Nucleus
(b) Endoplasmic reticulum
(c) Golgi apparatus
(d) Mitochondria
61. Anti-apoptotic gene:
(a) BIM
(b) P53
(c) BAX
(d) FLIP
62. Not a apoptotic gene:
(a) P53
(b) bax
(c) Mcl-1
(d) n-myc

CELLULAR ADAPTATION, INTRACELLULAR ACCUMULATION

63. Psammoma bodies are seen in all except:


(AI 2011,09)
(a) Follicular carcinoma of thyroid
(b) Papillary carcinoma of thyroid
(c) Serous cystadenoma of ovary
(d) Meningioma
64. True about metastatic calcification is:
(a) Calcium level is normal
(AIIMS May 2009)
(b) Occur in dead and dying tissue
(c) Occur in damaged heart valve
(d) Mitochondria involved earliest
65. Both hyperplasia and hypertrophy are seen in?
(a) Breast enlargement during lactation
(b) Uterus during pregnancy
(AIIMS May 2009)
(c) Skeletal muscle enlargement during exercise
(d) Left ventricular hypertrophy during heart failure
66. Which of the following is not a common site for metastatic
calcification?
(AIIMS Nov 2005)
(a) Gastric mucosa
(b) Kidney
(c) Parathyroid
(d) Lung
67. Calcification of soft tissues without any disturbance of
calcium metabolism is called:
(AIIMS Nov 2004)
(a) Inotrophic calcification
(b) Monotrophic calcification
(c) Dystrophic calcification
(d) Calcium induced calcification
68. The light brown perinuclear pigment seen on H & E staining
of the cardiac muscle fibres in the grossly normal appearing
heart of an 83 year old man at autopsy is due to deposition
as:
(AIIMS May 2003)
(a) Hemosiderin
(b) Lipochrome
(c) Cholesterol metabolite
(d) Anthracotic pigment
69. Dystrophic calcification is seen in:
(AIIMS Nov 2002)
(a) Rickets
(b) Hyperparathyroidism
(c) Atheromatous plaque
(d) Vitamin A intoxication
70. The Fenton reaction leads to free radical generation when:
(AIIMS Nov 2002)
(a) Radiant energy is absorbed by water
(b) Hydrogen peroxide is formed by Myeloperoxidase
(c) Ferrous ions are converted to ferric ions
(d) Nitric oxide is converted to peroxynitrite anion
71. Mallory hyaline is seen in:
(PGI Dec 2000)
(a) Alcoholic liver disease
(b) Hepatocellular carcinoma
(c) Wilson’s disease
(d) I.C.C. (Indian childhood cirrhosis)
(e) Biliary cirrhosis
72.Heterotopic calcification occurs in:
(a) Ankylosing spondylitis
(PGI Dec 2000)
(b) Reiter’s syndrome
(c) Forrestier’s disease
(d) Rheumatoid arthritis
(e) Gouty arthritis
73. Pigmentation in the liver is caused by all except:
(PGI Dec 01)
(a) Lipofuscin
(b) Pseudomelanin
(c) Wilson’s disease
(d) Malarial pigment
(e) Bile pigment
74. Wear and tear pigment in the body refers to:
(Karnataka 2006)
(a) Lipochrome
(b) Melanin
(c) Anthracotic pigment
(d) Hemosiderin
75. Mallory hyaline bodies are seen all except:
(a) Indian childhood cirrhosis
(AI 97) (UP 2004)
(b) Wilson’s disease
(c) Alcoholic hepatitis
(d) Crigler-Najjar syndrome
76. “Russell’s body” are accumulations of:
(UP 2006)
(a) Cholesterol
(b) Immunoglobulins
(c) Lipoproteins
(d) Phospholipids
MOST RECENT QUESTIONS
77. Dystrophic calcification is seen in:
(a) Atheroma
(b) Paget’s disease
(c) Renal osteodystrophy
(d) Milk-alkali syndrome
78. Brown atrophy is due to:
(a) Fatty necrosis
(b) Hemosiderin
(c) Lipofuscin
(d) Ceruloplasmin
79. Psammoma bodies are typically associated with all of the
following neoplasms except:
(a) Medulloblastoma
(b) Meningioma
(c) Papillary carcinoma of the thyroid
(d) Papillary serous cystadenocarcinoma of the ovary
80. Transformation of one epithelium to other epithelium is
known as:
(a) Dysplasia
(b) Hyperplasia
(c) Neoplasia
(d) Metaplasia
81. All are true about metaplasia except:
(a) Slow growth
(b) Reverse back to normal with appropriate treatment
(c) Irreversible
(d) If persistent may induce cancer transformation
82. About hyperplasia, which of the following statement is false?
(a) ↑ no of cells
(b) ↑ size of the affected cell
(c) Endometrial response to estrogen is an example
(d) All
83. Example of hypertrophy is:
(a) Breast in puberty
(b) Uterus during pregnancy
(c) Ovary after menopause
(d) Liver after resection
84. Metastatic calcification occurs in all except:
(a) Kidney
(b) Atheroma
(c) Fundus of stomach
(d) Pulmonary veins
85. An old man Muthoot has difficulty in urination associated
with increased urge and frequency. He has to get up several
times in night to relieve himself. There is no history of any
burning micturition and lower back pain. On rectal
examination, he has enlarged prostate. Which of the
following represents the most likely change in the bladder
of this patient?
(a) Hyperplasia
(b) Atrophy
(c) Hypertrophy
(d) Metaplasia
86. An increase in the size of a cell in response to stress is
called hypertrophy. Which of the following does not
represent the example of smooth muscle hypertrophy as an
adaptive response to the relevant situation?
(a) Urinary bladder in urine outflow obstruction
(b) Small intestine in intestinal obstruction
(c) Triceps in body-builders
(d) None of the above
87. Metastatic calcification is most often seen in:
(a) Lymph nodes
(b) Lungs
(c) Kidney
(d) Liver
88. True about psammoma bodies are all except:
(a) Seen in meningioma
(b) Concentric whorled appearance
(c) Contains calcium deposits
(d) Seen in teratoma
89. Russell bodies are seen in:
(a) Lymphocytes
(b) Neutrophils
(c) Macrophages
(d) Plasma cells
90. Psammoma bodies show which type of calcification:
(a) Metastatic
(b) Dystrophic
(c) Secondary
(d) Any of the above
91. Oncocytes are modified form of which of the following:
(a) Lysososmes
(b) Endoplasmic reticulum
(c) Mitochondria
(d) None of the above
92. Gamma Gandy bodies contain hemosiderin and:
(a) Na+
(b) Ca++
(c) Mg++
(d) K+

MISCELLANEOUS: FREE RADICAL INJURY: STAINS

93. Which of the following is the most common fixative used in


electron microscopy?
(AIIMS Nov 2012)
(a) Glutaraldehyde
(b) Formalin
(c) Picric acid
(d) Absolute Alcohol
94. The fixative used in histopathology:
(AIIMS May 2012)
(a) 10% buffered neutral formalin
(b) Bouins fixative
(c) Glutaraldehyde
(d) Ethyl alcohol
95. Which is the most commonly used fixative in
histopathological specimens?
(AI 2011)
(a) Glutaraldehyde
(b) Formaldehyde
(c) Alcohol
(d) Picric acid
96. Lipid in the tissue is detected by:
(AIIMS Nov 2009)
(a) PAS
(b) Myeloperoxidase
(c) Oil Red O
(d) Mucicarmine
97. The most abundant glycoprotein present in basement
membrane is:
(AI 2004)
(a) Laminin
(b) Fibronectin
(c) Collagen type 4
(d) Heparan sulphate
98. Enzyme that protects the brain from free radical injury is:
(AI 2001)
(a) Myeloperoxidase
(b) Superoxide dismutase
(c) MAO
(d) Hydroxylase
99. Increased incidence of cancer in old age is due to:
(a) Telomerase reactivation
(AIIMS May 2009)
(b) Telomerase deactivation
(c) Inactivation of protooncogene
(d) Increase in apoptosis
100. Stain not used for lipid:
(AIIMS Nov 2007)
(a) Oil red O
(b) Congo red
(c) Sudan III
(d) Sudan black
101. Acridine orange is a fluorescent dye used to bind:
(AIIMS Nov 2007)
(a) DNA and RNA
(b) Protein
(c) Lipid
(d) Carbohydrates
102. PAS stains the following except:
(AIIMS Nov 2007)
(a) Glycogen
(b) Lipids
(c) Fungal cell wall
(d) Basement membrane of bacteria
103. All are components of basement membrane except:
(a) Nidogen
(b) Laminin
(AIIMS Nov 2007)
(c) Entactin
(d) Rhodopsin
104. Which of the following pigments are involved in free radical
injury?
(AIIMS Nov 2006)
(a) Lipofuscin
(b) Melanin
(c) Bilirubin
(d) Hematin
105. True about cell ageing:
(AIIMS Nov 2001)
(a) Free radicals injury
(b) Mitochondria are increased
(c) Lipofuscin accumulation in the cell
(d) Size of cell increased
106. Neutrophil secretes:
(PGI Dec 2002)
(a) Superoxide dismutase
(b) Myeloperoxidase
(c) Lysosomal enzyme
(d) Catalase
(e) Cathepsin G

MOST RECENT QUESTIONS

107. Which of the following is a peroxisomal free radical


scavenger?
(a) Superoxide dismutase
(b) Glutathione peroxidase
(c) Catalase
(d) All of the above
108. Crooke’s hyaline body is present in:
(a) Yellow fever
(b) Basophil cells of the pituitary gland in Cushing’s syndrome
(c) Parkinsonism
(d) Huntington’s disease
109. Stain used for melanin is:
(a) Oil red
(b) Gomori methamine silver stain
(c) Masson fontana stain
(d) PAS stain
110. Which of the following statements about Telomerase is
true?
(a)Has RNA polymerase activity
(b) Causes carcinogenesis
(c) Present in somatic cells
(d) Absent in germ cells
111. Which of the following is a negative stain?
(a) Fontana
(b) ZN stain
(c) Nigrosin
(d) Albert stain
112. Oil red O staining is used for:
(a) Frozen section
(b) Glutaraldehyde fixed specimen
(c) Alcohol fixed specimen
(d) Formalin fixed specimen
113. The surgical registrar successfully performs a testicular
biopsy and hands over the specimen to the attending nurse.
The sister asks you how to send the specimen to the
pathologist. What fluid will you tell the sister to put the
specimen in:
(AIIMS May 2016)
(a) 95% Ethanol
(b) 10% formalin
(c) Bouin’s solution
(d) Zenker’s solution
114. Most reactive free radical is:
(AIIMS May 2016)
(a) Superoxide
(b) Hydroxyl
(c) Peroxide
(d) Carboxyl
115. Stain useful for identifying premalignant lesions of the lip
is:
(AIIMS Nov 2016)
(a) Hematoxylin and eosin
(b) Toluidine blue
(c) Giemsa
(d) Alizarin red
116. Resolving power of a light microscope is not affected by:
(AIIMS Nov 2016)
(a) Sample thickness
(b) Focal length of objective lens
(c) Numerical aperture of the lens
(d) Wavelength of light
117. All are sources of free radicals except:
(a) Fentons reaction
(b) Glutathione
(c) Myeloperoxidase
(d) Nitric oxide
118. Lipschutz bodies are seen in:
(a) Hodgkin’s disease
(b) Viral hepatitis
(c) Herpes
(d) Yellow fever
119. Which of the following is detected by Prussian blue stain?
(a) Ferric ions (b) Glycogen
(c) Lipids (d) Ferrous ions

1. Ans. (b) Extrinsic pathway of apoptosis
(Ref: Robbins 8/e p29-30, 9/e p56)
In the activation of Extrinsic pathway of apoptosis, binding of Fas
ligand takes place to CD95 (Fas; member of TNF receptor
family) or binding of TRAIL (TNF related apoptosis inducing
ligand) attaches to death receptors DR4 and DR5. This induces
the association of FADD (Fas- associated death domain) and
procaspase-8 to death domain motifs of the receptors resulting
in activation of caspase 8 (in humans caspase 10) which finally
activates caspases- 3 and 7 that are final effector caspases
2. Ans. (b) Amorphous densities in mitochondria
(Ref: Robbins 8/e p14-19, 9/e p42,50)
Two phenomena consistently characterize irreversibility:
1. The first is the inability to reverse mitochondrial dysfunction (lack
of oxidative phosphorylation and ATP generation) even after resolution of
the original injury.
2. The second is the development of profound disturbances in
membrane function.
So, the answer for the given question is ‘Amorphous densities in
mitochondria’.
However, please remember friends that the Robbins in its 8th edition
pg 14 mentions small amorphous densities to be present in
reversible cell injury also. Therefore, the best answer for
characterizing irreversibility of an injury is ‘profound disturbances
in membrane function’.
3. Ans. (c) Embryogenesis
(Ref: Robbins 8/e p25, 9/e p52)
Caspases are cysteine proteases and are critical for the process of
apoptosis. It is required at the time of different processes in
embryogenesis like implantation, organogenesis, developmental
involution and metamorphosis.
4. Ans. (b) Apoptosis
(Ref: Robbins 8/e p27, 9/e p53)
5. Ans. (d) Condensation of the nucleus
(Ref: Robbins 8/e p14-15, 26-27, 9/e p53)
The morphologic features characteristic of apoptosis includes
• Cell shrinkage: The cell is smaller in size having dense
cytoplasm and the organelles are tightly packed.
• Chromatin condensation: This is the most characteristic
feature of apoptosis.
• Formation of cytoplasmic blebs and apoptotic bodies
Regarding option ‘a’…’Plasma membranes are thought to remain
intact till late stage of apoptosis, as well as is a normal cell.
Regarding option “b”, eosinophilic cytoplasm, it is a common
feature of necrosis and apoptosis.
Regarding option “c”, nuclear moulding is defined as the “The shape of
one nucleus conforming around the shape of an adjacent nucleus”. It is a
characteristic of malignant cells.

6. Ans. (a) TB > (c) Gangrene


(Ref: Robbins 8/e p16, 9/e p43)
th
In the 7 edition of Robbins it was clearly stated that…“Caseous
necrosis, a distinctive form of coagulative necrosis, is
encountered most often in foci of tuberculous infection. The term
caseous is derived from the cheesy white gross appearance of
the area of necrosis.”
Regarding the option gangrene, it is not specified the type of
gangrene and therefore, we go with the better option as
tuberculosis in the given question. Moreover, according to
Robbins, gangrenous necrosis is not a specific pattern of
necrosis but is a term used in clinical practice.

Dry gangrene has coagulative necrosis whereas wet gangrene has


liquefactive necrosis.

7. Ans. (c) Mitochondria


(Ref: Harrison 18/e p681, 9/e p53)
8. Ans. (a) Formation of amorphous densities in mitochondrial
matrix
(Ref: Robbins 7/e p19, 9/e p42)
Formation of amorphous densities in the mitochondrial matrix is a
feature of irreversible injury (more commonly) and left commonly
can be seen in reversible injury.
9. Ans. (c) Diabetic glomerulosclerosis
(Ref: Robbins 7/e p214, 594, 1008, 9/e p44) ...see text for details
10. Ans. (d) Cytochrome C (Ref: Harrison 17/e p506, 9/e p55)
Apoptosis or programmed cell death can be induced by intrinsic
or extrinsic pathway. As can be seen in the intrinsic pathway; cyt
c gets associated with APAF-1 which activates caspase and
cause cell death.For detail see text.
11. Ans. (c) Bcl-2
(Ref: Harrison 17/e p506)
12. Ans. (a) Apoptosis
(Ref: Robbins 8/e p27, 9/e p56)
Apoptotic cells express phosphatidylserine in the outer layers of their
plasma membranes. This phospholipid moves out from the inner
layers where it is recognized by a number of receptors on the
phagocytes. These lipids are also detected by binding of a
protein called Annexin V. So, Annexin V staining is used to
identify the apoptotic cells.
13. Ans. (b) Amorphous densities in mitochondria
(Ref: Robbins 7/e p12, 9/e p42) ...See earlier explanation.
14. Ans. (b) Apoptosis
(Ref: Robbins 7/e p28, 9/e p53)
Caspases are present in normal cells as inactive proenzymes and
when they are activated they cleave proteins and induce
apoptosis. These are cysteine proteases.
15. Ans. (a) Inflammation is present
(Ref: Robbins 9/e p56)
In Apoptosis the dead cell is rapidly cleared, before its contents have
leaked out, and therefore cell death by this pathway does not
elicit an inflammatory reactionQ in the host.
16. Ans. (c) Bcl-X
(Ref: Robbins 7/e p29, 9/e p55)
17. Ans. (a) Apoptosis
(Ref: Robbins 7/e p26, 9/e p55)
• Cytosolic cytochrome C and Apaf-1 are involved in intrinsic
pathway of apoptosisQ.
• Mitochondrial cyt ‘c’ and not cytosolic cyt’c’ is involved in aerobic
respirationQ.

18. Ans. (a) Amorphous densities in mitochondria


(Ref: Robbin’s 7/e p12, 9/e p50)
19. Ans. (d) Apoptosis
(Ref: Robbin’s 8/e p27, 9/e p52)
The inter-nucleosomal cleavage of DNA into oligonucleosomes (in
multiples of 180-200 base pairs) is brought about by Ca2+ and
Mg2+ dependent endonucleases and is characteristic of
apoptosis.
20. Ans. (b) Apoptosis
(Ref: Robbins 7/e p26, 27, 9/e p52)
21. Ans. (a) Endonuclease
(Ref: Robbins 8/e pg28)
• Endonucleases are enzymes which cause
internucleosomal cleavage of DNA into
oligonucleosomes, the latter being visualized by agarose
gel electrophoresis as DNA ladders.
• In necrosis, smeared pattern is commonly seen
22. Ans. (c) Flocculent densities in mitochondria
(Ref: Robbins’s 7/e p12, 9/e p50)
23. Ans. (c) Apoptosis of cells induces inflammatory reaction
(Ref: Robbins 7/e p27, 9/e p56)
Remember important features of apoptosis
• Formation of cytoplasmic blebs and apoptotic bodiesQ
• Cell ShrinkageQ: The cells are smaller in size and the cytoplasm is
dense.
• Chromatin condensationQ: This is the most characteristic features
of apoptosis.
• Absence of inflammationQ
• Gel Electrophoresis of DNA shows ‘Step ladder’Q Pattern.

24. Ans. (a) Cytoplasmic blebs; (c) Nuclear fragmentation


(Ref: Robbins 7/e p26, 9/e p53)
Apoptosis is a programmed cell death.
During apoptosis, cells destined to die activate enzymes that degrade
the cell’s own nuclear DNA and nuclear and cytoplasmic
proteins. There is no inflammatory reaction elicited by host.
• Spindle formation is found in cell division in mitosis.
• During necrosis, cell swelling is seen.
25. Ans. (b) End products are phagocytosed by macrophage; (c)
Intranuclear fragmentation of DNA; (d) Activation of
caspases; (e) Annexin V is a marker of apoptotic cell
(Ref: Robbins 7/e p25-3l, 9/e p53)
26. Ans. (a) Apoptosis
(Ref: Robbins 8/e p25, 9/e p52)
27. Ans. (d) Bcl-2
(Ref: Robbins 7/e p29, 31, 32, 9/e p55)
28. Ans. (c) Inflammation
(Ref: Robbins 7/e p26, 9/e p53)
29. Ans. (c) Inflammation
(Ref: Robbin 7/e p27, 9/e p56)
30. Ans. (a) Deposition of Ca++ in mitochondria
(Ref: Robbins 8/e p13-14; 7/e p11, 9/e p47)
31. Ans. (b) Type of cell injury
(Ref: Robbins 9/e p52)
32. Ans. (b) Liquefactive necrosis
(Ref: Robbins 9/e p43)
33. Ans. (a) The death receptors induce apoptosis when they get
engaged by fas ligand system
(Ref: Robbins 9/e p56)
34. Ans. (b) Programmed cell death
(Ref: Robbins 9/e p52)
35. Ans. (a) Decreased oxidative phosphorylation in
mitochondria
(Ref: Robbins 8/e p18-19, 7/e p15, 9/e p45)
36. Ans. (c) Councilman bodies are associated with apoptosis
(Ref: Robbins 8/e p25; 7/e 26, 9/e p823)
37. Ans. (b) Scrotal skin
38. Ans. (c) Liver
(Ref: Robbins 7/e p21, 8/e p15; 9/e p43)
39. Ans. (b) Cell membrane
(Ref: Robbins 9/e p50-51)
40. Ans. (a) Mitochondrial densities
(Ref: Robbins 9/e p50)
41. Ans. (b) CNS
(Ref: Robbins 8/e p15, 7/e p22, 9/e p43)
42. Ans. (c) Mitochondria
(Ref: Robbins 8/e p28, 9/e p15, 53)
43. Ans. (b) Brain
(Ref: Robbins 8/e p15, 7/e p22, 9/e p43)
44. Ans. (a) Apaf 1
(Ref: Robbins 8/e p29, 9/e p55)
On being released in the cytosol, cytochrome c binds to a protein
called Apaf-1 (apoptosis-activating factor-1 which is responsible
for formation of a complex called apoptosome. This complex
binds to caspase-9 which is a critical initiator caspase of the
mitochondrial pathway of apoptosis.
NEET POINTS about APOPTOSIS
• Mitochondrion is the critical organelle required for apoptosis.
• Chromatin condensation is the most characteristic feature.
• Cell shrinkage is seen
• Gel electrophoresis demonstrates “step ladder pattern”
• Annexin V is the marker for apoptosis.
• CD 95 is the molecular marker for apoptosis

45. Ans. (b) Neurons


(Ref: Robbins 8/e p11-2)
Potential future questions!
The most sensitive neurons in the brain are in the pyramidal cell layer of the
hippocampus (especially area CA1, also referred to as Sommer sector),
cerebellar Purkinje cells and pyramidal neurons in cerebral cortex.

46. Ans. (a) Mitochondria


(Ref: Robbins 8/e p19, 9/e p65-66)
Direct quote.. “Initiation of intracellular calcification occurs in the
mitochondria of dead or dying cells that accumulate
calcium”.
47. Ans. (d) Glucocorticoids
(Ref: Underwood’s Pathology 6/e p80)
Glucocorticoids induce apoptosis while sex steroids inhibit
apoptosis. ..Underwood Pathology
Inducers of apoptosis Inhibitors of apoptosis
• Withdrawal of growth factor • Growth factors
• Loss of matrix attachment • Extracellular matrix
• Glucocorticoids • Sex steroids
• Free radicals • Some viral proteins
• Some viruses
• Ionising radiation
• DNA damage

48. Ans. (c) Inflammation


(Ref: Robbin 8/e p26-7)
Inflammation is not seen in apoptosis.
Chromatin condensation is the most characteristic feature of
apoptosis. Other findings like cell membrane shrinkage and DNA
fragmentation are also associated with apoptosis.
49.Ans (a) BCL-2
(Ref: Robbin 9th/ 8thed: pg606)
We need to identify a gene which should be able to inhibit apoptosis.
The answer therefore is BCL-2. It is seen to result in the
development of follicular lymphoma.
50. Ans (b) Both may be pathological
(Ref: Robbin 9th/40)
51. Ans. (a) Predominantly necrotic cells
(Ref: Robbins 8th/27)

• Presence of a smeared pattern on gel electrophoresis is associated with


necrosis...(C in figure)
• Presence of a step ladder pattern on electrophoresis is associated with
apoptosis (B in figure).

52. Ans (c) Chromatin condensation

(Ref: Robbins 9/e p15)


53. Ans (c) Post-pregnancy uterine atrophy
(Ref: Robbins 9/e p36)
Physiological atrophy includes:
a. Post-pregnancy uterine atrophy
b. Atrophy of embryonic structures, (like notochord and thyroglossal duct),
undergo during fetal development.

54. Ans (c) Treponemal infection


(Ref: Robbins 9/e p381)
Caseous necrosis is seen with tuberculosis, syphilis and certain
fungi (histoplasmosis, cryptococcosis, and coccidioidomycosis).
55. Ans (c) Cell membrane bound with organelles
(Ref: Robbins 9/e p52)
56. Ans (d) Polyarteritis nodosa
(Ref: Robbins 9/e p509)
57. Ans (b) Cell membrane
(Ref: Robbins 9/e p42)
Dead cells may be replaced by large, whorled phospholipid masses
called myelin figures that are derived from damaged cell
membranes.
58. Ans (b) Wet gangrene
(Ref: Robbins 9/e p43)
59. Ans (c) Liquefactive necrosis
(Ref: Robbins 9/e p43)
60. Ans (b) Endoplasmic reticulum
(Ref: Robbins 9/e p12)
Excess accumulation of misfolded proteins, exceeding the capacity
of the ER to edit and degrade them, leads to the ER stress
response (also called the unfolded protein response or UPR)
that triggers cell death through apoptosis.
61. Ans (d) FLIP
(Ref: Robbins 9/e p56)
62. Ans (d) n-myc
(Ref: Robbins 9/e p302)
Anti apoptotic genes include:
• Bcl-2
• Bcl-XL
• MCL-1
63. Ans. (a) Follicular carcinoma of thyroid
(Ref: Robbins 8/e p38, 9/e p65)
Tumors (MOST for PG)
• M – Meningioma
• O – Papillary carcinoma of Ovary (serous ovarian
cystadenoma)
• S – Papillary carcinoma of Salivary gland
• T – Papillary carcinoma of Thyroid
• Prolactinoma, Papillary type of renal cell carcinoma
• Glucagonoma
(Psammoma bodies are seen in papillary thyroid cancer and not
follicular thyroid cancer)
64. Ans. (d) Mitochondria involved earliest
(Ref: Robbins 8/e p38, Robbins 7/e p41-42, 9/e p65)
65. Ans. (b) Uterus during pregnancy
(Ref: Robbins 9/e p36)
• Hypertrophy refers to an increase in the size of cells,
resulting in an increase in the size of the organ. The
increased size of the cells is due the synthesis of more
structural components.
• The massive physiologic growth of the uterus during
pregnancy is a good example of hormone-induced increase
in the size of an organ that results from both hypertrophy
and hyperplasia
• Regarding the ‘a’ choice, Breast enlargement during
lactation; it is written in Robbins that prolactin and estrogen
cause hypertrophy of the breasts during lactation.
Hormonal hyperplasia is best exemplified by the
proliferation of the glandular epithelium of the female breast
at puberty and during pregnancy.
66. Ans. (c) Parathyroid
(Ref: Robbins 7/e p42, 9/e p65)
• Metastatic calcification may occur widely throughout the
body but principally affects:
• Interstitial tissues of gastric mucosaQ
• KidneysQ
• LungsQ
• Systemic arteriesQ and
• Pulmonary veinsQ
• The common feature of all these sites, which makes them
prone to calcification is that can loose acid and therefore
they have an internal alkaline component favorable for
metastatic calcification.
• Absence of derangement in calcium metabolism
• Often a cause of organ dysfunction.
67. Ans. (c) Dystrophic calcification
(Ref: Robbins 9/e p65)
68. Ans. (b) Lipochrome
(Ref: Robbins 7/e p39, 9/e p64)
Regarding other options
• Hemosiderin: It is a pigment deposited in conditions of
excess iron.
• Anthracotic pigment: It is pigment seen in the lung of coal
69.Ans. (c) Atheromatous plaque
(Ref: Robbins 9/e p65)
• Atheromatous plaque would have dead cells, so, there is
presence of dystrophic calcification.
Mnemonic: D for Dead and D for Dystrophic.

70. Ans. (c) Ferrous ions are converted to ferric ions


(Ref: Robbins’ 7/e p16, 9/e p48)
• Free radicals are generated through Fenton’s reaction
which is (H2O2 + Fe2+ → Fe3+ + OH+ + OH–)
• In this reaction iron is converted from its ferrous to ferric
form and a radical is generated.
• The other options are also examples of free radical injury
but the questions specifically about Fenton reaction.
• The effects of these reactive species relevant to cell injury
include: Lipid peroxidation of membranes, oxidative
modification of proteins and lesions in DNA.
71. Ans. (a) Alcoholic liver disease; (b) Hepatocellular
carcinoma; (c) Wilson’s disease; (d) I.C.C. (Indian childhood
cirrhosis); (e) Biliary cirrhosis
(Ref: Robbins’ 7/e p905)
Mallory bodies: Scattered hepatocytes accumulate tangled skeins of
cytokeratin intermediate filaments and other proteins, visible as
eosinophilic cytoplasmic inclusions in degenerating hepatocytes.
See details in chapter on ‘Liver’.
72. Ans. (a) Ankylosing spondylitis; (c) Forrestier’s disease
(Ref: Robbins’ 7/e p41-2; Harrison17/e p1952)
Pathologic calcification (Heterotopic calcification) is the abnormal
tissue deposition of calcium salts together with small amounts of
iron, manganese and other mineral salts. It may be of two
types: Dystrophic calcification or Metastatic calcification
*In ankylosing spondylitis - There is calcification and ossification usually
most prominent in anterior spinal ligament that gives “Flowing wax”
appearanceQ on the anterior bodies of vertebrae.
*Diffuse idiopathic skeletal hyperostosis (Forrestier’s diseaseQ,
ankylosing hyperostosis) affects spine and extra-spinal locations. It is an
enthesopathy, causing bony overgrowths and ligamentous ossification and
is characterized by flowing calcification over the anterolateral aspects of
vertebrae.

73. Ans. None


(Ref: Robbins 7/e p39, 910, 914)
Pigmentation in liver is caused by:
• Lipofuscin: It is an insoluble pigment known as lipochrome
and ‘wear and tear’ pigment. It is seen in cells undergoing
low, regressive changes and is particularly prominent in
liver and heart of ageing patient or patients with severe
malnutrition and cancer cachexia.
• Pseudomelanin: After death, a dark greenish or blackish
discoloration of the surface of the abdominal viscera results
from the action of sulfated hydrogen upon the iron of
disintegrated hemoglobin. Liver is also pigmented.
• Wilson’s disease: Copper is usually deposited in periportal
hepatocytes in the form of reddish granules in cytoplasm or
reddish cytoplasmic coloration stained by rubeanic acid or
rhodamine stain for copper or orcein stain for copper
associated protein. Copper also gets deposited in chronic
obstructive cholestasis.
• Malarial pigment: Liver colour varies from dark chocolate
red to slate-grey even black depending upon the stage of
congestion.
• In biliary cirrhosis liver is enlarged and greenish-yellow in
colour due to cholestasis. So liver is pigmented due to bile.
74. Ans. (a) Lipochrome
(Ref: Robbins 7/e p39, 9/e p64)
75. Ans. (d) Crigler-Najjar syndrome
(Ref: Robbins 7/e p905)
76. Ans. (b) Immunoglobulins
(Ref: Robbins 9/e p63)
77. Ans. (a) Atheroma
(Ref: Robbins 8/e p38, 9/e p65)
78. Ans. (c) Lipofuscin
(Ref: Robbins 8/e p10,532; 7/e 10, 9/e p64)
79. Ans. (a) Medulloblastoma
(Ref: Robbins 9/e p65)
80. Ans. (d) Metaplasia
(Ref: Robbins 8/e p10,11; 9/e p37)
81. Ans. (c) Irreversible
(Ref: Robbins 8/e p265; 9/e p37-38
82. Ans. (b) ↑ size of the affected cell
(Ref: Robbins 9/e p35)
83. Ans. (b) Uterus during pregnancy
(Ref: Robbins 8/e p6, 7/e p7-8 , 9/e p34-36)
Breast at Puberty Hyperplasia
Breast during lactation Hypertrophy
Uterus after resection Hyperplasia
Uterus during pregnancy Hyperplasia + Hypertrophy

84. Ans. (b) Atheroma


(Ref: Robbins 8/e p38; 7/e p41, 9/e p65)
85. Ans. (c) Hypertrophy
(Ref: Robbins 8/e p6-7, 9/e p36)
The patient is most likely suffering from benign hyperplasia of the
prostate. The question however asks about the change in
bladder which would be hypertrophy. This is secondary to the
obstruction in the urine outflow following which the smooth
muscle in the bladder undergoes hypertrophy.
Benign prostatic hyperplasia is due to action of the hormone
dihydrotestosterone and not testosterone.

86. Ans. (c) Triceps in body-builders


(Ref: Robbins 9/e p34)
The enlargement of the triceps is an example of skeletal muscle
hypertrophy (not smooth muscle hypertrophy).
87. Ans. (b) Lungs
(Ref: Dail and Hammar’s Pulmonary Pathology: Non-neoplastic
lung disease, Springer 3/e p777)
Direct quote…‘Lung are the most frequent involved of all organs.’
Ours is the only and the first book to give you an authentic
reference for this one friends. This is in sharp contrast to all
our competitors who give name and page number of books
where this info is just not there. Try that yourself. You would
find many such questions and answers in other chapters of
this edition. Happy reading!
88. Ans. (d) Seen in teratoma
(Ref: Robbins 8/e p38, 9/e p65)
The progressive acquisition of outer layers may create lamellated
configurations, called psammoma bodies because of their
resemblance to grains of sand. Some common cancers
associated with psammoma bodies are:
• M – Meningioma, Mesothelioma
• O – Papillary carcinoma of Ovary (serous ovarian
cystadenoma)
• S – Papillary carcinoma of Salivary gland
• T – Papillary carcinoma of Thyroid
• Prolactinoma
• Glucagonoma
89. Ans. (d) Plasma cells
(Ref: Robbins 8/e p35, 9/e p63)
Russell bodies are homogenous eosinophilic inclusions that result
from hugely distended endoplasmic reticulum.
90. Ans. (b) Dystrophic
(Ref: Robbins 8/e p38, 9/e p65)
Direct quote… “On occasion single necrotic cells may constitute
seed crystals that become encrusted by the mineral
deposits. The progressive acquisition of outer layers may
create lamellated configurations, called psammoma
bodies.”
91. Ans. (c) Mitochondria
(Ref: Robbins 8/e p35, 9/e p53)
Oncocytes are epithelial cells stuffed with mitochondria, which
impart the granular appearance to the cytoplasm.
92. Ans. (b) Ca++
(Ref: Robbins 7/e p705)
In chronic venous congestion of spleen, some of the hemorrhages
overlying fibrous tissue get deposits of hemosiderin and
calcium, these are called Gamma Gandy bodies or
siderofibrotic nodules.
93. Ans. (a) Glutaraldehyde
(Ref: Ackerman 9th/27)

• Commonest fixative used for light microscopic examination: 10%


buffered neutral formalin
• Commonest fixative used for electron microscopic examination:
Glutaraldehyde
94. Ans. (a) 10% buffered neutral formalin

(Ref: Bancroft 6/e p53, Ackerman 9th/27) ...see above answer.


95. Ans. (b) Formaldehyde
(Ref: Bancroft 6/e p53)
96. Ans. (c) Oil Red O
(Ref: Bancroft histology 6/e p53)
97. Ans. (a) Laminin
(Ref: Robbins Harrison 17/e p2462)
Laminin is the most abundant glycoprotein in basement membranes.
Type IV collagen, laminin and nidogen are present in basement
membranes.
Tendons and ligaments consist primarily of collagen type I whereas
cartilage is mainly consisted of Type II collagen.

98. Ans. (b) Superoxide dismutase


(Ref: Robbins 7/e p17, Harrison’s 17/e p2572 , 9/e p48)
99. Ans. (a) Telomerase reactivation
(Ref: Robbins 9/e p67)
• After a fixed number of divisions, normal cells become arrested in a
terminally non-dividing state known as replicative senescence. With
each cell division there is some shortening of specialized structures,
called telomeres, at the ends of chromosomes. Once the telomeres
are shortened beyond a certain point, the loss of telomere function
leads to activation of p53-dependent cell-cycle checkpoints, causing
proliferative arrest or apoptosis. Thus, telomere shortening functions
as a clock that counts cell divisions.
• In germ cells, telomere shortening is prevented by the sustained
function of the enzyme telomerase, thus explaining the ability of
these cells to self-replicate extensively. This enzyme is absent in
most somatic cells, and hence they suffer progressive loss of
telomeres.
• Cancer cells prevent telomere shortening by the reactivation of
telomerase activity. Telomerase activity has been detected in more
than 90% of human tumors. Telomerase activity and maintenance of
telomere length are essential for the maintenance of replicative
potential in cancer cells.
100. Ans. (b) Congo red
(Ref: Bancroft’s histopathology 5th/204)
Congo red is used for staining amyloid and not lipids
Stains for Lipids
• Oil red O • Sudan black • Sudan III and IV
• Filipin • Schultz • Nile blue sulfate

101. Ans. (a) DNA and RNA


(Ref: Bancroft 5th/236, 237, 238)
• Acridine orange is a nucleic acid selective fluorescent
cationic dye useful for cell cycle determination.
• It is cell-permeable, and interacts with DNA and RNA by
intercalation or electrostatic attractions respectively and
emits green and red right respectively.
• Acridine orange can be used in conjunction with
ethidium bromide to differentiate between live and
apoptotic cells.
102. Ans. (b) Lipids
(Ref: Bancroft’s histopathology 5th/204)
PAS (periodic acid-Schiff) stain is versatile and has been used to
stain many structures including glycogen, mucin, mucoprotein,
glycoprotein, as well as fungi. PAS is useful for outlining tissue
structures, basement membranes, glomeruli, blood vessels and
glycogen in the liver.
*Lipids are stained by oil red O and Sudan stains. (See explanation above)
*PAS can also stain glycolipids but here it is used for staining carbohydrate
moiety of these compounds and not lipid portion.

103. Ans. (d) Rhodopsin


(Ref: Robbins 7/e p103, 9/e p24)
Basement membrane is Periodic Acid Schiff (PAS) positive
amorphous structures that lie underneath epithelia of different
organs and endothelial cells. It consists of
• Laminin • Fibronectin • Tenascin
• Proteoglycans • Entactin • Perlecan (heparin
(Nidogen) sulphate)
• Collagen type IV

104. Ans. (a) Lipofuscin


(Ref: Robbins 7/e p39 , 9/e p64)
Important points about Lipochrome or Lipofuscin.
*Also called ‘wear and tear pigment’Q or ‘pigment of ageing’Q
*Perinuclear in location
*Derived through lipid peroxidationQ
*Indicative of free radical injury to the cell
*Prominent in ageingQ, severe malnutritionQ and cancer cachexiaQ

105. Ans. (c) Lipofuscin accumulation in the cell


(Ref: Robbins 9/e p64)
106. Ans. (b) Myeloperoxidase; (e) Cathepsin G
(Ref: Robbins 7/e p73)
107. Ans. (d) All of the above
(Ref: Robbins 9/e p48) ...see text
108. Ans. (b) Basophil cells of the pituitary gland in Cushing’s
syndrome
(Ref: Robbins 8/e p1149)
In Cushing’s syndrome, the normal granular, basophilic cytoplasm of the
ACTH producing cells in the anterior pituitary becomes paler and
homogenous. This is due to accumulation of intermediate keratin
filaments Q in the cytoplasm.

109. Ans. (c) Masson fontana stain


(Ref: Histopathology p150)
Stain Substance
Masson Fontana Melanin
Oil red O Neutral lipids and fatty acids
PAS Glycogen, mucin, mucoprotein, glycoprotein
and fungi
Gomori methamine silver Fungi (like Cryptococcus, Coccidiodes and
stain Pneumocystis jiroveci (carinii)
• Other stains for melanin are Schmorl’s method and enzyme
histochemical method called DOPA-oxidase (most
specific method).
110. Ans. (b) Causes carcinogenesis
(Ref: Robbins 9/e p67)
• Telomerase is a specialized RNA-protein complex that uses
its own RNA as a template for adding nucleotides to the
ends of chromosomes.
• Regulatory protein sense the telomere length and they
restrict the activity of telomerase to prevent unnecessary
elongation.
• Telomerase activity is highest in germ cells and present
at lower levels in stem cells, but it is usually undetectable in
most somatic tissues
Decreased activity of telomerase is associated with ageing
whereas its excessive activity is associated with cancers.
111. Ans. (c) Nigrosin

• Negative staining is a technique in which the background is stained, leaving


the actual specimen untouched, and thus visible. In contrast, with ‘positive
staining’, the actual specimen is stained.
• Examples of negative stains include nigrosin and India Ink.
• India ink is used to make a diagnosis of cryotococcal infection by making its
capsule prominent.

112. Ans. (a) Frozen section


(Ref: Bancroft Histology 7th/83)
Fresh cryostat sections must be used for oil red O stains
113. Ans. (b) 10% formalin
(Ref: Bancroft Histological techniques..page 70)
114. Ans. (b) Hydroxyl
(Ref: Robbins 9th/47)
Hydroxyl radical is the most reactive free radical.
115. Ans. (b) Toluidine blue
Premalignant lesions have abnormal cells containing high DNA/RNA
thereby making these cells hyperchromatic. The same can be
detected grossly with the help of stains like toluidine blue.
It is important to know that for detection of these lesions under the
microscope, we either use H&E (hemaotxylin and eosin) and
toluidine blue.
116. Ans (b) Focal length of objective lens
(Ref: Bancroft Histological techniques..page 45-47)
Resolving power of a light microscope is NOT affected by Focal length
of objective lens. Rest all mentioned options do affect the
resolving power.
117. Ans (b) Glutathione
(Ref: Robbins 9/e p47-48)
118. Ans (c) Herpes
(Ref: Robbins 9/e p336)
Lipschutz bodies are seen in herpes.
119. Ans (a) Ferric ions
(Ref: Robbins 9/e p650)

1. Which of the following is the etiology of Werner syndrome?
(NEET 2020 like pattern)
(a) Telomerase lengthening
(b) Lamin mutation
(c) Lipid peroxidation
(d) DNA helicase defect
Ans. (d) DNA helicase defect
(Ref: Robbins 9th/66)
• Werner syndrome in patients presents with premature
aging, and the defective gene product is a DNA helicase, a
protein involved in DNA replication and repair and other
functions requiring DNA unwinding.
• A defect in this enzyme causes rapid accumulation of
chromosomal damage that may mimic the injury that
normally accumulates during cellular aging.
2. A 45-year-old person who is a chronic smoker came to the
clinic with complaints of cough. The physician examines the
patient and take a biopsy. The picture in the biopsy was as
the description below. Which of the following cellular
changes has happened in this patient?
(NEET 2020 like pattern)

(a) Hyperplasia
(b) Dysplasia
(c) Metaplasia
(d) Anaplasia
Ans. (c) Metaplasia
(Ref: Robbins 9th/37-8)
• Chronic smoker with cough has a change in the nature of epithelium. As is
clear from the image, the pseudostratified ciliated columnar epithelium is being
replaced by squamous epithelial cells, this change is descriptive of metaplasia.
• This is also the commonest example of epithelial metaplasia.

3. Which of the following is activated by intrinsic or extrinsic


pathways?
(AIIMS Nov 2019 like pattern)
(a) Necroptosis
(b) Apoptosis
(c) Necrosis
(d) Pyroptosis
Ans. (b) Apoptosis
(Ref: Robbins 9th/ 15)
Programmed cell death like apoptosis is a central feature of normal
tissue development and turnover and can be triggered by
extrinsic signals (including cytotoxic T cells and inflammatory
cytokines), or intrinsic pathways (including DNA damage and
intracellular stress).
4. All are features of reversible cell injury except:
(AIIMS May 2019 like pattern)
(a) Endoplasmic reticulum swelling
(b) Dense deposition of mitochondria
(c) Bleb formation
(d) Detachment of ribosome
Ans. (b) Dense deposition of mitochondria
(Ref: Robbins 9th/40)
Reversible injury is characterized by generalized swelling of the cell
and its organelles, blebbing of the plasma membrane,
detachment of ribosomes from the ER, and clumping of nuclear
chromatin.
5. Which of the following process does not show phases of
acute inflammation?
(AIIMS May 2019 like pattern)
(a) Necrosis
(b) Pyroptosis
(c) Apoptosis
(d) Necroptosis
Ans. (c) Apoptosis
(Ref: Robbins 9th/56-7)
• Apoptosis is “caspase-dependent” programmed cell death. Cells that are
dying by apoptosis secrete soluble factors that recruit phagocytes. The
process of phagocytosis by apoptotic cells is so efficient that dead cells
disappear, often within minutes, without leaving a trace, and inflammation is
absent even in the face of extensive apoptosis.
• Necroptosis is sometimes called programmed necrosis and “caspase-
independent” programmed cell death. It resembles necrosis morphologically
which means it is associated with release of lysosomal enzymes and
ultimately rupture of the plasma membrane. So, it is associated with presence
of inflammation.
• Pyroptosis is characterized by swelling of cells, loss of plasma membrane
integrity, and release of inflammatory mediators.

6. Dystrophic calcification seen in:


(AIIMS May 2019 like pattern)
(a) Myositis ossificans
(b) Paget’s disease
(c) Metastasis
(d) Sarcoidosis
Ans. (a) Myositis ossificans
(Ref: Robbins 9th/38)
• Connective tissue metaplasia is the formation of cartilage, bone,
or adipose tissue in tissues that normally do not contain these
elements. The most common example is myositis ossificans.
(bone formation in muscle occurring after intramuscular
hemorrhage).
Also revise that
• MC Epithelial metaplasia is squamous metaplasia in lungs
smokersQ

7. Caspase involved in activation of IL-1 is which of the


following?
(NEET 2019 like pattern)
(a) Caspase 1
(b) Caspase 3
(c) Caspase 5
(d) Caspase 8
Ans. (a) Caspase 1
(Ref: Robbins 9th e/p 59)
Pyroptosis occurs in cells infected by microbes. It involves activation
of caspase-1 which cleaves the precursor form of IL-1 to
generate biologically active IL-1. Caspase-1 along with closely
related caspase-11 also cause death of the infected cell.
8. Which of the following is an antiapoptotic gene?
(NEET 2019 like pattern)
(a) Bcl 2
(b) Bcl–XS
(c) BAX
(d) BAD
Ans. (a) Bcl 2
(Ref: Robbins 9th e/p 53-5)

• Anti-apoptotic genes: BCL2, BCL-XL, and MCL1 are the principal members
of this group;
• Pro-apoptotic genes: BAX and BAK are the two prototypic members of this
group
• Sensors: Members of this group, including BAD, BIM, BID, Puma, and Noxa.
They regulate the balance between the other two groups, thus acting as
arbiters of apoptosis.

9. Which of the following type of necrosis is seen in blood


vessel due to immune complex mediated damage?
(NEET 2019 like pattern)
(a) Coagulative necrosis
(b) Liquefactive necrosis
(c) Fibrinoid necrosis
(d) Caseous necrosis
Ans. (c) Fibrinoid necrosis
(Ref: Robbins 9th e/p 44
Fibrinoid necrosis is a special form of necrosis usually seen in
immune reactions involving blood vessels. This pattern of
necrosis typically occurs when complexes of antigens and
antibodies are deposited in the walls of arteries. Deposits of
these “immune complexes,” together with fibrin that has leaked
out of vessels, result in a bright pink and amorphous
appearance.
10. False statement about Barret esophagus is:
(NEET 2019 like pattern)
(a) Goblet cells seen on histology
(b) Patient may lead to malignancy after few years
(c) Columnar to squamous metaplasia
(d) Chronic GERD is the predisposing factor
Ans. (c) Columnar to squamous metaplasia
(Ref: Robbins 9th e/p 757)
Barret esophagus is associated with squamous to intestinal
columnar metaplasia.
11. Which of the following is false about apoptosis?
(AIIMS Nov 2018 like pattern)
(a) No inflammation
(b) Plasma membrane intact
(c) Organelle swelling
(d) Affected by dedicated genes
Ans. (c) Organelle swelling
(Ref: Robbins 9th e/p 52-5)

Features of apoptosis

• Is controlled by genes
• May be physiological or pathological
• Important for development, homeostasis & elimination of pathogens & tumor
cells
• Affect single cells
• Cell size is shrunken
• Active
• No inflammatory reaction
• Plasma membrane is intact
• Step ladder pattern is seen

12. Identify the marked structure in given below image:


(AIIMS Nov 2018 like pattern)

(a) Lipofuscin granules


(b) Golgi apparatus
(c) Secondary lysosomes
(d) Mitochondria
Ans. (a) Lipofuscin granules
(Ref: Robbins 9th e/p 64)
Lipofuscin is identified in the electron microscopy by its perinuclear
and intralysosomal location.
13. Which of the following labels corresponds to the condenser
of the microscope?
(AIIMS May 2018 like pattern)
(a) A
(b) B
(c) C
(d) D
Ans. (c) C
See the image below to have an idea about the parts of the
microscope.
14. Which of the following stain is used for Acidic mucin?
(AIIMS May 2018 like pattern)
(a) Alcian blue
(b) PAS
(c) Masson’s trichrome stain
(d) PTAH
Ans. (a) Alcian blue
(Ref: Bancroft 7th e/ p 225)
• Sections are stained with a standard alcian blue (pH 2.5) method followed by
PAS.
• The alcian blue stains sialomucins, sulfomucins, and proteoglycans blue.
• Neutral mucins are stained deep red/magenta with the PAS.
• Tissues and cells that contain both neutral and acidic mucins will stain varying
shades of purple due to the binding of alcian blue and the reactivity with Schiff
reagent.

Stains for mucin with important features


• Alcian blue - The pH of this stain can be adjusted to give more specificity.
• PAS (peroidic acid-Schiff) - Stains glycogen as well as mucins, but tissue
can be pre-digested with diastase to remove glycogen.
• Mucicarmine: Very specific for epithelial mucins.

15. Toluidine blue staining is used for identification of which of


the following cells?
(AIIMS May 2018 like pattern)
(a) Mast cell
(b) Fibroblast
(c) Melanocyte
(d) Macrophages
Ans. (a) Mast Cell
(Ref: Robbins 9th e/p 202)

Mast cells are bone marrow–derived cells that are abundant near
blood vessels and nerves and in subepithelial tissues. Mast cells
have cytoplasmic membrane-bound granules which contain
acidic proteoglycans that bind basic dyes such as toluidine blue.
They may be difficult to differentiate from lymphocytes in routine,
hematoxylin and eosin–stained sections, and special
metachromatic stains (toluidine blue or Giemsa) must be
used to visualize their granules.
16. Fixative agent for PAP smear is which of the following?
(AIIMS Nov 2017 Pattern)
(a) Normal saline
(b) 95% ethanol
(c) Formalin
(d) Air drying
Ans. (b) 95% ethanol
(Ref: Robbins 9/e p340)
The rule about fixing the smear immediately applies here as well. A
solution of ether 50 % and alcohol (95%) 50 % in a small
widemouthed jar can be used, keeping the lid on. Two slides at a
time can be placed back to back in the jar for 15 minutes after
which they are removed and allowed to dry and examined under
microscope.
Important fixatives for future questions
Tissue Fixatives fluid
Light microscopy 10% formaldehyde
Electron microscopy 2% glutaraldehyde
Bone marrow biopsy Zenker fluid
Bone marrow aspirate Helly fluid

17. In the following liver biopsy, which special stain has been
used?
(AIIMS May 2017 Pattern)

(a) Masson’s trichrome stain


(b) Grimelius silver stain
(c) Steiner silver stain
(d) Sweet’s reticulin stain
Ans. (d) Sweet’s reticulin stain
The Science of Laboratory Diagnosis pg 19,
(Ref: Robbins Basic Pathology 10/e p163)
• Reticulin fibres are demonstrated by silver impregnation technique one of
which is Sweet’s reticulin stain.
• Reticulin fibres can also be demonstrated by the PAS technique.

Disclaimer
Any resemblance to an actual question is purely coincidental.
• Celsus was the frost person to describe the four cardinal signs of
inflammation: rubor (redness), tumor (swelling), calor (heat), and dolor
(pain). These signs are hallmarks of acute inflammation.
• Rudolf Virchow added the fifth clinical sign ‘loss of function’ (function
laesa).
• Elie Metchnikoff discovered the process of phagocytosis by observing
the ingestion of rose thorns by amebocytes of starfish larvae and of
bacteria by mammalian leukocytes.
• Sir Thomas Lewis established the concept that chemical substances,
such as histamine (produced locally in response to injury), mediate the
vascular changes of inflammation.
• Increased vascular permeability is the hallmark feature of acute
inflammation.
• Endothelial cell contraction is the most common mechanism of
increased vascular permeability.
• “Selectins” are responsible for ‘rolling’ whereas “integrins” are required
for “adhesion”.
• Hallmark of acute cytokine mediated acute inflammation: Endothelial
expression of E-selectin. Selectin family includes E-selectin, L-selectin and
P-selectin (not A-selectin).
• Transmigration (also called diapedesis) requires PECAM molecule or
CD31.
• Chemotaxis: single direction targeted movement of WBCs like neutrophils
caused by exogenous molecule (bacterial products) or endogenous
molecules (C5a, LTB4 or IL-8).
• Opsonisation requires special chemicals called opsonins (complement
proteins like C3b, lectins and antibodies).
• Phagocytic receptors include mannose receptors, scavenger
receptors and receptors for various opsonins.
• Scavenger receptors were originally defined as molecules that bind and
mediate endocytosis of oxidized or acetylated low-density lipoprotein (LDL)
particles that can no longer interact with the conventional LDL receptor.
• The H2O2-MPO-halide system is the most efficient bactericidal system of
neutrophils.
• Nitroblue tetrazolium test is used for monitoring the functioning of
phagocytes and is useful in patients suffering from chronic
granulomatous disease.
• Neutrophil extracellular traps (NETs) are extracellular fibrillar networks
which provide a high concentration of antimicrobial substances at sites of
infection. They are produced by neutrophils in response to chemicals
mainly interferons. NET formation is dependent on platelet activation
and it is associated with the pathogenesis of autoimmune conditions like
SLE.
• In the absence of effective TH17 responses, individuals are susceptible
to fungal and bacterial infections, and the skin abscesses that develop are
“cold abscesses,” lacking the classic features of acute inflammation, such
as warmth and redness.
• Histamine is the most important chemical mediator of acute inflammation.
• Arachidonic acid is derived from the conversion of essential fatty acid
linoleic acid.
• The prostaglandins are involved in the pathogenesis of pain and fever in
inflammation. Also know that PGE2 is hyperalgesic.
• Lipoxins are also generated from AA by the lipoxygenase pathway.
They suppress inflammation by inhibiting the recruitment of leukocytes.
Formation of lipoxins requires two cell populations (leucocytes and
platelets) for the biosynthesis.
• ‘C’ in CRP stands for: Carbohydrate antigen of pneumococcus.
• Complement proteins constitute 5-10% of plasma proteins.
• Eculizumab prevents the conversion of C5 to C5a. This inhibitor not only
reduces the hemolysis and attendant transfusion requirements in patients
of paroxysmal nocturnal hemoglobinuria (PNH), but also lowers the risk
of thrombosis by up to 90%.
• Catarrhal inflammation is the commonest type of acute inflammation.
• Adiponectin, IL-10, IL-6, IL-4 and TGF beta are anti-inflammatory
cytokines
• Stable tissues have a limited capacity to regenerate after injury the only
exception being liver.
• Not seen in acute inflammation: Granuloma formation.
• Not an immune granuloma: Silicosis (it causes non-immune granuloma).
• Important causes of necrotizing granuloma: TB syphilis, histoplasma, Cat’s
scratch disease, Wegner’s granulomatosis, RA, Hodkins disease,
Byssinosis. Please revise that Leprosy is not a cause of necrotizing
granuloma.
• Important macrophages: Histiocytes, Kupffer cells, osteoclasts,
mesangial cells, Hoffbouer cells, Littoral cells, type A synoviocytes.
• Components of basement membrane: Laminin Collagen IV, fibronectin,
tenascin, enatactin, proteoglycons perlecan. Not a component of basement
membrane is Rhodopsin.
• Most abundant glycoprotein in basement membrane: Laminin.
• Degradation of basement membrane is caused by: Metalloproteinases.
• Collagen in hyaline articular cartilage: Type II.
• Characteristic of protective epithelium is Regeneration. ‘Regeneration’ is
replacement of lost tissue by Living tissue of similar kind.
• Granulation tissue is formed by: Budding of new capillaries
(neovascularization). Angiogenesis is formation of new blood vessels.
• Sequence of appearance of cells in wound healing: Platelets-neutrophils-
macrophages-fibroblasts. Fibrosis is due to: TGF-β.
Response of the blood vessels and cells to an injurious stimulus is
called inflammation. It can be:
• Acute inflammation: It is of shorter duration (seconds, minutes, few
hours)
• Chronic inflammation: It is of longer duration (weeks, months and
years)
The changes seen in inflammation can be in the blood vessels (called
vascular changes) and in the cells (called cellular changes).
I. Vascular Changes
1. Vasoconstriction: It is the firstQ change in the blood vessels
which is transient in nature. Clinically it is responsible for the
blanching seen immediately after injury.
2. Vasodilation: Second change in the blood vessels lasting for a
longer duration is vasodilation. It results in increased blood
flow leading to redness (rubor) and the sensation of warmth
(color).
3. Increased permeability: It is the hallmark of acute
inflammationQ caused by separation of the endothelial cells
resulting in movement of fluid, cells and proteins out of the
blood vessels (collectively called exudate). The exudate is a
protein rich fluid which is responsible for the swelling (tumor)
associated with an injury. It is maximally seen in the venules.
The various mechanisms of increased vascular permeability
are explained below:
4. The loss of fluid results in concentration of red cells in small
vessels and increased viscosity of the blood leading to slower
blood flow and is called stasis.

II. Cellular Changes


The sequence of events in the journey of leukocytes from the vessel
lumen to the interstitial tissue, called extravasation, can be divided
into the following steps:
1. Margination: Movement of the leukocytes which are normally
moving in the centre of the blood vessel towards the periphery
of the blood vessel is called margination.
Affected blood Properties of
Mechanism Caused by vessels response
Mechanisms of Increased Vascular Permeability
Affected blood Properties of
Mechanism Caused by vessels response
1. Formation of Vasoactive Venules Rapid; Reversible;
endothelial gaps mediators like short lived (15 to
(Immediate transient histamine, 30 minutes)
responseQ) leukotrienes,
bradykinin
and
contraction of
endothelial
cell
cytoskeleton
2. Direct endothelial Toxins, Venules, capillaries Fast and may be
injury (immediate infections, and arterioles long lived
sustained burns,
responseQ) chemicals
causing
endothelial
cell necrosis
and
detachment
3. Cytoskeletal Due to Mostly venulesQ; Reversible,
reorganisation cytokines and capillaries may be delayed and
(Endothelial cell hypoxia also involved prolonged
retractionQ)
4. Delayed prolonged Thermal and Venules and Delayed and long
leakage radiation capillaries lived
injury induced
endothelial
cell damage
5. Leukocyte mediated Activated Venules (mostly); Late and long
endothelial injury leukocytes pulmonary and lived
causing glomerular
endothelial capillaries
injury or
detachment
Affected blood Properties of
Mechanism Caused by vessels response
6. Increased transcytosis Formation of Venules
vesiculo-
vacuolar
organelles
near inter
cellular
junctions by
histamine and
VEGF
7. Leakage from new Mostly by Sites of
blood vessel vascular angiogenesis
endothelial
growth factor
(VEGF) and
less
commonly by
histamine and
substance P

2. Rolling: It is the process of transient adhesion of leukocytes


with the endothelial cells. Selectins are the most important
molecules responsible for it. They interact with the
complementary molecules resulting in transient adhesion. The
selectins can be either.

E selectin (CD 62E) – Present on cytokine-activated endothelial cells and


interacts with sialyl lewis X receptor on the leukocyte.
L selectin (CD 62L) – Present on leukocytes and interacts with glycoprotein
adhesion molecules (GlyCAM-1), Mad CAM-1 and CD34 on endothelial cells.
P selectin (CD 62P) – Present on platelets and endothelial cells and interacts
with sialyl lewis X receptor on leukocytes.

3. Adhesion: It is firm attachment of the leukocytes to the


endothelial cells. Integrins are the most important molecules
promoting cell-cell or cell-matrix interactions by interacting
with vascular cell adhesion molecule (VCAM) or intercellular
adhesion molecule (ICAM). These can be of two types:
Contd...
β1-containing integrins: These are also called VLA molecules
and interact with VCAM-1 on endothelial cells.
β2-containing integrins: These are also called LFA-1 or Mac-1
and interact with ICAM-1 on endothelial cells.

4. Transmigration: The step in the process of the migration of


the leukocytesQ through the endothelium is called
transmigration or diapedesis. Neutrophils predominate in the
inflammatory infiltrate during the first 6 to 24 hours, then are
replaced by monocytes in 24 to 48 hours (except in
Pseudomonas infection in which neutrophils predominate over
2 to 4 days).
5. Chemotaxis: It is unidirectional movementQ of the
leukocytes towards antigens/bacteria in response to certain
chemicals. These chemicals are called chemotactic stimuli.
They can be:

a. Exogenous: Bacterial productsQ


b. Endogenous products: C5a, LTB 4, IL-8

All the chemotactic agents mentioned above bind to G-protein


coupled receptors (GPCRs) on the surface of leukocytes to
cause actin polymerization and all movements. Other actin-
regulating proteins like filamin, gelsolin, profilin, and
calmodulin also interact with actin and myosin to produce
contraction and cellular movement. The leukocytes
degranulate to release lysosomal enzymes, cytokines and
produce arachidonic acid metabolites. The leukocyte
activation takes place due to GPCRs, cytokine receptors and
Toll-like receptors (TLRs).

6. Opsonisation: Coating of the bacteria so that they are easily


phagocytosed by the white blood cells is known as
opsonisation.
Chemicals causing opsonisation are called opsonins. These are:
• C3bQ
• Fc fragment of antibody or IgGQ.
• Some serum proteins (like fibrinogenQ, mannose binding lectinQ and C reactive
proteinQ)

7. Phagocytosis: It is the process by which bacteria are


killed/eaten up by the white blood cells. Lysosomes are
important organelles required for phagocytosis.
Phagocytosis: It is characterized by 3 steps
a. Recognition and attachment: The particles to be ingested
by leukocytes (microbes and dead cells) are recognized by
receptors present on the surface of WBCs. These receptors
are.
i. Scavenger receptors: These bind microbes and oxidized
or acetylated LDL particles.
ii. Mac-1 integrins: These are present on the surface of
macrophages.
iii. Mannose receptors: These bind to mannose and fucose
residues of glycoproteins in microbial cell wall. The
presence of an additional terminal sialic acid or N-acetyl
galactosamine in human cells prevents their destruction
by WBCs.
b. Engulfment: There is formation of phagolysosome (due to
fusion of the lysosomes and the phagosome containing the
microbe) inside the leukocytes. This is followed by
degranulation of leukocytes.

Fig. 1: Chediak-Higashi syndrome: giant granules in leukocytes.


c. Killing and degradation: Final step in phagocytosis is the
killing of infectious organism within the leukocytes. It can be
accomplished by
i. Oxygen dependent killing mechanism
There is production of microbicidal reactive oxygen species
within phagocytic vesicles by the following mechanism:
The final step in the microbial killing is due to reactive oxygen
species called as ‘respiratory burst’.
Phagocytes (i.e., neutrophils, monocytes, and macrophages)
require an enzyme to produce reactive oxygen species to
destroy bacteria after they ingest the bacteria in a process
called phagocytosis. This enzyme is termed “phagocyte
NADPH oxidase” (PHOX). The initial step in this process
involves the one-electron reduction of molecular oxygen
to produce superoxide free radical. Superoxide then
undergoes a further series of reactions to produce
products such as peroxide, hydroxyl radical and
hypochlorite. The reactive oxygen species thus produced
are toxic to bacteria and help the phagocyte kill them
once they are ingested.
Chronic granulomatous disease (CGD) has a defective NADPH oxidase
activity with recurrent infection and granuloma formation affecting
gastrointestinal or genitourinary tract. CGD can be diagnosed with the
following tests:
1. Nitroblue-tetrazolium (NBT) test: It is negative in chronic granulomatous
disease and positive in normal individuals. This test depends upon the direct
reduction of NBT by superoxide free radical to form an insoluble formazan.
2. Dihydrorhodamine (DHR) test
3. Cytochrome C reduction assay.

In CGD, some phagocytosed organisms (catalase negative organisms like


steptococci) can be killed because these organisms produce their own
hydrogen peroxide which is used by neutrophilic myeloperoxidase to produce
free radicals and kill them.
In myeloperoxidase deficiency, the enzyme myeloperoxidase is absent, so
both catalase positive and catalase-negative organisms will survive within
phagocytes and cause infections.

Enzymes involved in respiratory burst


– NADPH oxidase is chiefly responsible for the formation of
hydrogen peroxide which plays the most important role in
microbial killing.
– Catalase degrades hydrogen peroxide into water and
oxygen.
– Superoxide dismutase (SOD) causes conversion of
superoxide ion into hydrogen peroxide.
– Glutathione peroxidase causes conversion of reduced
glutathione to its homodimer.
2OH– + 2GSH → 2H O + GSSG 2
Note: H2O2 – MPO – halide system is the most efficient way of killing the bacteriaQ.
ii. Oxygen independent killing mechanism
It can be done by various enzymes and proteins like
Lysozyme: Causes hydrolysis of bacterial glycopeptide coat
Lactoferrin: It is an iron binding protein.
– Bacterial permeability increasing protein.
– Major basic proteinQ (MBP).
– Defensins: These are arginine rich peptides toxic to
the microbes.
– Cathelicidins: These are antimicrobial proteins in the
neutrophils and other cells They are highly effective
against M. tuberculosis.
Fig. 2: Neutrophil extracellular traps.

Neutrophil extracellular traps (NETs)


• NETs are extracellular fibrillar networks which provide a high
concentration of antimicrobial substances at sites of infection.
They are produced by neutrophils in response to chemicals
mainly interferons. NET formation is dependent on platelet
activation and it is associated with the pathogenesis of
autoimmune conditions like SLE.

IMPORTANT CHEMICAL MEDIATORS


Chemical mediators of inflammation may be present in cells (cellular)
or in the plasma.
Preformed Cellular Mediators

a. Histamine: It is formed from the amino acid ‘histidine’. Mast cells


are the richest source of histamine. It is also present in platelets
and basophils. It causes vasodilation (but vasoconstriction of
large arteries), increased permeability (immediate transient
response) and bronchoconstriction.
b. Serotonin (5-HT): Richest source of serotonin (5-
hydroxytryptamine; 5- HT) is platelets. It has actions similar to
histamine. It is also present in enterochromaffin cells.
c. Lysosomal Enzymes: These are present in the lysosomes of
neutrophils and monocytes. Lysosomes contain two types of
granules; Primary (azurophilic) and secondary (specific) granules.
– Two major anti-proteases present in the body are a1
antitrypsin and a2 macroglobulin
– Neutrophils also have tertiary granules or C particles
which contain gelatinase and acid hydrolases.

Newly Synthesized Cellular Mediators

a. Nitric Oxide (NO): It is formed from l-arginine with the help of


enzyme nitric oxide synthase (NOS).
NADPH + O2
L-Arginine L-Citrulline + NO
NOS
Three isoforms of NOS are present in the body:
i. e NOS (Present in endothelium)
ii. n NOS (Present in neurons)
iii. i NOS (inducible form)
eNOS and nNOS are constitutively expressed whereas i NOS
production is induced by cytokines like TNF a and IFN-g.

Important Actions of NO

� Potent vasodilator
� Reduction of platelet aggregation
� Endogenous regulator of leucocyte recruitment
� Also possess microbicidal action: NO acts as a free radical and can also
be converted to highly reactive peroxynitrite anion (ONOO–) as well as
NO2 and NO3.

b. Cytokines: These are small proteinaceous molecules secreted


by the inflammatory cells. These include interleukins,
interferons and tumor necrosis factor-alpha (TNF-a). These
can produce local and systemic effects. Most important
cytokine responsible for systemic effects of inflammation are
interleukin-1 (IL-1) and tumor necrosis factor-alpha (TNF-a).
IL-1, TNF-a
Acute phase reaction Endothelial and fibroblast WBC effects
- Fever effects • ↑ Secretion of IL-1
- Increased sleep • ↑ PGI2 formation and IL-6
- Decreased appetite • ↑ Procoagulant
- Neutrophilia • ↑ Fibroblast
• ↑ Collagen

c. Arachidonic acid metabolites : Arachidonic acid (AA) is a 20-C


fatty acid containing four double bonds. It must be
released/mobilized from membrane phospholipids (PL) for
oxygenation to various compounds.

i. COX pathway: Two type of COX-enzymes (also known


as PGH synthase); COX-1 and COX-2 convert AA to
PGG2 first and then to PGH2 [both are called cyclic
endoperoxides]. Further fate of PGH2 depend upon the
enzyme present in a particular cell e.g. endothelium
contain PGI2 synthase and thus forms PGI2 whereas
platelets contain TXA2 synthase and therefore synthesize
TXA2.

FLAP- Five Lipoxygenase Activating Protein


ii. LOX-pathway: AA can be acted upon different types of
LOX enzymes.

1. 5-LOX (present in leukocytes, mast cells and dendritic cells) acts in the
presence of FLAP [5-LOX activating protein] to convert AA to LTA4. This
product can be converted either to LTB4 or to cysteinyl Leukotrienes (LTC4,
D4 and E4).
2. 15-LOX converts AA to 15-HETE which can be converted to Lipoxins
(LXA4 and LXB4) with the action of 5-LOX. Lipoxins can also be synthesized
by action of 12-LOX on LTA4.

iii. Epoxygenase pathway: Cytochrome P450 may convert


AA to 20-HETE or EET. Biological effects of EET are
reduced by metabolism to less active DHET with the help
of epoxide hydrolases.
• EET may function as endothelium derived
hyperpolarizing factor particularly in coronary
circulation. It also possesses anti-inflammatory, anti-
apoptotic and pro-angiogenic action.
• 20-HETE cause vasoconstriction of renal arteries and
has been implicated in the pathogenesis of
hypertension. In contrast, EET possess
antihypertensive properties via its vasodilating and
natriuretic actions. Inhibitors of epoxide hydrolase
[results in elevated levels of EET] are being
developed as antihypertensive drugs.
Abbreviations: DHET – Dihydroxyeicosatrienoic acid, EET – Epoxyeicosatrienoic
acid, HETE – Hydroxyeicosatetraenoic acid, HPETE –
Hydroxyperoxyeicosatetraenoic acid.
iv. Isoeicosanoid pathway: Isoprostanes are prostaglandin
stereoisomers. These are formed by non-enzymatic free-
radical based peroxidation of AA and related lipid
substrates. These have potent vasoconstrictor activity and
modulate WBC and platelet adhesive interactions and
angiogenesis.
d. Chemokines: Chemokines are a family of small proteins
that act primarily as chemoattractants for specific types
of leukocytes. They are classified into 4 major groups
according to the arrangement of conserved cysteine
residue in mature proteins. Most the chemokines have
four conserved cysteine residues (expressed as C). X
means amino acid other than C. Thus, C-X-C means two
conserved cysteines are separated by one amino acid
and C-X3-C means separation by three amino acids. C-C
means no separation and C-chemokines lack first and
third conserved cysteine residues.

Note: Chemokines mediate their actions through chemokine receptors


(CXCR or CCR). Certain receptors (CXCR4; CCR5) act as co-receptors for
binding and entry of HIV into CD4 cells.
Mediators Present in Plasma

a. Complement system
It consists of 20 complement proteins (and their breakdown products)
present in the plasma. They constitute 5-10% of plasma proteins.
These are numbered C1 to C9. The complement system has the
following four pathways:
– Classic activation pathway activated by antigen/antibody
immune complexes,
– Mannose binding lectin activation pathway activated by
microbes with terminal mannose groups
– Alternative activation pathway activated by microbes or
tumor cells
– Terminal pathway that is common to the first three pathways
and leads to the membrane attack complex that lyses cells.
Irrespective of the initial pathway, all the three cause break down
of activation of C3 and result in the formation of membrane attack
complex (MAC). This complex causes antigenic destruction.
FUNCTIONS OF IMPORTANT INDIVIDUAL COMPLEMENT
PROTEINS
• C3a and C5a are also called anaphylatoxins which are chemicals
causing release of histamine from mast cells. So, they cause
vasodilation and increased vascular permeability.
• C3b and inactive C3 (C3i) used for opsonisation.
• C5a also has important role in chemotaxis.
• C5b-9 (Membrane Attack Complex; MAC) attacks and kills the
antigen.

REGULATORY MOLECULES OF COMPLEMENT SYSTEM


• Decay accelerating factor (DAF) increases the dissociation of C3
convertase.
• Factor I proteolytically cleaves C3b.
• CD59 (Membrane inhibitor of reactive lysis) inhibits formation of
MAC.
• Factor H, factor I and CD46 prevent exessive alternate pathway
activation.
Deficiency of Disease/Syndrome
complement
component

1. C1 esterase Hereditary angioneurotic


Inhibitor edema (subcutaneous
edema because of
excessive complement
activation)

2. Early SLE and collagen


complement vascular disorders
proteins C1,
C2, C4

3. C3b and C3b Recurrent pyogenic


inactivator infections

4. C5 to C8 Bacterial infections with


Neisseria and
Toxoplasmosis

5. C9 No particular disease

6. DAF and Paroxysmal nocturnal


CD59 hemoglobinuria
(complement mediated
increased intravascular
lysis of RBCs, platelets
and neutrophils)

7. CD46, Atypical or ‘non


factors H and I epidemic’ hemolytic
uremic syndrome (HUS)

b. Clotting system
A brief overview is presented here and additional details of
coagulation cascade are mentioned in chapter 3.
The most important function of clotting system activation is
formation blood clot that helps to preventexcessive blood loss.
Some of the components of the clotting system also play other
roles e.g. fibrinogen is used for opsonisation and thrombin
causes chemotaxis.
c. Kinin System
It is initiated by activated factor XII (Hageman’s factor)

Kallikrein can also activate plasminogen into plasmin and cause


activation of complement protein C5a.

Functions of Bradykinin:
1. Contraction of smooth muscles
2. Pain
3. Dilation of the venules
The most important outcome of acute inflammation is clearance of
the injurious stimuli and replacement of injured cells (resolution).

Morphological patterns of inflammation


Serous Fibrinous Catarrhal Purulent
inflammation inflammation inflammation inflammation
*Presence of *Deposition of fibrin *Epithelial surface *Purulent exudate
outpouring of thin in extracellular inflammation is made of necrotic
fluid space due to large causes increased cells, neutrophils
*Effusion is fluid vascular leaks mucus secretion and edema fluid
accumulation in *Abscess is
*Characteristic of *Seen in common
cavities localized collection
inflammation in cold
of purulent
body cavity linings
inflammatory tissue
(meninges,
pericardium)

CHRONIC INFLAMMATION

Chronic inflammation is characterized by infiltration with mononuclear


cells (including macrophages, lymphocytes, and plasma cells), tissue
destruction and healing by replacement of damaged tissue via
angiogenesis and fibrosis. Macrophage is the dominant cell in chronic
inflammation. It accumulates inside the tissue because of recruitment
from circulation; local proliferation in tissue and immobilization at the
site of inflammation. Tissue destruction is the hallmark of chronic
inflammation.

Stem cell Gives rise to Monocyte Macrophage


(Bone marrow) monoblast (Present in blood) (Present in tissues)

Macrophages have a life span ranging from months to years and


they are given different names in different tissues e.g.
Liver - Kupffer cell
CNS - Microglia
Bone - Osteoclast
Lung - Alveolar macrophage or ‘Dust cells’
Connective tissue - Histiocyte
Placenta - Hoffbauer cells
Spleen - Littoral cells
Kidney - Mesangial cells
Synovium - Type A lining cells

Subsets of Activated Macrophages


Classically activated macrophages (M1) Alternatively activated
macrophages (M2)
Induced by microbial products and Induced by microbial products and
cytokines like IFN-gQ. cytokines like IL-4, IL-5Q
Release lysosomal enzymes, nitric oxide, Release IL-10Q, TGF-bQ
IL-1and IL-12
Involved in microbicidal activities and Involved in anti-inflammatory actions
pathogenic inflammationQ and wound repairQ

GRANULOMATOUS INFLAMMATION

It is a type of chronic inflammation characterized by formation of


granuloma. Granuloma is an aggregation of macrophages surrounded
by a collar of mononuclear cells principally lymphocytes. Macrophages
may get activated to form epithelioid cells (epithelium like cells). Some
of the cells may fuse together to form a bigger cell called a giant cell.
The giant cells can be primarily of the following types:
1. Langhans giant cell: The nuclei in this giant cell are present in the
periphery and in a horse shoe pattern. It is seen is tuberculosis.

Fig. 3: Langhans giant cell.

2. Foreign body giant cell: The nuclei are arranged randomly or


haphazardly here. It is seen in granuloma formed by foreign bodies
like sutures, talc etc.
3. Touton giant cells are seen in xanthomas, fat necrosis,
xanthogranulomatous inflammtion and dermatofibroma. They are
formed by fusion of epithelioid cells and contain a ring of nuclei
surrounded by foamy cytoplasm.
Fig. 4: Touton giant cells.

4. Physiological giant cells are seen in osteoclasts,


syncytiotrophoblasts and megakaryocytes.

Fig. 5: Warthin-Finkeldey giant cells of measles.


Fig. 6: Reed-Sternberg cells of Hodgkin’s lymphoma.

Common conditions resulting in granuloma formation with


important features
• Tuberculosis
• Sarcoidosis (Non caseating granulomaQ)
• Brucellosis
• Syphilis (GummaQ)
• Lymphogranuloma inguinale
• Leprosy
• Inflammatory bowel disease (IBD)
The formation of a granuloma is discussed later in the chapter of
‘immunity’.
Fig. 7: Stellate granuloma in cat scratch disease.

Types of Cells

Depending on the regenerative capacity, cells can be divided into 3


categories:
1. Permanent cells Cells of the body which never divide e.g. neurons,
skeletal muscle fibres and cardiac myocytes.
2. Stable cells They have a low rate of multiplication and are usually
present is the G0 phase. When given a stimulus, they
enter the G1 phase and multiply e.g. Cells of proximal
tubule of kidney, hepatocytes, pancreatic cells,
fibroblasts etc.
3. Labile cells These cells can regenerate throughout life e.g.
hematopoietic cells, cells of skin, gastrointestinal
mucosa etc.

WOUND HEALING

It is characterized by the process of regeneration of the damaged


tissue by cells of the same type and replacement of the lost tissue with
connective tissue. Regeneration results in complete restitution of lost
or damaged tissue.
Repair consists of a combination of regeneration and scar
formation by the deposition of collagen. It may restore some original
structures but can cause structural derangements.

Healing by Primary Intention

The healing of a clean uninfected wound is called healing by first


intention or primary union. It involves the following changes.

Day Features of wound

Day 0 (when the wound has Presence of blood clot in the incision
formed)

Day 1 (within 24 hours) Neutrophilic infiltration + blood clot

Day 2 (24 to 48 hours) Neutrophils + blood clot + continuous thin


epithelial layerQ

Day 3 Macrophages replace neutrophils, Appearance of


granulation tissue, type III collagen deposition
begins but do not bridge the incision

Day 5 Abundant granulation tissue


- Collagen fibrils bridge the incision
- Neovascularisation is maximumQ
- Full epithelial thickness with surface
keratinization

End of 2nd week Accumulation of collagen; fibroblast proliferation

1 month Replacement of collagen type III with collagen


type I (has greater tensile strength) due to action
of collagenase enzyme
Vitamin C is required for the conversion of tropocollagen to
collagen due to hydroxylation of lysine and proline residues providing
stability to collagen molecules.

Healing by Secondary Intention

During healing by secondary intention or secondary union;


inflammatory reaction being more intense, granulation tissue is
abundant and a large scar is formed. The scar decreases in size after
sometime; this is called scar contraction.

Wound strength is 10% after 1 weekQ; it increases rapidly during next 4


weeksQ and becomes 70% at the end of 3rd monthQ. The tensile strength of
the wound keeps on increasing as time progresses.
The predominant collagen in adult skin is type IQ whereas in early
granulation tissue, it is type III and IQ.
The balance between extracellular matrix (ECM) synthesis and
degradation results in remodeling of the connective tissue framework
which is an important feature of chronic inflammation and wound
repair. The collagen degradation is done by zinc dependent matrix
metalloproteinases (MMP). Collagen degradation is important for
tissue remodeling, angiogenesis and cancer metastasis. That is why
zinc deficiency is associated with impaired wound healing. MMPs
are synthesized by several cells like fibroblasts, macrophages,
neutrophils, synovial cells, and some epithelial cells. Activated
collagenases (a type of MMP) are rapidly inhibited by specific tissue
inhibitors of metalloproteinases (TIMPs), which are produced by most
mesenchymal cells, thus preventing uncontrolled action of these
proteases. The regulated activity is required for proper wound healing.

During wound healing, complications can arise from:


1. Delayed wound healing: Due to foreign body, ischemia, diabetes,
malnutrition, hormones (glucocorticoids), infection or scurvy.
2. Deficient scar formation: Dehiscence or rupture of a wound is most
common after abdominal surgery and is due to increased
abdominal pressure.
3. Excessive formation of the repair components: Certain conditions
may arise because of increased granulation tissue or excessive
collagen leading to keloid, hypertrophic scar and ‘proud flesh’.
Incisional scars or traumatic injuries may be followed by exuberant
proliferation of fibroblasts and other connective tissue elements
called desmoids or aggressive fibromatoses. These recur
frequently after excision.
4. Formation of contractures: Contractures are particularly prone to
develop on the palms, the soles, and the anterior aspect of the
thorax. These are commonly seen after serious burns.
Fig. 8: Keloid.

STEM CELLS

Definition: The most widely accepted stem cell definition is a cell with
a unique capacity to produce unaltered daughter cells (self-renewal)
and to generate specialized cell types (potency).
1. Self-renewal can be achieved in two ways:
Asymmetric cell division Symmetric cell division
• Produces one daughter cell that is identical • Produces two identical
to the parental cell and one daughter cell daughter cells
that is different from the parental cell and is
a progenitor or differentiated cell
• Asymmetric cell division does not increase
the number of stem cells.

For stem cells to proliferate in vitro, they must divide symmetrically.


Self-renewal alone cannot define stem cells, because any
established cell line, e.g., HeLa cells proliferate by symmetric cell
division.
2. Potency is used to indicate a cell’s ability to differentiate into
specialized cell types. This can be classified as:
Totipotent cells Multipotent cells Oligopotent Uni/Monopotent
cells cells
Can form an entire Can form multiple Can form more Can form a single
organism cell lineages but than one cell differentiated cell
autonomously. cannot form all of lineage but are lineage.
the body’s cell more restricted
lineages. than multipotent
cells.
Only zygoteQ HematopoieticQ NeuronQ stem SpermatogonialQ stem
(fertilized egg) has stem cells cells cells
this feature.

Terminally differentiated cells, such as fibroblast cells, also have a


capacity to proliferate (which may be called self-renewal) but maintain
the same cell type (e.g., no potency to form another cell type) and are
not, therefore, considered unipotent cells.
Embryonic stem cells are pluripotent, that is, they are capable of
forming all the tissues of the body
Adult stem cells are usually only able to differentiate into a
particular tissue.
• Stem cells are located in special sites called niches.
Name of the cell Location Function
Oval cellsQ Canals of Herring of the Forming hepatocytes and biliary
liver cells
Satellite cellsQ Basal lamina of Differentiate into myocytes after
myotubules injury
Limbus cellsQ Canals of SchlemmQ Stem cells for the corneaQ
Ito cellsQ Subendothelial space of Store vitamin AQ
DisseQ
Paneth cells Bottom of crypts Host defense against
microorganisms

Other sites for stem cells are the base of the crypts of the colon
and the dentate gyrus of the hippocampus.

Other Important Concepts in Stem Cell Biology


• Development naturally progresses from totipotent fertilized eggs to
pluripotent epiblast cells, to multipotent cells, and finally to
terminally differentiated cells.
• Nuclear reprogramming: The reversal of the terminally
differentiated cells to totipotent or pluripotent cells (called nuclear
reprogramming) has been achieved using nuclear transplantation,
or nuclear transfer (NT), procedures (often called “cloning”), where
the nucleus of a differentiated cell is transferred into an enucleated
oocyte.
• Stem cell plasticity or trans-differentiation: The prevailing
standard in develop-mental biology is that once cells are
differentiated, their phenotypes are stable. However, a number of
reports have shown that tissue stem cells, which are thought to be
lineage-committed multipotent cells, possess the capacity to
differentiate into cell types outside their lineage restrictions (called
trans-differentiation). For example, hematopoietic stem cells may
be converted into neurons as well as germ cells.
ACUTE INFLAMMATION, VASCULAR AND CELLULAR
CHANGES

1. In acute inflammation endothelial retraction leads to:


(AIIMS Nov 2011)
(a) Delayed transient increase in permeability
(b) Immediate transient increase in permeability
(c) Delayed prolonged increase in permeability
(d) Immediate transient decrease in permeability
2. After binding of complement and antibody on the surface of
encapsulated bacteria, the process of phagocytosis by
polymorphonuclear leukocytes involves which of the
following?
(AIIMS Nov 2011)
(a) Fc and C3b
(b) Receptor-mediated endocytosis
(c) Respiratory burst
(d) Pseudopod extension
3. Free radicals are generated by all except:
(AI 2011)
(a) Superoxide dismutase
(b) NADPH Oxidase
(c) Myeloperoxidase
(d) NO synthase
4. Which among the following is the hallmark of acute
inflammation?
(AI 2011, AIIMS May 2010)
(a) Vasoconstriction
(b) Stasis
(c) Vasodilation and increase in permeability
(d) Leukocyte margination
5. Main feature of chemotaxis is:
(AIIMS May 2010)
(a) Increased random movement of neutrophils
(b) Increase adhesiveness to intima
(c) Increased phagocytosis
(d) Unidirectional locomotion of the neutrophils
6. Characteristic of acute inflammation is:
(AI 2009)
(a) Vasodilation and increased vascular permeability
(b) Vasoconstriction
(c) Platelet aggregation
(d) Infiltration by neutrophils
7. Which of the following helps in generating reactive O2
intermediates in the neutrophils?
(AI ’11, 08, AIIMS Nov 2008)
(a) NADPH oxidase
(b) SOD (superoxide dismutase)
(c) Catalase
(d) Glutathione peroxidase
8. Basement membrane degeneration is mediated by:
(a) Metalloproteinases
(AI 2008)
(b) Oxidases
(c) Elastases
(d) Hydroxylases
9. Delayed prolonged bleeding is caused by:
(AI 2008)
(a) Histamine
(b) Endothelial retraction
(c) IL-1
(d) Direct injury to endothelial cells
10. Earliest transient change following tissue injury will be:
(AI 2007)
(a) Neutropenia
(b) Neutrophilia
(c) Monocytosis
(d) Lymphocytosis
11. All of the following vascular changes are observed in acute
inflammation, except:
(AI 2005)
(a) Vasodilation
(b) Stasis of blood
(c) Increased vascular permeability
(d) Decreased hydrostatic pressure
12. The following host tissue responses can be seen in acute
infection, except:
(AI 2002)
(a) Exudation
(b) Vasodilation
(c) Margination
(d) Granuloma formation
13. Oxygen dependent killing is done through
(AI 2007)
(a) NADPH oxidase
(b) Superoxide dismutase
(c) Catalase
(d) Glutathione peroxidase
14. Which of the following is not true?
(AIIMS May 2009)
(a) NADPH oxidase generate superoxide ion
(b) MPO kills by OCl–
(c) Chediak-Higashi syndrome is due to defective
phagolysosome formation
(d) In Bruton’s disease there is normal opsonization
15. Nitroblue tetrazolium test is used for?
(AIIMS Nov 2008)
(a) Phagocytes
(b) Complement
(c) T cell
(d) B cell
16. In acute inflammation due to the contraction of endothelial
cell cytoskeleton, which of the following results?
(AIIMS Nov 2006)
(a) Delayed transient increase in permeability
(b) Early transient increase
(c) Delayed permanent increase
(d) Early permanent increase
17. Diapedesis is:
(AIIMS Nov 2001)
(a) Immigration of leukocytes through the basement membrane
(b) Immigration of the leukocytes through the vessel wall to the
site of inflammation
(c) Aggregation of platelets at the site of bleeding
(d) Auto digestion of the cells
18. Endothelium leukocyte interaction during inflammation is
mediated by/due to
(PGI, Dec 2003)
(a) Selectins
(b) Integrins
(c) Defensins
(d) Endothelin

MOST RECENT QUESTIONS

19. In genetic deficiency of MPO the increased susceptibility to


infection is due to:
(a) Defective production of prostaglandins
(b) Defective rolling of neutrophils
(c) Inability to produce hydroxyl-halide radicals
(d) Inability to produce hydrogen peroxide
20. After extravasation, leukocytes emigrate in the tissue
towards the site of injury. It is called as:
(a) Margination
(b) Chemotaxis
(c) Diapedesis
(d) Pavementing
21. The complex process of leukocyte movements through the
blood vessels are all except?
(a) Rolling
(b) Adhesion
(c) Migration
(d) Phagocytosis
22. All are true about exudate except:
(a) More protein
(b) Less protein
(c) More specific gravity
(d) All
23. All of the following are signs of inflammation except:
(a) Pain
(b) Swelling
(c) Redness
(d) Absence of functional loss
24. Endogenous chemoattractant is:
(a) C5a
(b) Bacterial products
(c) Lipopolysaccharide A
(d) C8
25. Most important for diapedesis?
(a) PECAM
(b) Selectin
(c) Integrin
(d) Mucin like glycoprotein
26. All of the following are a family of selectin except:
(a) P selectin
(b) L selectin
(c) A selectin
(d) E selectin
27. In acute inflammation the tissue response consists of all
except:
(a) Vasodilatation
(b) Exudation
(c) Neutrophilic response
(d) Granuloma formation
28. Which of the following regarding cellular events in acute
inflammation is not correct?
(a)PECAM/CD31 is responsible for neutrophil activation
(b) Components of complement can assist in chemotaxis
(c) Neutrophil margination is assisted by selectins
(d) ICAM-1/VCAM-1 is responsible for neutrophil adhesion
29. The function common to neutrophils, monocytes, and
macrophages is:
(a) Immune response is reduced
(b) Phagocytosis
(c) Liberation of histamine
(d) Destruction of old erythrocytes
30. Correct sequence of events in acute inflammation is?
(a) Rolling, adhesion, margination, transmigration
(b) Margination, rolling, adhesions, transmigration
(c) Adhesion, rolling, transmigration, margination
(d) Margination, adhesion, rolling, transmigration
31. Most important amino acid for formation Neutrophilic
extracellular trap (NET) is?
(a) Leucine
(b) Methionine
(c) Citrulline
(d) Valine

CHEMICAL MEDIATORS OF INFLAMMATION

32. The role of bradykinin in process of inflammation is:


(a) Vasoconstriction
(AIIMS May 2012)
(b) Bronchodilation
(c) Pain
(d) Increased vascular permeability
33. Which of the following is not a pyrogenic cytokine?
(a) IL - 1
(b) TNF
(AI 2012)
(c) IFN - α
(d) IL - 18
34. All of the following are true in respect of angioneurotic
edema except?
(AI 2012)
(a) It is caused by deficiency of complement proteins
(b) It is more common in females
(c) It manifests as pitting edema
(d) It is an autosomal dominant disorder
35. Which of the following complement component can be
activated is both common as well as alternative pathways?
(AI 2011)
(a) C1
(b) C2
(c) C3
(d) C4
36. Which of the following is not an inflammatory mediator?
(AIIMS Nov 2010)
(a) Tumor Necrosis Factor
(b) Myeloperoxidase
(c) Interferons
(d) Interleukin
37. Nephrocalcinosis in a systemic granulomatous disease is
due to:
(AIIMS Nov 2010)
(a) Overproduction of 1,25 dihydroxy vitamin D
(b) Dystrophic calcification
(c) Mutation in calcium sensing receptors
(d) Increased reabsorption of calcium
38. Bradykinin is for:
(AI 2010)
(a) Pain
(b) Vasodilatation
(c) Vasoconstriction
(d) Increase vascular permeability
39. Most important bactericidal agent is:
(AI 2009)
(a) Cationic basic protein
(b) Lactoferrin
(c) Lysozyme
(d) Reactive O2 species
40. Bradykinin causes:
(AI 2008)
(a) Vasoconstriction
(b) Pain at the site of inflammation
(c) Bronchodilation
(d) Decreased vascular permeability
41. Lewis triple response is caused due to:
(a) Histamine
(AI 2008)
(b) Axon reflex
(c) Injury to endothelium
(d) Increased permeability
42. Factor present in the final common terminal complement
pathway is:
(AI 2007)
(a) C4
(b) C3
(c) C5
(d) Protein B
43. To which of the following family of chemical mediators of
inflammation, the Lipoxins belong?
(AI 2004)
(a) Kinin system
(b) Cytokines
(c) Chemokines
(d) Arachidonic acid metabolites
44. Both antibody dependent and independent complement
pathway converge on which complement component?
(a) C3
(AIIMS Nov 2008) (DNB 2008)
(b) C5
(c) C1q
(d) C8
45. C-C beta chemokines includes:
(AIIMS Nov 2006)
(a) IL-8
(b) Eotaxin
(c) Lymphotactin
(d) Fractalkine
46. All of the following are mediators of acute inflammation
except:
(AIIMS Nov 2005)
(a) Angiotensin
(b) Prostaglandin E2
(c) Kallikrein
(d) C3a
47. All of the following are mediators of inflammation except:
(AIIMS May 2005)
(a) Tumour necrosis factor-a (TNF-a)
(b) Interleukin-1
(c) Myeloperoxidase
(d) Prostaglandins
48. Interleukin secreted by macrophages, stimulating
lymphocytes is:
(AIIMS May 2001)
(a) IFN alpha
(b) TNF alpha
(c) IL-1
(d) IL-6
49. Cytokines are secreted in sepsis and Systemic Inflammatory
Response Syndrome (SIRS) by: (PGI Dec 01)
(a) Neutrophils
(b) Adrenal
(c) Platelets
(d) Collecting duct
(e) Renal cortex
50. Febrile response in CNS is mediated by
(PGI Dec 2003)
(a) Bacterial toxin
(b) IL-l
(c) IL-6
(d) Interferon
(e) Tumor necrosis factor (TNF)
51. Cytokines:
(PGI Dec 2005)
(a) Includes interleukins
(b) Produced only in sepsis
(c) Are polypeptide (complex proteins)
(d) Have highly specific action
52. Conversion of prothrombin to thrombin requires:
(a) V only
(b) V and Ca++
(Delhi PG-2008)
(c) XII
(d) X and Ca++
53. Which complement fragments are called ‘anaphylatoxins’?
(Delhi PG-2006)
(a) C3a and C3b
(b) C3b and C5b
(c) C5a and C3b
(d) C3a and C5a
54. Cryoprecipitate is rich in which of the following clotting
factors:
(Delhi PG-2005)
(a) Factor II
(b) Factor V
(c) Factor VII
(d) Factor VIII
55. Most important mediator of chemotaxis is:
(a) C3b
(b) C5a
(Delhi PG-2005)
(c) C5-7
(d) C2
56. Histamine causes:
(Delhi PG-2004)
(a) Hypertension
(b) Vasoconstriction
(c) Vasodilation
(d) Tachycardia
57. Which of the following is found in secondary granules of
neutrophils?
(UP 2000)
(a) Catalase
(b) Proteolytic enzyme
(c) Gangliosidase
(d) Lactoferrin
58. All are mediators of neutrophils except:
(a) Elastase
(b) Cathepsin
(UP 2004)
(c) Nitric oxide
(d) Leukotrienes
59. Ultra-structurally, endothelial cells contain:
(a) Weibel-Palade bodies
(UP 2004)
(b) Langerhan’s granules
(c) Abundant glycogen
(d) Kallikrein
60. Partial thromboplastin time correlates with:
(UP 2006)
(a) Intrinsic and common pathway
(b) Extrinsic and common pathway
(c) Vessel wall integrity and intrinsic pathway
(d) Platelet functions and common pathway
61. Bleeding time assesses:
(UP 2006)
(a) Extrinsic clotting pathway
(b) Intrinsic clotting pathway
(c) Fibrinogen level
(d) Function of platelets
62. The estimation of the prothrombin level is useful in the
following clotting factor deficiency, except:
(UP 2006)
(a) II
(b) V
(c) VII
(d) IX
63. Which of the following is secondary mediator of the
anaphylaxis is?
(UP 2006)
(a) Histamine
(b) Proteases
(c) Eosinophilic chemotactic factor
(d) Leukotriene B4
64. Birbeck’s granules in the cytoplasm are seen in:
(UP 2006)
(a) Langerhans cells
(b) Mast cells
(c) Myelocytes
(d) Thrombocytes
65. The Eosinophils secrete all except: (UP 2005)
(UP 2007)
(a) Major basic protein
(b) Hydrolytic enzyme
(c) Reactive form of O2
(d) Eosinophilic chemotactic factor
66. In Lipooxygenase pathway of the arachidonic acid
metabolism, which of the following products helps to
promote the platelet aggregation and vasoconstriction?
(UP 2008)
(a) C5a
(b) Thromboxane A2
(c) Leukotriene B4
(d) C1 activators
67. Chemotactic complement components are:
(RJ 2001)
(a) C3a
(b) C5a
(c) Both
(d) C3b
68. In inflammatory process, the prostaglandin E1and E2 cause:
(AP 2000)
(a) Vasodilatation
(b) Increased gastric output
(c) Decreased body temperature
(d) Vasoconstriction

MOST RECENT QUESTIONS


69. Opsonins are:
(a) C3a
(b) IgM
(c) Carbohydrate-binding proteins
(d) Selectins
70. Inflammatory mediator of generalized systemic inflammation
is:
(a) IL-1
(b) IL-2
(c) Interferon alpha
(d) TNF
71. All are cytokines except:
(a) Monoclonal antibody
(b) Interleukin
(c) Chemokine
(d) TNF
72. Cell-matrix adhesions are mediated by?
(a) Cadherins
(b) Integrins
(c) Selectins
(d) Calmodulin
73. Pro inflammatory Cytokines include all of the following
except:
(a) Interleukin 1
(b) Interleukin-10
(c) Interleukin 6
(d) TNF- Alpha
74. Fever occurs due to:
(a) IL-1
(b) Endorphin
(c) Enkephalin
(d) Histamine
75. E cadherin gene deficiency is seen in:
(a) Gastric cancer
(b) Intestinal cancer
(c) Thyroid cancer
(d) Pancreatic cancer
76. The most important source of histamine:
(a) Mast cells
(b) Eosinophil
(c) Neutrophil
(d) Macrophages
77. Following injury to a blood vessel, immediate haemostasis
is achieved by which of the following?
(a) Fibrin deposition
(b) Vasoconstriction
(c) Platelet adhesion
(d) Thrombosis
78. Platelet activating factor causes all except:
(a) Bronchoconstriction
(b) Vasoconstriction
(c) Decreased vascular permeability
(d) Vasodilation
79. Both antibody dependent and independent complement
pathway converge on which complement component?
(a) C3
(b) C5
(c) C1q
(d) C8
80. Cryoprecipitate is rich in which of the following clotting
factors?
(a) Factor II
(b) Factor V
(c) Factor VII
(d) Factor VIII
81. Prostaglandins are synthesized from:
(a) Linoleic acid
(b) Linolenic acid
(c) Arachidonic acid
(d) Butyric acid
82. Eosinophils are activated by:
(a) IL-1
(b) IL-5
(c) IL-4
(d) IL-6
83. Which chemical mediator is an arachidonic acid metabolite
produced by cyclo-oxygenase pathway?
(a) LXA4
(b) LXB4
(c) 5HETE
(d) PGH2
84. Which of the following factors is morphogenic as well as
mitogenic?
(a) Fibroblast growth factor
(b) Platelet derived growth factor
(c) Bone morphogenetic protein
(d) Insulin-like growth factor
85.Procalcitonin is used as marker of:
(a) Cardiac dysfunction in acute coronary syndrome
(b) Menstrual periodicity
(c) Pituitary function
(d) Sepsis
86. Complement complex that attacks cell membrane is:
(a) C12345
(b) C23456
(AIIMS May 2016)
(c) C34567
(d) C56789
87. Systemic inflammatory response syndrome, false is:
(a) Leucopenia
(b) Fever
(c) Leukocytosis
(d) Hypoglycemia
88.What is true about interferon?
(a) Are specific for individual viruses
(b) Are protective against only viruses
(c) Reduce protein synthesis in the target cell
(d) Are divided into 5 subtypes
89. Which of the following helps in movement and adhesion?
(a) MCP1
(b) PGE2
(c) LTB4
(d) CD31
90. C3 complement is cleared by:
(a) CD 59
(b) CD 55
(c) Factor D
(d) Factor E
91. Which of the following is true for macrophage chemotactic
factor?
(a) Heat labile
(b) High molecular weight
(c) Chymotrypsin sensitive
(d) Are antigenically similar to C3
92. Phagocytosis was discovered by:
(a) Elie Metchinkoff
(b) Aulus Cornelius Celsus
(c) Rudolf Virchow
(d) Emil Adolf von Behring
93. To which of the following family of chemical mediators of
inflammation do the lipoxins belong?
(a) Kinin system
(b) Cytokines
(c) Chemokines
(d) Arachidonic acid metabolites
94. Anti-inflammatory cytokines are all except:
(a) Interleukin 10
(b) Interleukin 4
(c) Interleukin 6
(d) TNF-alpha
95. Which of the following is derived from fibroblast cell?
(a) TGF-beta
(b) Collagen
(c) MMP2
(d) Angiopoietin
96. Slow mediators of inflammation are:
(a) Leukotrienes
(b) Prostaglandins
(c) Interleukins
(d) Vasoactive amines
97. Complement proteins constitute what percentage of serum
proteins?
(a) <1
(b) 1-5
(c) 5-10%
(d) >10%
98. Which of the following is not a cachexia gene?
(a) APEH
(b) MC4R
(c) Smad7
(d) Smad3
99. Negative acute phase reactant is:
(a) C-reactive protein
(b) Alpha-1 antitrypsin
(c) Transferrin
(d) Serum amyloid protein
100. Complement mediated lysis is mediated by which of the
following antibodies?
(a) IgE
(b) IgG
(c) IgM
(d) IgD
101. Deficiency of complement proteins C5 to C8 leads to
increased infection by which of the following?
(a) Streptococcus
(b) Neisseria
(c) Pseudomonas
(d) Staphylococcus
102. Which of the following increased in acute phase response?
(a) Alpha2 microglobulin
(b) Transferrin
(c) Albumin
(d) Retinal binding protein

CHRONIC INFLAMMATION; GRANULOMATOUS INFLAMMATION

103. The epithelioid cell and multinucleated giant cells of


Granulomatous inflammation are derived from:
(a) Basophils
(AI 2002)
(b) Eosinophils
(c) CD4-T lymphocytes
(d) Monocytes-Macrophages
104. Granuloma is pathological feature of all, except:
(a) Giant cell arteritis
(AIIMS Nov 2001)
(b) Microscopic polyangiitis
(c) Wegener’s granulomatosis
(d) Churg Strauss disease
105. Granulomatous inflammatory reaction is caused by all,
except:
(AIIMS Nov 2001)
(a) M. tuberculosis
(b) M. leprae
(c) Yersinia pestis
(d) Mycoplasma
106. Non-caseating granulomas are seen in all of the following
except:
(AIIMS May 2001)
(a) Byssinosis
(b) Hodgkin’s lymphoma
(c) Metastatic carcinoma of lung
(d) Tuberculosis
107. Epithelioid granuloma is caused by:
(PGI Dec 2002)
(a) Neutrophil
(b) Cytotoxic T-cells
(c) Helper T-cells
(d) NK cells
108. Caseous necrosis in granuloma are not found in:
(PGI June 2006)
(a) Tuberculosis
(b) Leprosy
(c) Histoplasmosis
(d) CMV
(e) Wegener’s granulomatosis

MOST RECENT QUESTIONS


109. The most important function of epithelioid cells in
tuberculosis is:
(a) Phagocytosis
(b) Secretory
(c) Antigenic
(d) Healing
110. Necrotizing epithelioid cell granulomas are seen in all,
except:
(a) Tuberculosis
(b) Wegener’s granulomatosis
(c) Cat Scratch disease
(d) Leprosy
111. Epithelioid granulomatous lesions are found in all of the
following diseases, except:
(a) Tuberculosis
(b) Sarcoidosis
(c) Berylliosis
(d) Pneumocystis carinii
112. Caseous granuloma is seen in:
(a) Histoplasmosis
(b) Silicosis
(c) Sarcoidosis
(d) Foreign body
113. Non-caseating granuloma is characteristically seen in:
(a) Syphilis
(b) Sarcoidosis
(c) Tuberculosis
(d) Histoplasmosis
114. All are granulomatous diseases except:
(a) Syphilis
(b) Sarcoidosis
(c) Schistosomiasis
(d) P. carinii
115. Which of the following is the most characteristic of
granuloma:
(a) Epithelioid cell
(b) Giant cell
(c) Fibroblasts
(d) Endothelial cell
116. Caseating granuloma are seen in:
(Bihar 2003)
(a) Histoplasmosis
(b) Sarcoidosis
(c) Coccidiodomycosis
(d) All
117. In a lymph node showing non necrotizing and non–
caseating granuloma which of the following is suspected?
(a) Toxoplasmosis
(b) Lymphogranuloma venereum
(c) Cat scratch disease
(d) Kikuchis lymphadenitis
118. Which of these is not a granulomatous disease?
(a) Leprosy
(b) Tuberculosis
(c) Sarcoidosis
(d) Amebiasis
119. In a granuloma, epithelioid cells and giant cells and derived
from:
(a) T – lymphocytes
(b) Monocyte – macrophages
(c) B – lymphocytes
(d) Mast cells

WOUND HEALING; STEM CELL BIOLOGY

120. Which one of the following statements is not correct


regarding ‘Stem cell’?
(DPG 2011)
(a) Developmental elasticity
(b) Transdifferentiation
(c) Can be harvested from embryo
(d) “Knockout mice” made possible because of it
121. An adult old man gets burn injury to his hands. Over few
weeks, the burned skin heals without the need for skin
grafting. The most critical factor responsible for the rapid
healing in this case is:
(AIIMS May 2003)
(a) Remnant skin appendages
(b) Underlying connective tissues
(c) Minimal edema and erythema
(d) Granulation tissue
122. Absolute lymphocytosis is seen in:
(a) SLE
(b) TB
(c) CLL
(d) Brucellosis
123. Which of the following is absolutely essential for wound
healing?
(Karnataka 2009)
(a) Vitamin D
(b) Carbohydrates
(c) Vitamin C
(d) Balanced diet
124. Chronic granulomatous disease is:
(Karnataka 2009)
(a) Associated with formation of multiple granulomas
(b) A benign neoplastic process
(c) A parasitic disease
(d) Acquired leukocyte function defect
125. In regeneration:
(UP 2002)
(a) Granulation tissue
(b) Repairing by same type of tissue
(c) Repairing by different type of tissue
(d) Cellular proliferation is largely regulated by biochemical
factors
126. Wound contraction is mediated by:
(Jharkhand 2005)
(a) Epithelial cells
(b) Myofibroblasts
(c) Collagen
(d) Elastin

MOST RECENT QUESTIONS

127. When a cell transforms itself into different lineage the ability
us know as?
(a) De-differentiation
(b) Re-differentiation
(c) Trans-differentiation
(d) Sub-differentiation
128. Prion disease is caused by:
(a) Misfolding of protein
(b) Denaturation of proteins
(c) Reduced formation of proteins
(d) Exces formation of proteins
129. Which of the following adhesion molecules is involved in
morphogenesis?
(a) Osteopontin
(b) Osteonectin SPARC
(c) Tenascin
(d) Thrombospondins
130. Maximum collagen in wound healing is seen at which stage
of healing?
(a) End of first week
(b) End of second week
(c) End of third week
(d) End of 2 months
131. First sign of wound injury is:
(a) Epithelialization
(b) Dilatation of capillaries
(c) Leukocytic infiltration
(d) Localized edema
132. “Oval cells” are seen in the stem cells of which of the
following tissues?
(a)Skin
(b)Cornea
(c) Liver
(d) Bone
133. Which of the following is the source of hepatic stem cells?
(a) Limbus cells
(b) Ito cell
(c) Oval cell
(d) Paneth cell
134. Tensile strength of wound after laparoscopic
cholecystectomy in a 30 year old woman depends upon:
(a) Replacement of type 3 collagen
(b) Extensive cross-linking of tropocollagen
(c) Macrophage activity
(d) Granulation tissue

135. One of the following statements about hematopoietic stem


cell is false?
(a) Stem cells have self renewal property
(b) Subset of stem cells normally circulate in peripheral blood
(c) Marrow derived stem cells can seed other tissues and
develop into non hematopoietic cells as well
(d) Stem cells resemble lymphoblasts morphologically
136. Vitamin used for post translational modification of glutamic
acid to gamma carboxy glutamate is:
(a) A
(b) D
(c) E
(d) K

137. After an incised wound, new collagen fibrils are seen along
with a thick layer of growing epithelium. The approximate
age of the wound is:
(a) 4–5 days
(b) About 1 week
(c) 12–24 hours
(d) 24 –72 hours
138. True about hypertrophic scar?
(a) No genetic predisposition
(b) More common in blood group A
(c) No HLA association
(d) Predominantly collagen type 4

139. Which of the following is a totipotent cell?


(a) Hematopoietic stem cell
(b) Embryonic stem cell
(c) Tissue stem cell
(d) Adult stem cell
1. Ans. (c) Delayed prolonged increase in permeability
(Ref: Robbins 8th/45, 9/e p74)
2. Ans. (d) Pseudopod extension
(Ref: Robbins 9/e p78)
Typically the phagocytosis of microbes and dead cells is initiated by
recognition of the particles by leukocyte receptors like mannose
receptors and scavenger receptors. During engulfment,
extensions of the cytoplasm (pseudopods) flow around the
particle to be engulfed, eventually resulting in complete
enclosure of the particle within a phagosome created by the
plasma membrane of the cell
• Option ‘a’ Both Fc fragment of IgG and C3b are required in
opsonisation. It takes place before phagocytosis.
• Option ‘c’ respiratory burst occurs after the formation of the
phagolysosome.
3. Ans. (a) Superoxide dismutase
(Ref: Robbins 9/e p48)
Superoxide dismutase (SOD) is an anti oxidant enzyme
Some clarification regarding option ‘d’…. ‘Nitric oxide (NO), an
important chemical mediator generated by endothelial cells,
macrophages, neurons, and other cell types can act as a free
radical and can also be converted to highly reactive peroxynitrite
anion (ONOO-) as well as NO2 and NO–3.
4. Ans. (c) Vasodilation and increase in permeability
(Ref: Robbins 8th/46-47 9/e p74)
Direct quote “a hallmark of acute inflammation is increased vascular
permeability leading to the escape of protein-rich exudate into the
extravascular tissue, causing edema”.
5. Ans. (d) Unidirectional locomotion of the neutrophils
(Ref: Robbins 8th/50 9/e p77)
6. Ans. (a) Vasodilation and increased vascular permeability
(Ref: Robbins 8th/45 9/e p74)
7. Ans. (a) NADPH oxidase
(Ref: Robbins 9/e p79)
Within the phagocytes, the following reaction takes place:
The initiating enzyme for this process is NADPH oxidase (also called
respiratory burst oxidase).
Glutathione peroxidase, glutathione reductase and superoxide
dismutase are examples of anti-oxidants. They reduce free
radical formation.
8. Ans. (a) Metalloproteinases
(Ref: Robbins, 9/e p105)
• Extracellular Matrix (ECM) comprises of interstitial matrix
and basement membrane. The degradation of collagen and
other ECM proteins is achieved by a family of matrix
metalloproteinases (MMPs) which are dependent on zinc
ions for their activity.
• MMP8 and MMP2 are collagenases which cleave type IV
collagen of basement membranes.
• MMPs also have a role in tumour cell invasion.
9. Ans. (d) Direct injury to endothelial cells... For details see text
(Ref: Robbins 9/e p74)
10. Ans. (b) Neutrophilia
(Ref: Robbins 7th/56)
11. Ans. (d) Decreased hydrostatic pressure
(Ref: Robbins 7th/50-51 9/e p73-74)
With acute inflammation, hydrostatic pressure is increased (due to
increased blood flow from vasodilation) and at the same time
osmotic pressure is reduced because of protein leakage (due
to increased permeability)
12. Ans. (d) Granuloma formation
(Ref: Robbins 9/e p97)
• Granuloma formation is characteristic of chronic
granulomatous inflammation and is not seen in acute
inflammation.
13. Ans. (a) NADPH oxidase
(Ref: Robbins 9/e p48)
14. Ans. (d) In Bruton’s disease there is normal opsonization
(Ref: Robbins 8th/231-232,55 9/e p240-241)
15. Ans. (a) Phagocytes
(Ref: Harrison 17th/384)
The nitroblue-tetrazolium (NBT) test is the original and most widely
known test for chronic granulomatous disease. It is negative
in chronic granulomatous disease and positive in normal
individuals. It is used for detecting the production of reactive
oxygen species in the phagocytes. The basis of the test has been
discussed in the text.
16. Ans. (b) Early transient increase
(Ref: Robbin’s 9/e p74)
Name of Mechanism Involved Mechanism
Early transient increase Endothelial cell contraction
Delayed transient increase in Direct endothelial injury
permeability
Delayed permanent increase Endothelial cell retraction, endothelial
cell damage

17. Ans. (b) Immigration of the leukocytes through the vessel


wall to the site of inflammation
(Ref: Robbins 9/e p76)
18. Ans. (a) Selectins; (b) Integrins
(Ref: Robbins 9/e p76)
• Endothelium and WBC interact through the molecules like
Immunoglobulins (family molecules e.g. ICAM-I, VCAM-I),
Integrins, Mucin-like glycoprotein and selectins.
Molecule Function Deficiency
disease
Integrin Firm adhesion LAD I
Selectin Rolling and loose adhesion LAD II
• Defensins are cationic arginine rich peptides having broad
antimicrobial activity found in Azurophil granules of
neutrophils.
• Endothelin is a potent endothelial derived vasoconstrictor
19. Ans. (c) Inability to produce hydroxyl-halide radicals
(Ref: Robbins 8th/53, 56, 9/e p79)
20. Ans. (b) Chemotaxis
(Ref: Robbins 9/e p77)
21. Ans. (d) Phagocytosis
(Ref: Robbins 9/e p75)
22. Ans. (b) Less protein
(Ref: Robbins 9/e p73)
23. Ans. (d) Absence of functional loss
(Ref: Robbins 8th/44; 7th/79, 9/e p71)
24. Ans. (a) C5a
(Ref: Robbins 8th/66; 7th/56, 9/e p77)
25. Ans. (a) PECAM
(Ref: Robbins 8/e p50, 9/e p77)
26. Ans. (c) A selectin
(Ref: Robbins 8/e p49, 9/e p76)
Selectins are a family of proteins that are involved in the cellular
process of rolling interactions. They are of three types: E selectin,
L selectin and P selectin. For details see text.
27. Ans. (d) Granuloma formation
(Ref: Robbins 9/e p97)
28. Ans. (a) PECAM/CD31 is responsible for neutrophil activation
(Robbins 9/e p76-7)
29. Ans. (b) Phagocytosis
30. Ans (b) Margination, rolling, adhesions, transmigration
(Ref: Robbins 9/e p75)
31. Ans (c) Citrulline
(Robbins 9th/82, NETosis 2: The Excitement Continues pg
205)
Conversion of arginine residues to citrulline in the histones is
an essential step in the formation of neutrophil extracellular traps.
32. Ans. (d) Increased vascular permeability > (c) Pain
(Ref: Robbins 8th/65-6, 9/e p89)
Functions of Bradykinin:
1. Increases vascular permeability
2. Contraction of smooth muscles
3. Dilation of the blood vessels
4. Pain when injected into the skin
• Out of these actions of bradykinin, the increase in vessel
permeability is a better answer as it is the hallmark of acute
inflammation.
33. Ans. (d) IL - 18
(Ref: Harrison 18th/144 , 9/e p99)
Pyrogens are substances which cause fever. These can be either
exogenous or endogenous. The endogenous pyrogens (also
called the pyrogenic cytokines) include:
• IL-1Q
• IL-6 Q
• Tumor necrosis factor Q (TNF)
• Ciliary neurotopic factor Q (CNTF)
• IFN-αQ (alpha)

Harrison clearly mentions that the IL-18Q which is a member of IL-1 family
does not appear to be a pyrogenic cytokine.

Important points about regulation of body temperature


PGE2Q is the final mediator responsible for causing elevation of the thermoregulatory
set point by increasing the concentration of cAMP.
Exogenous pyrogens include endotoxinQ produced by gram negative bacteria.
Body temperature is regulated at the level of the hypothalamus. Most individuals with
hypothalamic damage have subnormal and not supranormal body temperature.
34. Ans. (c) It manifests as pitting edema
(Ref: Harrison 17th/2066, 18th/2711-3)
35. Ans. (c) C3
(Ref: Robbins 8th/63-64 , 9/e p88)
The complement proteins can be activated by 3 pathways; classical,
lectin and alternate pathways. Terminal pathway is common to
the first three pathways and is present at the level of post
activation stage of C3. It eventually leads to the membrane attack
complex that lyses cells.
36. Ans. (b) Myeloperoxidase
(Ref: Robbins 9/e p83)
Myeloperoxidase (MPO) is an enzyme present in primary (or
azurophilic) granules of the neutrophils. In the presence of a
halide such as Cl-, MPO converts H2O2 to HOCl• (hypochlorous
radical) during the process of respiratory burst.
37. Ans. (a) Overproduction of 1, 25 dihydroxy vitamin D (Ref:
Robbins 8th/433-6, Heptinstall’s pathology of the kidney,
LWW, Volume 1; 6th/1051, Interstitial Lung Disease by
Schwarz 5th/458-9)
Nephrocalcinosis is defined as calcification of the renal interstitium and
tubules. It is associated with hypercalcemia. In chronic
granulomatous inflammation, the important cells involved are
macrophages and lymphocytes.
Direct quote Heptinstall’s … ‘Sarcoidosis and other granulomatous
diseases can be cause of hypercalcemia and hypercalciuria
owing to exces vitamin D from extra renal conversion of 1,25
(OH)2D3. Nephrocalcinosis was found to be associated with 22%
patients with chronic sarcoidosis’.
In other granulomatous conditions (like Sarcoidosis), there is
presence of metastatic calcification due to activation of
vitamin D precursor by macrophages…….Robbins
38. Ans. (a, b, d)
(Ref: Robbins 8th/65, 7th/45, 9/e p89)
Friends, in our opinion the question should have been asked with
an “except” because bradykinin has the following effects:
• Increases vascular permeability
• Arteriolar dilation
• Bronchial smooth muscle contraction
• Pain at the site of injections/inflammation

Since increased vascular permeability is the most characteristic feature of


acute inflammation, some people were of the opinion that this could be
single best option to be marked presuming the stem of question was correct.

39. Ans. (d) Reactive O2 species


(Ref: Robbins 9/e p79)
H2O2- MPO- halide system is the most efficient bactericidal system of
neutrophils.
40. Ans. (b) Pain at the site of inflammation
(Ref: Robbin 7th/e 45, 9/e p89)
41. Ans. (a) Histamine
(Ref: Goodman & Gilman 10th/650)
When histamine is injected intradermally it causes the triple response
consisting of:

• Red spot: Due to capillary dilatation


• Wheal: Due to exudation of fluid from capillaries and venules
• Flare: Redness in the surrounding area due to arteriolar dilation mediated
by axon reflex.

42. Ans. (c) C5


(Ref: Harrison 17th/2030-2032)
• The terminal pathway that is common to all pathways of the
complement system leads to the membrane attack complex
and consists of factors C5, C6, C7, C8 and C9 (C5b6789).
43. Ans. (d) Arachidonic acid metabolites
(Ref: Robbins 7th/69; Katzung 11th/314-323, 9/e p85)
Lipoxins are a recent addition to the family of bioactive products
generated from arachidonic acid. They have anti-inflammatory
activity (explained in text).
44. Ans. (a) C3
(Ref: Harrison 17th/2030, 9/e p88)
45. Ans. (b) Eotaxin
(Ref: Robbins 8th/62-63; 7th/71, 9/e p87)
46. Ans. (a) Angiotensin
(Ref: Robbin’s 9/e p83)
Kallikreins like bradykinin, PGs and complement components are
mediators of acute inflammation.
47. Ans. (c) Myeloperoxidase
(Ref: Robbins 9/e p83)
48. Ans. (c) IL-1 > (d) IL-6
(Ref: Javetz, 22nd/1291)
• Macrophages release IL – 1 which stimulates the T – helper
cells.
• The T – cells in response proliferate and release IL – 2 which
in turn further stimulates T – cell proliferation and B cell
proliferation and differentiation into plasma cells.
• Please note that even IL-6 (produced by macrophages) acts
on late stages of B cell differentiation enhancing antibody
formation. Still, IL-1 being the most important cytokine
having systemic effects of inflammation has been chosen as
the answer here in preference to IL-6.
49. Ans. (a) Neutrophils; (c) Platelets
(Ref: Harrison’ 18th/2223-5, Robbins 7th/202)
Cytokines are peptide mediators or intercellular messengers which
regulate immunological, inflammatory and reparative host
responses. They are produced by widely distributed cells like
macrophage, monocytes, lymphocytes, platelets, fibroblast,
endothelium, stromal cells etc.
50. Ans. (a) Bacterial toxin; (b) IL-1; (d) Interferon; (e) TNF
(Ref: Harrison 18th/143-5)
Fever is produced in response to substances called pyrogens that act
by stimulating prostaglandin synthesis in the vascular and
perivascular cells of hypothalamus. They can be classified as:

• Exogenous pyrogens - endotoxin of gram ‘-’ bacteria, superantigens


(gram ‘+’ bacteria)
• Endogenous pyrogens - IL-1, TNF-a, IL-6, Ciliary neurotropic factor and
IFN-a.

51. Ans. (a) Includes interleukins; (c) Are polypeptide (complex


protein)
(Ref: Ananthanarayan 7th/143; Harrison 16th/1915, Robbins
7th/202)
Cytokines are peptide mediators or intracellular messengers
produced by wide variety of haemopoietic and non- haemopoietic
type of cells in response to immuno, inflammatory or infectious
disease states. Most of the lymphokines exhibit multiple
biological effects and same effect may be caused by different
lymphokines.

Classification of cytokines
1. Cytokines that mediate innate immunity - IL-1, TNF, IFN, IL-6. IL-12

• Are involved in both innate and adaptive immunity.

2. Cytokines regulating lymphocyte growth, activation and differentiation


(IL-2, IL-4, IL-12, IL-15, TGF-b)

• IL-2 is a growth factor for T-cells.

• IL-4 stimulates differentiation to TH2 pathway.

• IL-12 stimulates differentiation to T H1 pathway.

• IL-15 stimulates the growth and activity of NK cells.

• IL-10 and TGF-b down regulate immune responses.

3. Cytokines that activate inflammatory cells:

• IL-5 activates eosinophils.

• TNF induces acute inflammation by acting on neutrophils and


endothelial cells.

4. Cytokines that affect movement of WBCs (chemotaxis).

5. Cytokines that stimulate hematopoiesis e.g. GM-CSF, G-CSF, stem


cell factor.

• Same cytokine can be produced by different cells. This


called Redundancy
• Same cytokine can have multiple actions (pleiotropic in
nature).

52. Ans. (d) X and Ca++


(Ref: Robbins 9/e p118)
53. Ans. (d) C3a and C5a

(Ref: Robbins 9/e p89)


54. Ans. (d) Factor VIII
(Ref: Robbins 7th/664)
55. Ans. (b) C5a
(Ref: Robbins 7th/56, 9/e p77)
56. Ans. (c) Vasodilation
(Ref: Robbins 7th/64, 9/e p73)
57. Ans. (d) Lactoferrin
(Ref: Robbins 8th/54; 7th/61, 9/e p80)
58. Ans. None
(Ref: Robbins 8th/53-54; 7th/69, 9/e p80)
Cathepsin G is a serine protease secreted by activated neutrophils that
play a role in the inflammatory response.
59. Ans. (a) Weibel-Palade bodies
(Ref: Robbins 9/e p76)
60. Ans. (a) Intrinsic and common pathway
(Ref: Robbins 8th/120,666; 7th/469, 9/e p118)
61. Ans. (d) Function of platelets
(Ref: Robbins 8th/666)
62. Ans. (d) IX
(Ref: Robbins 8th/119, 7th/128, 9/e p119)
63. Ans. (d) Leukotriene B4
(Ref: Robbins 9/e p203)
64. Ans. (a) Langerhans cells
(Ref: Robbins 9/e p622)
Birbeck granules are rod shaped/Tennis-racket shaped cytoplasmic
organelles with a central linear density and a striated appearance. They are
diagnostic microscopic feature in Langerhans cell histiocytosis
(Histiocytosis X)

65. Ans. (b) Hydrolytic enzyme


(Ref: Robbins 8th/54)
66. Ans. (b) Thromboxane A2
(Ref: Robbins 9/e p84)
67. Ans. (b) C5a
(Ref: Robbins 8th/64, 7th/56, 9/e p77)
68. Ans. (a) Vasodilatation
(Ref: Robbins 9/e p85)
69. Ans. (c) Carbohydrate-binding proteins
(Ref: Robbins 8th/51-53; 7th/59, 9/e p78)
70. Ans. (a) IL-1
(Ref: Robbins 8th/57, 61, 7th/71, 9/e p86)
71. Ans. (a) Monoclonal antibody
(Ref: Robbins 9/e p86)
72. Ans. (b) Integrins
(Ref: Robbins 8/e p96, 9/e p24)
The cell adhesion molecules (CAMs) are classified into four main
families:
Immunoglobulin family CAMs
• Cadherins
Integrins: Fibronectin, laminin, and osteopontin provid a connect
between cells and extracellular matrix (ECM) proteins).
• Selectins
• Cadherins and integrins link the cell surface with the
cytoskeleton through binding to actin and intermediate
filaments.
– Laminin is the most abundant glycoprotein in the
basement membrane and has binding domains for both
ECM and cell surface receptors.
73. Ans. (b) Interleukin-10
(Ref: Robbins 8/e p56; Cytokines and Pain/pg 3, 9/e p86)
The following are the anti inflammatory cytokines: IL-10, TGF-β, IL-4, IL-13).

74. Ans. (a) IL-1


(Ref: Robbins 8/e p61,66 , 9/e p83, 90)
75. Ans. (a) Gastric cancer
(Ref: Robbins 8/e p96, 9/e p297)
Cadherin is derived from the “calcium-dependent adherence protein.” It
participates in interactions between cells of the same type. The
linkage of cadherins with the cytoskeleton occurs through the
catenins. The cell -to-cell interactions mediated by cadherin and
catenins play a major role in regulating cell motility, proliferation,
and differentiation and account for the inhibition of cell
proliferation that occurs when cultured normal cells contact each
other (“contact inhibition”).
• Reduced function of E-cadherin is associated with certain types of
breast and gastric cancer.

76. Ans. (a) Mast cells

(Ref: Robbins 8/e p57, 9/e p83)


77. Ans. (b) Vasoconstriction
(Ref: Robbins 8/e p115 ... Not given in 8/9 edition of Robbins)
78. Ans. (c) Decreased vascular permeability
(Ref: Robbins 8/e p60, 9/e p73)
PAF is another phospholipid-derived mediator having the following
inflammatory effects:
• Platelet aggregation
• Vasoconstriction
• Bronchoconstriction
• At extremely low concentration, it may cause
vasodilation and increased venular permeability
79. Ans. (a) C3
(Ref: Robbins 9/e p88)
80. Ans. (d) Factor VIII
(Ref: Robbins 7/e p664)
Cryoprecipitate is a rich source of Factor VIII. Have a glance at the
flowchart given below.
81. Ans. (c) Arachidonic acid
(Ref: Robbins 9/e p84)
Arachidonic acid is an essential fatty acid which is acted on by the
enzyme cyclo-oxygenaseQ
(COX) leading to the formation of the prostaglandins.
Also know:
• Linoleic, linolenic and arachidonic acid are examples of
polyunsaturated essential fatty acidsQ (PUFA) which means they
cannot be synthesized in the human body.
• Docosahexaenoic acidQ is an essential fatty acid present in breast
milk which is required for myelination of nervesQ.
• Richest source of PUFA is safflower oilQ
• Coconut oilQ is the poorest source of PUFA.
82. Ans. (b) IL-5
(Ref: Robbins 8/e p200)
IL-5 is required for the development and maturation of the eosinophil.
83. Ans. (d) PGH2
(Ref: Robbins 9/e p84)
84. Ans. (a) Fibroblast growth factor
(Robbins 9th/ 19-20; 8th/87-88, Fetal and Neonatal Physiology
4th/867-8)
Fibroblast growth factor (FGF) contributes to wound healing
responses, hematopoiesis, and development. They can be
belonging to
• Acidic FGF (aFGF, or FGF-1)
• Basic FGF (bFGF, or FGF-2): necessary for angiogenesis
• FGF-7 is also referred to as keratinocyte growth factor (KGF)
Growth Factor Source Functions
Epidermal growth Activated marcophages, Mitogenic for keratinocytes and
factor (EGF) selivary glands, fibroblasts; stimulates
keratinocytes, and many karatinocytes migration;
other cells stimulates formation of
granulation tissue
Transforming growth Activated marcophages, Stimulates proliferation of
factor-α (TGF-α) keratinocytes, many other hepatocytes and many other
cells types epithelial cells
Hepatocyte growth Fibroblasts, stromal cells Enhances proliferation of
factor (HGF) scater in the liver, endothelial hepatocytes and other
factor cells epithelial cells; increases cell
motility.
Vascular endothelial Messenchymal cells Stimulates proliferation of
growth factor (VEGF) endothelial cells; increases
vascular parmeability
PLatelet-derived Platelets, macrophages, Chemotactic for neutrophils,
growth factor (PDGF) endothelial cells, smooth macrophages, fibroblasts, and
muscle cells, smooth muscle cells, activates
keratinocytes and stimulates proleferation of
fibroblasts, endothelial, and
other cells, stimulates ECM
protein synthesis
Fibroblast growth Macrophages, mast cells, Chemotactic and mitogenic for
factor (FGFs), endothelial cells, many fibroblasts; stimulates
including acidic (FGF- other cell types anglogenesis and ECM protein
1) and basic (FGF-2) synthesis.
Transforming growth Platelets, T Chemotactic for leukocytes and
factor b-(TGF-b) lymphovcytes, fibroblasts; stimulates ECM
macrophages, endothelial protein synthesis; suppresses
cells, keratinocytes, acute inflammation
smooth muscle cells,
fibroblasts
Growth Factor Source Functions
Keratinocyte growth Fibroblast Stimulates keratinocyte
factor (KGF) (i.e., migration, proliferation, and
FGF-7) differentiation.
ECM, EXtracellular
membrane

85. Ans. (d) Sepsis


(Ref: Harrison 18th/3419)
Procalcitonin is an acute phase reactant which is now useful for being a
marker of sepsis. It is in fact utilized for differentiating the bacterial and
aseptic meningitis.

86. Ans. (d) C56789


(Ref: Robbins 9th/88)
The complement complex attacking the cell membrane is membrane
attack complex having a composition of C56789. Action of this
complex leads to entry of fluid and ions inside the cells leading to
cell lysis.
87. Ans (d) Hypoglycemia
(Ref: CMDT 2018/ 502, Robbins 9/e p131)
Systemic inflammatory response syndrome is diagnosed when 2
or more of the following criteria are met:
• Core temperature <36° or >38°C
• Heart rate >90 beats/min
• Respirations >20/min or PCO2 <32 mmHg
• White blood cell count >12,000/μL, <4000/μL, or 10% bands

88. Ans (c) Reduce protein synthesis in the target cell


(Ref: Robbins 9/e p188)
Interferons act by reducing the protein synthesis in the target cell.
89. Ans (d) CD31
(Ref: Robbins 9/e p77)
CD31 is responsible for transmigration.
90. Ans (b) CD 55
(Ref: Robbins 9/e p642)
• CD55 is decay accelerating factor. It is responsible for breakdown of
C3.
• CD59 is membrane inhibitor of reactive lysis. It is a potent inhibitor of
C3 convertase that prevents the spontaneous activation of the
alternative complement pathway.

91. Ans (c) Chymotrypsin sensitive


(Ref: Immunology 278)
Macrophage chemotactic factor is a heat stable low molecular
weight protein. It is chymotrypsin sensitive and is resistant to
treatment by RNase and neuraminidase. It is different from C3
and C5.
92. Ans (a) Elie Metchinkoff
(Ref: Robbins 9/e p71)
Elie Metchnikoff discovered the process of phagocytosis by
observing the ingestion of rose thorns by amebocytes of starfish
larvae and of bacteria by mammalian leukocytes.
93. Ans (d) Arachidonic acid metabolites
(Ref: Robbins 9/e p85)
94. Ans (d) TNF-alpha
(Harrison 19/e p1753)
• Adiponectin, IL-10, IL-6, IL-4 and TGF beta are anti- inflammatory
cytokines…Harrison/19th.

95. Ans (b) Collagen


(Ref: Robbins 9/e p19,21)
• Interstitial matrix is produced by mesenchymal cells like
fibroblasts and is mainly composed of fibrillar and non-
fibrillar collagen.
• TGF-beta is chemotactic for leukocytes and fibroblasts. It
induces proliferation of fibroblasts.
96. Ans (a) Leukotrienes
(Ref: Robbins 9/e p85)
Slow-reacting substance of anaphylaxis consists of leukotrienes C4,
D4, and E4.
97. Ans (c) 5-10%
(Ref: Hepatic plasma proteins 78)
Complement proteins are accounting for 5-10% of the total plasma
proteins in the human body.
98. Ans (c) Smad7
(Ref: AIIMS Faculty)
Smad 7 is associated with reduction in cachexia.
99. Ans (c) Transferrin
100. Ans (c) IgM
(Ref: Robbins 9/e p88)
101. Ans (b) Neisseria
(Ref: Robbins 9/e p89)
102. Ans (a) alpha2 microglobulin
(Ref: Robbins 9/e p702, Anderson 10/e p411)
103. Ans. (d) Monocytes-Macrophages
(Ref: Robbins 9/e p97)
Delayed type hypersensitivity (as seen in TB) results from
accumulation of mononuclear cells around small veins and
venules, producing a perivascular cuffing. Monocytes transform
into macrophages which undergo morphological changes to
produce epithelioid cells.
104. Ans. (b) Microscopic polyangiitis
(Ref: Robbins Illustrated 7th/540)
• Microscopic polyangiitis is a small vessel vasculitis
showing the presence of necrotizing inflammation of the
affected vessels without the presence of granuloma.
Systemic vasculitis causing Systemic vasculitis causing
granulomas necrotizing inflammation
• Giant cell arteritis • Polyarteritis nodosa (PAN)
• Takayasu’s disease • Microscopic polyangiitis
• Wegener’s granulomatosis • Wegener’s granulomatosis
• Churg-Strauss syndrome • Churg-Strauss syndrome

105. Ans. (d) Mycoplasma

(Ref: Robbins 8th/802 9/e p97)


• Granulomatous inflammation is a distinctive pattern of
chronic inflammatory reaction characterized by focal
accumulations of activated macrophages, which often
develop an epithelial-like (epithelioid) appearance.
• Tuberculosis is the prototype of the granulomatous
diseases, but sarcoidosis, Crohn’s disease, cat-scratch
disease, lymphogranuloma inguinale, leprosy,
brucellosis, syphilis, some mycotic infections,
berylliosis, and reactions of irritant lipids are also
included.
• In Robbins (8th ed, page 802), Yersinia has also been
mentioned to be associated with granulomatous
inflammation. So, the answer of exclusion is Mycoplasma.
106.Ans. (c) Metastatic carcinoma of the lung
(see below)
Friends, remember that fungal and mycobacterial granulomas are
usually associated with central necrosis but all large caseating
granulomas come from small non-caseating granulomas.
Granuloma can be seen in both Byssinosis and Hodgkin’s
lymphomas.
• Granulomas are also seen in Hodgkin’s disease (Dorland’s,
28/p 716)
• So, metastatic carcinoma of lung is the answer of exclusion.
107. Ans. (c) Helper T-cells
(Ref: Robbins 7th/83
CD 4 Helper T-cells are involved in granuloma formation as it secretes
IFN-γ, IL-2 and IL-12.
108. Ans. (b) Leprosy; (d) CMV; (e) Wegener’s granulomatosis
(Ref: Robbins 7th/83,8th/73, 9/e p98)
Caseous necrosis is characteristic of tubercular granuloma, rare in
others type of granulomatous disease. TB granuloma is a
prototype of immune granuloma. These are caused by insoluble
particles; typically microbes that are capable of inducing a cell
mediated immune response.
Granulomatous lesions may develop in liver in CMV infection.
109. Ans. (b) Secretory
(Ref: Harsh Mohan 9/e p97)
110. Ans. (d) Leprosy
(Ref: Robbins 8th/73, 9/e p98)
111. Ans. (d) Pneumocystis carinii
(Ref: Robbins 9/e p98)
112. Ans. (a) Histoplasmosis
(Ref: Robbins 8th/718)
113. Ans. (b) Sarcoidosis
(Ref: Robbins 8th/701, 9/e p98)
114. Ans. (d) P. carinii
(Ref: Robbins 8th/246, 9/e p98)
115. Ans. (a) Epithelioid cell
(Ref: Robbins 9/e p97-98)
116. Ans. (a) Histoplasmosis
(Ref: Robbins 8th/717-718)
117. Ans. (a) Toxoplasmosis
This is a characteristic feature of toxoplasmosis.
Also know

• Toxoplasmosis is transmitted by catQ (definitive host). Man is the


intermediate hostQ.
• It is diagnosed by Sabin Feldman dye testQ.
• ChorioretinitisQ is the commonest manifestation of this disease when
transmitted congenitally. In the acquired disease, there is usually
absence of symptoms.
• Drug of choice for toxoplasmosis is combination of pyrimethamine and
sulfadiazineQ.
• Drug of choice for toxoplasmosis in pregnancy is spiramycinQ.

118. Ans. (d) Amebiasis


(Ref: Robbins 9/e p98)
119. Ans. (b) Monocyte – macrophages
(Ref: Robbins 9/e p97)
120. Ans. (a) Developmental elasticity
(Ref: Robbins 8th/82-5; Harrison 17th/426... Not in 9/Edition of
Robbins)
Stem cells show the property of developmental plasticity (Not
developmental elasticity) which is also known as
transdifferentiation.

A change in stem cell differentiation from one cell type to another is called
transdifferentiation, and the multiplicity of stem cell differentiation options is
known as developmental plasticity. …Robbins 8th/85.
121. Ans. (a) Remnant skin appendages
(Ref: Love & Bailey 23rd/189)
• Skin consists of two layers. Epidermis which is the most
superficial layer of the skin constantly replaced from the
basal layer and the dermis which is thicker than epidermis
and contains adnexal structures. The importance of these
adnexal structures is that they contain epithelial cells can
proliferate and can heal a partial thickness wound by
epithelialization.
122. Ans. (b) TB; (c) CLL; (d) Brucellosis
(Ref: P.J. Mehta 14th – 374)
Absolute lymphocytosis Relative lymphocytosis
1. Bacterial infections like 1. All causes of neutropenia.
tuberculosis, brucellosis, 2. Infective hepatitis
syphilis, pertussis, and 3. Convalescence from acute
toxoplasmosis. injection.
2. Viral infections like mumps, 4. Infant with infections,
rubella, and infectious malnutrition and avitaminosis.
mononucleosis.
3. Leukemia (chronic lymphocytic).
4. Thyrotoxicosis

123. Ans. (c) Vitamin C


(Ref: Robbins 7th/114, 9/e p106)
Healing is modified by a number of influences (including both systemic
and local host factors) frequently impairing the quality and
adequacy of both inflammation and repair.
Ascorbic acid deficiency causes reduced cross linking of
tropocollagen to collagenQ
So, the patient has increased bleeding tendencies and poor wound healing.

124. Ans. (a) Associated with formation of multiple granulomas


(Ref: Harrison 18th/387)
It is a congenital and not acquired leucocyte function defect.
125. Ans. (b) Repairing by same type of tissue
(Ref: Robbins 8th/92-94, 7th/88, 9/e p101)
126. Ans. (b) Myofibroblasts
(Ref: Robbins 9/e p105)
127. Ans. (c) Trans-differentiation
(Ref: Robbins 9/e p26-27)
Tissue stem cells which are thought to be lineage-committed
multipotent cells, possess the capacity to differentiate into cell
types outside their lineage restrictions (called trans-
differentiation). For example, hematopoietic stem cells may be
converted into neurons as well as germ cells.
128. Ans. (a) Misfolding of protein
(Ref: Robbins, 9/e p1281)
Prions are abnormal forms of a cellular protein that cause
transmissible neurodegenerative disorders. This group of
disorders includes:

� Creutzfeldt-Jakob diseaseQ (CJD),


� Gerstmann-Sträussler-Scheinker syndromeQ (GSS),
� Fatal familial insomniaQ, and
� KuruQ in humans;
� Scrapie in sheep and goats; mink-transmissible encephalopathy; chronic
wasting disease of deer and elk; and bovine spongiform
encephalopathyQ

• The prion protein (PrP) is both infectiousQ and


transmissibleQ.
• These disorders are predominantly characterized by
“spongiform change” caused by intracellular vacuoles in
neurons and glia.
• In pathogenesis, the disease occurs when the PrP
undergoes a conformational change from its normal α-helix-
containing isoform (PrPc) to an abnormal β-pleated sheet
isoform, usually termed PrPsc (for scrapie). It is associated
with resistance to the digestive action of proteases.
129. Ans. (c) Tenascin
(Ref: Robbins 8/e p96)
Name of adhesion molecule Function
Osteonectin (SPARC) Tissue remodeling in response to injury,
Angiogenesis inhibitor
Thrombospondins Inhibit angiogenesis
Osteopontin Regulates calcification
Mediator of leukocyte migration in inflammation,
Vascular remodeling
Fibrosis in various organs
Tenascin family Morphogenesis
Cell adhesion

SPARC is secreted protein acidic and rich in cysteine.


130. Ans. (c) End of third week
(Ref: Robbins 9/e p106-108) Measurements in Wound Healing:
Science and Practice Springer 2012 pg 112-3
Measurements in Wound Healing… “maximum collagen
production occurs at 20 days”. The remodeling of the collagen
continues beyond this duration.
Also know for future NEET questions

• TGF-β is the most important fibrogenic agent


• Wound strength is 10% after 1 weekQ; it increases rapidly during next 4
weeksQand becomes 70% at the end of 3rd monthQ.
• The tensile strength of the wound keeps on increasing as time progresses.
• Collagen is the most abundant protein in the body
• Type I collagen is the major component of extracellular matrix in skin.
• Type III collagen which is also normally present in skin, becomes more
prominent and important during the repair process.

131. Ans. (b) Dilatation of capillaries….


(Ref: Robbins 8/e p46, 102, 7/e p107, 9/e p106)
Direct quote.. “the cutaneous wound healing is divided into the three
phases: inflammation proliferation and maturation.”
One of the earliest manifestations of inflammation is dilation of the
capillaries.
132. Ans. (c) Liver
(Ref: Robbins 8/e p85-6, 9/e p28)
133. Ans. (c) Oval cell….discussed in detail in a different
question
(Ref: Robbins 8/e p83, 7/e p91, 9/e p28)
134. Ans. (b) Extensive cross-linking of tropocollagen
(Ref: Robbin 8/e p 105-6)
Fibrillar collagens (mostly type I collagen) form a major portion of the
connective tissue in repair sites and are essential for the
development of strength in healing wounds.
Net collagen accumulation, however, depends not only on increased
collagen synthesis but also on decreased degradation.
The recovery of tensile strength results from the excess of collagen
synthesis over collagen degradation during the first 2 months of
healing, and, at later times, from structural modifications of
collagen fibers (cross-linking, increased fiber size) after
collagen synthesis ceases.
135. Ans. (b) Subset of stem cells normally circulate in peripheral
blood
(Ref: Robbin 9/e p 581)
Hematopoietic stem cells have two essential properties that are
required for the maintenance of hematopoiesis: pluripotency and
the capacity for self-renewal. Pluripotency refers to the ability of a
single HSC to generate all mature blood cells. When an HSC
divides, at least one daughter cell must self renew to avoid stem
cell depletion. Self-renewing divisions occur within a specialized
marrow niche, in which stromal cells and secreted factors nurture
and protect the HSCs. During stress, HSCs are mobilized from
the bone marrow and appear in the peripheral blood.

Remember HSC resemble blasts morphologically.

136.Ans. (d) K
(Ref: Robbins 9/e p442, 119)
137. Ans. (a) 4–5 days
(Ref: Robbins 9th/106)

As directly mentioned in the book……….. “By day 3, neutrophils have been


largely replaced by macrophages, and granulation tissue progressively
invades the incision space. Collagen fibers are now evident at the incision
margins. Epithelial cell proliferation continues, forming a covering approaching
the normal thickness of the epidermis.

138. Ans. (b) More common in blood group A


(Ref: Robbins 9/e p702, Plastic Surgery Secrets Plus 120)
139. Ans. (b) Embryonic stem cell (Ref: Robbins 9/e p27)

ANNEXURE
Table 1: Endothelial Leukocyte adhesion molecules and their functions
Endothelial molecule WBC receptor Major role
P-selectin Sialyl- Lewis X Rolling
E-selectin Sialyl- Lewis X Rolling, adhesion to activated
endothelium
ICAM-1 CD 11/CD 18 (Integrins) Adhesion, arrest,
transmigration
VCAM-1 VLA 4, LPAM-1 Adhesion
Glycam-1 L-selectin Lymphocytes homing to high
endothelial venules
CD 31(PECAM) CD 31 WBC migration through
endothelium.

Table 2: Role of different mediators in inflammation


Increased vascular permeability Chemotaxis, leukocyte
recruitment and activation
(a) C3a and C5a (a) C5a
(b) Vasoactive amines (b) Leukotriene B4
(c) Leukotriene C4, D4, E4 (c) Chemokines
(d) PAF (d) IL-8
(e) Substance P (e) Bacterial products
(f) Bradykinin (f) TNF
Fever Vasodilatation
(a) IL-1 (a) NO
(b) TNF (b) Histamine
(c) Prostaglandins (PGE2) (c) Prostaglandins
Tissue damage Pain
(a) Neutrophil/macrophage lysosomal enzymes (a) Prostaglandin
(b) NO and reactive oxygen species (b) Bradykinin
1. A 33-year-old man presents with a 5-week history of calf pain
and swelling and low-grade fever. Serum levels of creatine
kinase are elevated. A muscle biopsy reveals numerous
eosinophils also she had peripheral blood eosinophilia.
Which of the following interleukins is primarily responsible
for the increase in eosinophils in this patient?
(NEET 2020 like pattern)
(a) IL2
(b) IL4
(c) IL1
(d) IL6
Ans. (b) IL4
(Ref: Robbins 9th/96)
• Direct line from Robbins…. “TH2 cells secrete IL-4, IL-5, and IL-
13, which recruit and activate eosinophils and are responsible for
the alternative pathway of macrophage activation”.
• Individually, single best cytokine for increasing eosinophils is IL5
>>> IL4.
2. An 11-year-old boy came with history of cough for15 days. On
examination he was found to have cervical
lymphadenopathy. The lymph node biopsy is shown below.
Which of the following is the most appropriate diagnosis?
(NEET 2020 like pattern)

(a) Tuberculosis
(b) Leprosy
(c) Sarcoidosis
(d) Syphilis
Ans. (a) Tuberculosis
(Ref: Robbins 9th/ 98)
Presence of history of chronic cough with cervical lymphadenopathy in
a patient is likely to be of infectious etiology. Nonetheless, biopsy
is helpful for confirmation of diagnosis. The presence of
granulomas with the classical Langerhans cell (multinucleated
giant cells with nuclei in periphery) is a feature of tubercular
infection.
• Sarcoidosis has noncaseating granulomas with abundant activ
macrophages (so called naked granuloma)
• Syphilis has Gumma (microscopic to grossly visible lesion, enclosing wa
histiocytes; plasma cell infiltrate; central cells are necrotic without los
cellular outline).
• Leprosy has acid-fast bacilli in macrophages with noncaseating granulomas

3. Which of the following causes vasodilation?


(AIIMS Nov 2019 like pattern)
(a) Thromboxane A2
(b) Prostaglandin E2
(c) Histamine
(d) Serotonin
Ans. (c) Histamine
(Ref: Robbins 9th/73)
Direct quote….“Vasodilation is induced by the action of several
mediators, notably histamine, on vascular smooth muscle.”
4. Match the following mechanisms regarding increased
vascular permeability. (AIIMS May 2019 like pattern)
(a) Immediate transient 1. Sun burn
(b) Immediate prolonged 2. Thorn pick
(c) Delayed transient 3. Sepsis
(d) Delayed prolonged 4. Necrosis with bacterial infection

Ans. a-2, b-3, c-4, d-1


• Immediate transient response…..contraction of endothelial
cells: due to histamine, bradykinin, leukotrienes, and other
chemical mediators mast cell degranulation…Thorn prick
• Immediate prolonged injury……it is due to endothelial injury,
resulting in endothelial cell necrosis and detachment. Direct
damage to the endothelium is encountered in severe injuries, for
example, in burns, or is induced by the actions of microbes
and microbial toxins that target endothelial cells. In most
instances leakage starts immediately after injury and is
sustained for several hours until the damaged vessels are
thrombosed or repaired..
• Delayed transient responce……… it is due to the action of
cytokines leading to endothelial cell retraction. This is seen in
bacterial infections.
• Delayed prolonged leakage..contraction of endothelial cells or
mild endothelial damage……….after gap of few hours and
present for several hours Late-appearing sunburn is a good
example.
5. The correct sequence in the coagulation intrinsic pathway is:
(AIIMS May 2019 like pattern)
(a) 12, 11, 9, 10
(b) 9, 10, 11, 12
(c) 11, 9, 10, 12
(d) 12, 11, 10, 9
Ans. (a) 12, 11, 9, 10
(Ref: Robbins 9th/117-8)
As discussed in the theory, the sequence of the clotting factors in the
intrinsic pathway is 12, 11, 9 and 10.
6. Marker for intrinsic pathway is:
(AIIMS May 2019 like pattern)
(a) Prothrombin time (PT)
(b) Bleeding time (BT)
(c) Clotting time (CT)
(d) Activated partial thromboplastin time (aPTT)
Ans. (d) Activated partial thromboplastin time (aPTT)
(Ref: Robbins 9th/119)
• Prothrombin time (PT) …… extrinsic pathway
• Activated partial thromboplastin time (aPTT)…. intrinsic pathway
• Bleeding time (BT) …… platelet function
• Clotting time (CT) …… capillary tube method

Important points to revise about coagulation profile testing!


• Sample: platelet poor plasma
• Within 2 hours of collection
• To be done at room temperature
• Plastic syringe is used
• 3.2 trisodium citrate is used……. lavender color top tube

7. Which of the following is/are is endogenous pyrogens?


(NEET 2019 like pattern)
(a) IL-1
(b) TNF
(c) Lipopolysaccharide
(d) Both a and b
Ans. (d) Both a and b
(Ref: Robbins 9th e/ p99 )
Exogenous pyrogens include bacterial products like
lipopolysaccharride (LPS) stimulate leukocytes to release
cytokines such as IL-1 and TNF (called endogenous pyrogens)
that increase the enzymes (cyclooxygenases) that convert AA
into prostaglandins. In the hypothalamus, the prostaglandins,
especially PGE2, stimulate the production of neurotransmitters
that reset the temperature set point at a higher level.
8. Which one of following is functions of PGI2?
(NEET 2019 like pattern)
(a) Vasoconstriction and inhibits platelet aggregation
(b) Vasodilator and inhibits platelet aggregation
(c) Vasoconstriction and increases platelet aggregation
(d) Vasodilator and increases platelet aggregation
Ans. (b) Vasodilator and inhibits platelet aggregation
(Ref: Robbins 9th e/p 85)
Prostacyclin is a vasodilator and a potent inhibitor of platelet
aggregation.
9. Macrophage engulfs different cells as shown in the image, the
process is called:
(AIIMS Nov 2018 like pattern)
(a) Phagocytosis
(b) Emperipolesis
(c) Killing
(d) Cytotoxicity
Ans. (b) Emperipolesis
(Ref: Blood and Bone Marrow Pathology 2nd e/p 25)
Emperipolesis describes the presence and movement of one cell
within the cytoplasm of another; the engulfed cell can
subsequently leave the engulfing cell and appears
morphologically unaltered by the interaction. It is very commonly
seen in megakryocytes.
10. Integrin connects actin to which macromolecule in the
extracellular matrix?
(AIIMS Nov 2018 like pattern)
(a) Laminin
(b) Collagen
(c) Fibronectin
(d) Vitronectin
Ans. (c) Fibronectin
(Ref: Robbins 9th e/p 24)
Important adhesive glycoproteins and adhesion receptors
• Fibronectin consists of a disulfide-linked dimer, with several distinct dom
that allow binding to ECM and to integrins.
• Laminin is the most abundant glycoprotein in basement membran
connects cells to underlying ECM components such as type IV collagen
heparan sulfate.
• Integrins are a large family of transmembrane heterodimeric glycoproteins
allow cells to attach to ECM constituents such as laminin and fibronectin,
functionally and structurally linking the intracellular cytoskeleton with
outside world.

11. Which of the following is the correct sequence of cellular


events of acute inflammation?
(AIIMS May 2018 like pattern)
a. Rolling----Stable adhesion---Activation of integrins----
Migration via endothelium
b. Rolling----Activation of Integrins---- Stable Adhesion----
Migration via endothelium
c. Stable adhesion---Rolling----Activation of integrins----
Migration via endothelium
d. Activation of integrins----Migration via endothelium---Stable
adhesion--- Rolling.
Ans. (b) Rolling ----Activation of integrins---- Stable Adhesion----
Migration via endothelium
(Ref: Robbins 9th e/p 76)
The journey of leukocytes from the vessel lumen to the tissue is a
multistep process that is mediated and controlled by adhesion
molecules and cytokines called chemokines. This process can be
divided into sequential phases:
1. In the lumen: margination, rolling, and adhesion to
endothelium.
2. Migration across the endothelium and vessel wall
3. Migration in the tissues toward a chemotactic stimulus.
12. A 12-year-old boy had a cut in his forearm 4 days ago. Now
the bleeding has been stopped due to granulation tissue
formation. While taking a skin biopsy a part of the
granulation tissue was also included in the specimen. The
histology of granulation tissue is shown below. Which type
of collagen is found in this granulation tissue?
(AIIMS May 2018 like pattern)
a. Type 1 b. Type 2
c. Type 3 d. Type 4
Ans. (c) Type 3
• The type of collagen found in the granulation tissue within 2 weeks (after 4
days as is mentioned in the stem of thequestion) is predominantly collagen
type 3.
• After 2 weeks, the collagen in the matrix is collagen type 1.

13. Stellate granuloma is seen in:


(AI 2018 Pattern)
(a) Crohn disease
(b) Berryliosis
(c) Cat scratch disease
(d) Syphilis
Ans. (c) Cat scratch disease
(Ref: Robbins 9/e p27)
14. Warthin Finkeldey cells are seen in:
(AI 2018 Pattern)
(a) Mumps
(b) Measles
(c) Rubella
(d) Chicken pox
Ans. (b) Measles
(Ref: Robbins 9/e p355)
15. Which of the following has pyrogenic activity?
(AI 2018 Pattern)
(a) TGF-beta
(b) IL-6
(c) IL-5
(d) IL-2
Ans. (b) IL-6
(Ref: Robbins 9/e p86, Harrison 19/e p124)
IL-1, IL-6, TNF-a, IFN-a and ciliary neurotropic factor (a member of the
IL-6 family) are pyrogenic cytokines.
16. Which of the following is an opsonin?
(AI 2018 Pattern)
(a) C5a
(b) C3b
(c) C3a
(d) C5b
Ans. (b) C3b
(Ref: Robbins 9/e p78)
17. Which of the following is true about keloid?
(AI 2018 Pattern)
(a) It is associated with elevated levels of growth factors
(b) It has greater amount of collagen and vascularity
(c) It does not spread beyond wound site
(d) Wide excision is the treatment of choice
Ans. (b) It has greater amount of collagen and vascularity
(Ref: Robbins 9/e p109)
Disclaimer
Any resemblance to an actual question is purely coincidental.
• Increased blood volume in a tissue can be due to Hyperemia (due to increased
flow) or congestion (due to impaired venous return).
• Blood in blood vessels normally does not clot because Endothelium is smooth
and coated with glycocalyx.
• Vitamin K dependent clotting factors are factors 2,7,9,10 as well as anti
clotting factors like protein C and protein S.
• Thrombin is a procoagulant but thrombin-thrombomodulin complex is an
anticoagulant.
• Virchow’s triad of thrombosis include: Vascular (endothelial) injury; abnormal
blood flow (stasis or turbulence); hypercoagulability.
• Cancers causing migratory thrombophlebitis are Pancreas (most commonly),
lung (2nd ), prostate, stomach, brain, breast, ovary, lymphomas and AML-M3.
• SLE is most commonly associated with secondary anti phospholipid antibody
syndrome and formation of anti beta 2 glycoprotein antibody.
• Leiden mutation (factor V mutation) is a missense mutation the most
common inheritable cause of hypercoagulability.
• Important laboratory findings of fat embolism: Thrombocytopenia, fat
microglobulinemia, fat globules in urine, anemia, hypocalcemia and
hypoalbuminemia.
• Most common site of DVT is Calf veins whereas most common source of
pulmonary embolism: DVT of proximal veins of lower limb (above knee), i.e.
popliteal femoral or iliac veins.
• White infarct occurs with arterial occlusions in solid organs with end-arterial
circulation (e.g., heart, spleen, and kidney).
• Red infarcts occur in venous occlusions (e.g., testis), in loose tissues (e.g.,
lung), in organs with dual circulations (e.g.,lung and small intestine).
• Lines of Zahn are seen in antemortem clots.
• “Chicken fat” appearance is seen in post mortem clots.
• Characteristic feature of shock: Poor tissue perfusion.
• Endotoxic shock is initiated by Cytokine action. (most important cytokine is
TNF-α followed by IL-1.
• Disseminated intravascular coagulation, hypotensive shock, and metabolic
disturbances (including insulin resistance and hyperglycemia) are referred to as
the clinical triad of septic shock.

Criteria for Systemic Inflammatory Response Syndrome (2 or more of the


following conditions)

• FeverQ (oral temperature >38oC) or hypothermiaQ (<36oC)


• TachypneaQ (>24 breaths/minute)
• Tachycardia Q (>90 beats/minute)
• LeukocytosisQ (>12,000/μl), leucopeniaQ (<4,000/μl), or >10%Q bands

HEMODYNAMICS

In a normal blood vessel like capillary, there are two forces (Starling
forces) acting on the fluid in the circulation. The hydrostatic pressure
causes fluid movement from inside the vessel to outside and the colloid
osmotic pressure (mostly due to proteins) is responsible for the reverse
movement of fluid from outside the vessel to the inside. The capillary
hydrostatic and osmotic forces are normally balanced so that there is no
net loss or gain of fluid across the capillary bed. However, increased
hydrostatic pressure or diminished plasma osmotic pressure leads to a net
accumulation of extravascular fluid (edema).
In edema, the excessive interstitial fluid can be either an exudate or a
transudate.

A transudate is a fluid with low protein content (most of which is albumin) and a
specific gravity of less than 1.012. It is essentially an ultrafiltrate of blood plasma
that results from osmotic or hydrostatic imbalance across the vessel wall without
an increase in vascular permeability.
An exudate is an inflammatory extravascular fluid that has a high protein
concentration, cellular debris, and a specific gravity above 1.020. It is formed
mainly due to alteration in the normal permeability of small blood vessels in the
area of injury.

Causes and conditions associated with edema


↑ Hydrostatic ↓ Plasma osmotic Lymphatic Sodium Inflammation
pressure pressure obstruction retention
(Lymphedema)
• CHF • Liver cirrhosis • After surgery • ↑ Salt • Acute and
• Ascites • Malnutrition or irradiation intake chronic
(Cirrhosis) • Protein-losing • Neoplasia • ↓ Renal inflammation
• Venous gastroenteropathy • Inflammatory perfusion
obstruction • ↑ RAAS
due to activity
thrombosis
of physical
inactivity
• Arteriolar
dilation

When edema is influenced by gravity, it is called dependent


edema and it is a characteristic feature of congestive heart failure
(particularly the right ventricle).
• Edema due to a renal cause (as in Nephrotic syndrome) is more
severe and affects all parts of body equally. However, it is initially
appreciated in tissue with loose tissue matrix such as around eyes
and is called periorbital edema.

Hyperemia Congestion
• Active process due to arteriolar • Passive process due to impaired venous
dilation outflow
• Edema is absent • Edema is present
• Red color of the tissues • Blue red color of the tissue (due to
deoxyhemoglobin)
• Seen in Inflammation • Seen in Right heart failure, portal venous
obstruction in cirrhosis.

In chronic passive congestion, there is chronic hypoxia and capillary


rupture leading to hemorrhage foci. Phagocytosis of red cells
results in hemosiderin-laden macrophages.
Pulmonary Congestion
In acute pulmonary congestion there is presence of engorged alveolar capillaries and
focal intra-alveolar hemorrhage.
In chronic pulmonary congestion there is presence of thickened and fibrotic septa and
alveoli contain hemosiderin laden macrophages (heart failure cells).

Hepatic Congestion
Acute hepatic congestion manifest as central vein and sinusoidal distension with
degeneration of central hepatocytes.
In chronic hepatic congestion, central region of lobule shows loss of cells and have red
brown color which is accentuated against surrounding normal liver (called nutmeg
liver). Initially there is centrilobular necrosis and presence of hemosiderin laden
macrophages. In long standing congestion (as in heart failure), there is presence of
hepatic fibrosis called cardiac cirrhosis.

Fig. 1: Nutmeg liver. ...(AIIMS Image)

HEMOSTASIS

It is defined as a sequence of events leading to cessation of bleeding by


the formation of a stable fibrin-platelet hemostatic plug. The process
involves the vascular endothelium, platelets and the coagulation system.
1. Vascular Endothelium
After an injury, there is transient vasoconstriction of the vessel due to
endothelin. This is followed by the activation of thrombogenic
factors which include:
a. Alteration in the blood flow resulting in turbulence and stasis
favoring the clot formation.
b. There is release of tissue factor from the injured cells activating
the factor VII (extrinsic pathway) and exposure to subendothelial
collagen causing activating of factor XII (intrinsic pathway). The
endothelial cells also release von Willebrand factor (vWF) which
binds to exposed collagen and facilitates platelet adhesion.
c. There is also release of inhibitors of plasminogen activator (PAIs)
which inhibit fibrinolysis.
2. Platelets
Initially vWF binds with the collagen followed by the binding of the
platelets with vWF through the glycoprotein Ib factor. The platelets
then undergo a shape change and their degranulation occurs. The
granules in the platelets can be Alpha granules or delta granules.
The release of granule mediators result in release of calcium
(required in the coagulation cascade), ADP (potent mediator of
platelet aggregation) and the surface expression of phospholipid
complexes, which provide the platform for the coagulation
cascade. The platelets also synthesize thromboxane A2 (TXA2)
which is a potent vasoconstrictor and is also responsible for
platelet aggregation. The platelets bind to each other by binding to
fibrinogen using Gp IIb-IIIa.

3. The Coagulation System


The clotting system can be activated by intrinsic (through factor XII) or
the extrinsic pathway (through factor VII). The intrinsic pathway is
activated by exposing factor XII to thrombogenic surfaces (like
glass and other negatively charged surfaces) whereas extrinsic
pathway requires exogenous trigger (provided originally by tissue
extracts). The division is artifact because extrinsic pathway is
physiologically relevant for after vascular damage whereas
intrinsic pathway is of relevance in vitro.

Prothrombin time (PT)


• It is used to monitor the functioning of the extrinsic and the common coagulation
pathways.
• Normal PT is 12-16 seconds.

Activated partial thromboplastin time (aPTT)


• It is used to monitor the functioning of the intrinsic and the common coagulation
pathways.
• Normal aPTT is 26-34 seconds.
• A relatively rare cause of prolonged aPTT is presence of antibodies against
coagulation plasma proteins called inhibitors. It can be seen due to the following
reasons:
Hemophilia A and B patients receiving clotting factors to control their bleeding
episodes, Pregnancy, Autoimmune diseases, Malignancies (lymphoma, prostate
cancer) and Dermatologic conditions This has been dealt extensively in chapter-8 of
this book.

Thrombin time (TT)


• It is used for testing the conversion of fibrinogen into fibrin and depends on
adequate fibrinogen levels.

Bleeding time (BT)


• It is the time taken for a standardized skin puncture to stop bleeding
• It tests the ability of blood vessels to constrict and platelets to form a hemostatic
plug.

Fibrin degradation products (FDPs)


• They are used to assess the fibrinolytic activity and they are increased in
disseminated intravascular coagulation (DIC).

Important anticoagulant substances are:


1. Antithrombin III – It inhibits activity of thrombin and other factors
like XIIa, XIa, Xa and IXa. It is activated by binding to heparin-
Like molecules.
2. Proteins C and S – Vitamin K dependent proteins which
inactivate Va and VIIIa.
3. Endothelial prostacyclin (PGI2) and nitric oxide (NO) are potent
vasodilators and inhibitors of platelet aggregation.
4. Tissue factor pathway inhibitor – Derived from endothelium
and inhibits tissue factor VIIa and Xa molecules.
5. Thrombomodulin binds to thrombin and this complex activates
protein C (in the presence protein S), which finally inactivates
factor V and VIII. This action result in anticoagulant effect.
The binding of clotting factors II, VII, IX and X to calcium depends on the addition
of g carboxylation of glutamic acidQ residues on these proteins using vitamin K
as a cofactor. These clotting proteins are prothrombotic. Proteins C and S are
two other vitamin K-dependent proteins which can inactivate factors Va and VIIIa.
These are anticlotting factors.

THROMBOSIS

It is defined as the pathologic formation of intravascular fibrin-platelet


thrombus.
Virchow’s triad is required for thrombus formation. Its components are:
1. Endothelial injury: It can be due to the factors like vasculitis,
hypertension, turbulent flow, bacterial endotoxins, homocystinuria,
hypercholesterolemia, radiation, etc. It is particularly important for
thrombus formation occurring in the heart or in the arterial
circulation.
2. Alterations in the normal blood flow: Both turbulence and stasis
contribute to the development of thrombosis. Turbulence causes
arterial whereas stasis causes venous thrombosis. It can also be
seen with hyperviscosity syndromes like polycythemia and with
deformed red cells as in sickle cell anemia.
3. Blood hypercoagulability: It can be either primary or secondary
hypercoagulable state.

Hypercoagulable states
Primary (Genetics) Secondary (Acquired)
• Mutations in factor V (Most common) • Prolonged bed rest or immobilization
• Antithrombin III deficiency • Homocysteinemia
• Protein C or S deficiency • Tissue damage (Surgery, fracture,
• Fibrinolysis defects burns)
• Homocysteinemia • Cancer
• Allelic variations in prothrombin levels • MI, Prosthetic cardiac valves
• Mutations in the methyl tetra hydro • DIC (Disseminated intravascular
folate (MTHF) gene coagulation)
• Heparin induced thrombocytopenia
• Antiphospholipid antibody syndrome

Relationship between coagulation defect and site of thrombosis (updated


from Harrison18th/462)

• Factor V mutation (also called Leiden mutation) is the most


common inherited cause of hypercoagulability in which normal
arginine is replaced by glutamine at position 506 making it
resistant to degradation by protein C. This causes unchecked
coagulation and it manifest with recurrent deep venous
thrombosis.
• Antithrombin III, Protein C or Protein S deficiency are other genetic
causes of hypercoagulability manifesting typically as venous
thrombosis and recurrent thromboembolism in adolescence or
early adult life.
• All conditions mentioned above would have presence of venous
thrombosis except for the following:

Conditions with both arterial and venous thrombi


• HomocysteinuriaQ • Essential thrombocythemiaQ
• Antiphospholipid antibodyQ • CancerQ
• HyperhomocysteinemiaQ • PNHQ
• Disseminated intravascular • Polycythemia veraQ
coagulationQ • DysfibrinogenemiaQ
• Heparin induced thrombocytopeniaQ

Arterial and Venous Thrombi

• An area of attachment to the underlying vessel or heart wall,


frequently firmest at the point of origin, is characteristic of all
thromboses. Venous thrombi are called as stasis thrombi
because they are formed in the sluggish venous circulation.
These are also known as red thrombi as they contain more
enmeshed red cells and relatively few platelets. Arterial
thrombus contains more platelets and relatively less fibrin.

Feature Arterial thrombus Venous thrombus


Pathogenesis Endothelial injury or site of Stasis of blood
turbulence
Blood flow Associated with active Associated with sluggish blood
blood flow flow
Sites Coronary, cerebral and Superficial and deep leg veins,
femoral arteries ovarian/periuterine veins
Propagation Grows in a retrograde Grows in an antegrade manner
manner from point of from point of attachment
attachment
Gross Lines of Zahn present Lines of Zahn absent
Microscopic Pale platelet layer Red cells mixed with relatively
alternating with dark red cell less platelets, so also called as
layer so also called as red thrombi
white thrombi
Occlusion Incomplete lumen Complete vessel occlusion
occlusion
Complications Ischemia and infarction of Embolism, edema and
organs ulceration

Postmortem clots are gelatinous with a dark red dependent portion where red
cells have settled by gravity and a yellow chicken fat supernatant resembling
melted and clotted chicken fat. These are usually not attached to the underlying
wall whereas as discussed above, an area of attachment is characteristic of all
thrombosis. Thrombi may form on heart valves as seen in infective endocarditis;
nonbacterial thrombotic endocarditis and verrucous (Libman-Sacks)
endocarditis.
EMBOLISM

An embolus is a detached intravascular solid, liquid, or gaseous mass that


is carried downstream from its site of origin. It is most commonly
composed of thromboembolism. The emboli may also be composed of
other types like atheroemboli, fat emboli (most commonly with skeletal
injuries), air emboli, amniotic fluid emboli and tumor emboli.

Fig. 2: Pulmonary embolus. ...(All India Image)


Pulmonary Emboli

Most of the pulmonary emboli arise in the deep leg veins above the level of the
knee. Paradoxical embolus is a rare embolus that can pass through an inter-atrial or
inter-ventricular defect, thereby entering the systemic circulation. Most pulmonary
emboli (60% to 80%) are clinically silent because they are small. They rarely may
cause pulmonary infarction (because lungs have dual blood supply from pulmonary
and bronchial vessels) manifesting clinically as breathlessness, pleuritic pain,
hemoptysis and pleural effusion. Sudden death may occur if >60% of the pulmonary
circulation is obstructed. Recurrent pulmonary emboli may also cause pulmonary
hypertension which may lead to cor pulmonale.

Systemic Thromboembolism

Most of them arise in the heart and the major sites of arterial embolization are the lower
extremities (75%), the brain (10%) and less commonly, the intestines, kidneys, spleen,
and upper extremities.

Fat Embolism

Fat embolism syndrome is characterized by pulmonary insufficiency, neurologic


symptoms (irritability, restlessness and even coma), anemia, and thrombocytopenia
(manifesting as diffuse petechial rash). It is seen after fractures of long bones (which
contain fatty marrow) or after soft-tissue trauma. It is fatal only in 10% of cases. The
pathogenesis involves both mechanical obstruction and free fatty acids causing local
toxic injury to endothelium.

Fig. 3: Fat embolism. ...(AIIMS Image)

Infarct

It is an area of necrosis caused by occlusion of either the arterial supply or


the venous drainage in a particular tissue. Nearly 99% of all infarcts result
from thrombotic or embolic events, and almost all result from arterial
occlusion. Venous thrombosis usually causes venous obstruction and
congestion but can cause infarction (more in organs with a single venous
outflow like testis and ovary). The infarcts may be either red (hemorrhagic)
or white (anemic) and may be either septic or bland.
Fig. 4: White infarct

Feature Red infarcts White infarcts

Cause • Arterial occlusion in loose • Arterial occlusions in solid


tissues or organs having organs with end arterial
dual blood supply circulation
• Venous occlusion (in
ovarian torsion)

Affected organs Lung and small intestine Solid organs (heart, spleen,
kidney)

Properties Ill define hemorrhagic Well defined margins and


margins which change in color progressively paler with time
to brown

Edema Usually present Usually absent

Disseminated Intravascular Coagulation (DIC)/Consumption


Coagulopathy

DIC is an acute, subacute, or chronic thrombohemorrhagic disorder


occurring as a secondary complication in a variety of diseases. It is
characterized by activation of the coagulation sequence that leads to the
formation of microthrombi throughout the microcirculation of the body. As a
consequence of the thrombotic diathesis, there is consumption of platelets,
fibrin, and coagulation factors and, secondarily, activation of fibrinolytic
mechanisms.
Causes of DIC
Obstetrics Infections Neoplasm Massive Miscellaneous
complications tissue
injury

- Abruptio - Gram negative - Ca pancreas - - Acute


placenta sepsis - Ca prostate Traumati intravascular
- Retained - - Ca lung c hemolysis
dead fetus Meningococce - Ca stomach - Burns - Snake bite
- Septic mia - Acute - Extensive - Shock
abortion - Histoplasmosis promyelocytic surgery - Heat stroke
- Amniotic - Aspergillosis leukemia - Vasculitis
fluid - Malaria - Aortic
embolism - Toxemia aneurysm
- Toxemia - Liver
disease

Laboratory investigations in DIC reveal that


• Platelet count is decreased
• Prolonged PT/TT
• Decreased fibrinogen
• Elevated fibrin split products (D-dimers)
The management is to treat the underlying disorder.

SHOCK

It is defined as systemic hypoperfusion caused by reduction either in


cardiac output or in the effective circulating blood volume.

Causes of Shock

• Cardiogenic shock: Presence of cardiac pump failure as seen with


myocardial infarction, cardiac arrhythmia, cardiac tamponade and
pulmonary embolism.
• Hypovolemic shock: Caused by reduction of blood volume due to
hemorrhage, severe burns and severe dehydration.
• Neurogenic shock: Presence of generalized vasodilation due to
anesthesia and CNS injury.
• Anaphylactic shock: Presence of generalized vasodilation due to
type I hypersensitivity reaction
• Septic shock: It is due to release of endotoxins or bacterial cell
wall lipopolysaccharides (gram negative infections) resulting in
production of cytokines like TNF-alpha, IL-1 and IL-6, vasodilation
and hypotension, acute respiratory distress syndrome and
multiple organ dysfunction syndrome. Pathogenesis of septic
shock is explained in the flowchart below:

LPS: Lipopolysaccharide LBP: LPS Binding Protein TLR: Toll Like


Receptor

STAGES OF SHOCK
Stage I: Stage of compensation in which perfusion to the vital
organs is maintained by mechanisms like increased sympathetic
tone, catecholamine release and activation of renin-angiotensin
system.
Stage II: Stage of decompensation in which there is tissue
hypoperfusion and other features like development of metabolic
acidosis, electrolyte disturbances and renal insufficiency.
Stage III: Irreversible stage having irreversible tissue injury and
multiple organ failure which is not even corrected by the removal
of the underlying cause or correction of the hemodynamic
disturbance.

FEATURES OF SHOCK

Brain Ischemic encephalopathy

Heart Coagulative necrosis or contraction band necrosis

Liver Fatty change with hemorrhagic central necrosis; ‘shock liver’

Kidneys Extensive tubular ischemic injury (Acute tubular necrosis)


leading to oliguria and electrolyte disturbances.

Adrenal Cortical cell lipid depletion.

GIT Hemorrhagic enteropathy.


Lungs are uncommonly affected but may show features of diffuse alveolar damage or
shock lung.

Clinical Features
In hypovolemic and cardiogenic shock, the patient presents with
hypotension; a weak, rapid pulse; tachypnea; and cool, clammy, cyanotic
skin. In septic shock, however, the skin may initially be warm and flushed
because of peripheral vasodilation. Then, patients develop a second
phase dominated by renal insufficiency and marked by a progressive fall in
urine output as well as severe fluid and electrolyte imbalances.
HEMODYNAMICS AND HEMOSTASIS

1. All are true about blood coagulation except?


(AI 2011)
(a) Factor X is a part of both intrinsic and extrinsic pathway.
(b) Extrinsic pathway is activated by contact of plasma with
negatively charged surfaces.
(c) Calcium is very important for coagulation.
(d) Intrinsic pathway can be activated in vitro.
2. Vitamin K is responsible for the carboxylation of which amino
acid in the clotting factors?
(AI 2011)
(a) Aspartate
(b) Glutamate
(c) Proline
(d) Lysine
3. Edema in nephrotic syndrome occurs due to
(a) Na+ and water restriction
(AIIMS Nov. 2010)
(b) Increased venous pressure
(c) Decreased serum albumin
(d) Decreased fibrinogen
4. Thrombomodulin thrombin complex prevents clotting because:
(DPG 2011)
(a) Thrombomodulin inhibits prothrombin activator
(b) The complex activates antithrombin III
(c) Thrombomodulin-thrombin complex activates heparin
(d) The complex removes thrombin and also activates protein C
which inactivates the activated factors V and VIII
5. Vitamin K associated clotting factors are:
(a) IX, X
(b) I, V
(AI 2010)
(c) VII, VIII
(d) I, VIII
6. All endothelial cells produce thrombomodulin except those found
in:
(AI 2005)
(a) Hepatic circulation
(b) Cutaneous circulation
(c) Cerebral microcirculation
(d) Renal circulation
7. Which of the following is a procoagulation protein?
(AI 2004)
(a) Thrombomodulin
(b) Protein C
(c) Protein S
(d) Thrombin
8. All of the following are correct about Thromboxane A2 except:
(AI 2001)
(a) Low dose aspirin inhibits its synthesis
(b) Causes vasoconstriction in blood vessels
(c) Causes bronchoconstriction
(d) Secreted by WBC
9. Coagulation defects associated with increased coagulation are
seen in:
(PGI Dec 2006)
(a)Increased protein C
(b) Increased protein S
(c) Increased anti-thrombin III
(d) Protein C resistance
(e) Dysfibrinogenemia
10. All of the following are anticoagulant substances except:
(Karnataka 2006)
(a) Antithrombin III
(b) Protein S
(c) vWF
(d) Nitric oxide

MOST RECENT QUESTIONS

11. Cause of edema is:


(a) Decreased plasma protein concentration
(b) Increased lymph flow
(c) Increased ECF volume
(d) Increased plasma protein concentration
12. Endothelium derived relaxing factor (EDRF) is associated with:
(a) Ras
(b) C-myc
(c) Bcl
(d) nNOS
13. Which is not involved in local hemostasis?
(a) Fibrinogen
(b) Calcium
(c) Vitamin K
(d) Collagen
14. Which is the following not synthesized in the liver?
(a) Factor II
(b) Factor VII
(c) Factor IX
(d) Factor VIII
15. A 54-year-old chronic alcoholic Adhiya Kumar is brought by his
son as he has developed progressively increasing abdominal
distension from past 3 months. The physician aspirates the
abdominal fluid which is straw-colored and clear and is found to
have protein content (mainly albumin) of 2.3 g/dl. Which of the
following is a major contributor to the fluid accumulation in this
patient?
(a) Blockage of lymphatics
(b) Decreased oncotic pressure
(c) Decreased capillary permeability
(d) Inflammatory exudation
16. Gandy gamma body is typically seen in chronic venous
congestion of which of the following?
(a) Lung
(b) Kidney
(c) Spleen
(d) Liver
17.Extrinsic pathway of clotting factors is measured by?
(a) Prothromin time
(b) Activated partial thromboplastin time
(c) Bleeding time
(d) Clotting time
18. Tissue thromboplastin activates:
(a) Factor VII
(b) Factor IV
(c) Factor VI
(d) Factor XII
19. Platelet adhesion to collagen is mediated by which of the
following?
(a) Factor VIII
(b) Factor IX
(c) Von Willebrand factor
(d) Fibronectin
20. Adhesion of platelets to collagen is due to:
(a) Factor IX
(b) Fibrinogen
(c) Von Willebrand factor
(d) Fibronectin
21. The clot formed after coagulation cascade is not stable unless
extensive cross-linking occurs. This is done by:
(a) Plasmin
(b) Thrombin
(c) Factor XII
(d) High molecular weight kininogen

THROMBOSIS: EMBOLISM: INFARCT

22. Histologic sections of lung tissue from 66-year-old woman,


Sheena with congestive heart failure and progressive breathing
problems reveal numerous hemosiderin-laden cells within the
alveoli. Which of the following is the cell of origin of these
“heart failure cells”?
(a) Endothelial cells
(b) Pneumocytes
(c) Lymphocytes
(d) Macrophages
23. At autopsy, the spleen of a patient is noted to have a thickened
capsule and many small, scarred areas. Microscopic
examination of the scarred areas reveals fibrosis with
hemosiderin and calcium deposition. This type of spleen is
usually seen in conjunction with which of the following
disorders?
(a) Hepatic cirrhosis
(b) Hodgkin’s disease
(c) Rheumatoid arthritis
(d) Sickle cell anemia
24. Antiphospholipid syndrome is associated with all except:
(AI 2012)
(a) Recurrent abortion
(b) Venous thrombosis
(c) Pancytopenia
(d) Antibody to lupus
25. Pale infarct is seen in all except:
(AIIMS Nov 2010)
(a) Lungs
(b) Spleen
(c) Kidney
(d) Heart
26. Congenital hypercoagulability states are all of the followings
except:
(a) Protein C deficiency
(AIIMS Nov 2010)
(b) Protein S deficiency
(c) Anti-phospholipid antibody syndrome
(d) MTHFR gene mutation
27. Fat embolism is commonly seen in:
(DPG 2011)
(a) Head injuries
(b) Long bone fractures
(c) Drowning
(d) Hanging
28. Virchow’s triad includes all except:
(a) Injury to vein
(b) Venous thrombosis
(c) Venous stasis
(d) Hypercoagulability of blood
29. Hypercoagulability due to defective factor V gene is called:
(AIIMS Nov 2003)
(a) Lisbon mutation
(b) Leiden mutation
(c) Antiphospholipid syndrome
(d) Inducible thrombocytopenia syndrome
30. Arterial thrombosis is seen in:
(PGI June 2003)
(a) Homocysteinemia
(b) Antiphospholipid syndrome
(c) Protein S deficiency
(d) Protein C deficiency
(e) Antithrombin III deficiency
31. Hemorrhagic infarction is seen in:
(PGI Dec 2002)
(a) Venous thrombosis
(b) Thrombosis
(c) Septicemia
(d) Embolism
(e) Central venous thrombosis
32. Hyperviscosity is seen in:
(PGI Dec 2003, 04)
(a) Cryoglobulinemia
(b) Multiple myeloma
(c) MGUS
(d) Lymphoma
(e) Macroglobulinemia
33. Predisposing factor for venous thrombosis:
(a) AT III deficiency
(b) Protein S deficiency
(c) Protein C deficiency
(d) Dysfibrinogenemia
34. Inherited coagulation disorders are:
(PGI Dec 2005)
(a) Protein C deficiency
(b) Protein S deficiency
(c) Leiden factor mutation
(d) Lupus anticoagulant
(e) Anti-cardiolipin
35. Which of the following statements about pulmonary emboli is
not correct?
(Delhi PG 2009 RP)
(a) 60-80% pulmonary emboli are clinically silent
(b) In more than 95% cases venous emboli originate from deep leg
veins
(c) Embolic obstruction of pulmonary vessels almost always cause
pulmonary infarction
(d) Embolic obstruction of medium sized arteries may result in
pulmonary infarction

MOST RECENT QUESTIONS

36. Which one of the following inherited disorders produces arterial


thrombosis?
(a) Factor V Leiden mutation
(b) Antithrombin deficiency
(c) Homocysteinemia
(d) Protein S deficiency
37. Heart failure cells are seen in:
(a) Chronic venous congestion of liver
(b) Chronic venous congestion of lung
(c) Acute venous congestion of lung
(d) Acute venous congestion of liver
38. Necrosis with putrefaction is called as:
(a) Desiccation
(b) Gangrene
(c) Liquefaction
(d) Coagulative necrosis
39. Lines of Zahn are found in:
(a) Thrombus
(b) Infarct tissue
(c) Postmortem clot
(d) All
40. Chicken fat clot is:
(a) Postmortem clot
(b) Thrombus
(c) Infarct
(d) All
41. Lines of Zahn occur in which of the following?
(a) Postmortem clot
(b) Infarct
(c) Embolus
(d) Coralline thrombus
42. White infarcts are seen in the following except:
(a) Liver
(b) Kidney
(c) Spleen
(d) Heart
43. White infarct is seen in:
(a) Lung
(b) Intestine
(c) Heart
(d) Ovary
44. A 58-year-old man suffered road traffic accident and came to the
hospital. He had multiple fractures in his lower limbs, ribs and
lung contusion. Ultimately he succumbed to his injuries. At
autopsy, a biopsy from the lung showed the following
appearance. What is the likely cause of his death?
(AIIMS Nov 2016)

(a) Fat embolism


(b) Emphysema
(c) Pulmonary embolism
(d) Congestive heart failure
45. Leiden mutation is an example of which of the following
mutations:
(a) Non sense mutation
(b) Mis-sense mutation
(c) Frameshift mutation
(d) Trinucleotide repeat mutation
46. Lines of Zahn are seen in:
(a) Heart
(b) Liver
(c) Lungs
(d) Kidneys
SHOCK, DIC AND MISCELLANEOUS

47. All of the following are true about DIC except?


(AI 2012)
(a) Increased fibrinogen
(b) Increased activated partial thromboplastin time
(c) Decreased prothrombin time
(d) Increased fibrin degradation products
48. The initiating mechanism in endotoxic shock is
(a) Peripheral vasodilatation
(AIIMS Nov 2010)
(b) Endothelial injury
(c) Increased vascular permeability
(d) Reduced cardiac output
49. Reverse transcriptase is a RNA dependent DNA polymerase.
Which of these viruses has it?
(a) Hepatitis A virus
(b) Hepatitis B virus
(c) Hepatitis E virus
(d) Hepatitis C virus
50. The initiating mechanism in endotoxic shock is:
(AIIMS Nov 2010)
(a) Peripheral vasodilatation
(b) Endothelial injury
(c) Increased vascular permeability
(d) Cytokine release
51. D-Dimer is the most sensitive diagnostic test for:
(DPG 2011)
(a) Pulmonary embolism
(b) Acute pulmonary oedema
(c) Cardiac tamponade
(d) Acute myocardial infarction
52. Shock lung is characterized by:
(AIIMS May 2008; Nov 2007)
(a) Alveolar proteinosis
(b) Bronchiolitis obliterans
(c) Diffuse pulmonary hemorrhage
(d) Diffuse alveolar damage
53. The histological features of shock includes:
(a) ATN
(b) Pulmonary congestion
(c) Depletion of lipids in adrenal cortex
(d) Hepatic necrosis
(e) Depletion of lymphocytes
54. Conditions associated with incoagulable state are:
(a) Abruption placentae
(PGI Dec 2003, 2004)
(b) Acute promyelocytic leukemia
(c) Severe falciparum malaria
(d) Snake envenomation
(e) Heparin overdose
55. Which of the following is a feature of Disseminated Intravascular
Coagulation (DIC)?
(Karnataka 2006)
(a) Normal prothrombin time
(b) Reduced plasma fibrinogen
(c) Normal platelet count
(d) Normal clotting time
1. Ans. (b) Extrinsic pathway is activated by contact of plasma with
negatively charged surfaces
(Ref: Robbins 8th/119, 9/e 118)
Contact of plasma with negative charged surface activates intrinsic and
not extrinsic pathways.
2. Ans. (b) Glutamate
(Ref: Robbins 8th/119, 9/e p119)
The binding of clotting factors II, VII, IX and X to calcium depends on the
addition of γ carboxylation of glutamic acid residues on these
proteins. This step requires vitamin K as a cofactor.
Vitamin K dependent factors
Increasing clotting Inhibiting clotting
• Clotting factors II, VII, IX and X • Protein C and protein S
Proteins C and S are two other vitamin K-dependent proteins which can
inactivate factors Va and VIIIa. These are anticlotting factors.
3. Ans. (c) Decreased serum albumin
(Ref: Robbins 8th/922, 9/e p115)
+
Na and water retention is now the more important cause of edema in
nephrotic syndrome. For details see the Chapter on Kidney.

Receptor Na+ and water retention is not to be confused with option (a) Na+ and
water restriction.

4. Ans. (d) The complex removes thrombin and also activates


protein C which inactivates the activated factors V and VIII
(Ref: Harrison 17th/364-5, Robbins 8th/116, 9/e p121)
5. Ans. (a) IX, X
(Ref:Robbins 8th/118-119 9/e p119)
6. Ans. (c) Cerebral microcirculation
(Ref: Ganong 21st/546, Robbins 7th/85, Robbins 8th/116, Harrison
17th/365, 9/e p121)
‘All endothelial cells except those in the cerebral microcirculation
produce thrombomodulin, a thrombin protein, and express it on their
surface’.
7. Ans. (d) Thrombin
(Ref: Robbins 7th/127, Harrison 17th/364, 9/e p120-121)
8. Ans. (d) Secreted by WBC
(Ref: Robbins 9/e p118)
Thromboxane A2 (TXA2) is synthesized and released from activated
platelets (Not WBCs)
• TXA2 is also a powerful vasoconstrictor and bronchoconstrictor.
• Low does aspirin (50-325 mg) is used as antiplatelet drug
because it inhibits COX irreversibly and decreases formation of
TXA2 by platelets.
9. Ans. (d) Protein C resistance; (e) Dysfibrinogenemia
(Ref: Harrison 16th/149l, 9/e p123)
10. Ans. (c) Von Willebrand factor (vWF)
(Ref: Robbins 7th/125-126/129-130, 9/e p121,118)
VWF (von Willebrand Factor) is produced by endothelial cells and it is required
for platelet binding to collagen and other substances. So, it is a procoagulant
factor.

11. Ans. (a) Decreased plasma protein concentration


(Ref: Robbins 8th/112; 7th/120-121, 9/e p114)
12. Ans. (d) nNOS
(Ref: Robbins 8th/60, 7th/72-73, 9/e p80)
13. Ans. (c) Vitamin K
(Ref: Robbins 9/e p118-119)
14. Ans. (d) Factor VIII
(Ref: Robbins 9/e p47-118)
15. Ans. (b) Decreased oncotic pressure
(Ref: Robbins 8th/112, 9/e p114)
The patient in the stem of the question is most likely having liver cirrhosis
secondary to chronic alcoholism. An important manifestation of this
disease is reduced hepatic synthesis of albumin which is the most
important contributor to plasma oncotic pressure. Also, ascites is
associated with increased sodium and water retention because of
stimulation of the renin-angiotensin aldosterone system (RAAS). A
minor contribution is also because of hydrostatic forces (due to intra-
hepatic scarring and partial obstruction of the portal venous return)
resulting in fluid transudation and increased secretion of hepatic
lymph.
16. Ans. (c) Spleen
(Ref: Harsh Mohan 6th/52)
Gamna-Gandy bodies in chronic venous congestion (CVC) of the spleen is
characterized by calcific deposits admixed with haemosiderin on
fibrous tissue.
17. Ans. (a) Prothromin time
(Ref: Robbin 9/e p119)
Prothrombin time Extrinsic pathway Factor 5/7

Activated partial Intrinsic pathway Factor 8


thromboplastin time

Bleeding time Platelet function and Platelet function and


platelet count count

18. Ans. (a) Factor VII


(Ref: Robbins 9/e p118)
The extrinsic pathway is activated by tissue factor (thromboplastin)
causing activation of factor VIIa. For details, refer to text.
19. Ans. (c) Von Willebrand factor
(Ref: Robbin 8/e p116-8)
Direct lines.. “Von Willebrand factor functions as an adhesion bridge
between subendothelial collagen and the glycoprotein Ib (Gp Ib)
platelet receptor. Aggregation is accomplished by fibrinogen bridging
GpIIb-IIIa receptors on different platelets.”
20. Ans. (c) Von Willebrand factor
(Ref: Robbins 9th/117)
Direct quote …. “Platelet adhesion is mediated largely via interactions with
vWF, which acts as a bridge between the platelet surface receptor
glycoprotein Ib (GpIb) and exposed collagen.”
21. Ans. (b) Thrombin
(Ref: Robbins 9th/119)
Thrombin also stabilizes the secondary hemostatic plug by activating
factor XIII, which covalently cross-links fibrin.
22. Ans. (d) Macrophages
(Ref: Robbins 9/e p116)
• Example of tissue macrophages are Kupffer cells (liver), alveolar
macrophages (lung), osteoclasts (bone), Langerhan’s cells
(skin), microglial cells (central nervous system)
• In the lung, alveolar macrophages can phagocytose the red
blood cells that accumulate in alveoli in individuals with
congestive heart failure. These cells contain hemosiderin and
are referred to as “heart failure cells.”
23. Ans. (a) Hepatic cirrhosis
(Ref: Robbins 9/e p530)
The spleen shows the changes of chronic congestive splenomegaly,
typically associated with hepatic cirrhosis. The described small scars
are called Gandy-Gamma nodules which are due to the result of
organization of old hemorrhages.
• Hodgkin’s disease (choice B) produces large splenic nodules in which
Reed-Sternberg cells can be found surrounded by mature lymphocytes,
eosinophils, and neutrophils.
• Rheumatoid arthritis (choice C) and many other chronic inflammatory
disorders induce reactive hyperplasia of the spleen with formation of many
large germinal centers in the splenic follicles.
• Sickle cell anemia (choice D) produces many small (often triangularly
shaped) infarctions in the spleen.

24. Ans. (c) Pancytopenia


(Ref: Harrison 17th/1795, Robbins 8th/123,215, 7th/133, 229, 9/e p124-
125)

Antiphospholipid antibody syndrome is characterized by antibodies against


plasma proteins in complex with phospholipid. In primary antiphospholipd
antibody syndrome there is hypercoagulable state without evidence of
autoimmune disorders. In association with SLE or lupus, the name given is
secondary antiphospholipd antibody syndrome. There is formation of antibody
against phospholipid beta– 2-glycoprotein 1 complexQ. It also binds to
cardiolipin antigen and lead to false positive test for syphilisQ. It also interferes
with in vitro clotting time and so, called as lupus anticoagulant. In vivo, these
patients have hypercoagulable state resulting in arterial and venous thrombosis
resulting spontaneous recurrent miscarriage and focal or cerebral ischemia.

25. Ans. (a) Lungs


(Ref: Robbins 8th/128, 9/e p129)
The Lungs have dual blood supply and so, they exhibit red infarct. The
infarcts may be either red (hemorrhagic) or white (anemic) and may
be either septic or bland.
All infarcts tend to be wedge shaped with the occluded vessel at the apex and
the periphery of the organ forming the base. The infarct microscopically has
features of ischemic coagulative necrosis.

26. Ans. (c) Anti-phospholipid antibody syndrome


(Ref: Robbins 8th/123, 9/e p123)
Anti-phospholipid antibody syndrome is an acquired causes of
hypercoagulability
Hypercoagulable states
Primary (Genetics) Secondary (Acquired)
• Mutations in factor V (Most common) • Prolonged bed rest or immobilization
• Antithrombin III deficiency • Homocysteinemia
• Protein C or S deficiency • Tissue damage (Surgery, fracture,
• Fibrinolysis defects burns)
• Homocysteinemia • Cancer
• Allelic variations in prothrombin levels • MI, Prosthetic cardiac valves
• Mutations in the methyl tetra hydro • DIC (Disseminated intravascular
folate (MTHF) gene coagulation)
• Heparin induced thrombocytopenia
• Antiphospholipid antibody syndrome

27. Ans. (b) Long bone fractures (Ref:


Robbins 8th/126, 9/e p128)
28. Ans. (b) Venous thrombosis
(Ref: Anderson 10th/387, Robbin’s 7th/130, 9/e p122)
The factors that predispose to venous thrombosis were initially described
by Virchow in 1856 and are known as Virchow’s triad. These
include:
• StasisQ • Vascular damageQ • HypercoagulabilityQ
29. Ans. (b) Leiden mutation
(Ref: Robbins 7th/p131, Robbins 8th/122, 9/e p123)
• Mutation in factor V gene is caused by the substitution of
glutamine for the normal arginine residue at position 506. It is
known as Leiden mutation and it is the most common
inherited cause of hypercoagulability.

Note: Lisbon mutation is associated with a mutation in thyroid peroxidase


geneQ.

30. Ans. (a) Homocysteinemia; (b) Antiphospholipid syndrome


(Ref: Harrison 18th/462, 9/e p123)
31. Ans. (a) Venous thrombosis; (b) Thrombosis; (d) Embolism
(Ref: Anderson’s10th/2707)
Hemorrhagic infarction is seen in:
• Hypercoagulable states (OCP use, pregnancy, polycythemia
vera, malignancy, etc.)
• Embolism - Infarct is attributed to lysis of clot exposing infarct
tissue and its permeable capillary bed to recirculating blood.
• Venous thrombosis.
32. Ans. (a) Cryoglobulinemia; (b) Multiple myeloma; (d)
Lymphoma; (e) Macroglobulinemia
(Ref: William’s Hematology 6/1268)
Hyperviscosity is seen in
• Multiple myeloma
• Lymphoplasmacytic lymphoma (Waldenstrom’s
macroglobulinemia)
• Cryoglobulinemia
• Myeloproliferative disorders
MGUS (Monoclonal Gammopathy of uncertain significance): Here, M
Protein can be identified in the serum of 1% of healthy individual >50
years of age and 3% in older than 70 yrs. It is the most common form
of monoclonal gammopathy. In MGUS less than 3g/dL of monoclonal
protein is present in serum and there is no Bence Jones proteinuria.
Normal viscosity of blood is 1.8

33. Ans. All.


(Ref: Harrison 16th- 686, Robbin’s 7th/132)
Dysfibrinogenemia produces hypercoagulable state and leads to thrombus
formation in some patients. Other causes have already been
discussed.
34. Ans. (a) Protein C deficiency; (b) Protein S deficiency; (c) Leiden
factor mutation
(Ref: Harrison 16th/685; de Gruchy’s 5th/420, Robbins 9/e p123)
Congenital coagulation disorders
• Hemophilia A and B • Factor VII deficiency.
• von Willebrand’s disease • Factor X (Stuart) deficiency.
• Fibrinogen absence or deficiency. • Factor XII, XI deficiency.
• Prothrombin absence or • Factor XIII deficiency.
deficiency. • Fitzgerald factor (HMWK)
• Factor V deficiency. deficiency.

35. Ans. (c) Embolic obstruction of pulmonary vessels almost


always cause pulmonary infarction
(Ref: Robbins 8th/126)
As discussed is text, most pulmonary emboli (60% to 80%) are clinically
silent because they are small. With time, they undergo organization
and are incorporated into the vascular wall. Embolic obstruction of
medium-sized arteries may result in pulmonary hemorrhage but
usually does not cause pulmonary infarction because of the dual
blood flow into the area from the bronchial circulation.
36. Ans. (c) Homocysteinemia
(Ref: Robbins 7th/131, 8th/122, Harrison 18th/462, 9/e p123)
HyperhomocystenemiaQ is a mixed disorder (inherited as well as acquiredQ)
which can cause both venous and arterial thrombosisQ.

37. Ans. (b) Chronic venous congestion of lung


(Ref: Robbins 8th/535, 7th/122, 9/e p116)
38. Ans. (b) Gangrene
(Ref: Robbins 9/e p43, 129)
39. Ans. (a) Thrombus
(Ref: Robbins 9/e p125)
40. Ans. (a) Postmortem clot
(Ref: Robbins 9/e p125)
41. Ans. (d) Coralline thrombus
(Ref: Robbins 9/e p125)
Thrombi often have grossly and microscopically apparent laminations
called lines of Zahn; these represent pale platelet and fibrin deposits
alternating with darker red cell-rich layers. Such laminations signify
that a thrombus has formed in flowing blood; their presence can
therefore distinguish antemortem thrombosis from the bland non
laminated clots occurring in postmortem clots.
In veins thrombi form coral-like system with framework of platelets, fibrin
and trapped white blood cells; this is a coralline thrombus.
42. Ans. (a) Liver…..see explanation of earlier question….
(Ref: Robbins 8/e p128, 7/e p138)
43. Ans. (c) Heart
(Ref: Robbins 8/e p128, 9/e p129-130)
44. Ans. (a) Fat embolism
(Ref: Robbins 9th/128)
Presence of empty cells in the slide is consistent with the presence of fat
in the slide. A patient who has met with a road traffic accident with fat
cells in the lung histopathology is highly suggestive of fat embolism.
45. Ans (b) Mis-sense mutation
(Ref: Robbins 9/e p124)
Factor V Leiden mutation is a mis-sense mutation resulting in
glutamine to arginine substitution at amino acid residue 506 that
renders factor V resistant to cleavage and inactivation by protein C.
46. Ans (a) Heart
(Ref: Robbins 9/e p125)
Lines of Zahn are seen in thrombi and used for differentiating it
from postmortem clots. Thrombi occurring in heart chambers or in
the aortic lumen are designated mural thrombi. So, the best answer
in the given question.
47. Ans. (c) Decreased prothrombin time
(Ref: Robbins 8th/674, 9/e p134, 664-665)
48. Ans. (b) Endothelial injury (Ref: Robbins 9/e p131-132)
The principle mechanisms for septic shock include:
• Peripheral vasodilation and pooling of blood
• Endothelial activation/injury
• Leukocyte-induced damage
• Disseminated intravascular coagulation
• Activation of cytokine cascades
If the question talks about the initiating mechanism, it should be preferably
answered as cytokin release. If this option is not given then
endothelial injury is the next best answer.
1. Thrombosis
2. Increase in vascular permeability
3. Vasodilation
The other three options are following this primary event of endothelial
injury due to cytokine release.
49. Ans. (b) Hepatitis B virus
(Ref: Robbins 9th/832)
Hepatitis B virus contains the pol gene. This exhibits both DNA
polymerase activity and reverse transcriptase activity. Replication of
the viral genome occurs via an intermediate RNA template, through a
unique replication cycle: DNA → RNA → DNA.
50. Ans. (d) Cytokines release
(Ref: Robbins 9/e p132-133)
51. Ans. (a) Pulmonary embolism
(Ref: Robbins 9/e p127)
D-dimer is a fibrin degradation product, a small protein fragment
present in the blood after a blood clot is degraded by fibrinolysis. It is
so named because it contains two crosslinked D fragments of the
fibrinogen protein. D-dimer concentration may be determined by a
blood test to help diagnose thrombosis. D-dimer testing is of clinical
use when there is a suspicion of deep venous thrombosis (DVT) or
pulmonary embolism (PE). In patients suspected of disseminated
intravascular coagulation (DIC), D-dimers may aid in the diagnosis.
52. Ans. (d) Diffuse alveolar damage
(Ref: Harrison 17th/1680-1681; Robbin’s 7th/715, 9/e p134)
Shock lung is also known as acute respiratory distress syndrome, diffuse
alveolar damage, acute alveolar injury and acute lung injury.

53. Ans. (a) ATN; (b) Pulmonary congestion; (c) Depletion of lipid in
adrenal cortex and (d) Hepatic necrosis
(Ref: Robbins 7th/141, 142, 9/e p134)
Shock is characterized by failure of multiple organ systems due to
systemic hypoperfusion caused by reduction either in cardiac output
or in effective circulating blood volume.
Liver → Fatty changes with hemorrhagic central necrosis.
Kidneys → Extensive tubular ischemic injury (Acute tubular necrosis).
Lungs → Pulmonary congestion with diffuse alveolar damage.
Adrenal → Cortical cell lipid depletion.
Brain → Ischemic encephalopathy.
Heart → Coagulation necrosis or contraction band necrosis.
GIT → Hemorrhagic enteropathy.

54. Ans. (a) Abruptio placentae; (b) Acute promyelocytic leukemia;


(c) Severe falciparum malaria (d) Snake envenomation; (e)
Heparin overdose.
(Ref: Robbins 7th/657, KDT 5th/562, 9/e p664-665)
55. Ans. (b) Reduced plasma fibrinogen level
(Ref: Robbins 7th/656-658, 9/e p664-665)
1. Which of the following factors play a major role in initiation of
thrombus formation?
(NEET 2020 like pattern)
(a) Vasoconstriction
(b) Coagulation cascade activation
(c) Platelets activation
(d) Endothelial injury
Ans. (d) Endothelial injury
(Ref: Robbins 9th/122)
The primary abnormalities that lead to thrombosis (Virchow triad) are:
• Endothelial injury,
• Stasis or turbulent blood flow, and
• Hypercoagulability of the blood.

2. Factor V mutation is commonly associated with:


(AI 2018 Pattern)
(a) Hemarthrosis
(b) Hematemesis
(c) Thrombosis
(d) Epistaxis
Ans. (c) Thrombosis
(Ref: Robbins 9/e p123-124)
Disclaimer
Any resemblance to an actual question is purely coincidental.
• Approximate number of genes in human genome is 30,000.
• Coding DNA constitutes only 2% of the entire genome.
• Watson and Crick are associated with Double helical DNA model.
• Fuelgen reaction is for DNA.
• Multifactorial inheritance is known for: cleft lip, hypertension, gout, Cardiac septal
defects, diabetes mellitus and coronary artery disease.
• Inheritance of ABO blood group and HLA antigens is Codominance
• Autosomal dominant disorders are characterized by expression in heterozygous state;
they affect males and females equally, and both sexes can transmit the disorder. It
affects receptors and structural proteins.
• Autosomal recessive diseases occur when both copies of a gene are mutated;
enzyme proteins are frequently involved. Males and females are affected equally.
• X-linked disorders are transmitted by heterozygous females to their sons, who
manifest the disease.
• DiGeorge syndrome (thymic hypoplasia with diminished T-cell immunity and
parathyroid hypoplasia with hypocalcemia) and Velocardiofacial syndrome (congenital
heart disease involving outflow tracts, facial dysmorphism, and developmental delay)
• Down syndrome is due to: Trisomy 21; it is most commonly caused by : Maternal non-
disjunction in meiosis-I .
• Patau syndrome karyotype: Trisomy 13 (47 XX, + 13).
• Edward syndrome karyotype: Trisomy 18 (47 XX, + 18).
• Increasing severity of mental retardation of male members over generations is due to
Triple repeat mutations (anticipation or dynamic mutation).
• Barr body is not seen in females in the condition of Turner syndrome.
• Gene regulate normal morphogenesis: Homeobox. This gene mutation is not cause
ventricular septal defect.
• Defective DNA repair syndromes: Bloom syndrome, Fanconi anemia, Ataxia
telangiectasia and Xeroderma pigmentosum.
• Genes involved in RETT syndrome: MECP2 (most common), FOXG1, CDKL5.
• Normal parents transmitting autosomal dominant disorders is usually due to: Germ line
(gonadal) mosaicism. Seen with tuberous sclerosis and osteogenesis imperfecta.
• Mother transmitting the disease to all children: Mitochondrial disorders.
• Genomic Imprinting: The disorders associated with imprinting are also called as the
parent-of-origin gene disorders. Examples include Prader Willi syndrome and
Angelman syndrome.
• In Prader- Willi syndrome: deletion of paternal chromosome 15 occurs with imprinting
of maternal chromosome 15. Patients have mental retardation, short stature, hypotonia,
hyperphagia, small hands and feet, and hypogonadism.
• In Angelman syndrome: deletion of maternal chromosome with imprinting of paternal
chromosome 15. Affected patients have mental retardation, ataxia, seizures, and
inappropriate laughter.
• Inheritance of both chromosomes of a pair from one parent is called uniparental
disomy. Angelman syndrome can also result from uniparental disomy of paternal
chromosome 15.
• Microarray’ is used for study of multiple genes.
• In-situ DNA nick end labelling can quantitate the fraction of cells in apoptotic pathway.
• Comparative genomic hybridization is for comparing cancer cells and normal cells.

Genetics is the study of the genes. Genes are a part of chromosome and they code for a trait or
character. The position of gene on a chromosome is called as a locus. Out of a total number of 46
chromosome, 22 pairs of chromosomes are homologous and are called autosomes. The 23rd pair
is alike only in the females (have 2 similar X chromosomes) whereas in a male there is one X
chromosome and one Y chromosome. The X and Y are therefore referred to as sex
chromosomes.

The genetic makeup of an individual is called genotype whereas the manifested physical feature is called as
phenotype.

The Gene are made up of nucleic acids like ribonucleic acid; RNA or deoxyribonucleic acid;
DNA. RNA is present only in the nucleus whereas DNA is present in both the nucleus and
mitochondria of a cell. These nucleic acids are made up of nucleotides whose composition
includes nitrogenous base, a sugar (deoxyribose in DNA and ribose in RNA) and a phosphate
group. The nitrogenous base can be either a purine (adenine, guanine) or pyrimidine (thymine,
cytosine, uracil). The purines bind with the pyrimidines complementarily.

Alternate form of gene coding for different forms of a character is called allele. A normal gene has 2 alleles.
When these code for same trait, it is known as homozygous state whereas if the alleles code for different traits,
it is called heterozygous state.

If an allele manifests itself in a heterozygous state, it is called as dominant. The alternate


allele which is unable to manifest itself in the heterozygous state is called as a recessive allele.
(So, ‘A’ is Dominant and ‘a’ is Recessive)

SINGLE GENE DISORDERS WITHMENDELIAN/CLASSICAL INHERITANCE

a. Autosomal Dominant (AD) Inheritance Diseases


– Mutated genes can express themselves in heterozygous state.
– Usually cause defect in the synthesis of structural Q or non-enzyme proteins.
– These have variable onset (so, onset may be into adulthood).
– These are characterized by reduced penetranceQ (individuals inherit the gene but can
be phenotypically normal) and variable expressibility Q (the trait is seen in the
individuals carrying the mutant gene but is expressed differently among individuals, e.g.
patients of neurofibromatosis have variant from brownish skin spots to multiple skin
tumors in different patients).
Examples of autosomal dominant disorders

Vo Von Willebrand DiseaseQ

Familial Familial Adenomatous PolyposisQ

Hypercholesterolemia Hypercholesterolemia (Familial)

Autosomal Adult polycystic kidneyQ

D Dystrophia myotonicaQ

O Osteogenesis imperfectaQ

M Marfan syndromeQ

I Intermittent porphyriaQ

N Neurofibromatosis-1Q

A AchondroplasiaQ

N Neurofibromatosis – 2Q

T Tuberous sclerosisQ

Hai Huntington’s diseaseQ; Hereditary spherocytosisQ

There are the following types of autosomal dominant gene mutations:


Autosomal dominant disease mutations
Gain of function Loss of function
Protein product of the mutant allele has properties not Leads to the reduced production of a gene product or
normally associated with the natural/wild-type protein give rise to an inactive protein
Less common More common
Affects normal proteins with toxic properties Affects regulatory proteins & subunits of multimeric
proteins
e.g. Huntington’s disease e.g. Osteogenesis imperfecta

Concept
Dominant negative mutant allele is associated with the more common “loss of function” mutation. This type of
mutation leads to not only reduced production of a gene product but the inactive polypeptide interferes with the
functioning of a normall allele in a heterozygote. This usually affects structural proteins. At times, the inactive
protein is a part of multiunit protein complex and it interferes with the normal functioning of other units of the same
complex.
Example: Osteogenesis imperfecta
The collagen molecule is made up of triple helical molecule made up of three collagen chains arranged in a helical
configuration. Each collagen chains in the helix must be normal for the normal assembly and stability of the
collagen molecule. If there is a single mutant collagen chain, normal collagen trimers cannot be formed, and hence
there is a marked deficiency of collagen.

SOME IMPORTANT AUTOSOMAL DOMINANT DISEASES

Marfan Syndrome

It is an autosomal dominant disease having mutation in the fibrillin geneQ on the chromosome
15q21. Fibrillin behaves as a scaffolding protein for the alignment of elastic fibers. So, any defect
affects the following systems:

Skeletal defects CVS changes Ocular changes


• Tall and thinbuiltQ • Mitral valve prolapse causing • Bilateral dislocation or
• Long and slender fingers and hands mitral regurgitaitonQ subluxation of the lens
(Arachnodactyly)Q • Medial degeneration causing (know as Ectopia lentisQ)
• Hyperextensible joints (especially dissecting aortic aneurysm and
thumbQ) aortic regurgitationQ.
• Inward depressed stemum (Pigeon
breast deformityQ)

Neurofibromatosis
Neurofibromatosis
NF-1 (von Recklinghausen DiseaseQ) NF-2 (Bilateral acoustic neurofibromatosisQ)

• More common, seen in 90% patientsQ • Less common, seen in 10% patients
• Presence of neural tumors of neurofibromas in the body • Bilateral acoustic neuromasQ
which can be cutaneous, subcutaneous or plexiform • Multiple meningiomaQ
• 6 or more pigmented skin lesions called ‘cafe au lait’ • Cafe au lait spot are present but Lisch nodules
spots
are absentQ
• Pigmented iris hamartoma called Lisch NodulesQ
• Associated skeletal muscle defects like scoliosis, bone
cysts or tibial pseudoarthrosis
• Risk of development of meningioma,
pheochromocytomas and Wilm’s tumorQ

Note:
• NF1 gene is present on chromosome 17Q and its product called neurofibromin is a tumor suppressor
gene which normally causes decreased activity of p21 ras oncoprotein.
• NF2 gene is present on chromosome 22Q and it normally produces merlin which is a protein causing
contact inhibition of proliferation of Schwann cells.
• So, any mutation in NF1 or NF2 causes increased chances of tumor formation.

Ehlers-Danlos Syndrome (EDS)


It is an inherited tissue disease due to defect in collagen structure or synthesis. The clinical
features include presence of hyperextensible skin (cigarette paper skin Q) which can be easily
injured as it is fragile. There is also presence of hyperextensible joints. There are different variants
of EDS having different modes of inheritance.

EDS type 3 (Hypermobility type) AD inheritance; presence of joint hypermobility; pain and dislocation
EDS type 4 (Vascular type) AD inheritance; defect in collagen type III; presence of thin skin; easy
bruising; arterial and uterine rupture, small joint hyperextensibility

EDS type 6 (Kyphoscoliosis type) AR inheritance, mutation in the enzyme lysyl hydroxylase resulting in
formation of unstable collagen; there is presence of hypotonia, joint laxity,
congenital scoliosis and ocular fragility.

b. Autosomal Recessive (AR) Inheritance Diseases


– Mutant genes express themselves only in Homozygous state.Q
– Usually cause defect in the synthesis of an enzyme protein.Q
– These have an early uniform onsetQ (usually in childhood).
– There is complete penetranceQ (persons having defective gene in homozygous state will
have disease as well).
– “Inborn errors of metabolism”are usually inherited as autosomal recessive disorders.Q
– If one parent is carrier and the other one is normal, autosomal recessive disorders usually
donot manifest but if an affected baby is born to such a couple, Uniparental disomy
(UPD) should be suspected. UPD occurs when a person receives two copies of a
chromosome, or part of a chromosome, from one parent and no copies from the other
parent.

Pedigree in Autosomal Recessive Disorders

Fried Friedrich’s ataxiaQ


Poori aur PhenylketonuriaQ
Garam GalactosemiaQ
C Cystic fibrosisQ
H HemochromatosisQ
A a1 – Antitrypsin deficiencyQ
W Wilson’s diseaseQ
A AlkaptonuriaQ
L Lysosomal and glycogen storage diseasesQ
M Muscular atrophy (both spinal as well as neurogenic)
A Adrenal hyperplasiaQ (congenital)
S Sickle cell diseaseQ
T ThalassemiaQ
Hai HomocystinuriaQ

Some Important Autosomal Recessive Diseases

1. Phenylketonuria (PKU): It is caused by the deficiency of enzyme phenylanine hydroxylase


resulting in inability to convert phenylalanine to tyrosine and resultant hyperphenylalaninemia.
The clinical presentation is a child normal at birth but developing profound mental retardation
by 6 months of age. The absence of tyrosine results in light-colored skin and hair. There is
also presence of a mousy or musty odor to the sweat and urine due to secondary
accumulation of a metabolite called phenylacetate.

The management is done by the dietary restriction of phenylalanine.

2. Alkaptonuria (Ochronosis): It is caused by the deficiency of homogentistic acid oxidase


resulting in the accumulation of homogentistic acid. The latter has an affinity for connective
tissues (especially cartilage), resulting in a black discoloration (ochronosis). The clinical
features include the passage of normal coloured urine (which turn black on exposure to air),
black cartilage, discoloration of the nose and ears and early onset of degenerative arthritis.

3. Glycogen Storage Diseases: These are a group of rare diseases that have in common a
deficiency in an enzyme necessary for the metabolism of glycogen, which results in the
accumulation of glycogen in the liver, heart, and skeletal muscle. Some salient types include:
Type Name of disorder Enzyme deficiency
Type I Von Gierke’s disease Glucose-6-phosphatase
Type II Pompe’s disease Acid maltase
Type III Cori’s disease Debranching enzyme
Type IV Anderson’s disease Branching enzyme
Type V McArdle’s disease Muscle phosphorylase
Type VI Her’s disease Hepatic phosphorylase
Type VII Tarui’s disease Phosphofructokinase 1 (PFK-1)
4. Lysosomal Storage Diseases
Disease Enzyme deficiency Accumulating substance
Tay-Sachs disease Hexosaminidase A GM2 ganglioside
Niemann-Pick disease Sphingomyelinase Sphingomyelin
Gaucher disease Glucocerebrosidase Glucocerebroside
Fabry disease a-Galactosidase A Ceramide trihexoside
Metachromatic leukodystrophy Aryl sulfatase A Sulfatide

Hurler syndrome a-1-Iduronidase Dermatan sulfate


Heparan sulfate
Hunter syndrome L-Iduronosulfate sulfatase Dermatan sulfate
Heparan sulfate

Disease Deficiency
Tarun – Tay Sachs Has – Hexosaminidase
Nine – Neimann Pick Shirts – Sphingomyelinase
Most – Metachromatic Are Saffron – Aryl Sulfatase
leukodystrophy
Few – Fabry Are Green – Alpha Galactosidase

a. Tay-Sachs Disease: It is caused by the deficiency of the enzyme hexosaminidase A


leading to accumulation of GM2 ganglioside in the lysosomes of the CNS and retina. It is
common in Ashkenazi Jews. Cherry red spot is seen in the retina whereas dilated
neurons with cytoplasmic vacuoles are seen in the CNS. The clinical presentation
includes normal child at birth with onset of symptoms by 6 months. It is associated with
progressive mental deterioration and motor incoordination and death by the age of 2-3
years. Electron microscopy shows the presence of distended lysosomes with whorled
membranes.

b. Niemann-Pick Disease: It is caused by the deficiency of the enzyme sphingomyelinase


leading to the accumulation of sphingomyelin within the lysosomes of the CNS and
reticuloendothelial system. It is also commoner in Ashkenazi Jews. There is presence of
a retinal cherry-red spot and CNS having distended neurons with a foamy cytoplasmic
vacuolization. The clinical presentation includes normal child at birth with onset of
symptoms by 6 months. It is associated with progressive massive splenomegaly,
lymphadenopathy, mental deterioration and motor manifestations resulting in death by the
age of 2 years.

c. Gaucher Disease: It is the most common lysosomal storage disorder caused by the
deficiency of glucocerebrosidase leading to the accumulation of glucocerebroside
predominantly in the lysosomes of the reticuloendothelial system. It is characterized by
the presence of hepatosplenomegaly, hypersplenism leading to
thrombocytopenia/pancytopenia, lymphadenopathy and bone marrow involvement
leading to bone pain, deformities, and fractures. A subgroup of patients may also have
CNS manifestations.

d. Mucopolysaccharidosis (MPS): These are a group of lysosomal storage disorders


characterized by deficiencies of the lysosomal enzymes required for the degradation of
mucopolysaccharides (glycosaminoglycans). The clinical features of the patients include
mental retardation, cloudy cornea, hepatosplenomegaly, skeletal deformities and coarse
facial features, joint abnormalities and cardiac lesions.

X-LINKED RECESSIVE DISORDER

Males have an X and a Y chromosome. There is no corresponding locus for a mutant allele of the
X chromosome on the Y chromosome. The mutant recessive gene on the X chromosome
expresses itself in a male child because it is not suppressed by a normal allele whereas in the
female, the presence of a normal allele on other X-chromosome prevents the expression of the
disease. So, females only act as carriers.Q
Examples of X-linked recessive disorders
Less Lesch-Nyhan syndromeQ
H Hemophilia A and BQ; Hunter syndromeQ
C Chronic granulomatous diseaseQ
G is G6PD deficiencyQ
Detected Duchhene muscular dystrophyQ, Diabetes insipidusQ
Clinically in Color blindnessQ
A AgammaglobulinemiaQ (Bruton’s disease)
Fragile Fragile X syndromeQ, Fabry DiseaseQ
Woman Wiskott Aldrich syndromeQ

X-LINKED DOMINANT DISORDERS

These are the conditions in which both heterozygous males and females are affected. All the sons
of the affected male are normal and all the daughters are affected. The affected female transmits
the disease to half of the sons and daughters.
Examples: Hypophosphatemic type of vitamin D resistant rickets; Incontinentia pigmenti,
Alport Syndrome and oro-facio-digital syndrome.
SINGLE GENE DISORDERS WITH NON-MENDELIAN INHERITANCE

Non-Mendelian Inheritance can be classified into the following categories:


A. Mitochondrial inheritance
B. Genomic Imprinting
C. Triple Repeat Mutations
D. Germline Mosaicism

A. MITOCHONDRIAL INHERITANCE
Mutation in the mitochondrial DNA has the characteristic feature of maternal inheritanceQ
because the ovum contains the mitochondria with their abundant cytoplasm whereas sperms
contains minimal number of mitochondria. The fertilized oocyte degrades mtDNA carried from the
sperm in a complex process involving the ubiquitin proteasome system. So, while mothers
transmit their mtDNA to both their sons and daughters, only the daughters are able to transmit the
inherited mtDNA to future generations.
• d denotes diseased whereas N denotes normal individuals.

Salient features of these diseases


• The organs most commonly affected in these diseases are the ones having large number of
mitochondria inside them. Such organs include CNS, skeletal muscle, cardiac muscle, liver
and kidneys.
• Tissues may have both normal/wild and mutant mitochondrial DNA.
• Sons do not transmit the disease to progeny Q

B. GENOMIC IMPRINTING

A person gets two alleles for a character; one from mother and second from father. Normally these
two alleles are similar. But, in some cases these alleles are differentially expressed. i.e. either
maternal gene become silent (only paternal gene express) or paternal gene become silent (only
maternal express). In such a condition, if the chromosome containing the gene which is expressed
undergoes deletion, there will be disease, whereas if homologous undergoes deletion, nothing will
happen.

Example of genomic imprinting disorder is microdeletion of chromosome 15q11-13. If


microdeletion occurs in maternal chromosome; Angelman syndrome results whereas paternal
chromosomal microdeletion may cause Prader-Willi syndrome.
It is also seen in Beckwith-Wiedmann syndrome and Albright’s hereditary
osteodystrophy.

• Molecular studies of cytogenetically normal patients with Prader Willi syndrome reveal that they have two
maternal copies of chromosome 15.
• Inheritance of both chromosomes of a pair from one parent is called uniparental disomy. So, Prader Willi
Syndrome may be due to UPD of maternal chromosome 15.
• Similarly Angelman syndrome patients might have uniparental disomy of paternal chromosome 15.

C. TRIPLE REPEAT MUTATIONS


The mutation in this disease group is characterized by a long repeating sequence of three
nucleotides. It is characteristically different from other types of mutations because it is dynamic in
nature. Dynamicity means that the degree of amplification of a sequence of three nucleotides
increases during gametogenesis.
Trinucleotide repeat disorders

Expansions in non-coding regions Expansions in coding regions


“Loss of function” type mutation “Gain of function” type mutation
Mutant proteins aggregate as intranuclear Mutant proteins interfere with other proteins
inclusions
Examples: Examples:
• Fragile X syndromeQ • Huntington’s diseaseQ
• Friedrich’s ataxiaQ • Spinobulbar muscular atrophyQ (Kennedy’s disease)
• Myotonic dystrophyQ • Spinocerebellar ataxia types 1, 2, 3, 6, 7Q

The trinucleotide repeat expansions in the non-coding regions involve different repeats as Fragile X syndrome
(CGGQ), Friedrich’s ataxia (GAAQ) and Myotonic dystrophy (CTGQ).
Fragile X Syndrome

There is presence of triplet repeat mutations of CGG nucleotides. The mutation affects the FMR-1
gene (Familial Mental Retardation – 1 gene) present on the X chromosome. On karyotyping, the
chromosome appears as broken (so, called fragile site). It is the second most common cause of
mental retardation (Down syndrome is the commonest cause). The clinical features of patient
include long face with a large mandible, large everted ears and large testicles (macro-orchidism).Q

In the normal people, the number of CGG repeats is from 10 to 55. There is amplification of
CGG repeat in carrier females to 55-200 CGG repeats which is called premutation. In diseased
individuals, the CGG repeats range from 200-4000 repeats called full mutations. During the
process of oogenesis (Not spermatogenesis), amplification causes conversion of premutations to
full mutations. This is responsible for Sherman’s Paradox Q (the risk of mental retardation is much
higher in grandsons than the brothers of transmitting males as the grandsons acquire a
premutation from their grandfather which gets amplified to a mutation in their mother ova).

• Southern blot is useful for genetic counseling (it can differentiate between premutation and
mutation prenatally and postnatally).

Huntington’s Chorea
There is presence of CAG repeats associated with chromosome 4 that are responsible for the
production of an abnormal neurotoxic protein called HuntingtonQ. It is associated with caudate
nucleus atrophy. Clinical features include early onset of progressive dementia and presence of
choreiform movements (due to inhibition of GABAergic neurons).
D. GONADAL/GERMLINE MOSAICISM
Normally autosomal dominant disorders have affected parents but in some patients with
autosomal dominant disorders, the parents are not affected. In such patients, the disorder results
from a new mutation in the egg or the sperm from which they were derived; as such, their siblings
are neither affected nor at increased risk of developing the disease.
However, in certain autosomal dominant disorders, exemplified by osteogenesis imperfectaQ and tuberous
sclerosisQ, phenotypically normal parents have more than one affected child. This may appear to clearly violate the
laws of Mendelian inheritance but is explained by gonadal mosaicism.

Gonadal mosaicism results from a mutation that occurs postzygotically during early
(embryonic) development. If the mutation affects only cells destined to form the gonads, the
gametes carry the mutation, but the somatic cells of the individual are completely normal. Such an
individual is said to exhibit germ line or gonadal mosaicism. A phenotypically normal parent
who has germ line mosaicism can transmit the disease-causing mutation to the offspring through
the mutant gamete. Since the progenitor cells of the gametes carry the mutation, there is a definite
possibility that more than one child of such a parent would be affected. Gonadal mosaicism
should not be confused with mosaicism (explained below).
MOSAICISM

CHROMOSOMAL DISORDERS

Study of chromosomes is called karyotyping. It is done in cells like skin fibroblasts, peripheral
blood lymphocytes and amniotic cells. The normal number of chromosomes in a somatic cell is
diploid and is expressed as 46, XX or 46, XY.
• Mitosis is arrested in dividing cells in metaphase stage by use of colchicine. In this stage,
individual chromosomes take the form of two chromatids connected at the centromere. The
Short arm of chromosome is called “p“ (petite) and long arm is reffered to as “q“.
Banding technique and selected features

Q banding G banding R (Reverse) banding C banding

Dye used Quinacrine mustard Trypsin Alkaline solution followed by Chemical followed by
followed by Giemsa Giemsa
Giemsa

Microscope Fluorescence Light Light microscopy Light microscopy


used microscopy microscopy

Special *Temporary *Permanent *Used for analyzing *Used for studying


features So, not usedQ for *MC rearrangements involving the chromosomal
routine cytogenetic techniqueQ terminal endsQ of translocations involving
analysis for routine chromosomes centromeric regionsQ
cytogenetic *Gives pattern opposite to G
analysis banding

Appearance
of
chromosomes

Note: Q, G and R banding produce bands along entire length of chromosomes whereas for specific chromosomal
structures, other types of banding may be used. Some of these include T- banding (for Telomeres), C banding (for
Constitutive heterochromatin) and NOR-banding (for nucleolus-organizing regions).

Sometimes, fluorodeoxyuridine (FUdR)Q banding is also done. It is a direct inhibitor of thymidylate synthetase and it
can induce folate-sensitive fragile sites in chromosomes. Chromosomal fragile sites can induce mental retardation as
is seen in fragile X syndromeQ.

TYPES OF CHROMOSOMES

Metacentric Centromere is in the center of the chromosome (the


chromosome has two equal arms).

Submetacentric Centromere is away from the center so that the arms


are unequal in size (one arm shorter than the other).
.

Acrocentric Centromere is almost at the tip (one end) of the


chromosome (one arm is much longer than the
other).

Telocentric Centromere is at the extreme end of the replicating


chromosome
(chromosome has only one arm). Not seen in
humansQ.
CYTOGENETIC ABNORMALITIES

Deletion
(Loss of genetic material)

Balanced translocation

Robertsonian translocation – Translocation between the


2 acrocenteric chromosomes with breakpoint occurring
close to the centromeres. So, very large and very small
chromosomes are obtained. The small chromosomes
are usually lost.

Inversion

• The gametes contain half the number of chromosomes (haploid) and are represented as (23,
X) or (23, Y).

Isochromosome
Isochromosome formation results when one arm of a chromosome is lost and the remaining arm
is duplicated, resulting in a chromosome consisting of two short arms only or of two long arms. It
has morphologically identical genetic information in both arms.
• The reason for the formation of an isochrome is the centromere misdivision. Instead of dividing
longitudinally to separate the two sister chromatids, the centromere undergoes a transverse split that
separated the two arms from one another.

Formation of isochromosome
Contd...
Contd...

• An exact multiple of haploid chromosomes is called Euploidy (2n, 3n, 4n … etc).


• When exact multiple of haploid chromosomes is not present, it is called Aneuploidy.
Genome mutations involve loss or gain of whole chromosomes, giving rise to monosomy or
trisomy.

DOWN SYNDROME (TRISOMY 21)Q


• It is the most common of the chromosomal disordersQ and a major cause of mental
retardation.Q

• Genetics of Down syndrome.


Meiotic nondisjunction of chromosome 21 occuring in the ovum
• Seen in 95% cases with trisomy 21. So, it is the commonest cause of Down syndrome
• The extra chromosome is of maternal origin.
• Strong relation with maternal age
Robertsonian translocation
• Seen in about 4% of cases of Down syndrome.
• The extra chromosomal material derives from the presence of a robertsonian translocation of the long arm of
chromosome 21 to another acrocentric chromosome (e.g., 22 or 14).
• Most cases are frequently (but not always) familial.
• No relation with maternal age
Mosaicism
• Seen in ~1% of Down syndrome patients
• Results from mitotic nondisjunction of chromosome 21 during an early stage of embryogenesis.
• Patients have a mixture of cells with 46 and 47 chromosomes (mosaicism).
• No relation with maternal age.
• Important signs of the disease include:
– Flat facial profile
– Mental retardation
– Microgenia (abnormally small chin)
– Oblique palpebral fissures with epicanthic skin folds (mongoloid slant)
– Muscle hypotonia (poor muscle tone)
– Flat nasal bridge
– Single palmar fold (Simian crease)
– Curvature of little finger towards other four fingers (Clinodactyly)
– Protruding tongue or macroglossia
– White spots on the iris known as Brushfield spots
– Excessive joint laxity including atlanto-axial instability
– Excessive space between large toe and second toe (Sandle toe)
– A single flexion furrow of the fifth finger
– Higher number of ulnar loop dermatoglyphs
Screening for Down Syndrome
Many standard prenatal screens can discover Down syndrome. Amniocentesis and chorionic
villous sampling (CVS) are considered invasive procedures, in that they involve inserting
instruments into the uterus, and therefore carry a small risk of causing fetal injury or miscarriage.
The risks of miscarriage for CVS and amniocentesis are often quoted as 1% and 0.5%
respectively. There are several common non-invasive screens that can indicate a fetus with Down
syndrome. These are normally performed in the late first trimester or early second trimester. Due
to the nature of screens, each has a significant chance of a false positive, suggesting a fetus with
Down syndrome when, in fact, the fetus does not have this genetic abnormality. Screen positives
must be verified before a Down syndrome diagnosis is made. Common screening procedures for
Down syndrome are given in the table below.
First and second trimester Down syndrome screens
Screen When performed (weeks Detection False positive Description
gestation) rate rate
Triple test 15-20 70% 5% Maternal serum α-feto protein (Low) +
Quad 15–20 81% 5% Estriol (Low) + hCG (High)
screen Triple test + inhibin-Alpha (High)
First 10–13 85% 5% Ultrasound to measure:
Trimester *Nuchal Translucency (Increased)
Combined *Ductus venosus flow (reversed)
Test *Nasal bone (hypoplasia)
+ hCG + Pregnancy associated plasma
protein A; PAPPA (Low)
Integrated 10-13 and 15–20 95% 5% Measurements from both the 1st
Test Trimester Combined test and the 2nd
trimester Quad test to yield a more
accurate screening result.
Common features of both are mental retardation, rocker bottom feet and congenital heart
defects (VSD and PDA).

TRISOMY 22
Cat Eye Syndrome is a rare condition caused by the partial trisomy of chromosome 22 (The
short arm (p) and a small section of the long arm (q) of Chromosome 22 is present three instead
of the usual two times. The term “Cat Eye” syndrome was coined due to the particular appearance
of the vertical colobomas in the eyes of some patients.

Klinefelter Syndrome

• It is the most common chromosomal disorder of males associated with hypogonadism and
infertility.
• It is due to extra-X-chromosome. Classically, it is 47, XXY. Other variants can have 48 XXXY,
rarely 49 XXXY or mosaics can be there with some cells containing normal 46, XY and others
47, XXY.
• Classically, it results from meiotic non-disjunction of sex chromosomes [40% during
spermatogenesis and 60% during oogenesis]. Mostly, non-disjunction occur during 1st
meiotic division.
• Extra X- chromosomes increase the female like features, i.e. feminization [as shown by
atrophic testes, lack of secondary sexual characteristics, gynecomastia]. Further, Extra
inactive X-chromosome appear as Barr body
• Presence of single Y-chromosome is enough for male phenotype. Thus XY, XXY, XXXY all are
males.

Clinical Features of Klinefelter Syndrome

• Male sex
• Hypogonadism
• Loss of secondary sexual characteristics
• Subnormal IQ
• Disproportionately long arms and legs
• Gynecomastia
• There is increased risk of breast carcinoma, germ cell tumors (like embryonal cell carcinoma,
teratoma and mediastinal germ cell tumors) and autoimmune diseases like SLE.
• Patients can develop cardiovascular problems. Most commonly associated is mitral valve
prolapse followed by varicose veins.
• Due to less testosterone (hypogonadism), feedback inhibition is less and pituitary produces
more LH and FSH.

Turner Syndrome

• Most common cause of sex chromosomal abnormality in the females.Q


• Usually results from complete or partial monosomy of X chromosome and associated with
hypogonadism in phenotypic females.Q Patients have the (45X) karyotype or may be mosaics.

• Clinical features in infancy include edema of dorsum of hands and feet,Q neck webbing or edema of
nape of neck (also produces cystic hygromaQ) and congenital cardiac defect (particularly preductal
coarctation of the aortaQ and bicuspid aortic valveQ).
• Clinical features in adolescence and adulthood include short stature, low posterior hairline, webbing of
neck, cubitus valgus (increased carrying angle),Q streak ovariesQ (contributing to infertility and
amenorrhea), coarctation of the aorta, broad chest and widely spaced nipples, short 4th metacarpal.
Noonan Syndrome

It is a relatively common autosomal dominant (chromosome 12 defect) congenital disorder


considered to be a type of dwarfism, that affects both males and females equally. It used to be
referred to as the male version of Turner’s syndrome; however, the genetic causes of Noonan
syndrome and Turner syndrome are distinct. Genetics of Turner syndrome shows monosomy of X
chromosome (XO) whereas in Noonan syndrome, mostly mutation in genes on chromosome
12 is noted. The principal features include congenital heart defect, short stature, learning
problems, pectus excavatum, impaired blood clotting, and a characteristic configuration of facial
features which is quite similar to Turner syndrome.

LYON’S HYPOTHESIS

• Only one of the X chromosome is genetically active.


• Other X of the paternal or maternal origin undergoes pyknosis and is rendered inactive.
• Inactivation of either maternal or paternal X occurs at random among all the cells of the
blastocyst by about 16th day of embryonic life.Q
• Inactivation of the same X chromosome persists in all the cells derived from each precursor
cell. Inactivation of X is because of a gene called Xist which is causing gene silencing DNA
methylation. So, all normal females are actually mosaics.
1. Polymerase chain reaction (PCR)
PCR analysis involves the synthesis of relatively short DNA fragments from a DNA template.
It can be of the following subtypes:
Sanger sequencing Pyrosequencing Single-base primer extension
*Analysis of large genes or *More sensitive than *Useful approach for
multiple genes Sanger sequencing identifying mutations at a specific
*Gold standard” for sequence *Detection of as little as nucleotide position
determination 5% mutated alleles in a background of
normal alleles.
*Used to analyze DNA obtained from
cancer biopsies, in which tumor cells are
often “contaminated” with large numbers
of admixed stromal cells.

Restriction fragment length


analysis Amplicon length analysis Real-time PCR
*Useful for molecular diagnosis *Detection of mutations that affect the *Can detect and quantify the
when the causal mutation length presence of particular nucleic acid
always occurs at an invariant of DNA (e.g., deletions or expansions) sequences in “real time” (i.e.,
nucleotide during the exponential phase of DNA
position amplification
* Also be used to detect somatic
point mutations in oncogenes such
as KRAS and BRAF

Real-time PCR performed on cDNA


is the method of choice for
monitoring residual disease in
patients with CML

Also know
2. Fluorescence in Situ Hybridization (FISH)
• A technique used when we want to recognize sequences specific to particular
chromosomal regions.
• These DNA clones are labeled with fluorescent dyes and labels a specific chromosomal
region that can be visualized under a fluorescent microscope.

Fig. 1: FISH

Advantages of FISH
a. Does not require dividing cells especially when a rapid diagnosis is warranted (e.g.,
deciding to treat a patient with acute myeloid leukemia with retinoic acid).
b. Can be performed on prenatal samples, peripheral blood cells, touch preparations from
cancer biopsies, and even fixed archival tissue sections.
c. FISH is used to detect aneuploidy; subtle microdeletions or complex translocations that
are not demonstrable by routine karyotyping; and gene amplification (e.g., HER2 in breast
cancer or NMYC amplification in neuroblastomas).
Variants of FISH

a. Chromosome painting: Extension of FISH whereby probes are prepared that span
entire chromosomes. It can detect limited number of chromosomes simultaneously.
b. Spectral karyotyping (also called multicolor FISH): Use of different fluorochromes
permitting visualization of entire human genomeQ. Since it is so powerful, it is also
called as “spectacular karyotyping.”

3. Multiplex Ligation-Dependent Probe Amplification (MLPA)


MLPA blends DNA hybridization, DNA ligation, and PCR amplification to detect deletions and
duplications of any size, including anomalies that are too large to be detected by PCR and too
small to be identified by FISH.
4. Southern Blotting
Changes in the structure of specific loci can be detected by Southern blotting. It is useful in
the detection of certain large-trinucleotide-expansion diseases, including the fragile X
syndrome.
5. Cytogenomic Array Technology
Genomic abnormalities can also be detected without prior knowledge by using microarray
technology to perform a global genomic survey. The techniques include:
I. Comparative genomic hybridization (CGH)
It is a method that can be used only when DNA is available from a specimen of interest.
The entire DNA specimen from the sample of interest is labeled in one color (e.g., red), and
the normal control DNA specimen is indicated by another color (e.g., green). These are mixed
in equal amounts and hybridized. The red-to-green ratio is analyzed by a computer program
with the following interpretation:
a. Red color: gain of genetic material
b. Green color: loss of genetic matrial
c. Yellow color: due to equal mixture of green and red color indicating equal amount of
control and test samples. It is currently used for detection of cancer, mutations in
mental retardation and the detection of microdeletions.

Fig. 2: Comparative genomic hybridization (CGH)


Disadvantage of CGH
• Cannot detect balanced chromosomal abnormalities such as reciprocal translocations, inversions or ring
chromosomes do not affect copy number.

II. SNP genotyping approaches:


SNP arrays are routinely used to uncover copy number abnormalities in pediatric patients
when the karyotype is normal but a structural chromosomal abnormality is still suspected. It is
the mainstay of genome wide association studies (GWAS). It serves as both a physical
landmark within the genome and as a genetic marker whose transmission can be followed
from parent to child:

SUMMARY OF CYTOGENTIC TECHNIQUES

• PCR analysis: it can detect relatively short DNA fragments from a DNA template.
• FISH: use of DNA probes that recognize sequences specific to particular chromosomal regions
• Spectral karyotyping (also called multicolor FISH): Use of different fluorochromes permitting visualization
of entire human genomeQ.
• Multiplex Ligation-Dependent Probe Amplification (MLPA): to detect deletions and duplications of any
size, including anomalies that are too large to be detected by PCR and too small to be identified by FISH.
• Comparative genomic hybridization (CGH): to detect unbalanced chromosomes and to differentiate
normal and cancer cells.
• SNP genotyping approaches: is the mainstay of genome wide association studies

Epigenetic Alterations
It is defined as the study of heritable chemical modification of DNA or chromatin that does not
alter the DNA sequence itself but alters its expression. Examples include:
a. Methylation of DNA
b. Methylation of histones
c. Acetylation of histones

DNA methylation can be detected with the treatment of genomic DNA with sodium bisulfite

Epigenetic modifications are critical for normal human development including the regulation of
tissue-specific gene expression, X chromosome inactivation, genomic imprinting, aging and
cancer. Some disease states having this alteration include fragile X syndrome and Prader-Willi
and Angelman syndromes.
Polymorphic Markers and Molecular Diagnosis
Linkage is the tendency for genes and other genetic markers to be inherited together because
of their location near one another on the same chromosome. It is used particularly when the
specific gene is not known or if a polygenic condition is being analysed. The two types of
genetic polymorphisms most useful for linkage analysis are SNPs and repeat-length
polymorphisms known as minisatellite and microsatellite repeats.
Single nucleotide polymorphism (SNPs) Repeat-length polymorphisms
• Most common type of DNA polymorphism, • Subdivided microsatellite repeats and minisatellite repeats.
occurring every 1000 nucleotides throughout • Microsatellites are usually < 1 kilobase and are characterized
the genome by a repeat size of 2 to 6 base pairs.
• Transmission can be followed from parent • Minisatellite repeats are larger (1 to 3 kilobases), and the
to child repeat motif is usually 15 to 70 base pairs.
Significance of microsatellite markers
• Are scattered throughout the human genome and have such a high level of polymorphism,
they are ideal for differentiating between two individuals and to follow transmission of the
marker from parent to child.
• Validated panels of microsatellite marker PCR assays have been routinely used for
determination of relatedness and identity in transplantation, cancer genetics, paternity
testing, and forensic medicine.
Polymorphisms and Genome-Wide Analyses
• In genome wide association studies (GWAS), large cohorts of patients with and without a
disease (rather than families) are examined across the entire genome for common genetic
variants or polymorphisms that are overrepresented in patients with the disease.
• It identifies regions of the genome that contain a variant gene or genes that confer disease
susceptibility.
• Important for diseases like type 2 diabetes, hypertension.
Next-generation sequencing (NGS)
• Next-generation sequencing describes several newer DNA sequencing technologies that are
capable of producing large amounts of sequence data in a massively parallel manner.
• In contrast to Sanger sequencing which requires a single, simple, homogenous template DNA
(usually either a specific PCR product or prepared plasmid), NGS can use are well suited to
heterogeneous DNA samples. It means any DNA from almost any source can be used.
• Sequence reads from NGS instruments are approximately less than 500 bp.

PEDIGREE ANALYSIS

Symbols Used

Male is represented as a square and female is represented as a circle. Affected individuals are
represented by filling the circle or square by shading.
Normal male Normal female Affected male Affected female

Two parents are joined by horizontal line and progeny is indicated by a vertical line.

Analysis
Step 1: First of all see whether there is mitochondrial inheritance or not. If female is transmitting
the disease to all offsprings (both males and females) and male is not transmitting the disease to
any child, it is mitochondrial inheritance.

Step 2: If mitochondrial inheritance is not present, now see whether the disease is inherited as
dominant or recessive trait. In dominant inheritance, at least one member in all generations will
have disease whereas in recessive inheritance, there will be some generations without disease
also. Means, if offsprings of both unaffected parents carry the disease/character, it is recessive
whereas if both affected parents produce normal offspring, it is dominantly inherited.

Step 3: Now see, whether it is sex-linked or autosomal by looking at the sex-predilection as under
SINGLE GENE DISORDERS WITH CLASSICAL INHERITANCE

1. Which of the following is an autosomal recessive condition?


(AI 2012)
(a) Ataxia telangiectasia
(b) Peutz Jeghers syndrome
(c) Neurofibromatosis
(d) Tuberous sclerosis
2. Which of the following is an autosomal dominant metabolic disorder?
(AI 2004)
(a) Cystic fibrosis
(b) Phenylketonuria
(c) a-1 antitrypsin deficiency
(d) Familial hypercholesterolemia
3. The approximate number of genes contained in the human genome is:
(AIIMS Nov 2002)
(a) 40,000
(b) 30,000
(c) 80,000
(d) 1,00,000
4. True statements about α-l anti-trypsin deficiency is:
(a) Autosomal dominant disease
(PGI June 2003)
(b) Emphysema
(c) Fibrosis of portal tract
(d) Diastase resistant positive hepatocytes
(e) Orcein positive granules
5. Autosomal recessive diseases are:
(PGI June 2003)
(a) Hereditary spherocytosis
(b) Thalassemia
(c) Sickle cell anemia
(d) Cystic fibrosis
(e) Hemophilia A
6. Autosomal dominant disorders are all except:
(a) Hereditary spherocytosis
(PGI Dec 2003)
(b) Thalassemia
(c) Sickle cell anemia
(d) Cystic fibrosis
(e) Hemophilia
7. Which of the following disorders has been shown to be genetically transmitted by
single autosomal dominant genes?
(Delhi PG 2010)
(a) Catatonic schizophrenia
(b) Phenylketonuria
(c) Creutzfeldt-Jakob’s disease
(d) Huntington’s disease
8. Duchenne dystrophy is a:
(Delhi PG-2006)
(a) Autosomal dominant disorder
(b) X-linked dominant disease
(c) Autosomal recessive disease
(d) X-linked recessive disease
9. Catastrophic variant of Ehler Danlos syndrome is:
(a) I
(b) II
(c) III
(d) IV
10. Sickle cell disease is due to:
(UP 2005)
(a) Point mutation
(b) Frame shift mutation
(c) Nucleotide receptor blockage
(d) Non-sequence mutation
11. All are autosomal dominant disorders except:
(a) Albinism
(UP 2007)
(b) Marfan’s syndrome
(c) Familial adenomatous polyposis
(d) Von-Hippel Lindau syndrome
12. In Marfan’s syndrome there is defect in protein:
(a) Collagen
(RJ 2000)
(b) Elastin
(c) Fibrillin
(d) All
13. Neurofibromatosis is:
(RJ 2001)
(a) Autosomal dominant
(b) AR
(c) X-linked recessive
(d) All
14. Blue black pigmentation in alkaptonuria is due to:
(a) Homogentisic acid
(RJ 2002)
(b) Oxalic acid
(c) Glucouronic acid
(d) All
15. A 26-year-old woman presents because of trouble with her vision. Physical
examination reveals a very tall, thin woman with long, thin fingers. Examining her
eyes reveals the lens of her left eye to be in the anterior chamber. Her blood levels of
methionine and cystathionine are within normal levels. Which of the following is the
most likely cause of this patient’s signs and symptoms?
(a) Abnormal copper metabolism
(b) Decreased levels of vitamin D
(c) Decreased lysyl hydroxylation of collagen
(d) Defective synthesis of fibrillin
(e) Defective synthesis of type I collagen
MOST RECENT QUESTIONS

16. Which of the following is the inheritance of Huntington’s chorea?


(a) Autosomal dominant
(b) Autosomal recessive
(c) X-linked
(d) Mitochondrial
17. Hemophilia is associated with:
(a) X chromosome
(b) Y Chromosome
(c) Chromosome 3
(d) Chromosome 16
18. Which one of the following is an autosomal dominant disorder.
(a) Duchenn’s muscular dystrophy
(b) Fragile X syndrome
(c) Fanconi’s anemia
(d) Huntington’s chorea
19. Adult polycystic kidney disease is inherited by:
(a)Autosomal dominant
(b) Autosomal recessive
(c) X-linked
(d) Mitochondrial
20. Neurofibroma is having which of the following inheritance?
(a)Autosomal dominant
(b) Autosomal recessive
(c) X-linked recessive
(d) X-linked dominat
21. Which of the following is not X-linked condition:
(a) Duchenne muscular dystrophy
(b) Emery-Dreifuss muscular dystrophy
(c) Facioscapulohumeral muscular dystrophy
(d) Becker muscular dystrophy
22. Which one is not a feature of cystic fibrosis?
(a) Autososmal recessive disease
(b) Abonormal chloride transport
(c) Affects intestine only
(d) Increased risk of pulmonary infections
23. Which of the following is an X-linked dominant disorder?
(a) Vitamin D resistant rickets
(b) Familial hypercholesterolemia
(c) Red green color blindness
(d) Achondroplasia
24. Duchenne muscular dystrophy is inherited as:
(a) X-linked
(b) Autosomal dominant
(c) Autosomal recessive
(d) Codominant
25. Inheritance of Gardner syndrome is:
(a) Autosomal recessive
(b) Autosomal dominant
(c) X linkeddominant
(d) X linked recessive
26. In the entire human genome, coding DNA constitutes: (AIIMS May’ 14)
(a) 2%
(b) 1%
(c) 0.1%
(d) 4%
27. Methylation of cytosine leads to: (AIIMS May’ 14)
(a) Increased expression of gene
(b) Decreased expression of gene
(c) No effect on gene expression
(d) Mutation
28. ABO blood group inheritance is an example of:
(a) Codominance
(b) Mitochondrial inheritence
(c) Allelic exclusion
(d) Sex-linked Inheritance
29. Which of the following does not show X-linked dominant inheritance?
(a) Incontinentia pigmenti
(b) Duchene muscular dystrophy
(c) X-linked hypophosphatemic rickets
(d) Fragile X syndrome
30. Which is not an autosomal dominant disorder?
(a) Gardner’s syndrome
(b) Polycystic kidney
(c) Ataxia telangiectasia
(d) Achondroplasia

SINGLE GENE DISORDERS WITH NON CLASSICAL INHERITANCE

31. In Prader Willi syndrome, which of the following is increased?


(AI 2012)
(a) LH
(b) FSH
(c) TSH
(d) Ghrelin
32. NARP syndrome is seen in:
(AI 2011)
(a) Mitochondrial diseases
(b) Glycogen storage diseases
(c) Lysosomal storage diseases
(d) Lipid storage diseases
33. Maternal disomy of chromosome 15 is seen in:
(a) Prader-Willi syndrome
(AIIMS Nov 2010)
(b) Klinefelter’s syndrome
(c) Angelman syndrome
(d) Turner’s syndrome
34. Two siblings with osteogenesis imperfecta have normal parents. The mode of
inheritance is explained by which of the following?
(AIIMS May 2010)
(a) Anticipation
(b) Genomic imprinting
(c) Germline mosaicism
(d) New mutation
35. Mitochondrial DNA (mt-DNA) is known for all except:
(a) Maternal inheritance
(DPG 2011)
(b) Heteroplasmy
(c) Leber hereditary optic neuropathy is the prototype
(d) Nemaline myopathy results due to mutations in mt-DNA
36. Preferential expression of the gene depending upon the parent of origin is called:
(AI 2009)
(a) Mosaicism
(b) Genomic imprinting
(c) Alleles
(d) Chimerism
37. Preferential expression of the gene depending upon the parent of origin is called:
(AI 2008)
(a) Anticipation
(b) Germ line mosaicism
(c) Genomic imprinting
(d) Aneuploidy
38. Differential expression of same gene depending on parent of origin is referred to as:
(AI 2005)
(a) Genomic imprinting
(b) Mosaicism
(c) Anticipation
(d) Non-penetrance
39. A couple has two children affected with tuberous sclerosis. On detailed clinical and
laboratory evaluation (including molecular studies) both parents are normal. Which
one of the following explains the two affected children in this family?
(AIIMS May 2006)
(a) Non penetrance
(b) Uniparental disomy
(c) Genomic imprinting
(d) Germline mosaicism
40. Genomic imprinting is associated with:
(PGI Dec 01)
(a) Silencing of paternal chromosome
(b) Silencing of maternal chromosome
(c) Angelman syndrome
(d) Prader Willi syndrome
(e) Gonadal mosaicism
41. Dominant negative inheritance is seen in:
(a) Ehler-Danlos syndrome
(PGI Dec 2002)
(b) Marfan’s syndrome
(c) Hunter syndrome
(d) Osteogenesis imperfecta
(e) Hereditary retinoblastoma
42. True statements regarding the mitochondrial genes are:
(a) Paternal transmission
(PGI Dec 2002)
(b) Maternal transmission
(c) Mendelian inheritance
(d) Mitochondrial myopathy
(e) Horizontal inheritance
43. Mitochondrial DNA is:
(PGI June 2003)
(a) Paternally inherited
(b) Maternally inherited
(c) Horizontal inheritance
(d) Vertical inheritance
(e) Mendelian inheritance

MOST RECENT QUESTIONS

44. Which of the following is/are an example/examples of non-Mendelian inheritance?


(a) Genomic imprinting
(b) Uniparental disomy
(c) Mitochondrial inheritance
(d) All of the above
45. Increasing severity of mental retardation in male members over generations is a
result of:
(a) Mitochondrial DNA mutation
(b) Frameshift mutation
(c) Y-linked disorder
(d) Trinucleotide repeat mutation
46. All of the following are chromosomal breakage syndromes except:
(a) Fanconi’s anemia
(b) Ehler-Danlos syndrome
(c) Bloom’s syndrome
(d) Ataxia telangiectasia
47. All of the following are characterized by ‘trinucleotide repeats’ affecting the non-
coding regions except:
(a) Friedrich’s ataxia
(b) Fragile X syndrome
(c) Huntington’s disease
(d) Myotonic dystrophy
48. In Huntington chorea the causative mutation in the protein huntingtin is a:
(a) Point mutation
(b) Gene deletion
(c) Frameshift mutation
(d) Trinucleotide repeat expansion
49. Which of the following is NOT an example of a syndrome caused by uniparental
disomy?
(AIIMS May’ 14)
(a) Prader-Willi syndrome
(b) Angelman syndrome
(c) Russell-Silver syndrome
(d) Bloom syndrome
50. Genomic imprinting is seen in:
(a) Klinefelter’s syndrome
(b) Down’s syndrome
(c) Angelman syndrome
(d) Hydatidi form mole
51. Mitochondrial chromosomal abnormality leads to:
(a) Leber’s hereditary optic neuropathy
(b) Angelman syndrome
(c) Prader villi syndrome
(d) Myotonic dystrophy
52. One of the following disorders is due to maternal disomy:
(a) Prader-Willi syndrome
(b) Angelman syndrome
(c) Hydatidiform mole
(d) Klinefelter’s syndrome
53. In genomic imprinting, DNA is modified by:
(a) Acetylation
(b) Methylation
(c) Phosphorylation
(d) Deamination
54. A 48-year-old lady presented with bone pains and hepatosplenomegaly. On
examination of biopsy speci-men from spleen, crumpled tissue paper appearance was
seen. Which is the product is likely to have accumulated?
(a) Glucocerebroside
(b) Sphingomyelin
(c) Sulfatide
(d) Ganglioside
55. The phenomenon where subsequent generations are at risk of earlier and more severe
disease is known as:
(a) Anticipation
(b) Pleiotropy
(c) Imprinting
(d) Mosaicism

CHROMOSOMAL DISORDERS AND KARYOTYPING

56. No change of genetic material occurs in which of the following cytogenetic


abnormalities?
(AIIMS Nov 2012)
(a) Deletion
(b) Insertion
(c) Translocation
(d) Inversion
57. Patient present with skin bullae on sun exposure. There is a defect in which of the
following?
(AIIMS Nov 2012)
(a) Thymidine dimers
(b) Trinucleotide repeats
(c) Sugar changes
(d) DNA methylation
58. Which of the following tests is used to differentiate the chromosome of normal and
cancer cells?
(AIIMS Nov 2012)
(a) PCR
(b) Comparative genomic hybridization
(c) Western blotting
(d) Karyotyping
59. Down’s syndrome is associated with the clinical manifestation of mental retardation.
Which of the following is not associated with Down’s syndrome?
(AIIMS Nov 2011)
(a) Trisomy 21
(b) Mosaic 21
(c) Translocation t (14,21), t (21,21)
(d) Deletion of 21
60. The genetics involved in Down syndrome is:
(AI 2010)
(a) Maternal non-disjunction
(b) Paternal non-disjunction
(c) Mosacism
(d) Monosomy
61. Karyotyping is done for:
(AI 2009)
(a) Chromosomal disorders
(b) Autosomal recessive disorders
(c) Autosomal dominant disorders
(d) Linkage disorders
62. Males who are sexually underdeveloped with rudimentary testes and prostate glands,
sparse pubic and facial hair, long arms and legs and large hands and feet are likely to
have the chromosome complement of:
(AI 2004)
(a) 45, XYY
(b) 46, XY
(c) 46, XXY
(d) 46, X
63. Which of the following procedures as routine technique for karyotyping using light
microscopy?
(AI 2003)
(a) C-banding
(b) G-banding
(c) Q-banding
(d) Brd V-staining
64. A married middle aged female gives history of repeated abortions for the past 5 years.
The prenatal karyogram of the conceptus is given below:

This karyogram suggests the following:


(AI 2003)
(a) Klinefelter’s syndrome
(b) Turner’s syndrome
(c) Down’s syndrome
(d) Patau’s syndrome
65. A 19-year-old female with short stature, wide spread nipples and primary amenorrhea
most likely has a karyotype of:
(AI 2003)
(a) 47, XX + 18
(b) 46, XXY
(c) 47, XXY
(d) 45 X
66. Karyotyping most commonly done under light microscopy:
(AIIMS Nov 2009)
(a) G banding
(b) Q banding
(c) C banding
(d) R banding
67. Effective polymerase reaction was repeated for 3 cycles on a DNA molecule. What will
be the resulting formation of the copies?
(AIIMS Nov 2001)
(a) Double number of copies
(b) Three times the number of DNA molecule
(c) Four times the number of DNA molecule
(d) Eight times

MOST RECENT QUESTIONS

68. Y-chromosome is:


(AIIMS May 2007)
(a) Telocentric
(b) Metacentric
(c) Submetacentric
(d) Acrocentric
69. Which of the following is true of Klinefelter’s syndrome.
(PGI Dec 01)
(a) Chromosome pattern in 47XXY
(b) Mental retardation is present
(c) Hypogonadism occurs
(d) Increased FSH level
(e) Eunuchoid proportions
70. The classic karyotype of Klinefelter’s syndrome is:
(Karnataka 2009)
(a) 47XXY
(b) 45XO
(c) 48XXXY
(d) 46XY/47XXY
71. Chromosomal abnormality in Mongolism is:
(a) Trisomy 21
(Karnataka 2005)
(b) Trisomy 22
(c) Trisomy 17
(d) Trisomy 5
72. Trisomy 13 is identified as:
(Karnataka 2005)
(a) Edward’s syndrome
(b) Patau’s syndrome
(c) Down’s syndrome
(d) Klinefelter’s syndrome
73. In Down syndrome, there is non-disjunction of chromosome:
(a) 13
(b) 15
(c) 18
(d) 21
74. Barr body is not seen in:
(a) Klinefelter syndrome
(b) Turner syndrome
(c) Normal female
(d) XXX syndrome
75. Karyotype is:
(a) Size, shape and number of chromosome
(b) Gene packing
(c) DNA assay
(d) None
76. Osteogenesis imperfecta defect in:
(a) Collagen type I
(b) Elastin
(c) Collagen type IV
(d) Fibrillin 2
77. Karyotyping is done in which phase of cell cycle?
(a) Anaphase
(b) Metaphase
(c) Telophase
(d) S phase
78. The number of chromosomes in Turner syndrome is:
(a) 47
(b) 46
(c) 45
(d) 44
79. Patau syndrome is due to which of the following?
(a) Trisomy 21
(b) Trisomy 18
(c) Trisomy 21
(d) Trisomy 13
80. The number of chromosomes in Klinefelter syndrome is:
(a) 47
(b) 46
(c) 45
(d) 44
81. Chromosomes are visualized through light microscope with resolution of:
(a) 5 Kb
(b) 50 Mb
(c) 5 Kb
(d) 500 Kb
82. Which of the following techniques can be used to detect exact localisation of a genetic
locus? (AIIMS May,
Nov 2013)
(a) Chromosome painting
(b) FISH
(c) Comparative genomic hybridization
(d) Western blot
83. In Marfan syndrome, the defect is in:
(a) Fibrillin I
(b) Fibrillin II
(c) Collagen
(d) Elastin
84. If a chromosome divides in an axis perpendicular to usual axis of division it is going
to form:
(a) Ring chromosome
(b) Isochromosome
(c) Acrocentric chromosome
(d) Subtelocentric chromosome
85. Most lethal combination:
(a) Autosomal trisomy
(b) Chromosomal monosomy
(c) Autosomal monosomy
(d) Chromosomal trisomy
86. All of the following are true about the Downs syndrome except:
(a) Most common cause is trisomy21
(b) Extra chromosome is of maternal origin
(c) Incidence of the Robertsonian translocation is 1:1000
(d) Mosaicism 21 has no association with maternal age
87. All are true about turner syndrome except:
(a) Short stature
(b) Webbed neck
(c) Dysgenetic gonads
(d) True hermaphroditism
88. SnRNA mutation is associated with which syndrome?
(a) Turner’s syndrome
(b) Prader-Willi syndrome
(c) Klinefelter syndrome
(d) Patau syndrome

PEDIGREE ANALYSIS, GENE LOCATION, LYON HYPOTHESIS

89. BRCA 1 gene is located on?


(AIIMS May 2011)
(a) Chromosome 13
(b) Chromosome 11
(c) Chromosome 17
(d) Chromosome 22
90. Males are more commonly affected than females in which of the following genetic
disorders?
(a) Autosomal recessive disorder
(AI 2010)
(b) Autosomal dominant disorder
(c) X-linked recessive disorder
(d) X-linked dominant disorder
91. In-situ DNA nick end labeling can quantitate:
(a) Fraction of cells in apoptotic pathways
(b) Fraction of cells in S phase
(AI 2005)
(c) p53 gene product
(d) bcr/abl gene
92. The chances of having an unaffected baby, when both parents have achondroplasia,
are:
(AI 2005)
(a) 0%
(b) 25%
(c) 50%
(d) 100%
93. Study the following carefully:
(AI 2005)

Read the pedigree. Inheritance pattern of the disease in the family is:
(a) Autosomal recessive type
(b) Autosomal dominant type
(c) X-linked dominant type
(d) X-linked recessive type
94. Kinky hair disease is a disorder where an affected child has peculiar white stubby hair,
does not grow, brain degeneration is seen and dies by age of two years. Mrs. A is
hesitant about having children because her two sisters had sons who had died from
kinky hair disease. Her mother’s brother also died of the same condition. Which of the
following is the possible mode of inheritance in her family?
(AI 2004)
(a) X-linked recessive
(b) X-linked dominant
(c) Autosomal recessive
(d) Autosomal dominant
95. An albino girl gets married to a normal boy, what are the chances of their having an
affected child and what are the chances of their children being carriers?
(AI 2003)
(a) None affected, all carriers
(b) All normal
(c) 50% carriers
(d) 50% affected, 50% carriers
96. The mother has sickle cell disease; Father is normal; Chances of children having
sickle cell disease and sickle cell trait respectively are:
(AI 2001)
(a) 0 and 100%
(b) 25 and 25%
(c) 50 and 50%
(d) 10 and 50%
97. Father has a blood group B; Mother has AB; Children are not likely to have the
following blood group:
(AI 2001)
(a) O
(b) A
(c) B
(d) AB
98. Gene therapy is used for:
(AIIMS May 2009)
(a) Cystic fibrosis
(b) Sickle cell anemia
(c) Thalassemia
(d) All of the above
99. Gene for major histocompatibility complex is located on which chromosome?
(AIIMS Nov 2008)
(a) Chromosome 10
(b) Chromosome 6
(c) X chromosome
(d) Chromosome 13
100. Gene for folate carrier protein is located on chromosome:
(AIIMS Nov 2008, May 2008)
(a) Chromosome 10
(b) Chromosome 5
(c) Chromosome 21
(d) Chromosome 9
101.Ability of stem cells to cross barrier of differentiation to transform into a cell of another
lineage expressing the molecular characteristics of different cell type with the ability to
perform the function of the new cell type is referred as:
(AIIMS Nov 2007)
(a) De differentiation
(b) Re differentiation
(c) Trans-differentiation
(d) Sub differentiation
102. If both husband and wife are suffering with achondroplasia, what are their chances of
having a normal child.
(AIIMS May 2004)
(a) 0%
(b) 25%
(c) 50%
(d) 100%
103. A one year old boy presented with hepatosplenomegaly and delayed milestones. The
liver biopsy and bone marrow biopsy revealed presence of histiocytes with PAS-
positive Diastase-resistant material in the cytoplasm. Electron-microscopic
examination of these histiocytes is most likely to reveal the presence of:
(AIIMS Nov 2003)
(a) Birbeck’s granules in the cytoplasm
(b) Myelin figures in the cytoplasm
(c) Parallel rays of tubular structures in lysosomes
(d) Electron dense deposit in the mitochondria
104. The gene that regulates normal morphogenesis during development is:
(AIIMS Nov 2002)
(a) FMR-1 gene
(b) Homeobox gene
(c) P-16
(d) PTEN
105. A baby’s blood group was determined as O Rh negative. Select the blood group the
baby’s mother or father will not have:
(AIIMS Nov 2002)
(a) A, Rh positive
(b) B, Rh positive
(c) AB, Rh negative
(d) O, Rh positive
106. Thalassemia occurs due to which mutation?
(PGI Dec 2000)
(a) Missense
(b) Splicing
(c) Transition
(d) Frame-shift
(e) Truncation
107. Congenital syndrome associated with lympho-proliferative malignancy is:
(PGI June 2005)
(a) Bloom syndrome
(b) Fanconi’s anemia
(c) Turner syndrome
(d) Chediak Higashi syndrome
(e) Ataxia telangiectasia
108. Loss of heterozygosity means:
(PGI Dec 2005)
(a) Loss of single arm of chromosome.
(b) Loss of mutant allele in mutant gene
(c) Loss of normal allele in mutant gene
(d) Loss of normal allele in normal gene

MOST RECENT QUESTIONS

109. Long and short arm of chromosome are called respectively:


(a) p and q
(b) q and p
(c) m and n
(d) r and s
110. Cystic fibrosis transmembrane conductance regulator gene is located on
chromosome:
(a) 5
(b) 6
(c) 7
(d) 8
111. Which one of the following is not a germ cell tumor?
(a) Dermoid
(b) Granulosa cell tumor
(c) Choriocarcinoma
(d) Gynandroblastoma
112. The CFTR gene associated what cystic fibrosis is located on chromosome:
(a) 5
(b) 12
(c) 4
(d) 7
113. In cystic fibrosis the most frequent pulmonary pathogen is:
(a) Pseudomonas
(b) Enterococci
(c) Staphylococci
(d) Klebsiella
114. Male to male transmission is not seen in:
(a) Autosomal dominant diseases
(b) Autosomal recessive disease
(c) X-linked dominant disease
(d) Genomic imprinting
115. Microarray is best characterised by:
(a) Study of multiple genes
(b) Study of disease
(c) Study of organisms
(d) Study of blood group
116. The technique used for separation and detection of RNA is which one of the
following:
(a) Northern blot
(b) Southern blot
(c) Eastern blot
(d) Western blot
117. True statement about inheritance of an X-linked recessive trait is:
(a) 50% of boys of carrier mother are affected
(b) 50% of girls of diseased father are carrier
(c) Father transmits disease to the son
(d) Mother transmits the disease to the daughter
118. Gene for Wilm’s tumor is located on:
(a) Chromosome 1
(b) Chromosome 10
(c) Chromosome 11
(d) Chromosome 12
119. Which of the following potentially represents the most dangerous situation?
(a) Rh+ve mother with 2nd Rh-ve child
(b) Rh-ve mother with 2nd Rh+ve child
(c) Rh+ve mother with 1st Rh-ve child
(d) Rh-ve mother with 1st Rh+ve child
1. Ans. (a) Ataxia telangiectasia
(Ref: Robbins 8th/302-3, 1323-4, 9/e p242-243)
• Ataxia telangiectasiais an autosomal recessive condition.
• Patients have increased sensitivity to X-ray-induced chromosome abnormalities.
• Characterized by an ataxic-dyskinetic syndrome beginning in early childhood, caused by
neuronal degeneration predominantly in the cerebellum, the subsequent development
of telangiectasias in the conjunctiva and skin, and immunodeficiency.
• The nuclei of cells in many organs (e.g., Schwann cells in dorsal root ganglia and
peripheral nerves, endothelial cells as well as pituicytes) show a bizarre enlargement of
the cell nucleus and are referred to as amphicytes.
• The lymph nodes, thymus, and gonads are hypoplastic.
• Clinical features include recurrent sinopulmonary infections and unsteadiness in
walking.
• Increased risk of development of lymphoid malignant disease (T-cell leukemia
and lymphomaoma); gliomas and carci

All other options are autosomal dominant conditions.

2. Ans. (d) Familial hypercholesterolemia


(Ref: Robbins 7th/152, 9/e p141)
Most of the metabolic disorders have autosomal recessive inheritance except:
Her – Her’s disease [Liver phosphorylase deficiency]
Left – Lesch Nyhan syndrome
Eye – Ocular albinism
Has – Hunter syndrome and Hypercholesterolemia (familial)
Five – Fabry’s disease
Pimples – Porphyria [Acute intermittent]

Note: All these exceptions have X-linked recessive inheritance except acute intermittent porphyria and familial
hypercholesterolemia which are autosomal dominant disorders.

3. Ans. (b) 30,000 genes


(Ref: Robbins’s 7th/1219, 8th/136)
Humans have a mere 30,000 genes rather than the 100,000 predicted only recently. Recent
Robbins mentions the number of genes to 20,000 to 25,000.
4. Ans. (b) Emphysema; (c) Fibrosis of portal tact; (d) Diastase resistance positive
hepatocytes
(Ref: Robbins 9/e p850-851)
• This is an autosomal recessive disease characterized by deficiency of a1 - antitrypsin
(important protease inhibitor).
• There is portal tract fibrosis in neonatal hepatitis. About 10 - 20% of newborn with a1 –
antitrypsin deficiency develop neonatal hepatitis and cholestasis.
• Hepatocellular carcinoma develops in 2-3 % a1 - antitrypsin deficiency in adults.
• The treatment and cure, for severe hepatic disease is orthotropic liver transplantation.
5. Ans. (b) Thalassemia; (c) Sickle cell anemia; (d) Cystic fibrosis
(Ref: Robbins 7th/151, 8th/141-2, 9/e p141)
• Hereditary spherocytosis is an autosomal dominant disorder
• Hemophilia is an X-linked recessive disease.
A broad generalization is that the physiologic metabolic enzyme deficiencies are all autosomal recessive
whereas Structural defects are autosomal dominant.
6. Ans. (b) Thalassemia; (c) Sickle cell anemia; (d) Cystic fibrosis; (e) Hemophilia
(Ref: Robbins 8th/652 , 9/e p141)
7. Ans. (d) Huntington’s disease
(Ref: Harrison 17th/401 Robbins 7th/1393, 8th/141,168, 9/e p141)
8. Ans. (d) X-linked recessive disease
(Ref: Robbins 7th/1336 , 9/e p142)
9. Ans. (d) IV
(Ref: Robbins 9/e p146)
10. Ans. (a) Point mutation
(Ref: Robbins 9/e p138)
11. Ans. (a) Albinism
(Ref: Robbins 9/e p141-142)
12. Ans. (c) Fibrillin
(Ref: Robbins 9/e p144)
13. Ans. (a) Autosomal dominant
(Ref: Robbins 9/e p140)
14. Ans. (a) Homogentisic acid
(Ref: Robbins 9/e p64)
15. Ans. (d) Defective synthesis of fibrillin
(Ref: Robbins 7th/104, 154-155 , 9/e p144)
The stem describes a patient of Marfan syndrome which is an autosomal dominant disorder that
results from defective synthesis of fibrillin.
16. Ans. (a) Autosomal dominant…see earlier explanation (Ref: Robbins 9/e p141)
17. Ans. (a) X chromosome…explained earlier
(Ref: Robbins 8/e p142, 9/e p142)
18. Ans. (d) Huntington’s chorea
(Ref: Robbins 8/e p141)
19. Ans. (a) Autosomal dominant…see earlier explanation (Ref: Robbins 9/e p141)
20. Ans. (a) Autosomal dominant…see earlier explanation (Ref: Robbins 9/e p141)
21. Ans. (c) Facioscapulohumeral muscular dystrophy
(Ref: Robbins 8/e p142, 1270)
• Facioscapulohumeral muscular dystrophy is an autosomal dominant disorder
characterised by facial weakness (difficulty with eye closure and impaired smile) and
scapular winging.
• Progressive Muscular Dystrophies
Type Inheritance Clinical Features
Duchenne’s XR Progressive weakness of girdle muscles
Unable to walk after age 12
Progressive kyphoscoliosis
Respiratory failure in 2d or 3d decade
Becker’s XR Progressive weakness of girdle muscles
Able to walk after age 15
Respiratory failure may develop by 4th decade
Type Inheritance Clinical Features
Limb-girdle AD/AR Slow progressive weakness of shoulder and
hip girdle muscles
Emery-Dreifuss XR/AD Elbow contractures, humeral and peroneal
weakness
Congenital AR Hypotonia, contractures, delayed milestones
Progression to respiratory failure in some;
static course in others
Myotonica (DM1, DM2) AD Slowly progressive weakness of face,
shoulder girdle, and foot dorsiflexion
Preferential proximal weakness in DM2
Facioscapulohumeral ADQ Slowly progressive weakness of face,
shoulder girdle, and foot dorsiflexion
Oculopharyngeal AD Slowly progressive weakness of extraocular,
pharyngeal, and limb muscles

22. Ans. (c) Affects intestine only


(Ref: Robbins 9/e 466-471)
• Cystic fibrosis is also known as salty baby syndromeQ or mucoviscidosisQ. It is an
autosomal recessiveQ genetic disorder that affects most critically the lungs, and also
the pancreas, liver, and intestine.
• It is characterized by abnormal transport of chloride and sodium across epithelium,
leading to thick, viscous secretions
• Affected patients are prone to Pseudomonas infectionsQ for which a combination of
3rd generation cepahalosporins and aminoglycosideQ is used.
• The most commonly used form of testing is the sweat test using the drug that stimulates
sweating (pilocarpine iontophoresisQ).
• Patients require repeated use of antibiotics and lung transplantation (in later stages) to
survive.
23. Ans. (a) Vitamin D resistant rickets
(See Below)
Vitamin D resistant rickets X-linked dominant due to PHEX gene
Familial hypercholesterolemia Autosomal recessive
Red green colour blindness X-linked recessive
Achondroplasia Autosomal dominant

24. Ans. (a) X-linked


(Ref: Robbins 8/e p142 , 9/e p142) ...see text for details
X-linked recessive disorder can be remembered by the following line: Less hCG is Detected
Clinically in A Fragile Woman.
25. Ans. (b) Autosomal dominant
(Ref: Robbins 8/e p816 , 9/e p806,809)

• It is a subtype of familial adenomatous polyposis inherited as an autososmal dominant disorder.


• Gardener syndrome Q Intestinal polyps + epidermal cysts + fibromatosis + osteomas (of the mandible, long
bones and skull).

26. Ans. (a) 2%


(Robbins 9th /2)
‘98.5% of the human genome that does not encode proteins’….. direct quote from Robbins. So,
it means that about 1.5% of the genome is used for coding proteins. The best answer
therefore is option “a”.
27. Ans. (b) Decreased expression of gene
(Robbins 9th/4)
High levels of DNA methylation in gene regulatory elements typically result in transcriptional
silencing.
28. Ans. (a) Codominance
(Ref: Robbins 9th/140)
When both of the alleles of a gene pair contribute to the phenotype, this is called as codominance.
Good examples of codominant inheritance are:

• Histocompatibility
• Blood group antigens

29. Ans. (b) Duchene muscular dystrophy


(Ref: Robbins 9/e p142)
30. Ans (c) Ataxia telangiectasia
(Ref: Robbins 9/e p141)
31. Ans. (d) Ghrelin
(Ref: Robbins 8th/441-2, and 9/e 444, Pediatric endocrinology: mechanisms,
manifestations, management 1st/26-8)

• Ghrelin is a growth hormone secretagogue and the only gut hormone with orexigenic (means
increasing food intake) property.
• It is primarily produced in the stomach. In children, its value is inversely related with body mass index and
insulin values. It is postulated to play an important role in hyperphagia.

Direct quote from Pediatric endocrinology… ‘fasting ghrelin levels were obtained in children
with Prader Willi syndrome and found to be elevated 3-4 times when compared to children
who are obese’.
Pancreatic polypeptide Y (PYY) is normally secreted from endocrine cells of the ileum and colon.
It reduces energy intake and its reduced levels in the patients of Prader Willi syndrome
may contribute to hyperphagia and obesity.
32. Ans. (a) Mitochondrial diseases
(Ref: Harrison 17th/316-317, Robbins 8th/1328)
NARP syndrome (Neuropathy, ataxia, and retinitis pigmentosa), is a condition related to changes
in mitochondrial DNA.
For details, see text.
33. Ans. (a) Prader-Willi syndrome
(Ref: Robbins 8th/172)
Prader Willi syndrome could be present because of the following:
a. Deletion of paternal chromosome 15 or
b. Uniparental disomy of maternal chromosome 15.
For details, see text.
34. Ans. (c) Germline mosaicism
(Ref: Robbins 8th/173)
Gonadal mosaicism results from a mutation that occurs postzygotically during early (embryonic)
development. If the mutation affects only cells destined to form the gonads, the gametes
carry the mutation, but the somatic cells of the individual are completely normal. Such an
individual is said to exhibit germ line or gonadal mosaicism. A phenotypically normal parent
who has germ line mosaicism can transmit the disease-causing mutation to the offspring
through the mutant gamete. Since the progenitor cells of the gametes carry the mutation,
there is a definite possibility that more than one child of such a parent would be affected. It is
seen with tuberous sclerosis and osteogenests imperfecta.
35. Ans. (d) Nemaline myopathy results due to mutations in mt-DNA
(Ref: Harrison 17th/2688, Robbins 8th/171)
Nemaline myopathy is not a mitochondrial disease
Nemaline Myopathy

Nemaline myopathy is a clinically heterogeneous condition and not a mitochondrial disease. Five genes
have been associated with this myopathy. All code for thin filament–associated proteins, suggesting
disturbed assembly or interplay of these structures as a pivotal mechanism. Mutations of the nebulin
(NEB) gene account for most cases, including both severe neonatal and early childhood forms, inherited
as autosomal recessive disorders.

36. Ans. (b) Genomic imprinting


(Ref: Robbins 9/e 173)
37. Ans. (c) Genomic imprinting
(Ref: Robbin 9/e 173)
38. Ans. (a) Genomic imprinting (Ref: Harrison’s 17th/413,
18th/518; Robbins 7th/186. 8th/171-2 , 9/e 173)
Genomic imprinting is the phenomenon that leads to preferential expression of an allele
depending on its, parental origin. It is also seen in (updated from HARRISON 18th):

• Wiedemann syndrome (have two paternal but no maternal copies of chromosome 11).
• Albright’s hereditary osteodystrophy (short stature, brachydactyly and PTH resistance). There is mutation
in the Gs a subunit; individuals express the disease only when the mutation is inherited from the mother).

39. Ans. (d) Germline mosaicism


(Ref: Robbins 8th/173, 7th/187, Harrison 17th/1800, 9/e 174)
Germline mosaicism is seen with osteogenesis imperfecta and tuberous sclerosis. For
details, see text.
40. Ans. (a) Silencing of paternal chromosome; (b) Silencing of maternal chromosome; (c)
Angelman syndrome; (d) Prader-Willi syndrome
(Ref: Robbins’ 7th-1856, 8th/171-3 , 9/e 173)
41. Ans. (a) Ehler-Danlos syndrome; (b) Marfan’s syndrome; (d) Osteogenesis imperfecta
(Ref: Robbins 7th/151 , 9/e 144-146)
Dominant negative effects occurs when a mutant polypeptide not only loses its own
function but also interferes with the product of normal allele in a heterozygote, thus
causing more severe effects than deletion or non-sense mutations in the same gene.
Structural proteins that contribute to multimeric structures are vulnerable to dominant
negative effects, e.g. collagen.

Seen in: Osteogenesis imperfecta, Ehler-Danlos syndrome, Marfan’s syndrome.

42. Ans. (b) Maternal transmission; (d) Mitochondrial myopathy


(Ref: Harrison 18th/501, Robbins 8th/171) ...see text
43. Ans. (b) Maternally inherited
(Ref: Robbins 9/e 171)
44. Ans. (d) All of the above
(Ref: Robbins 8th/167 , 9/e 168)
Non-Mendelian inheritance can be classified into following four categories:
Trinucleotide repeat mutation Mitochondrial gene mutations Genomic Gonadal
disorders imprinting Mosaicism
• Fragile- X syndrome • Leber Hereditary Optic • Prader-Willi • Tuberous
• Friedreich’s Ataxia Neuropathy syndrome sclerosis
• Myotonic dystrophy • Kearns-Sayre syndrome (Paternal • Osteogenesis
• Huntington disease • Chronic progressive external deletion) imperfecta
• Spinobulbar muscular atrophy ophthalmoplegia • Angelman
(Kennedy disease) • Pearson syndrome syndrome
• Spinocerebellar ataxias • Neurogenic muscular weakness (maternal
with ataxia and retinitis deletion)
pigmentosa (NARP)

45. Ans. (d) Trinucleotide repeat mutation


(Ref:Robbins 8th/169-171 , 9/e 169)
46. Ans. (b) Ehlers-Danlos syndrome
(Ref: Robbins 7th/155-6, Table 5.5 174 , 9/e 145-146)
Chromosome breakage syndromes are associated with high level of chromosomal
instability. Such conditions include. Fanconi anemia, Bloom syndrome and Ataxia
telangiectasia.
Ehlers-Danlos syndrome (EDS)
• Genetic disorder resulting from defective synthesis of fibrillar collagen
• Skin is extraordinary stretchable, extremely fragile and vulnerable to trauma, joints are
hypermobile
• Internal complications: rupture of colon, large arteries
47. Ans. (c) Huntington’s disease
(Ref: Robbins 9/e 168)
Huntington’s disease is characterized by trinucleotide repeats affecting the coding region. Rest all
conditions mentioned in the options affects the non-coding regions.
48. Ans. (d) Trinucleotide repeat expansion
(Ref: Robbins 9/e p168)
Expansion of trinucleotide repeats is an important genetic cause of human disease, particularly
neurodegenerative disorders. There are three key mechanisms by which unstable repeats
cause diseases:

• Loss of function of the affected gene occurs in fragile X syndrome. In such cases the repeats are generally
in non-coding part of the gene.
• A toxic gain of function by alterations of protein structure as in Huntington disease and spinocerebellar
ataxias. In such cases the expansions occur in the coding regions of the genes.
• A toxic gain of function mediated by mRNA as is seen in fragile X tremor-ataxia syndrome. In this condition,
the non coding parts of the gene are affected.

49. Ans. (a) Bloom syndrome


(Robbins 9th/314-5) ...see text
50. Ans. (c) Angelman syndrome
(Ref: Robbins 9/e 172)
51. Ans. (a) Leber’s hereditary optic neuropathy
(Ref: Robbins 8/e p171 , 9/e 172)
Examples of mitochondrial inheritance are Leber’s optic neuropathy, Leigh’s disease,
MELAS (mitochondrial encephalopathy, lactic acidosis and stroke like syndrome) NARP
syndrome (Neuropathy, ataxia, and retinitis pigmentosa), Kearns-Sayre syndrome, Chronic
progressive external ophthalmoplegia and Pearson syndrome.
52. Ans. (a) Prader-Willi syndrome
(Ref: Robbins 9/e 173)
Maternal disomy is associated with disorders like Prader-Willi syndrome and Angelman syndrome.
It is also seen in other conditions like molar pregnancy and Beckwith-Wiedemann syndrome.
53. Ans. (b) Methylation
(Ref: Robbins 9th/172)
Imprinting selectively inactivates either the maternal or paternal allele. Thus, maternal imprinting
refers to transcriptional silencing of the maternal allele, whereas paternal imprinting implies
that the paternal allele is inactivated. Imprinting is associated with differential patterns of
DNA methylation at CG nucleotides. Other mechanisms include histone H4 deacetylation
and methylation.
54. Ans. (a) Glucocerebroside
(Ref: Robbins 9th/153)

Presence of bone pain and hepatosplenomegaly with a splenic biopsy cell having “crumpled paper”
appearance is highly suggestive of Gaucher’s disease.

55. Ans. (a) Anticipation (Ref: Robbins 9th/169)


Anticipation refers to the observation that clinical features of a genetic disease worsen with each successive
generation. It is observed in association with Fragile X syndrome and Huntington disease commonly.

56. Ans. (d) Inversion (Ref: Robbins 8th/138, 160)


Inversion refers to a rearrangement that involves two breaks within a single chromosome with reincorporation of the
inverted, intervening segment. It is not associated with change in genetic material.
57. Ans. (a) Thymidine dimers (Ref: Robbins 8th/275, 302)
Xeroderma pigmentosum

• Autosomal recessiveQ inherited disorder of defective DNA repairQ.


• Affected individuals are at increased risk for the development of skin cancers particularly following
exposure to the UV light contained in sun rays.
• UV radiation causes cross-linking of pyrimidine residues, preventing normal DNA replication. Such DNA
damage is repaired by the nucleotide excision repair systemQ.
• Deficiency of enzymes like UV specific endonuclease (commonest), DNA polymerase and DNA
ligase is implicated

58. Ans. (b) Comparative genomic hybridization (Ref: Robbins


8th/179, Harrison 18th/512)
• Comparative genomic hybridization (CGH) differentiates between cancer and normal
cells.
• Comparative genomic hybridization (CGH) is a method that can be used only when DNA
is available from a specimen of interest. The entire DNA specimen from the sample of
interest is labeled in one color (e.g., red), and the normal control DNA specimen is
indicated by another color (e.g., green). These are mixed in equal amounts and
hybridized to normal metaphase chromosomes. The red-to-green ratio is analyzed by
a computer program that determines where the DNA of interest may have gains or
losses of material.
59. Ans. (d) Deletion of 21 (Ref: Robbins 8th/161-162)
Down syndrome is characterized by trisomy 21 (NOT deletion of chromosome 21). For details
see text.
60. Ans. (a) Maternal non-disjunction
(Ref: Robbins 8th/161)
Genetics of Down syndrome

• Meiotic nondisjunction of chromosome 21 occuring in the ovum is seen in 95% cases with trisomy
21 and is the so, commonest cause of Down syndrome. The extra chromosome is of maternal origin.
• There is a Strong relation with maternal age
• It may also be seen with Robertsonian translocation and mosaicism.

61. Ans. (a) Chromosomal disorders (Ref: Robbins 8th/158)

• Karyotyping is the study of chromosomes


• Chromosomes are arrested in metaphase by Colchicine
• These are then stained by many stains. Most commonly used stain is Giemsa stain, so called G-banding
• The chromosomes are arranged in order of decreasing length
• Any alteration in number or structure of chromosomes can be easily detected by karyotyping

62. Ans. (c) 46, XXY (Ref: Harrison’s 17th/2340 –


2341, 411, Robbins 7th/179)
It is a typical case of Klinefelter syndrome. The features pointing towards this diagnosis are:
• Male phenotype
• Hypogonadism [rudimentary testes]
• Decreased secondary sexual characteristics [sparse pubic and facial hairs]
• Disproportionately long arms and legs.
Patients with Klinefelter syndrome have extra X chromosome, so they may be 47,XXY or
46XY/47 XXY mosaics.
None of the options appear correct as an extra X chromosome in the male should increase the
total number of chromosomes to 47 and hence 47 XXY should be the most appropriate
answer. However an extra X–chromosome is the most essential aspect and hence, within
the available options, the best answer is 46 XXY.
63. Ans. (b) G-Banding (Ref: Harsh Mohan 6th/17,
Robbins 8th/159, Cancer cytogenetics3rd/2010)
G banding is the most widely used technique for routine cytogenetic analysis.’ So, G banding is
chosen as the answer of choice. For details of different banding techniques, please see text.
64. Ans. (c) Down’s syndrome (Ref: Robbins 7/175)
• The given karyogram shows three chromosomes at 21 instead of a pair. It is called
Trisomy 21.
• Trisomy 21 is synonymous with Down’s syndrome and is the most common of the
chromosomal disorders. It is a major cause of mental retardation.
• Other trisomies are Edward syndrome (Trisomy 18) and Patau syndrome (Trisomy 13)
65. Ans. (d) 45X (Ref: Robbins 7/179-180)
Given features (Female, primary amenorrhea, short stature, widely spaced nipples) suggests the
diagnosis of Turner’s syndrome.
Turner’s Syndrome
Turner’s syndrome is the most common sex chromosomal disorder in phenotypic females.
Turner’s syndrome results from complete or partial loss of one X chromosome (45, XO) and is
characterized by hypogonadism in phenotypic females

66. Ans. (a) G banding (Ref: Harsh Mohan 6th/17,


Robbins 8th/159, Cancer cytogenetics 2010/3rd edition)
Refer to the text for detail.
Q banding is easily ruled out because it does not require light microscopy. Cancer cytogenetics clearly
mentions that ‘sequence specific techniques like T banding; C banding and NOR banding have been
replaced by in situ hybridization techniques. G banding is the most widely used technique for routine
cytogenetic analysis.’ So, G banding is chosen as the answer of choice.

67. Ans. (d) Eight times (Ref: Harrison 17th/391)


Friends, the number of copies of the particle after ‘n’ cycles of polymerase chain reaction is given
by the formula 2n times the original copies. So, the number of copies after 3 cycles would be
23 = 8 times the original copies.
PCR is used to amplify DNA but RT – PCR (reverse transcriptase PCR) can be used for studying
mRNA.
68. Ans. (d) Acrocentric (Ref: Nelson 17th/382)
X chromosome is Submetacentric and Y chromosome is Telocentric.
69. Ans. (a) Chromosome pattern is 47XXY; (b) Mental retardation is present; (c)
Hypogonadism occurs; (d) Increased FSH level; (e) Eunuchoid proportions
(Ref: Harrison’ 16th-2215, 2216, Robbins 7th/179, 8th/165) ...see text
70. Ans. (a) 47XXY (Ref:Robbins 7th/179, 8th/161-2)
71. Ans. (a) Trisomy 21 (Ref: Robbins 7th/179, 8th/165)
72. Ans. (b) Patau’s syndrome (Ref: Robbins 7th/177)
73. Ans. (d) 21 (Ref: Robbins 8th/161, 7th/175-176)
74. Ans. (b) Turner syndrome (Ref: Robbins 8th/165-167)
75. Ans. (a) Size, shape and number of chromosome
(Ref: Robbins 8th/158-159; 7 th/170)
76. Ans. (a) Collagen type I (Ref: Robbins 8/e p1211-2)
• Osteogenesis imperfecta (or brittle bone diseaseQ) is a phenotypically diverse
disorder caused by deficiencies in the synthesis of type 1 collagenQ.
• It is the most common inherited disorder of connective tissue.
• It principally affects bone, but also impacts other tissues rich in type 1 collagen (joints,
eyes, ears, skin, and teeth). It is characterized by bone fragility, hearing loss, blue
scleraQ and dentinogenesis imperfect.
• Osteogenesis imperfecta usually results from autosomal dominant mutations in the
genes that encode theα1 and α2 chains of collagen.
77. Ans. (b) Metaphase (Ref: Robbins 8/e p158)
Revise NEET info!
• Karyotyping is the study of chromosomes
• Chromosomes are arrested in metaphase by ColchicineQ
• These are then stained by many stains. Most commonly used stain is Giemsa stainQ, so called G-
bandingQ
• The chromosomes are arranged in order of decreasing lengthQ. Any alteration in number or
structure of chromosomes can be easily detected by karyotyping

78. Ans. (c) 45 (Ref: Robbins 8/e p165)


Turner syndrome is the most common cause of sex chromosomal abnormality in the
females. It usually results from complete or partial monosomy of X chromosome and
associated with hypogonadism in phenotypic females.Q Patients have the (45X) karyotype or
may be mosaics.
79. Ans. (d) Trisomy 13
(Ref: Robbins 8/e p162)
Trisomy 13 is known as Patau syndrome characterized by:
• Mentalretardation, rocker bottom feet and congenital heart defects (VSD and PDA)
• Cleft lip/palate
• Polydactyly
• Microcephaly
• Eye defects (microphthalmia, iris coloboma, cataract, retinal dysplasia)
• Capillary hemangiomata
Mnemonic: useful for AIIMS and NEET questions!
• p for Patau and polydactyly as well as palate defects: Patau syndrome (trisomy 13)
• Edward syndrome (trisomy 18): e for Edward and Extra occiput (prominent occiput)

80. Ans. (a) 47 (Ref: Robbins 8/e p165)


Klinefelter syndrome is the most common chromosomal disorder of males associated with
hypogonadism and infertility. It is due to extra-X-chromosome. Classically, it is 47, XXY.
Other variants can have 48 XXXY, rarely 49 XXXY or mosaics can be there with some cells
containing normal 46, XY and others 47, XXY.
81. Ans. (c) 5 Mb (Ref: Encyclopedia of Genetics 3/e p29)
Karyotype analysis detects both numerical and structural chromosomal aberrations (overall
resolution is 5 Mega bases (Mb); breakpoint resolution is 5 to 15 Mb).
82. Ans. (b) FISH (Ref: Robbin 8/e p 179)
FISH uses DNA probes that recognize sequences specific to particular chromosomal
regions.
83. Ans. (a) Fibrillin I (Ref: Robbins 8/e p144)
• Fibrillin occurs in two homologous forms, fibrillin-1 and fibrillin-2, encoded by two
separate genes, FBN1 and FBN2 mapped on chromosomes 15q21.1 and 5q23.31,
respectively.
Contd...
Contd...
• Mutations of FBN1 (affecting fibrillin-1Q) underlie Marfan syndrome
• Mutations of the related FBN2 gene (fibrillin-2Q) are less common, and are associated
with congenital contractural arachnodactyly, an autosomal dominant disorder
characterized by skeletal abnormalities.
84. Ans. (b) Isochromosome (Ref: Robbin 8/e p160, The
Principles of Clinical Cytogenetics 3/e p157-8)
Isochromosome formation results when one arm of a chromosome is lost and the remaining
arm is duplicated, resulting in a chromosome consisting of two short arms only or of two long
arms. It has morphologically identical genetic information in both arms.
• The reason for the formation of an isochrome is the centromere misdivision. Instead of dividing
longitudinally to separate the two sister chromatids, the centromere undergoes a transverse
split that separated the two arms from one another.

85. Ans. (c) Autosomal monosomy (Ref: Robbins 9/e p159)


Monosomy involving an autosome generally causes loss of too much genetic
information to permit live birth or even embryogenesis.
86. Ans (c) Incidence of the Robertsonian translocation is
1:1000 (Ref: Robbins 9/e p161)
87. Ans (d) True hermaphroditism (Ref: Robbins 9/e p166)
88. Ans (b) Prader-Willi syndrome (Ref: Robbins 9/e p173)
Prader-Willi syndrome is characterized by interstitial deletion in the long arm of chromosome 15.
In all cases the deletion affects the paternally derived chromosome 15. SNORP family of
genes encodes small nucleolar RNAs which are involved in modifications of ribosomal
RNAs. Loss of SNORP functions is believed to contribute to Prader-Willi syndrome.
89. Ans. (c) Chromosome 17 (Ref: Robbins
8th/1078, Harrison 17th/563)
Features BRCA1 BRCA2
Chromosome 17q21 13q12.3
Function Tumor suppressor, Transcriptional Tumor suppressor, Transcriptional
regulation, Role in DNA repair regulation, Role in DNA repair

Age at onset Younger age (40s to 50s) 50 years


% Single gene hereditary 52% 32%
disorders
Risk of other tumors (varies Ovarian, Male breast cancer (lower Ovarian, ale breast cancer
with specific mutation) than BRCA2), Prostate, colon, Prostate, pancreas, stomach,
pancreas melanoma, colon
Pathology of breast cancers Greater incidence of medullary Similar to sporadic breast cancers
carcinomas, poorly differentiated
carcinomas, ER-, PR-, and
Her2/neu-negative carcinomas,
carcinomas with p53 mutations

90. Ans. (c) X-linked recessive disorder


(Ref: Robbins 8th/142)
Analysis of X-linked recessive disorders
Males have an X and a Y chromosome. There is no corresponding locus for a mutant allele of the
X chromosome on the Y chromosome. The mutant recessive gene on the X chromosome
expresses itself in a male child because it is not suppressed by a normal allele whereas in
the female, the presence of a normal allele on other X-chromosome prevents the expression
of the disease so, females only act as carriers.Q
91. Ans. (a) Fraction of cells in apoptotic pathways (Ref: The journal of
Histochemistry & Cytochemistry: Volume 47 (5): 711-717, 1996: Wikipedia)
In situ DNA nick end – labeling is an in-situ method for detecting areas of DNA which are
nicked during apoptosis.
Terminal deoxynucleotidyl transferase mediated dUTP–biotin nick end labeling (TUNEL) is a
method for detecting apoptotic cells that exhibit DNA fragmentation.
92. Ans. (b) 25% (Ref: Harrison’s 17th/385)
Achondroplasia is an autosomal dominant condition.
Only one mutant allele is enough to cause disease. Thus, AA and Aa will be affected whereas aa
will be unaffected. [‘A’ is mutant allele whereas ‘a’ is normal].

As is clear from the diagram, 3 out of 4, i.e. 75% of children will be affected and 1 out of 4,
i.e. 25% children will be unaffected. However, please note that clinically the baby with AA
genotype usually donot survive.
93. Ans. (d) X-linked recessive type (Ref: Robbins 8th/142)
Presentation of disease only amongst males identifies the disorder as sex (X) linked. Because
carrier mothers are not manifesting the disease, yet their sons do, the disorder can only be
recessive. The disorder is thus X– linked recessive.
94. Ans. (a) X –linked recessive (Ref: Robbins 8th/142)
The given disease is manifesting only in males, therefore it is sex-linked disease. Females are not
affected, so they must be carriers. This is a classical inheritance feature of X-linked
recessive disorder.
Remember:
• Male is having XY, i.e. only X-chromosome. So, even if one mutant allele is present on X-
chromosome, it will manifest (whether recessive or dominant).
• Females are XX, so if one gene is mutated in X-chromosome, female will be phenotypically normal
and genotypically carrier, if inheritance is recessive but female will suffer from disease, if it is X-
linked dominant.
• There is no sex predilection in autosomal dominant or recessive disorders.

95. Ans. (a) None affected, all carriers


(Ref: Harrison 17/2332)
Albinism is an autosomal recessive (AR) disorder
• AR disorders express only in homozygous state, i.e. if both alleles are mutant
• If ‘A’ is normal allele and ‘a’ is mutant, then the given cross in the question can be made
as

• Thus genotypically all offsprings are carriers and Phenotypically, all of then will be
normal.
96. Ans. (a) 0 and 100% (Ref: Harrison 17th/637)
Sickle cell anemia is an autosomal recessive disorder.
• Sickle cell disease is the homozygous state of HbS (SS) where S stands for gene
coding HbS.
• Sickle cell trait is the heterozygous state of HbS (SA) where A stands for absent gene.
• Normal individual has no gene for HbS (AA)
If the mother has sickle cell disease ‘SS’ and father is normal ‘AA’ all the offsprings will be ‘SA’.
Thus % of sickle cell disease (SS) will be zero and that of sickle cell trait (SA) will be 100%.
97. Ans. (a) Blood group O (Ref: Harrison 17th/708)
Major blood group system is ABO system. These are based on the presence of antigen on surface
of RBCs Four blood groups according to this system are.
Blood Group Antigen Anti body (Isoagglutinins)
A A Anti – B
B B Anti – A
AB A and B None
O None Anti – A and Anti – B

• The genes that determine A and B phenotypes are found on chromosome 9p and are
expressed in a Mendelian co-dominant manner.
• AB group is universal recipient (No antibody) and Blood group O is universal donor (No
antigen)
• Blood group according to alleles can be
A AA or Ai
B BB or Bi
AB AB
O ii
[i means allele containing gene for no antigen]
• In the given question, father is blood group B [i.e. BB or Bi] and mother is AB. The cross
can be

• Thus, phenotypically blood group can be A (Ai), B (Bi, BB) or AB (AB). Thus, none of
the children can have O blood group.
98. Ans. (d) All of the above (Ref: Harrison 18th/547-551)
Gene transfer is a novel area of therapeutics in which the active agent is a nucleic acid sequence
rather than a protein or small molecule. Most gene transfers are carried out using a vector or
gene delivery vehicle because delivery of naked DNA or RNA to a cell is an inefficient
process. More clear-cut success has been achieved in a gene therapy trial for another form
of SCID, adenosine deaminase (ADA) deficiency. Other diseases likely to be amenable to
transduction of hemaopietic stem cells (HSCs) include
• Wiskott-Aldrich syndrome
• Chronic granulomatous disease
• Sickle cell disease
• Thalassemia.
Clinical trials using recombinant adeno-associated vectors are now ongoing for muscular
dystrophies, alpha-1 antitrypsin deficiency, lipoprotein lipase deficiency, hemophilia B, and a
form of congenital blindness called Leber’s congenital amaurosis.
99. Ans. (b) Chromosome 6 (Ref: Harrison 17th/2045)
100. Ans. (c) Chromosome 21 (Ref: Harrison 17th/644)
101. Ans. (c) Trans-differentiation (Ref: Robbins
7th/92 & Harrison 17th/426)
Stem cell is defined as a cell with a unique capacity to produce unaltered daughter cells (self-
renewal) and to generate specialized cell types (potency).
The prevailing paradigm in developmental biology is that once cells are differentiated, their
phenotypes are stable. However, tissue stem cells, which are thought to be lineage-
committed multipotent cells, possess the capacity to differentiate into cell types outside their
lineage restrictions (called trans-differentiation or stem cell plasticity). For example,
hematopoietic stem cells may be converted into neurons as well as germ cells.
102. Ans. (b) 25% (Ref: Robbins 8th/141, 7th/151)
Achondroplasia is an autosomal dominant disease.

So, only 1 out of 4 children will be unaffected, i.e. 25% of children will be normal.
103. Ans. (c) Parallel rays of tubular structure in lysosomes
(Ref: Robbin 8th/152-153)
• PAS stain is a widely used stain which gives positive reaction with glycogen (primarily)
and non glycogen substances like glycoprotein, glycolipids, proteoglycans and neutral
mucins.
• Whether the PAS positivity of a particular cell is due to presence of glycogen or due to
latter can be differentiated with diastase (glycogen digesting enzyme).
• If the cell is PAS positive due to glycogen, the pretreatment with diastase will make it
PAS negative. But if the cell is PAS positive due to non glycogen substances, the cell
will retain its PAS positivity even after pretreatment with diastase. So, in the given
question, the presence of PAS positive and diastase resistant material indicates
presence of non glycogen substances.
• The clinical feature of delayed milestones and hepatosplenomegaly in 1 year old boy is
suggestive of some lysosomal storage disorder (like Niemann Picks disease).
• This disease is characterized by the presence of large foam cells in bone marrow, liver
and spleen. There is presence of pleomorphic inclusion of lipids in lysosomes enclosed
in concentric or parallel lamellae.
104. Ans. (b) Homeobox gene (Ref: Robbins 8th/452-3)
Classes of genes known to be important in normal morphogenesis during
development include
• Homeobox genes (HOX): The HOX genes have been implicated in the patterning of
limbs, vertebrae, and craniofacial structures. HOX genes possess retinoic acid
response elements (RAREs), and that the latter are required for mediating both
physiologic and pathologic effects of retinoids during development. Mutations of
HOXD13 cause synpolydactyly (extra digits) in heterozygous individuals and
mutations of HOXA13 cause hand-foot-genital syndrome, characterized by distal limb
and distal urinary tract malformations.
• PAX genes: PAX genes are characterized by a 384 base pair sequence — the paired
box. They code for DNA-binding proteins that are believed to function as transcription
factors. In contrast to HOX genes, however, their expression patterns suggest that they
act singly, rather than in a temporal or spatial combination.
• Mutation in PAX3 causes Waardenburg syndrome (congenital pigment abnormalities and
deafness).
• Mutation in PAX6 causes Aniridia (congenital absence of the iris)
• PAX2 mutations cause the “renal-coloboma” syndrome (developmental defects of the
kidneys, eyes, ears, and brain).
• Translocations involving PAX3 and PAX7 are seen in alveolar rhabdomyosarcomas.
• Translocations involving PAX5 are seen in subsets of lymphomas
• Translocations involving PAX8 are seen in thyroid cancers.

Other options

FMR gene: It is involved in fragile X-syndrome.

PTEN gene: located on chromosome 10q is associated with endometrial cancers and
glioblastoma.(phosphatase and tensin homologue)

p16 blocks cell cycle and is a tumor suppressor gene.

105. Ans. (c) AB, Rh negative (Too obvious to explain friends)


106. Ans. (b) Splicing; (d) Frame-shift: (Ref: Harper’
27th/415, Robbins 8th/648-9)
Important causes of thalassemia are:
b-thalassemia: A wide variety of mutations in b-globin gene may cause this including
• Promoter region mutations
• Chain terminator mutations include frameshift mutations.
• Splicing mutations

b0-thalassemia, associated with total absence of b-globin chains in the homozygous state. Its
commonest cause is chain termination.

b+-thalassemia, characterized by reduced (but detectable) b-globin synthesis in the homozygous state.
Its commonest cause is splicing mutation.

a-thalassemia: Mutations in a-globin gene are mainly unequal crossing over and large deletions and less
commonly nonsense and Frame-shift mutations.

107. Ans. (a) Bloom syndrome; (b) Fanconi’s anemia; (d) Chediak-Higashi syndrome; (e)
Ataxia telangiectasia (Ref: Harrison 16th/643, 631, Robbins 7th/307)
Bloom’s syndrome, Fanconi anemia, Klinefelter syndrome, Ataxia telangiectasia and Kostman
syndrome are associated with myeloid leukemia.
108. Ans. (c) Loss of normal allele in mutant gene
(Ref: Robbins 7th/299)
“A child with inherited mutant RB allele in somatic cells is perfectly normal. Because such a child is a
heterozygous at the Rb locus, it implies that heterozygosity for the Rb gene does not affect cell behavior.
Cancer develops when the cell becomes homozygous for the mutant allele or in other words when the
cell loses heterozygosity for the normal Rb gene (a condition known as LOH loss of heterozygosity)”
Thus, from these lines we interpret that loss of heterozygosity means loss of normal allele in
mutant gene.
109. Ans. (b) q and p (Ref: Robbins 6th/166, 7th/171)
• Short arm of a chromosome is designated ‘p’ (for petit) and long arm is referred to as ‘q’
• In a banded karyotype, each arm of the chromosome is divided into two or more regions
by prominent bands.
• The regions are numbered (e.g. 1, 2, 3) from centromere outwards.
• Each region is further subdivided into bands and sub bands and these are ordered
numerically as well.
• Thus, the notation Xp 21.2 refers to a chromosomal segment located on the short arm of
the ‘x’ chromosome, in region 2, band 1 and sub band 2.

110. Ans. (c) 7 (Ref: Robbins 8th/465; 7th/490)


111. Ans. (b) Granulosa cell tumor (Ref: Robbins 8th/1338; )
112. Ans. (d) 7 (Ref: Robbins 9/e p466)
113. Ans. (a) Pseudomonas (Ref: Robbins 9/e p469)
Pseudomonas aeruginosa species, in particular, colonize the lower respiratory tract, first
intermittently and then chronically.
114. Ans. (c) X-linked dominant disease
(Ref: Robbins 8/e p142)
Since a male transmits only the ‘Y’ chromosome to his son, so obviously he cannot transmit any X
linked disease.
115. Ans. (a) Study of multiple genes (Ref: Robbins 8/e p174)
Microarrays are gene chips used to sequence genes or portions of genes. In this technique,
short sequences of DNA (oligonucleotides) that are complementary to the wild-type
sequence and to known mutations are “tiled” adjacent to each other on the gene chip, and
the DNA sample to be tested is hybridized to the array. Before hybridization the sample is
labeled with fluorescent dyes. The hybridization (and consequently, the fluorescent signal
emitted) will be strongest at the oligonucleotide that is complementary to wild-type sequence
if no mutations are present, while the presence of a mutation will cause hybridization to
occur at the complementary mutant oligonucleotide.
116. Ans. (a) Northern blot

• Norther n Blot : RNA North : Roti


• Souther n Blot : DNA South : Dosa
• Wester n Blot : Proteins West : Poha/Pizaa

117. Ans. (a) 50% of boys of carrier mother are affected


(Ref: Robbins 8/e p142)
Looking at the statements one by one:
Carrier mother contributes to the transfer of one of the two boys getting the affected chromosome.
So, 50% of boys of carrier mother are affected.
• Option “b”…All girls having diseased father are affected as carriers because they get
the affected ‘X’ sperm from their fathers. However, these girls are not going to manifest
the disease.
• Option “c”…fathers cannot transmit the disease to their sons as they transmit just the
“Y” chromosome to them whereas the disease is “X” linked.
• Option “d”… mothers contribute to just one “X” chromosome and so, 50% of the
daughters would become carriers if t other was a carrier.
118. Ans. (c) Chromosome 11
(Ref: Robbins 8/e p479-80)
Easiest way to remember that info….. count the number of letters in Wilms tumour..yea it is
exactly 11…the location of both genes associated with Wilms tumour
So, the two genes associated with Wilms tumour WT1 gene (located on chr 11p13)and WT2 gene
(located on chr 11p15).

119. Ans. (b) Rh-ve mother with 2nd Rh+ve child


• Rh-ve mother with 2nd Rh+ve child can result in the development of hemolytic disease
of newborn or erythroblastosis fetalis. So, it is a dangerous condition.
• This condition is a type II hypersensitivity reaction.
• This is Not to be confused with Hemorrhagic disease of the newborn which is a
coagulation disturbance in the newborns due to vitamin K deficiency. As a consequence
of vitamin K deficiency there is an impaired production of coagulation factors II, VII, IX,
X, C and S by the liver.

ANNEXURE

LOCATION OF IMPORTANT GENES ON CHROMOSOMES

Gene Chromosome
p73 1p
Folate transporter 21q
Neuroblastoma 1p
Rhodopsin 3
VHL 3p
ADPKD-2 4q
ADC 5p
MHC 6p
ARPKD 6
Cystic fibrosis 7q
MET 7
RET 10
WT-1 11p
vWF 12
Retinoblastoma 13q
BRCA-1 17q
Fibrillin-1 15
Fibrillin-2 5
BRCA-2 13q
NF-1 17q
p53 17p
NF-2 22q
1. A 25-year-old man presents for a routine physical examination. The patient is tall (6 ft. 5
in) and on examination he was found to have an early diastolic murmur. His family
pedigree is as given below. Which of the following is the mode of inheritance by
which the disease is likely to be transmitted?
(NEET 2020 like pattern)

(a) Autosomal dominant disorder


(b) Autosomal recessive disorder
(c) X-linked recessive disorder
(d) X-linked dominant disorder
Ans. (a) Autosomal dominant disorder
Few basic steps to be followed for taking care of these questions friends,
Since the disease affects both male and females, it is autosomal in nature and not sex linked.
• There is presence of one affected member in each generation and each affected
individual transmits the disease to at least 50% people in the next generation, so, it is
likely to be autosomal dominant.
• 2. Which of the following is the correct
statement related to Huntington’s Chorea? (NEET 2020 like pattern)
(a) It is a tri-nucleotide expansion mutation related disorder
(b) There is loss of function type of mutation
(c) There are abnormal repeats of CUG
(d) Abnormality is seen due to mutation in chromosome 6
Ans (a) It is a tri-nucleotide expansion mutation related disorder (Ref:
Robbins 9th/1297)
• Huntington disease is characterized by choreiform movements and progressive dementia that appear after the
age of 30.
• It is an autosomal dominant disorder that results from an abnormal gene (showing CAG repeats) on
chromosome-4.
• The expansion of the polyglutamine region bestows a toxic gain-of-function on huntingtin.
• There is degeneration of GABA neurons in the striatum, which leads to decreased function (decreased inhibition)
and increased movement.

3. Which of the following is autosomal recessive disorder?


(NEET 2019 like pattern)
(a) Huntington’s chorea
(b) Neurofibromatosis 1
(c) Marfan’s syndrome
(d) Sickle cell anemia
Ans. (d) Sickle cell anemia (Ref: Robbins 9th e/p 141)
4. Which of the following is autosomal dominant in inheritance? (NEET
2019 like pattern)
(a) Sickle cell disease
(b) Achondroplasia
(c) Hemochromatosis
(d) Wiskott Aldrich syndrome
Ans. (b) Achondroplasia
(Ref: Robbins 9th e/p 141, 1184)
• Achondroplasia is the most common skeletal dysplasia and a major cause of dwarfism.
• It is an autosomal dominant disorder resulting in retarded cartilage growth.
• It is caused by gain of function mutation in FGFR3 Receptor
• Affected individuals have shortened proximal extremities, a trunk of relatively normal length, and an enlarged
head with bulging forehead and conspicuous depression of the root of the nose.
• The skeletal abnormalities are usually not associated with changes in longevity, intelligence, or reproductive
status.

5. All are true regarding mitochondrial DNA except:


(NEET 2019 like pattern)
(a) Inherited from mother
(b) High prevalence
(c) Non classical Mendelian inheritance
(d) Kidneys, liver and brain are affected more
Ans. (b) High prevalence (Ref: Robbins 9th e/p 171)
• Mitochondrial inheritance is a non classical Mendelian inheritance associated with maternal inheritance.
• mtDNA encodes enzymes involved in oxidative phosphorylation, mutations affecting these genes exert their
deleterious effects primarily on the organs most dependent on oxidative phosphorylation such as the
central nervous system, skeletal muscle, cardiac muscle, liver, and kidneys.
• Diseases associated with mitochondrial inheritance are rare.

6. True about Fragile X syndrome is:


(NEET 2019 like pattern)
(a) Triple nucleotide CAG Sequence mutation
(b) 10% Female carriers mentally retarded
(c) Males have IQ 20-40
(d) Gain of function mutation
Ans. (c) Males have IQ 20-40 (Ref: Robbins 9th e/p 169)
• Fragile X syndrome is associated with “loss of function” mutation.
• There is presence of triplet repeat mutations of CGG nucleotides. The mutation affects
the FMR-1 gene (Familial Mental Retardation – 1 gene) present on the X chromosome.
• On karyotyping, the chromosome appears as broken (so, called fragile site). It is the
second most common cause of mental retardation.
• In fragile X syndrome, the affected males are mentally retarded, with an IQ in the
range of 20 to 60.
• The clinical features of patient include long face with a large mandible, large everted
ears and large testicles (macro-orchidism).
• 30% to 50% of carrier females are affected (i.e., mentally retarded).
7. Identify the inheritance pattern in the given pedigree: (AIIMS
Nov 2018 like pattern)
(a) Y linked recessive
(b) Y linked dominant
(c) X linked recessive
(d) X linked dominant
Ans. (c) X linked recessive
In the pedigree analysis as we can understand the following concepts:
1. See whether the disease is inherited as dominant or recessive trait. In dominant
inheritance, at least one member in all generations will have disease whereas in
recessive inheritance, there will be some generations without disease also. So, in the
given question, we have recessive means of inheritance.
2. Now see, whether it is sex-linked or autosomal by looking at the sex-predilection as
under:
• If the male is transmitting the disease only to daughters (all daughters), and not to the
sons whereas females transmit the disease to half daughters and half sons, it is X
linked dominant.
• If only males are affected in a pedigree (as is given in our question), it is likely to be
X linked recessive. (Females act as carriers, they transmit the disease to next
generation but they remain unaffected).
8. Which of the following is not a part of the quadruple test for antenatal detection of
Down syndrome?
(AIIMS Nov 2018 like pattern)
(a) AFP
(b) Estriol
(c) Beta hCG (d. Inhibin B
Ans. (d) Inhibin B (Ref: Robbins 9th e/p 175)
Quad screening is done at 15-20 weeks. It consists of the following:
• α-feto protein (Low)
• Estriol (Low)
• hCG (High)
• Inhibin-Alpha (High)
Inhibin-Alpha is also not a part of the triple test screening.
9. Abnormality in elastin protein can lead to all except: (AIIMS
May 2018 like pattern)
a. Joint laxity
b. Aortic aneurysm
c. Fractures d. Subluxation of lens
Ans. (c) Fractures Ref: Robbins 9th e/p 23,
The ability of tissues to recoil and recover their shape after physical deformation is conferred by
elastin. Morphologically, elastic fibers consist of a central core of elastin with an associated
mesh-like network composed of fibrillin. So, fibrillin synthetic defects lead to skeletal
abnormalities and weakened aortic walls, as in individuals with Marfan syndrome.
Marfan syndrome is a disorder of connective tissues, manifested principally by changes in the
following:
• Skeleton: Long extremities with lax joint ligaments in the hands and feet, long head with bossing of the frontal
eminences and prominent supraorbital ridges and spinal deformities (kyphosis, scoliosis, or rotation or slipping
of the dorsal or lumbar vertebrae). There may be presence of pectus excavatum or a pigeon-breast deformity.
• Eyes: Most characteristic is bilateral subluxation or dislocation usually outward and upward of the lens), and
• Cardiovascular system: Mitral valve proplase and dilation of ascending aorta.

10. Biopsy from an eight-year-old child with leg swelling showed small round blue tumor
cells consistent with diagnosis of Ewing’s sarcoma. What will be the best method to
detect translocation t(11;22) in this malignancy? (AIIMS May 2018 like
pattern)
(a) Conventional karyotyping
(b) Next generation sequencing
(c) FISH
(d) PCR
Ans. (c) FISH
The question is testing basics friends as in it, we have to choose a technique which can help us in
the detection of a specific translocation (11;22). For this purpose, FISH is the best technique
as it uses DNA probes that recognize sequences specific to particular chromosomal
regions. FISH is also used to detect:
• Numeric abnormalities of chromosomes (aneuploidy)
• Subtle microdeletions or complex translocations that are not demonstrable by routine
karyotyping; and
• Gene amplification (e.g., HER2 in breast cancer or NMYC amplification in
neuroblastomas).
11. Which of the following is X linked disease?
(AI 2018 Pattern)
(a) Thalassemia
(b) Galactosemia
(c) Color blindness
(d) Sickle cell disease
Ans. (c) Color blindness (Ref: Robbins 9/e p142)
12. Which of the following is a tool used in gene editing?
(AIIMS May 2017 Pattern)
(a) CRISPR
(b) Big Data
(c) Gene Xpert
(d) HealthCare App
Ans. (a) CRISPR (Ref: Robbins 9/e p28-29)
Genomic editing is a process using a nuclease called Cas9 that was originally identified in
prokaryotes that can be used together with guide RNAs called CRISPRs (Clustered
Regularly Interspaced Short Palindromic Repeats) to selectively alter or correct DNA
sequences, such as disease-causing mutations.
Disclaimer
Any resemblance to an actual question is purely coincidental.
• Willis gave the definition of neoplasm.
• Excessive fibrosis in tumor is called as desmoplasia. It is responsible for linitis plastic
appearance of stomach cancer.
• Abnormal differentiation at normal site: Hamartoma. It is now considered as a
neoplasm.
• Normal differentiation at abnormal (ectopic) site: Choristoma.
• Replacement of a mature (differentiated) cell type into other mature cell is called
Metaplasia. Squamous metaplasia in the lungs of smokers is the commonest
example.
• Lack of differentiation is called Anaplasia. It has irreversible loss of polarity.
• Disordered differentiation is called dysplasia. It has reversible loss of polarity.
• Carcinoma in situ and invasive carcinoma are differentiated by: involvement of
basement membrane (penetration is seen in invasive carcinoma).
• Most reliable sign of malignancy is Metastasis.
• 2nd most reliable sign of malignancy: Local invasion.
• Sarcomas metastalize usually by hematogenous route whereas carcinomas
metastatize usually by lymphatic route.
• Cancer cells derive energy from Aerobic glycolysis (Warburg effect).
• Most common target for genetic alteration in human tumor: p53.
• DNA damage by irradiation/UV light causes: Increase in p53 level and cell cycle
arrest in G1 (due to blockade at G1S transition).
• Mutated form of p53 gene associated with cancer whereas the non-mutated form
(wild form) is associated with reduced risk of development of cancer.
• p53 acts through: CDK inhibitor p21 (p53 itself is not a CDK inhibitor).
• p53 acts at: G1S check point.
• Retinoblastoma (RB) gene regulates: G1- S transition.
• Important autosomal dominant familial cancers: FAP, Wilms tumor, breast/ovary
cancers, retinoblastoma, MEN-1 and 2, Neurofibromatosis 1 and 2.
• Defective DNA repair syndromes predisposing to cancer: Bloom syndrome,
Fanconi anemia, Ataxia telangiectasia and Xeroderma pigmentosa.
• BRCA-1 is located on chromosome I7q whereas BRCA-2 is located on Chromosome
13q
• Retinoblastoma gene is located on: Chromosome 13q14.
• Conditions predisposing to cancer include regeneration, hyerplasia, dysplasia,
metaplasia, chronic inflammation and atrophy. Hypertrophy does not cause cancer.
• The detachment of epithelial cells from basement membranes and from cell-cell
interactions can lead to a particular form of cell death called anoikis.
• COX2 inhibitors decrease the incidence of colonic adenomas and are now approved
for treatment of patients with familial adenomatous polyposis
• All trans-retinoic acid is a highly effective therapy is the first example of
differentiation therapy, in which immortal tumor cells are induced to differentiate
into their mature progeny, which have limited life spans. It is used for treating patients
with acute promyelocytic leukemia.
• Chromothrypsis is a process in which a chromosome is “shattered” and then re-
assembled in a haphazard way. It is seen in is found in up to 25% of osteosarcomas
and at a relatively high frequency in gliomas as well.
• Tumor markers can be used for screening to assess response of therapy/surgery,
follow- up (to see recurrence), prognosis and to assess the growth. They cannot be
used to confirm diagnosis.
• Marker of testicular tumors: AFP, HCG, placental alkaline phosphatase, placental
lactogen, LDH.
• AFP (alpha feto-protein) is genetically and structurally related to Albumin.
• Important markers for different cancers: CD-99 (Ewing’s sarcoma), Placental alkaline
phosphatase (Seminoma of testis), Cytokeratin(carcinoma), CD34 (Alveolar soft part
sarcoma) Desmin (rhabdomyosarcoma/leiomyosarcoma) and ‘BRAF’ gene mutation
(Melanoma)
• SYT-SSX1 and SYT-SSX2 genes are associated with synovial sarcoma.
• Mutagenicity test is called as Ames test.
• Most important predictor of mesenchymal tumors: Grade.
• Commonest carcinoma in male is carcinoma of Lung. However, the commonest
carcinoma in elderly male is Prostate.
• Cachexia in tumor is due to TNF.
• Migratory thrombophlebitis is seen in Pancreas (most common), lung (2nd
common), GIT, Prostate, breast, ovary, brain and lymphomas.
• Most common hormone producing hypercalcemia in tumors: Parathormone related
peptide.
• Features of tumor lysis syndrome include: Hypocalcemia, hyperkalemia,
hyperuricemia, hyperphosphatemia and lactic acidosis. It does not include
hypercalcemia
• Spontaneous regression is seen in the following tumors: Neuroblastoma, malignant
melanoma and retinoblastoma.
• Exfoliative cytology is useful in: Cancers of cervix, endometrium, lung
(bronchogenic), bladder, prostate and stomach.
• Intraoperative histopathological analysis can not be used for immediate definite
diagnosis of a cancer. It is only useful for detecting positive margins after resection
and is used to confirm suspected metastasis.
• Lewis Thomas and Macfarlane Burnet coined the term immune surveillance,
which implies that a normal function of the immune system is to constantly “scan” the
body for emerging malignant cells and destroy them.
NEOPLASIA

Neoplasia refers to the process of new growth. The important feature of the growth associated
with neoplasms is the fact that it is an uncoordinated growth of the tissue persisting even after
the cessation of the stimulus which evoked the change. Oncology is the study of tumors or
neoplasms. The tumors are usually composed of the:

1. Parenchyma – Made up of proliferating neoplastic cells


2. Stroma – Made up of connective tissue and blood vessels.

Benign Tumors
These are usually denoted by adding a suffix “- oma” to the cell of origin, so, these may be
arising from fibroblastic cells (fibroma); cartilage cells (chondroma) or osteoblasts (osteoma).
Adenoma and Papilloma are examples of benign tumors.

MALIGNANT TUMORS

Cancer is a generalized term used for all malignant tumors. These tumors can be of the
following types:
1. Sarcoma - Arising from mesenchymal tissue.
2. Carcinoma – Tumor of epithelial cell origin derived from any germ layer. If this tumor is
having a glandular pattern, it is called adenocarcinoma.

The divergent differentiation of parenchymal cells produces mixed tumors or pleomorphic tumors.
In teratoma, parenchymal cells are made up from more than one germ layer e.g. Dermoid cyst.

Fig. 1: Dermoid cyst


Characteristics of Neoplasia

I. Anaplasia: The extent to which neoplastic cells resemble normal cells both
morphologically and functionally is called differentiation. An absence of differentiation is
called anaplasia.
The features of anaplasia are:
1. Pleomorphism – It is the variation in the size and shape of the cells.
2. Hyperchromasia – Increased nuclear material or DNA is responsible for dark staining
of the cells called hyperchromasia. In normal cells, the nuclear cytoplasmic (or N: C)
ratio is 1:4 whereas it becomes 1: 1 in anaplastic cells.
3. Increased mitosis gives rise to atypical bizarre mitotic figures.

Fig. 2: Tripolar mitotic spindle in anaplasia

4. Loss of polarity due to disturbed orientation of anaplastic cells.


5. Presence of tumor giant cells having big hyperchromatic nuclei.
Key differentiating features between Metaplasia, Dysplasia and Anaplasia

Feature Metaplasia Dysplasia Anaplasia

Definition Reversible change A change having loss in the An absence of


in which one uniformity of the individual cells differentiation (extent to
differentiated cell and loss of architectural which neoplastic cells
type (epithelial or orientation resemble normal cells both
mesenchymal) is morphologically and
replaced by another functionally)
cell type

Pleomorphism Absent PresentQ in low grade PresentQ in high grade


(variation in the size
and shape of
cells/nuclei)

Reversibility ReversibleQ Reversible in early stages IrreversibleQ


(irreversible if whole epithelium is
involved)

N:C ratio Normal (1:4) Increased (↑) Increased (↑↑↑)

Hyperchromatism Absent Present (small degree) Present (high degree)

Mitotic figures Absent/minimal at Typical mitotic figuresQ present Atypical mitotic figuresQ
normal places at abnormal places (multipolar spindles)
present at abnormal places

Other features Absent Absent PresentQ


(Tumor giant cellsQ,
hemorrhage, necrosis)

Example Barret’s esophagus, Cervical dysplasia of squamous Carcinoma of the cervix,


myositis ossificans cells carcinoma esophagus

II. Rate of growth: The growth of a tumor correlates with the level of differentiation. Well-
differentiated tumors have a slow proliferation rate. Recently, cancer stem cells or tumor-
initiating cells (T-ICs) have been identified in breast cancer, glioblastoma multiforme (a
brain tumor), and acute myeloid leukemia. These T-ICs are cells that allow a human tumor
to grow and maintain itself definitely when transplanted into an immunodeficient mouse.
III. Local invasion: It is the second most reliable feature that differentiates malignant from
benign tumors. The benign tumors are slow growing, cohesive, expansile masses that are
usually capsulated. The malignant tumors show invasion, infiltration and destruction of the
surrounding tissue.

IV. Metastasis: It is the most reliable feature of a malignant tumor, characterized by the
spread of the tumor to other parts because of penetration into blood vessels, lymphatics
and the body cavities.

Pathways of spread
Direct seeding of body
cavities and surfaces Lymphatic spread Hematogenous sprad
• Involvement of • Usual for carcinomas • Usual for sarcomas
peritoneal cavity (in • Pattern of lymph node • Thinner walls of veins
ovarian cancer). involvement follows causes preferential venous
• Surface of abdominal natural route of lymphatic invasion in comparison to
viscera (mucus drainage. arteries.
secreting appendicle • Sentinel lymph node is • Liver and lungs are the
cancer causing the first lymph node to most commonly affected
pseudomyxoma receive lymph flow from a organs.
peritonei). primary tumor. • Invasion of veins is
• Seen in melanoma, particularly prominent is
breast and colon cancer. hepatocellular cancer and
renal cell cancer.

PRECURSOR LESIONS

Precursor lesions are localized morphologic change that are associated with a high risk of cancer. They can be
of the following subtypes:

Cancer-Enabling Inflammation
Inflammatory cells increase the risk of cancers by different mechanisms like:
i. Release of factors that promote proliferation
ii. Removal of growth suppressors,
iii. Enhanced resistance to cell death,
iv. Inducing angiogenesis,
v. Activating invasion and metastasis
vi. Evading immune destruction

GENETIC PREDISPOSITION TO CANCER/INHERITED CANCER SYNDROMES

I. Autosomal dominant cancer syndrome


– Inheritance of a single mutant gene increases the risk of development of cancer.
– Can be remembered as
Very - Von Hippel Lindau (VHL) syndrome causing renal cell cancer
Rich - Retinoblastoma
Cute and - Cowden syndrome (PTEN gene)
Nice - Neurofibromatosis 1, 2; Nevoid basal cell cancer syndrome
Men - Melanoma: MEN (Multiple Endocrine Neoplasia) 1 and 2
Hereditarily - Hereditary nonpolyposis colon cancer
Like - Li-Fraumeni syndrome, LKB1 gene in Peutz Jeghers syndrome
Familiar - Familial adenomatous polyposis
Females - ovarian and breast tumor (will occur is females obviously)
II. Familial cancers
– These are cancers occurring at high frequency in families without a clear defined
pattern of transmission. These usually show early age of onset and are present is 2
or more close relatives of the index case.
– Include cancer of colon, ovary, breast, pancreas, etc.
III. Autosomal recessive cancer syndrome (all of these are caused due to Defective DNA
Repair)

The seven fundamental changes in cell physiology that together determine the malignant
phenotype are:
1. Self-sufficiency in growth signals: Due to oncogene activation
2. Insensitivity to growth-inhibitory signals
3. Evasion of apoptosis: Presence of resistance to apoptosis
4. Limitless replicative potential: Presence of unrestricted proliferative capacity due to
telomerase activity
5. Development of sustained angiogenesis
6. Ability to invade and metastasize
7. Genomic instability resulting from defects in DNA repair
8. Aerobic glycolysis (Warburg effect).

CELL CYCLE AND CONTROL MECHANISMS

• Cell cycle consists of five phases. These include:


G1 : Pre-synthetic phase
S : Synthetic phase (DNA synthesis)
G2 : Post-synthetic pre-mitotic phase
M : Mitotic phase: Cells divide and produce new cells, which either directly re-enter next cycle or
pass into non- proliferative G0 phase
G0 : Quiescent state: Cells in this state remain quiescent for variable periods, but can be recruited in
cell cycle if stimulated later.
• Resting (non-dividing) cells are in the G0 stage of the cell cycle and must enter G1 stage for
replication. The orderly progression of cells through the various phases of cell cycle is
controlled by cyclins and cyclin-dependent kinases (CDKs), and by their inhibitors.
• CDKs are expressed constitutively during the cell cycle but in an inactive form whereas
cyclins are synthesized during specific phases of the cell cycle, and their function is to
activate the CDKs.
• Cyclins D, E, A, and B appear sequentially during the cell cycle and bind to one or more
CDKs.

• During the G1 phase of the cell cycle, cyclin D binds to and activates CDK4, forming a
cyclin D-CDK4 complex. This complex has a critical role in the cell cycle by
phosphorylating the retinoblastoma susceptibility protein (RB). The phosphorylation of RB
is a molecular ON-OFF switch for the cell cycle. Phosphorylation of RB results in activation
of transcription factor E2F. Activated E2F results in transcription of target genes like cyclin
E, DNA polymerases, thymidine kinase, dihydrofolate reductase etc.

• Further progression through the S phase and the initiation of DNA replication involve the
formation of an active complex between cyclin E and CDK2. The next decision point in the
cell cycle is the G2/M transition. This transition is initiated by the E2F-mediated
transcription of cyclin A, which forms the cyclin A-CDK2 complex that regulates events at
the mitotic prophase. The main mediator that propels the cell beyond prophase is the
cyclin B-CDK1 complex, which is activated by a protein phosphatase (Cdc 25). Cyclin B-
CDK1 activation causes the breakdown of the nuclear envelope and initiates mitosis.

Cell-Cycle Inhibitors
The activity of cyclin-CDK complexes is tightly regulated by inhibitors, called CDK inhibitors.
There are two main classes of CDK inhibitors: the Cip/Kip and the INK4/ARF families.
• The Cip/Kip family has three components, p21, p27, and p57, which bind to and inactivate
the complexes formed between cyclins and CDKs. Transcriptional activation of p21 is
under the control of p53
• The human INK4a/ARF locus (a notation for “inhibitor of kinase 4/alternative reading
frame”) encodes two proteins, p16INK4a and p14ARF, which block the cell cycle and act
as tumor suppressors. p16INK4a competes with cyclin D for binding to CDK4 and inhibits
the ability of the cyclin D-CDK4 complex to phosphorylate RB, thus causing cell-cycle
arrest at late G1 whereas p14ARF prevents p53 degradation.

Cell-Cycle Checkpoints
The cell cycle has its own internal controls, called checkpoints. There are two main
checkpoints, one at the G1/S transition and another at G2/M.
• The S phase is the point of no return in the cell cycle, and before a cell makes the final
commitment to replicate, the G1/S checkpoint checks for DNA damage. If DNA damage is
present, the DNA repair machinery gets activated and arrests the cell cycle. If the damage
is not repairable, apoptotic pathways are activated to kill the cell. Thus, the G1/S
checkpoint prevents the replication of cells that have defects in DNA.

• DNA damaged after its replication can still be repaired as long as the chromatids have not
separated. The G2/M checkpoint monitors the completion of DNA replication and checks
whether the cell can safely initiate mitosis and separate sister chromatids. This checkpoint
is particularly important in cells exposed to ionizing radiation.

• To function properly, cell-cycle checkpoints require sensors of DNA damage, signal


transducers, and effector molecules. In the G1/S checkpoint, cell-cycle arrest is mostly
mediated through p53, which induces the cell-cycle inhibitor p21. Arrest of the cell cycle by
the G2/M checkpoint involves both p53-dependent (via cyclin A/cdK-2) and independent
(via cdc 25) mechanisms.
• p53 links cell damage with DNA repair, cell-cycle arrest, and apoptosis. In response to
DNA damage, it is phosphorylated by genes that sense the damage and are involved in
DNA repair. p53 assists in DNA repair by causing G1 arrest and inducing DNA repair
genes. A cell with damaged DNA that cannot be repaired is directed by p53 to undergo
apoptosis. With homozygous loss of p53, DNA damage goes unrepaired and mutations
increase the chances of malignant transformation.

Regulation of Cell Cycle


CARCINOGENESIS
It is a multi-step process which requires accumulation of multiple genetic changes either as
germline or somatic mutations. The following four are the principal targets of genetic damage:
I. Proto-Oncogenes
Proto-oncogenes (Normal genes required for cell proliferation and differentiation)
Oncogenes (Genes promoting autonomous cell growth in cancer cells)
Oncoproteins (Proteins lacking regulatory control and responsible for promoting cell
growth)

Selected Oncogenes, their mode of activation and associated human tumors


Category Proto- Mode of activation Associated human tumor
oncogenes
Growth factors
PDGF-b chain SIS Overexpression Astrocytoma
Osteosarcoma

Fibroblast growth HST-1 Overexpression Stomach cancer


INT-2 Amplification Bladder cancer
factors
Breast cancer
Melanoma
TGF-a TGFA Overexpression Astrocytomas
Hepatocellular carcinomas
HGF HGF Overexpression Thyroid cancer
Growth factor: Receptors
EGF-receptor family ERB-B1 Overexpression Squamous cell carcinomas of lung,
(EGFR) Amplification gliomas
ERB-B2 Breast and ovarian cancers
CSF-1 receptor FMS Point mutation Leukemia
Receptor for neurotrophic RET Point mutation Multiple endocrine neoplasia 2A and
factors B, familial medullary thyroid
carcinomas
PDGF receptors PDGF-R Overexpression Gliomas
Receptor for stem cell KIT Point mutation Gastrointestinal stromal tumors and
(steel factor) other soft tissue tumors
Proteins involved in signal transduction
GTP-binding K-RAS Point mutation Colon, lung and pancreatic tumors
H-RAS Point mutation Bladder and kidney tumors
N-RAS Point mutation Melanomas, hematologic
malignancies

Non-receptor tyrosine ABL Translocation Chronic myeloid leukemia


Acute lymphoblastic leukemia
kinase
RAS signal transduction BRAF Point mutation Melanomas
WNT signal transduction b-catenin Point mutation Hepatoblastomas, hepatocellular
Overexpression carcinoma
Nuclear regulatory proteins

Transcriptional activators C-MYC Translocation Burkitt lymphoma


N-MYC Amplification Neuroblastoma, small cell carcinoma
L-MYC Amplification of lung
Small cell carcinoma of lung

Cell-cycle regulators

Cyclins CYCLIN D Translocation Mantle cell lymphoma


CYCLIN E Amplification Breast and esophageal cancers
Overexpression Breast cancer

Cyclin dependent kinase CDK4 Amplification or point Glioblastoma, melanoma, sarcoma


mutation

The Important Oncogenes Include

1. RAS – It is an example of signal transducing protein. Normally, inactive RAS binds to GDP
and the presence of growth factor causes GDP to be exchanged by GTP causing RAS
activation. The activated RAS binds to its farnesyl transferase receptor causing increased
activation of MAP kinase and promoting mitogenesis. The activated RAS comes back to its
normal inactive state due to the intrinsic GTPase activity which gets augmented due to a
group of proteins called GTPase Activating Proteins (or GAP). Mutated RAS proteins bind
to GAP without augmentation of GTPase activity resulting is uncontrolled mitogenesis and
tumor formation.

The different types of RAS affected in different tumors are:


K – RAS Colon, lung and pancreatic tumors
N – RAS Melanoma, blood tumors (AML)
H – RAS Bladder and kidney tumors
2. ABL – It possess non-receptor associated tyrosine kinase activity. C-ABL present on
chromosome 9 fuses with BCR gene on chromosome 22 and result in Philadelphia
chromosome (t 9;22). The fusion gene possesses uncontrolled tyrosine kinase activity
responsible for causing cancer development.
3. MYC – Normally in the presence of growth factors, increased levels of MYC along with
another protein, MAX form a heterodimer and cause activation of transcription.
Uncontrolled nuclear transcription gives rise to development of cancer. In the absence of
growth factor, MYC can cause apoptosis. The types of MYC and the tumors associated
with their mutation include:
a. C – MYC - Burkitt’s lymphoma
b. L- MYC - Small cell lung cancer [L- for lung]
c. N- MYC - Neuroblastoma [N for Neuroblastoma]

• Loss of function mutations in RET oncogene result in intestinal aganglionosis and Hirschsprung
disease
• Gain of function mutations in RET oncogene result in Multiple Endocrine Neoplasia (MEN 2A/2B)
syndromes.

II. Tumor Suppressor Genes


These genes normally regulate cell growth (they do not prevent tumor formation, so the
name is actually a misnomer). Any failure of growth regulation causes development of
cancer.

Selected Tumor Suppressor Genes involved in Human Neoplasms

Tumors associated Tumors associated


Subcellular with Somatic with Inherited
Location Gene Function Mutations Mutations

Cell surface TGF-b Growth inhibition Carcinoma of colon Unknown


receptor Cell adhesion Carcinoma of stomach Familial gastric cancer
E-cadherin

Inner aspect of NF-1 Inhibition of RAS signal Neuroblastoma Neurofibromatosis type


plasma transduction and of p21 1 and sarcomas
membrane cell-cycle inhibitor

Cytoskeleton NF-2 Cytoskeleton stability Schwannomas and Neurofibromatosis type


meningiomas 2, acoustic
schwannomas and
meningiomas

Cytosol APC/b- Inhibition of signal Carcinomas of stomach, Familial adenomatous


catenin transduction colon, pancreas; polyposis coli/colon
PTEN PI-3 kinase signal melanoma cancer
SMAD 2 and transduction Endometrial and Unknown
SMAD 4 TGF-b signal prostate cancers Unknown
transduction Colon, pancreas tumors

Nucleus RB Regulation of cell cycle Retinoblastoma; Retinoblastomas,


osteosarcoma, osteosarcoma
carcinomas of breast,
colon, lung

P53 Cell-cycle arrest and Most human cancers Li-Fraumeni syndrome;


apoptosis in response to multiple carcinomas and
DNA damage sarcomas

WT-1 Nuclear transcription Wilms tumor Wilms tumor

P16 (INK4a) Regulation of cell cycle Pancreatic, breast and Malignant melanoma
by inhibition of cyclin- esophageal cancers
dependent kinase

BRCA-1 and DNA repair Unknown Carcinoma of female


BRCA-2 breast and ovary;
carcinomas of male
breast

KLF-6 Transcription factor Prostate Unknown

Some important tumor suppressor genes include:


1. RB gene
– In the active or hypophosphorylated state, it is present in the non- multiplying cells.
The oncogenic viruses cause phosphorylation of RB resulting in its inactivation. The
RB inactivation results in cell multiplication by activation of the transcription factor E2F
(as discussed eariler under control mechanism of cell cycle).
Concept of Loss of Heterozygosity (LOH)

Knudson’s Two Hit Hypothesis


According to this hypothesis, both the normal alleles should be inactivated for the development
of retinoblastoma. Retinoblastoma can be of the following types:

a. Inherited/Familial Retinoblastoma (40%)


In hereditary cases of Retinoblastoma one genetic change (first hit) is inherited from the affected parent
therefore it is present in all the somatic cells of the body. But one genetic change is not sufficient to produce
cancer. The second mutation (second hit) is required to produce cancer. It occurs in the retinal cells (which
are carrying the first mutation).

b. Sporadic Retinoblastoma (60%)


In sporadic cases both mutations (hits) occur somatically within a single retinal cell whose progeny then form the
tumor.

2. p 53 gene
It is also known as ‘molecular policeman’ and ‘guardian of the genome’. The gene is
present on chromosome 17p. The non-mutated p53 gene is also called as the ‘wild type’
of p53 gene and is associated with reduced risk of development of cancers. Any
inactivation of p53 prevents successful DNA repair in a cell leading to the development of
a tumor. The p53 gene codes for 53 KDa nuclear phophoprotein.
– In most of the cases, the inactivating mutations in both the alleles of p53 are acquired
in somatic cells. However, sometimes individual may inherit one mutant p53 allele and
the second acquired ‘hit’ may inactivate the normal p53 allele. This later condition is
called Li-Fraumeni syndrome associated with development of sarcoma, breast
cancer, leukemia and brain tumors.
– Human papilloma virus (HPV) causes inactivation of p53 through its E6 protein and
so, is responsible for development of cancer of anal and genital region.
Note: p73 (big brother of p53) and p63 are other members of the family of p53 gene. p63 is esential for the
differentiation of stratified squamous epithelia. p73 has pro-apoptotic effects after DNA damage induced by
the chemotherapeutic agents.

3. NF-1 and NF2 gene


NF- 1 gene gives rise to neurofibromin which is a GTPase activating protein (GAP). NF2
gives rise to neurofibromin 2 or merlin protein which inhibits the proliferation of Schwann
cells. So, any mutation affecting any of these genes increases the chances of
development of cancer.
Fig. 3: Neurofibromatosis-1
III. Genes Regulating Apoptosis
Apoptosis (or programmed cell death) is promoted by the genes bax, bad, bcl- Xs and p53
whereas it is inhibited by bcl-2. Understandably, any increase in bcl-2 would cause
inhibition of apoptosis and development of cancer. Normally chromosome 14 has
immunoglobulin heavy chain gene whereas chromosome 18 has bcl-2 gene. In
follicular lymphoma, there is presence of translocation t (14:18)Q which causes
increased expression of bcl-2 thereby preventing apoptosis and inducing the
development of cancer.

IV. Genes Inhibiting DNA Repair


Defective DNA repair increases DNA instability increasing the chances of development of
a cancer. This can be of the following three types:
MULTI-STEP CARCINOGENESIS

The normal epithelium undergoes sequential mutations in different genes eventually leading to
development of carcinoma.

Fig. 4: Hyperproliferative epithelium in polyp

TUMOR GROWTH

SPREAD OF TUMORS
1. Detachment of tumor cells
– Due to ↓ E-cadherin and
abnormal catenin

2. Attachment to matrix
component fibronectin and
laminin due to integrins (on
cancer cells)

3. Degradation of extracellular
matrix by serine, cysteine
and matrix
metalloproteinases

4. Intravasation

5. Tumor cell embolus

6. Extravasation – CD44 on T
cells used by tumor cells
for migration in lymphoid
tissue

7. Metastatic deposit

ETIOLOGY OF CANCERS

I. Chemical Carcinogens
Chemical carcinogenesis has two steps called initiation and proliferation. Initiation can be
by two types of agents

a. Direct acting agents – These are mutagens causing cancer by direct damage or
modification of DNA.
b. Indirect acting agents (also called as procarcinogens) – These require metabolic
conversion to form active carcinogens.
Initiators cause irreversible DNA damage. The proliferation of the tumor cells is done
by promoters (chemicals causing multiplication of already mutated cells). The promoters
cause reversible DNA damage. The carcinogenic potential of a chemical is tested by Ames
test.
Contd...
Contd...

Chemical Carcinogens

Acute myeloid
leukemia, bladder
Alkylating agents cancer
Androgens Prostate cancer
Aromatic amines (dyes) Bladder cancer
Arsenic Cancer of the lung, skin
Asbestos Cancer of the lung,
pleura, peritoneum
Benzene Acute myelocytic
leukemia
Chromium Lung cancer
Diethylstilbestrol (prenatal) Vaginal cancer (clear cell)
Estrogens Cancer of the
endometrium, liver,
breast
Ethyl alcohol Cancer of the liver,
esophagus, head and
neck
Immunosuppressive agents (azathioprine, cyclosporine, glucocorticoids) Non-Hodgkin’s lymphoma

Nitrogen mustard gas Cancer of the lung, head


and neck, nasal sinuses
Nickel dust Cancer of the lung, nasal
sinuses
Oral contraceptives Bladder and cervical
cancer
Phenacetin Cancer of the renal pelvis
and bladder
Polycyclic hydrocarbons Cancer of the lung, skin
(especially squamous cell
carcinoma of scrotal skin)
Sunlight (ultraviolet) Skin cancer (squamous
cell and melanoma)
Tobacco (including smokeless) Cancer of the upper
aerodigestive tract,
bladder
Vinyl chloride Liver cancer
(angiosarcoma)

Infectious Organisms

BACTERIA (H. PYLORI)


H. pylori is the first bacterium classified as a carcinogen. H. pylori infection is implicated in the
genesis of both gastric adenocarcinomas and gastric lymphomas. The gastric lymphomas are
of B-cell originQ and are called MALT lymphomas (marginal zone-associated lymphomas)
because the transformed B cells normally reside in the marginal zones of lymphoid follicles. H.
pylori infection results in the formation of H. pylori-reactive T cells, which cause polyclonal B-
cell proliferations. The MALT lymphoma is associated with t(11;18)Q translocation.
VIRUSES
Carcinogenic Viruses
RNA viruses DNA viruses
• Human T cell leukemia virus-1 (HTLV-1) • Hepatitis B virus (HBV)
• Hepatitis C virus (HCV) • Human herpes virus 8 (HHV8)
• Human papilloma virus (HPV)
• Epstein-Barr virus (EBV)

Pathogenesis

HTLV-1
It is a RNA oncogenic virus which is transmitted by blood products, sexual intercourse or
breastfeeding. It has attraction for CD4 T cells (similar to HIV). HTLV-1 has a gene TAX. The
TAX proteinQ causes

1. Transcription of host genes involved in proliferation and differentiation of T-cells (e-FOS,


IL-2 genes)
2. Genomic instability by inhibiting DNA repair function and by inhibiting cell cycle
checkpoints activated by DNA damage
These contribute to increased chances of cancer by HTLV-1.

HCV
Hepatitis C virus (HCV) is also strongly associated with the development of hepatocellular
carcinoma. This is associated with its ability to cause chronic liver cell injury and inflammation
that is accompanied by liver regeneration. Mitotically active hepatocytes, surrounded by an
altered environment, are presumably prone to genetic instability and cancer development.

EBV
It is a DNA oncogenic virus. It causes infection of epithelial cells of oropharynx and B- cells
because of the presence of CD21 moleculeQ on the surface of these cells. LMP-1 geneQ
present in the EBV causes activation of NF-κβ and JAK/STAT signaling pathways thereby
promoting B-cell survival and proliferation. (This increases the chances of B- cell lymphoma).
Another EBV-encoded gene, EBNA-2, transactivates several host genes like cyclin D and the
src family genes. The EBV genome also contains a viral cytokine, vIL-10 which prevents
macrophages and monocytes from activating T cells and is required for EBV-dependent
transformation of B cells. EBV acts a polyclonal B-cell mitogen followed by acquisition of
t(8;14)Q translocation which ultimately results in development of Burkitt’s lymphoma.

*EBV belongs to the herpes family and can cause the following cancers:
• African form of Burkitt’s lymphoma
• B- cell lymphoma in immunosuppressed (post transplant) individuals
• Hodgkin’s lymphoma
• 1° CNS diffuse large B-cell lymphoma
• Nasopharyngeal cancer

HBV
It encodes for HBx proteinQ which disrupts the normal growth control of infected liver cells by
activation of several growth promoting genes. HBx also causes inactivation of the tumor
suppressor gene p53. This results in HBV causing hepatocellular cancer.

HPV
It is responsible for development of squamous cell carcinoma of cervix and anogenital lesion
and in some cases, oral and laryngeal cancers. The virus gets integrated in the genome of
host cells which is essential for the malignant transformation of the affected cells HPV 16
(more commonly) and HPV 18 (less commonly) are particularly important in carcinogenesis as
they have viral genes E6 and E7 which causes Rb and p53 gene inactivation respectively.
Since both p53 and Rb are tumor suppressor genes, so, their inactivation increases the
chances of cancer development.

PARANEOPLASTIC SYNDROMES

Clinical syndromes Major forms of underlying Causal mechanism


cancer
Endocrinopathies
Cushing syndrome Small cell carcinoma of lung ACTH or ACTH-like substance
Pancreatic carcinoma
Neural tumors
Syndrome of inappropriate antidiuretic Small cell carcinoma of lung; Antidiuretic hormone or atrial
hormone secretion Intracranial neoplasms natriuretic hormones
Hypercalcemia Squamous cell carcinoma of Parathyroid hormone-related
lung protein (PTHRP), TGF-a, TNF,
Breast carcinoma IL-1
Renal carcinoma
Adult T-cell leukemia/lymphoma
Ovarian carcinoma
Hypoglycemia Fibrosarcoma Insulin or insulin-like
Other mesenchymal sarcomas substance
Hepatocellular carcinoma
Carcinoid syndrome Bronchial adenoma (carcinoid) Serotonin, bradykinin
Pancreatic carcinoma
Gastric carcinoma
Polycythemia Renal carcinoma Erythropoietin
Cerebellar hemangioma
Hepatocellular carcinoma
Nerve and Muscle Syndromes
Myasthenia Bronchogenic carcinoma Immunologic
Disorders of the central and peripheral Breast carcinoma
nervous systems
Dermatologic Disorders
Acanthosis nigricans Gastric carcinoma Immunologic; secretion of
Lung carcinoma epidermal growth factor
Uterine carcinoma
Dermatomyositis Bronchogenic, breast carcinoma Immunologic
Osseous, Articular, and Soft Tissue Changes
Hypertrophic osteoarthropathy and clubbing Bronchogenic carcinoma Unknown
of the fingers
Vascular and Hematologic Changes
Venous thrombosis (Trousseau Pancreatic carcinoma Tumor products (mucins that
phenomenon) Bronchogenic carcinoma activate clotting)
Other cancers
Nonbacterial thrombotic endocarditis Advanced cancers Hypercoagulability
Anemia Thymic neoplasms Unknown
Others
Nephrotic syndrome Various cancers Tumor antigens, immune
complexes
ACTH, adrenocorticotropic hormone; TGF, transforming growth factor; TNF, tumor necrosis factor; IL,
interleukin.
IMMUNOHISTOCHEMISTERY

It is a method for diagnosis of cancer.


1. Categorization of undifferentiated malignant tumor: Sometimes, many tumors like
anaplastic carcinoma, lymphoma, melanoma and sarcoma are difficult to distinguish with
routine H and E staining because of poor differentiation. So, immunohistochemical stains
can help in diagnosis e.g.
– Presence of cytokeratin points to epithelial origin (carcinoma).
– Presence of desmin is specific for muscle cell origin.
– Presence of Leucocyte Common Antigen (LCA) points to lymphoma.

2. Determination of site of origin of metastatic tumor: There are markers that point to the
origin of tumor (primary) in a biopsy specimen of metastasis. Examples include PSA (for
prostate cancer) and thyroglobulin (for thyroid cancer).
3. Prognostic or therapeutic significance: Estrogen/progesterone receptor detection has
therapeutic value in breast carcinomas. Receptor positive breast cancers are susceptible
to anti-estrogen therapy. Similarly, over-expression of erb-B2 protein suggests a poor
prognosis.

TUMOR MARKERS

Tumor markers are biochemical substances (include cell-surface antigens, cytoplasmic


proteins, enzymes, and hormones) which indicate the presence of a tumor.

Markers Associated Cancers


Hormones
Human chorionic gonadotropin (hCG) Trophoblastic tumors, non-seminomatous testicular
tumors
Calcitonin Medullary carcinoma of thyroid
Catecholamine and metabolites Pheochromocytoma and related tumors
Ectopic hormones See above
Oncofetal Antigens
a-Fetoprotein Liver cell cancer, non-seminomatous germ cell tumors of
testis
Carcinoembryonic antigen Carcinomas of the colon, pancreas, lung, stomach, and
heart
Isoenzymes
Prostatic acid phosphatase Prostate cancer
Neuron-specific enolase Small cell cancer of lung, neuroblastoma
Specific Proteins
Immunoglobulins Multiple myeloma and other gammopathies
Prostate-specific antigen and prostate-specific Prostate cancer
membrane antigen
Mucins and Other Glycoproteins
CA-125 Ovarian cancer
CA-19-9 Colon cancer, pancreatic cancer
CA-15-3 Breast cancer
New Molecular Markers
p53, APC, RAS mutations in stool and serum Colon cancer
p53 and RAS mutations in stool and serum Pancreatic cancer
p53 and RAS mutations in sputum and serum Lung cancer
p53 mutations in urine Bladder cancer

The important antigens for the determination of specific tumor cell origin are:
ithelial Tumors
• Breast: Alpha lactalbumin, GCDP-15, estrogen/progesterone
• Thyroid: Thyroglobulin, calcitonin
• Liver: AFP (a fetoprotein), HBsAg, keratin
• Prostate: Prostatic acid phosphatase, prostate specific antigen
• Mesothelioma: Keratin, Calretinin, mesothelin

rm Cell Tumors
• Human chorionic gonadotropin, AFP (a-fetoprotein)
senchymal Tumors
• Endothelial tumors: Factor VIII, CD 34,
• Melanoma: HMB 45, S 100
• Fibrohistiocytic tumors: Lysozyme, HAM 56
• Myogenic tumors: Desmin, smooth muscle specific antigen, myoglobin
uroendocrine Tumors
• Neuron specific enolase (NSE), chromogranin, synaptophysin
– Malignant melanoma expresses HMB 45, S-100 and vimentin. HMB 45 is present in
melanosomes and is more specific. S-100 is more sensitive but is non-specific (also
present in Langerhans’ cell histiocytosis, neural tumors, and sarcomas like
liposarcoma and chondrosarcoma)
– Neurofibroma (a neural tumor) shows the presence of S100 and GFAP (Glial Fibrillary
Acid Protein). Malignant tumors often lose expression of S-100 antigen.
– Neuroblastoma expresses NSE, chromogranin and synaptophysin. NSE is more
specific whereas chromogranin and synaptophysin are more sensitive tumor markers.
• Angiosarcoma expresses factor VIII, vimentin and CD34 antigen

Fig. 5: Neuroblastoma showing Homer-Wright pseudorosettes

Type of tumors associated with intermediate filaments:

Intermediate Filament Normal Tissue Expression Tumor

Cytokeratin All epithelial cells Carcinoma

Vimentin Mesenchymal cells Sarcomas

Desmin Muscle cells Leiomyoma


Rhabdomyosarcoma

Glial Fibrillary Acidic Protein Glial cells Astrocytoma


(GFAP) Ependymoma

Neurofilament Neurons and Neural crest Pheochromocytoma


derivatives Neuroblastoma

CONCEPT OF TUMOR LYSIS SYNDROME

Tumor lysis syndrome is caused by destruction of large number of rapidly proliferating


neoplastic cells. It is characterized by
• Hyperuricemia (due to increased turnover of nucleic acids)
• Hyperkalemia (due to release of the most abundant intracellular cation potassium)
• Hyperphosphatemia (due to release of intracellular phosphate)
• Hypocalcemia (due to complexing of calcium with the elevated phosphate)
• Lactic acidosis
• Hyperuricemia can cause uric acid precipitation in the kidney causing acute renal failure.
NEOPLASIA: GENERAL ASPECTS

1. Which of the following helps in differentiation of follicular carcinoma from


follicular adenoma of thyroid gland?
(AI 2011, AIIMS May 2010)
(a) Hurthle cell change
(b) Lining of tall columnar and cuboidal cells
(c) Vascular invasion
(d) Increased mitoses
2. All are malignant tumors, except:
(AI 2008)
(a) Chloroma
(b) Fibromatosis
(c) Askin’s tumor
(d) Liposarcoma
3. The following is not a feature of malignant transformation by cultured cells:
(AI 2005)
(a) Increased cell density
(b) Increased requirement for growth factors
(c) Alterations of cytoskeletal structures
(d) Loss of anchorage
4. Which of the following carcinoma most frequently metastasizes to brain?
(AIIMS Nov 2005)
(a) Small cell carcinoma of lung
(b) Prostate cancer
(c) Rectal carcinoma
(d) Endometrial cancer
5. Chemotherapeutic drugs can cause:
(AIIMS May 2005)
(a) Only necrosis
(b) Only apoptosis
(c) Both necrosis and apoptosis
(d) Anoikis
6. Reversible loss of polarity with abnormality in size and shape of cells is known as:
(AIIMS Nov 2001)
(a) Metaplasia
(b) Dysplasia
(c) Hyperplasia
(d) Anaplasia
7. Predisposing factors for skin cancer are:
(PGI Dec 2000)
(a) Smoking
(b) U-V-light
(c) Chronic ulcer
(d) Infrared light
8. Increased risk of cancer is seen in: (Delhi PG 2009 RP)
(a) Fibroadenoma of breast
(b) Bronchial asthma
(c) Chronic ulcerative colitis
(d) Leiomyoma of the uterus
9. A strong propensity for vascular invasion is seen in:
(a) Prostatic carcinoma
(Karnataka 2008)
(b) Hepatocellular carcinoma
(c) Bronchogenic carcinoma
(d) Gastric carcinoma
10. Earliest changes of neoplastic transformation as seen at a microscopic level is
called:
(Karnataka 2004)
(a) Hyperplasia
(b) Metaplasia
(c) Dysplasia
(d) Carcinoma in situ
11. Squamous cell carcinoma spreads by:
(RJ 2000)
(a) Hematogenous route
(b) Lymphatic route
(c) Direct invasion
(d) All
12. Which one of the following tumors does not cause bony metastasis?
(AP 2000)
(a) Renal cell carcinoma
(b) Gastric carcinoma
(c) Thyroid carcinoma
(d) Breast carcinoma
13. Hamartoma is:
(AP 2003)
(a) Proliferation of cells in foreign site
(b) Proliferation of native cells in tissue
(c) Malignant condition
(d) Acquired condition

MOST RECENT QUESTIONS

14. Sure sign of malignancy is:


(a) Mitoses
(b) Polychromasia
(c) Nuclear pleomorphism
(d) Metastasis
15. A lesion 3 cm away from gastroesophageal junction contain columnar epithelium,
such a type of lesion is:
(a) Metaplasia
(b) Hyperplasia
(c) Dysplasia
(d) Anaplasia
16. Malignancy is typically associated with disordered differentiation and maturation.
Which of the following mentioned options best describes anaplasia?
(a) Hepatic tumor cells synthesizing bile
(b) Skin tumor cells producing keratin pearls
(c) Bronchial epithelial cells producing keratin pearls
(d) Muscle tumor cells forming giant cells
17. Which of the following criteria can be used to determine if a pheochromocytoma
lesion is benign or malignant?
(a) Blood vessel invasion
(b) Cannot be determined by microscopic examination
(c) Hemorrhage and necrosis
(d) Nuclear pleomorphism
18 Which one is not the pre cancerous condition?
(a) Crohn’s disease
(b) Ulcerative colitis
(c) Leukoplakia
(d) Xeroderma pigmentosum
19. Which of the following features differentiates invasive carcinoma from carcinoma in
situ?
(a) Anaplasia
(b) Number of mitosis
(c) Basement membrane invasion
(d) Pleomorphism
20. Cell-matrix adhesions are mediated by?
(a) Cadherins
(b) Integrins
(c) Selectins
(d) Calmodulin
21. Which of the following is not a labile cell?
(a)Bone marrow
(b) Epidermal cells
(c) Small intestine mucosa
(d) Hepatocytes
22. Sure sign of malignancy is:
(a) Mitoses
(b) Polychromasia
(c) Nuclear pleomorphism
(d) Metastasis
23. Bimodality of incidence occurs in all, except
(a) Cancer penis in male
(b) Hodgkin’s diseases
(c) Breast cancer in females
(d) Leukemia
24.All of the following are premalignant except:
(a) Ulcerative colitis
(b) Peutz-Jegher syndrome
(c) Crohn disease
(d) Familial adenomatous polyposis
25. Which of the following is most reliable feature of malignant transformation of
pheochromocytoma?
(a) Presence of mitotic figures
(b) Presence of metastasis to other organs
(c) Vascular/ capsular invasion
(d) All of the above
26. Overgrowth of a skin structure at a localised region is:
(a) Hamartoma
(b) Malignant tumor
(c) Choristoma
(d) Polyp
27. Peau d’orange in carcinoma breast is due to:
(a) Obstruction of sub-dermal lymphatics
(b) Infiltration of Cooper’s ligament
(c) Hematogenous dissemination
(d) Nipple involvement
28.Molecular study is important in the management of which malignancy?
(a) Multiple myeloma
(b) Renal cell carcinoma
(c) Seminoma
(d) Basal cell carcinoma
29. Tumor cells secrete which of the following?
(a) Tyrosine kinase
(b) Collagenase IV
(c) Catenins
(d) E-cadherins
30. Substance playing a role in tumor metastasis cascade is?
(a) Collagenase IV
(b) TNF-alpha
(c) CD99
(d) NM23
31. Carcinoma with no or minimal metastasis is?
(a) Squamous cell carcinoma
(b) Basal cell carcinoma
(c) Melanoma
(d) Leydig’s cell carcinoma

CELL CYCLE AND ITS REGULATION

32. Ionizing radiation affects which stage of cell cycle


(a) G2 S
(b) G1 G2
(c) G2 M
(d) G0 G1
33. The correct sequence of cell cycle is:
(AI 2003)
(a) G0-G1-S-G2-M
(b) G0-G1-G2-S-M
(c) G0-M-G2-S-G1
(d) G0-G1-S-M-G2
34. During which phase of the cell cycle the cellular content of DNA is doubled:
(AIIMS Nov 2005)
(a) Mitotic phase
(b) G1 phase
(c) G2 phase
(d) S phase
35. The tumor suppressor gene p 53 induces cell cycle arrest at:
(AIIMS Nov 2005)
(a) G2-M phase
(b) S-G2 phase
(c) G1-S phase
(d) G0 phase

36. Transition from G2 to M phase of the cell cycle is controlled by:


(AIIMS Nov 2003)
(a) Retinoblastoma gene product
(b) p53 protein
(c) Cyclin E
(d) Cyclin B
37. Fixed time is required for which steps of cell cycle:
(a) S
(PGI Dec 2000)
(b) M
(c) G1
(d) G2
(e) Go
38. Regarding oncogenesis:
(PGI June 2002)
(a) Topoisomerase II causes breaks in strands
(b) p53 is the most common oncogene mutation causing malignancy in humans
(c) At G2-M phase there is loss of inhibitors controlling cell-cycle
(d) Decrease in telomerase activity causes anti-tumor effects
39. Regarding oncogenesis:
(PGI Dec 2002)
(a) Proto-oncogenes are activated by chromosomal translocation
(b) Malignant transformation involves accumulation of mutations in proto-oncogenes
and tumor suppressor gene in stepwise fushion
(c) Point mutation of somatic cells
(d) Increase in telomerase activity causes anti-tumor effects
(e) At G2-M phase there is loss of inhibitors controlling cell cycle

MOST RECENT QUESTIONS

40. The tumor suppressor gene P53 induces cell arrest at:
(a) G2- M phase
(b) S- G2 phase
(c) G1- S phase
(d) G0- phase
41. Not a premalignant condition:
(a) Fragile X syndrome
(b) Down’s syndrome
(c) Blount’s syndrome
(d) Fanconi’s syndrome
42. Which is associated with G2M transition in cell cycle:
(a) Cyclin A
(b) Cyclin B
(c) Cyclin E
(d) Cyclin D
43. Which of the following is not a cyclin dependent kinase (CDK) inhibitor?
(a) p21
(b) p27
(c) p53
(d) p57
44. Cells are most radiosensitive in:
(a) S - phase
(b) M -phase
(c) Gl - phase
(d) G0 - phase
45. E cadherin gene deficiency is seen in:
(a) Gastric cancer
(b) Intestinal cancer
(c) Thyroid cancer
(d) Pancreatic cancer
46. Li Fraumeni syndrome is due to mutation of which gene?
(a) p21
(b) p53
(c) p41
(d) p43

GENETIC MECHANISMS OF CARCINOGENESIS: PROTO-ONCOGENE, TUMOUR


SUPPRESSOR GENE, DEFECTIVE DNA REPAIR

47. All are true about Fanconi anemia, except:


(Bihar 2006)
(a) Defect in DNA repair
(b) Bone marrow hyperfunction
(c) Congenital anomaly present
(d) Increased chances of cancer
48. HER2/neu receptor plays a role in
(AIIMS Nov. 2010)
(a) Predicting therapeutic response
(b) Diagnosis of breast cancer
(c) Screening of breast cancer
(d) Recurrence of tumor
49. The most common secondary malignancy in a patient having retinoblastoma is:
(AI 2010)
(a) Osteosarcoma
(b) Renal cell carcinoma
(c) Pineoblastoma
(d) Osteoblastoma
50. Regarding Fanconi anemia, the wrong statement is:
(a) Autosomal dominant
(AI 2010)
(b) Bone marrow show pancytopenia
(c) Usually aplastic anemia
(d) It is due to defective DNA repair
51. True statements about p53 gene are all except:
(AI 2008)
(a) Arrests cell cycle at G1 phase
(b) Product is 53 kD protein
(c) Located on chromosome 17
(d) Wild/non-mutated form is associated with increased risk of childhood tumors
52. Growth factor oncogene is:
(AI 2008)
(a) Myc
(b) Fos
(c) Sis
(d) Jun
53. Rosettes are characteristically seen in:
(AI 2008)
(a) Retinoblastoma
(b) Melanoma
(c) Dysgerminoma
(d) Lymphoma
54. The normal cellular counterparts of oncogenes are important for the following
functions, except:
(AI 2006)
(a) Promotion of cell cycle progression
(b) Inhibition of apoptosis
(c) Promotion of DNA repair
(d) Promotion of nuclear transcription
55. An example of a tumor suppressor gene is:
(AI 2005)
(a) myc
(b) fos
(c) ras
(d) Rb
56. Lynch syndrome is associated with cancers of the:
(AIIMS Nov 2009)
(a) Breast, colon, ovary
(b) Breast, endometrium, ovary
(c) Breast, colon, endometrium
(d) Colon, endometrium, ovary
57. Loss of heterozygosity associated with:
(AIIMS May 2008)
(a) Acute myeloid leukemia
(b) ALL
(c) Retinoblastoma
(d) Promyelocytic leukemia
58. Which is not a tumor suppressor gene?
(AIIMS May 2008)
(a) WT-1
(b) Rb
(c) p53
(d) ras
59. The inheritance pattern of familial Retinoblastoma is:
(AIIMS Nov 2005)
(a) Autosomal recessive
(b) Autosomal dominant
(c) X-linked dominant
(d) X-linked recessive
60. Which of the following is known as the “guardian of the genome”?
(AIIMS May 2005)
(a) p53
(b) Mdm2
(c) p14
(d) ATM
61. The following statements are true about Tumor Suppressor Gene p53, except:
(AIIMS Nov 2004)
(a) It regulates certain genes involved in cell cycle regulation
(b) Its increased levels can induce apoptosis
(c) Its activity in the cells decreases following UV irradiation and stimulates cell cycle
(d) Mutations of the p53 gene are most common genetic alteration seen in human
cancer
62. In the mitogen activated protein kinase pathway, the activation of RAS is
counteracted by:
(AIIMS May 2004)
(a) Protein kinase C
(b) GTPase activating protein
(c) Phosphatidyl inositol
(d) Inositol triphosphate
63. Which of the following mutations in a tumor suppressor agent causes breast
carcinoma?
(AIIMS May 2002)
(a) p43
(b) p53
(c) p73
(d) p83
64. True about proto-oncogenes is: (PGI June’ 06)
(a) Important for normal cell growth
(b) May get converted into oncogenes
(c) C-myc over-expression causes lymphoma
(d) Their mutation causes retinoblastoma
(e) Deletion cause Sickle cell disease
65.True about oncogene is:
(PGI Dec 2002)
(a) Present in normal cell
(b) They are of viral origin
(c) They are transduced from virus infected cells
(d) P53 is most common oncogene mutation causing malignancy
(e) Viral oncogenes are identical with humans cellular oncogenes
66. Cancer cell survival is enhanced by:
(a) Suppression of p53 protein
(PGI June 2003)
(b) Over expression of p53 gene
(c) bcl-2
(d) bax
(e) bad
67. Following are required for normal growth:
(PGI Dec 2003)
(a) Proto-oncogenes
(b) Tumor suppressor genes
(c) Oncogenes
(d) DNA repair genes

MOST RECENT QUESTIONS

68. Xeroderma pigmentosum is caused due to a group of closely related abnormalities


in:
(a) Mismatch repair
(b) Base excision repair
(c) Nucleotide excision repair
(d) SOS repair
69. Increased expression of which of the following causes oncogenesis
(a) IGF receptor
(b) EGF receptor
(c) GH receptor
(d) Aldosterone receptor
70. Tumor suppressor genes are all, except
(a) APC
(b) p53
(c) Rb
(d) C-myc
71. Angiogenesis is:
(a) Formation of the new blood vessels
(b) Repair by connective tissues
(c) Formation of the blood clot
(d) All of the above
72. Medullary carcinoma of thyroid is associated with mutation in:
(a) RET
(b) RAS
(c) NF
(d) Rb
73. APC gene is located on which chromosome:
(a) Chromosome 5
(b) Chromosome 6
(c) Chromosome 9
(d) Chromosome 11
74. Endometrial carcinoma is associated with which of the following tumor
suppression gene mutation?
(a) P53
(b) Rb
(c) PTEN
(d) APC
75. Histopathologically, rosettes are not seen in:
(a) Retinoblastoma
(b) Neurocysticercosis
(c) PNET
(d) Medulloblastoma
76. The tumor suppressor gene p 53 induces cell cycle arrest at:
(a) G2 – M phase
(b) S – G2 phase
(c) G1 – S phase
(d) G0 phase
77. Which of the following gene defect is associated with development of medullary
carcinoma of thyroid:
(a) RET gene
(b) FAP gene
(c) Rb gene
(d) BRCA 1 gene
78. All of the following are tumor markers, except:
(a) Beta-2 macroglobulin
(b) HCG
(c) Alpha-fetoprotein
(d) CEA
79. MYC gene is:
(a) Protein kinase inhibitor
(b) Growth factor inhibitor
(c) GTPase
(d) Transcription activator
80. Retinoblastoma is associated with which of the following tumours?
(a) Osteoclastoma
(b) Hepatocellular cancer
(c) Squamous cell cancer
(d) Osteosarcoma
81. RET gene mutation is associated with which malignancy?
(a) Pheochromocytoma
(b) Medullary carcinoma thyroid
(c) Lymphoma
(d) Renal cell carcinoma
82. An example of a tumour suppressor gene is:
(a) Myc
(b) Fos
(c) Ras
(d) RB
83. Which of the following is DNA repair defect?
(a) Retinoblastoma
(b) Neurofibromatosis
(c) Xeroderma pigmentosum
(d) MEN-I
84. An example of a tumor suppressor gene is:
(a) Myc
(b) Fos
(c) Ras
(d) Rb
85. Knudson two hit hypothesis is seen with
(a) Melanoma
(b) Retinoblastoma
(c) Ulcerative colitis
(d) Crohn disease
86. Retinoblastomas arising in the context of germ-line mutations not only may be
bilateral, but also may be associated with _______ (so called “trilateral”
retinoblastoma)
(a) Medulloblastoma
(b) Pinealoblastoma
(c) Neuroblastoma
(d) Hemangioblastoma
87. HER-2/neu gene causes breast carcinoma due to?
(a) Overexpression
(b) Suppression
(c) Mutation
(d) Translocation
88. Gene involved in medullary carcinoma thyroid is:
(a) Ret proto oncogene
(b) FAP gene
(c) Rb gene
(d) BRCA 1 gene
89. Which chromosome mutation is associated with medulloblastoma?
(a) Chromosome 16
(b) Chromosome 17
(c) Chromosome 18
(d) Chromosome 19
90. VHL syndrome is associated most commonly with which carcinoma?
(a) Lung carcinoma
(b) Renal cell carcinoma
(c) Endometrial carcinoma
(d) Hepatocellular carcinoma

ETIOLOGICAL AGENTS FOR CANCER: CHEMICAL, RADIATION, MICROBES,


MULTISTEP CARCINOGENESIS

91. Post transplant lymphoma is caused by which of the following?


(AIIMS May 2012)
(a) CMV
(b) EBV
(c) Herpes simplex
(d) HHV-6
92. H. pylori infection is associated with development of which malignancy:
(DPG 2011)
(a) MALTomas
(b) Atherosclerosis
(c) Sarcoma
(d) Gastrointestinal stromal tumor (GIST)
93. Helicobacter pylori infection is associated with all of the following conditions,
except:
(DPG 2011)
(a) Peptic ulcer disease
(b) Gastric adenocarcinoma
(c) B cell lymphoma
(d) Burkitt’s lymphoma
94. Tumors associated with organisms are all except:
(a) Hepatocellular cancer
(AIIMS Nov 2009)
(b) Non-small Cell Carcinoma of Lung
(c) Gastric cancer
(d) Nasopharyngeal cancer
95. Which of the following is essential for tumor metastasis?
(AIIMS Nov 2008, DNB 2009)
(a) Angiogenesis
(b) Tumorogenesis
(c) Apoptosis
(d) Inhibition of tyrosine kinase activity
96. Which of the following statements about carcinogenesis is false?
(AIIMS May 2006)
(a) Asbestos exposure increases the incidence of lung cancer
(b) Papilloma viruses produce tumors in animals but not in humans
(c) Exposure to aniline dyes predisposes to cancer of the urinary bladder
(d) Hepatitis B virus has been implicated in hepatocellular carcinoma
97. Which of the following is an oncogenic RNA virus?
(Delhi PG 2009 RP)
(a) Hepatitis B virus
(b) Human papilloma virus
(c) Epstein Barr virus
(d) Hepatitis C virus
98. LMP-1 gene plays a role in oncogenesis induced by:
(Karnataka 2008)
(a) Human T cell leukemia virus type I
(b) Hepatitis B virus
(c) Epstein-Barr virus
(d) Human papilloma virus

MOST RECENT QUESTIONS

99. Skin cancers develop due to sunlight exposure induced by:


(a) UVA rays
(b) UVB rays
(c) UVC rays
(d) UVD rays
100. Most radiosensitive tumor is:
(a) Renal cell carcinoma
(b) Carcinoma colon
(c) Hepatocellular carcinoma
(d) Testicular seminoma
101. Smoking is a risk factor for all carcinomas, except:
(a) Oral
(b) Bronchial
(c) Bladder
(d) Thyroid
102. Workers exposed to polyvinyl chloride may develop following liver malignancy:
(a) Cholangiocarcinoma
(b) Fibrolamellar carcinoma
(c) Angiosarcoma
(d) All of the above
103. Which among the following is not a neoplastic virus:
(a) Cytomegalovirus
(b) Hepatitis B virus
(c) Human papilloma virus
(d) All of these
104. One of the following leukemia almost never develops after radiation?
(a) Acute myeloblastic leukemia
(b) Chronic myeloid leukemia
(c) Acute lumphoblastic leukemia
(d) Chronic lymphocytic leukemia
105.The following parasitic infections predispose to malignancies?
(a) Paragonimus westermani
(b) Guinea worm infection
(c) Clonorchiasis
(d) Schistosomiasis
106. Kaposi’s sarcoma is seen with:
(a) HCV
(b) HPV
(c) HSV
(d) HHV
107. UV radiation has which of the following effects on the cells?
(a) Prevents formation of pyrimidine dimers
(b) Stimulates formation of pyrimidine dimers
(c) Prevents formation of purine dimers
(d) All of the above
108. Thorium induced tumor is which of the following?
(a) Renal cell carcinoma
(b) Lymphoma
(c) Angiosarcoma of liver
(d) Astrocytoma
109. Radiation exposure during infancy has been linked to which one of the following
carcinoma?
(a) Breast
(b) Melanoma
(c) Thyroid
(d) Lung
110. The most radiosensitive cells are:
(a) Neutrophils
(b) Lymphocytes
(c) Erythrocytes
(d) Megakaryocytes
111. The SI unit of radiation absorbed dose is
(a) Rad
(b) Becquerel
(c) Gray
(d) Sievert
112. Tropical spastic paraparesis is caused by:
(a) Human T-cell Lymphotropic Virus
(b) Hepatitis B virus
(c) Human Immunodeficiency virus
(d) Epstein Barr Virus
113. Which of the following does not predispose to leukemia?
(a) Genetic disorder
(b) Alcohol
(c) Smoking
(d) Chemical exposure
114. HPV oncogene expression is due to:
(a) E1E2
(b) E1E3
(c) E3E5
(d) E6E7
115. Hematological malignancies are commonly linked to which of the following:
(a) Nicotine
(b) Lithium
(c) Benzene
(d) Alcohol
116. Which of the following has tumor promoting effect?
(a) BRAC
(b) myc
(c) RB
(d) p16
117. All are true about chromosomal instability syndrome except?
(a) DNA repair defects
(b) AD inheritance
(c) Increased risk of malignancy
(d) May be associated with immunodeficiency
118. BRCA1 gene is associated with:
(a) Lobular carcinoma
(b) Medullary carcinoma
(c) Tubular carcinoma
(d) Papillary carcinoma
119. PTEN gene mutation is seen in:
(a) Ovarian carcinoma
(b) Li-Fraumeni syndrome
(c) Endometrial carcinoma
(d) MEN2A
120. Wilm’s tumor gene is located on chromosome?
(a) 13q2.3
(b) 17p21
(c) 17q21
(d) 11p13

PARANEOPLASTIC SYNDROMES, TUMOUR MARKERS, TUMOUR LYSIS SYNDROME


121. A 20 year old female was diagnosed with granulose cell tumor of the ovary. Which
of the following bio markers would be most useful for follow-up of patient?
(AIIMS Nov 2011)
(a) CA 19-9
(b) CA50
(c) Inhibin
(d) Neuron – specific enolase
122. Alpha fetoprotein is a marker of:
(AI 2010)
(a) Hepatoblastoma
(b) Seminoma
(c) Renal cell carcinoma
(d) Choriocarcinoma
123. Hyperglycemia associated with:
(AI 2010)
(a) Multiple myeloma
(b) Ewing sarcoma
(c) Osteosarcoma
(d) Chondroblastoma
124. Which of the following is Not associated with thymoma?
(a) SIADH
(AI 2010)
(b) Myasthenia gravis
(c) Polymyositis
(d) Hypogammaglobinemia
125. Which of the following is not true about Neuroblastoma?
(AI 2009)
(a) Most common extracranial solid tumor in childhood
(b) >50% patients present with metastasis at time of diagnosis
(c) Lung metastases are common
(d) Involve aorta and its branches early
126. Migratory thrombophlebitis is associated with all of the following malignancies,
except:
(AI 2008)
(a) Prostate
(b) Lung
(c) GIT
(d) Pancreas
127. HMB 45 is a tumor marker for:
(AI 2008, DNB 2008)
(a) Neuroblastoma
(b) Neurofibroma
(c) Malignant melanoma
(d) Angiosarcoma
128. AFP is a marker of:
(AIIMS Nov 2009)
(a) Hepatoblastoma
(b) Seminoma
(c) Sertoli-Leydig cell tumor
(d) Choriocarcinoma
129. An undifferentiated malignant tumor on immunohistochemical stain shows
cytoplasmic positivity of most of the tumor cells for cytokeratin. The most
probable diagnosis of the tumor is:
(AIIMS May 2006)
(a) Sarcoma
(b) Lymphoma
(c) Carcinoma
(d) Malignant melanoma
130. For which one of the following tumors Gastrin is a biochemical marker?
(AIIMS May 2005)
(a) Medullary carcinoma of thyroid
(b) Pancreatic neuroendocrine tumor
(c) Pheochromocytoma
(d) Gastrointestinal stromal tumor
131. All of the following are examples of tumor markers, except:
(AIIMS Nov 2004)
(a) Alpha-hCG (a-hCG)
(b) Alpha-Feto protein
(c) Thyroglobulin
(d) Beta 2-microglobulin
132. Which of the following tumors have an increased elevation of placental alkaline
phosphatase in the serum as well as a positive immunohistochemical staining for
placental alkaline phosphatase?
(AIIMS May 2004)
(a) Seminoma
(b) Hepatoblastoma
(c) Hepatocellular carcinoma
(d) Peripheral neuroectodermal tumor
133. In tumor lysis syndrome, all of the following are seen, except:
(AIIMS May 2002)
(a) Hypernatremia
(b) Hypercalcemia
(c) Hyperkalemia
(d) Hyperphosphatemia
134. Uses of tumor marker are:
(PGI Dec 2000)
(a) Screening of a cancer
(b) Follow up of a cancer patient, esp. for knowing about recurrence
(c) Confirmation of a diagnosed cancer
(d) For monitoring the treatment of a cancer
135. True about Carcinoembryonic antigen (CEA):
(PGI June 01
(a) Useful for screening of carcinoma colon)
(b) Gives confirmative evidence of Ca. colon
(c) Helpful for follow-up after resection
(d) Levels decrease immediately after resection of tumor
(e) Tumor size correlates with CEA level
136. CA·125 is associated with:
(PGI June 2002)
(a) Colon ca
(b) Breast ca
(c) Ovarian ca
(d) Bronchogenic ca
(e) Pancreatic ca
137. Secondaries are common in all, except:
(PGI June 01)
(a) Skull
(b) Hand and feet bones
(c) Proximal limb bones
(d) Pelvic
(e) Vertebrae
138. Hybridoma refers to
(Delhi PG 2009 RP)
(a) Collision tumor
(b) A tumor of brown fat
(c) A hamartoma
(d) A technique for raising monoclonal antibodies
139. BCL2 is a marker for:
(Delhi PG-2007)
(a) Follicular lymphoma
(b) Mycosis fungoides
(c) B-cell lymphoma
(d) Mantle cell lymphoma
140. Alpha-fetoproteins are a marker of:
(Karnataka 2005)
(a) Secondaries in liver
(b) Cholangiocarcinoma
(c) Hepatoma
(d) None of the above
141. Increased level of alpha fetoprotein is found in
(UP 2001)
(a) Yolk sac tumor
(b) Seminoma
(c) Teratoma
(d) Choriocarcinoma
142. Migratory thrombophlebitis is seen in:
(UP 2007)
(a) Disseminated cancer
(b) Rheumatic heart disease
(c) Libman-Sachs endocarditis
(d) All of the above
143. A 65 years old male diagnosed by biopsy a case of lung carcinoma, with
paraneoplastic syndrome and increased calcium. Probable cause is
(UP 2008)
(a) Parathyroid hormone
(b) Parathyroid hormone related peptide
(c) Calcitonin
(d) Calcitonin related peptide
144. Which is associated with polycythemia:
(RJ 2001)
(a) Gastric carcinoma
(b) Fibrosarcoma
(c) Cerebellar hemangioblastoma
(d) All
145. Serum AFP is increased in all, except:
(RJ 2003)
(a) Acute hepatitis
(b) Hepatocellular carcinoma
(c) Hepatoma
(d) Bladder carcinoma
146. Carcinoembryonic antigen is elevated in all, except:
(a) Alcoholic cirrhosis
(RJ 2004)
(b) Ca colon
(c) Ulcerative colitis
(d) Emphysema
147. Desmoid tumor arises from:
(TN 1991)(AP 2000)
(a) Wall of the intestine
(b) Anterior abdominal wall
(c) Submucosa
(d) Appendix
148. Alpha-fetoprotein is a tumor marker of:
(AP 2001)
(a) Carcinoma ovary
(b) Liver malignancies
(c) Endodermal sinus tumor of testis
(d) Both (b) and (c)
149. a-fetoprotein is seen in all except:
(AP 2002)
(a) Hepatocellular carcinoma
(AI 1997)
(b) Carcinoma colon
(UP 1996)
(c) Pancreatic carcinoma
(d) Germ cells of testes
150. The diagnostic tumor marker of liver carcinoma is:
(AP 2007)
(a) CEA
(b) AFP
(c) CA - 125
(d) All of the above
151. Spontaneous regression of tumor is seen in:
(Kolkata 2002)
(a) Wilm’s tumor
(b) Neuroblastoma
(c) Acute monocytic leukemia
(d) Hepatoblastoma

MOST RECENT QUESTIONS

152. All of the following about tumor markers are properly matched, except:
(a) Prostate cancer - PSA
(b) Colon cancer - CEA
(c) Ovarian cancer – CA 125
(d) Cholangiocarcinoma - AFP
153. Popcorn calcification is seen in:
(a) Chondrosarcoma
(b) Fibrous dysplasia
(c) Osteoblastoma
(d) Wilms’ tumor
154. Which one of the following is a frequent cause of serum alpha- fetoprotein level
greater than 10 times the normal upper limit?
(a) Seminoma
(b) Hepatocellular carcinoma of liver
(c) Cirrhosis of liver
(d) Oat cell tumor of lung
155. Rise of AFP is noted in all except:
(a) Hepatocellular carcinoma
(b) Cirrhosis
(c) Germ cell tumor
(d) Kidney tumor
156. Catecholamines are increased in:
(a) Neuroblastoma
(b) Retinoblastoma
(c) Medulloblastoma
(d) Nephroblastoma
157. Which of the following is a marker for carcinoma of lung and breast?
(a) CEA
(b) AEP
(c) HCG
(d) CA-15-3
158. Secondaries of all the following cause osteolytic lesions except:
(a) Prostate
(b) Kidney
(c) Bronchus
(d) Thyroid
159. Sacrococcygeal teratoma, marker is:
(a) CEA
(b) β- HCG
(c) S100
(d) CA-125
160. Which of the following mutation is seen in malignant melanoma?
(a) N-myc
(b) CDKN2A
(c) RET
(d) Rb
161. Marker of small cell cancer of lung is:
(a) Chromogranin
(b) Cytokeratin
(c) Desmin
(d) Vimentin
162. Which of the followingis a squamous cell carcinoma marker?
(a) Vimentin
(b) Desmin
(c) Cytokeratin
(d) Glial fibrillary acid protein
163. Marker for ovarian carcinoma in serum is:
(a) CA-125
(b) Fibronectin
(c) Acid Phosphatase
(d) PSA
164. Which of the following is tumor marker of seminoma?
(a) AFP
(b) LDH
(c) PLAP
(d) HCG
165. Commonest cancer in which metastasis is seen in brain in
(a) Breast
(b) Lung
(c) Kidney
(d) Intestines
166. Which of the following is incorrect about neuro-blastoma?
(a) Most common abdominal tumor in infants
(b) X-ray abdomen shows calcification
(c) Can show spontaneous regression
(d) Urine contains 5H.I.A.A
167. Tumor that follows rule of 10 is:
(a) Pheochromocytoma
(b) Oncocytoma
(c) Lymphoma
(d) Renal cell carcinoma
168. Immuno-histopathological markers wrongly matched:
(a)Desmin-Carcinomas
(b) Vimentin – Sarcomas
(c) Leukocyte specific antigen-Lymphoma
(d) S100-melanoma
169. Which of the following is a special stain for rhabdomyosarcoma?
(a) Cytokeratin
(b) Synaptophysin
(c) Desmin
(d) Myeloperoxidase
170. The most common cause of malignant adrenal mass is
(a) Adrenocortical carcinoma
(b) Malignant Phaeochromocytoma
(c) Lymphoma
(d) Metastasis from another solid tissue tumor
171. About intraoperative histopathological analysis, all are true except:
(a) Gives an immediate definitive diagnosis of tumor
(b) Used for detecting positive margins after resection
(c) Used to confirm suspected metastasis
(d) Sentinel lymph node biopsy in breast carcinoma is an example
172. Hypercalcemia is seen in which cancer?
(a) Renal cell cancer
(b) Carcinoma stomach
(c) Small cell carcinoma lung
(d) Hepatocellular carcinoma
173. Krukenberg tumor associated mostly with which cancer?
(a) Stomach
(b) Breast
(c) Liver
(d) Pancreas
174.Most common carcinoma is associated with inferior vena caval metastasis?
(a) Small cell carcinoma lung
(b) Gastric adenocarcinoma
(c) Renal cell carcinoma
(d) Papillary carcinoma thyroid
175. Carcino Embryonic Antigen is:
(a) Hormone
(b) Glycoprotein
(c) Enzyme
(d) Tumor associated protein
176. Herringbone pattern on histology is seen in which tumor?
(a) Fibrosarcoma
(b) Lipoma
(c) Carcinoma
(d) Liposarcoma
1. Ans. (c) Vascular invasion (Ref: Robbins 9/e p1094)
Robbins clearly write…. ‘Microscopically, most follicular carcinomas are composed of fairly
uniform cells forming small follicles. Follicular carcinomas may be grossly infiltrative or
minimally invasive. The latter are sharply demarcated lesions that may be impossible to
distinguish from follicular adenomas on gross examination. This distinction requires
extensive histologic sampling of the tumor-capsule-thyroid interface, to exclude
capsular and/or vascular invasion. Extensive invasion of adjacent thyroid parenchyma
makes the diagnosis of carcinoma obvious in some cases’.
• Ideal answer for a question for diagnosis of follicular cancer is capsular invasionQ (better than
even vascular invasion) but in the given options, vascular invasion is the answer of choice.

2. Ans. (b) Fibromatosis:


(Ref: Robbins 7th/770, 783-4, Harrison 16th/633, 9/e p1221-1222)
• Fibromatosis are a group of fibroblastic proliferations. Though they are locally
aggressive, they do not metastasize.
Fibromatosis
Superficial Fibromatosis Deep Fibromatosis
*Palmar fibromatosis (Dupuytren contracture) *Also called desmoids tumors
*Plantar fibromatosis *Greater tendency to recur
*Penile fibromatosis (Peyronie’s disease) *Grow in a locally aggressive manner
*Arise isolated or as component of Gardner
syndrome

3. Ans. (b) Increased requirement of growth factors


(Ref: Biology of the cell (2003) 357-364)
Both normal cells and cancer cells can be cultured in-vitro. However, they behave quite
differently
Normal cell Cancer cell
Show replicative senescence i.e. cells pass through They are immortal i.e. proliferate
limited number of cell divisions before they decline in vigor indefinitely in culture.
and die. It may caused by inability to synthesize Cancer cells in culture produce
telomerase telomerase
Normal cells show the phenomenon of contact inhibition Show no contact inhibition. Even after
i.e. they proliferate until the surface of culture dish is the surface of dish is covered, the cells
covered by single layer of cells just touching each other continue to divide
Nutrients and growth factors must be supplied to them in Do not require growth factors
their tissue culture medium

Normal karyotype is present Mostly show abnormal karyotype

4. Ans. (a) Small cell carcinoma of lung


(Ref: Harrison 17th/2458; Robbins 8th/1339, 7th/1410, 9/e p1315)
Small cell carcinoma of lung most commonly metastasize to the brain. It accounts for about
40% of brain metastases.
Other Tumors Metastasizing to Brain are carcinomas of • Breast, • Melanoma, • Kidney, • GIT

5. Ans. (c) Both necrosis and apoptosis


(Ref: Harrison 17th/519 , 9/e p303, 315)
• Chemotherapeutic drugs can cause both necrosis and apoptosis, but it is apoptosis
which is the basis of action of chemotherapeutic drugs.
• Anoikis refers to death of epithelial cells after removal from the normal milieu of substrate,
particularly from cell to cell contact.
6. Ans. (b) Dysplasia discussed in details in text.
(Ref: Robbins 7th/273-274 , 9/e p271)
7. Ans. (a) Smoking; (b) U-V-light; (c) Chronic ulcer:
(Ref: Harrison’ 16th/497, Robbins 9/e p1155)
Risk factors
Melanoma Basal cell and squamous cell carcinoma
• Family history of melanoma • Exposure to UV light principally UV-B
• Persistently changing mole • Male sex and Older age
• Presence of clinically atypical mole • Exposure to sun, arsenic, smoking, cyclic aromatic
• Immunosuppression hydrocarbons in tar, soot or shale
• Sun exposure • HIV, HPV infection, immunosuppression.
• Ionizing radiations, thermal burns
• Certain scars and chronic ulcerations
• Heritable conditions like albinism, Xeroderma
pigmentosum, genetic mutations (PATCHED
gene)
• Premalignant conditions like Actinic keratosis,
Bowen’s disease, Erythroplasia of Queyrat.

8. Ans. (c) Chronic ulcerative colitis


(Ref: Robbins 8th/276 , 9/e p279)
Certain non-neoplastic disorders—the chronic atrophic gastritis of pernicious anemia, solar
keratosis of the skin, chronic ulcerative colitis, and leukoplakia of the oral cavity, vulva,
and penis—have such a well-defined association with cancer that they have been termed
precancerous conditions.
9. Ans. (b) Hepatocellular carcinoma
(Ref: Robbins 7th/925 , 9/e p274)
Renal cell cancer and hepatocellular cancer have high tendency invasion of vascular channels.
10. Ans. (c) Dysplasia (Ref: Robbins 9/e p271-272)
• Dysplasia is the loss of uniformity of individual cells as well as their architectural orientation.
• Carcinoma in situ (dysplastic changes are marked but lesion remains confined to normal tissue:
pre-invasive neoplasm). Basement membrane is intact.
• Anaplasia is Complete lack of differentiation of cells both morphologically and functionally
(Invasive Ca)

11. Ans. (b) Lymphatic route (Ref: Robbins 9/e p273)


12. Ans. (b) Gastric carcinoma (Ref: Robbins 9/e p1207)
13. Ans. (b) Proliferation of native cells in tissue
(Ref: Robbins 8th/262; 7th/272 , 9/e p267)
14. Ans. (d) Metastasis (Ref: Robbins 8th/269; 9/e p272)
15. Ans. (a) Metaplasia (Ref: Robbins 8th/10,265; 9/e p271)
16. Ans (d) Muscle tumor cells forming giant cells
(Ref: Robbins 8th/262-5, 9/e p270)
Neoplastic cells may be similar to normal cells found in the tissue of origin, which defines the
malignancy as “well differentiated” or “low grade.” Alternatively, the neoplastic cells may
lack most of the characteristic features of normal cells found in the tissue of origin, which
defines the malignancy as “poorly differentiated” or “high grade.” Tumors that contain
neoplastic cells in the midst of this spectrum are termed “moderately differentiated” or
“medium grade.” If the neoplastic cells are described as anaplastic, they demonstrate a
complete lack of differentiation.
The appearance of giant multinucleated cells in a muscle tumor would therefore suggest anaplasia.

(Choice A) Cells in hepatic tissue would be expected to synthesize bile. Therefore a hepatic
tumor that synthesizes bile is described as being well-differentiated not anaplastic.
(Choice B and C) Cells in the epithelium would be expected to produce keratin pearls.
Therefore an epithelial tumor that produces keratin pearls would be described as well-
differentiated not anaplastic.
17. Ans. (b) Cannot be determined by microscopic examination (Ref: Robbins
8th/1159-1161, 9/e p1135)
Pheochromocytomas, and their related counterparts in extra-adrenal sites called
paragangliomas, are notorious because the only reliable indicator of metastatic potential
is the presence of distant metastases. Very malignant-appearing tumors may not
metastasize and benign-appearing tumors may produce metastases. These tumors
should all be considered “potentially malignant.”
18. Ans. (a) Crohn’s disease (Ref: Robbins 9/e p279)
• Ideal answer to this question is none but in the given situation, the answer of choice
is Crohn disease because in the comparison of the two types of inflammatory bowel
disease, Crohn disease is less likely to be associated with progression to cancer of
the bowel.
19. Ans. (c) Basement membrane invasion
(Ref: Robbins 9/e p271)
• Basement membrane invasionQ
is the most important differentiating feature between invasive
carcinoma from carcinoma in situ.

20. Ans (b) Integrins (Ref: Robbins 8/e p49, 9/e p24)
The cell adhesion molecules (CAMs) are classified into four main families:
Immunoglobulin family CAMs
• Cadherins
• Integrins: bind to extracellular matrix (ECM) proteins such as fibronectin, laminin,
and osteopontin providing a connection between cells and extracellular matrix
(ECM)
• Selectins
21. Ans. (d) Hepatocytes
(Ref: Robbins 8/e p81-4, 9/e p101)
Permanent cells Quiescent cells Labile cells
• Cannot divide in postnatal • Low level of replication which increases • Rapid rate of replication
life only on stimulation • Skin, GIT, oral cavity
• Neurons, skeletal muscles • Liver cells, kidney cells

22. Ans. (d) Metastasis (Ref: Robbins 9/e p272)


Metastasis is the most reliable feature of a malignant tumor, characterized by the spread of
the tumor to other parts because of penetration into blood vessels, lymphatics and the
body cavities.
23. Ans. (a) Cancer penis in male
(Ref: Various books, internet)
Diseases showing bimodality of age presentation (Mnemonic: ABCDEGH)
1. Aortic stenosis/acute leukemia – A.L.L > A.M.L
2. Breast cancer (Before advent of mammography
3. Crohn’s disease
4. Dermatomyositis
5. Enthesioneurobalstoma
6. Thyroglossal cyst
7. Hodgkin’s lymphoma
8. Vulvar carcinoma but NOT penile cancer
24. Ans. (c) Crohn disease (Ref: Robbins 9th/800-1,806,809)
Robbins pg 806……Peutz-Jeghers syndrome is associated with a markedly increased risk of
several malignancies. Lifetime risk is approximately 40% for these, and regular
surveillance is recommended.
Pg 809…Colorectal adenocarcinoma develops in 100% of untreated FAP patients, often before
age 30 and nearly always by age 50.

The risk of colonic adenocarcinoma is increased in patients with long-standing IBD affecting the colon.
Please understand that increased risk of cancer is seen in colonic variant of Crohn disease and not
otherwise. Ulcerative colitis is a premalignant condition.

25. Ans. (b) Presence of metastasis to other organs


(Ref: Robbins 8/e p1159-1161, 9/e p1135)
“There is no histologic feature that reliably predicts clinical behavior. In fact, cellular
and nuclear pleomorphism, including the presence of giant cells, and mitotic
figures are often seen in benign pheochromocytomas, while cellular monotony is
paradoxically associated with an aggressive behavior. Even capsular and vascular
invasion may be encountered in benign lesions. Therefore, the definitive diagnosis
of malignancy in pheochromocytomas is based exclusively on the presence of
metastases.
26. Ans (a) Hamartoma (Ref: Robbins 8/e p262, 9/e p13)
An overgrowth of a skin structure at a localized region is likely to be indigenous as well as
benign; this is more likely to be a hamartoma. It is now considered as a neoplasm.
27. Ans. (a) Obstruction of sub-dermal lymphatics
(Ref: Robbins 9th/1067)
Breast cancers presenting with breast erythema and skin thickening have a very poor
prognosis, as most patients prove to have distant metastases. The edematous skin is
tethered to the breast by Cooper ligaments and mimics the surface of an orange peel, an
appearance referred to as peau d’orange. These clinical signs are caused by dermal
lymphatics filled with metastatic carcinoma that blocks lymphatic drainage.
28. Ans (a) Multiple myeloma (Ref: Robbins 9/e p306)
Molecular study is important in the management of:

• Multiple myeloma
• Neuroblastoma
• Prostate cancer
• Myelodysplastic disorders

29. Ans (b) Collagenase IV (Ref: Robbins 9/e p308)


Benign tumors of the breast, colon, and stomach show little type IV collagenase activity,
whereas their malignant counterparts over express this enzyme.
30. Ans (a) Collagenase IV (Ref: Robbins 9/e p308)
31. Ans (b) Basal cell carcinoma (Ref: Robbins 9/e p1158)
32. Ans. (c) G2 M
(Ref: Robbins 8th/286; Harrison 17th/516, 9/e p289)
Direct quote from Robbins…. ‘The G2/M checkpoint monitors the completion of the DNA
replication and checks whether the cell can safely initiate the mitosis and separate sister
chromatids. This checkpoint is particularly important in cells exposed to ionizing radiation.
Cells damaged by ionizing radiation activate G2/M checkpoint and arrest in G2’.
33. Ans. (a) G0-G1-S-G2-M (Ref: Robbins 7th/90, 9/e p25)
34. Ans. (d) S phase (Ref: Robbins 8th/86, 7th/90, 9/e p25)
35. Ans. (c) G1-S Phase (Ref: Robbins 7th/292, 9/e p295)
• G1/S check-point is controlled by p53 whereas G2/M check-point has both p53
dependent as well as independent mechanisms.
• p53 induces the synthesis of p21 which inhibits cyclin D/Cdk4. This results in
stoppage of activation of Rb and cell cycle is arrested in G1/S phase.

36. Ans. (d) Cyclin B


(Ref: Robbins 7th/290-291, Robbins 8th/285-286 , 9/e p25-26)
Examples of cyclin/CDK complexes controlling the cell cycle.
Cyclin B/CDK1 Regulates the transition from G2 to M phase

Cyclin D/CDK4 Regulates the transition from G1-S


Cyclin D/CDK6
Cyclin E/CDK2

Cyclin A/CDK2 and cyclin Active in S phase


B/CDK1

37. Ans. (a) S; (b) M; (d) G2: (Ref: Gray’s anatomy 38th/55)
• The time taken for S, G2 and M phases are similar for most cell types, occupying
about 6, 4 and 2 hours respectively.
Contd...
• The duration of G1 shows considerable variation. It can be as short as 2 hours in
rapidly dividing cells like embryonic tissues or as long as 12 hours in some adult
tissues.
• G1 phase is most variable because, in this phase cells are not committed to DNA
replication. They can either enter resting state or progress to next cell division.
38. Ans. (a) Topoisomerase II causes break in strands; (c) At G2-M phase there is loss
of inhibitors controlling cell-cycle and (d) Decrease in telomerase activity causes
anti-tumor effects. (Ref: Harrison 16th/453,
454, Robbins 7th/43, 292)
• Topoisomerase I nicks DNA, relieving torsional tension of the replicating helix.
• Topoisomerase II introduces the double strand break to avoid DNA tangle.

• p53 it is a tumor supresser gene.


• With each cell division, there is some shortening of specialized structures called telomeres
(present at ends of chromosomes). In germ cells, telomere shortening is prevented by the
enzyme telomerase. This enzyme is absent from most somatic cells, and hence they suffer
progressive loss of telomeres. Introduction of telomerase into normal human cells causes
considerable extension of their life span, thus supporting that telomerase loss is causally
associated with loss of replication activity. So, decrease telomerase activity causes anti
tumor effect.
• Loss of inhibitors controlling cell cycle occurs at G1-S phase/G2-M phase.

39. Ans. (a) Proto-oncogenes are activated by chromosomal translocation; (b)


Malignant transformation involves accumulation of mutations in the proto-
oncogenes and tumor suppressor gene in stepwise fashion; (c) Point mutation of
somatic cells; (e) At G2-M phase there is loss of inhibitors controlling cell cycle
(Ref: Robbins’ 9/e p284-286)
Activation of proto-oncogene results in cancer causing oncogenes by following mechanisms:
1. Single point mutation e.g. ras oncogene.
2. Gene amplification e.g. n-myc amplification in neuroblastoma.
3. Chromosomal translocation e.g. t (8:14) in Burkitt lymphoma results in c-myc over-expression
and hence the tumor.
4. Promoter insertion.
5. Enhancer insertion.

The cell cycle has its own internal controls called checkpoints. There are two main
checkpoints:
1. G1-S transition- The S-phase is the point of no return in cell cycle. G1-S checkpoint
checks for DNA damage and prevents replication of defective cells.
2. At G2M transition- The G2M checkpoint monitors the completion of DNA replication
and checks whether the cell can safely initiate cell division. Loss of inhibitors at this
stage can lead to division of faulty cells and can lead to carcinogenesis.
• Defect in cell-cycle checkpoint components is a major cause of genetic instability in
cancer cells.
40. Ans. (c) G1- S phase (Ref: Robbins 9/e p294-295)
41. Ans. (a) Fragile X syndrome (Ref: Robbins 9/e p169)
42. Ans. (b) Cyclin B (Ref: Robbins 8th/286, 7th/290, 9/e p26)
43. Ans. (c) p53 (Ref: (Ref: Robbins Illustrated 7/e p p 292,
(Ref: Robbins 8/e p286, 9/e p25-26)
Cyclins form complex with cyclin dependent kinases and regulate the transition of cell cycle
from one stage to the other. These CDK complexes in turn are regulated by CDK
inhibitor. The inhibitors control the cell cycle by balancing the activity of CDKs. The
signals from these inhibitors determine whether a cell progresses through the cell cycle.
Changes in the level of these inhibitors may occur in some tumors, or possibly in aging cells.
• The cyclin dependent kinase inhibitors are

- p21 - p27 - p57 - p15 - p16 - p18 - p19

44. Ans. (b) M–phase

• Cells are most radiosensitive in G2M interphase


• Cells are least radiosensitive in S phase

45. Ans. (a) Gastric cancer (Ref: Robbins 9/e p291)


Cadherin is derived from the “calcium-dependent adherence protein.” It participates in
interactions between cells of the same type. The linkage of cadherins with the
cytoskeleton occurs through the catenins. The cell -to-cell interactions mediated by
cadherin and catenins play a major role in regulating cell motility, proliferation, and
differentiation and account for the inhibition of cell proliferation that occurs when cultured
normal cells contact each other (“contact inhibition”).
• Reduced function of E-cadherin is associated with certain types of breast and gastric cancer.
• Mutation and altered expression of the Wnt/β-catenin pathway is implicated in gastrointestinal
and liver cancer development.

46. Ans. (b) p53 (Ref: Robbins 8/e p274, 290, 9/e p293-294)
47. Ans. (b) Bone marrow hyperfunction
(Ref: Robbins 8th/663; 7th/647, 9/e p630)
48. Ans. (a) Predicting therapeutic response
(Ref: Robbins 8th/1090, 9/e p1062)
Friends, direct quote from Robbins….’HER2/neu overexpression is associated with
poorer survival but its main importance is as a predictor of response to agents that
target this transmembrane protein (examples trastuzumab or lapatinib).’
• The overexpression is due to amplification of the gene HER2/neu located on
17q21.
49. Ans. (a) Osteosarcoma (Ref: Robbins 9/e p293)
• Retinoblastoma is the most common primary intraocular malignancy of
children. Involvement of both eyes with pineal gland is called as trilateral
retinoblastoma.
• The pinealoblastoma in association with retinoblastoma is a primary tumor.
• In approximately 40% of cases, retinoblastoma occurs in individuals who inherit a
germ-line mutation of one RB allele. This variant of retinoblastoma (familial
retinoblastoma) is inherited as an autosomal dominant trait and is associated with
osteosarcoma. Osteosarcoma is therefore the commonest secondary maligancy
associated with retinoblastoma.
50. Ans. (a) Autosomal dominant
(Ref: Robbins 8th/302, Harrison 665, 9/e p314-315)
• Fanconi’s anemia is an autosomal recessiveQ disease characterized by
progressive pancytopeniaQ, increased risk of malignancy (solid tumors and
AMLQ) and congenital developmental anomalies like short stature, café au lait
spots, abnormalities affecting thumb, radius and genitourinary tract.
• Fanconi’s anemia is associated with BRCA gene. The Fanconi anemia proteins and
BRCA proteins form a DNA-damage repair proteins to correct intrastrand and
interstrand DNA cross links induced by chemical cross-linking agents.
51. Ans. (d) Wild/non-mutated form is associated with increased risk of childhood
tumors
(Ref: Robbin 7th/302-303, Harrison 17th/499-500, 8th/290-2, 9/e p294)
• p53 gene is a tumor suppressor gene also known as “guardian of the genome”Q
located on short arm of chromosome 17Q (17p). Its wild/non mutated form is
associated with reduced risk of tumors.
52. Ans. (c) Sis (Ref: Robbins 7th/182)
• A number of nuclear transcription factors are the products of oncogenes like myc,
fos, jun, myb and rel. Out of these myc is most commonly involved in tumorsQ.
• SIS oncogene is the only example of a growth factor oncogene in the given options.
Its over expression is seen in cancers like astrocytoma and osteosarcoma. The
other growth factor are described is text.
53. Ans. (a) Retinoblastoma (Ref: Robbins 7th/1442;
Neuropathology for the Neuroradiologist: Rosettes and Pseudorosettes by F.J.
Wippold and A. Perry)
Rosettes consist of a halo or spoke-wheel arrangement of cells surrounding a central core or
hub.
Rosettes may be considered primary or secondary manifestations of tumor architecture.
Primary rosettes form as a characteristic growth pattern of a given tumor type, whereas
secondary rosettes result from the influence of external factors on tumor growth.
*Neuropil-rich rosettes are referred to as pineocytomatous rosettes in pineocytomas and neurocytic
rosettes in central neurocytoma. These are similar to the Homer Wright rosette, but they are generally
larger and more irregular in contour.
54. Ans. (c) Promotion of DNA repair
(Ref: Robbins 7th/293, 295), http://www.nature.com/scitable/topicpage/proto-oncogenes-
to-oncogenes-to-cancer-883 by Heidi Chial, 9/e p284)
The normal cellular counterpart of oncogene is known as proto-oncogene. Proto-
oncogenes are important for cellular function related to growth and proliferation. Proteins
encoded by these genes may function as growth factor ligands and receptors, signal
transducers, transcription factors and cell cycle components.
Chial writes that ‘proto-oncogenes encode proteins that function to stimulate cell division,
inhibit cell differentiation, and halt cell death. All of these processes are important for
normal human development and for the maintenance of tissues and organs. Oncogenes,
however, typically exhibit increased production of these proteins, thus leading to
increased cell division, decreased cell differentiation, and inhibition of cell death’. So, we
can say that ‘These genes may also inhibit apoptosis’.
Promotion of DNA repair is the function of tumor suppressor genes. Promotion of DNA
repair is protective from oncogenesis and is not the function of proto-oncogenes.
55. Ans. (d) Rb
(Ref: Robbins 7th/300, Harrison 17th/499, 496, 9/e p290)
• Tumor suppressor genes are the genes whose products down regulate the cell
cycle, and thus apply brakes to cellular proliferation.
• Rb gene is a tumor suppressor gene whereas Myc, fos and ras are all examples of
proto-oncogenes.
56. Ans. (d) Colon, endometrium, ovary
(Ref: Harrison 17/page 575, Robbins 8th/821-822, 9/e p810)

• Lynch syndrome is an autosomal dominantQ disorder


• It is also called as Hereditary Non-polyposis Colon Cancer (HNPCC) syndromeQ
• It is caused because of defective DNA repair genesQ leading to microsatellite instability.
• There is increased chance of multiple cancers (colorectal area, endometrium, ovary, stomach,
ureter, brain, small intestine, hepatobiliary tract and skin)

57. Ans. (c) Retinoblastoma (Ref: Robbins 9/e p292-293)


58. Ans. (d) ras (Ref: Robbins 7th/295, 9/e p286)
59. Ans. (b) Autosomal dominant
(Ref: Robbins 7th/1442, 299-300, 9/e p292-293)
60. Ans. (a) p53 (Ref: Robbins 7th/302, 303, 304, 9/e p293)
61. Ans. (c) Its activity in the cells decreases following UV irradiation and stimulates
cell cycle
(Ref: Robbins 7th/302, 303, 8th/290-291, 9/e p293-294)
• p53 is a tumor suppressor gene, located on chromosome 17. It is also called as
“Guardian of the genome”.
• At the time of DNA injury following irradiation, its level increases and it acts to
cause cell cycle arrest (G1/S)
• The cell cycle arrest is to allow time for DNA repair. If repair is unsuccessful, p53
causes apoptosis of the cell by activating bax (apoptosis inducing gene). So, any
exposure to UV irradiation would cause increased activity of p53 gene resulting in
apoptosis and cell death.
62. Ans. (b) GTPase activating protein (Ref: Robbins 9/e 286)
Activated ras is present in association with GTP. Enzyme GTPase will degrade GTP to GDP
and result in inactivation of ras. Thus, GTPase activating protein will counteract the
activation of ras.
63. Ans. (b) p53 (Ref: Robbins Illustrated, 9/e 294)
• Mutation in p53 tumor suppressor gene is strongly associated with breast cancer, as
well as many other sarcomas and carcinomas. This condition is called as Li-
Fraumeni syndrome.
64. Ans. (a) Important for normal cell growth; (b) May get converted into oncogenesis;
(c) C-myc over-expression causes lymphoma (Ref: Robbins 9/e 284)
65. Ans. (c) They are transduced from virus infected cells (Ref: Ananthnarayan
7/580-1, Robbins’ 7th/293, 302)
• Viral oncogenes (V-onc) commonly known as ‘cancer genes’ which encode
proteins triggering transformation of normal cells into cancer cells.
• Proto-oncogenes are the normal cellular genes that promote normal growth and
differentiation.
• Oncogenes isolated from cancer cells are called cellular oncogenes (C-onc).
• Proto-oncogenes are converted to oncogenes and cause cancer by:
– Transduction into retrovirus (V-oncs) or
– Changes in situ that affect their expression and function thereby converting
them into cellular oncogene (C-oncs).
• The transduction of oncogenes by the virus (e.g. retrovirus) is through recombination
with DNA of a (normal) host cell that had been infected by the virus. Thus, they are
of host cell origin. The virus act as transducing agent, carrying oncogenes from one
cell to another.

Viral oncogenes do not contain introns and that’s how they are different from human oncogenes.

66. Ans. (a) Suppression of p53 protein; (c) bcl-2


(Ref: Robin’s 7th/306, 274, 9/e 302)
• Cell survival would be seen when they are prevented from apoptosis. Genes that
favor cell survival and protect from apoptosis are: - bcl-2 , bcl-xL
• Genes that favor programmed cell death are: bax, bad, bcl-xL and p53.
67. Ans. (a) Proto-oncogenes; (b) Tumor suppressor genes; (d) DNA repair genes
(Ref: Robbins 7th/290, 298, 9/e 280)
68. Ans. (c) Nucleotide excision repair (Ref: Harrison 17th/d/387
Robbins 7th/d 287, 9/e 314)
69. Ans. (b) EGF Receptor (Ref: Robbins 7th/295, 9/e 285)
70. Ans. (d) C-myc (Ref: Robbins 7th/295, 300, 9/e 288)
• C-myc is a proto-oncogene of transcriptional activator category and is associated
with Burkitt’s lymphoma.
71. Ans. (a) Formation of the new blood vessels (Ref: Robbins
8th/297; 7th/71-72, 9/e 305)
72. Ans. (a) RET (Ref: Robbins 8th/1124-1126, 9/e 284)
73. Ans. (a) Chromosome 5 (Ref: Robbins 9/e 296)
74. Ans. (c) PTEN (Ref: Robbins 8/e p294, 9/e 298)

Direct quote from Robbins… “PTEN (Phosphatase and tensin homologue) is a membrane-associated
phosphatase encoded by a gene on chromosome 10q23 that is mutated in Cowden syndrome, an
autosomal dominant disorder marked by frequent benign growths, such as tumors of the skin appendages,
and an increased incidence of epithelial cancers, particularly of the breast, endometrium, and thyroid.

75. Ans. (b) Neurocysticercosis


(Robbins 9th/ 1203,1312, 1339)
• Retinoblastoma has the presence of Flexner-Wintersteiner rosette’s and fleurettes
reflecting photoreceptor differentiation. …. (Robbins 9th/1339)
• PNET (Primitive Neuro-Ectodermal Tumour): It is composed of sheets of uniform
small, round cells that are slightly larger than lymphocytes. ….
(Robbins 9th/1203)
• Medulloblastoma: The tumor may express neuronal (neurosecretory) granules, form
Homer- Wright rosettes…. (Robbins 9th/1312)
For details see the answer of a question in chapter of ‘Neoplasia’
76. Ans. (c) G1 – S phase (Ref: Robbins 8/e p290-1, 9/e 294)
• The cell cycle has its own internal controls, called checkpoints. There are two main
checkpoints, one at the G1/S transition and another at G2/M.
• In the G1/S checkpoint, cell-cycle arrest is mostly mediated through p53, which
induces the cell-cycle inhibitor p21.
• Arrest of the cell cycle by the G2/M checkpoint involves both p53-dependent (via
cyclin A/cdK-2) and independent (via cdc 25) mechanisms.
As can be deduced from above mentioned information that the p53 is associated with both the
types of checkpoints. However, G2/M checkpointcan take place even without p53.
Hence, option “c” is the preferred answer.
77. Ans. (a) RET gene (Ref: Robbins 9/e 284)
78. Ans. (a) Beta-2 macroglobulin (Ref: Robbins 8/e p327)
Beta-2 microglobulin and not beta macroglobulin may be used as a tumor marker (as in
multiple myeloma).
79. Ans. (d) Transcription activator (Ref: Robbins 8/e p288)
• A host of oncoproteins, including products of the MYC, MYB, JUN, FOS, and REL
oncogenes, are transcription factors that regulate the expression of growth-
promoting genes, such as cyclins.
• Out of all these,MYC is most commonly involved in human tumorsQ
• MYC gene is associated with “Conflict model” in carcinogenesis
Tumors associated with different subtypes of MYC
C-MYC Translocation Burkitt lymphoma
N-MYC Amplification Neuroblastoma, small-cell carcinoma of lung
L-MYC Amplification Small-cell carcinoma of Lung

80. Ans. (d) Osteosarcoma (Ref: Robbins 8/e p293)


81. Ans. (b) Medullary carcinoma thyroid
(Ref: Robbins 8/e p280, 9/e 284)
The RET protein is a receptor for the glial cell line–derived neurotrophic factor and structurally
related proteins that promote cell survival during neural development. RET is normally
expressed in neuroendocrine cells, such as parafollicular C cells of the thyroid, adrenal
medulla, and parathyroid cell precursors. Point mutations in the RET proto-oncogene
are associated with dominantly inherited MEN types 2AQ and 2BQ and familial
medullary thyroid carcinomaQ.
RET gene mutation is more commonly associated with medullary thyroid cancer than
pheochromocytoma.
Clinical significance of RET… (Ref: Robbins 8/e p)

All individuals carrying germline RET mutations are advised to undergo prophylactic thyroidectomy to
prevent the inevitable development of medullary carcinomas.

82. Ans. (d) RB…explained earlier (Ref: Robbins 9/e p291)


83. Ans. (c) Xeroderma pigmentosum
(Ref: Robbins 8/e p275, 9/e 314)
84. Ans. (d) Rb (Ref: Robbins 8/e p286, 9/e p291)
85. Ans. (b) Retinoblastoma (Ref: Robbins 9/e p290)
86. Ans. (b) Pinealoblastoma (Ref: Robbins 9/e p1339)
87. Ans. (a) Overexpression (Ref: Robbins 9/e p284)
HER-2/neu gene is a proto-oncogene whose amplification/ overexpression cause breast
carcinoma.
88. Ans. (a) Ret proto oncogene (Ref: Robbins 9/e p1099)
89. Ans. (b) Chromosome 17 (Ref: Robbins 9/e p1312)
90. Ans. (b) Renal cell carcinoma (Ref: Robbins 9/e p1134)
91. Ans. (b) EBV (Ref: Robbins 8th/230,
Harrison 18th/921, 1124, 9/e 327)
Infectious Agent Lymphoid Malignancy

Epstein-Barr virus • Burkitt’s lymphoma


• Post–organ transplant lymphoma
• Primary CNS diffuse large B cell lymphoma
• Hodgkin’s disease
• Extranodal NK/T cell lymphoma, nasal type
HIV • Diffuse large B cell lymphoma
• Burkitt’s lymphoma
Hepatitis C virus • Lymphoplasmacytic lymphoma

Helicobacter pylori • Gastric MALT lymphoma

Human herpesvirus 8 • Primary effusion lymphoma


• Multicentric Castleman’s disease
HTLV-I • Adult T cell leukemia/lymphoma

92. Ans. (a) MALToma (Ref: Harrison 17th/1858,


Robbins 8th/316, 9/e 329)
93. Ans. (d) Burkitt’s lymphoma (Ref: Robbins 7th/814,
Robbins 8th/315-6, 9/e 329)
94. Ans. (b) Non-small Cell Carcinoma of Lung (Ref:
Robbins 8th/277, 878, 9/e 325-329)
CANCERS ASSOCIATED WITH INFECTIOUS AGENTS
Opisthorchis, cholangitis Cholangiosarcoma, colon Liver flukes (Opisthorchis
carcinoma viverrini)
Bile acids
Chronic cholecystitis Gallbladder cancer Bacteria, gallbladder stones
CANCERS ASSOCIATED WITH INFECTIOUS AGENTS
Gastritis/ulcers Gastric adenocarcinoma, MALT Helicobacter pylori
Hepatitis Hepatocellular carcinoma Hepatitis B and/or C virus
Mononucleosis B-cell non-Hodgkin lymphoma and Epstein-Barr virus
Hodgkin lymphoma, nasopharyngeal
cancer
AIDS Non-Hodgkin lymphoma, squamous Human immunodeficiency virus,
cell carcinoma, Kaposi’s sarcoma human herpesvirus type 8

Osteomyelitis Carcinoma in draining sinuses Bacterial infection


Pelvic inflammatory Ovarian carcinoma, cervical/anal Gonorrhea, chlamydia, human
disease, chronic cervicitis carcinoma papillomavirus

Chronic cystitis Bladder, liver, rectal carcinoma Schistosomiasis

Non small cell lung cancer is not reported to be associated with any infectious organism.
95. Ans. (a) Angiogenesis (Ref: Harrison 17th/.
509, Robbins 9/e 305-306)
Metastasis is a complex series of steps in which cancer cells leave the original tumor site and
migrate to other parts of the body via the bloodstream or the lymphatic system. To do so,
malignant cells break away from the primary tumor and degrade proteins of the
extracellular matrix (ECM). One of the critical events required for metastasis is the
growth of a new network of blood vessels, called tumor angiogenesis.

• Without vascularization or angiogenesis, the tumor can grow only 1-2 mmQ. Vessels are also required
for nutrition.
• Vascularisation promoted by VEGF and bFGF and inhibited by Angiostatin, Endostatin and
TumstatinQ.
• It has been found that angiogenesis inhibitors would therefore prevent the growth of metastases.

96. Ans. (b) Papilloma viruses produce tumors in animals but not in humans
(Ref: Harrison 17th/487)
All the options mention about the carcinogens. “Human papilloma virus is the most common
etiological factor for cervical cancer”
97. Ans. (d) Hepatitis C virus (Ref: Robbins 8th/315, 9/e 328)
Hepatitis C virus (HCV) is only oncogenis RNA virus in the options. Others mentioned are
oncogenic DNA viruses.
98. Ans. (c) Epstein-Barr Virus
(Ref: Robbins 9/e 328)
LMP-1 gene plays a role in oncogenesis induced by EBV. For details, see text.
99. Ans. (b) UVB rays (Ref: Robbins 7th/323 , 9/e 324)
100. Ans. (d) Testicular seminoma (Ref: Robbins 8th/989)
101. Ans. (d) Thyroid (Ref: Robbins 9/e 415)
102. Ans. (c) Angiosarcoma (Ref: Robbins 9/e 519)
103. Ans. (a) Cytomegalovirus (Ref: Robbins 9/e 325-326)
104. Ans. (d) Chronic lymphocytic leukemia
(Ref: Robbins 9/e p 431)
The main sources of ionizing radiation are x-rays and gamma rays (electromagnetic waves of
very high frequencies), high-energy neutrons, alpha particles (composed of two protons
and two neutrons), and beta particles, which are essentially electrons.
Diagnostics is the most common source of radiation exposure in human beings.
Cancers associated with radiation Cancers not associated with radiation
• ALL, AML and CML • CLL
• Cancer of thyroid, breast and lung. • Hodgkins lymphoma
• Cancer of CNS, bladder, ovary • Cancer prostate/testis/cervix

105. Ans. (c) Clonorchiasis (Ref: Robbins 8/e p880, 9/e p 874)
The following two parasites have definitive etiological association with malignancies:

• Clonorchis sinensis: cholangiocarcinomaQ


• Opisthorchis viverrini: cholangiocarcinomaQ
• Schistosoma haematobium : squamous cell cancer of urinary bladderQ
• Schistosoma japonicum: colorectal cancerQ

106. Ans. (d) HHV (Ref: Robbins 8/e p313, 9/e p 254)
107. Ans. (b) Stimulates formation of pyrimidine dimers
(Ref: Robbins 8/e p312, 9/e p314)
Direct quote from Robbins.. “The carcinogenicity of UV-B light is attributed to its
formation of pyrimidine dimers in DNA”. This type of DNA damage is repaired by the
nucleotide excision repair pathway. The importance of the nucleotide excision repair
pathway of DNA repair is illustrated by the high frequency of cancers in individuals with
the hereditary disorder xeroderma pigmentosum..
108. Ans. (c) Angiosarcoma of liver (Ref: Robbins 9/e 519)
Angiosarcoma of the liver is a highly aggressive tumor which is associated with exposure
to:
• Vinyl chlorideQ,
• ArsenicQ, or
• ThorotrastQ.
Thorotrast is a suspension containing particles of the radioactive compound thorium
dioxide. It emits alpha particles due to which it has been found to be extremely
carcinogenic.
109. Ans. (c) Thyroid
(Ref: Robbins 8/e p312, 425 and internet, 9/e p 325)
The most radiosensitive organ sites in children in order of sensitivity are the thyroid gland,
breasts, bone marrow (leukemia), brain and skin.
110. Ans. (b) Lymphocytes (Ref: Robbins, 9/e p430)
• The most radiosensitive cell in the blood is the lymphocytesQ.
• The least radiosensitive cell in the blood is the plateletsQ.
• DNAQ is the most sensitive intracellular organelle to radiation.
111. Ans. (c) Gray (Ref: Robbins 9/e p428)
Gray (Gy) is a unit that expresses the energy absorbed by the target tissue per unit
mass.
112. Ans. (a) Human T-cell Lymphotropic Virus…was given in this book in chapter 6:
Immunity
113. Ans. (b) Alcohol (Ref: Robbins 9/e p589)
114. Ans. (d) E6E7 (Ref: Robbins 9/e p326)
The oncogenic potential of HPV can largely be explained by the activities of the two viral
genes encoding E6 and E7.
115. Ans. (c) Benzene (Ref: Robbins 9/e p413)
116. Ans. (b) myc (Ref: Robbins 9/e p284)
117. Ans. (b) AD inheritance (Ref: Robbins 9/e p304)
118. Ans. (b) Medullary carcinoma (Ref: Robbins 9/e p1065)
Among cancers arising in BRCA1 carriers, 13% are of medullary type, and up to 60% have a
subset of medullary features. Please remember, BRCA1 is also associated with
mucinous carcinomas.
119. Ans. (c) Endometrial carcinoma (Ref: Robbins 9/e p291)
120. Ans. (d) 11p13 (Ref: Robbins 9/e p298)
121. Ans. (c) Inhibin (Ref: Robbins 8th/1050, 9/e p1032)
Granulosa cell tumor

• The most common type of ovarian tumor that is composed of cells that stain positively with inhibinQ.
• Histologically, the cells may form Call-Exner bodiesQ
• The tumor cells may secrete estrogens and cause precocious sexual development in girls or
increase the risk for endometrial hyperplasia and carcinoma in women.
• Less commonly granulosa cell tumors can secrete androgens and produce masculinization.

Concept: for future exam

• Tumor cells in Sertoli-Leydig tumors (Androblastomas) may stain positively with inhibin, but Call-
Exner bodies are not present. Sertoli-Leydig tumors also may secrete androgens and produce
virilization in women.
• The granulosa cell tumors vary in their clinical behavior, but they are considered to be potentially
malignant.

122. Ans. (a) Hepatoblastoma (Ref: Robbins 8th/327, 7th/339, 9/e p869-870, Harsh
Mohan 6th/637)
• AFP is glycoprotein synthesized in fetal life by yolk sac, fetal liver and fetal
gastrointestinal tract. It is a marker of hepatocellular cancer and non-
seminomatous germ cell tumors of testes. Elevated plasma AFP is also found
less regularly in carcinomas of the colon, lung, and pancreas.

Important points about hepatoblastoma

• Arising from hepatic parenchymal cells


• The most common tumor of young childhood
• More commonly seen in boys
• The concentration of AFP is very high
• A characteristic feature of hepatoblastomas is the frequent activation of the Wnt/β-catenin signaling
pathway by stabilizing mutations of β-catenin, contributing to the process of carcinogenesis.

123. Ans. (c) Osteosarcoma > (a) Multiple myeloma


(Ref: Journal….Cancer 42:603-610, 1978)
The journal writes that………….. “The clinical manifestations, resulting from the
production of metabolically active substances by neoplastic tissue, have been labeled
paraneoplastic syndromes. Adolescent patients with primary osteosarcoma demonstrate
abnormal glucose, insulin and growth hormone responses to oral glucose loading in 78%
of the study population. No statistical association exists between any two of the three
factors and, therefore, no primary abnormality can be identified. High somatomedin levels
were noted in 72% of the group studied accompanied by simultaneous elevations of
growth hormones. Studies of adrenal, gonadal and gonadotropic hormones were
essentially normal, thereby ruling these out as associated endocrine abnormalities”.
• Hyperglycemia is also associated with chondrosarcoma and fibrosarcoma.
• Some reports associate hyperglycemia with multiple myeloma too but we would
prefer osteosarcoma as the answer here because the chances of hyperglycemia is
more with osteosarcoma than multiple myeloma.
124. Ans. (a) SIADH (Ref: Robbins 8th/636-637, 9/e p626-627)
Thymoma is the commenest anterior mediastinal tumor which causes symptoms due to
compression on the mediastinal structures. It is associated with the following
paraneoplastic syndromes:
• Myasthenia gravis (most common)Q
• Acquired hypogammaglobulinemiaQ
• Pure red cell aplasiaQ
• Graves disease
• Pernicious anemia
• Dermatomyositis-polymyositis
• Cushing syndrome.

Epstein-Barr virus may be associated with thymomas.


Thymoma is the commonest anterior mediastinal tumor.

125. Ans. (d) Involves aorta and its branches early (Ref:
Robbins 8th/475-479, 9/e p476-479)
See text in chapter-18
126. Ans. (a) Prostate (Ref: Robbin 7th/354, 9/e p332)
• Migratory thrombophlebitis (Trousseau signQ) is particularly associated with
adenocarcinomas of the pancreas, colon and lungQ because of associated
paraneoplastic syndrome resulting in hypercoagulability.
127. Ans. (c) Malignant melanoma: (Ref Anderson
pathology 144-152, 9/e p1149)

• Malignant melanoma expresses HMB 45, S-100 and vimentin.


• HMB 45 is present in melanosomes and is more specificQ.
• S-100 is more sensitiveQ but is non-specific (also present in Langerhans’ cell histiocytosis, neural
tumors, and sarcomas like liposarcoma and chondrosarcoma)

128. Ans. (a) Hepatoblastoma (Ref: Robbins 8th/327,


7th/339, 9/e p869-870, Harsh Mohan 6th/637)
129. Ans. (c) Carcinoma (Ref: Robbins 9/e p334)
Immunohistochemistry is used for making categorization of undifferentiated tumors. The
important examples include: Cytokeratin (carcinoma), Desmin (Leiomyoma and
Rhabdomyosarcoma) and vimentin (Sarcomas).
130. Ans. (b) Pancreatic neuroendocrine tumor
(Ref: Harrison 17th/2354, Robbins 9/e 1121)
• Gastrin is secreted by Gastrinomas, which are neuroendocrine tumors most
commonly found in duodenum.
• Marker of medullary carcinoma of thyroid is calcitonin and GIST is CD117 (c-kit).
131. Ans. (a) Alpha-hCG (a-hCG) (Ref: Robbins 7th/1045, 1046,
9/e 337-338, C.S.D.T. 11th/1071)
Human chorionic gonadotropin (hCG)
The beta subunit of hCG is typically measured as a tumor marker because it has unique
sequence that are not shared with other human glycoprotein hormones. So, it is quite
specific.

Alpha hCG is not used as tumor marker because it is similar to the FSH, LH and TSH. So there can
be cross reactivity between beta subunits of these hormones.
132. Ans. (a) Seminoma (Ref: Harrison
17th/1925 & Robbins 8th//988-989)
• The normal serum alkaline phosphate consists of 4 isoenzymes secreted from the
following sites:
(a) Liver (b) Bone
(c) Intestine (d) Placenta
They are best differentiated by electrophoresis. Another approach is based on the
differentiation between the different isoenzymes on the basis of heat susceptibility.
• Alkaline phosphatase from individual tissues differ in susceptibility to inactivation by heat. The
finding of an elevated serum alkaline phosphatase level in a patient with a heat-stable fraction
strongly suggests that the placenta or a tumor is the source of the elevated enzyme in serum.
Susceptibility to inactivation by heat increases, respectively, for the intestinal, liver, and
bone alkaline phosphatase, bone being by far the most sensitive and the liver being most
resistant.
Mnemonic: bone burns but liver lasts

• The conditions having elevated placental alkaline phosphatase include:


– Seminoma
– Choriocarcinoma
– Third trimester of pregnancy
133. Ans. (b) Hypercalcemia (Ref: Harrison, 17th/1736)
Tumor lysis syndrome is associated with hyperphosphatemia due to release of intracellular
phosphate by the destroyed cancer cells. This is followed by a decrease in serum
calcium levels.
134. Ans. (a) Screening of a cancer; (b) Follow up of a cancer patient, esp. for knowing
about recurrence; (d) For monitoring the treatment of a cancer
(Ref: Robbins’ 7th/338, 9/e p338)
• Uses of Tumor Markers
– Screening of Cancer e.g. in prostate carcinoma (PSA), ovarian carcinoma (CA-
125).
– Follow-up a cancer patient especially for knowing recurrence e.g. AFP in
hepatocellular carcinoma, CEA in colon carcinoma.
– For monitoring of a cancer e.g. AFP + HCG in testicular· malignancy, AFP in
hepatocellular carcinoma.
– Prognosis of a cancer e.g. HCG; AFP in testicular malignancy, CEA in colon
Ca.

Tumor markers are not specific, so, cannot be used for confirmation of diagnosis. Confirmation is done by
biopsy

135. Ans. (a) Useful for screening of Carcinoma colon; (c) Helpful for follow-up after
resection; (d) Levels decrease immediately after resection of tumor
(Ref: Harrison’ l6th/530, 531, Robbin 9/e p338)
• Carcino embryonic antigen is used in Colon carcinoma as follows;
– For screening of carcinoma colon
– For follow-up after resection
– Early knowledge about tumor recurrence and metastasis
– Levels of CEA are elevated in 70% of patients but are poorly correlated with
cancer stage.
– After complete surgical resection, CEA level should be normalized, persistent
levels imply a poor prognosis.
– Diagnosis of colon carcinoma
is confirmed by colonoscopy and biopsy as some benign and other
malignant conditions also show high values of CEA:
• Pancreatic, breast and stomach carcinoma
• Alcoholic cirrhosis
• Hepatitis
• IBD
136. Ans. (c) Ovarian ca
(Ref: Harrison 16th/439, Robbins 9/e p337)
137. Ans. (b) Hand and feet bones:
(Ref: Robbins 9/e 1207)
• Metastasis may occur any bone but most commonly involve axial skeleton (e.g.
vertebra, pelvis, ribs. skull. sternum) > Proximal femur > humerus.
• Metastasis in small bone of hand and feet are uncommon and usually originates in
cancer of lung, kidney and colon

• Skeletal metastasis are typically multifocal, however carcinoma of kidney and thyroid produce
solitary lesions.

138. Ans. (d) A technique for raising monoclonal antibodies


(Ref: Harsh Mohan 6th/15)
139. Ans. (a) Follicular lymphoma (Ref: Robbin 9/e 594-595)
• The hallmark of follicular lymphoma is a (14; 18) translocation, which leads to the
juxtaposition of the IgH locus on chromosome 14 and BCL 2 locus on chromosome
18.
• This translocation is seen in most but not all follicular lymphomas and leads to over-
expression of BCL2 protein.

• B-cell lymphoma is associated with breakpoint involving the BCL 6 locus on chromosome 3.
• Mantle cell lymphoma is associated with a locus on chromosome 11 variously known as BCL1 or
PRAD1.

140. Ans. (c) Hepatoma (Ref: Robbins 7th/338, 9/e p337)


AFP is glycoprotein synthesized in fetal life by yolk sac, fetal liver and fetal GIT. It is a marker
of HCC, hepatoma and non-seminomatous germ cell tumors of testes.
141. Ans. (a) Yolk sac tumor (Ref: Robbins 9/e p977)
142. Ans. (a) Disseminated cancer; (this is known as Trosseau signQ) (Ref: Robbins
8th/322, 7th/335, 9/e p332)
143. Ans. (b) Parathyroid hormone related peptide
(Ref: Robbins 8th/728, 7th/333, 9/e p330)
144. Ans. (c) Cerebellar hemangioblastoma
(Ref: Robbins 8th/665, 7th/334, 9/e p331)
145. Ans. (d) Bladder carcinoma (Ref: Robbins 9/e p338)
146. Ans. (c) Ulcerative colitis (Ref: Robbins 9/e p338)
147. Ans. (b) Anterior abdominal wall(Ref: Robbins 9/e p1222)
148. Ans. (d) Both (b) and (c) (Ref: Robbins 9/e p337)
149. Ans. (b) Carcinoma colon (Ref: Robbins 9/e p337)
150. Ans. (b) AFP (Ref: Robbins 8th/327, 7th/338, 9/e p873)
151. Ans. (b) Neuroblastoma (Ref: Robbins 9/e p476)
152. Ans. (d) Cholangiocarcinoma - AFP
(Ref: Robbins 8th/880-881, 7th/926-927, 9/e p337)
153. Ans. (a) Chondrosarcoma (Ref: Robbins 8th/1230)
154. Ans. (b) Hepatocellular carcinoma of liver
(Ref: Robbins 8th/876, 7th/338, 9/e p873)
155. Ans. (d) Kidney tumor (Ref: Robbins 9/e p337)
156. Ans. (a) Neuroblastoma (Ref: Robbins 9/e p476)
157. Ans. (a) CEA……….See text (Ref: Robbins 9/e p337)
158. Ans. (a) Prostate (Ref: Robbins 8/e p1235, 9/e p1207)
• Carcinomas of the kidney, lung, and gastrointestinal tract and malignant
melanoma produce lytic bone destruction.
• Other metastases elicit a sclerotic response, particularly prostate adenocarcinoma,
which may do so by secreting WNT proteins that stimulate osteoblastic bone
formation.
• Most metastases induce a mixed lytic and blastic reaction
159. Ans. (b) β- HCG
(Ref: Robbins 8/e p474, 9/e p474-475 The Essentials of Clinical Oncology p490)

• Sacrococcygeal teratomas are the most common teratomas of childhood, accounting for 40% or more
of cases). They occur with a frequency of 1 in 20,000 to 40,000 live births, and are four times more
common in girls than boys
• Serum alpha fetoprotein is a useful marker for sacrococcygeal teratoma. Some books mention that even
beta HCG is elevated in some patients.

160. Ans. (b) CDKN2A (Ref: Robbins 8/e p1174, 9/e p1147)
Please do not get confused with the first option friends. Melanomas are associated with N-Ras
and not N-myc.
Coming to the other options,
Direct quote from Robbins…. “The CDKN2A gene (is mutated in approximately 40% of
pedigrees with autosomal dominant familial melanoma”.
161. Ans. (a) Chromogranin (Ref: Robbins 9/e p717)
Direct quote from Robbins.. “The occurrence of neurosecretory granules, the ability of some
of these tumors to secrete polypeptide hormones, and the presence of neuroendocrine
markers such as chromogranin, synaptophysin and CD57 (in 75% of cases) and
parathormone-like and other hormonally active products suggest derivation of this tumor
from neuroendocrine progenitor cells of the lining bronchial epithelium.
162. Ans. (c) Cytokeratin
(Ref: Robbins 8/e p324, 9/e p334) ...see text for details
163. Ans. (a) CA-125 (Ref: Robbins 8/e p327, 9/e p337)
164. Ans. (c) PLAP (Ref: Robbin 8/e p988-9)
Robbins … “Seminoma cells are diffusely positive for c-KIT, OCT4 and placental alkaline
phosphatase (PLAP), with sometimes scattered keratin-positive cells”
165. Ans. (b) Lung
(Ref: Robbin 9/e p 1315)
166. Ans. (d) Urine contains 5H.I.A.A (Ref: Robbin 9/e p 478)
• Neuroblastoma is the most common extracranial solid cancer in childhood and the most common cancer in
infancy.
• About 90% of neuroblastomas, regardless of location, produce catecholamines, which are an important
diagnostic feature (i.e., elevated blood levels of catecholamines and elevated urine levels of the metabolites
vanillylmandelic acid and homovanillic acid.
Increased urinary 5HIAA is a feature of carcinoid tumour and not neuroblastoma.

167. Ans. (a) Pheochromocytoma (Ref: Robbins 9/e p1134)


Pheochromocytomas have been associated with a“rule of 10s”.
• 10% of pheochromocytomas are extra-adrenalQ, occurring in sites such as the organs of
Zuckerkandl and the carotid body.
• 10% of sporadic adrenal pheochromocytomas are bilateralQ.
• 10% of adrenal pheochromocytomas are biologically malignantQ, defined by the presence of
metastatic disease.
• 10% of adrenal pheochromocytomas are not associated with hypertensionQ.
• One “traditional” 10% rule that has now been modified pertains to familial cases. Now almost25%
of individuals with pheochromocytomas and paragangliomas harbor a germline mutation in the
succinate dehydrogenase genes.

168. Ans. (a) Desmin-carcinoma ...see text for details


169. Ans. (c) Desmin (Ref: Robbin 8/e p1253)
• Rhabdomyosarcoma is the most common soft-tissue sarcoma of childhood and
adolescence. It usually appears before age 20
• Ultrastructurally, rhabdomyoblasts contain sarcomeres, and immunohistochemically
they stain with antibodies to the myogenic markers desmin, MYOD1 and
myogenin.
Other questions from same topic: for AIIMS/NEET

• Most common location is the head and neck or genitourinary tract, where there is little if any skeletal
muscle as a normal constituent.
• Rhabdomyoblasts are also known as tadpole or strap cells
• Rhabdomyosarcoma is histologically subclassified into embryonal, alveolar, and pleomorphic
variants. The embryonal variant is the commonest.

170. Ans. (d) Metastasis from another solid tissue tumor


(Ref: Robbin 9/e p 1133)
Metastases to the adrenal cortex are significantly more common than primary
adrenocortical carcinomas.
Even Harrison says… “The most common cause of adrenal tumors is metastasis from another
solid tumor like breast cancer and lung cancer”.
Malignant Prevalence
Adrenocortical carcinoma 2-5
Malignant pheochromocytoma <1
Adrenal neuroblastoma <0.1
Lymphoma (incl primary adrenal lymphoma) <1
Metastases (most frequent: breast, lung) 15

171. Ans. (a) Gives an immediate definitive diagnosis of


tumor (Ref: Internet)

• The frozen section procedure/cryosection is rapid microscopic analysis of a specimen.


• It is used most often in oncological surgery.
• The quality of the slides produced by frozen section is lower than formalin-fixed paraffin-embedded
tissue processing.
• While presumptive diagnosis can be rendered in many cases, fixed tissue processing is required for
more accurate diagnosis

172. Ans. (a) Renal cell cancer (Ref: Robbins 9/e p331)
• Tumors most often associated with paraneoplastic hypercalcemia are carcinomas of
the kidney, lung, breast and ovary.
• The most common lung neoplasm associated with hypercalcemia is squamous cell
carcinoma.
173. Ans. (a) Stomach (Ref: Robbins 9/e p1034)
174. Ans. (c) Renal cell carcinoma (Ref: Robbins 9/e p515)

• Hepatocellular carcinoma and renal cell carcinoma show a striking tendency to grow within veins,
and these can ultimately occlude the IVC.
• Bronchogenic carcinoma or malignant lymphoma may cause invasion of the superior vena cava

175. Ans (b) Glycoprotein (Ref: Robbins 9/e p337)


176. Ans. (a) Fibrosarcoma (Ref: Robbins 9/e p1159)

ANNEXURE
I. Types of rosette
Types Flexner-
of Wintersteiner Homer-Wright True Ependymal Perivascular
rosette rosettes rosettes Rosette Pseudorosette Neurocytic rosette

Diagram

Feature *A halo of cells *A halo of cells *The halo-like *A halo of cells *Rosette is similar to
surrounds a largely surrounds a cluster of cells in surrounds a blood the Homer Wright
empty central hub. central hub that each rosette vessel rosette,but the
Small cytoplasmic contains a surrounds an empty *Called ‘pseudo’ central fiber-rich
extensions from the meshwork of central lumen because the neuropil island is
cells project into the fibers central structure is larger and more
lumen not actually irregular
formed by the
tumor itself, but
instead represents
a native, non-
neoplastic element
Related RetinoblastomaQ, Supratentorial Ependymoblastoma Medulloblastomas, Central
tumors Pineoblastomas, PNETs, (rare form of PNET) PNETs, neurocytoma
Medulloepitheliomas RetinoblastomaQ, Central
Pineoblastomas neurocytomas,
Glioblastomas,
Pilomyxoid
astrocytomas

II. Epigenetics in cancer

“Epigenetics” refers to factors other than the sequence of DNA that regulate gene
expression (and, thereby, cellular phenotype). These factors include histones
modifications catalyzed by enzymes associated with chromatin regulatory complexes;
DNA methylation, and other less well characterized proteins that regulate the higher
order organization of DNA (e.g., looping of enhancer elements onto gene promoters).
Epigenetic changes have important roles in many aspects of the malignant phenotype,
including the expression of cancer genes, the control of differentiation and self renewal, and
even drug sensitivity and drug resistance.

Cancers Showing Epigenetic Changes

Gene(s) Function Tumor (Approximate Frequency of Mutation)

DNMT3A DNA methylation Acute myeloid leukemia (20%)

MLL1 Histone methylation Acute leukemia in infants (90%)

MLL2 Histone methylation Follicular lymphoma (90%)

CREBBP/EP Histone acetylation Diffuse large B cell lymphoma (40%)


300

ARD1A Nucleosome Positioning/chromatin Ovarian deer cell carcinoma (60%) endometrial


remodeling carcinoma (30%-40%)

SNF5 Nucleosome Positioning/chromatin Malignant rhaboid tumor (100%)


remodeling

PERM1 Nucleosome Positioning/chromatin Renal carcinoma (30%)


remodeling
1. Which of the following is not a small round blue cell tumor?
(AIIMS Nov 2019 like pattern)
(a) Neuroblastoma
(b) Retinoblastoma
(c) Hemangioblastoma
(d) Ewing’s sarcoma
Ans. (c) Hemangioblastoma (Ref: Robbins 9th/278, 475
• Many childhood tumors have been collectively referred to as small round blue cell tumors because of their
primitive histologic appearance.
• The differential diagnosis of such tumors includes neuroblastoma, Wilms tumor, lymphoma,
rhabdomyosarcoma, Ewing sarcoma/primitive neuroectodermal tumor, medulloblastoma, and
retinoblastoma.
• If the anatomic site of origin is known, diagnosis is usually possible on histologic grounds alone.
Occasionally, a combination of chromosome analysis, immunoperoxidase stains, or electron microscopy is
required.

2. Arrangement of the CDK and cyclins in cell cycle from G1 to S checkpoint:


(AIIMS Nov 2019 like pattern)
a. CDK 2/Cyclin E
b. CDK 4/Cyclin D
c. CDK 1/Cyclin B
d. CDK 2/Cyclin A
Ans. Correct sequence is b…a…d….c (Ref: Robbins 9th /25)
The sequential appearance of the cyclins and the kinases dependent on them is as follows:
• CDK 4/Cyclin D → CDK 2/Cyclin E → CDK 2/Cyclin A → CDK 1/Cyclin B.

3. Look at the options below:


A. Promoter initiation
B. Gene amplification
C. Enhancer insertion
D. Point mutation
Which of the following is correct for the protooncogene to oncogene
transformation?
(AIIMS Nov 2019 like pattern)
a. A, B, C and D are correct
b. Only A, B and C are correct
c. Only A and B are correct
d. Only C and D are correct
Ans. (a) A, B, C and D are correct (Ref: Robbins 9th/ 284)
Protooncogenes get converted to oncogenes by any of the following mechanisms:
1. Point mutation
2. Gene amplification
3. Genetic translocation: This translocation can activate proto-oncogenes in two
ways:
– By promoter or enhancer substitution, in which the translocation results in over
expression of a proto-oncogene by swapping its regulatory elements with those of
another gene, which is highly expressed.
– By formation of a fusion gene in which the coding sequences of two genes are
fused leading to the expression of a novel chimeric protein with oncogenic
properties.
So, it is easy for you to understand friends that proto-oncogene to oncogene transformation
can be either due to promoter initiation, gene amplification, enhancer insertion as well as
point mutation.
4. Cell arrest due to DNA damage is done by:
(AIIMS May 2019 like pattern)
(a) Rb
(b) P53
(c) P16
(d) Notch signal ………proto-oncogene involved in signal transduction…leukemia and
lymphoma
Ans. (b) P53 (Ref: Robbins 9th/289)
• P53 causes cell cycle arrest and apoptosis. Acts mainly through p21 to cause cell cycle arrest.
• Causes apoptosis by inducing the transcription of pro-apoptotic genes such as BAX. Levels of p53 are
negatively regulated by MDM2 through a feedback loop.
• p53 is required for the G1/S checkpoint and is a main component of the G2/M checkpoint.

About other options,

• Rb gene is normally required for G1/S transition


• Notch signal is a proto-oncogene involved in signal transduction. It is seen in the development of leukemia
and lymphoma.
5. Which of the following malignancy is least commonly associated with lymphatic
spread?
(AIIMS May 2018 like pattern)
(a) Basal cell carcinoma
(b) Squamous cell carcinoma
(c) Malignant melanoma
(d) Merkel cell carcinoma
Ans. (a) Basal cell carcinoma
(Ref: Robbins 9th e/p 272, 1155)
Basal cell carcinoma is a slow-growing tumor that rarely metastasizes. The vast majority is
recognized at an early stage and is cured by local excision. However, a small number of
tumors (<0.5%) are locally aggressive and potentially disfiguring.
6. A 5-years old child was presented with proptosis. Microscopic examination of the
mass revealed a round cell tumor positive for desmin immunohistochemical
marker. Most likely diagnosis is:
(AIIMS May 2018 like pattern)
(a) Leukemia
(b) Embryonal rhabdomyosarcoma
(c) Lymphoma
(d) Primitive Neuroectodermal Tumor (PNET)
Ans. (b) Embryonal rhabdomyosarcoma
(Ref: Robbins 9th e/11, Ocular Pathology E book, p 491)
• Desmin is a marker for tumor with muscle cell origin like leiomyosarcoma (smooth muscle origin) and
rhabdomyosarcoma (striated muscle origin).
• The most common sarcoma in infancy or childhood is embryonal rhabdomyosarcoma. They are seen in
different parts of the body most commonly being the genitourinary tract.
• Orbit is also a common site for involvement. Rhabdomyosarcoma is the most common primary malignant
orbital tumor in children. Its embryonal subtype is the commonest. Immunohistochemistry is positive for
desmin, vimentin, myoglobin and muscle specific actin.

7. RET proto-oncogene is associated with:


(AI 2018 Pattern)
(a) Medullary thyroid cancer
(b) Paraganglioma
(c) Papillary thyroid cancer
(d) CNS tumors
Ans. (a) Medullary thyroid cancer (Ref: Robbins 9/e p1099)
8. Which of the following mutation is seen in Cowden syndrome?
(AI 2018 Pattern)
(a) STK11 mutation
(b) SMAD4 mutation
(c) PTEN mutation
(d) PTCH mutation
Ans. (c) PTEN mutation (Ref: Robbins 9/e p291)
• PTEN (phosphatase and tensin homologue) is a membrane-associated phosphatase
coded by a gene on chromosome 10q23.
• It is mutated in Cowden syndrome, an autosomal dominant disorder marked by
frequent benign growths, such as skin appendage tumors, and an increased
incidence of epithelial cancers, particularly of the breast, endometrium, and thyroid.
9. Absence of differentiation is known as:
(AI 2018 Pattern)
(a) Metaplasia
(b) Anaplasia
(c) Dysplasia
(d) Any of the above
Ans. (b) Anaplasia (Ref: Robbins 9/e p270)
10. Neuroendocrine tumor is having which of the following as an
immunohistochemical marker?
(AI 2018 Pattern)
(a) Cytokeratin
(b) Calretin
(c) Synaptophysin
(d) Carcinoembryonic antigen
Ans. (c) Synaptophysin (Ref: Robbins 9/e p717)
Neuroendocrine markers include chromogranin, synaptophysin and neuron specific enolase.
11. Inhibition of phosphorylation of the Rb gene will have which of the following effect
on cell cycle?
(AIIMS May 2017 Pattern)
(a) Inhibition of cell cycle at G1 phase
(b) Inhibition of cell cycle at G2 phase
(c) The cell cycle will progress as it is and the cell will divide
(d) There will be no effect on cell cycle as for Rb gene phosphorylation is not needed
Ans. (a) Inhibition of cell cycle at G1 phase
(Ref: Robbins 9/e p292)
RB is a key negative regulator of the G1 /S cell cycle transition. For details, see the diagram in
the theory of the chapter on Neoplasia.
Disclaimer
Any resemblance to an actual question is purely coincidental.
• Cell mediated immunity is mediated by T-cells whereas humoral immunity is due to
B cells.
• Helper T cells are positive for CD4 whereas cytotoxic T-cells are positive for: CD8.
• T cells undergo both positive and negative selection whereas B cells undergo only
negative selection.
• Antibodies production is a function of B cells and not CD4 Helper T cells.
• TH-1 helper t cells produce: IL-2, IFN-gamma, IL-12 whereas TH-2 helper T cells
produce: IL-4, IL-5, IL-6, IL-13.
• Production of specific antibodies against a particular antigen is due to the Clonal
selection.
• NK cells are Not MHC restricted, not require antibodies.
• Markers of NK cells: CD56 and CD16.
• Primary function of Toll-like receptors: Activation of innate immune system.
• Toll-like receptors activate immune system by: Activation of transcription factors
(NF-kb and AP-1).
• Toll like receptors recognise bacterial endotoxin of all gram negative bacteria
except leptospira.
• Antigen presenting cells (APC) are professional APC (Macrophages, B-cells,
Dendritic cells, Langerhans cells) and non professional APC (fibroblasts, thymic
epithelial cells, endothelial cells.
• Most potent stimulator of Naive T-cells is Langerhans dendritic cell.
• Superantigens bind to: Directly to lateral portion of T-cell receptor (TCR) b-chain
and MHC-II b-chain.
• Major function of MHC (HLA): Present antigen to T-cell for recognition by T-cell
receptors.
• MHC-I is present on all nucleated cells and platelets (but not present on RBCs)
whereas the HLA-II is present on the antigen presenting cells. Medullary
macrophages do not express MHC-II.
• Mixed lymphocyte culture (mixed leukocyte reaction) is used to identify: HLA-II
(MHC-II).
• Markers of B-cells (CD-10, CD-19, CD-20, CD-21, CD-23, CD-79a), memory T-
cells (CD-45 RO), hematopoietic stem cell (CD-34).
• Epitheloid granuloma is caused by: CD-4 Helper T-cells.
• Allograft is: Graft from genetically unrelated member of same species.
• Transplant rejection involves: Both cellular (cell mediated) and humoral (antibody
mediated) rejections. C4d deposition in the glomeruli is an indicator of antibody
mediated rejection.
• Hyperacute rejection is due to: Performed antibodies.
• Mechanism of corneal endothelial graft rejection: Cell mediated (Type-IV)
reaction.
• Most important target in graft rejection: Blood vessels (endothelitis, necrotizing
vasculitis, fibrinoid necrosis).
• Graft versus host disease (GVHD) occurs when immune-competent donor cells
(like bone marrow) is transplanted into immune-compromised host.
• Most commonly affected tissues in GVHD: Skin (most common) >> liver>> gut.
• Not a feature of scleroderma: Calcification of long bones.
• Anticentromere antibody is seen with localized scleroderma/ CREST syndrome
whereas Anti-DNA topoisomerase type-I (anti-Scl 70) is seen with diffuse
scleroderma.
• Anti-U1RNP antibodies are seen in: Mixed connective tissue disease (MCTD).
• Antibodies in Sjogren syndrome: Anti-Ro (SSA), Anti-La (SSB).
• Biopsy of minor salivary gland in Sjogren’s syndrome shows: Lymphocytic
infiltration.
• A common primary immunodeficiency is Selective IgA deficiency (patients
commonly present with recurrent sinopulmonary infections and diarrhea. In
addition, they have an increased risk of autoimmune diseases, particularly SLE
and rheumatoid arthritis). They develop anaphylactic reactions on blood
transfusion with normal IgA containing blood.
• Adenosine deaminase deficiency is associated with: Severe combined
immunodeficiency (SCID).
• Features of common variable immunodeficiency: Hypogammaglobulinemia,
normal number of B-cells, inabilitiy of B-cells to become plasma cells.

• Antibodies in Wiskott-Aldrich syndrome:–↓↓ IgM, ↑IgE but normal IgA and IgG.
• Raji cell assay is used to quantify the circulating immune complexes.
• Features of amyloid: Non-branching, fibrillary congophilic protein with a beta-pleated
sheet conformation and is PAS (+)ve. On electron microscopy, it shows Non-branching
fibrils with diameter of 7.5–10 nm and indefinite length.
• Most commonly affected organ in amyloidosis: Kidney.
• Most common cause of death in amyloidosis: Cardiac failure.
• Characteristic staining feature of amyloidosis: Apple green birefringence under
polarized light.
Immunity is the defensive power of the body (protecting the body from
various infections). It can be of two types: innate immunity and adaptive
immunity. Innate immunity (also known as natural or native immunity) refers
to defense mechanisms that are present since birth and have evolved to
recognize microbes. It is the first line of defense. It is non-specific and has
no memory. Adaptive immunity (also called acquired or specific immunity)
consists of mechanisms that are stimulated by microbes and are capable of
recognizing non-microbial substances also. Adaptive immunity develops later
(after exposure to antigens). It is more specific as well as powerful as well
as has memory.
The major components of innate immunity are
1. Epithelial barriers like intact skin that blocks entry of environmental
microbes
2. Cells like phagocytic cells (mainly neutrophils and macrophages), Natural
killer (NK) cells, Dendritic cells
3. Plasma proteins (proteins of the complement system, mannose binding
lectin and C-reactive protein)
The innate immunity is due to presence of pattern recognition
receptors (PRR). These are peptide molecules on the leukocytes which
recognize particular structural pattern on a micro-organism called pathogen
associated molecular patterns (PAMPs). A similar group of molecules
released by injured cells is called danger associated molecular patterns
(DAMP, uric acid is an example). The PRR can be of the following two types:
Soluble pattern recognition receptors Surface pattern recognition receptors
• Mannose receptors (for mannose • Scavenger receptors (on macrophages)
binding lectin) • Toll like receptors
• C-reactive protein • NOD-like receptors
• RIG-like receptors

Some important Toll like receptors (TLR) and the molecules they recognize
are:

• TLR-2: peptidoglycan of gram + bacteria; lipopolysaccharide of leptospiraQ


• TLR-3: dsRNA viruses
• TLR-4Q: Chlamydia and lipopolysaccharide of Gram (–) bacteriaQ except
leptospira

Signaling by Toll-like receptors causes the activation of nuclear


transcription factors (NF-κb and AP-1). This result in recruitment of
inflammatory cytokines, endothelial adhesion molecules (E-selectin) and
proteins involved in microbial killing mechanisms (inducible nitric oxide
synthase).

The adaptive immune system consists of lymphocytes and their


products like antibodies. It has two components: cellular (or cell mediated)
and humoral immunity. The former is protective against intracellular microbes
whereas the latter is effective against extracellular microbes.

IMMUNE CELLS

Apart from the leucocytes, our focus here would be to discuss the other
important immune cells (lymphocytes and antigen presenting cells) in detail.

1. T-lymphocytes (Thymus Derived)

They constitute 60-70% of peripheral blood lymphocytes and are located in


the paracortical areas of lymph node and the periarteriolar sheaths of spleen.
These cells have an antigen specific T cell receptor (TCR) [composed of a
and b polypeptide chains in 95% cases] to bind with the antigen. The ab T
cells are present in blood and tissues. The other 5% cells have TCR
composed of g/d chains and are present mostly at the epithelial/mucosal
surface. A large number of TCRs can be generated because of
rearrangement of genes coding for α and β polypeptide chains. When an
antigenic peptide comes in contact with TCR, it activates a particular T cell
only and not all the cells. This is called as clonal selection.

Demonstration of TCR gene rearrangement by southern blot is a molecular marker of T cell


lineage.

The cells have on their surface cluster differentiating (CD) molecules by


which they can be readily identified. The CD molecules present on T cells
are CD1, CD2, CD3, CD4, CD5, CD7, CD8 and CD28. The T cell having
CD4 molecule is called CD4+ T cell or the Helper T cell and that having
CD8 molecule is called as CD8+ T cell or Cytotoxic/Killer T cell. CD4+ T
cells secrete cytokines and help macrophages and B cells to fight infections
whereas CD8+ T cells destroy host cells having microbes like viruses and
tumor cells.
CD3 is involved in signal transduction and is also called as a Pan T
cell marker. The T cells also have presence of CD40 ligand on their surface
which is required for B cell activation and induction of immunoglobulin
‘isotype switching’. (Described later).

The activation of T cells requires two signals... see Figure 1

Signal 1: Comes from binding of the TCR to MHC bound antigen. The CD4 or CD8
act as co-receptors and enhance this signal.
Signal 2: Comes from the interaction of CD28 with co-stimulatory molecules B7-1 and
B7-2 present on the antigen presenting cells.

The activated T cells gives rise to two groups of cells: effector T cells
which manage the antigen at that time only and some differentiate into long
lived memory cells (for future exposure to the same antigen).
Location of cell Molecular marker
All leucocytes CD45 (Leukocyte common antigen; LCA) and CD45RB
Medullary thymocytes (‘Naive’ T- CD45 RA and CD45RC
cells)
Cortical thymocytes (Memory T- CD45RO
cells)

Fig. 1: Interaction between APC and T cell.


Importance of signal 2 in immunity
• Signal 2 ensures that the activation of the T cells is not taking place by chance. It is
due to a particular antigen only. So, it acts like a safety signal.
• Secondly, if by any mistake, the APC present a self antigen (normal body tissue),
signal 2 is not generated. In the absence of signal 2, T cell undergoes anergy. This is
an important mechanism of peripheral immunological tolerance.

Concept of Superantigen

Fig. 2: Superantigen.
Types of CD4+ Helper Cells

• Recently a new group of T cells have been discovered called as NK-T


cells. These T cells express markers normally present on Natural Killer
cells and recognize glycolipid antigens displayed by MHC-like molecule
CD1. These are important defense mechanisms against microorganisms
like Listeria monocytogenes and M tuberculosis.
Naive T cell (TH0) can get differentiated into either TH1cell or TH2 cell.
The differentiation towards TH1 cell is driven by strong innate response and
intracellular organisms (TB, Listeria). TH1 cells release IL-2 and IFN-g which
causes cytotoxic T cell activation and granuloma formation. Constitutively
which means most of the times physiologically, TH0 cell differentiates into
TH2 cell. TH2 cells release IL-4 and IL-5 which cause B cell activation and
multiplication leading to antibody formation.

Diagram of different T helper cells

2. Antigen Presenting Cells

When an antigen enters inside the body, it is phagocytosed by the neutrophils


following which the antigenic peptides are released in the circulation.
However, if antigen presenting cells phagocytose the antigen, they process it
inside themselves and present on their surface in association with MHC
molecule. This processed antigen is now presented to the T cells.
The part of the antigen associated with MHC molecule on the APC is
called as aggretope whereas the antigenic part in contact with TCR is called
epitope. MHC molecules can be of two types: MHC I and MHC II (described
later).

Details about APCs

MACROPHAGES
These cells have a role in induction (in cellular immunity) and the effector (in
humoral immunity) phase of immune response. They process and present
antigen to T cells for induction of cell mediated immunity (CMI). They get
activated by the presence of IFN-γ and are the effector cells in humoral
immunity as they phagocytose opsonised microbes.

DENDRITIC CELLS
These are important antigen presenting cells in the body and can be of the
following types:
a. Interdigitating dendritic cells (dendritic cellsQ) are the most important
antigen-presenting cells for initiating primary immune responses
against protein antigens.
b. Follicular dendritic cells are present in the germinal centers of
lymphoid follicles in the spleen and lymph nodes.
– These cells bear Fc receptors for IgG and receptors for C3b and can
trap antigen bound to antibodies or complement proteins. Such cells
are required for the process of ‘Affinity Maturation’ (production of
antibodies having high affinity for antigens).

3. B-lymphocytes (Bone Marrow Derived)

They constitute 10-20% of peripheral blood lymphocytes and are located in


the cortical areas of lymph node, white pulp of spleen and mucosa
associated lymphoid tissue of pharyngeal tonsils and Peyer’s patches of GIT.
These cells have a B cell receptor (BCR) composed of IgM and IgD on
their surface to bind with the antigen. BCR has unique antigen specificity. The
rearrangement of immunoglobulin gene can give rise to different types of
BCRs. The antigen however binds to the complementary BCR only (clonal
selection).

The presence of rearranged immunoglobulin genes in a lymphoid cell is used as a


molecular marker of B-lineage cells.

B cell associated markers are CD10 (CALLA), CD19, CD20, CD21


(EBV receptor), CD22, CD23.
B cells have Iga and Igb on their cell membrane which are required for
signal transduction (similar to CD3 of the T cells). They also have CD 40
molecule on its surface.

Detail of antigenic interaction with B cells and T cells


The activation of T cells requires two signals

Signal 1: Comes from binding of the TCR to MHC bound antigen.


Signal 2: Comes from the interaction of CD28 with co-stimulatory molecules
B7-1 and B7-2 present on the antigen presenting cells.

In addition to these two, there is a Competence signal is due to


interaction between CD40 molecules on B cells with CD 40 ligand on T
helper cells. It results in the release of cytokines like IL-4 and IL-5. These
cytokines cause B cell proliferation resulting in formation of plasma cells and
memory cells.

The plasma cells secrete immunoglobulins. These can of different


classes or isotypes like IgG, IgM, IgD, IgA and IgE. Initially, the first antibody
produced by the plasma cell is IgM and later, other antibodies like IgG, IgA
etc. are produced due to change in the nature of heavy chains. Isotype
Switching is due to IFN-γ and IL-4. Polysaccharide and lipid antigens
produce mainly IgM whereas protein antigens induce production of different
isotypes of antibodies (IgG, IgA, IgE etc.).
Isotype/Class Switching
IgM IgA, IgE, IgG
(Due to change in heavy chain)

IMPORTANT POINTS ABOUT ANTIBODIES


IgG • Present in maximum concentrationQ the human body
• Important for secondary immune responseQ
• Can cross the placentaQ
IgA • Resent in physiological secretionsQ of the body
• Present in monomer form in serum and as dimerQ form in glandular
secretions
• Responsible for activation alternate pathwayQ
IgM • Important for primary immune responseQ
• Having maximum molecular weightQ
• Having maximum sizeQ
• Present as a pentamerQ
• Also known as ‘Millionaire’s antibody’Q
• Functions as B cell receptorQ
• IgMQ and IgG (IgMQ > IgG) are responsible for activation of classical
pathwayQ
IgD • Functions as B cell receptorQ
IgE • Increased in allergic conditionsQ
• Also known as ‘homocytotropic antibodyQ’
• Also called ‘reaginic antibody’Q
The antibodies produced remove the antigen by different mechanisms
like complement activation (by membrane attack complex formation),
opsonisation (for preferential killing) and antibody dependent cytotoxicity.

Cell Mediated Immunity

It is for more important for intracellular pathogens, virus infected/malignant


cells and endogenous antigens. It is mediated by CD8 T cells, macrophages
and natural killer cells. Endogenous antigen is expressed with MHC I
molecule by the nucleated cells. These cells are also destroyed in the
process, so, the preferred name for them is Target cells and not APCs.

Antigen activated CD8T cells undergo proliferation. They release


perforin-granzyme molecules and express Fas Ligand both of which initiate
apoptosis of Target cells.

NATURAL KILLER CELLS (NK CELLS) OR NULL CELLS OR NON-T,


NON-B LYMPHOCYTES
NK cells are also called ‘Large granular lymphocytes’ as they are
morphologically larger than both T and B lymphocytes and contain
azurophilic granules (which are absent in both T and B lymphocytes). They
constitute 5-10% of peripheral blood lymphocytes. They arise in both bone
marrow and thymic microenvironments. NK cells are activated in presence of
IL-2 to Lymphocyte activated killer (LAK) cells. These cells express the
following molecules:
Contd...
Contd...

They are first line defense against cancer and virus infected cells. So, functionally NK cells
share features of both monocyte-macrophages and neutrophils. The hyporesponsiveness
of NK cells is seen in patients of Chediak-Higashi syndrome.

The NK cells express activating and inhibitory receptors. The functional


activity of the NK cells is regulated by a balance between signals from these
receptors. Normal cells are not killed because inhibitory signals from normal
MHC class I molecules override activating signals. The ability of NK cells to
kill target cells is inversely related to target cell expression of MHC
class I molecules.

Major Histocompatibility Complex (MHC) or Human Leucocyte Antigen


(HLA) Complex

MHC is a cluster of genes located on short arm of chromosome 6 (6pQ)


whose main physiologic function is to bind peptide fragments of foreign
proteins for presentation to antigen-specific T cells.
It is classified into three classes namely class I, II and III genes. The
class I genes includes HLA-A, -B, -C, - E, -F and -G. HLA-A,-B and -C gene
codes for MHC I molecule and the HLA-E molecule is the major self-
recognition target for the natural killer (NK) cell inhibitory receptors. HLA-G
is expressed selectively in extravillous trophoblasts, the fetal cell population
directly in contact with maternal tissues. It provides inhibitory signals to both
NK cells and T cells and maintains maternofetal tolerance and the function
of HLA-F remains largely unknown. The class II genes include HLA-D and
code for MHC II molecule whereas the class III gene codes for the
complement and other proteins.
MHC-I MOLECULE

It consists of a chain (heavy chain) linked to b2 microglobulin (light chain; not


encoded within MHC) and binds to peptides that are derived from proteins
synthesized within the cell like the viral antigens. The antigen binding cleft is
formed by a1and a2 chain of the MHC molecule (distal a domains of
MHCQ). The antigens binding with MHC I molecule are presented to CD8+ T
cells. As discussed earlier, cytotoxic T cells/CD8+ T cells are MHC-I
restricted.
Structure of antigen binding cleft of MHC molecule on the surface
of APC
The antigen binding
cleft is made up α1
and α2Q chains of MHC
I molecule which is
structurally the
distal domain of
these chains.

The antigen binding


cleft is made up distal
α and βchainsQ (α1
and β1chains) of MHC
II molecule.

MHC-II molecule

It consists of a chain linked to b-chain. The antigen binding cleft is formed by


a1 and b1 chain of the MHC molecule (distal a and b domains of MHCQ).
The antigens binding with MHC II molecule are presented to CD4+ T cells.
As discussed earlier, helper T cells/CD4+ T cells are MHC-II restricted.
HLA and Disease Association

HLA- class II
DR-2 - Japanese SLE
- Multiple sclerosis
- Narcolepsy
- Goodpasture’s syndrome
DR-3 • Myasthenia gravis
• Graves’ disease
• Type I DM
• Dermatitis herpetiformis
• Chronic active hepatitis
• Caucasian SLE
• Sjogren’s syndrome
DR-4 Type 1 DM
Pemphigus vulgaris
Rheumatoid arthritis
DR-5 Juvenile (pauciarticular) arthritis
DR-8 Type I DM
DQ-1 Pemphigus vulgaris
DQ - Gluten sensitive enteropathy [celiac sprue]
DQ-7 Bullous pemphigoid
DQ-8 Type 1 DM

DISORDERS OF THE IMMUNE SYSTEM

Hypersensitivity Reactions

These are caused by the activity of the immune system detrimental to the
host in response to exposure of the antigens.

TYPE I HYPERSENSITIVITY REACTION/ANAPHYLACTIC


TYPE/IMMEDIATE TYPE OF HYPERSENSITIVITY REACTION
It is defined as a rapidly developing immunologic reaction occurring within
minutes after the combination of an antigen with antibody bound to mast cells
in individuals previously sensitized to the antigen. It is commonly referred to
as allergy.

PATHOGENESIS
The first step is the stage of sensitization or priming in which there is entry
of the antigen inside the body for the first time where it is captured by the
antigen presenting cells and presented to the T cell which then differentiates
into TH2 cell. The TH2 cell releases mediators like IL-3, IL-4 and IL-5. IL-4
causes activation of B cell leading to the release of IgE from them whereas
IL-5 is responsible for activating the eosinophils. The secreted IgE then binds
to mast cells in the circulation because of presence of Fc receptors on the
mast cells. So, in the initial exposure or sensitization, there is presence of
mast cells in the circulation having the presence of IgE on their surface.
Contd...
Contd...

The subsequent exposure to the same antigen causes the features in


two phases. In the initial phase (within minutes of antigen exposure), there is
release of preformed mediators of the mast cell due to their degranulation
causing the release of histamine, proteases and chemotactic factors.
Histamine causes vasodilation, bronchoconstriction and increased
permeability. Late phase (2-24 hours after antigen exposure) is marked by
the release of secondary mediators from the mast cells that include
prostaglandins, leukotrienes, cytokines and platelet activating factor (PAF).
PGD2 is abundant in lung mast cells and causes bronchoconstriction as well
as increased mucus production. The secondary mediators are responsible for
the effects like bronchospasm, increased mucus production and recruitment
of the inflammatory cells at the site of inflammation. PAF causes
bronchospasm, increased permeability and release of histamine and is
considered to be important in the initiation of the late-phase response. The
release of various mediators is responsible for the clinical features seen in
type I hypersensitivity reaction.
Examples of type I hypersensitivity includes:
Localized hypersensitivity Systemic hypersensitivity
• Bronchial asthma Anaphylaxis due to:
• Hay fever/allergic rhinitis • Antibiotics: Most commonly penicillin (therefore,
• Food allergies a test dose should always be given before
• Atopic dermatitis administration of penicillin to any patient)
• Urticaria • Bee stings
• Angioedema • Insect bites

TYPE II HYPERSENSITIVITY REACTION/ANTIBODY


MEDIATED/CYTOLYTIC HYPERSENSITIVITY REACTION
Type II hypersensitivity is mediated by antibodies directed toward
endogenous or exogenous specific antigens present on cell surfaces or
extracellular matrix. The effector mechanisms for this reaction include:

I. Opsonisation and Complement- and Fc Receptor-Mediated


Phagocytosis
The antibodies are formed against the antigens and these are responsible for
complement system activation resulting in the formation of membrane
attack complex (MAC) leading to destruction of the antigen. The
antibodies may also cause opsonisation (through C3b and C4b) and Fc
receptor mediated phagocytosis.
II. Complement and Fc Receptor-Mediated Inflammation
When antibodies deposit in extracellular tissues, the injury is because of
inflammation and not phagocytosis or lysis of cells. The deposited
antibodies activate complement system leading to recruitment of
neutrophils and monocytes. These cells also bind to the deposited
antibodies via their Fc receptors. The activated leukocytes release
enzymes resulting in tissue damage.
III. Antibody mediated cellular dysfunction
In this mechanism, the antibodies directed against cell-surface receptors
impair or dysregulate function without causing cell injury or inflammation.

Examples of type II hypersensitivity reaction


Opsonization and • Transfusion reactions
Complement- and Fc • Erythroblastosis fetalis
Receptor-Mediated • Autoimmune hemolytic anemia
Phagocytosis • Autoimmune thrombocytopenic purpura

Complement and Fc • Goodpasture syndrome


Receptor-Mediated • Vasculitis due to ANCA
Inflammation • Acute rheumatic fever
• Vascular rejection in organ grafts

Antibody mediated cellular • Myasthenia gravis (against acetylcholine receptor)


dysfunction • Graves’ disease (against TSH receptor)
• Pemphigus vulgaris (against epidermal cadherin)
• Pernicious anemia (against intrinsic factor)
• Insulin resistant diabetes (against insulin receptor)

TYPE III HYPERSENSITIVITY REACTION OR IMMUNE COMPLEX


DISEASE
Antigen-antibody complexes produce tissue damage mainly by eliciting
inflammation at the sites of deposition. The antigen can be either
endogenous or exogenous. The immune complexes once formed may be
present in the circulation (circulating immune complexes) or may get
deposited inside the vessels or extravascular sites (in situ immune complex).
They may either be generalized or localized. Systemic or generalized
immune complex disease has the following phases:
Phase I or Immune Complex Formation

It is characterized by the formation of the antibody about 5 days after


introduction of the antigen. The small or intermediate immune complexes are
most pathogenic. The large complexes are rapidly removed by the
macrophages.

Phase II or Immune Complex Deposition

In this phase, the immune complexes get deposited in the glomeruli, joints,
skin, heart, serosal surfaces and the blood vessels.

Phase III

Immune complex mediated inflammation is seen 10 days after antigen


administration and results in the development of vasculitis,
glomerulonephritis and arthritis. The immune complexes cause inflammation
by activation of the complement system resulting in the neutrophilic
infiltration, vasodilation and edema. They also cause activation of the intrinsic
pathway of coagulation system and microthrombi formation contributing to
tissue ischemia and necrosis.
The blood vessels show intense neutrophilic infiltration and necrotizing
vasculitis having the presence of fibrinoid necrosis.
Examples of type III hypersensitivity include:
Localized hypersensitivity Systemic hypersensitivity
*Arthus reaction *SLE
*Farmer’s lung *Reactive arthritis
*Polyarteritis nodosa *Henoch-Sch nlein purpura
*Post streptococcal glomerulonephritis
*Serum sickness
*Type II lepra reaction
TYPE IV OR CELL MEDIATED HYPERSENSITIVITY REACTION
The cell-mediated type of hypersensitivity is initiated by antigen-activated
(sensitized) T lymphocytes. It includes the delayed type hypersensitivity
reactions mediated by CD4+ T cells, and direct cell cytotoxicity mediated by
CD8+ T cells.

1. Pathogenesis of delayed type hypersensitivity reactions (mediated


by CD4+ T cells)
The first step is the entry of the antigen inside the body where it is captured
by the APCs and presented to the T cell which then differentiates into
TH1 cell (remember that in type I hypersensitivity, the naïve T-cells
differentiate into TH2 cells). The sensitized TH1 cells enter the circulation
and remain in the memory pool of the body. When there is re-exposure of
the same individual to the antigen for the subsequent time, there is
release of cytokines like TNF-a, lymphotoxin, IFN-γ, IL-2 and IL-12.
• TNF-a and lymphotoxins have effects on endothelial cells leading to
extravasation of lymphocytes and monocytes.
• IL-2 causes proliferation of antigen specific T-cells.
The collective release of these mediators recruits a lot of inflammatory cells
at the site of inflammation. The activated macrophages give rise to
epithelioid cells and these cells surrounded by a collar of lymphocytes all
around lead to formation of a granuloma. This granuloma formation is
seen with tuberculin test, and other intracellular pathogens like
mycobacterium, fungi and some parasites. Delayed type hypersensitivity
reaction is also important in transplant rejection.

2. Pathogenesis of T cell mediated cytotoxicity (mediated by CD8+ T


cells)
Cytotoxic T lymphocytes (CTL) cause destruction of antigen bearing target
cells particularly the tumor cells, the virus infected cells and allogeneic
tissue during graft rejection. There are two mechanisms involved in this:
– Perforin granzyme dependent killing: The mediators present in the
lysosomal granules of the CTLs like perforin cause pore formation
and the granzyme activates apoptosis on entering the cells via these
pores.
– Fas-Fas ligand dependent killing: Activated CTL express Fas ligand
which can bind to Fas expressed on the target cells leading to
apoptosis.
Examples of Type IV Hypersensitivity Include:
*Tuberculin reaction *Lepromin test *Multiple sclerosis
*Chronic graft rejection *Contact dermatitis *Sarcoidosis
*Temporal arteritis *Primary biliary cirrhosis *Type I lepra reaction
*Tumor immunity *Resistance to viral infections *Crohn’s disease
*Rheumatoid arthritis

TRANSPLANT REJECTION

It is a complex process in which both cellular and humoral immunity plays a


role.

T cell Mediated Rejection


The T cell mediated rejection is also called as cellular rejection and it has two
pathways:
• Direct pathway in which the interstitial dendritic cells of the donor present
the antigen to the CD4 and CD8 T cells of the host. The host CD4 Tcells
differentiate into TH1 cells and similar to delayed hypersensitivity cause
graft injury. CD8 T cells differentiate into cytotoxic T lymphocytes and
cause graft tissue damage by perforin-granzyme and Fas–Fas ligand
pathways.
• Indirect pathway in which the dendritic cells of the recipient present the
antigen to CD4 Tcells. There is no involvement of the CD8 T cells.

Antibody Mediated Rejection or Humoral Rejection


It is also known as rejection vasculitis and takes place by two mechanisms:

• Hyperacute rejection takes place when there is preformed anti-donor


antibodies present in the circulation of the recipient. It takes place within
minutes to hours and is associated with previous blood transfusions,
multiparous lady or already rejected transplant. It is an example of type II
hypersensitivity reaction.
• Acute rejection is seen within days to months after transplantation. The
mechanisms involved include inflammation, complement dependent
cytotoxicity and ADCC.

MORPHOLOGY OF TRANSPLANT REJECTION


Hyperacute rejection
It takes place in individuals with preformed antibodies usually within minutes to hours of
transplantation. The preformed antibodies result in immune complex disease with the
presence of neutrophils within arterioles, glomeruli and peritubular capillaries and fibrin-
platelet thrombi in vessel wall. There is presence of necrosis of the renal cortex.
Acute rejection
It is seen days to months after transplantation. It can be acute humoral rejection or acute
cellular rejection.
The acute cellular rejection is seen within few months after transplantation. There is
presence of endothelitis with the presence of CD4 and CD8 T cells in the interstitium along
with the mononuclear cells in the glomerular and peritubular capillaries.
The acute humoral rejection or rejection vasculitis is mediated primarily by anti-donor
antibodies and it manifests mainly as damage to the blood vessels in the form of
necrotizing vasculitis with endothelial cell necrosis, neutrophilic infiltration, deposition of
immunoglobulins, complement, and fibrin, and thrombosis. There is associated necrosis of
the renal parenchyma.
Chronic rejection
It occurs months to years after transplantation. In this, the vascular changes consist of
dense, obliterative intimal fibrosis in the cortical arteries resulting in glomerular loss,
interstitial fibrosis and tubular atrophy, duplication of basement membranes of the glomeruli
(also called as chronic transplant glomerulopathy). The renal interstitium also has
mononuclear cell infiltrates containing large numbers of plasma cells and eosinophils.

Fig. 3: Acute humoral (antibody mediated) rejection: C4d deposition in peritubular capillaries
and a glomerulus.

GRAFT VERSUS HOST DISEASE (GVHD)/RUNT DISEASE (in animals)

Graft versus host disease occurs in any situation in which immunologically


competent cells or their precursors are transplanted into immunologically
crippled patients and the transferred cells recognize alloantigens in the
host. It occurs most commonly in the setting of allogenic bone marrow
transplantation. The recipients of bone marrow transplants are
immunodeficient because of either their primary disease or prior treatment
of the disease with drugs or irradiation. When such recipients receive normal
bone marrow cells from allogenic donors, the immunocompetent T cells
present in the donor marrow recognize the recipient’s HLA antigen as
foreign antigen and reacts against them. Both CD4+ and CD8+T cells
recognize and attack host tissues.

ACUTE GVHD
• It is characterized by an erythematous maculopapular rash; persistent
anorexia or diarrhea, or both; and by liver disease with increased serum
levels of bilirubin, alanine and aspartate aminotransferase, and alkaline
phosphatase.
• Diagnosis usually requires skin, liver, or endoscopic intestinal biopsy for
confirmation. In all these organs, endothelial damage and lymphocytic
infiltrates are seen.
*Grade I acute GVHD is of little clinical significance, does not affect the
likelihood of survival, and does not require treatment. In contrast, grades II to
IV GVHD are associated with significant symptoms and a poorer probability
of survival, and they require aggressive therapy.

CHRONIC GVHD

Chronic GVHD resembles an autoimmune disorder with malar rash, sicca


syndrome, arthritis, obliterative bronchiolitis, and bile duct degeneration and
cholestasis.
• Because patients with chronic GVHD are susceptible to significant
infections, they should receive prophylactic trimethoprim-
sulfamethoxazole.
• Infection with cytomegalovirus is particularly important.
SYSTEMIC LUPUS ERYTHEMATOSUS (SLE)

SLE is an autoimmune multi-system disorder of unknown etiology


characterised by loss of self tolerance and production of auto-antibodies. It is
more commonly seen in the females affecting them around the age of 20-
30’s. The deficiency of early complement proteins (C1, C2 and C4) has been
postulated to be associated with increased incidence of SLE. The auto-
antibodies in this condition are formed against DNA, histones, non histone
proteins bound to RNA and nucleolar antigens. These are collectively called
as antinuclear antibodies (ANA).
The clinical criteria for the diagnosis of SLE include any 4 of the
following mentioned 11 criteria:
• Malar rash • Serositis-Pleuritis or pericarditis
• Discoid rash • Renal disorder
• Photosensitivity • Hematological disorder
• Oral ulcer • Immunological disorder
• Arthritis • Antinuclear antibody
• Neurological disorder-Seizure or psychosis in the absence of known drug/metabolic
abnormality.
ORGAN INVOLVEMENT
1. Kidney: WHO classification of renal involvement or ‘lupus nephritis’ is
as follows:
• Class I - Minimal or no change
• Class II - Mesangial lupus glomerulonephritis
• Class III - Focal proliferative glomerulonephritis
• Class IV - Diffuse proliferative glomerulonephritis.
• Class V - Membranous glomerulonephritis

2. Heart: There is development of pericarditis (more commonly) as well


as Libman-Sacks endocarditisQ (less commonly) having vegetations on both
the sides of the valvular surface. There is also presence of.
3. Mouth: Oral ulcers are usually painless.
4. Joints: Non-erosive arthritis involving 2 or more peripheral joints with
tenderness and effusion.
5. Skin: Erythematous rash present over malar region is also called
‘butterfly rash’. Exposure to sunlight accentuates the erythema.
6. Lung: There is presence of pleuritis (more commonly) as well as
diaphragmatic weakness (shrinking lung syndrome).
7. Blood: Presence of autoimmune cytopenia (anemia, neutropenia or
thrombocytopenia). The presence of LE cell or hematoxylin body is also
seen.
SJOGREN SYNDROME
It is an autoimmune disorder characterised by the destruction of lacrimal and
salivary glands resulting in the inability to produce tears and saliva. It is more
commonly seen in females. It can be primary when it is called sicca
syndrome and it may also be secondary to other autoimmune disorders;
rheumatoid arthritis being most commonly associated disorder.

There is presence of anti-ribonucleoprotein antibodies like SS-A (Ro)


and SS-B (La). The presence of former is associated with early disease
onset, longer disease duration, and extraglandular manifestations, such as
cutaneous vasculitis and nephritis.
Clinical features include dry mouth (xerostomia) and dry eyes
(keratoconjunctivitis sicca), the latter due to lymphocytic infiltration and
destruction of the lacrimal gland. Mickulicz syndrome include lacrimal and
salivary gland enlargement of whatever cause. Patients with Sjogren
syndrome have an increased risk of developing lymphoid malignancies.
MIXED CONNECTIVE TISSUE DISEASE (MCTD)
It is a disease seen in a group of patients who are identified clinically by the
coexistence of features suggestive of SLE, polymyositis, rheumatoid arthritis,
and systemic sclerosis. These patients have high titers of antibodies to RNP
particle-containing U1 RNP. The factors lending distinctiveness to mixed
connective tissue disease include the reduced incidence of renal involvement
and a good response to corticosteroids.

SCLERODERMA
• It is an autoimmune disorder characterised by fibroblast stimulation and collagen
deposition in the skin and internal rgans. The skin is most commonly affected, but
the gastrointestinal tract, kidneys, heart, muscles, and lungs also are frequently
involved.
• It is more commonly seen in the females and is due to release of growth factors
acting on the fibroblasts like fibroblast growth factor (FGF), platelet derived growth
factor (PDGF) and cytokines like IL-1.

The disease has two categories:


• Diffuse scleroderma is characterized by presence of anti-DNA
topoisomeraseQ antibodies (Scl-70). There is widespread skin
involvement at onset, with rapid progression and early visceral
involvement. The symptoms include dysphagia, malabsorption,
arrhythmia (due to cardiac fibrosis), exertional dyspnea and renal
insufficiency.

• Limited scleroderma is characterized by the presence of anti-


centromereQ antibodies. The skin involvement is often confined to
fingers, forearms, and face. Since the visceral involvement occurs late;
so, the clinical course is relatively benign. Some patients develop CREST
syndromeQ.

Recent exam topic: Antinuclear antibody staining patterns


Pattern Antigen Image
Homogeneous Chromatin,
or diffuse histones
nuclear
staining

Rim or Double stranded


peripheral DNA
staining
Speckled Antibody against
pattern extractable (Non-
(MC and least DNA) nuclear
specific antigens like
pattern) ribonucleoprotein,
Sm antigen, SS-A
and SS-B
reactive antigens

Nucleolar RNA
pattern (Bright
(seen in fluorescence is
systemic seen within the
sclerosis) nucleoli)

Centromeric Centromeres
pattern
(seen in
CREST
syndrome)

INFLAMMATORY MYOPATHIES
The inflammatory myopathies represent the largest group of acquired and
potentially treatable causes of skeletal muscle weakness. They are classified
into three major groups: polymyositis (PM), dermatomyositis (DM), and
inclusion body myositis (IBM).

Fig. 4: Polymyositis: lymphocytes surround individualmuscle cells.

Criteria for Diagnosis of Inflammatory Myopathies

Inclusion body
Criterion Polymyositis Dermatomyositis myositis
Myopathic muscle Yes Yes Yes; slow onset,
weakness early
involvement of
distal muscles,
frequent falls
Electromyographic Myopathic Myopathic Myopathic with
findings mixed potentials
Muscle enzymes Elevated (up to Elevated (up to 50-fold) Elevated (up to
50-fold) or normal or normal 10-fold) or
normal
Muscle biopsy findings “Primary” Perifascicular, Primary
inflammation with perimysial, or inflammation with
the CD8/MHC-I perivascular infiltrates, CD8/MHC-I
complex and no perifascicular atrophy complex;
vacuoles vacuolated fibers
with amyloid
deposits;
cytochrome
oxygenase–
negative fibers;
signs of chronic
myopathy
Rash or calcinosis Absent Present Absent

Fig. 5: Dermatomyositis: Perivascular lymphocytic infiltrate.

X-LINKED AGAMMAGLOBULINEMIA OF BRUTON

It is an X-linked immunodeficiency disorder characterized by the failure of B-


cell precursors (pro-B cells and pre-B cells) to mature into B cells due to
mutation of B-cell tyrosine kinase (Btk). Btk is required for the maturation of
pre-B cell to mature B cell. The disease is seen almost entirely in males. In
most cases, recurrent bacterial infections of the respiratory tract, such as
acute and chronic pharyngitis, sinusitis, otitis media, bronchitis, and
pneumonia, are present. The causative organisms are Haemophilus
influenzae, Streptococcus pneumoniae, or Staphylococcus aureus in most of
the patients. It is associated with an increased risk of other autoimmune
disorders.
WISKOTT-ALDRICH SYNDROME (WAS)

• X-linked recessive diseaseQ.


• Affecting males more commonly.
• Mutations in the WASP gene located on short arm of chromosome X.
• The WASP protein is important in platelets and T cells.
• Cytoskeletal integrity and signal transduction
• Characterized by bruising caused by thrombocytopeniaQ, eczemaQ, recurrent
infectionsQ, bloody diarrhea (due to thrombocytopenia) and increased risk of
autoimmune disorders and malignanciesQ.

Diagnosis is made on the basis of clinical parameters, the blood film and low
immunoglobulin levels. Treatment it is done with bone marrow
transplantation. The alternatives include intravenous immunoglobulin
infusions or splenectomy.
COMMON VARIABLE IMMUNODEFICIENCY
Most patients with common variable immunodeficiency have normal or near-
normal numbers of B cells in the blood and lymphoid tissues which are not
able to differentiate into plasma cells. Patients have intrinsic B-cell defects
(defective cytokine receptor called BAFF which normally promotes B cell
differentiation and survival) as well as abnormalities of T cell-mediated
regulation of B cells. The clinical manifestations include recurrent
sinopulmonary pyogenic infections, recurrent herpesvirus infections and
persistent diarrhea caused by G. lamblia. It affects both sexes equally, and
the onset of symptoms is relatively late (in childhood or adolescence).
These patients have a high frequency of autoimmune diseases like
rheumatoid arthritis and increased risk of lymphoid malignancy (particularly in
women).

DIGEORGE SYNDROME OR VELOCARDIOFACIAL SYNDROME


It is a T-cell deficiency due to deletion of chromosome 22q11.2. The patients
have a loss of T cell-mediated immunity (owing to hypoplasia or lack of the
thymus), tetany (owing to lack of the parathyroids), and congenital defects of
the heart and great vessels. They may also have abnormal facies with
defects in the mouth and ears. The absence of cell-mediated immunity
results in the development of recurrent fungal and viral infections.

ACQUIRED IMMUNODEFICIENCY SYNDROME (AIDS)


This retroviral disease is caused by the human immunodeficiency virus (HIV).
It is characterized by the triad of immunosuppression associated with
opportunistic infections, secondary neoplasms, and neurologic
manifestations. The major routes of HIV infection are:
1. Sexual contact
It is the most common mode of spread of the infection throughout the
world. It is usually through heterosexual contact. The presence of any
other concomitant sexually transmitted disease causing genital
ulcerations increases the risk of transmission of HIV also. Gonorrhea and
Chlamydia also act as cofactors for HIV transmission primarily by
increasing the seminal fluid content of inflammatory cells carrying HIV.
2. Parenteral inoculation
Parenteral transmission of HIV is a broad term which includes transmission
through:

a. Intravenous drug abusers (the largest group): transmission occurs


through shared needles, syringes etc.
b. Patients receiving blood or blood components (like hemophiliacs
receiving factor VIII or IX concentrates)
c. Infected patient to the physician through needle stick injury:
The transmission through the parenteral route can be prevented with
the screening of the blood and taking precautions like the universal
precautions like not recapping the needle after taking blood sample
of a patient and use of disinfectants like hypochlorite for blood
spillages.

3. Passage of the virus from infected mother to newborn (mother to child


transmission or vertical transmission)
The transmission from an infected to the child can take place through:
a. Transplacental spread.
b. Infected birth canal during normal vaginal deliveryQ: it is the MC
route for vertical transmission.
c. Ingestion of breast milk
The transmission through the vertical route can be reduced by the use
of elective caesarean section and the use of antiviral drugs like
nevirapine and zidovudine. (For details, refer to Review of
Pharmacology by the same authors).

ETIOLOGY AND PATHOGENESIS


HIV is a retrovirus belonging to the lentivirus family and is of two types HIV-1
and HIV-2. There are two strains of HIV which are:

a. Macrophage-tropic (R5 virus) strain: it infects both


monocytes/macrophages and T cells.
b. T-cell tropic (X4 virus) strain: it infects only activated T cell lines.

The HIV-1 virion is spherical and contains an electron-dense, cone-


shaped core containing the major capsid protein p24, nucleocapsid protein
p7/p9, the viral RNA, and viral enzymes (protease, reverse transcriptase, and
integrase) surrounded by a lipid envelope derived from the host cell
membrane. p24 is the most readily detected viral antigen and is the target for
the antibodies used to diagnose HIV infection in blood screening. The viral
envelope has two glycoproteins (gp120 and gp41) required for HIV infection
of cells.
The two major targets of HIV infection are the immune system and
the CNS. The profound immunodeficiency is the hallmark of AIDS. The viral
envelope gp120 interacts with CD 4 molecule followed by confirmational
change in gp 41. The virus then fuses with the host cell membrane. The
commonly affected CD 4 cells in the human body include helper T cellsQ
(worst affected), monocyte-macropahges and dendritic cells.
Defective CCR5 receptors lead to protective effect of providing resistance to the
development of AIDS.

The HIV pro-virus causes latent infection or damages host cell by


apoptosis or direct killing. This leads to decline in CD4+ cell count and the
patient developing clinical symptoms.

Natural History of HIV

The acute retroviral syndrome


It is seen for 3-12 weeks and is characterized by high levels of plasma viremia, and
widespread seeding of the lymphoid tissues. Clinically there is a self-limited acute illness
with nonspecific symptoms, including sore throat, myalgias, fever, rash, weight loss, and
fatigue, and clinical features, such as rash, cervical adenopathy, diarrhea, and vomiting.
The middle chronic phase
This is characterized by a period of clinical latency. It is usually lasting for an average
duration of 10 years. In this phase, there is a continuous battle between the virus and the
host immune cells. The immune system is intact, but there is continuous HIV replication,
predominantly in the lymphoid tissues, which may last for several years. Patients are either
asymptomatic or develop persistent generalized lymphadenopathy.

The final phase or the stage of crisis


It is associated with the loss of host immune cells in the battle and the progression to AIDS.
It is characterized by the patient presenting with a long-lasting fever (>1 month), fatigue,
weight loss, and diarrhea.
CLINICAL FEATURES
The typical adult patient with AIDS presents with fever, weight loss, diarrhea,
generalized lymphadenopathy, multiple opportunistic infections, neurologic
disease and secondary neoplasms.
The opportunistic infections seen are:

Bacterial infections Viral infections Fungal Protozoal


infections infections
• M. tuberculosis • Cytomegalovirus • Candidiasis • Cryptosporid
• Salmonella • Herpes simplex virus • Pneumocystis ium
• Nocardiosis • Varicella zoster virus jiroveci • Isosporidium
• Atypical • JC virus causing • Cryptococcos • Toxoplasmo
mycobacterial Progressive multifocal is sis
infections leukoencephalopahty • Histoplasmosi
s
• Coccidiomyco
sis

Neoplasms in AIDS

1. Kaposi’s sarcomaQ: It is caused due to infection with Kaposi sarcoma


herpesvirus (KSHV) or human herpes virus -8 (HHV8). It is characterized
by the proliferation of spindle-shaped cells that express markers of both
endothelial (vascular or lymphatic) and smooth muscle cellsQ. KSHV
infection is related to rare B cell lymphomas in AIDS patients known as
body cavity based primary effusion lymphoma and to a multicentric B-cell
lymphoproliferative disorder called as Castleman disease.

Fig. 6: Kaposi sarcoma with spindle shaped cells.

2. Lymphomas: AIDS related lymphomas include


i. Systemic lymphomas having the CNS as the most common
extranodal site for development of lymphomaQ,
ii. Primary CNS lymphoma found more commonly in AIDS than in
general population
iii. Body cavity lymphomas present as pleural, peritoneal or pericardial
effusions.
These tumors are more frequently seen in patients with CD4+ T cell count
<50 per microlitre.
3. Genital cancers including cancer of the cervix and the anal cancers due
to infection with human papilloma virus (HPV).

Neurological Manifestations in AIDS


The neurological manifestations are due to the involvement of the
microgliaQ. These include
1. Opportunistic infections
2. Neoplasms
3. Aseptic meningitis
4. Peripheral neuropathies
5. AIDS-dementia complexQ
6. Vacuolar myelopathy: It is a disorder of the spinal cord found in 20% to 30% of
patients with AIDS. The findings resemble those of subacute combined degeneration,
though serum levels of vitamin B12 are normal.
7. Meningoencephalitis: HIV encephalitis is characterized microscopically as a chronic
inflammatory reaction with widely distributed infiltrates of microglial nodules around
the small blood vessels showing abnormally prominent endothelial cells and
perivascular foamy or pigment-laden macrophages. These nodules also contain the
macrophage-derived multinucleated giant cell.
8. Inflammatory myopathyQ: The histological findings include muscle fiber necrosis and
phagocytosis, interstitial infiltration with HIV-positive macrophages. Characteristically
vasculitis is absentQ.

Diagnosis of HIV infection or AIDS


The diagnosis of HIV is established with the following tests:
• ELISAQ is used for the detection of antibodies against viral proteins. This is the most
sensitive and the best screening test for the diagnosis of AIDS.
• Western blotQ is the most specific or the confirmatory test for HIV.
• Direct detection of the viral infection is with p24 antigen capture assay, reverse
transcriptase polymerase chain reaction (RT-PCR), DNA-PCR and culture of the virus
from the monocytes and CD4+ T cells.

The management of the disease is done by the Highly active


antiretroviral therapy (HAART) details of which can be referred from ‘Review
of Pharmacology’ by the same authors.
Some patients with advanced disease in HIV paradoxically deteriorate on initiating the
antiviral therapy. This ironical disorder whose basis in not understood is called Immune
reconstitution inflammatory syndrome.

Amyloidosis (Beta-Fibrillosis)

It is a group of diseases having in common the deposition of amyloid (a


pathologic proteinaceous substance, deposited between cells in various
tissues and organs of the body). Amyloid appears as an amorphous,
eosinophilic, hyaline, extracellular substance with the light microscope. Its
progressive accumulation can cause pressure atrophy of adjacent cells.
Nature of Amyloid
Amyloid is seen to be made up of nonbranching fibrils of indefinite length and
a diameter of approximately 7.5 to 10 nm by the electron microscope. X-ray
crystallography and infrared spectroscopy demonstrate a characteristic
cross-β-pleated sheet conformation responsible for the birefringence.
Chemically, 95% of the amyloid is made up of fibril proteins.

CLASSIFICATION OF AMYLOIDOSIS
Primary Amyloidosis
It is associated with immunocyte dyscrasias like multiple myeloma or any
other B cell neoplasm.
The tumor cells in multiple myeloma secrete light chains of the
immunoglobulins of either lamda or kappa type which get deposited in the
tissues as amyloid. The chemical nature of the amyloid is ALQ (A for amyloid
and L for light chain).
Secondary Amyloidosis (also called as Reactive Systemic Amyloidosis)
It is usually seen secondary to chronic inflammatory conditions like
rheumatoid arthritisQ (most commonly), tuberculosis, bronchiectasis,
chronic osteomyelitis, inflammatory bowel disease, ankylosing spondylitis
and two cancers namely renal cell cancer and Hodgkin’s disease. There is
release of IL-1 and IL-6 which act on the liver cells leading to the secretion of
SAA protein which gives rise to AA protein being deposited in this condition.
The chemical nature of amyloid is AAQ.

Hemodialysis Associated Amyloidosis


It is caused by the deposition of the b2 microglobulin which is a component
of MHC class I molecule and can not be filtered through the cuprophane
dialysis membrane. It gets deposited in the synovium, joints and the tendon
sheaths leading to carpal tunnel syndrome. The chemical nature of the
amyloid is Ab2Q.

Heredofamilial Amyloidosis
i. Familial Mediterranean fever is an autosomal recessive condition
characterized by development of attacks of fever associated with
inflammation of serosal surfaces (pleura, peritoneum and synovial
membrane). The amyloid protein deposited is AA protein and the protein
associated with this condition is called pyrinQ.
ii. Familial amyloidotic neuropathies (several types):
This is a group of autosomal dominant conditions in which both peripheral
and autonomic nerves are involved. There is deposition of ATTR (A for
amyloid and TTR is for transthyretin, a protein which transports thyroxine
and retinol). The transthyretin deposited in this condition is a mutant
form of the normal proteinQ.
iii. Systemic senile Amyloidosis
This is a condition characterized by the deposition of structurally
normal transthyretinQ, the chemical nature of amyloid is ATTR and it is
usually deposited in the heart of aged individuals leading sometimes to
the development of restrictive cardiomyopathy.

LOCALIZED AMYLOIDOSIS
There is presence of nodular deposits most often in lung, larynx, skin, urinary
bladder, tongue and around the eyes.
i. Senile cerebral amyloidosis
It is seen in Alzheimer’s disease in which there is deposition of b-amyloid
protein. So, chemical nature of amyloid is AbQ.
ii. Endocrine
It is associated with:
– Medullary carcinoma of thyroid having the deposition of ACalQ
derived from calcitonin
– Islet of Langerhans in Type II DM having deposits of AIAPPQ derived
from Islet Amyloid Peptide
iii. Isolated Atrial Amyloidosis
In this condition, there is deposition of AANF derived from Atrial natriuretic
factor.
iv. Prion disease
In this condition, there is deposition of misfolded prion proteins APrP derived
from normal prion protein PrP.
Summary of clinical conditions and the chemical nature of amyloid
S. Amyloid
No. protein Precursor Disease
1. AL Ig light chain Multiple myeloma
(primary amyloidosis)
2. AA SAA Secondary or reactive
amyloidosis
3. Ab2m b2 microglobulin Hemodialysis
Associated
amyloidosis
4. ATTR Mutant Transthyretin Familial amyloidotic
Normal Transthyretin neuropathy
Systemic senile
amyloidosis
5. Ab Ab precursor protein Senile cerebral
Alzheimer’s
6. ACal Calcitonin Medullary carcinoma
of thyroid
7. AIAPP Islet amyloid polypeptide Type II diabetes
8. AANF ANP Isolated atrial
amyloidosis
Misfolded prion
protein (APrP)
disease
9. Aa Fibrinogen Familial renal
amyloidosis
10. ACys Cystatin Cerebral amyloid
angiopathy

Morphology in Amyloidosis

Kidney
It is the most common and most serious form of organ involvement and is usully involved in
secondary amyloidosis. There is deposition primarily in the mesangiumQ (initial affected
site) followed by glomerular basement membrane and the interstitial peritubular tissue.
Arteries and arterioles are also affected (venules are spared).
Spleen
There is splenomegaly. If there is involvement of splenic follicles, it is called as Sago
spleen and if there is involvement of splenic sinuses and red pulp it is called as
Lardaceous spleen.
Liver
It is first deposited in the space of Disse and later result in hepatomegaly. The liver
function tests are usually normal.
Heart
It is more commonly associated with primary amyloidosis. It is the most important organ
involved in senile systemic amyloidosis. Clinically, there may be development of arrhythmia
and it is also the most important cause of restrictive cardiomyopathy. There is deposition in
the focal subendocardial region.
Adrenals
The intercellular deposits begin initially in zona glomerulosa.
GIT
The GI tract may be involved through the gingiva to the anus. The deposition of the
amyloid in the tongue results in the nodular enlargement of tongue called macroglossia or
the tumor forming amyloid of the tongue.

Clinical features are non-specific and the symptoms are seen depending
on the organ predominantly affected in the disease. Deposition of the amyloid
in long term hemodialysis takes place in joints and in the carpal ligament of
the wrist, the latter leading to development of ‘carpal tunnel syndrome’.

Diagnosis
The diagnosis is made by the microscopic examination of the biopsy from
renal tissue, rectum, abdominal fat aspiration and gingiva. The best site for
taking the biopsyQ is abdominal fat aspirateQ followed by rectal biopsy.
Grossly, the organs are enlarged and firm with a waxy appearance. The cut
surface on painting with iodine imparts a yellow color which on application of
sulfuric acid (H2SO4) gives a blue violet color.

STAINING FOR AMYLOID


• Congo red: It is the most widely used specific stain for amyloid.
• Iodine staining: It is used for unfixed specimen or histological
section. Amyloid stains mahogany brown and if sulfuric acid is
added, it turns violet.
• Thioflavin ‘T’ and ‘S’ give secondary immunofluorescence with
ultraviolet light. Thioflavin T is more useful for demonstrating
juxtaglomerular apparatus of the kidney.
• Metachromatic stains like crystal violet and methyl violet give rose pink
appearance.
• Amyloid is PAS positive.

Fig. 7: Amyloid with congo red staining (Left) and Apple green birefringence (Right). ...(All
India Image).

The condition has usually poor prognosis.


Summary of Appearance of Amyloid for exam questions
On light microscopy and standard tissue Amorphous eosinophilic extracellular
stains (H and E) substance
Congo red stain on ordinary light Pink or red color to tissue deposits
Congo red stain on polarizing Apple green birefringenceQ
microscopy
Fluorescent stains (thioflavin T and S) Yellow color under UV light
Electron microscopy Nonbranching fibrilsQ of indefinite length
and a diameter of approximately 7.5 to 10
nm.
X-ray crystallography and infrared Characteristic cross b pleatedQ sheet
spectroscopy conformation
IMMUNE CELL: GENERAL ASPECTS

1. Which of the following features is not shared between ‘T cells’ and B


cells’?
(AIIMS Nov 2012)
(a) Antigen Specific Receptors
(b) Class I MHC Expression
(c) Positive selection during development
(d) All of the above
2. CD4 is not important for which of the following?
(a) Antibody production
(AIIMS May 2011)
(b) Cytotoxicity of T cells
(c) Memory B cells
(d) Opsonisation
3. Type 1 MHC presents peptide antigen to T cell , so that peptide
binding site is formed by:
(AI 2010)
(a) Alfa and Beta chain
(b) Distal domain alfa 1 and 2
(c) Alfa and beta microglobulin
(d) Proximal domain alfa 1 and 2
4.Function of CD4 is all except:
(AI 2009)
(a) Memory
(b) Immunoglobulin production
(c) Activation of macrophages
(d) Cytotoxicity
5. A super-antigen is a bacterial product that
(AI 2008)
(a) Binds to B7 and CD28 co-stimulatory molecules
(b) Binds to the beta chain of TCR and MHC class II molecules of APC
stimulating T cell activation
(c) Binds to the CD4 + molecule causing T cell activation
(d) Is presented by macrophages to a larger-than-normal number of T
helper CD4 + lymphocytes
6. Memory T cells can be identified by using the following marker:
(AI 2003)
(a) CD45RA
(b) CD45RB
(c) CD45RC
(d) CD45RO
7. All of the following statements about NK cells are true except:
(AI 2003)
(a) They are derived from large granular cells
(b) They comprise about 5% of human peripheral lymphoid cells
(c) They are MHC restricted cytotoxic cells
(d) They express IgG Fc receptors
8. The following feature is common to both cytotoxic T-cells and NK
cells:
(AI 2002)
(a) Synthesize antibody
(b) Require antibodies to be present for action
(c) Effective against virus infected cells
(d) Recognize antigen in association with HLA class II markers
9. MHC restriction to antigen presentation is not done for:
(AIIMS May 2009)
(a) Killing of viruses by cytotoxic cells
(b) Killing of bacteria by helper cells
(c) T cell activation in autoimmunity
(d) Graft rejection
10. Most potent stimulator of naive T cell is:
(a) Mature dendritic cell (AI 2011, AIIMS Nov 08)
(b) Follicular dendritic cell
(c) Macrophages
(d) B cell
11. Natural killer cells attacks which of the following cells:
(AIIMS Nov 2006)
(a) Cells which express MHC1
(b) Cells which are not able to express MHC1
(c) MHC cells which express MHC2
(d) Cells which are not able to express MHC
12. Toll like receptors, recognize bacterial products and stimulates
immune response by:
(AIIMS Nov 2006)
(a) Perforin and granzyme mediated apoptosis
(b) FADD ligand apoptosis
(c) Transcription of nuclear factor mediated by N-FκB which recruits
cytokines
(d) Cyclin
13. The following interleukin is characteristically produced in a TH1
response:
(AIIMS Nov 2004)
(a) IL-2
(b) IL-4
(c) IL-5
(d) IL-10
14. CD-95 has a major role in:
(AIIMS Nov 2003)
(a) Apoptosis
(b) Cell necrosis
(c) Interferon activation
(d) Proteolysis
15. Which of the following chemical mediators of inflammation is an
example of a C-X-C or alpha Chemokine?
(AIIMS Nov 2003)
(a) Lipoxin LXA4
(b) Interleukin IL-8
(c) Interleukin IL-6
(d) Monocyte Chemo-attractant Protein MCP-1
16. The complement is fixed best by which of the following
immunoglobulins:
(AIIMS May 2002)
(a) IgG
(b) IgM
(c) IgA
(d) IgD
17. Antigen presenting cells are which of the following:
(a) Astrocytes
(AIIMS May 2002)
(b) Endothelial cells
(c) Epithelial cells
(d) Langerhan’s cells
18. Antigen presenting cells are:
(PGI June 2006)
(a) Langerhan’s cell
(b) Macrophage
(c) Cytotoxic T cells
(d) Helper T cells
(e) B-lymphocyte
19. Perforins are produced by:
(PGI Dec 2001)
(a) Cytotoxic T cells
(b) Suppressor T cells
(c) Memory helper T cells
(d) Plasma cells
(e) NK cells
20. Cell surface molecules involved in peripheral tolerance induction
are:
(PGI Dec 2003)
(a) B7 and CD28
(b) CD40 and CD40L
(c) CD34 and CD51
(d) B7 and CD3
21. Marker for B-Lymphocyte:
(PGI Dec 2004)
(a) CD34
(b) CD33
(c) CD19
(d) CD20
(e) CD22
22. IL-1 causes
(Delhi PG-2008)
(a) Increased leukocyte adherence
(b) Fibroblast proliferation
(c) Increased collagen synthesis
(d) All of the above

MOST RECENT QUESTIONS

23. Antigen presenting cells present in skin are called


(a) Langerhan’s cells
(b) Kupffer’s cells
(c) Microglia
(d) Melanocytes
24. Plasma cells
(a) Contain nucleus
(b) Helps in the formation of antibody
(c) Are deficient in cytoplasm
(d) Are derived from T-cells
25. The normal ratio of CD4 to CD8 is
(a) 1: 1
(b) 2: 1
(c) 8: 1
(d) 10: 1
26. CD4 cells is used to identify which of the following
(a) MHC I
(b) MHC II
(c) T cells
(d) B cells
27. CD3 is marker for:
(a) Monocyte
(b) T cell
(c) B cell
(d) None
28. Which of the following is not true about innate immunity?
(a) It is present prior to antigenic exposure
(b) It is relatively non-specific
(c) Memory is seen
(d) It is the first line of defense
29. Which one of the listed receptors is the type of receptor on
leukocytes that binds to pathogen-associated molecular patterns
(PAMPs) and mediates immune response to bacterial
lipopolysaccharide?
(a) Cytokine receptor
(b) G-protein-coupled receptor
(c) Mannose receptor
(d) Toll-like receptor
30. Immunity against cancer cells:
(a) Basophils
(b) Eosinophils
(c) NK cells
(d) Neutrophils
31. NK cells express:
(a) CD 15, CD 55
(b) CD 16, CD 56
(c) CD 16, CD 57
(d) CD 21, CD 66
32. NK cell CD marker is:
(a) 16
(b) 60
(c) 32
(d) 25
33. The following interleukin is characteristically produced in a TH1
response?
(a) IL-2
(b) IL-4
(c) IL-5
(d) IL-10
34. Most potent stimulator of Naïve T-cells:
(a) Mature dendritic cells
(b) Follicular dendritic cells
(c) Macrophages
(d) B-cell
35. Which of the following immune cells have the expression of CD8 on
their surface?
(a) T-cells
(b) B-cells
(c) Null cells
(d) Macrophages
36. Kupffer cells are found in:
(a) Heart
(b) Lungs
(c) Liver
(d) Spleen
37. Birbeck granules are present in:
(a) Merkel cell
(b) Langerhans cell
(c) Langhans cell
(d) Melanocyte
38. Macroglobulin is derived from:
(a) B cells
(b) T cells
(c) Both
(d) Natural killer cells
39. Which of the following is not true regarding IgE antibodies?
(a) It mediates release of histamine and other chemical mediators
(b) It is the primary antibody involved in allergic reactions
(c) It is involved in anti-parasitic immune responses
(d) May cross the placenta and fix complement
40. Which of the following immunoglobulin does not fix complement?
(a) IgA
(b) IgG
(c) IgM
(d) IgE
41.B cells are located in which region of lymph nodes:
(a) Paracortical region
(b) Cortical follicles
(c) Medullar sinuses
(d) Subcapsular region
42. Plasma cells produce specific antibodies by
(a) Isotope switching
(b) Class selection
(c) Isotope selection
(d) Clonal selection
43. Surface Immunoglobulin is found in which cell?
(a) T-cell
(b) B-cell
(c) NK cell
(d) Plasma cells

MHC

44. MHC class III genes encode:


(AI 2003)
(a) Complement component C3
(b) Tumor necrosis factor
(c) Interleukin 2
(d) Beta 2 microglobulin
45. The HLA class III region genes are important elements in:
(AI 2003)
(a) Transplant rejection phenomenon
(b) Governing susceptibility to autoimmune diseases
(c) Immune surveillance
(d) Antigen presentation and elimination
46. HLA is located on:
(AIIMS Nov 2009)
(a) Long arm of chromosome 6 (DNB 05,09)
(b) Long arm of chromosome 3
(c) Short arm of chromosome 6
(d) Short arm of chromosome 3
47. HLA B27 is positive in:
(AIIMS Nov 2009)
(a) Ankylosing spondylitis
(b) Rheumatoid arthritis
(c) SLE
(d) Behçet syndrome
48. Mixed lymphocyte culture is used to identify:
(a) MHC class I antigen
(AIIMS Nov 2002)
(b) MHC class II antigen
(c) B lymphocytes
(d) T helper cells
49. HLA typing is useful in:
(PGI June 2006)
(a) Disputed paternity
(b) Thanatology
(c) Organ transplant
(d) Dactylography
50. True about MHC-class II:
(PGI June 2006)
(a) Not involved in innate immunity
(b) Cytotoxic T-cell involved
(c) Present in nucleated cells
(d) Present in B-cells
51. MHC-II positive cells are all except:
(PGI Dec 2000)
(a) B cells
(b) T cells
(c) Macrophages
(d) Platelets
(e) RBCs
52. True about MHC:
(PGI June 2003)
(a) Transplantation reaction
(b) Autoimmune disease
(c) Immunosuppression
(d) Involved in T-cell function
(e) Situated at long arm of chromosome 6

MOST RECENT QUESTIONS

53. Epitope binding floor of the MHC molecule consists of


(a) Alpha helices
(b) Beta pleated structure
(c) Alpha and beta-1 chain
(d) Beta-2 microglobulin
54. HLA B27 is not seen in which of the following?
(a) Ankylosing spondylitis
(b) Reiter’s syndrome
(c) Rheumatoid arthritis
(d) Psoriatic arthritis
55. The role played by Major Histocompatibility Complex 1 and 2:
(a) Transduce the signal to T cells following antigen recognition
(b) Mediate immunogenic class switching
(c) Present antigens for recognition by T cell antigen receptors
(d) Enhance the secretion of cytokines
56. MHC class I are present on all except:
(AIIMS Nov 2016)
(a) Platelets
(b) All nucleated cells
(c) RBCs
(d) WBCs
57. Major histocompatibility complex class I is seen on which of the
following cell?
(a) Macrophages only
(b) All body cells
(c) B cell only
(d) All blood cells except erythrocytes
58.Antigen presented along with HLA class II stimulate
(a) CD8 cell
(b) CD4 cell
(c) CD2 cell
(d) CD19 cell
59. Which of the following is having a 90% association with HLA B27?
(a) Ankylosing spondylitis
(b) Rheumatoid arthritis
(c) Psoriasis
(d) Reiter syndrome
60.Which of the following is the function of MHC I and II?
(a) Signal transduction in T cells
(b) Antibody class switching
(c) Antigen presentation to T cells
(d) Increase the secretion of cytokines
61. Antigen presenting cells are all except:
(AIIMS May 2016)
(a) M-cells
(b) Thymocytes
(c) Macrophages
(d) Langerhans cells
62. MHC-2 protein is present in all except:
(a) Cortical macrophages
(b) Medullary macrophages
(c) Cortical epithelial cells
(d) Medullary epithelial cells
63. HLA-Cw6 is associated with:
(a) Myasthenia gravis
(b) Behcets disease
(c) Pemphigus vulgaris
(d) Psoriasis vulgaris
64. HLA class II is linked with which of the following?
(a) Graft rejection
(b) Graft versus host disease
(c) Killing of virus infected cells
(d) Susceptibility to autoimmune diseases
65. HLA association with myasthenia gravis is:
(a) HLA-B27
(b) HLA-B51
(c) HLA-B47
(d) HLA-B8
66. HLA associated with pustular psoriasis is:
(a) HLA CW6
(b) HLA B13
(c) HLA B27
(d) HLA B17
67. To rule out rheumatoid arthritis, most important among the
following is:
(a) HLA DR8
(b) HLA DR4
(c) HLA DQ1
(d) HLA B27

HYPERSENSITIVITY REACTIONS

68. What type of hypersensitivity reaction is seen in myasthenia


gravis?
(AI 2012)
(a) Type 1 hypersensitivity reaction
(b) Type 2 hypersensitivity reaction
(c) Type 3 hypersensitivity reaction
(d) Type 4 hypersensitivity reaction
69. Hemolytic disease of newborn is an example of:
(a) Type 3 hypersensitivity reaction
(DPG 2011)
(b) Type 2 hypersensitivity reaction
(c) Arthus reaction
(d) Type 4 hypersensitivity reaction
70. Raji cell assays are used to quantitate:
(DPG 2011)
(a) Complement levels
(b) Immune complexes
(c) T cells
(d) Interferon levels
71. Hypersensitivity pneumonitis is classically a/an:
(a) Allergic reaction
(AI 2009)
(b) Type II hypersensitivity
(c) Immune complex mediated hypersensitivity
(d) Cell mediated hypersensitivity
72. The immunoglobulin involved in type I hypersensitivity reaction is:
(AI 2007)
(a) IgE
(b) IgM
(c) IgA
(d) IgG
73. Arthus reaction is what type of hypersensitivity reaction:
(AI 2007), (UP’03)
(a) Localized immune complex
(b) Ag-Ab reaction
(c) Complement mediated
(d) Ab mediated
74. A 40 year old man has chronic cough with fever for several months.
The chest radiograph reveals a diffuse reticulonodular pattern.
Microscopically on transbronchial biopsy there are focal areas of
inflammation containing epithelioid cell granuloma, Langhans
giant cells, and lymphocytes. These findings are typical for which
of the following type of hypersensitivity immunologic responses:
(a) Type I
(AIIMS May 2003)
(b) Type II
(c) Type III
(d) Type IV
75. Ram Devi presented with generalized edema sweating and flushing
tachycardia and fever after bee sting. This is:
(AIIMS Nov 2001)
(a) T cell mediated cytotoxicity
(b) IgE mediated reaction
(c) IgG mediated reaction
(d) IgA mediated hypersensitivity reaction
76. Example of Type IV Hypersensitivity is/are:
(a) Farmer’s lung
(PGI June 2006)
(b) Contact hypersensitivity
(c) Immediate hypersensitivity
(d) Myasthenia gravis
77. Example of Type II Hypersensitivity is/are:
(a) Blood transfusion reaction
(PGI June 2006)
(b) Arthus reaction
(c) Hay Fever
(d) Post-streptococcal glomerulonephritis
78. Which of the following diseases is/are mediated through
complement activation:
(PGI Dec 03)
(a) Atopic dermatitis
(b) Graft versus Host disease
(c) Photoallergy
(d) Necrotizing vasculitis
(e) Urticaria
79. Which of following statements is not true about Mycobacterium
tuberculosis infection?
(Delhi PG 2009 RP)
(a) M. tuberculosis leads to development of delayed hypersensitivity
(b) Lymphocytes are the primary cells infected by M. tuberculosis
(c) Positive tuberculin test signifies cell mediated hypersensitivity
(d) Tuberculin test does not differentiate between infection and
disease.

MOST RECENT QUESTIONS


80. A man after consuming sea food develops rashes. It is due to:
(a) IgE mediated response
(b) Complement activation
(c) Cell mediated response
(d) None of the above
81. Granuloma in Sarcoidosis is called
(a) Hard sore
(b) Soft sore
(c) Hard tubercle
(d) Caseating granuloma
82. Myasthenia gravis may be associated with
(a) Thymoma
(b) Systemic lupus erythematosus
(c) Hyperthyroidism
(d) All of the above
83. Which of the following type of hypersensitivity reaction is found in
blood transfusion reaction?
(a) Anaphylactic type
(b) Cytotoxic type
(c) Type III hypersensitivity
(d) Cell mediated hypersensitivity
84. Which of the following type of hypersensitivity reactions occurs in
Farmer’s lung?
(a) Type I
(b) Type II
(c) Type III
(d) Type IV
85. Tuberculin test positivity indicates:
(a) Good humoral immunity
(b) Infection with mycobacterium
(c) Good cell mediated immunity
(d) None
86. Cell mediated immunity is:
(a) Type I
(b) Type II
(c) Type III
(d) Type IV
87. Antibody found in patients with myasthenia gravis is directed
against
(a) Acetylcholine
(b) Acetylcholine receptors
(c) Acetylcholine vesicles in nerve terminal
(d) Actin-myosin complex of the muscle
88. Myasthenia gravis is most commonly associated with which of the
following?
(a) Thymoma
(b) Thymic carcinoma
(c) Thymic hyperplasia
(d) Lymphoma
89. Patient has been given penicillin 48 hours ago, with no history of
drug allergy. Now he develops wheeze and hemolysis. Antibody
for penicillin is positive. Type of hypersensitivity is which of the
following:
(a) Type I
(b) Type II
(c) Type III
(d) Type IV
90. IgE receptors are present on:
(a) Mast cells
(b) NK cells
(c) B cells
(d) Histiocytes
91. A 45-year-old patient presents with history of fever, night sweats
and weight loss. On X-ray a mass in apical lobe of lung is seen. On
histopathology it was found to have caseous necrosis. What is the
likely underlying process involved?
(a) Enzymatic degeneration
(b) Hypersensitivity reaction with modified macrophages, lymphocytes
and giant cells
(c) Acute decrease in blood supply
(d) Decreased supply of growth factors
92. Serum sickness is:
(AIIMS May 2016)
(a) Type 1 Hypersensitivity reaction
(b) Type 2 Hypersensitivity reaction
(c) Type 3 Hypersensitivity reaction
(d) Type 4 Hypersensitivity reaction
93. Which of the following is true about serum sickness ?
(a) Type -2 hypersenstivity
(b) Can lead to leukocytoclastic vasculitis
(c) Hypercomplementenemia
(d) Can occue due to homologus antigen
94. Centre of tubercular granuloma is formed by:
(a) T-lymphocytes
(b) B-lymphocytes
(c) Langhan’s giant cells
(d) Necrotic zone

TRANSPLANT REJECTION, GVHD

95. Hyperacute rejection is due to


(AIIMS Nov 2012)
(a) Preformed antibodies
(b) Cytotoxic T-lymphocyte mediated injury
(c) Circulating macrophage mediated injury
(d) Endothelitis caused by donor antibodies
96. All are affected in Graft-Versus host reaction:
(AIIMS May 2007)
(a) Skin
(b) GIT
(c) Liver
(d) Lung
97. Preformed antibodies cause:
(PGI June 2006)
(a) Hyperacute rejection
(b) Acute rejection
(c) Chronic rejection
(d) Acute humoral rejection
98. True about graft versus host disease is:
(PGI Dec 2005)
(a) Associated with solid organ transplantation
(b) Graft must contains immunocompetent T cell
(c) It is seen in immunosuppressed persons
(d) Also called as Runt disease in animals
99. Acute humoral renal transplant rejection is characterized by the
following, except: (Delhi PG 2009 RP)
(a) Presence of anti-donor antibodies
(b) Interstitial and tubular mononuclear cell infiltrate
(c) Necrotizing vasculitis
(d) Acute cortical necrosis
100. Transfer of the graft of different species are called as:
(UP 2002)
(a) Isograft
(b) Allograft
(c) Homograft
(d) Xenograft
101. Acute graft versus host disease reaction occurs in all except:
(UP 2007)
(a) Liver
(b) Adrenal
(c) Gut
(d) Skin

MOST RECENT QUESTIONS

102. Principal cause of death in renal transplant patient is:


(a) Uraemia
(b) Malignancy
(c) Rejection
(d) Infection
103. Preformed antibodies cause:
(a) Hyperacute rejection
(b) Acute rejection
(c) Chronic rejection
(d) Acute humoral rejection
104. Which of these complement factor is a marker of humoral
rejection?
(AIIMS May 2016)
(a) C3d
(b) C5a
(c) C3b
(d) C4d
105. Method of prevention of GVHD in bone marrow transplantation is:
(a) T-cell removal
(b) Prior immune suppression
(c) Post procedure immune suppression
(d) All of the above
106. True about adult autologous stem cell transplant are all except:
(a) Used in the treatment of leukemia
(b) Stem cells are collected directly from the bone marrow
(c) G-CSF is given to expand the number of stem cells
(d) It allows high dose of chemotherapy
107. Number of criteria for HLA matching are:
(a) 10 (b) 4
(c) 16 (d) 22
108. The commonest type of graft rejection is:
(a) Hyperacute rejection
(b) Acute rejection
(c) Chronic rejection
(d) All are equal in incidence
109.Organ with least chance of rejection is:
(a) Blood
(b) Liver
(c) Kidney
(d) Heart
110. Runt disease is associated with:
(a) Acute rejection
(b) Hyperacute rejection
(c) Chronic rejection
(d) Graft versus host disease
111. Prior immune suppression is not helpful in which type of graft
rejection:
(a) Acute rejection
(b) Hyperacute rejection
(c) Chronic rejection
(d) None of the above
112. Which of the following is true about GVHD?
(a) Occurs when host is immunocompetent
(b) Occurs when donor cells are immunocompetent
(c) Most common organ involved is lung
(d) Most common in renal transplant
113. Microcytotoxicity is used for:
(a) Tissue typing
(b) Drug allergy
(c) Infection Susceptibility
(d) Substance toxicity
114. Acute graft rejection occurs within:
(a) 3 hours
(b) 3 days
(c) 3 months
(d) 3 years

AUTOIMMUNE AND IMMUNODEFICIENCY DISEASES


115. Autoimmunity in EBV infection is the result of:
(a) Molecular mimicry
(AI 2012)
(b) Polyclonal B cell activation
(c) Expressing sequestrated antigens
(d) Antigenic cross reactivity
116. A 14 years old girl on exposure to cold has pallor of extremities
followed by pain and cyanosis. In later ages of life she is prone to
develop?
(AIIMS May 2011)
(a) Systemic lupus erythematosus
(b) Scleroderma
(c) Rheumatoid arthritis
(d) Histiocytosis
117. Which is not autoimmune disease?
(AI 2011)
(a) Systemic Lupus Erythematosus
(b) Grave’s Disease
(c) Myasthenia Gravis
(d) Sickle Cell Disease
118. Which among the following is seen in antiphospholipid antibody
syndrome?
(AI 2011)
(a) Beta 2 microglobulin antibody
(b) Anti nuclear antibody
(c) Anti centromere antibody
(d) Anti glycoprotein antibody
119. Necrotizing lymphadenitis is seen in:
(AI 2011)
(a) Kimura disease
(b) Kikuchi Fujimoto disease
(c) Hodgkin disease
(d) Castelman disease
120. Wire loop lesions are seen in:
(DPG 2011)
(a) SLE
(b) Diabetic nephropathy
(c) Benign nephrosclerosis
(d) Wegener’s granulomatosis
121. Tissue from rat used for detection of antinuclear antibodies?
(AIIMS Nov 2009)
(a) Kidney
(b) Brain
(c) Stomach
(d) Liver
122. Which is not found in CNS in a case of AIDS?
(a) Perivascular giant cell
(AIIMS May 2009)
(b) Vacuolization
(c) Inclusion bodies
(d) Microglial nodule
123. A person present with recurrent swelling on face and lips due to
emotional stress. Likely cause is:
(a) C1 esterase inhibitor deficiency
(AIIMS May 2009)
(b) Allergy
(c) Anaphylaxis
(d) None of the above
124. All of the following statements are true about Wiskott Aldrich
syndrome except:
(AIIMS Nov 2008)
(a) It is an autosomal recessive disorder
(b) There is failure of aggregation of platelets in response to agonists
(c) Thrombocytopenia is seen
(d) Patient presents with eczema
125. Hematoxylin bodies seen in:
(AIIMS May 2008)
(a) SLE
(b) PAN
(c) Rheumatoid arthritis
(d) Wegener’s granulomatosis
126. Wire loop lesions are often characteristic for the following class of
lupus nephritis:
(AIIMS May 2004)
(a) Mesangial proliferative glomerulonephritis (WHO class II)
(b) Focal proliferative glomerulonephritis (WHO class III)
(c) Diffuse proliferative glomerulonephritis (WHO class IV)
(d) Membranous glomerulonephritis (WHO class V)
127. A renal biopsy from a 56 year old woman with progressive renal
failure for the past 3 years shows glomerular and vascular
deposition of pink amorphous material. It shows apple-green
birefringence under polarized light after Congo red staining. These
deposits are positive for lambda light chains. The person is most
likely to suffer from:
(a) Rheumatoid arthritis
(AIIMS May 2003)
(b) Tuberculosis
(c) Systemic lupus erythematosus
(d) Multiple myeloma
128. A young lady presented with bilateral nodular lesions on shins.
She was also found to have bi-lateral hilar lymphadenopathy on
chest X-ray. Mantoux test reveals indurations of 5 mms. Skin
biopsy would reveal:
(a) Non caseating granuloma
(AIIMS May 2002)
(b) Vasculitis
(c) Caseating granuloma
(d) Malignant cells
129. Anti ds-DNA antibodies are commonly seen in:
(PGI June 01)
(a) SLE
(b) Scleroderma
(c) PAN
(d) Dermatomyositis
(e) Rheumatoid arthritis
130. Low complement levels seen in:
(PGI Dec 2006)
(a) PSGN
(b) MPGN
(c) Good pasture’s syndrome
(d) Wegner’s granulomatosis
(e) Infective endocarditis.
131. Which is seen in Chediak-Higashi syndrome?
(a) Leucocytosis
(PGI Dec 2001)
(b) Neutropenia
(c) Defective microbial killing
(d) Presence of large granules in neutrophil
(e) Immunodeficiency
132. Adenosine deaminase deficiency is seen in:
(a) Severe combined immunodeficiency
(PGI Dec 2003)
(b) Wiskott Aldrich Syndrome
(c) Agammaglobulinemia as HIV
133. True about alpha-1 antitrypsin deficiency, is/are:
(PGI June 01)
(a) Autosomal dominant
(b) Pulmonary emphysema
(c) Diastase resistant hepatic cells
(d) Hepatic cells are orcein stain positive
(e) Associated with berry aneurysm
134. All are true regarding Hyper IgE syndrome except:
(Delhi PG 2009)
(a) Inheritance is as a single locus autosomal dominant trait with
variable expression
(b) Coarse facial features
(c) Recurrent staphylococcal abscesses involving skin, lungs
(d) High serum IgE with low IgG, IgA and IgM
135. All are true about Wiskott-Aldrich Syndrome except:
(Delhi PG 2009)
(a) Bloody diarrhea during infancy
(b) Low IgM and elevated IgA and IgE
(c) Large size platelets
(d) Atopic dermatitis
136. Diagnosis of X linked Agammaglobulinemia should be suspected
if:
(Delhi PG 2009)
(a) Absent tonsils and no palpable lymph nodes on physical
examination
(b) Female sex
(c) High isohemagglutinins titers
(d) Low CD3
137. Which of the following cell types is not a target for initiation and
maintenance of HIV infection?
(Delhi PG 2009 RP)
(a) CD4 T cell
(b) Macrophage
(c) Dendritic cell
(d) Neutrophil
138. All of the following are found in SLE except:
(a) Oral ulcers
(Delhi PG-2006)
(b) Psychosis
(c) Discoid rash
(d) Leucocytosis
139. Which of the following immunoglobulin is absent in Ataxia
telangiectasia:
(Delhi PG-2005)
(a) IgG
(b) IgM
(c) IgA
(d) IgD
140. Scl-70 antibody is characteristic of:
(a) Systemic lupus erythematosus
(Karnataka 2007)
(b) Scleroderma
(c) Dermatomyositis
(d) Sjogren’s syndrome
141. LE cell phenomenon is seen in:
(Karnataka 2005)
(a) Lymphocyte
(b) Neutrophil
(c) Monocyte
(d) Eosinophil
142. Most sensitive test for screening of “Systemic Lupus
Erythematosus” (SLE) is:
(Karnataka 2005, RJ 2002)
(a) LE phenomenon
(b) Rheumatoid factor
(c) Anti-nuclear factor (ANF)
(d) Double stranded DNA test
143. According to WHO, the feature of class II lupus is:
(a) Transient proteinuria
(UP 2000)
(b) Massive proteinuria
(c) Hematuria
(d) RBC casts
144. ANCA antibody with peripheral rim distribution is indicative of:
(UP 2000)
(a) Antihistone antibody
(b) Anti smith antibody
(c) Anti double stranded DNA antibody
(d) Anti double stranded RNA antibody
145. Basic pathology in cystic fibrosis is:
(UP 2001)
(a) Defect in the transport of chloride across epithelia
(b) Defect in the transport of sodium across epithelia
(c) Defect in the transport of potassium across epithelia
(d) Defect in the transport of bicarbonate across epithelia
146. Besbuer Boeck Schaumann disease is also called as:
(a) Sarcoidosis
(UP 2003)
(b) Crohn’s disease
(c) Whipple’s disease
(d) Hodgkin’s disease
147. Most common viral antigen used for diagnosis of HIV in blood
before transfusion is:
(UP 2005)
(a) p24
(b) p17
(c) p7
(d) p14
148. Most common vascular tumor in AIDS patients is:
(a) Kaposi ’s sarcoma
(UP 2000) (UP 2007)
(b) Angiosarcoma
(c) Lymphangioma
(d) Lymphoma
149. Which in not an autoimmune disease?
(RJ 2001)
(a) Syphilis
(b) SLE
(c) Systemic sclerosis
(d) RA
150. Bilateral parotid gland enlargement is seen in all except:
(RJ 2001)
(a) Sarcoidosis
(b) Sjogren’s syndrome
(c) SLE
(d) Viral infections
151. Sarcoidosis does not involve
(RJ 2004)
(a) Brain
(b) Heart
(c) Lung
(d) Kidney
152. Characteristic of SLE of kidney is: (RJ 2004,
Jharkhand 05)
(a) Focal sclerosis
(b) Focal necrosis
(c) Wire loop lesions
(d) Diffuse glomerulosclerosis
153. Libman-Sacks endocarditis is seen in:
(AP 2001)
(a) Rheumatoid arthritis
(b) SLE
(c) Infective endocarditis
(d) Nonbacterial thrombotic endocarditis
154. Chediak-Higashi syndrome is due to defect in:
(Kolkata 2003)
(a) Opsonisation
(b) Chemotaxis
(c) LAD
(d) Extracellular microbicidal killing

MOST RECENT QUESTIONS

155. Anti-double stranded DNA is highly specific for:


(a) Systemic sclerosis
(b) SLE
(c) Polymyositis
(d) Rheumatic sclerosis
156. Anti-topoisomerase I is marker of:
(a) Systemic sclerosis
(b) Classic polyarteritis nodosa
(c) Nephrotic syndrome
(d) Rheumatoid arthritis
157. An 8-year-old boy presents with sarcoidosis. Which of the
following is correct?
(a) Hilar lymphadenopathy with perihilar calcification
(b) Basal infiltrates
(c) Rubbery lymph nodes
(d) Egg-shell-calcification
158. Most common site for lymphoma in AIDS patients is:
(a) CNS lesions
(b) Spleen
(c) Thymus
(d) Abdomen
159. All are true about histological features of Kaposi’s sarcoma
except:
(a) Microscopically lesion similar to granulation tissue
(b) Dilated and irregular blood vessels with interspersed infiltrate of
lymphocyte and plasma cells
(c) Atypical blood vessels have solid spindle cell appearance
(d) Nodule is the initial lesion of Kaposi’s sarcoma
160. HIV affects which of the following most commonly?
(a) Helper cells
(b) Suppressor cell
(c) RBCs
(d) Platelets
161. Which of the following lesions/conditions shows most specific
anatomic changes in HIV infection?
(a) Lymph nodes
(b) Opportunistic infections
(c) CNS lesions
(d) Kaposi’s sarcoma (blood vessels)
162. Which of the following autoantibody is least likely associated with
SLE?
(a) Anti ds DNA
(b) Anti Sm
(c) Anti topoisomerase
(d) Anti histone
163. Which of the following autoantibody is specific for SLE?
(a) ds DNA
(b) Anti RO
(c) Anticentromere
(d) Anti topoisomerase
164. Regarding severe combined immunodeficiency disease, which of
the following statement is true?
(a) Adenosine deaminase deficiency
(b) Decreased circulating lymphocytes
(c) NADPH oxidase deficiency
(d) C1 esterase deficiency
165. Which of the following is a finding in lymphoid tissues in
individuals with common variable hypogammaglobulinemia?
(a) Decreased B cell count
(b) Increased B cell count
(c) Normal B cell count
(d) Absent B cells
166. Thymic hypoplasia is seen in which of the following?
(a)Wiskott-Aldrich syndrome
(b) Digeorge syndrome
(c) IgA deficiency
(d) Agammaglobulinemia
167. Onion peel appearance of splenic capsule is seen in:
(a)SLE
(b) Scleroderma
(c) Rheumatoid arthritis
(d) Sjogren syndrome
168. Following is not a feature of AIDS related lymphadenopathy:
(a) Florid reactive hyperplasia
(b) Follicle lysis
(c) Haematoxylin bodies
(d) Collection of monocytoid B – Cells in sinuses
169. Most common CNS neoplasm in HIV patient is:
(a) Meduloblastoma
(b) Astrocytoma
(c) Primary CNS Iymphoma
(d) Ependymoma
170. A false negative tuberculin reaction may be obtained in all of the
following situations except:
(a) Children previously tested with tuberculin test
(b) Post – measles test
(c) Corticosteroid therapy
(d) Miliary tuberculosis
171. Risk of HIV transmission is not seen with:
(a) Whole blood
(b) Platelets
(c) Plasma derived Hepatitis B vaccine
(d) Leucocytes
172.All of the following methods are used for the diagnosis of HIV
infection in a 2 months old child, except:
(a) DNA –PCR
(b) Viral culture
(c) HIV ELISA
(d) P 24 antigen assay
173. Mantoux test reading of less than 5 mm indicates:
(a) Tuberculous infections
(b) Disseminated TB
(c) Susceptibility to TB
(d) Immunity to TB
174. Epitope spreading refers to:
(a) A type of mechanism of spread of malignant tumors
(b) One type of mechanism of HIV dissemination
(c) A mechanism for the persistence and evolution of autoimmune
disease
(d) One of the mechanisms of apoptosis
175. Heerfordt’s syndrome consists of fever, parotid enlargement,
facial palsy and
(a) Arthralgia
(b) Bilateral hilar adenopathy
(c) Erythema nodosum
(d) Anterior uveitis
176.HIV affects CD4 cells by which protein?
(a) Gp 120
(b) Gp 41
(c) CCR5
(d) CXCR4
177. Treatment for Asymptomatic HIV is done when CD4 count is below
(a) 200
(b) 350
(c) 400
(d) 500
178. The poly-arthritic condition that is NOT common in males
(a) Gout
(b) Psoriatic arthritis
(c) Ankylosing spondylitis
(d) Systemic lupus erythematosus
179. Hodgkins lymphoma caused for by:
(a) EBV
(b) CMV
(c) HHV6
(d) HHV8
180. Which of these is an immune-privileged site?
(a) Area postrema
(b) Loop of Henle
(c) Optic nerve
(d) Seminiferous tubules
181. Which of the following is not an autoimmune disorder?
(a)Ulcerative colitis
(b) Grave’s disease
(c) Rheumatoid arthritis
(d) SLE
182.Autoimmunity is caused by all except:
(a) Infections
(b) Expression of cryptic antigens
(c) Negative selection of T- cells in the thymus
(d) Inappropriate expression of the MHC proteins
183. Which of the following is not seen in SLE affected kidneys?
(a) Focal glomerulonephritis
(b) Diffuse glomerulonephritis
(c) Membranous glomerulonephritis
(d) Lipoid nephrosis
184. Anti RO (SSA) antibodies are seen in:
(a) Systemic sclerosis
(b) Subacute cutaneous lupus
(c) Myasthenia gravis
(d) Mixed connective tissue disorder

AMYLOIDOSIS

185. Secondary amyloidosis is associated with:


(AI 2012)
(a) Ab
(b) AL
(c) AA
(d) APrP
186. A 60 year old female is suffering from renal failure and is on
hemodialysis since last 8 years. She developed carpal tunnel
syndrome. Which of the following finding will be associated?
(AIIMS Nov 2011)
(a) AL
(b) AA
(c) ATTR
(d) β2 microglobulin
187. The best investigation for the diagnosis of amyloidosis is:
(AIIMS May 2010)
(a) Colonoscopy
(b) Rectal biopsy
(c) Upper GI endoscopy
(d) CT scan
188. Which type of amyloidosis is caused by mutations in transthyretin
gene?
(DPG 2011, AI 2005)
(a) Familial Mediterranean fever
(b) Familial amyloidosis polyneuropathy
(c) Dialysis associated amyloidosis
(d) Prion protein associated amyloidosis
189. In Hemodialysis associated amyloidosis, which of the following is
seen:
(AI 2008)
(a) Transthyretin
(b) b2 Microglobulin
(c) SAA
(c) a2 Microglobulin

190. Bone marrow in AL amyloidosis shows:


(a) Bone marrow plasmacytosis
(AI 2007)
(b) Granulomatous reaction
(c) Fibrosis
(d) Giant cell formation
191. A diabetic patient is undergoing dialysis. Aspiration done around
the knee joint would show:
(a) A beta 2 microglobulin
(AI 2007)
(b) AA
(c) AL
(d) Lactoferin
192. What is the best method for confirming amyloidosis?
(a) Colonoscopy
(AI 2007)
(b) Sigmoidoscopy
(c) Rectal biopsy
(d) Tongue biopsy
193. Neointimal hyperplasia causes vascular graft failure as a result of
hypertrophy of:
(AI 2006)
(a) Endothelial cells
(b) Collagen fibers
(c) Smooth muscle cells
(d) Elastic fibers
194. Which one of the following stains is specific for Amyloid?
(AI 2005)
(a) Periodic Acid Schiff (PAS)
(b) Alizarin red
(c) Congo red
(d) Von-Kossa
195. In amyloidosis Beta pleated sheet will be seen in:
(a) X-ray crystallography
(AIIMS Nov 2006)
(b) Electron microscope
(c) Spiral electron microscope
(d) Congo red stain
196. A 50-year-old presented with signs and symptoms of restrictive
heart disease. A right ventricular endo-myocardial biopsy revealed
deposition of extracellular eosinophilic hyaline material. On
transmission electron microscopy, this material is most likely to
reveal the presence of:
(AIIMS May 2006)
(a) Nonbranching filaments of indefinite length
(b) Cross banded fibers with 67 m periodicity
(c) Weibel Palade bodies
(d) Concentric whorls of lamellar structures
197. Amyloid deposits stain positively with all of the following except:
(AIIMS May 2006)
(a) Congo-red
(b) Crystal violet
(c) Methanamine silver
(d) Thioflavin T
198. On electron microscopy amyloid characteristically exhibits:
(a) Beta-pleated sheat
(AIIMS Nov 2005)
(b) Hyaline globules
(c) 7.5-10 nm fibrils
(d) 20-25 nm fibrils
199. Familial amyloidotic polyneuropathy is due to amyloidosis of
nerves caused by deposition of:
(a)Amyloid associated protein
(AIIMS Nov 2002)
(b) Mutant calcitonin
(c) Mutant transthyretin
(d) Normal transthyretin
200. Lardaceous spleen is due to deposition of amyloid in:
(a) Sinusoids of red pulp
(AIIMS Nov 2002)
(b) White pulp
(c) Pencillary artery
(d) Splenic trabeculae
201. What are the stains used for Amyloid?
(PGI Dec 2007)
(a) Thioflavin
(b) Congo red
(c) Eosin
(d) Auramine
(e) Rhodamine
202. Gingival biopsy is useful in the diagnosis of:
(a) Sarcoidosis
(Delhi PG 2010)
(b) Amyloidosis
(c) Histoplasmosis
(d) Scurvy
203. Amyloid is:
(UP 2000)
(a) Mucopolysaccharide
(b) Lipoprotein
(c) Glycoprotein
(d) Intermediate filament

MOST RECENT QUESTIONS

204. Serum amyloid associated protein is found in:


(a) Alzheimer’s disease
(b) Chronic inflammatory states
(c) Chronic renal failure
(d) Malignant hypertension
205. Most common site of biopsy in amyloidosis:
(a) Liver
(b) Spleen
(c) Kidney
(d) Lung
206. Correctly matched pairs in amyloidosis are:
(a) Multiple myeloma - light chain
(b) Chronic inflammation - AA
(c) Cardiac - ATTR
(d) Neural – Beta-2 microglobulin
207. A diabetic patient is undergoing dialysis. Aspiration done around
the knee joint would show:
(a) A-β2 Microglobulin
(b) AA
(c) AL
(d) Lactoferrin
208. Amyloidosis is most commonly seen in:
(a) Maturity onset DM
(b) Type 1 DM
(c) Type 2 DM
(d) Equally seen with all forms of DM
209. Which of the following is the most serious organ involvement in
amyloidosis?
(a)Cardiac tissue
(b) Renal tissue
(c) Splenic tissue
(d) Hepatic tissue
210. Which type of Amyloidosis is caused by mutation of the
transthyretin protein?
(a) Familial Mediterranean fever
(b) Familial amyloidotic polyneuropathy
(c) Dialysis associated amyloidosis
(d) Prion protein associated amyloidosis
211. Cause of death in amyloidosis involving kidney:
(a) Cardiac failure
(b) Renal failure
(c) Sepsis
(d) Liver failure
212. Secondary amyloidosis complicates which of the following:
(a) Pneumonia
(b) Chronic glomerulonephritis
(c) Irritable bowel syndrome
(d) Chronic osteomyelitis
213. On Congo- red staining, amyloid is seen as:
(a) Dark brown color
(b) Blue color
(c) Brilliant pink color
(d) Khaki color
214. Lardaceous spleen is due to deposition of amyloid in:
(a) Sinusoids of red pulp
(b) White pulp
(c) Pencillary artery
(d) Splenic trabeculae
215. Which of the following is the chemical nature of Hemodialysis
associated with amyloid?
(a) AA
(b) AL
(c) Beta – 2-microglobulin
(d) ATTR
216. Familial amyloidosis is seen in:
(a) Alzheimer’s disease
(b) Senile cardiac amyloidosis
(c) Renal amyloidosis
(d) Splenic amyloidosis
217. Excessive accumulation of which hormone protein causes organ
dysfunction:
(a) Growth hormone
(b) Prolactin
(c) Calcitonin
(d) Parathormone
218. Serum amyloid associated protein is increased in:
(a) Alzheimer’s disease
(b) Ankylosing spondylitis
(c) Chronic renal failure
(d) Malignant hypertension
1. Ans. (c) Positive selection during development (Ref:
Immunology by SK Gupta 1st/142-150, Robbins 8th/209)
This appeared to be a tough question but let us analyze all the
options step wise.
• Both B cells and T cells have antigen specific receptors. T cells
have an antigen specific T cell receptor (TCR) composed of a and
b polypeptide chains in 95% cases to bind with the antigen. B cells
have a B cell receptor (BCR) having unique antigen specificity
composed of IgM and IgD on their surface to bind with the antigen.
• Since MHC I is expressed on all the nucleated cells, so, it is likely
to be present on both ‘B’ as well as ‘T’ cells.
• As discussed in our accompanying DVD on Immunology, the T
cells undergo both negative and positive selection. Both these are
described below:
• Positive selection: T cells in the thymic cortex are allowed to
survive only if their T cell receptor has affinity for the MHC
molecule. If the T cells do not have any affinity for the MHC
molecule, they are programmed to die. This is important because
only if this affinity is present, the T cells can interact with the
antigen presenting cells. So, positive selection is required for self
MHC restriction.
• Negative selection: T cells come in the thymic medulla after
being already positively selected in the thymic cortex. In the
medulla, if a T cell has affinity for ‘self antigens’, they are
eliminated. This is called as negative selection. It is therefore
required for the self tolerance.
• Similar to the negative selection of the T cells, the B cells may also
recognize ‘self antigens’ in the bone marrow. In this situation, the B
cell undergoes antigen receptor gene rearrangement so as to
express new antigen receptors. These new receptors are designed
as to not recognize ‘self antigens’. This process is described as
‘receptor editing’. If because of any reason receptor editing does
not take place, the B cells undergo apoptosis. This is the negative
selection of B cells in the bone marrow.
Thus, it can be concluded that both T and B cells undergo negative
selection but only the T cells undergo positive selection.

2. Ans. None (Ref: Robbins 8th/194-5, 9/e 198)


It is recommended to go through the chapter review for the best
understanding of this question. However, I would try to summarize the
important points as follows;
Option ‘a’ and ‘d’….CD4 is present on helper T cell and is required for
antibody production because it is interacts with B cells for causing
activation, conversion into plasma cells and antibody production. The
antibody IgG is required for opsonisation (making the bacteria coated
for preferential killing).
Option ‘b’….Helper T cell subtype 1 is responsible for the secretion of
cytokines like IFN-γ and IL-2 which cause naïve T cells to get converted
into cytotoxic T cells.
Option ‘c’….CD 4 is also important for the following:
• B cell mitogenQ
• Required for isotype switchingQ.
• Affinity maturationQ
• Presence of memory in immune cellsQ
So, the answer is none in this question.
3. Ans. (b) Distal domain alfa 1 and 2
(Ref: Robbins 8th/190-191, 9/e 195 see tex
The antigen binding cleft is made up a1 and a2 chains of MHC I molecule
which is structurally the distal domain of these chains….Kuby
immunology.
4. Ans. (b) Immunoglobulin production
(Ref: Robbins 8th/186-187, 9/e 190-191)
• On antigenic stimulation, the naïve helper T cells get differentiated
into either effector cells or memory cells. The T helper cells can
be of two types (either TH1 or TH2 cells).
• The TH1 cells can secrete cytokines like IL-2 and IFN-γ which
cause activation of macrophages and cause activation of
CD8+T cells into cytotoxic T cells.
• The TH2 cell can secrete IL-4 and IL-5 which cause B cell
proliferation and differentiation into plasma cells which secrete
antibodies or immunoglobulins.
• So, we need to understand that the helper T cells is only helping
in the production of immunoglobulins by plasma cells (they
themselves don’t produce antibodies).
5. Ans. (b) Binds to the beta chain of TCR and MHC class II molecules
of APC stimulating T cell activation
(Ref: Harrison 16th/1920)
6. Ans. (d) CD45RO (Ref: Harrison 17th/2021)
CD 45 is called as Leukocyte common antigen (LCA)Q
Location of cell Molecular marker
All leucocytes CD45 and CD45RB
Medullary thymocytes (‘Naive’ T- cells) CD45 RA and CD45RC
Cortical thymocytes (Memory T-cells) CD45RO

7. Ans. (c) They are MHC restricted cytotoxic cells


(Ref: Harrison’s 17th/2024-2028, 9/e 192)
*NK cells are unique as they are capable of MHC – unrestrictedQ direct cell
lysis which is not mediated by an immune responseQ
8. Ans. (c) Effective against virus infected cells
(Ref: Harrison 17th/2024 – 2028, 9/e 192)
9. Ans. (d) Graft rejection (Ref: Robbins 9/e 231-233)
• CD8+ cytotoxic T lymphocytes (CTLs) recognize cell-bound
antigens only in association with class I MHC molecules, so, CD8+
T cells are said to be class I MHC-restricted.
• CD4+ T cells can recognize antigens only in the context of self-
class II MHC molecules; they are referred to as class II MHC-
restricted.
So, killing of viruses by cytotoxic cells, killing of bacteria by helper cells and T cell
activation in autoimmunity all require MHC molecules for their normal function.
• Talking now about option ‘D’ i.e. graft rejection;
Rejection is a complex process in which both cell-mediated immunity and
circulating antibodies play a role. T cell-mediated graft rejection is
called cellular rejection, and it is induced by two mechanisms:
destruction of graft cells by CD8+ CTLs and delayed hypersensitivity
reactions triggered by activated CD4+ helper cells. Both these as
discussed above would require MHC molecules.
Antibodies evoked against alloantigens in the graft can also mediate
rejection. This process is called humoral rejection. It can be of two
types
1. Hyperacute rejection occurs when preformed antidonor antibodies
are present in the circulation of the recipient
2. In recipients not previously sensitized to transplantation antigens,
exposure to the class I and class II HLA antigens of the donor may
evoke antibodies which are usually formed against graft
vasculature.
So, Humoral graft rejection does not involve T-cells and is NOT MHC restricted.
10. Ans. (a) Mature dendritic cell (Ref: Robbins 9/e 191)
11. Ans. (b) Cells which are not able to express MHC 1
(Ref: Robbins 7th/201, 8th/188, 9/e 192)
• The NK cells express activating and inhibitory receptors. The
functional activity of the NK cells is regulated by a balance
between signals from these receptors. Normal cells are not killed
because inhibitory signals from normal MHC class I molecules
override activating signals. The ability of NK cells to kill target
cells is inversely related to target cell expression of MHC
class I moleculesQ. If virus infection or neoplastic transformation
disturbs or reduces the expression of class I MHC molecules,
inhibitory signals delivered to NK cells are interrupted and lysis
occurs.
12. Ans. (c) Transcription of nuclear factor mediated by NF-κβ which
recruits cytokines (Ref: Robbins 9/e 187)
The Toll-like receptors are membrane proteins that recognize a variety of
microbe-derived molecules and stimulate innate immune responses
against the microbes. These derive there name due to homology to a
Drosophila protein called ‘Toll’. The Toll-like receptors are expressed on
many immune cells of the body. Signaling by Toll-like receptors results
in the activation of transcription factors, notably NF-κB and AP-1.
13. Ans. (a) IL-2 (Ref: Robbins 7th/198, 8th/195, 9/e 198)
T-helper cells can be divided in three distinct types on the basis of different
cytokines they produce.
• T-helper – 1 (TH1) secretes: IL-2 and interferon γ, these cells are important
for type IV hypersensitivity
• T-helper– 2 (TH2) secretes: IL-4, IL-5, these cells are important for type I
hypersensitivity reaction.
• TH 17 Cells: Secrete IL-17, IL-22, these cells provide defense against
extracellular bacteria and fungi.

14. Ans. (a) Apoptosis (Ref: Robbins 9/e 56)


15. Ans. (b) Interleukin IL – 8 (Ref: Robbins 9/e 87)
16. Ans. (b) IgM (Ref: Harrison, 17th/2036)
17. Ans. (b) Endothelial cells (c) Epithelial cells and (d) Langerhan’s
cells (Ref: Robbins Illustrated 9/e 195)
As already discussed in the text also, the antigen presenting cells include
macrophages, Dendritic Cells (found in lymphoid organs) and
Langerhans’ cells (found in epidermis). The question ideally should
have been …with ‘all except’
18. Ans. (a) Langerhan’s cell; (b) Macrophages; (e) B-lymphocyte
(Ref: Robbins 7th/197, 9/e 195)
Antigen presenting cells are: B-cell, Langerhan’s cell in skin, Macrophages
19. Ans. (a) Cytotoxic T cells: (Ref: Robbins 9/e p210)
• Perforins are hole forming proteins synthesized by cytotoxic T-cells.
They can perforate the plasma membrane of the target cells that
are under attack by CD8+ lymphocytes. Granzymes are delivered
into the target cells through these holes formed by perforins. In
addition the perforin pores allow water to enter the cells, thus
causing osmotic lysis.
20. Ans. (a) B7 and CD28; (Ref:
Robbins 7th/225, Harrison 16th/1907, 9/e p213)
Immunological tolerance in peripheral lymphoid organs is called as peripheral
tolerance. It requires signal to (interaction between CD28 and B7).
CD28 molecules (co-stimulatory molecule) bind to their ligands-CD80
(B7-l) and/or CD86 (B7-2) and activates T-cell. If the antigen presented
by cell do not bear CD28 ligand, a negative signal is delivered and cell
become tolerant and anergic.
21. Ans. (c) CD19; (d) CD20; (e) CD22
(Ref: Immunology by Roitt, 6th/29, 30, 19 9/e p191)
CD19, 20 and 22 are main markers of human B cells. Other B cell markers are
CD72 to CD78.
CD33 is present in monocyte whereas CD34 is marker of hematopoetic stem cell.

22. Ans. (d) All of the above (Ref: Robbins 9/e p87)
23. Ans. (a) Langerhan’s cells (Ref: Robbins 9/e p192)
24. Ans. (b) Helps in the formation of antibody
(Ref: Robbins 8th/183-184; 7th/82 , 9/e p191)
25. Ans. (b) 2 : 1 (Ref: Robbins 9/e p190-191)
26. Ans. (c) T cells (Ref: Robbins 8th/192, 9/e p191)
27. Ans. (b) T cell (Ref: Robbins 7th/670 , 9/e p191)
28. Ans (c) Memory is seen
(Ref: Robbins 8th/184, 9/e p186-188) ...see text
29. Ans. (d) Toll-like receptor (Ref: Robbins 9/e p187-188)
Toll-like receptors (TLRs), stimulate one of the immune responses directed against
microbes,
TLRs bind to pathogen-associated molecular patterns (PAMPs), which are
small molecular sequences found commonly on pathogens.
Examples of PAMPs include bacterial lipopolysaccharide (LPS), lipoteichoic acid,
and peptidoglycan.
LPS is probably the prototypical PAMPQ.
TLRs, in conjunction with CD14, bind to LPS (endotoxin), and activate leukocytes
to produce cytokines and reactive leukocytes to produce cytokines and reactive
oxygen intermediates (ROIs).

30. Ans. (c) NK cells (Ref: Robbins 8/e p188, 9/e p192)
• The NK cells are also known as large granular lymphocytes as they
have a larger size and contain abundant azurophilic granules.
• NK cells are endowed with the ability to kill a variety of infected
and tumor cells, without prior exposure to or activation by these
microbes or tumors.
31. Ans. (b) CD 16, CD 56…explained earlier
(Ref: Robbins 9/e p192)
32. Ans. (a) 16 (Ref: Robbins 8/e p188, 9/e p192)
• Natural Killer cell is identified with the molecules as CD16 and
CD56 Q.
• CD16 is an Fc receptor for IgG, and it confers on NK cells the
ability to lyse IgG-coated target cells. This phenomenon is known
asantibody-dependent cell-mediated cytotoxicity (ADCC)Q.
33. Ans. (a) IL-2 (Ref: Robbins 8/e p195, 9/e p198)

After coming in contact with antigen presenting cells, CD4+ helper T


cells secrete IL-2 and expresses high-affinity receptors for IL-2. IL-2 is a
growth factor that acts on these T lymphocytes and stimulates their
proliferation, leading to an increase in the number of antigen-
specific lymphocytes.
34. Ans. (a) Mature dendritic cells (Ref: Robbins 9/e p191)
Direct quote.. “interdigitating dendritic cells, or just dendritic cells are the
most important antigen-presenting cells (APCs) for initiating primary T-
cell responses against protein antigens’
35. Ans. (a) T-cells (Ref: Robbins 8/e p186, 9/e p191)
36. Ans. (c) Liver…refer to text for details
(Ref: Robbins 8/e p834, 7/e p79, 9/e p102)
37. Ans. (b) Langerhans cell (Ref: Robbins 9/e p622)
38. Ans. (a) B cells (Ref: Robbins 9/e p191)
Macroglobulin is the other name for antibodies. So, the answer
becomes obvious. i.e. B cells. The activated B cells are called as
plasma cells and are responsible for secretion of antibodies.
39. Ans. (d) May cross the placenta and fix complement
IgG (and not IgE) is the antibody which may cross the placental barrier and fix
complement.

40. Ans. (d) IgE... see text table for details


41. Ans. (b) Cortical follicles (Ref: Robbins 9/e p191)
42. Ans. (d) Clonal selection (Ref: Robbins 9/e p190)
43. Ans. (b) B-cell (Ref: Robbins 9/e p192)
44. Ans. (b) Tumor necrosis factor
Ref: Ananthanarayan 6th/121, Harrison 17th/2047, 9/e p194-195)
*HLA class III contains genes for

• Complement components C2 and C4Q of classical pathway (Not C3Q)


• Properdin factor BQ of alternate pathway
• Tumor necrosis factorQ: Alpha and Beta
• Heat shock protein 70 Q
• Enzyme tyrosine hydroxylaseQ
• Genes for MHC (also known as HLA) are located on short arm of chromosome
6Q.

45. Ans. (b) Governing susceptibility to autoimmune diseases (Ref:


Ananthanarayan 6th/108, Roilt’s Essential Immunology – 262,
Robbins 9/e p215)
*HLA class III region contains genes for early complement components C2
and C4 of classical pathway.
*Deficiency of these early components of the classical pathway viz C1, C2
and C4 is associated with autoimmune diseases like SLE and other
collagen vascular diseases.
• These genes are thus, important in regulating susceptibility to
autoimmune disease
• Class III genes do not participate in MHC restriction or graft
rejection components.
46. Ans. (c) Short arm of chromosome 6
(Ref: Robbins 8th/190, 7th/203, 9/e p194)
47. Ans. (a) Ankylosing spondylitis
(Ref: Robbins 8th/193, Harrison 17th/2051, 9/e p215)
48. Ans. (b) MHC class II antigen
(Ref: Harrison 16th/1933, 17th/2047, 9/e p194)
• The MHC class II region was originally termed the D-region. The
allelic gene products were first detected by their ability to stimulate
lymphocyte proliferation by mixed lymphocyte reaction. So,
mixed lymphocyte culture is used to identify HLA II. It is present on
all antigen presenting cells (B cells, dendritic cells and
macrophages) and can be induced on endothelial cells and
fibroblasts.
49. Ans. (a) Disputed paternity, (c) Organ transplant

(Ref: Robbins, 9/e p195, 215)


Uses of MHC/HLA typing
Anthropology Transfusion
Paternity Testing Forensic science
Transplantation Disease Correlation

50. Ans. (a) Not involved in innate immunity; (c) Present in nucleated
cells; (d) Present in B-cells
(Ref: Robbins 7th/203, 9/eP194-195)
Class II MHC Proteins are glycoprotein present on the surface of certain
cells including macrophages, B-lymphocytes, dendritic cells of the
spleen and Langerhan’s cells of the skin.
Endothelial cells and fibroblasts can be induced to express Class II
MHC by IFN-g.
51. Ans. (b) T cells, (d) Platelets; (e) RBCs
(Ref: Ananthanarayan’ 7th/130, Robbins 7th/203, 9/e p194)
52. Ans. (a) Transplantation reaction; (b) Autoimmune disease; (d)
Involved in T-cell function
(Ref: Harrison 16th-1930, 1934; Robbins 7th-204-205)
The principal physiologic function of Major histocompatibility complex (MHC)
is to bind peptide fragments of foreign proteins for presentation to
appropriate antigen specific T-cells. Thus MHC is involved in
transplantation reaction, disease susceptibility (i.e. autoimmune
disease, inflammatory disease, infections, etc.), immune response and
tolerance.
53. Ans. (a) Alpha helices and (c) Alpha and beta-1 chain
(Ref: Robbins 7th/203-204, 9/e p195)
Friends, the examiner should have specified the type of MHC molecule so
that question becomes clear.
a1 and a2 domains form a cleft/groove where the peptides bind to MHC I molecule.
The antigen binding cleft in MHC II is formed by an interaction of a1 and b1
domains of both chains.

54. Ans. (c) Rheumatoid arthritis (Ref: Robbins 9/e p215)


HLA B27 is associated with Seronegative spondyloarthropathies. Please
revise the following important features about these.
Seronegative spondyloarthropathies Salient features of these
(Mnemonic: PAIR) diseases
P: Psoriatic arthritis * Absence of serum auto-
A: Ankylosing spondylitis (AS) antibodies
I: Inflammatory bowel disease arthritis * Associated with HLA B27 (MC
R: Reactive arthritis (Reiter syndrome) AS)
* Onset before the age of 40 years
* Presence of uveitis, spine/large
peripheral joint arthritis

55. Ans. (c) Present antigens for recognition by T cell antigen


receptors (Ref: Robbin 8/e p191)
The physiologic function of MHC molecules is to display peptide fragments of
proteins for recognition by antigen-specific T
cells.... (Ref: Robbin 8/e p190)
Also now that:
• Class I MHC molecules are required to display antigens to CD8 T cells
• Class II MHC molecules are required to display antigens to CD4 T cells.

56. Ans. (c) RBCs (Ref: Robbins 8/e p190, 9/e p194)
• Class I MHC molecules are expressed on all nucleated cellsQ
and plateletsQ
57. Ans. (d) All blood cells except erythrocytes
(Ref: Robbins 9/e p194)
58. Ans. (b) CD4 cell (Ref: Robbins 9/e p195)
59. Ans. (a) Ankylosing spondylitis (Ref: Robbins 9th/205)
Direct quote… “Approximately 90% of patients are HLA-B27 positive;
associations have also been found with the IL-23 receptor gene”.
Ankylosing spondylitis (also rheumatoid spondylitis and Marie-
Strümpell disease)
It causes destruction of articular cartilage and bony ankylosis, especially of
the sacroiliac and apophyseal joints (between tuberosities and
processes).
It becomes symptomatic in the 2nd and 3rd decades of life as lower back
pain and spinal immobility.
60. Ans. (c) Antigen presentation to T cells
(Ref: Robbins 9th/194)
The function of MHC molecules is to display peptide fragments of
protein antigens for recognition by antigen specific T cells.
61. Ans. (a) M-cells (Ref: Atlas of Immunology 3rd/ 206)
Types of Antigen Presenting Cells
Professional APC Non-professional APC
Have high expression of MHC II Expression of MHC II molecule can be
molecule physiologically induced by cytokines like IFN-gamma under
stress
• Dendritic cells • Thymic epithelial cells
• B cells • Fibroblasts
• Macrophages • Glial cells
• Endothelial cells
• Pancreatic beta cells

M cells are specialized epithelial cells of the mucosa-associated lymphoid


tissues. They transport antigens from the lumen to cells of the immune
system, thereby initiating an immune response or tolerance.
62. Ans. (b) Medullary macrophages
The Elements of Immunology/165
• Cortical macrophages, epithelial cells and dendritic cells express high levels of
MHCII
• Medullary macrophages express only MHCI molecule.
• Medullary epithelial cells and macrophages express both MHCI and MHC II
molecules

63. Ans. (d) Psoriasis vulgaris (Ref: Robbins 9/e p1165)


64. Ans. (b) Graft versus host disease
(Ref: Robbins 9/e p195)
65. Ans. (d) HLA-B8 (Ref: Robbins 9/e p1235)
66. Ans. (c) HLA B27 (Ref: Robbins 9/e p1165, internet)

• Psoriasis is associated with HLA-Cw*0602 allele.


• Pustular psoriasis and Psoriatic spondylitis are associated with HLA- B27

67. Ans. (b) HLA DR4 (Ref: Robbins 9/e p1210, Harrison 19/e p2139,
Rheumatology Secrets/35)
Genes associated with rheumatoid arthritis.
• HLA-DRB1 gene
• HLA-DR 4
• PTPN22 gene

68. Ans. (b) Type 2 hypersensitivity reaction


(Ref: Robbins 8th/203, 9/e p206)
Myasthenia gravis is a type 2 hypersensitivity reaction. Other important
examples can be remembered from the mnemonic “My blood group is
R h positive”. For details see text.
69. Ans. (b) Type 2 hypersensitivity reaction
(Ref: Robbins 7th/211, 9/e p206)
70. Ans. (b) Immune complexes (Ref: Internet, 9/e p207)
A Raji cell assay identifies the presence of circulating immune complexes. A
positive result suggests the presence of antigen-nonspecific immune
complexes in the circulation. The raji cell assay may be helpful in
differentiating diseases. Additionally, raji cell tests may assist with the
assessment of disease activity. A positive raji cell assay that turns
negative may suggest that the disease activity has improved.
71. Ans. (c) Immune complex mediated hypersensitivity
(Ref: Robbins 8th/703, 9/e p207)
Hypersensitivity pneumonitis (allergic alveolitis) is ideally an example of
type III and type IV hypersensitivity. Complement and immunoglobulins
demonstrated within vessel walls by immunofluorescence as well as
presence of specific antibodies in the serum of affected patients
indicate type III (immune complex) hypersensitivity. The presence of
non-caseating granulomas in 2/3rd patients suggest the development of
a T cell-mediated (type IV) delayed-type hypersensitivity against the
implicated antigen(s).
However, the single best answer to be marked would be type III
hypersensitivity reaction because immune complex formation plays a
relatively more important role in hypersensitivity pneumonitis.
72. Ans. (a) IgE (Ref: Robbins 9/e p202)
73. Ans. (a) Localized immune complex
(Ref: Robbins 8th/205, 7th/215, 9/e p207)
The Arthus reaction is a localized area of tissue necrosis resulting from acute
immune complex vasculitis, usually elicited in the skin.
Revise the mnemonics “SHARP“ from the text.
74. Ans. (d) Type IV (Ref: Robbins 9/e p210)
• Presence of epitheloid cell granuloma, langhans giant cells and
lymphocytes is characteristic of chronic granulomatous
inflammation, which is associated with type IV hypersensitivity
action.
75. Ans. (b) IgE mediated reaction (Ref: Robbins 9/e p202)
• The symptoms of the patient are due to hypersensitivity type I
reaction Type I is mediated by IgE and it flairs up within minutes.
• The symptoms range form rashes to anaphylactic shock with
vasodilation hypotension and bronchiolar spasm.
Type I Type II
• The symptoms range form rashes to • Is characterized by an antigen antibody
anaphylactic shock with vasodilation reaction on the surface of a host cell*
hypotension and bronchiolar spasm • Mediated by IgG or IgM
• Mediated by IgE Examples of Type II
Examples of Type I • Blood transfusion reactions*
• Eczema* • Transplant rejection*
• Hay Fever* • Autoimmune hemolytic anemia*
• Asthma* • Good Pasture’s syndrome*
• Anaphylactic shock* • Graves disease*
• Urticaria* • Myasthenia gravis*
• Acute dermatitis* • Pemphigus vulgaris
• Theobald Smith Reaction* • Pernicious anemia*
• Rheumatic fever*

Type III Type IV


• Mediated by antigen/antibody complex* • Cell mediated reaction (delayed
Examples of type III hypersensitivity)*
• S: Serum sickness, Post - streptococcal Examples of type IV
glomerulonephritis, SLE, Schick test • Tuberculosis
• H: HSP • Sarcoidosis*
• A: Arthus reaction, Acute vira l hepatitis • Temporal arteritis
• R: Reactive arthritis • Contact dermatitis*
• P: Penicillamine toxicity, • Lepromin test and PPD (Mantoux
• Polyarteritis nodosa (PAN) test)*
• Patch test*
• Type I DM

76. Ans. (b) Contact hypersensitivity (Ref: Robbins 9/e p209)


77. Ans. (a) Blood transfusion reaction
(Ref: Robbins 9/e p206)
78. Ans. (d) Necrotizing vasculitis
(Ref: Harrison’s 16th/327, 328, Robbins 9/e p207)
• Acute necrotizing vasculitis is the dominant morphological
consequences of immune complex injury [Type-III hypersensitivity
reaction]. The immune complexes incite an activation of
complement and produce inflammatory reaction and necrosis.
• Atopic dermatitis and urticaria –Type I.

• Photoallergy- type IV hypersensitivity or delayed hypersensitivity

• Graft versus host disease is mediated by T-cells.

79. Ans. (b) Lymphocytes are the primary cells affected by M.


tuberculosis (Ref: Robbins 8th/368, 9/e p371)
• Macrophages are the primary cells infected by M. tuberculosis.
80. Ans. (a) IgE mediated response (Ref: Robbins 9/e p202)
81. Ans. (a) Hard sore (Ref: Robbins 7th/738, 9/e p693)
• Granulomas found in sarcoidosis are non-caseating and so, refered
to as “Hard sore.” They contain.
– Asteroid Bodies
– Schaumann bodies and
– Birefringent crystals
82. Ans. (d) All of the above
(Ref: Robbins 7th/1344, 9/e p1235-1236; Harrison’s 16th/2521 table 366-
3)
Myasthenia gravis: revision of key points
Autoimmune mediated neuromuscular disease example of type II
hypersensitivity reaction
Distinct finding: ↓ ACh receptors (in muscles) and circulating antibodies to ACh
receptors
Associations
1. Hyperthyroidism
2. Thymic hyperplasia – 65%
3. Thymoma – 15%
4. Autoimmune disorders (Hashimoto’s thyroiditis, Graves’ disease, Rheumatoid
arthritis/SLE, positive family history of autoimmune diseases)

83. Ans. (b) Cytotoxic type (Ref: Robbins 9/e 205)


84. Ans. (c) Type III (Ref: Robbins 9/e 207)
85. Ans. (c) Good cell mediated immunity
(Ref: Robbins 8th/207, 7 th/381, 9/e 210)
86. Ans. (d) Type IV (Ref: Robbins 8/e p197, 9/e 24)
The hypersensitivity reactions have been given the following names:
• Immediateor (type I) hypersensitivity
• Antibody-mediated or (type II) hypersensitivity
• Immune complex–mediated or (type III) hypersensitivity
• Cell-mediated or (type IV) hypersensitivity
87. Ans. (b) Acetylcholine receptors (Ref: Robbins 9/e 195)
88. Ans. (c) Thymic hyperplasia
(Ref: Robbins 9/e 1235-1236)
Direct quote... “Thymic hyperplasia is found in 65% and thymoma in 15%
of affected patients”.
Myasthenia gravis: revision of key points for NEET/AIIMS!
• Autoimmune mediated neuromuscular disease example of type II
hypersensitivity reaction
• When arising before age 40 years it is most commonly seen in women, but it
occurs equally in both sexes in older patients.
• Distinct finding: ↓ ACh receptors (in muscles) and circulating antibodies to ACh
receptors
• Most sensitive test: single fibre electromyography
• Most specific test: antibodies to ACh receptorsQ
• Electrophysiological studies: ↓ in motor responseQ on repeated stimulation
• Nerve conduction studies: NormalQ
• Treatment is done with drugs (neostigmine with atropineQ) and thymectomyQ

89. Ans. (b) Type II (Ref: Robbins 9th/205)


Administration of penicillin causing no symptoms in 48 hours with no previous
history of allergy rules out type I hypersensitivity reaction.
The patient presented with hemolysis which can be because of antibody
formation against red cells. The formation of autoantibody is a feature
associated with type II hypersensitivity reaction. Thus, it becomes the
answer over here.
Clinically, antibody-mediated cell destruction and phagocytosis occur in
multiple situations:
• Transfusion reactions, in which cells from an incompatible donor
react with and are opsonized by preformed antibody in the host
• Hemolytic disease of the newborn: antigenic difference between
the mother and fetus
• Autoimmune hemolytic anemia, agranulocytosis, and
thrombocytopenia, in which individuals produce antibodies to their
own blood cells, which are then destroyed.
• Certain drug reactions, in which a drug acts as a “hapten” by
attaching to plasma membrane proteins of red cells and antibodies
are produced against the drug-protein complex.
90. Ans. (a) Mast cells (Ref: Robbins 9th/201)
Immeiate, or type I, hypersensitivity is a rapid immunologic reaction occurring
in a previously sensitized individual that is triggered by the binding of an
antigen to IgE antibody on the surface of mast cells.
91. Ans. (b) Hypersensitivity reaction with modified macrophages,
lymphocytes and giant cells
(Ref: Robbins 9th/372)
Presence of a history of fever, night sweats and weight loss with apical lesion
having caseous necrosis is a pointer to the presence of tuberculosis in
the patient. This is characterized by the stimulation of the macrophages
to kill mycobacteria, the TH1 response leading to the formation of
granulomas and caseous necrosis. The formation of granuloma is a
feature of type IV hypersensitivity reaction.
92. Ans. (c) Type 3
hypersensitivity reaction
(Robbins 9th/ 207)
Serum sickness is a type of immune complex disease or type 3
hypersensitivity reaction.
93. Ans. (b) Can lead to leukocytoclastic vasculitis
(Ref: Robbins 9/e p207-8)
• Acute serum sickness is the prototype of a systemic immune complex disease;
it was once a frequent sequela to the administration of large amounts of foreign
(heterologous antigen) serum.
• It is a type III hypersensitivity reaction.
• It may lead to hypocomplementemia.
• Serum sickness may lead to leukocytoclastic vasculitis.

94. Ans. (d) Necrotic zone (Ref: Robbins 9/e p375)


95. Ans. (a) Preformed antibodies (Ref: Robbins 9/e 233-234)
Direct quote from Robbins. ‘Hyperacute rejection occurs when preformed
antidonor antibodies are present in the circulation of the recipient’.
Such antibodies may be present:
• In a recipient who has previously rejected a kidney transplant
• Multiparous women who develop anti-HLA antibodies against
paternal antigens shed from the fetus may have preformed
antibodies to grafts taken from their husbands or children
• Prior blood transfusions
• In recipients not previously sensitized to transplantation antigens,
exposure to the class I and class II HLA antigens of the donor graft
may evoke antibodies. The initial target of these antibodies in
rejection seems to be the graft vasculature. Thus, antibody-
dependent acute humoral rejection is usually manifested by a
vasculitis, sometimes referred to as rejection vasculitis
Also know that endothelitis is caused by injury to the vascular
endothelial cells mediated by CD8+ T cells. This is a component of
acute cellular rejection.
96. Ans. (d) Lung (Ref: Robbin 9/e 236, Harrison’s 17th/717)
• GVHD affects skin (earliest organ), intestine and liver
• Lungs are not affected in GVHD. For details see text.
97. Ans. (a) Hyperacute rejection (Ref: Robbins 9/e 233-234)
Hyperacute rejection takes place when there are preformed antibodies in the
circulation of the recipient. It can be due to:
• Patient who has already rejected a transplant
• Multiparous females
• Prior blood transfusions

98. Ans. (a) Associated with solid organ transplantation; (b) Graft
must contains immunocompetant T cell; (c) It is seen in
immunosuppressed persons; (d) Also called as Runt disease in
animals (Ref: Robbins 7th/222, 9/e 232-233;
Harrison 16th/670; Ananthanarayan 7th/180)
Graft versus host reaction (GVH) occurs in any situation in which
immunologically competent cells or there precursors are
transplanted into immunologically crippled recipient cells and the
transferred cells recognize alloantigens in the host.
GVHD occurs most commonly in allogenic bone marrow
transplantationQ but may also follow transplantation of solid organs
rich in lymphoid cells.
99. Ans. (b) Interstitial and tubular mononuclear cell infiltrate
(Ref: Robbins 8th/228-229 , 9/e 232-233)
100. Ans. (d) Xenograft (Ref: Harsh Mohan 6th/65)
• Isograft: Is a graft from a different individual genetically identical with recipient
e.g. identical twin
• Autograft: Is to self
• Allograft: Graft from same species but different genotype (from one human to
another human)
• Xenograft: Graft from different species (from animal to human)

101. Ans. (b) Adrenal (Ref: Robbins 9/e 236, 8th/230; 7th/125)
102. Ans. (d) Infection (Ref Cambell’s Urology, 8/e p346,349)
Principal causes of death in renal transplant patients (in decreasing
order): Heart disease, Infection, Stroke
103. Ans. (a) Hyperacute rejection (Ref: Robbins 9/e 233-234)
Hyperacute rejection Acute rejection Chronic rejection

*Takes place in individuals with *Seen days to months after *Occurs months to years
preformed antibodiesQ transplantation. It can be after transplantation
usually within minutes to acute humoral rejection or
hours of transplantation acute cellular rejection

104. Ans. (d) C4d (Ref: Robbins 9th/ 234)


Acute antibody mediated reaction is manifested by lesions consisting
of inflammation in the glomeruli and peritubular capillaries associated
with deposition of the complement breakdown product C4d which is
produced by the activation of the complement dependent classical
pathway.
105. Ans. (d) All of the above Ref: cancerresearch.uk.org
The following are the steps taken to reduce the risk of graft versus host
disease:
1. Getting the best donor match: donor is as closely matched to recipient
with the technique of tissue typing.
2. Use of immunosuppressive drugs like ciclosporin, tacrolimus etc
3. T cell depletion: Removal of T cells from the donor’s bone marrow is
called as T cell depletion.
4. Post procedure immune suppression using the drugs like methotrexate
and cyclophosphamide.
106. Ans. (b) Stem cells are collected directly from the bone
marrow (Ref: Harrison 19/chapter 139, Robbins 9/e p27-8)
• Autologous transplantation involves the removal and storage of the patient’s
own stem cells with subsequent reinfusion after the patient receives high-dose
myeloablative therapy.
• Unlike allogeneic transplantation, there is no risk of GVHD or graft rejection
with autologous transplantation.
• Donors are typically treated with 4 or 5 days of hematopoietic growth
factor, following which stem cells are collected in 4-hour pheresis sessions.
107. Ans. (a) 10 (Ref: Immunology & Serology in Laboratory
Medicine/464)
HLA markers like HLA-A/ HLA-B/ HLA-C and HLA-DRB1 are the most
important markers to ensure the success of the transplantation. Some
centres also use HLA-DQ as an additional marker which is
minimally/less significant than first 4 markers. Since we have 2 alleles
for each of them, so, the total HLA score is 10.
• For adult donors, 6 out of 8 HLA markers (HLA-A/ HLA-B/ HLA-C and HLA-
DRB1) should match.
• For cord blood units, the criteria are less stringent and match of 4 out of 6
(HLA-A/ HLA-B and HLA-DRB1) is required.

108. Ans. (b) Acute rejection


(Ref: Fundamentals of Surgical Practice; 258)
Acute rejection is mediated by T cells and is the commonest type of
organ rejection seen in tissue mismatch after an allotransplant, or
when insufficient immuno-supression is employed.
109. Ans. (b) Liver (Ref: Robbins 9/e p236)
The rejection reaction against liver transplants is not as vigorous as
might be expected from the degree of HLA disparity.
110. Ans. (d) Graft versus host disease
(Ref: Robbins 9/e p236, Blood chapter 60)
Runt disease is a graft versus host disease which occurs when
immunologically competent cells or their precursors are transplanted
into immunologically crippled recipients, and the transferred cells
recognize alloantigens in the host and attack host tissues.
111. Ans. (b) Hyperacute rejection (Ref: Robbins 9/e p???)
Hyperacute rejection is caused by preformed antibodies against the
donor antigens. So, cross matching prior to transplantation is
helpful in its prevention.
112. Ans. (b) Occurs when donor cells are immunocompetent
(Ref: Robbins 9/e p236)
GVHD occurs when immunologically competent cells or their
precursors are transplanted into immunologically crippled
recipients, and the transferred cells recognize alloantigens in the host
and attack host tissues. It is seen most commonly in hematopoetic
stem cell transplantation.
113. Ans. (a) Tissue typing (Ref: Immunology/290)
• Tissue typing is the technique in which the tissues of a prospective
donor and recipient are tested for compatibility before carrying out
transplantation.
• The two commonly used techniques are: "mixed leukocyte
reaction" and micro-cytotoxicity assay.
114. Ans. (c) 3 months
(Ref: Robbins 9/e p???)
• Hyperacute rejection: Within minutes to hours (due to preformed antibodies)
• Acute rejection: Within days to months (due to cell mediated and antibody
mechanisms)
• Chronic rejection: Within months to years (due to T cell mediated mechanism)

115. Ans. (b) Polyclonal B cell activation


(Ref: Robbins 8th 212, 9/e 216)
Polyclonal B cell activation is caused by EBVQ and HIVQ resulting in
production of autoantibodies for other mechanisms see text
116. Ans. (b) Scleroderma
(Ref: Robbins 8th/225, 518, Harrison 18th/2096, 9/e 228-229)
The symptoms present in this girl are suggestive of Raynaud’s phenomenon
(pallor and cyanosis of the digits of hands and feet due to exaggerated
vasoconstriction of digital arteries and arterioles). It can either be:
• Primary Raynaud’s phenomenon or
• Secondary Raynaud’s phenomenon (due to SLE, scleroderma,
Buerger’s disease, atherosclerosis). Since, Raynaud’s
phenomenon may the first manifestation of these diseases, the
patient with new symptoms need to be evaluated.
• Direct quote Robbins 8th/225.... ‘though systemic sclerosis shares
many features with SLE, rheumatoid arthritis and polymyositis, its
distinctive features are the striking cutaneous changes,
notably skin thickening. Raynaud phenomenon, manifested as
episodic vasoconstriction of the arteries and arterioles of the
extremities, is seen in virtually all patients and precedes other
symptoms in 70% of cases.’ Dysphagia is seen in 50% patients.
Ruling out SLE, the presentation in SLE is….‘Typically, the patient is a young
woman with some of the following features: a butterfly rash over the
face, fever, pain but no deformity in one or more peripheral joints (feet,
ankles, knees, hips, fingers, wrists, elbows, shoulders), pleuritic chest
pain, and photosensitivity.’
117. Ans. (d) Sickle Cell Disease
(Ref: Harrison 17th/2074)
Sickle cell disease is caused by a point mutation in the b6 chain of
hemoglobin. It is not an auto immune disease.
118. Ans. (d) Anti glycoprotein antibody

(Ref: Robbins 8th/215, Harrison 17th/2073, 9/e 219)


Antiphospholipid antibody syndrome is characterized by antibodies
against phospholipid beta– 2-glycoprotein 1 complexQ. For detail,
see text under SLE.
119. Ans. (b) Kikuchi Fujimoto disease
(Ref: Harrison 17th/1011)
The other name of Kikuchi Fujimoto disease is histiocytic necrotizing
lymphadenitis. This will answer our question.
Kikuchi-Fujimoto disease (KFD)/Histiocytic necrotizing lymphadenitis
• Benign and self-limited disorder in young individuals characterized by regional
cervical lymphadenopathy with tenderness, usually accompanied with mild
fever and night sweats.
• May be viral in etiology
• Diagnosed on the basis of an excisional biopsy of affected lymph nodes which
shows fragmentation, necrosis and karyorrhexis, presenting with posterior
cervical lymphadenopathy.
• Patients should be followed-up because of increased chances of development
of SLE (systemic lupus erythematosus).

Castleman disease (CD)


• Defined by lymph node hypertrophy with angiofollicular lymphoid hyperplasia.
• Has localised form or multicentric form (several Lymph nodes are affected).
• Clinical features inlcude Peripheral lymphadenopathy, hepatomegaly and/or
splenomegaly and POEMS syndrome (polyneuropathy, organomegaly,
endocrinopathy, monoclonal gammopathy, skin changes)
• Human herpes virus 8 (HHV-8) is the etiological agent
• Lymph node histological analysis with immunohistochemical staining, shows
polyclonal angiofollicular lymphoid hyperplasia, most often of the
hyalinovascular type (especially in localised CD) and more rarely of the
plasma-cell type (particularly in multicentric CD) or mixed/intermediate type.

120. Ans. (a) SLE


(Ref: Harrison 17th/2077, Robbins 8th/218, 9/e 224)
Subendothelial deposits create a homogeneous thickening of the capillary
wall called wire loop lesion, which can be seen by means of light
microscopy when they are extensive. They usually reflect active
disease.
121. Ans. (d) Liver
(Ref: Immunofluorescence Methods for Microscopic Analysis in
Methods in Nonradioactive Detection, Lange Publications/247)
Quote from the book…. “Serum anti-nuclear antibodies (ANA) bind to the
corresponding antigens present in rat liver sections. The antigen-antibody
complexes are detected by means of a fluorescein labeled anti-human
immunoglobulin, and visualized with the aid of a fluorescence microscope”.

122. Ans. (c) Inclusion bodies (Ref: Robbins 9/e 250-255)


123. Ans. (a) C1 esterase inhibitor deficiency
(Ref: Robbins 8th/235, 9/e 238)
124. Ans. (a) It is an autosomal recessive disorder
(Ref: Robbins 9/p242)
125. Ans. (a) SLE (Ref: Robbins 9/e 218)
126. Ans. (c) Diffuse proliferative glomerulonephritis (WHO class IV)
(Ref: Harrison 17th/2077, Robbins illustrated 8th/218, 9/e p224)
127. Ans. (d) Multiple myeloma (Ref: Robbins 9/e p258-259, 8th/252-
254, 7th/261)
Pink and amorphous material that shows apple green birefringence under
polarized light confirms the diagnosis of Amyloidosis. The presence of
Lambda light chains is suggestive of multiple myeloma.
128. Ans. (a) Non caseating granulomas
(Ref: Robbins 9/e p693, Harrison, 17th/2135)
• The presence of bilateral nodules on the shin; bilateral hilar
lymphadenopathy and negative Mantoux test in a female
patient point to a probable diagnosis of Sarcoidosis.
• The skin lesions characteristically show the presence of non –
caseating granulomasQ in sarcoidosis.
129. Ans. (a) SLE: (Ref: Robbins 9/e p218)
Antibodies to ds DNA and the so called Smith (Sm) antigens are virtually
diagnostic of SLE. Anti ds-DNA is common in SLE (40-60%).
Anti nuclear antibody is present in all the mentioned diseases but anti double
stranded DNA is very specific for SLE.
130. Ans. (a) PSGN; (b) MPGN; (e) Infective endocarditis
(Ref: Harrison 16th-/680)
Causes of hypocomplementemia
• Glomerulonephritis
– Idiopathic proliferative GN
– Cresenteric GN
– MPGN
– Post-infectious GN
• Lupus nephritis
• Cryoglobulinemia
• Bacterial endocarditis
• Shunt nephritis
• Atheroembolic renal disease
• Sepsis
• Acute pancreatitis
• Advanced liver disease
131. Ans. (b) Neutropenia; (c) Defective microbial killing; (d) Presence
of large granules in neutrophils; (e) Immunodeficiency:
(Ref: Harrison’ 16th/353, 354, Robbins 9/e p238)
Chediak-Higashi syndrome
• Autosomal recessive inheritance
• Due to defect in lysosomal transport protein LYST.
• Clinical features include: primary immune deficiency, neutropenia, defective
microbial killing, impaired chemotaxis, hypopigmentation of skin, eyes and
hair, photophobia and nystagmus
• Microscopic examination shows giant peroxidase positive inclusions in the
cytoplasm of leukocytes.

132. Ans. (a) Severe combined immunodeficiency.


(Ref: Robbins 7th/244, 9/e p239)
133. Ans. (b) Pulmonary emphysema; (c) Diastase resistant hepatic
cells: (Ref: Robbins 9/e p850-851)
• a1-anti-trypsin deficiency is an autosomal recessive disease having
abnormally low levels of a1-anti-trypsin
• Deficiency of the enzyme leads to pulmonary panacinar emphysema
• Gene located on Chr 14.
• Characterized by PAS positive and diastase resistant inclusions in
hepatocytes.

134. Ans. (d) High serum IgE, with low IgG, IgA and IgM (Ref: Robbins
9/e p242, Harrison 17th/384, 2061, 2056, 381)
Hyper IgE syndrome is also known as Job’s syndrome
Abnormal chemotaxis is a variable feature.
Patients have characteristic facies with broad nose, kyphoscoliosis,
osteoporosis and eczema.
Recurrent abscesses (known as cold abscesses) involving skin, lungs and other
organs is a prominent feature
Serum IgE level is significantly elevated whereas IgM, IgG and IgA level are
normal.

Note: In Hyper- IgM syndrome, IgM is elevated and IgG, IgA are normal.

135. Ans. (c) Large size platelet


(Ref: Robbins 9e/p242, 8th/235, Harrison 17th/2060, OP Ghai pediatrics
6th/326)
136. Ans. (a) Absent tonsils and no palpable lymph nodes on physical
examination (Ref: Robbins 9/e p240-241)
137. Ans. (d) Neutrophil (Ref: Robbins 8th/238, 9/e p248)
138. Ans. (d) Leucocytosis (Ref: Robbins 9/e p218)
139. Ans. (c) IgA (Ref: Harrison 16th/2423, 9/e p242-243)

• Patients with ataxia telangiectasia (AT) present in the first decade of life with
progressive telangiectatic lesions associated with deficits in cerebellar
function and nystagmus. There is a high incidence of recurrent pulmonary
infections (bronchiectasisQ) and neoplasms of the lymphatic and
reticuloendothelial system.
• It is caused due to defect in DNA repair genesQ.
• Thymic hypoplasia with cellular and humoral (IgAQ and IgG2)
immunodeficiencies, premature agingQ and endocrine disorders such as
insulin resistance or type-I DMQ.
• The most striking neuropathologic changes include loss of Purkinje,
granule and basket cells in the cerebellar cortex as well as of neurons in
the deep cerebellar nuclei.
• A poorly developed or absent thymus gland is the most consistent defect
of the lymphoid system.

140. Ans. (b) Scleroderma (Ref: Robbins 7th/229)


141. Ans. (b) Neutrophil (Ref: Robbins 9/e p222)
LE cell or hematoxylin body is a phagocytic leukocyte (neutrophil or
macrophage) that has engulfed the denatured nucleus of an injured
cell.
Tart Cell is usually a monocyte which has ingested another cell or nucleus of
another cell.

142. Ans. (c) Anti-nuclear factor (Ref: Robbins 9/e p218-219)


143. Ans. (c) Hematuria (Ref: Robbins 9/e p222)
144. Ans. (c) Anti double stranded DNA antibody
(Ref: Robbins 9/e p219, 8th/214; 7th/228)
145. Ans. (a) Defect in the transport of chloride across epithelia
(Ref: Robbins 9/e p466-467)
146. Ans. (a) Sarcoidosis (Ref: Harsh Mohan 6th/164-165)
147. Ans. (a) p24 (Ref: Robbins 7th/246, 9/e p245)
148. Ans. (a) Kaposi’s sarcoma (Ref: Robbins 8th/523-524)
149. Ans. (a) Syphilis (Ref: Robbins 9/e p217)
150. Ans. (c) SLE (Ref: Robbins 9/e p225-227)
151. Ans. (a) Brain (Ref: Robbins 9/e p693)
152. Ans. (c) Wire loop lesions (Ref: Robbins 9/e p224)
153. Ans. (b) SLE (Ref: Robbins 9/e p224)
154. Ans. (b) Chemotaxis (Ref: Walter and Israel 7th/150)
The direct quote from the book is “Chediak Higashi syndrome is an
autosomal recessive condition in which polymorphs exhibit defective
random movements, defective chemotaxis and impaired
degranulation on phagocytosing particles.
155. Ans. (b) SLE (Ref: Robbins 9/e p219)
156. Ans. (a) Systemic sclerosis (Ref: Robbins 9/e p228)
157. Ans. (a) Hilar lymphadenopathy with perihilar calcification
(Ref: Robbins 9/e p693, 8th/703, 7th/738)
158. Ans. (a) CNS lesions (Ref: Robbins 9/e p254-255)
159. Ans. (d) Nodule is the initial lesion of Kaposi’s sarcoma
(Ref: Robbins 9/e p254, 8th/529; 7th/549)
160. Ans. (a) Helper cells (Ref: Robbins 9/e p246)
161. Ans. (c) CNS lesions (Ref: Robbins 9/e p254-255)
Direct quote from Robbins…. “The anatomic changes in the tissues (except
of lesions in the brainQ) are neither specific nor diagnostic”.
162. Ans. (c) Anti topoisomerase (Ref: Robbins 9/e p219)
Anti topoisomerase is least commonly associated with SLE amongst the
given options. The following is a modified table given for a reference
from Robbins.
Nature of Antigen Antibody System % Positive in SLE

Many nuclear antigens (DNA, Generic ANA (indirect IF) >95


RNA, proteins)

Native DNA Anti–double-stranded DNA 40–60

Histones Antihistone 50–70

Core proteins of small nuclear Anti-Sm 20–30


RNP particles (Smith antigen)

RNP (U1RNP) Nuclear RNP 30–40

RNP SS-A(Ro) 30–50


RNP SS-B(La) 10–15

DNA topoisomerase I Scl-70 <5

Centromeric proteins Anticentromere <5

Histidyl-tRNA synthetase Jo-1 <5

163. Ans. (a) ds DNA


(Ref: Robbins 8/e p214-215, 9/e p218-219) ...see text for detail
164. Ans. (a) Adenosine deaminase deficiency
(Ref: Robbins 8/e p234, 9/e p239)
• Severe combined immunodeficiency (SCID) represents a
constellation of genetically distinct syndromes, all having in
common defects in both humoral and cell-mediated immune
responses
• The most common form, accounting for 50% to 60% of cases, is
X-linked, and hence SCID is more common in boys than in girls.
The genetic defect in the X-linked form is a mutation in the
common γ-chain (γc) subunit of cytokine receptors
• The remaining cases of SCID are inherited as autosomal
recessive. The most common cause of autosomal recessive SCID
is a deficiency of the enzyme adenosine deaminase (ADA).

165. Ans. (c) Normal B cell count


(Ref: Robbins 8/e p233, 9/e p241) ...see text for detail
166. Ans. (b) Digeorge syndrome (Ref: Robbins 9/e p241)
167. Ans. (a) SLE (Ref: Robbins 9/e p224)
In the splenic tissue involvement in SLE, splenomegaly, capsular
thickening, and follicular hyperplasia are common features.
Central penicilliary arteries may show concentric intimal and
smooth muscle cell hyperplasia, producing so-called onion-skin
lesionsQ.
168. Ans. (c) Haematoxylin bodies (Ref: Robbins 9/e p256)
• Biopsy specimens from enlarged lymph nodes in the early stages
of HIV infection reveal a marked follicular hyperplasia.
• Monocytoid cells along the blood vessels can be seen in acute
lymphadenitis.
• With disease progression, the frenzy of B-cell proliferation subsides
and gives way to a pattern of severe follicular involution. The
follicles are depleted of cells, and the organized network of
follicular dendritic cells is disrupted. The germinal centers may
even become hyalinized.
• During this advanced stage viral burden in the nodes is reduced, in
part because of the disruption of the follicular dendritic cells. These
“burnt-out” lymph nodesQ are atrophic and small.
169. Ans. (c) Primary CNS Iymphoma
(Ref: Robbins 9/e p254-255)
Q
• Primary CNS lymphoma is the most common CNS neoplasm in
immunosuppressed individuals, including those with AIDS and
immunosuppression after transplantation.
• It is of B-cell originQ and most have infection caused by EBVQ in
the setting of immunosuppression.
• Histologically, reticulin stains demonstrate that the infiltrating cells are
separated from one another by silver-staining material; this pattern, referred
to as “hooping,” is characteristic of primary brain lymphoma.

• In addition to expressing B-cell markers, most of the cells also


express BCL-6; when tumors arise in the setting of
immunosuppression,
170. Ans. (a) Children previously tested with tuberculin test
(Ref: Robbins 9/e p371)
Children previously tested with tuberculin test may show a FALSE POSITIVE
TUBERULIN TEST.
False-negative Mantoux test False-positive Mantoux test

• SarcoidosisQ • Infection by atypical mycobacteriaQ


• MalnutritionQ • Previous vaccination with BCGQ
• Hodgkin diseaseQ
• ImmunosuppressionQ
• Fulminant tuberculosisQ

171. Ans. (c) Plasma derived Hepatitis B vaccine. ...see text of AIDS for
details

172. Ans. (c) HIV ELISA


Excellent question testing your basics buddies!
• HIV ELISA is required for detection of antibodies against the virus.
The antibodies may be present in the infant because of maternal
infection also because IgG antibodies can cross the placental
barrier. So,the presence of anti HIV antibodies is not reliable for
diagnosis of HIV in a 2 month old child.
• Please don’t get foxed by the option viral culture friends. There is
indeed a HIV culture method being used which is called as
‘peripheral blood mononuclear cells’ (PBMC) using the virus
microculture in macrophages concept. PBMC are drawn in high
concentration from centrifugation of freshly drawn anticoagulated
venous blood.
• Viral DNA detection using DNA-PCR andp24 antigen assay are
standard techniques used for viral detection.

173. Ans. (b) Disseminated TB


(Ref: Robbins 9/e p371)
• Mantoux test is simply able to predict the presence or absence of
cell mediated immunity against the tubercular antigens. It
CANNOT differentiate between infection and disease.
• Negative Mantoux test just indicates that the individual has never
been exposed to tubercle bacilli earlier; it can’t indicate the
susceptibility to the disease.
174. Ans. (c) A mechanism for the persistence and evolution of
autoimmune disease (Ref: Robbins 9/e p217)
Epitope spreading is a phenomenon in which an immune response
against one self antigen causes tissue damage, releasing other
antigens, and resulting in the activation of lymphocytes by these newly
encountered epitopes. This is responsible for the persistence and
progression of autoimmune diseases.
175. Ans. (d) Anterior uveitis (Ref: Harrison 18th/2806)
• Löfgren’s syndrome consists of erythema nodosum, hilar adenopathy on
chest X ray and uveitis.
• Heerfordt’s syndrome: fever, parotid enlargement, facial palsy and uveitis.

176. Ans. (a) Gp 120 (Ref: Robbins 8th/246-7)


HIV-1 uses CD4 to gain entry into host T-cells and achieves this through its
viral envelope protein known as gp120.
177. Ans. (b) 350
Under the revised guidelines the treatment of AIDS patients with respect to
opportunistic infections has undergone a change with H.A.A.R.T being
initiated at a threshold of CD4 count <350 cells/cu.mm instead of
previous 200 cells/cu.mm.
178. Ans. (d) Systemic lupus erythematosus
(Ref: Robbins 9/e p218)
Similar to many autoimmune diseases, SLE predominantly affects women. A
female-to-male ratio of 9:1 is seen during the reproductive age group.
By comparison, the female-to-male ratio is only 2:1 for disease
developing during childhood or after the age of 65.
179. Ans. (a) EBV (Ref: Robbins 9th/607)
180. Ans. (d) Seminiferous tubules (Ref: Robbins 9th/214)
The testis, eye, and brain, all of which are called immuneprivileged sites
because it is difficult to induce immune responses to antigens
introduced into these sites. If the antigens of these tissues are released,
for example, as a consequence of trauma or infection, the result may be
an immune response that leads to prolonged tissue inflammation and
injury. This is the postulated mechanism for post-traumatic orchitis and
uveitis.
181. And. (a) Ulcerative colitis (Ref: Robbins 9th/211)
The given table in Robbins clears all doubts regarding the answer or this
question.
` Table 6-6 Autoimmune Diseases
Organ-Specific Systemic
Diseases Mediated by Antibodies
Autoimmune haemolytic anemia Systemic lupus erythematosus
Autoimmune thrombocytopenia
Autoimmune atrophic gastritis of pemicious
anemia
Myasthenia gravis
Graves diseases
Goodpasture syndrome
Diseases Mediated by T Cells*
Type 1 diabetes mellitus Rheumatoid arthritis
Multiple sclerosis Systemic sclerosis (scleroderma)†
Sjogren syndrome†
Diseases Postulated to be Autoimmune
Inflammatory bowel diseases {Crohn
disease, ulcerative colitis
Primary Biliary Cirrhosis Polyartieritis nodosa†
Organ-Specific Systemic
Autoimmune (chronic active) hepatitis Inflammatory myopathies†

*A role for T cells has been demonstrated in these disorders, but


antibodies may also be involved in tissue injury.

An autoimmune basis of these disorders is suspected but the
supporting evidence is not strong
182. Ans. (c) Negative selection of T- cells in the thymus
(Ref: Robbins 9/e p???)
Negative selection of T- cells in the thymus is a mechanism of central
tolerance. Autoimmune diseases would develop only when there is
failure of tolerance. So, the answer.
183. Ans. (d) ) Lipoid nephrosis (Ref: Robbins 9/e p222-223)
184. Ans. (b) ) Subacute cutaneous lupus
(Ref: Robbins 9/e p219, Harrison 19/ table 319-1)
Anti RO (SSA) antibodies are not specific for SLE; it predisposes to
subacute cutaneous lupus, and to neonatal lupus with congenital
heart block. It is associated with decreased risk for nephritis.
185. Ans. (c) AA (Ref: Robbins 8th/252-3, 9/e p257)
186. Ans. (d) β2 microglobulin
(Ref: Robbins 8th/254, 9/e p258) ...see text
187. Ans. (b) Rectal biopsy
(Ref: Harrison 17th/2145-6; Robbins 8th/255, 9/e p262, Harsh Mohan
6th/88)
The histological examination of the biopsy material is the commonest
and confirmatory method for the diagnosis in a suspected case of
amyloidosis. The sites for the biopsy can be the renal tissue,
rectum, abdominal fat aspiration and gingiva. The rectumQ is the best
site for taking the biopsy in the options provided however as per
Harrison 19th ed/p 945 the abdominal fat aspirate is positive in
80% of cases.

Note: Congo red staining of aspirated abdominal fat is initial test of choice in
most cases. If it is found to be negative, more invasive biopsy of other affected organ
can be taken.

188. Ans. (b) Familial amyloidosis polyneuropathy


(Ref: Robbins 8th/252-253, 9/e p259)
189. Ans. (b) b2 microglobulin

(Ref: Robbins 7th/159-160, 9/e p258) ...see text


190. Ans. (a) Bone marrow plasmacytosis
(Ref: Robbins 7th/pg 260, 8th/252, 9/e p257)
191. Ans. (a) A beta 2 microglobulin (Ref: Robbins 9/e p258)
192. Ans. (c) Rectal biopsy
(Ref: Harrison 16th/2028, 17th/2145 Robbins 9/e p262)
193. Ans. (c) Smooth muscle cells (Ref: Robbins 7th/515)
The proliferation of smooth muscle cells is a critical event in the
neointimal hyperplastic response. Several studies have clearly
demonstrated that blockade of smooth muscle cell proliferation resulted
in preservation of normal vessel phenotype and function, causing the
reduction of neointimal hyperplasia and graft failure.
194. Ans. (c) Congo red
(Ref: Robbins 9/e p257 Harrison 17th/2145)
195. Ans. (a) X-ray crystallography (Ref: Robbin 9/e p257)

• b Pleated structure is seen on X-ray crystallography


• Electro Microscope shows non branching fibrils of indefinite length.

196. Ans. (a) Nonbranching filaments of indefinite length (Ref: Robbins


7th/259, 9/e p257)
197. Ans. (c) Methanamine silver (Ref: Harsh Mohan 5th/89, 6th/87,
Robbins 9/e p262)
198. Ans. (c) 7.5-10 nm fibrils
(Ref: Robbins 6th/259, 8th/249, 9/e p257)
Remember b-pleated structure of amyloid is seen on X-ray crystallography,
whereas it is seen as a non-branching fibril of 7.5-10 nm diameter and
infinite length on electron microscopy.
199. Ans. (c) Mutant transthyretin (Ref: Robbin’s 9/e p259)
200. Ans. (a) Sinusoids of red pulp (Ref: Robbins 9/e p261)
Amyloidosis of spleen
Sago spleen Lardaceous spleen (Mnemonic: Lal)

Amyloid deposition is Amyloid deposition spares the follicles and involve


largely limited to splenic the walls of the splenic sinuses in red pulp
follicles
Mnemonic: Red is ‘lal’ in hindi, so, similar sounding lardaceous.
201. Ans. (a) Thioflavin; (b) Congo red.
(Ref: Robbins 7th/254, 9/e p262)
202. Ans. (b) Amyloidosis
(Ref: Harrison 17th/2146; Robbins 7th/264, 9/e p262)
203. Ans. (c) Glycoprotein (Ref: Robbins 9/e p256)
204. Ans. (b) Chronic inflammatory states
(Ref: Robbins 9/e p257, 8th/251-252; 7th/159)
205. Ans. (c) Kidney (Ref: Robbins 8th/254, 9/e p261)
206. Ans. (a) Multiple myeloma-Light chain; (b) Chronic inflammation-
AA; (c) Cardiac-ATTR
(Ref: Harsh Mohan 5th/87, Robbins 9/e p259)
207. Ans. (a) A-β2 Microglobulin…………..See earlier explanation
(Ref: Robbins 9/e p258)
208. Ans. (c) Type 2 DM (Ref: Robbins 8/e p253, 9/e p259)
Amyloid replacement of islets is a characteristic finding in individuals
with long-standing type 2 diabetesQ. It is believed that the islet
amyloid protein is directly cytotoxic to islets, analogous to the role
played by amyloid plaques implicated in the pathogenesis of Alzheimer
disease
209. Ans. (b) Renal tissue (Ref: Robbins 8/e p254, 9/e p261)
• Amyloidosis of the kidney is the most common and potentially the
most serious form of organ involvement
210. Ans. (b) Familial amyloidotic polyneuropathy
(Ref: Robbins 8/e p253, 9/e p259)
211. Ans. (b) Renal failure (Ref: Robbins 8/e p254, 9/e p261)
“Amyloidosis of the kidney is the most common and potentially the
most serious form of organ involvement”…..direct lines from
Robbins.
Renal involvement gives rise to proteinuria that may be severe enough
to cause the nephrotic syndrome. Progressive obliteration of glomeruli
in advanced cases ultimately leads to renal failure and uremia. Renal
failure is a common cause of death.
212. Ans. (d) Chronic osteomyelitis
(Ref: Robbins 9/e p257, 8/e p253, 7/e p261)
Direct quote… “tuberculosis, bronchiectasis, and chronic osteomyelitis were
the most important underlying conditions, but with the advent of
effective antimicrobial chemotherapy the connective tissue disorders
such as rheumatoid arthritisQ (most common), ankylosing
spondylitis, and inflammatory bowel disease, particularly Crohn disease
and ulcerative colitis”.
213. Ans. (c) Brilliant pink color (Ref: Robbins 9/e p257)
Congo red under ordinary light imparts a pink or red color to amyloid
deposits. Under polarized light, the Congo red–stained amyloid shows a
green birefringence.
214. Ans. (a) Sinusoids of red pulp
(Ref: Robbins 9/e p261)
215. Ans (c) Beta – 2-microglobulin (Ref: Robbins 9/e p258)
216. Ans (b) Senile cardiac amyloidosis
(Ref: Robbins 9/e p258)
217. Ans (c) Calcitonin (Ref: Robbins 9/e p259)
Calcitonin is a protein which gets converted to amyloid and can lead to
organ dysfunction.
218. Ans (b) Ankylosing spondylitis (Ref: Robbins 9/e p259)
ANNEXURE

Non HLA genes associated with autoimmune diseases


Putative Gene Postulated Function of Encoded Protein and
Involved Diseases Role of Mutation/Polymorphism in Disease
Genes involved in immune regulation:
PTPN22 RA, T1 D, IBD Protein tyrosine phosphatase, may affect
signalling in lymphocytes and may after negative
selection or activation of self-reactive T cells
IL23R IBD, PS, AS Receptor for the T117-includeing cytokine IL-23;
may after differentiation of CD4+ T cells into
pathogenic T1 17 effector cells
CTLA4 T1D, RA Inhibits T cell responses by terminating activation
and promoting activity of regulatory T cells; may
interfere with self-tolerance
IL2RA MS, T1D a chain of the receptor for IL-2, which is a growth
and survival factor for activated and regulatory T
cells; may affect development of effector cells
and/or regulation of immune responses
Genes involved in immune responses to microbes:
NOD2 IBD Cytoplasmic sensor of bacteria expressed in
Paneth and other intestinal epithelial cells; may
control resistance to gut commensal bacteria
AT616 IBD Involved in autophagy; possible role in defense
against microbes and maintenance of epithelial
barrier function
IRF5, IFIH1 SLE Role in type 1 interferon production; type I IFN is
involved the pathogenesis of SLE (See text)

AUTOANTIBODIES IN AUTOIMMUNE DISORDERS


Specificity of Autoanti Association with specific
Disease Body Disease Features
Systemic lupus Double-stranded DNA Nephritis; specific for SLE
erythernatosus (SLE) U1-RNP
Smith (Sm) antigen (Core Specific for SLE
protein of small RNP
particles)
Ro (SS-A)/La (SS-B) Congenital heart block; neonatal
nucleoproteins lupus
Phospholipid-protein Antiphospholipid syndrome
complexes (anti-PL) (in~10% of SLE patients)
Multiple nuclear antigen Found in other autoimmune
(“generic ANAs”) diseases, not specific.
Systemic sclerosis DNA topoisomerase 1 Diffuse skin disease, lung
disease; specific for systemic
sclerosis
Centromeric proteins Limited skin disease, ischemic
(CENPs) A, B, C digital loss, putomonery
hypertension
RNA polymerase III Acute onset, sclerodema renal
crisis, cancer
Sjögren syndrome Ro/SS-A
La/SS-B
Autoimmune myositis Histidyl-tRNA synthetase, Interstitial lung disease,
Jo1 Raynaud phenomenon
Mi-2 nuclear antigen Dermatomyositis, skin rash
MDAS (cytoplasmic receptor Vascular skin lesions, interstitial
for viral RNA) lung disease
TF1y nuclear protein Dermatomyositis, cancer
Rheumatoid arthritis CCP (cyclic citrullinated Specific for rheumatoid
peptides); various arthritis
citrullinated proteins
Rheumatoid factor (not
specific)

For Special Attention... Potential Future AIIMS Question

• Anti RNA polymerase III antibody is associated with Acute onset, scleroderma
renal crisis and cancer.
• Autoimmune myositis is associated with antibody against Histidyl aminoacyl-tRNA
synthetase, Jo1 25, Mi-2 nuclear antigen, MDA5 (cytoplasmic receptor for viral
RNA) and TIF1γ nuclear protein

IgG4-RELATED DISEASE

• IgG4-related disease (IgG4-RD) is an idiopathic newly recognized constellation of


disorders affecting middle aged to old men characterized by tissue infiltrates dominated
by IgG4 antibody-producing plasma cells and T lymphocytes, storiform fibrosis,
obliterative phlebitis, and usually increased serum IgG4.Autoimmune myositis is
associated with antibody against Histidyl aminoacyl-tRNA synthetase, Jo1 25, Mi-2
nuclear antigen, MDA5 (cytoplasmic receptor for viral RNA) and TIF1γ nuclear protein.
• It includes disorders like Mikulicz syndrome (enlargement and fibrosis of salivary and
lacrimal glands), Riedel thyroiditis, idiopathic retroperitoneal fibrosis, autoimmune
pancreatitis, and inflammatory pseudotumors of the orbit, lungs, and kidneys.

X-linked Lymphoproliferative Disease


It is characterized by an inability to eliminate Epstein-Barr virus (EBV),
eventually leading to fulminant infectious mononucleosis and the
development of B-cell tumors.
In most of the cases it is caused by mutations in the gene for SLAM-
associated protein (SAP) that are associated with the activation of NK cells
and T and B lymphocytes. This leads to attenuated NK and T cell activation
and result in increased susceptibility to viral infections. SAP is also required
for the development of follicular helper T cells, and so, XLP patients are
unable to form germinal centers or produce high affinity antibodies.
1. A child with presented to the clinic with history of recurrent
infections. He also had a reduced platelet count and a skin rash as
shown in the image below. What is the likely diagnosis for this
patient?
(NEET 2020 like pattern)

(a) Henoch-Schonlein purpura


(b) Job syndrome
(c) Bruton’s agammaglobulinemia
(d) Wiskott-Aldrich syndrome
Ans. (d) Wiskott-Aldrich syndrome (Ref: Robbins 9th/242)
• The presence of recurrent infections is suggestive of immune
deficiency. Additionally, the child has reduced platelet count and a
skin rash. So, this completes the triad of Wiskott-Aldrich
syndrome.
• This is an X-linked disease caused by mutations in the gene
encoding Wiskott-Aldrich syndrome protein (WASP), which is
located at Xp11.23.
• IgM levels in the serum are low, but levels of IgG are usually
normal. Paradoxically the levels of IgA and IgE are often elevated.
• There is increased risk of developing B-cell lymphomas.
• The only treatment is HSC transplantation.
2. Graft used from an identical twin is called as:
(NEET 2020 like pattern)
(a) Xenograft
(b) Allograft
(c) Autograft
(d) Isograft
Ans. (d) Isograft (Ref: Robbins 9th/231)
• Donor and recipient are same: autograft
• Donor and recipient are different but have identical genes (twins): isograft
• Donor and recipient are different but have different genes: allograft
• Donor and recipient are from different species: xenograft

3. Basophils are activated by: (AIIMS Nov 2019 like pattern)


(a) IL– 5
(b) Neutrophils
(c) Killer inhibitory peptide
(d) Cell fixed IgE
Ans. (d) Cell fixed IgE (Ref: Robbins 9th/ 202)
Direct lines… “mast cells (and their circulating counterpart, basophils) are
activated by the cross-linking of high-affinity IgE Fc receptors; in
addition, mast cells may also be triggered by several other stimuli, such
as complement components C5a and C3a (called anaphylatoxins
because they elicit reactions that mimic anaphylaxis), both of which act
by binding to receptors on the mast cell membrane.
4. Which of the following is not involved in the genetics of severe
combined immunodeficiency?
(AIIMS Nov 2019 like pattern)
(a) BTK
(b) JAk-3
(c) ZAP 70
(d) IL2R
Ans. (a) BTK
(Ref: Robbins 9th/ 239-240, Harrison 20th/ 2493)
B-cell tyrosine kinase is required for maturation signals from the pre-B cell
and B-cell. The mutation in the BTK gene is associated with Bruton’s
disease characterized by defect in this enzyme associated with failure
of B-cell maturation and absence of antibodies.
Severe combined immunodeficiency (SCID) can be caused due to the
following reasons:
1. In 50–60% patients, it is X-linked and the genetic defect in this
condition is mutation in the common γ-chain (γc) subunit of cytokine
receptors including the receptor for IL-2, IL-7 etc.
2. In remaining patients, it is an autosomal recessive disorder due to
either:
• Deficiency of the enzyme adenosine
deaminase (ADA) or mutations in
recombinase-activating genes (RAG)
blocking the development of T and B
cells.
• An intracellular kinase called Jak3 is
essential for signal transduction through
the common cytokine receptor γ chain.
• The zeta chain-associated protein kinase
of 70 kD (ZAP-70) is involved in T cell
receptor (TCR) signaling and is critical
for T cell differentiation and function.
Deficiency of ZAP-70 causes a
combined immunodeficiency (CID).

5. Which of the following diseases in graph ‘C’ and ‘D’ with respect to
a normal graph (A and B) is diagnosed with the help of the given
flow cytometry pattern?
(AIIMS Nov 2019 like pattern)

(a) Bruton disease


(b) Bare lymphocyte syndrome
(c) Hyper IgM syndrome
(d) Severe combined immunodeficiency disease
Ans. (c) Hyper IgM syndrome (Ref: Read
explanation below and Robbins 9th/ 241)
Friends, let us understand the basics of the flow cytometry scatter plot
pattern as these questions have high chance to be repeated in the
future examinations too!
Scatterplot has the levels of two parameters. Two axes represent two
detectors and cells are presented by dots.

We separate the cells into four principal populations:


• Q1 and Q4 are cells strongly expressing one and only one color (single
positives),
• Q2 are the cells with both red and blue positivity (double positives) and
• Q3 are cells negative for both colors.
Coming now to our question, graphs ‘A’ and ‘B’ represent the pattern in a
normal individual and since both CD19 (X axis) and CD40 (Y axis) are
present, this is suggestive of the presence of B cells.
The graphs labelled ‘C’ and ‘D’ are of the patient. They have the normal
expression of CD19 (X axis) but reduced expression of CD40 (Y axis).
This means that the B cells of the patient have reduced expression of CD40
which is seen in a patient with hyper IgM syndrome.
As Robbins mentions, hyper-IgM syndrome is characterized by the failure to
produce isotype-switched high-affinity antibodies (IgG, IgA, IgE) due to a
defect in CD40–CD40L interaction. So, these people have decreased
concentrations of serum IgG and IgA and normal or elevated IgM, leading to
increased susceptibility to infections.
Revise the previously asked questions!
• Forward scatter (FS) is most sensitive to the size of the cell whereas side scatter
(SS) is most influenced by the optical homogeneity/granularity of the cell.
• Live cells have greater forward scatter in comparison to dead cells and granulocytes
have more side scatter.

6. True/false amyloidosis: (AIIMS May 2019 like pattern)


(a) Ab2 microglobulin is accumulated in senile amyloidosis
(b) ATTR is senile systemic amyloidosis
(c) Malignancy is the most common cause of amyloidosis in western
countries
(d) Mostly it contains kappa light chains
(e) Apple green under UV light when stained with congo red
Ans. Key
(a) False
(b) True
(c) True
(d) False
(e) False
• Ab
b2 microglobulin is accumulated in patients who had chronic kidney disease or those
who had undergone hemodialysis.
• Normal ATTR is deposited in the heart of aged individuals (senile systemic
amyloidosis). Abnormal ATTR deposition is seen with familial amyloid
polyneuropathies.
• Malignancy is the most common cause of amyloidosis in western countries as plasma
cell cancers are the most common reason for amyloidosis.
• Mostly it contains lambda 6 light chains which have a tendency for fibril formation.
• Apple green under UV light when stained with Congo red is false because it is observed
with polarized light.

7. Which of the following antibodies is involved in allergic


hypersensitivity? (AIIMS May 2019 like pattern)
(a) IgM
(b) IgA
(c) IgG
(d) IgE
Ans. (d) IgE (Ref: Robbins 9th/201)
Immediate, or type I, hypersensitivity is a rapid immunologic reaction
occurring in a previously sensitized individual that is triggered by the
binding of an antigen to IgE antibody on the surface of mast cells.
These reactions are often called allergy, and the antigens that elicit
them are allergens.
8. A 26-year-old female patient presented with fever, oral ulcers,
sensitivity to light and rash over the malar area of the face sparing
the nasolabial folds of both side. Which of the following indicates
the condition associated with these manifestations?
(NEET 2019 like pattern)
(a) Sturge weber syndrome
(b) SLE
(c) Dermatitis
(d) Psoriasis
Ans. (b) SLE (Ref: Robbins 9th e/p 222)
The description (fever, oral ulcers, photosensitivity and malar rash with a
butterfly distribution) fits perfectly in the clinical criteria required for
the diagnosis of SLE.
9. If a mother is donating here kidney to her son- This is an example of
which if the following?
(NEET 2019 like pattern)
(a) Isograft
(b) Allograft
(c) Autograft
(d) Xenograft
Ans. (b) Allograft (Ref: Robbins 9th e/p 231)
Grafts exchanged between individuals are as under:

• From individual to self: autograft


• From individuals of the same species: allografts
• Between individuals with identical genes (identical twins): isograft
• From one species to another: xenografts

10. Which of the following is not an AIDS defining illness?


(NEET 2019 like pattern)
(a) Hodgkin’s lymphoma
(b) Cervical cancer
(c) Primary CNS lymphoma
(d) Kaposi sarcoma
Ans. (a) Hodgkin’s lymphoma (Ref: Robbins 9th/253)
Patients with AIDS have a high incidence of certain tumors:
• Kaposi sarcoma (KS),
• B-cell lymphoma,
• Cervical cancer in women and anal cancer in men.

11. MHC II is associated with: (NEET 2019 like pattern)


(a) Antigen presenting cells
(b) Red blood cells
(c) Platelets
(d) Epithelial cells
Ans. (a) Antigen presenting cells (Ref: Robbins 9th e/p 195)
MHCII is associated with antigen presenting cells (dendritic cells, B cells and
macrophages).
12. Naive T cell get activated by:
(AIIMS Nov 2018 like pattern)
(a) NK cell
(b) Dendritic cell
(c) Macrophage
(d) B-lymphocyte
Ans. (b) Dendritic cell (Ref: Robbins 9th e/p 191)
Direct lines….. “Dendritic cells are the most important antigen-presenting
cells for initiating T-cell responses against protein antigens”.
13. Antibody dependent killing is a feature of which of the following
cell/s? (AIIMS Nov 2018 like pattern)
(a) Neutrophil only
(b) NK cell only
(c) Macrophage only
(d) NK cell, neutrophils & macrophage
Ans. (d) NK cell, neutrophils & macrophage
(Ref: Robbins 9th e/p 205)
Antibody-mediated destruction of cells may occur by another process called
antibody-dependent cellular cytotoxicity (ADCC). Cells that are
coated with IgG antibody are killed by a variety of effector c ells, mainly
NK cells and macrophages, which bind to the target by their receptors
for the Fc fragment of IgG, and cell lysis proceeds without
phagocytosis.
14. Which of the following can recognize dead material?
(AIIMS Nov 2018 like pattern)
(a) NET
(b) Inflammasome
(c) Necrosis
(d) Toll like receptor
Ans. (b) Inflammasome (Ref: Robbins 9th e/188)
The inflammasome is a protein complex that recognizes products of dead
cells and some microbes and induces the secretion of biologically
active interleukin 1. The inflammasome consists of a sensor protein (a
leucine-rich protein called NLRP3), an adapter, and the enzyme
caspase-1.
15. Which of the following is an immune privileged site?
(AIIMS Nov 2018 like pattern)
(a) Optic nerve
(b) Seminiferous tubule
(c) Area postrema
(d) Spinal cord
Ans. (b) Seminiferous tubule (Ref: Robbins 9th e/p 214)
Some antigens are hidden (sequestered) from the immune system, because
the tissues in which these antigens are located do not communicate
with the blood and lymph. As a result, self antigens in these tissues fail
to elicit immune responses and are essentially ignored by the immune
system. This is believed to be the case for the testis (seminiferous
tubules), eye (cornea and retinal pigment epithelial cells), and
brain, all of which are called immuneprivileged sites because it is
difficult to induce immune responses to antigens introduced into these
sites.
If the antigens of these tissues are released, for example, as a consequence
of trauma or infection, the result may be an immune response that
leads to prolonged tissue inflammation and injury.
16. Anti-centromere antibodies seen in which of the followings
conditions? (AIIMS Nov 2018 like pattern)
(a) Drug induced lupus
(b) SLE
(c) Sjögren syndrome
(d) Scleroderma
Ans. (d) Scleroderma (Ref: Robbins 9th e/p 230)
• Two ANAs strongly associated with systemic sclerosis:
– One of these, directed against DNA topoisomerase I (anti-
Scl 70), is highly specific. Patients who have this antibody
are more likely to have pulmonary fibrosis and peripheral
vascular disease.
– The other, an anticentromere antibody, is found in 20% to
30% of patients, who tend to have the CREST syndrome.
Patients with this syndrome have relatively limited
involvement of skin. Involvement of the viscera, including
esophageal lesions, pulmonary hypertension, and biliary
cirrhosis, may not occur at all or occur late.
17. Nobel prize for medicine/physiology in 2018 for the discovery
of: (AIIMS Nov 2018 like pattern)
a. Apoptotic pathway
b. Crispr-Cas9
c. Negative immune regulation
d. Molecular mechanisms controlling circadian rhythm
Ans. (c) Negative immune regulation (Ref: nobelprize.org )
Nobel Prize in Physiology or Medicine 2018: was awarded to James P.
Allison and Tasuku Honjo for their discovery of cancer therapy by
inhibition of negative immune regulation.” The Laureates have
shown how different strategies for inhibiting the brakes on the immune
system can be used in the treatment of cancer.
2017: Jeffrey C. Hall, Michael Rosbash and MichaelW. Young “for their
discoveries of molecular mechanisms controlling the circadian
rhythm.
18. Which of the following cannot be diagnosed without positive
ANA? (AIIMS Nov 2018 like pattern)
(a) SLE
(b) Sjögren
(c) Drug induced lupus
(d) Scleroderma
Ans. (a) SLE
(Ref: Robbins 9th e/p 218, Harrison 20th e/ p 2517 )
Antinuclear antibodies (ANAs) are seen in most patients with SLE. It is
considered as the best screening test; repeated negative tests by
immunofluorescence make SLE unlikely…Harrison.
19. Which of the following is not false about ataxia
telangiectasia? (AI 2018 Pattern)
(a) It is associated with normal immune function
(b) Serum levels of IgM are increased
(c) It is X linked recessive disease
(d) There is presence of amphicytes in different organs.
Ans (c) It is X linked recessive disease
(Ref: Robbins 9/e p1299)

• Ataxia-telangiectasia is an autosomal recessive disorder characterized disorder


characterized by an ataxic-dyskinetic syndrome beginning in early childhood, with
the subsequent development of telangiectasias in the conjunctiva and skin, along
with immunodeficiency.
• The abnormalities are predominantly in the cerebellum, with loss of Purkinje and
granule cells.
• Cells in many organs (e.g., Schwann cells in dorsal root ganglia and peripheral
nerves, endothelial cells, pituicytes) show a bizarre enlargement of the nucleus to
two to five times normal size and are referred to as amphicytes.
• The lymph nodes, thymus, and gonads are hypoplastic.
• Many affected individuals develop lymphoid neoplasms, which are most often T-
cell leukemias.
20. Hyperacute
rejection occurs within: (AI 2018 Pattern)
(a) 12 hours
(b) 2 weeks
(c) 1 month
(d) 3 month
Ans. (a) 12 hours (Ref: Robbins 9/e p233)
21. Nude mice is able to accept xenograft because they do not
have: (AI 2018 Pattern)
(a) B cells
(b) T cells
(c) Natural killer cells
(d) Macrophages
Ans. (b) T cells (Ref:
The Nude Mouse in Oncology Research/ch 2)
Nude mouse is a laboratory mouse strain which has a nonfunctioning or
absent thymus due to a mutation in the FOXN1 gene. So, it has no T
cells and therefore, has no graft rejection.
22. Mean transformation time for HIV to AIDS is:
(AI 2018 Pattern)
(a) 5 years
(b) 9 years
(c) 10 years
(d) 12 years
Ans. (c) 10 years (Ref: Robbins 9/e p252, Harrison 19/e p1229)
A chronic infection develops and persists with varying degrees of continual
virus replication in the untreated patient for a median of ~10 years
before the patient becomes clinically ill.
23. MHC Class II proteins are expressed by:
(AIIMS May 2017 Pattern)
(a) B-cells, dendritic cells and macrophages
(b) Platelets
(c) T-cells
(d) All nucleated cells
Ans. (a) B-cells, dendritic cells and macrophages
(Ref: Robbins 9/e p195)
B-cells, dendritic cells and macrophages are all antigen presenting cells.
MHC II proteins are expressed on the surface of antigen presenting
cells.
24. In thymus, which gene is responsible for recognition of self-
antigens? (AIIMS May 2017 Pattern)
(a) NOTCH 1
(b) AIRE
(c) Rb
(d) PTEN
Ans. (b) AIRE (Ref: Robbins 9/e p213)

• A protein called AIRE (autoimmune regulator) stimulates expression of some


“peripheral tissue-restricted” self antigens in the thymus and is thus critical for
deletion of immature T cells specific for these antigens.
• Mutations in the AIRE gene are the cause of an autoimmune
polyendocrinopathy called Autoimmune polyendocrine syndrome type 1
(APS1).

25. In a 30-year-old female patient with polyarthritis, testing reveals


nucleolar pattern of ANA staining. What is the likely course of this
patient?
(AIIMS May 2017 Pattern)
(a) Malar rash, alopecia and renal failure
(b) Sclerodactyly, esophageal dysmotility and Raynaud’s phenomenon
(c) Sjogren syndrome
(d) Painful genital and oral blisters and ulcers
Ans. (b) Sclerodactyly, esophageal dysmotility and Raynaud’s
phenomenon (Ref: Robbins 9/e p218)
Disclaimer
Any resemblance to an actual question is purely coincidental.
• Erythropoiesis in organs follows the sequence of yolk sac,
liver and then bone marrow.
• Ratio of fat cells to hematopoietic cells in bone marrow: 1:1.
• Myeloid to erythroid ratio in bone marrow: 3:1 or 4:1.
• Ratio of fat cells to red cells (erythroid) in bone marrow: 4:1.
• Largest number of bone marrow cells: Metamyelocytes.
• Best indicator of anisocytosis: red cell distribution width.
• Hereditary spherocytosis is the only important anemia with
increased MCHC.
• Not a feature of hemolytic anemia Increased heptoglobin (it is
decreased). However, reduced haptoglobin level in hemolysis
is masked by bile duct obstruction.
• Biconcave shape of RBC is due to Spectrin.
• Most common defect in hereditary spherocytosis is Ankyrin
(most common) followed by Band 3 (2nd MC). Others include
defective spectrin and Band 4.2 (palladin).
• Test for increased osmotic fragility in Hereditary
spherocytosis: Pink test.
• Most common cause of spherocytes: Immune haemolytic
anemia.
• Young female with spherocytes, investigation to be done:
Coomb’s test (to rule out immune hemolytic anemia).
• Bite cells and Heinz body are seen in G-6PD deficiency.
• Sickle cell anemia is due to a Point mutation (and not
deletion).
• Sickling is affected by: Concentration of HbS (most
important), deoxygenation and pH, duration of deoxygenation
in microcirculation.
• Best investigation for hemoglobinopathies is HPLC.
• On Hb electrophoresis: HbS moves Slower than HbA towards
positive electrode.
• Sickle cell trait provides pretection against: Falciparum
malaria.
• Gamma Gandy bodies are seen in Sickle cell anemia, chronc
myeloid leukemia and cirrhosis.
• HbE is common in: East India (Bengal, Assam)
• Mutation in thalassemia: Mainly point mutation (missense
mutation) causing aberrant splicing.
• HbA2 is raised (> 3·5%.) in thalassemia trait whereas HbF is
highly raised in thalassemia major (Cooley’s anemia).
• HbH disease is due to three alpha genes deletion whereas
Barts Hb is due to four gene deletions.
• Hair on end (Crew cut appearance) appearance on skull x-
ray: Thalassemia, SCA, HS, G6PD deficiency.
• Paroxysmal nocturnal
hemoglobinuria is due to Acquired defect in red cell PIG-A
gene leading to defective Glycosylphosphatidylinositol (GPI)-
linked proteins. It is best diagnosed by FLAER flow
cytometry. (Ref. CMDT 2018)
• M/C collagen vascular disorder causing Coomb’s positive
hemolytic anemia is SLE whereas the leukemia causing
Coomb’s positive test is CLL.
• Donath-Landsteiner antibody is seen in: Parxysmal cold
hemoglobinuria.
• Important infection causing non-immune hemolytic anemia:
Malaria.
• Drugs causing warm antibody hemolytic anemia: Methyldopa,
qunidine, cephalosporins, penicillin.
• Findings of microangiopathic hemolytic anemia (MAHA):
Fragmented RBCs (schistocytes), Burr cells, Helmet cells,
Triangle cells. Causes of microangiopathic anemia are TTP,
HUS, DIC and malignant hypertension. Macroangiopathic
anemia is seen with cardiac prosthetic valve.
• Important causes of microcytic hypochromic anemia: Iron
deficiency, sideroblastic anemia, thalassemia, anemia of
chronic disease.
• Blood index which reflects iron deficiency more accurately:
MCHC.
• Mentzer index < 13: Thalassemia minor whereas Mentzer
index > 13 is seen in iron deficiency anemia.
• M/C anemia in chronic renal failure is Normocytic
normochromic anemia.
• Sideroblastic anemia is having production of Ringed
sideroblasts and its causes include collagen vascular
disorders (SLE), lead (lead poisoning leads to lnhibition of
enzymes involved in heme synthesis like ferrochelatase and
aminolevulinic acid dehydratase), porphyria, myelofibrosis,
iron overload, alcoholism, myelodysplasia.

• Triad of megaloblastic anemia: Oval macrocytes (earliest


finding), hypersegmented neutrophils, Howell-Jolly bodies.
• MCHC is Not increased in megaloblastic anemia.
• False positive Schilling test: Renal insufficiency.
• Copper deficiency causes: normocytic hypochromic
anemia
• Aplastic anemia is characterized by pancytopenia and
absence of splenomegaly.
• Most common cause of Howell-Jolly bodies: Splenectomy.
• Cabot ring is seen in: Megaloblastic anemia (most
common), lead poisoning (2nd MC), hemolytic anemia,
thalassemia, myelodysplastic syndrome and
postsplenectomy.
• Types of anemia in lead poisoning: Microcytic hypochromic
(with reticulocytosis, basophilic stippling, target cells),
sideroblastic anemia, hemolytic anemia.
• Blood group antigens are not found in: CSF.
• Feature of Swachman-Diamond syndrome: Pancreatic
insufficiency; bone marrow dysfunc- tion; neutropenia; short
stature; skeletal abnormalities.
• Best method for hemoglobin estimation: Cyanmethemoglobin
method.
• Stain used for reticulocytes: Supravital stains (methylene blue
and Brilliant cresyl blue).
• Conditions protecting against falciparum malaria: G-6PD
deficiency, pyruvate kinase deficiency, sickle cell trait, beta
thalassemia trait and HbC.
Hematology is the study of the various cells and components of the
blood. Hematopoeisis is the process of production of blood cells which
primarily takes place in the following organs:
Sites of Hematopoeisis
Till 3rd week of intrauterine life: Yolk sac.
By 3rd month of intrauterine life: Liver is the main site of blood formation.
At 4th month of intrauterine life: Bone marrow is the main site of hematopoiesis.
Finally by birth, the bone marrow in whole of the skeleton is hematopoietically
active and is the chief source of blood cells and it remains so till puberty.
After puberty, the red marrow is present in vertebrae, ribs, sternum, skull, pelvis
and proximal epiphyseal regions of humerus and femur.

SALIENT FEATURES OF BONE MARROW

• The ratio of the fat cells and the hematopoeitic cells in an adult is
1:1. The number of the myeloid cells is more than the number of
the erythroid cells (normal M:E ratio is 3 to 15:1).
• The investigations for the information about bone marrow are bone
marrow aspiration and bone marrow biopsy.
Hematopoietic stem cells are pluripotent stem cells which are
CD34+ cells. They give rise to the trilineage myeloid cells,
lymphoblasts and monoblasts. The trilineage myeloid cell gives rise to
the following three cells:
• Normoblast (Gives rise to RBCs)
• Myeloblast (Gives rise to neutrophils, eosinophils and basophils)
• Megakaryocyte (Gives rise to platelets).
Monoblast gives rise to monocytes whereas lymphoblast gives rise
to lymphocytes.

Stages of Erythropoiesis
General Information About RBCs

The normal red cell is biconcave in shape and has a diameter of 7-8
mm. The cytoskeleton of the RBC is made up of proteins like spectrin,
ankyrin, band 2.1, band 3, band 4.1, etc. that provide deformability to
the RBCs so that they can cross through tiny blood vessels like
capillaries. Importance of these proteins can be appreciated in
disorders like hereditary spherocytosis. When the RBCs are of unequal
size, this is referred to as anisocytosis and when of different shapes, it
is called poikilocytosis.

• Reticulocytes are nonnucleated spherical cells bigger than normal RBCs


and are polychromatic (having a blue color) due to the presence of free
ribosomes and RNA.
• Reticulocyte count: Percentage of reticulocytes among the red cells
present in the peripheral blood is called reticulocyte count. Normal value
is around 1.5% in adults and 1.7% in the cord blood cells.
• Absolute reticulocyte count (ARC): Number of reticulocytes present in
1 mm3 of blood.
ARC = (Reticulocyte %) X Erythrocyte Count/100.
• Reticulocyte Index (RI): It adjusts reticulocyte count for hematocrit. It
reflects bone marrow activity and is also known as “Poor man’s Bone
Marrow Aspirate”. Normal reticulocyte index is 1-3%.
RI = Reticulocyte Count X (Hb/Age and sex adjusted normal Hb level).

Table 1: Conditions affecting reticulocyte count.

Reticulocytosis (Increased RBC Reticulocytopenia (Decreased RBC


production) production)
*Criteria *Criteria
Reticulocyte Index > 3% Reticulocyte Index < 1%
Reticulocyte Count > 1.5% Reticulocyte Count < 0.5%
Conditions Conditions
• Acute blood loss or hemorrhage • Aplastic anemia
• Postsplenectomy • Bone marrow infiltration
• Microangiopathic Anemia • Bone marrow suppression
• Autoimmune Hemolytic Anemia (Sepsis/Chemotherapy radiotherapy)
• Hemoglobinopathy (Sickle cell • Blood transfusion
anemia and Thalassemia) • Liver disease
• Post anemia Treatment like • Disordered RBC maturation (Iron, B12,
Folate Supplementation, Iron Folate Deficiency, Hypothyroidism,
Supplementation and vitamin B12 Sideroblastic Anemia or Anemia of
Supplementation. Chronic Disease)

ANEMIAS
Anemia is defined as any reduction below normal limits of the total
circulating red cell mass which is characterized by the clinical features
of pallor of skin and nails, dizziness, palpitations, lethargy and fatigue.
Some of the important terms used in context of anemias are as follows:
• MCV (Mean cell volume): It is the average volume (in femtolitres) of a
red blood cell (normal value is 82-96 fl).
• MCH (Mean corpuscular hemoglobin): Average mass of hemoglobin (in
picograms) per red blood cell is MCH. Normal value is 27-33 pg.
• MCHC (Mean corpuscular hemoglobin concentration): MCHC is
average concentration of hemoglobin in a given volume of packed red
blood cells. Normal value is 33-37g/dl.
• Normal RBCs have central pallor of around a third of the diameter
(normochromic). If the color is decreased which means pallor more than
one-third, the RBCs are called hypochromic and if color is increased
(central pallor is lost), the RBCs are called hyperchromic.
Anemia can be caused by blood loss, reduced red cell production and
excessive red cell destruction.

A. BLOOD LOSS
Blood loss causes decrease in hematocrit resulting in compensatory
increased release of erythropoietin from the renal juxtaglomerular cells.
Erythropoietin stimulates increased bone marrow activity. However, the
earliest change in the peripheral blood is leucocytosis (caused by
increased mobilization from the marginal pools) followed by
reticulocytosis and thrombocytosis. Chronic blood loss is usually due to
GIT lesions and gynecological disturbances.

B. IMPAIRED RBC PRODUCTION


1. DUE TO DECREASED ERYTHROPOIESIS
This category of anemias may result from the defective DNA synthesis
due to vitamin B12 or folic acid deficiency (called megaloblastic anemia)
or impaired heme synthesis due to iron deficiency.

Fig. 1: Megaloblastic Anemia.


a. Megaloblastic Anemia
The chief feature in this anemia is impaired DNA synthesis resulting in
delayed mitosis while RNA and protein synthesis is not impaired. This
leads to nuclear/cytoplasmic asynchrony which affects all proliferating
cell lines particularly cells of bone marrow and GIT cells. The chief
findings are a hypercellular bone marrow with megaloblasts in the
bone marrow along with presence of abnormal granulocytic precursors
(giant metamyelocyte and band forms) and large megakaryocytes with
bizarre multilobated nuclei. The presence of ineffective erythropoiesis
can result in pancytopenia associated with features of hemolytic
anemia including jaundice and increased levels of serum bilirubin and
LDH enzyme. In the peripheral smear, there is pancytopenia with
presence of macrocytes (RBCs having MCV >100 fl) lacking a central
pallor. There is characteristically presence of large and
hypersegmented neutrophils (neutrophils having > 5 lobes). The
earliest manifestation of megaloblastic anemia is presence of
hypersegmented neutrophils. Diagnosis is made if even a single
neutrophil with ≥ 6 lobes is seen or > 5% neutrophils with 5 lobes
are seen.

The two main causes of this anemia are vitamin B12 and folic acid
deficiency.
(i) Vitamin B12 deficiency
Normal vitamin B12 metabolism
The vitamin B12 (or cobalamin) is present in the bound form (bound
with dietary proteins) in the diet. It is freed by the action of pepsin
in stomach and then binds with salivary proteins called R-binders
(also known as cobalaphilins). In the duodenum, this cobalamin-
cobalaphilin complex is broken by the action of pancreatic
proteases. Free cobalamin now binds with the intrinsic factor
(Castle’s factor) secreted from the parietal cells of the stomach.
Vitamin B12-intrinsic factor complex is taken by the ileal
enterocytes. Within the intestinal cells, the cobalamin gets bound
with a transport protein called transcobalamin II which delivers it to
the rapidly proliferating cells of the body (bone marrow and GIT
cells). So, the causes of vitamin B12 deficiency can be:

Pernicious anemia
It is an autoimmune disorder against parietal cells of the stomach by
auto-reactive T cells resulting in chronic atrophic gastritis and parietal
cells loss (responsible for decreased intrinsic factor production). The
antibodies which are present in the patients are:

• Type I antibodiesQ (most common): Block binding of vitamin B12 to IF (in


75% patients).
• Type II antibodies: Prevent cobalamin-IF binding with ileal receptors.
• Type III antibodies: formed against the a and b subunits of gastric proton
pump (seen in 90% patients but not specific as it also seen in idiopathic
chronic gastritis).

It is also associated with other autoimmune disorders like


autoimmune thyroiditis and adrenalitis.

Morphology
The principal organs affected are the bone marrow, GIT and CNS
which show the following features:
Bone marrow
Megaloblasts, hypersegmented neutrophils and precursors of granulocytes along
with megakaryocytes are seen.
GIT
There is presence of shiny and “beefy” tongue due to atrophic glossitis, almost
complete loss of parietal cells and replacement of gastric mucosa by mucus
secreting goblet cells (intestinalization).
CNS
The combined involvement of the axons in the ascending tracts of posterior column
and the descending pyramidal tract is a characteristic feature of vitamin B12
deficiency giving the term as subacute combined degeneration of the spinal
cord.

Clinical Features
They are as follows:
• Megaloblastic anemia
• Pancytopenia (Leucopenia with hypersegmented neutrophils,
thrombocytopenia)
• Jaundice due to ineffective hematopoiesis and peripheral hemolysis
• Neurological features due to posterolateral spinal tract involvement.

Laboratory tests

• Serum antibodies against intrinsic factor are present.


• Achlorhydria even after histamine stimulation.
• Increased serum levels of methylmalonic acid and homocysteine.
• Schilling test: It is performed to distinguish between different causes of
vitamin B12 deficiency.

(ii) Folic acid deficiency


Megaloblastic anemia caused due to folic acid deficiency is
clinically indistinguishable from vitamin B12 deficiency anemia.
However, folic acid deficiency is NOT associated with neurological
abnormalities.
b. Due to Defective Hemoglobin Synthesis
(i) Iron deficiency
It is the commonest cause of anemia worldwide.
Normal iron metabolism
The metabolism of iron can be divided in the following headings:
Absorption
Iron is present in two forms in the food: heme and nonheme iron.
The iron is absorbed more completely from the heme form
(present in the nonvegetarian food) as compared to nonheme
form. The factors affecting absorption of iron include:
Factors increasing absorption Factors decreasing absorption
• Ferrous form (Fe2+) • Ferric form (Fe3+)
• Acid (HCl) in the stomach • Achlorhydria (absence of HCl secretion)
• Ascorbic acid • Alkaline food (pancreatic secretions)
• Amino acid and sugars in the • Phytates, tannates and phosphates in diet
food • Iron overload
• Iron deficiency • Tetracyclines and EDTA
• Physiological conditions • Inflammatory disorders
(pregnancy and hypoxia)

Iron is absorbed primarily from the duodenum in the ferrous form.


It is transported inside the enterocytes by an apical transporter called
DMT1 (Divalent Metal Transporter 1) and from here, it enters the
plasma by two basal membrane transporters (ferroportin and
hephaestin). A fraction of ferrous iron gets converted into ferric state
by intracellular oxidation. Most of the iron absorbed from the gut is lost
because of mucosal lining shedding whereas in increased
requirements it is absorbed in a greater percentage.

Transport and storage of iron


From the enterocytes, the absorbed iron is transferred to a plasma
protein called transferrin that delivers it to different cells of the body
expressing high levels of transferrin receptors on their surface. These
cells include hepatocytes and the developing erythroblasts in the bone
marrow. The serum transferrin saturation is an indicator of serum iron
concentration. Normally, transferrin is 33% saturated (one-third
saturation) with iron. Since, the serum transferrin concentration is
nearly 300-350 μg/dl (also called as total iron binding capacity or
TIBC), the normal serum iron levels are in the range of 100-120 μg/dl.
The iron which is not immediately required by the cells is stored in the
form of ferritin which is a protein iron complex present in all the
tissues especially liver, spleen and bone marrow. It is the ferric form of
iron which is present in ferritin. A small amount of ferritin is also
present in the plasma which is derived from the storage pools of the
body iron; so, serum ferritin is an indicator of body iron stores.
Intracellular iron is converted into hemosiderin which stains positively
with potassium ferrocyanide giving a positive Prussian blue stain.

The normal requirement of iron in the diet is nearly 1 mg/d. The


causes of iron deficiency anemia include the following:
Dietary lack Impaired Increased Chronic blood loss
absorption requirement
• Infants • Steatorrh • Growing infants • GIT (peptic ulcer,
• Children ea and children gastric cancer,
• Low socio- • Sprue • Pregnant hemorrhoids,
economic • Chronic females hookworm disease)
status diarrhea • Premenopausal • Urinary tract (renal,
• Elderly • Gastrecto women pelvic or bladder
my cancers)
• Genital tract (uterine
cancer, menorrhagia)
Fig. 2: Normal iron metabolism.

Features of Iron Deficiency Anemia


It is characterized by the following stages:
Stage I or stage of negative iron balance

This is a stage characterized by decreased amount of storage iron manifesting as


decreased serum ferritin concentration and reduced amount of bone marrow iron
staining with Prussian blue stain. The serum iron and red cell protoporphyrin levels
are absolutely normal. Though TIBC is marginally increased, the red cell indices
and morphology are normal.

Stage II or stage of iron deficient erythropoiesis

This is a stage of reduced circulating iron in addition to decrease storage form of


iron. So, this stage is characterized by deficient iron stores, reduced serum ferritin,
decreased % saturation of serum transferrin and increased TIBC. The red cell
morphology is normal.
Stage III or stage of iron deficiency anemia
It is characterized by all features of stage II and in addition abnormal morphology of
the red cells, i.e. the presence of microcytic and hypochromic cells.

Clinical features include fatigue, impaired growth and


development, pica (eating noedible substances like mud, etc. in
children), koilonychia (angular or spoon shaped nails), angular
stomatitis (ulceration at the angle of mouth), dysphagia (as in Plummer
Vinson syndrome) and palpitations (because of hyperdynamic
circulation which can even precipitate congestive heart failure).
Peripheral blood

Microcytic and hypochromic red cells with slight reticulocytosis whereas TLC is
normal. Usually, microcytosis is seen before appearance of hypochromia.
Poikilocytosis is seen in form of small and elongated red cells called pencil cells.
It is also characteristic feature of this disease. There is increased red cell
distribution width also.

Bone marrow

Hypercellular bone marrow (having increased erythroid progenitors) with depleted


bone marrow iron stores. There is presence of micronormoblasts.

Additional findings

• Serum ferritin and serum iron are decreased whereas serum transferrin and
TIBC are increased.
• Red cell protoporphyrin levels are increased because there is decrease in
the availability of heme (due to reduced iron availability) resulting in elevated
free erythrocytic protoporphyrin levels. RBC free protoporphyrin is normally 30-
50 μg/dl whereas its value reaches > 200 μg/dl in iron deficiency anemia.

Fig. 3: Iron Def Anemia. (H: Hypochromic cell; E: Elliptocyte)


The treatment of anemia is with the help of either oral or
parenteral iron therapy the response of which is clinically assessed
with improvement in symptoms and increase in the reticulocyte count
on about 8th - 9th day.

Differential diagnosis of microcytic hypochromic anemia

Iron Sideroblastic
Tests deficiency Inflammation Thalassemia anemia
Peripheral Micro/hypo Normal Micro/hypo with Variable
smear micro/hypo targeting
SI < 30 < 50 Normal to high Normal to high

TIBC > 360 < 300 Normal Normal


Percent TS < 10 10-20 30-80 30-80

Ferritin < 15 30-200 50-300 50-300


(mcg/L)
Hemoglobin Normal Normal Abnormal Normal
pattern
Contd...
Contd...

SI, serum iron; TIBC, total iron-binding capacity; TS, transferrin


saturation.
(ii) Thalassemia
It is discussed in detail with other hemolytic diseases.
There are many indices to differentiate between iron deficiency
anemia (IDA) and beta-thalassemia (BT): Two examples
include:
Value for Iron Value for beta-
Index Formula deficiency anemia thalassemia
Mentzer index MCV/RBC count > 13 < 13
Srivastava index MCH/RBC > 3.8 < 3.8

a. Miscellaneous
(i) Anemia of Chronic Disease (AOCD)
It is characterized by the decreased utilization of iron from the
storage from of iron, i.e. ferritin. In chronic inflammatory
conditions, there is increased secretion of cytokines like IL-1,
TNF, IFN-g, etc. that cause release of the protein hepcidinQ
because of which release of iron from the storage pool is
inhibited. This result in the high serum ferritin levels, reduced
TIBC, reduced % transferrin saturation and decreased serum
iron levels.

– In the peripheral blood smear there is presence of


microcytic and hypochromic red cells.
– In the bone marrow there is absence of hypercellularity
because of inhibition of erythropoetin secretion by renal
cells due to the action of cytokines like IL-1, etc.
– Common clinical conditions having AOCD include
diseases like rheumatoid arthritis, regional enteritis,
osteomyelitis, lung abscess and cancers like Hodgkin’s
lymphoma, carcinomas of breast and lung.

(ii) Anemia due to marrow infiltration


This type of anemia is caused due to infiltration of the bone
marrow resulting in myelophthisic anemia. It is characterized
by presence of immature erythroid and myeloid precursors in
the blood (this is called as leukoerythroblastosis). Metastasis
from cancers like breast, lung and prostate are the most
common cause of marrow infiltration.

(iii) Sideroblastic anemia


Sideroblastic anemia is characterized by the presence of ringed
sideroblasts. These are normoblasts having pin point iron
granules (easily demonstrable with the help of Prussian blue
dye) in the cytoplasm or perinuclear region. Sideroblastic
anemia can be hereditary (due to decreased ALA synthase
activity) or acquired (secondary to leukemias, myelodysplastic
syndrome, alcoholism, copper deficiency, pyridoxine
deficiency or lead poisoning). The pathogenesis of the
diseases involves defective heme synthesis resulting in
ineffective erythropoiesis which thereby contributes to iron
overload.

– Peripheral smear is characterized by presence of


microcytic hypochromic cells which also demonstrate the
presence of anisopoikilocytosis. There is increase in
serum iron, serum ferritin, % transferrin saturation and
free erythrocyte porphyrin whereas TIBC is decreased.

Note: Abnormal sideroblasts are also seen in thalassemia, megaloblastic


anemia and hemolytic anemias.

2. DUE TO DECREASED STEM CELLS PROLIFERATION


a. Aplastic anemia
This is a disorder characterized by marrow failure associated with
pancytopenia (anemia, thrombocytopenia and leukopenia). The
causes are:
Acquired Inherited

• Primary stem cell defect • Fanconi anemia


• Irradiation
• Viral infections [hepatitis (non A, non B , non C, non
G), CMV, EBV, varicella zoster virus]
• Chemical agents (alkylating agents, antimetabolites,
benzene, chloramphenicol, phenylbutazone,
arsenicals, insecticides like DDT, parathion)
It has been postulated that the etiological agents cause alteration
in the stem cells thereby activating the T cells of the body against
them resulting in destruction of the stem cells contributing to the
pancytopenia.

The bone marrow biopsy shows it is characteristically hypocellular being


replaced by fat cells (in contrast to aleukemic leukemia and myelodysplastic
syndrome in which we have pancytopenia associated with hypercellular
marrow) whereas bone marrow aspiration reveals “dry tap”.

Fig. 4: Aplastic Anemia. (Fat cells replace hematopoeitic cells in bone


marrow)
Clinical features are caused because of anemia (pallor, weakness
and dyspnea), thrombocytopenia (petechiae) and neutropenia
(recurrent infections). Red cells are normocytic and normochromic.
It is treated with either bone marrow transplantation (in young patients)
or antithymocyte globulin (in old patients).
b. Anemia of chronic disease (has been discussed above)
c. Anemia of renal failure
It is characterized by inadequate release of erythropoetin resulting
in development of anemia. The other contributory factors are:
Iron deficiency secondary to increased bleeding tendency (seen in
uremia)
– Extracorpuscular defect induced hemolysis

The severity of anemia is proportional to uremia and is usually


managed with recombinant erythropoetin.

C. HEMOLYTIC ANEMIA
This type of anemia can be due to intracorpuscular or
extracorpuscular defects.
Hemolysis can result due to destruction of RBCs inside the
circulation (intravascular) or outside the blood vessels (extravascular).

Note: Serum haptoglobin decreases but not as much as in intravascular


hemolysis.
1. Hereditary Spherocytosis
Hereditary spherocytosis (HS), an autosomal dominantQ disorder, is
an important cause of hemolytic anemia.

Normal RBC membrane skeleton


Normally, RBC membrane consists of a protein spectrin, which has
two subunits a and b. This spectrin is attached to cell membrane at two
sites. At head region, spectrin binds to ion transporter, band 3 of
membrane with the help of ankyrin and band 4.2 whereas at the tail
region, spectrin binds to glycophorin A of the membrane by protein 4.1
and actin.
Fig. 5: Hereditary Spherocytosis.(Spherical cells lacking central pallor)

Mutations in these proteins can result in HS. Mostly the mutations


are seen in head region most commonly in ankyrinQ [Robbins 8th/e
pg 642] and the next common mutation is in band 3 (Anion channel).
Rarely, the mutations can be seen in band 4.2 (Palladin), spectrin and
glycophorin A.

Pathogenesis
HS is characterized by reduced life span of RBC [10-20 days as
compared to 120 days] and has increased osmotic fragility (the
pathogenesis is explained above). The main clinical findings are
jaundice, splenomegaly and gallstones. A characteristic feature of HS
is increase in MCHCQ due to dehydration caused by loss of K+ and
water. It is almost the only condition where high MCHC is seen.
Pink testQ is done to measure the osmotic fragility. SplenectomyQ
is almost always beneficial in HS. After splenectomy anemia is
corrected but spherocytes will remain in blood. The vaccination against
encapsulated organisms like pneumococccus and H. influenza is also
must.
2. Glucose 6-Phosphate Dehydrogenase Deficiency (G-6PD
Deficiency)
Abnormalities in the hexose monophosphate shunt or glutathione
metabolism resulting from deficient or impaired enzyme function
reduce the ability of red cells to protect themselves against oxidative
injuries. This results in hemolytic disease. The most important of these
is G6PD deficiency. Normal G6PD functioning is required to decrease
oxidative damage to RBCs.
Conditions increasing oxidative stress
• Food (fava beans)
• Infections (WBC induced free radicals)
• Drugs (antimalarials, sulphonamide).
G6PD deficiency manifests in several distinct clinical patterns.
Most common is hemolysis after exposure to oxidant stress.
Acute intravascular hemolysis with anemia, hemoglobinemia, and
hemoglobinuria usually begins 2 to 3 days following exposure of
G6PD-deficient individuals to oxidants. Since only older red cells are at
risk for lysis, the episode is self-limited, as hemolysis stops when
only the younger red cells remain. Reticulocytosis is seen in the
recovery phase. The features of chronic hemolytic anemias like
splenomegaly and cholelithiasis are absent because the hemolytic
episodes occur intermittently.

3. Paroxysmal Nocturnal Hemoglobinuria


Paroxysmal nocturnal hemoglobinuria (PNH) is the only hemolytic
anemia caused by an acquired intrinsic defect in the cell
membrane. The stem cells of the bone marrow acquire mutations in
Phosphatidyl inositol glycan A (PIGA) gene, which is essential for the
synthesis of the glycosylphosphatidyl inositol (GPI) anchor. GPI is
responsible for providing an anchor for cell membrane attachment of
some proteins. In normal persons, these proteins are also required for
inactivating the complement. The examples of these proteins include:

• Decay-accelerating factor or CD55


• Membrane inhibitor of reactive lysis, or CD59 (most important)
• C8 binding protein

Out of the above mentioned proteins, CD 59 is most important. It


is a potent inhibitor of C3 convertase, and thereby prevents
spontaneous activation of the alternative complement pathway in vivo.
In PNH patients, due to the absence of these protective proteins, red
blood cells, platelets and granulocytes become more sensitive to lysis
by complement system.

Clinical features include intravascular hemolysis with


hemoglobinuria. The complement system is activated by acidotic
conditions like exercise (accumulation of lactic acid) or sleep (due to
decreased respiratory rate). Since the respiration decreases at night,
so, patient experiences intermittent attacks (paroxysmal) of hemolysis
at night (nocturnal) resulting in passage of red urine
(hemoglobinuria) in the morning. The dysfunction of the GPI linked
proteins on the platelets is responsible for the prothrombotic state.
Diagnosis

• It is best made with flow cytometry in which there is presence of


bimodal distribution of the red cells i.e. cells which are deficient in
CD55/CD59 as well normal cells which are CD55+/CD59+.
• Other tests demonstrating increased susceptibility to the complement
system which can be used for diagnosis include:
1. Ham’s acidified serum test: lysis of erythrocytes on addition of
acidified serum.
2. Sucrose lysis test: complement system is increased by the
presence of sucrose.

4. Immune Hemolytic Anemias


Immune hemolytic anemias are caused because of the formation of
anti-RBC antibodies.

Types of Immune Hemolytic Anemia


Warm antibody type Cold antibody type
Mostly IgGQ; rarely IgA Mostly IgMQ, rarely IgG
Causes Causes
• Primary (Idiopathic) • Primary (Idiopathic)
• SLEQ, rheumatoid arthritis • Mycoplasma infection,
• B cell lymphoid neoplasms • Infectious Mononucleosis
• Drugs (α-methyldopa, penicillin) • Lymphoid neoplasms
• Paroxysmal cold
hemoglobinuria (IgG)
Mechanism of hemolysis Mechanism of hemolysis
Extravascular hemolysis (in spleen) Extravascular hemolysis in cold
agglutinin (in liver)
Intravascular hemolysis in cold
hemolysins
The antibody does not usually fix Antibodies reacts at 4-6°C, dissociate
complement, and is active at 37°C. at 30°C or above
Cold-antibody autoimmune hemolytic anemia (cold AIHA) is
subdivided into two clinical categories based on the type of antibodies
involved. These two types of cold antibodies are:
a. Cold agglutinins: these are monoclonal IgM antibodies that react
at 4 to 6°C. They are called agglutinins because the IgM directed
against the I antigen present on the RBCs can agglutinate red cells
due to its large size (pentamer). In addition IgM can activate
complement resulting in the cells being coated with C3b followed
by extravascular hemolysis. Examples include Mycoplasma
pneumonitits and infectious monoucleosis. Vascular obstruction
by the red cell agglutination can produce Raynaud’s
pheonomenon, which is characterized by ischemia in the fingers
when expose to the cold.

Fig. 6: Cold agglutinin disease.

b. Cold hemolysins: these are seen in patients with paroxysmal


cold hemolglobinuria (PCH). They are unique because they are
biphasic antierythrocyte autoantibodies. These antibodies are IgG
that is directed against the P blood group antigen. They are called
biphasic because they attach to red cells and bind complement at
4°C but the complement activation takes place when the
temperature is increased. This is followed by intravascular
hemolysis. The antibody is called the Donath-Landsteiner
antibody (previously associated with syphilis, but also with
mycoplasma pneumonia measles, mumps, and ill defined viral and
“flu” syndromesQ.
5. Microangiopathic hemolytic anemia (MAHA)

MAHA is a microangiopathic subgroup of hemolytic anemia (anemia, loss of


red blood cells through destruction) caused by factors in the small blood
vessels. The endothelial layer of small vessels is damaged with resulting fibrin
deposition and platelet aggregation. As red blood cells travel through these
damaged vessels, they are fragmented resulting in intravascular hemolysis. It
is identified by the finding of anemia and schistocytes, “burr cells,” “helmet
cells,” and “triangle cells”on microscopy (should be > 3/5000 cells) and
these should have 1-3 sharp spicules. It is associated with conditions like
DICQ (most commonly), malignant hypertension, SLE, thrombotic
thrombocytopenic purpura (TTP), hemolytic-uremic syndrome (HUS), and
disseminated cancer.

6. Hemoglobinopathies

A. Sickle cell anemia


This is characterized by the presence of an abnormal type of
hemoglobin called HbS. It results from a point mutation that causes the
glutamic acid to be replaced by valine at the b6 position of the globin
chain. If the individual is homozygous, it is represented as HbSS (1
gene each from both the parents) whereas the heterozygous is HbAS
(1 gene from one parent is for HbS and the other gene is for HbA).
Heterozygotes are protected against falciparum malaria.

Pathogenesis
When deoxygenated, HbS molecules becomes insoluble, undergoes
aggregation and polymerization producing a sickle cell or holly leaf
shape of the RBCs. Initially, this process is reversible (on getting
oxygenated, the cells attain there normal shape) but repeated attacks
of aggregation can cause irreversible sickling of the RBCs which also
causes oxidative damage to the red cells.

Reversible sickled cells exhibit increased adhesiveness within the


microcirculation of organs with sluggish blood flow thereby causing
episodes of hypoxia and infarction called as vasoocclusive crisis or
pain crisis. Hemoglobin released from the lysed red cells causes
inactivation of NO thereby increasing the severity of ischemia.
Irreversible sickled cells get sequestrated in the spleen thereby
contributing to extravascular hemolysis.

Factors Affecting Sickling of the Hemoglobin


Amount of HbS (most important factor) and its interaction with other
hemoglobins
The presence of relatively low concentration of HbS (25-40%) and the presence of
HbA in heterozygotes prevents efficient HbS sickling thereby contributing to
decreased severity of the disease in them. In comparison, the homozygotes have
full blown disease.
Other hemoglobins like fetal hemoglobin (HbF) and HbC (having a substitution of
lysine for glutamic acid at β6 position) have inhibitory effect on the disease.
Hemoglobin concentration of the red cell
Hb concentration of the cell, i.e. MCHC affects polymerization to a great extent.
Decrease in pH
Acidosis increases chances of sickling.
Duration of time red cells are exposed to decreased oxygen tension
Organs having slow or sluggish circulation (bone and spleen) have an increased
chance of sickling.

Clinical features
• Severe anemia results in jaundice and pigment gallstone formation
and is associated with reticulocytosis. Vaso-occlusive crisis
clinically manifests as painful episodes in affected organs of the
body. In the bone, it presents as dactylitis or inflammation of the
bones of hands and feet, so called Hand foot syndrome, increased
chances of Salmonella osteomyelitis, avascular necrosis of femoral
head, fish mouth appearance of vertebra (due to occlusion of
vertebral arteries) and prominent cheek bones and crew cut
appearance of skull (both because of extramedullary
hematopoeisis).
• Other organs of the body may also be affected, e.g. lungs (acute
chest syndrome characterized by cough, fever and chest pain),
brain (seizures or stroke), skin (leg ulcers), penis (stagnation in
corpora cavernosa leads to priapism) or spleen. In the initial
stages, there is splenomegaly due to congestion and trapping of
red cells in the vascular sinusoids (Gamma gandy bodies;
consisting of foci of fibrosis having iron or calcium salts deposited
in connective tissue are seen).
• Prolonged hypoxia and infarction can lead to autosplenectomy
which increases susceptibility to infection with capsulated
organisms like Hemophilus influenzae, Pneumococcus, etc.
• Parvovirus infection can precipitate an attack of aplastic crisis also.
Chronic anemia can cause hyperdynamic circulation resulting in
cardiomegaly.

Fig. 7: Sickle Cell Anemia. (S: Sickled cells; T: Target cells)

Fig. 8: Spleen in sickle Cell Anemia. (G: Gamma gandy body)


Peripheral smear shows anisopoikilocytosis, presence of sickle
cells, target cells, polychromatophilia and ovalocytes. Howell Jolly
bodies (composed of chromatin aggregates in red cells) are seen
particularly after autosplenectomy.
Diagnosis is done with the help of the following:
• Sickling test: Sickling is induced by a reducing agent like 2% metabisulfite or
dithionite to blood. However, this test cannot differentiate between sickle cell
disease and sickle cell trait.
• Hb electrophoresis: It is carried out on a cellulose acetate membrane (pH
8.6). HbS is slower moving as compared to normal HbA, so, heterozygotes
show 2 bands of hemoglobin.
• HbF estimation (by alkali denaturation method) shows HbF to be 10-30% in
homozygotes.
• HPLC is the best investigation for the diagnosis of sickle cell disease.
• Prenatal genetic testing can be done using the enzyme MstII endonuclease.
Chorionic villus sampling at 10-12 weeks of gestation is used to estimate
fetal DNA abnormality.

B. Thalassemia
It is a group of autosomal recessive inherited disorders characterized
by decreased synthesis of either α or β globin chain of HbA. It is the
most common type of hemoglobinopathy in the world. β and α
thalassemia is caused by deficient synthesis of β and α chains
respectively.

The α globin chain is coded by a gene on chromosome 16 and the


gene for β globin chain is located on chromosome 11. The clinical
features therefore result from deficiency of one chain and the relative
excess of the other chain.
Pathogenesis of β thalassemia
i. β thalassemia syndromes
This type of thalassemia is caused by point mutations. These are
of two types:
1. β0 thalassemia – Characterized by total absence of β chains
in the homozygous state.
2. β+ thalassemia – Characterized by reduced synthesis of β
chains in the homozygous state.

Mutations can be caused due to the following mechanisms:


a. Promoter region mutation: Causes reduced transcription
of the β chains leading to β+ thalassemia.
b. Chain terminator mutations: Either creation of a stop
codon in exon or frameshift mutation inducing a
downstream stop codon leads to premature chain
termination resulting in β° thalassemia.
c. Splicing mutation: They are the most common cause of
thalassemia resulting in unspliced mRNA being degraded
in the nucleus leading to the development of either β+
thalassemia or β° thalassemia.

Fig. 9: Thalassemia smear heaving hypochromic cells, target cells (T).


Inset shows nucleated RBC.
Clinical features
Clinically β, thalassemia is of three types: thalassemia major,
thalassemia intermedia and thalassemia minor.
a. Thalassemia major (also called Cooley anemia): It is seen in
individuals homozygous for the β thalassemia genes (β+/β+or β°/
β°); these individuals have a severe transfusion dependent anemia
which manifests at usually 6 to 9 months after birth. There is
presence of prominent frontal and cheek bones,
hepatosplenomegaly (due to extramedullary hemopoiesis),
jaundice, increased risk of pigment stones and endocrinological
manifestations as delayed puberty (due to GH deficiency), bone
fractures (hypoparathyroidism) and/or diabetes mellitus (iron in
islet of Langerhans).
Peripheral smear shows moderate to severe anemia, anisocytosis,
microcytic hypochromic red cells, target cells, nucleated red cells,
basophilic stippling, Howell Jolly bodies, etc. There is
reticulocytosis and left shift in the leukocytes. Since β chains are
not produced but γ chains are synthesized normally, HbF is
markedly increased and is the major constituent of red cells
(90%). MCH, MCV and MCHC are reduced.
Bone marrow is hypercellular with erythroid hyperplasia causing
reversal of normal M:E ratio (it becomes 1:3 in thalassemia).
Pink inclusions are seen in the normoblasts (caused by α chain
accumulation). Widening of the diploe gives rise to crew cut
appearance on skull X-ray.

b. Thalassemia minor or trait: It is seen in individuals heterozygous


with one β thalassemia gene and one normal gene (β+/β or β0/β). It
is more common clinically than the major variant and offers
resistance against falciparum malaria. These patients are usually
asymptomatic with only mild anemia.
Peripheral smear shows microcytic, hypochromic cells with basophilic
stippling and presence of target cells. MCH, MCV and MCHC are
reduced. The levels of HbA2 are characteristically elevated (It is
normally 3-3.5% but in thalassemia trait, the level is 3.6-8%; this is
a diagnostic feature of this disease). HbF is mildly increased (5%).
Fig. 10: NESTROF test to screen thalassemia trait.

NESTROF Test

A Screening test used for this condition is Naked Eye Single Tube Red cell
Osmotic Fragility (NESTROF) test. In this test 2 blood samples (1of a normal
person serving as control and 1 of patient) are added to 2 tubes with 0.35%
saline. After 30 min a white paper with a black line is placed behind both the
tubes. The RBCs in control sample undergo hemolysis so the black line is
visible whereas cells in thalassemia trait are resistant so black line is not
clearly visible.

c. Thalassemia intermedia: The patients show anemia but do not


require transfusions. The features of the disease are intermediate
between the two other types of thalassemia discussed above.

Important Investigations

• Apart from the above mentioned investigations, the thalassemia patients


must undergo Hb electrophoresis to determine the nature of the hemoglobin
present. HbA2Q is characteristically elevated in a patient of thalassemia
minorQ.
• Globin chain synthesis can be studied by calculating a: β ratio. It is
normally 1:1 but in thalassemia, it is 5-30:1.
• Alkali denaturation methodQ is done to determine the concentration of
HbF which is relatively resistant to denaturation by strong alkali like
NaOH/KOH as compared to HbA. In the RBCs, HbF is shown by acid elution
method (Kleihauer’s cytochemical methodQ).

Management of these patients is done with blood transfusions


(to maintain hemoglobin concentration), iron chelators like
desferrioxamine, deferiprone and defrasirox (to chelate excessive iron)
and bone marrow transplantation (if HLA matched donor is
available).
Preventive measures include marriage counseling and chorionic
villus sampling at 9-10 weeks followed by PCR analysis. All antenatal
females with Hb <11 gm% should undergo NESTROF test.
ii. a thalassemia syndromes
This type of thalassemia is caused by gene deletionQ. These are
of four types as depicted in the flowchart.
In the silent carrier state, the patients are clinically asymptomatic.
The clinical picture in a-thalassemia trait is similar to that discussed in
β-thalassemia minor which means there is presence of microcytosis,
minimal or no anemia, and no abnormal physical signs. HbHQ is a
major cause of anemia, as precipitates of oxidized HbH form in older
red cells, which are then removed by splenic macrophages. This
produces a moderately severe anemia resembling β-thalassemia
intermedia. Hydrops fetalis is the most dangerous form of α-
thalassemia and severe tissue anoxia leads to intrauterine fetal death.
The fetus shows severe pallor, generalized edema, and massive
hepatosplenomegaly.
RBC: GENERAL ASPECTS

1. Which of the following is associated with an intrinsic defect in


the RBC membrane?
(AIIMS May 2012)
(a) Autoimmune hemolytic anemia
(b) Hereditary spherocytosis
(c) Microangiopathic haemolytic anemia
(d) Thermal injury causing anemia
2. Which of the following is not a stem cell of the bone marrow?
(AI 2012)
(a) Lymphoblast
(b) Myeloblast
(c) Myoblast
(d) Normoblast
3. Which of the following surface glycoproteins is most often
expressed in human hematopoietic stem cell?
(a) CD 22
(b) CD 40
(AIIMS May 2008)
(c) CD 15
(d) CD 34
4. Reticulocytosis is seen in all except:
(AIIMS May 2007)
(a) P.N.H.
(b) Hemolysis
(c) Nutritional anemia
(d) Dyserythropoietic syndrome
5. Which of these are seen on Romanowsky stain?
(a) Reticulocytes
(PGI Dec 2004)
(b) Basophilic stippling
(c) Heinz bodies
(d) Howell-Jolly bodies
(e) Cabot ring
6. Which of the following surface glycoproteins is most often
expressed in human hematopoietic stem cell?
(a) CD22
(b) CD40
(Delhi PG 2010)
(c) CD15
(d) CD34
7. Inappropriate erythropoietin level is found in all except:
(UP 2001)
(a) Renal cell carcinoma
(b) Lung disease
(c) High altitude
(d) Benign liver tumor
8. The size of the red blood cells is measured by:
(a) MCV
(b) MCHC
(UP 2007)
(c) ESR
(d) MCH

MOST RECENT QUESTIONS

9. Anemia which is associated with pancytopenia is:


(a) Hemolytic
(c) Megaloblastic
(b) Iron deficiency
(d) All
10. Hematuria with dysmorphic RBCs are seen in:
(a) Acute glomerulonephritis
(b) Renal TB
(c) Renal calculi
(d) Chronic renal failure
11. MCHC is increased in:
(a) Iron deficiency anemia
(b) Spherocytosis
(c) Thalassemia
(d) All
12. In polycythemia vera, all are raised except:
(a) Hematocrit
(b) Platelet count
(c) RBCs
(d) Erythropoietin
13. The type of anemia seen in chronic renal failure is:
(a) Microcytic
(b) Normocytic
(c) Macrocytic
(d) All of the above
14. Burr cell is seen in:
(a) Uremia
(b) Hepatocellular carcinoma
(c) Gastric carcinoma
(d) Ovarian carcinoma
15. Acanthocytes are seen in:
(a) Abetalipoproteinemia
(b) Hartnup disease
(c) Whipple disease
(d) None
16. Reticulocytes are stained with:
(a) Methyl violet
(b) Brilliant Cresyl blue
(c) Sudan black
(d) Indigo carmine
17. Storage form of iron:
(a) Ferritin
(b) Transferrin
(c) Hepcidin
(d) Ferroportin
18. Hb is a good buffer because of:
(a) Histidine residues
(b) Protein nature
(c) Acidic nature
(d) Iron molecule
19. Normal platelet count is found in:
(a) Wiskott Aldrich syndrome
(b) Henoch Schonlein purpura
(c) Immune thrombocytopenia
(d) Dengue fever
20. In an adult man, there is about how much grams of
hemoglobin in the circulating blood?
(a) 350
(b) 500
(c) 900
(d) 1000
21. The longest living WBC is which one of the following
(a) Lymphocyte
(b) Eosinophil
(c) Neutrophil
(d) Monocyte
22. Freezing point of normal human plasma is:
(a) 4°C
(b) 0°C
(c) –0.54°C
(d) –1.54°C
23. The normal albumin: globulin (A/G) ratio blood is
(a) 5:1
(b) 2:1
(c) 1:2
(d) 1:1
24. Thrombosthenin is:
(a)Coagulation protein
(b) Contractile protein
(c) Thrombus inhibiting protein
(d) Protein for platelet production
25. The best method for estimation of hemoglobin concentration
in blood is:
(a) Acid hematin method
(b) Alkali hematin method
(c) Cyanmethemoglobin method
(d) Any of the above

26. The number of Fe2+ atoms in one Hb molecule:


(a) 1
(b) 2
(c) 4
(d) 8
27. Linzenmeyer is used to measure:
(a)Bleeding time
(b) Clotting time
(c) Prothrombin time
(d) ESR
28. Serum contains all the clotting factors except:
(a) Plasma thromboplastin
(b) Labile factor
(c) Hageman factor
(d) Christmas factor
29. Progenitor hematopoetic stem cells originate in which of the
following?
(a) Bone marrow
(b) Thymus
(c) Lymph node
(d) Spleen
30. Haematocrit is the ratio of:
(a) WBC to whole blood
(b) Platelets to whole blood
(c) RBCs to whole blood
(d) Total blood cells to plasma
31 The anaemia associated with leukaemia is:
(a) Iron deficiency
(b) Megaloblastic type
(c) Myelophthisic type
(d) None of the above
32. Which of the following is a distinguishing feature of
reticulocyte?
(a) Constitute 10% of the red cells
(b) No nucleus
(c) Smaller in the size than RBCs
(d) Mature in lymph nodes
33. Life span of RBCs in infant is:
(a) 100–120 days
(b) 60–80 days
(c) 80–100 days
(d) 40–60 days
34. Increase in MCHC is associated with?
(a) Iron deficiency anemia
(b) Megaloblastic anemia
(c) Anemia of chronic disease
(d) Hereditary spherocytosis
35. MCHC criteria to diagnose iron deficiency anemia:
(a) <32
(b) <34
(c) <28
(d) <30

MEGALOBLASTIC ANEMIA, APLASTIC ANEMIA

36. Which of these does not indicate megaloblastic anemia?


(a) Increased reticulocyte count
(AIIMS Nov 2012)
(b) Raised Bilirubin
(c) Mild splenomegaly
(d) Nucleated RBC
37. A patient with Hb-6 gm%, TLC 1200, platelet-60,000, MCV 12fl,
what is the diagnosis?
(AIIMS May 2008)
(a) Aplastic anemia
(b) Megaloblastic anemia
(c) PNH
(d) Myelofibrosis
38. Macrocytosis in complete blood count can be diagnosed by:
(PGI Dec 2006)
(a) ↑ MCV
(b) ↑ MCHC
(c) ↑ Hematocrit
(d) ↑ Red cell distribution width
39. Which is the true statement regarding megaloblastic anemia?
(PGI Dec 01)
(a) Megaloblastic precursors are present in bone marrow
(b) Mean corpuscular volume is increased
(c) Serum LDH is increased
(d) Thrombocytosis occurs
(e) Target cells are found
40. Macrocytic anemia may be seen in all of these except:
(a) Liver disease
(PGI June 2002)
(b) Copper deficiency
(c) Thiamine deficiency
(d) Vitamin B12 deficiency
(e) Orotic aciduria
41. Causes of vitamin B12 deficiency megaloblastic anemia are:
(PGI June 2005)
(a) Fish tap worm infestation
(b) Dilantin therapy
(c) Gastrectomy
(d) Ileal resection
(e) Methotrexate
42. Aplastic anemia can progress to all except:
(a) AML
(Delhi PG 2010)
(b) Myelodysplastic anemia
(c) Pure red cell aplasia
(d) Paroxysmal nocturnal hemoglobinuria
43. Serum vitamin B12 level is increased in all except:
(a) Hepatitis
(UP 2000)
(b) Cirrhosis of liver
(c) Hepatocellular carcinoma
(d) Cholestatic jaundice

MOST RECENT QUESTIONS

44. Normocytic normochromic anemia is seen in all except:


(a) Aplastic anemia
(b) Chronic renal disease
(c) Pure red cell aplasia
(d) Thalassemia
45. A 76 years old male presented with anemia with
splenomegaly. PBS shows tear drop shaped cells and bone
marrow examination was normal. The diagnosis is:
(a) Myelofibrosis
(b) Iron deficiency anemia
(c) Folic acid deficiency
(d) CML
46. Abnormality in Schilling test can be seen in all of the
following except:
(a) B12 deficiency
(b) Folic acid deficiency
(c) Ileal disease
(d) Bacterial overgrowth
47. Cause of macrocytic anemia is:
(a) Sideroblastic anemia
(b) Iron deficiency
(c) Thalassemia
(d) Hypothyroidism
48. Pure red cell aplasia is associated with:
(a) Thymoma
(b) Renal cell carcinoma
(c) Hepatocellular carcinoma
(d) Prostate carcinoma
49. Vitamin B12 malabsorption is caused by:
(a) Ankylostoma duodenale
(b) Diphyllobothrium latum
(c) Giardiasis
(d) Taenia solium
50. Maturation failure in poor absorption of the vitamin B12 is
associated with:
(a) Microcytic hypochromic anemia
(b) Sickle cell anemia
(c) Anemia occurs after 3-4 months of poor absorption
(d) Causes polycythemia
51. FIGLU test is done for:
(a) Cyanocobalamin deficiency
(b) Folic acid deficiency
(c) Thiamine deficiency
(d) Riboflavin deficiency
52. Hypersegmented neutrophils are present in which of the
following anemia?
(a) Hemolytic
(b) Iron deficiency
(c) Megaloblastic
(d) Aplastic
53. Hypersegmented neutrophils are seen in:
(a) Thalassemia
(b) Iron deficiency
(c) Megaloblastic anemia
(d) All
54. Howell-Jolly bodies are seen in:
(a) Alcoholics
(b) Cirrhosis
(c) Nephrotic syndrome
(d) Postsplenectomy
55. Macrocytic anemia is caused by:
(a) Hookworm infestation
(b) Iron deficiency
(c) Diphyllobothrium latum infestation
(d) All of the above
56. An adult who develops pure red cell aplasia should be
explicitly evaluated for which of the following?
(a) Gastric adenocarcinoma
(b) Pancreatic adenocarcinoma
(c) Papillary thyroid cancer
(d) Thymoma
57. All of the following can cause reticulocytosis except:
(a) Aplastic anemia
(b) Thalassemia
(c) Sickle cell anemia
(d) Chronic blood loss
58. Which of following viruses causes hemolysis of red blood
cells?
(a) Rubella
(b) Human parvo virus B19
(c) Measles
(d) Dengue virus
59. Which of the does not indicate megaloblastic anemia?
(a) Raised bilirubin
(b) Mild splenomegaly
(c) Increased reticulocyte count
(d) Nucleated red cells
60. Hb A2 is raised in which of the following conditions?
(a) Beta thalassemia trait
(b) Sickle cell anemia
(c) Hereditory spherocytosis
(d) G6 PD deficiency
61. Reticulocytosis is not seen in which of the following
conditions?
(a) Thalassemia
(b) Hereditary spherocytosis
(c) Chronic renal failure
(d) Sickle cell anemia
62. Schilling test is used for identification of which of the
following?
(a) Fat absorption
(b) Vit K absorption
(c) Vitamin B12 absorption
(d) Vitamin D absorption

MICROCYTIC ANEMIA: IDA, AOCD, SIDEROBLASTIC ANEMIA

63. A 60-year-old male patient with history of rheumatoid arthritis


presents with the following: Hb:4.5g/dL. platelet count is 2
lakh/mm3. TLC: 6000/mL, serum ferritin is 200µg/dL, serum
iron 30mg/dL and TIBC 280ng/L. Which of the following is
the most likely diagnosis?
(a) Anaemia of chronic disease
(AIIMS Nov 2011)
(b) Thalassemia minor
(c) Iron deficiency anemia
(d) Autoimmune haemolytic anemia
64. A 20-year-old female presents with the following laboratory
values: hemoglobin 9gm%, MCV is 55%, RBC is 4.5
million/mm3. There is no history of blood transfusion. What
is the most likely diagnosis out of the following?
(AIIMS Nov 2011)
(a) Thalassemia major
(b) Thalassemia minor
(c) Iron deficiency anemia
(d) Anemia of chronic disease
65. A 13-year-girl with fatigue and weakness was found to be
having reduced hemoglobin. Her MCV 70fl, MCH 22pg and
RDW was 28. What is her most likely diagnosis?
(a) Iron deficiency anemia
(AI 2010)
(b) Thalassemia minor
(c) Sideroblastic anemia
(d) Thalassemia major
66. Ringed sideroblasts are seen in:
(AI 2008)
(a) Iron deficiency anemia
(b) Myelodysplastic syndrome
(c) Thalassemia
(d) Anemia of chronic disease
67. A 30 years old female, RBC count 4.5 million, MCV 55fl, TLC
8000/mm3. There is no history of blood transfusion. What is
the likely diagnosis?
(a) Iron deficiency anemia
(AIIMS May 2008)
(b) Thalassemia major
(c) Thalassemia minor
(d) Megaloblastic anemia
68. The pathogenesis of hypochromic anemia in lead poisoning
is due to:
(AIIMS Nov 2002)
(a) Inhibition of enzymes involved in heme biosynthesis
(b) Binding of lead to transferrin, inhibiting the transport of iron
(c) Binding of lead to cell membrane of erythroid precursors.
(d) Binding of lead to ferritin inhibiting their breakdown into
hemosiderin
69. A patient presents with increased serum ferritin, decreased
TIBC, increased serum iron, % saturation increased. Most
probable diagnosis is:
(a) Anemia of chronic disease
(AIIMS Nov 2006)
(b) Sideroblastic anemia
(c) Iron deficiency anemia
(d) Thalassemia minor
70. Anemia in CRF is due to:
(PGI Dec 2006)
(a) ↓ erythropoietin
(b) ↓RBC survival
(c) ↓ folate
(d) Bone marrow hypoplasia
(e) Iron deficiency
71. Iron deficiency anemia is seen in:
(PGI Dec 2006)
(a) Chronic renal failure
(b) Billroth II operation
(c) Hookworm infection
(d) Celiac sprue
(e) Carcinoma cecum
72. Bone marrow iron is increased in:
(PGI Dec 2003)
(a) Thalassemi(a)
(b) Iron deficiency anemi(a)
(c) Anemia in chronic disease.
(d) PNH
(e) Megaloblastic anemi(a)
73. Microcytosis is seen in:
(PGI June 2004)
(a) Thalassemia
(b) Hb Lepore
(c) Hb Barts
(d) Gastrectomy
(e) Systemic sclerosis
74. True about iron deficiency anemia is:
(PGI June 2005)
(a) Microcytic hypochromic anemia
(b) Decreased TIBC
(c) Increased ferritin
(d) Bone marrow iron decreased earlier than serum iron
75. In Anemia of chronic disease, what is seen?
(a) TIBC ↑
(b) S. Iron ↑
(Delhi PG-2008)
(c) BM iron ↓
(d) S. ferritin ↑
76. A 30 years old female asymptomatic not requiring blood
transfusion has Hb-13 gm%, HbF-95%, HbA2 1.5%. Which of
the following is the most likely diagnosis?
(a) Beta-Heterozygous thalassemia
(Delhi PG-2008)
(b) Beta-Homozygous thalassemia
(c) Intermediate thalassemia
(d) Persistently raised HbF
77. The condition which does not cause microcytic hypochromic
anemia is:
(Karnataka 2008)
(a) Iron deficiency
(b) Hookworm infestation
(c) Absence of intrinsic factor
(d) Prolonged bleeding episodes
78. Hypochromic microcytic blood picture is seen in all of the
following conditions except:
(Karnataka 2007)
(a) Iron deficiency anemia
(b) Lead poisoning
(c) Rheumatoid arthritis
(d) Sideroblastic anemia

MOST RECENT QUESTIONS

79. Lead poisoning is associated with:


(a) Microcytic hypochromic anemia
(b) Macrocytic anemia
(c) Decreased levels of zinc protoporphyrin
(d) Howell-Jolly bodies
80. Microspherocytes in peripheral blood smear are seen in:
(a) Congenital spherocytosis
(b) Autoimmune acquired hemolytic anemia
(c) Thalassemia
(d) All of the above
81. “Macropolycytes” in peripheral smear is a feature of:
(a)Hereditary spherocytosis
(b) Iron deficiency anemia
(c) Sickle cell anemia
(d) Megaloblastic anemia
82. Anemia of chronic disease is characterized by of all except:
(a) ↓ Serum iron level
(b) ↓ TIBC
(c) ↓ Serum ferritin level
(d) Increased macrophages iron in marrow
83. Most common cause of anemia is:
(a) Iron deficiency
(b) Folic acid deficiency
(c) Sideroblastic anemia
(d) Pernicious anemia
84. A patient of anemia due to chronic inflammation, the positive
finding is:
(a) Serum iron is increased
(b) S. ferritin is decreased
(c) TIBC is decreased
(d) Presence of normal iron in blasts
85. Which of the following glycoproteins is transported in plasma
in iron metabolism?
(a) Spectrin
(b) Transferrin
(c) Ferritin
(d) Hemosiderin
86. All are laboratory finding in iron deficiency anemia except:
(a) Decreased serum iron
(b) Increased total iron binding capacity
(c) Decreased serum ferritin
(d) Increased mean corpuscular volume
87. Best parameter for assessment of body iron stores is:
(a) Serum iron
(b) Serum TIBC
(c) Serum ferritin
(d) Serum transferin
88. Hemochromatosis affects all of the following organs except:
(a) Liver
(b) Pancreas
(c) Heart
(d) Salivary gland
89. Skin pigmentation in hemochromatosis occurs due to:
(a) Melanin
(b) Ferritin
(c) Hemosiderin
(d) All
90. Storage form of iron in body is:
(a) Ferritin
(b) Transferrin
(c) Ceruloplasmin
(d) Ferriportin
91. Microcytic hypochromic anemia is seen in:
(a) Hereditary spherocytosis
(b) Thalassemia major
(c) Iron deficiency anemia
(d) Pernicious anemia
92. Earliest feature of correction of iron deficiency anemia is:
(a) Reticulocytosis
(b) Increase in serum ferritin
(c) Increase in RBC count
(d) Increase in serum iron level immediately
93. Lead causes following except:
(a) Uroporphyrinuria
(b) Sideroblastic anemia
(c) Basophilic stippling
(d) Macrocytic anemia
94. Low iron and low TIBC is seen in:
(a) Anaemia of chronic disease
(b) Sideroblasticanaemia
(c) Iron deficiency anaemia
(d) Aplastic anemia
95. Anemia in humans can be caused by which of the following
worm?
(a) Roundworm
(b) Hookworm
(c) Strongyloides
(d) Tapeworm
96. Sideroblastic anemia is seen in chronic poisoning with:
(a) Lead
(b) Arsenic
(c) Copper
(d) Mercury
97. Echinocytes are types of:
(a) RBCs
(b) Lymphocytes
(c) Monocytes
(d) Platelets
98. Response of iron therapy in a patient with iron deficiency
anemia is denoted by:
(a)Restoration of enzymes
(b) Reticulocytosis
(c) Increase in iron binding capacity
(d) Increase in hemoglobin
99. Sideroblastic anemia is caused by all except:
(a) Collagen vascular disease
(b) Erythropoetic porphyria
(c) Lead poisoning
(d) Cutaneous porphyria
100. Rate of iron uptake is regulated by which one of the
following:
(a) Mucosal cell iron stores
(b) Route of administration
(c) Preparation administered
(d) Age of the patient
101. All of the following if present provide protection against
malaria except:
(a) Duffy blood group
(b) Sickle cell anemia
(c) Thalassemia
(d) G6PD deficiency
102. Most important but nonspecific regulator of iron metabolism
is:
(a) Hepcidin
(b) DMT1
(c) Ferroportin
(d) Ferritin

HEMOLYTIC ANEMIA: PNH, HS, G6PD, IMMUNE HEMOLYTIC


ANEMIA

103. A 23-year-old female presented with jaundice and pallor for


2 months. Her peripheral blood smear shows the presence
of spherocytes. The most relevant investigation to arrive at a
diagnosis is which of the following?
(AIIMS May 2012)
(a) Tests for PNH
(b) Osmotic fragility test
(c) Coombs test
(d) Reticulocyte count
104. An abnormal Ham test is most likely associated with which
of the following?
(AIIMS Nov 2011)
(a) Spectrin
(b) Defect in complement activating proteins
(c) Defective GPI anchor
(d) Mannose-binding residue effect
105. A 5-year-old male child presents with episodic anemia and
jaundice since birth. He is least likely to have which of the
following?
(AIIMS Nov 2011)
(a) Hereditary spherocytosis
(b) Sickle cell disease
(c) G6PD deficiency
(d) Paroxysmal nocturnal hemoglobinuria
106.Thrombotic event is seen in all of following except:
(AIIMS May 2011)
(a) Paroxysmal nocturnal hemogloninuria
(b) Disseminated intravascular coagulation
(c) Idiopathic thrombocytopenic purpura
(d) Heparin induced thrombocytopenia
107. PNH associated with somatic mutation affecting:
(a) Decay accelerating factor (DAF)
(AI 2010)
(b) Membrane inhibitor of reactive lysis (MIRL)
(c) Glycosylphosphatidylinositol (GPI)
(d) C8 binding protein
108. Cold hemagglutinin is associated with:
(AI 2008)
(a) Anti IgM
(b) Anti IgG
(c) Anti IgA
(d) Donath-Landsteiner antibody
109. The following protein defects can cause hereditary
spherocytosis except:
(AI 2007)
(a) Ankyrin
(b) Palladin
(c) Glycophorin C
(d) Anion transport protein
110. Autoimmune hemolytic anemia is seen in:
(AI 2001)
(a) ALL
(b) AML
(c) CLL
(d) CML
111. Microangiopathic hemolytic anemia is seen in all of the
following diseases except:
(AIIMS Nov 2008)
(a) Antiphospholipid antibody syndrome
(b) Thrombotic thrombocytopenic purpura
(c) Microscopic polyangiitis
(d) Metallic cardiac valves
112. An Rh -ve woman became pregnant with Rh +ve fetus.
Within few days after birth, the infant developed jaundice,
ascites, hepatomegaly and edema. The likely substance(s)
deposited in skin and sclera in jaundice is/are given below.
Which is the best possible answer?
(a) Biliverdin
(AIIMS Nov 2003)
(b) Conjugated and unconjugated bilirubin
(c) Unconjugated bilirubin
(d) Conjugated bilirubin
113. Features seen in hemolytic anemia are all except:
(a) Teardrop and Burr cells
(b) ↓ Haptoglobin
(c) Reticulocytosis
(d) Hemoglobinuria
114. Intravascular hemolysis occurs in:
(a) Hereditary spherocytosis
(PGI Dec 2000)
(b) Acute G6PD deficiency
(c) Sickle cell disease
(d) Thalassemia
(e) PNH
115. Microangiopathic hemolytic anemia is seen in:
(a) HUS (PGI June 01)
(b) ITP
(c) Malignant hypertension
(d) Prosthetic valves
(e) TTP
116. Spherocytosis in blood smear is seen in:
(a) Hemoglobin C
(PGI June 2004)
(b) Mechanical trauma
(c) Hereditary spherocytosis
(d) Hereditary elliptosis
117. Cause of fragmented RBC in peripheral blood:
(a) Microangiopathic hemolytic anemia
(b) DIC
(PGI June 2005)
(c) Hemophilia-A
(d) Malignant hypertension
(e) HELLP syndrome
118. Intravascular hemolysis is seen in:
(PGI June 2005)
(a) Glucose-6 phosphate dehydrogenase deficiency
(b) Thalassemia
(c) Sickle cell anemia
(d) Hemophilia
119. The peripheral smear of hereditary spherocytosis will show
spherocytes:
(PGI Dec 2006)
(a) Usually of same size
(b) Reticulocytosis seen
(c) Smaller size
(d) Anemia is negligible
(e) Always associated with ↑ MCHC
120. Microangiopathic hemolytic anemia seen in:
(a) Thrombotic thrombocytopenic purpura
(b) Hemolytic uremic syndrome
(PGI Dec 2003)
(c) Henoch-Schonlein purpura
(d) DIC
(e) IgA nephropathy

MOST RECENT QUESTIONS

121. Donath-Landsteiner antibodies are seen in:


(a) Warm agglutination
(b) Cold agglutination
(c) Paroxysmal nocturnal hemoglobinuria
(d) ITP
122. In hereditary spherocytosis an inherited abnormality is seen
in which of the following red blood cell component:
(a) a-globin chain
(b) b-globin chain
(c) Phosphatidyl inositol glycan A
(d) Spectrin
123. Hereditary spherocytosis is due to:
(a) Acquired membrane defect
(b) Ankyrin deficiency
(c) Defective hemoglobin synthesis
(d) Mechanical trauma to red cells
124. Not seen in paroxysmal nocturnal hemoglobinuria is:
(a) LDH levels are raised
(b) Increased hemosiderin in urine
(c) Decreased leukocyte alkaline phosphatase
(d) Increased platelets
125. ‘Warm’ autoantibodies are seen in:
(a) SLE
(b) Mycoplasma
(c) Syphilis
(d) Varicella
126. Hot agglutinin is found in all except:
(Bihar 2006)
(a) Mycoplasma infection
(b) SLE
(c) Methyl dopa
(d) Rheumatoid arthritis
127. PNH due to defect in:
(a) CD 59
(b) CD 15
(c) CD 100
(d) CD 20
128. D.I.C is seen in:
(a) Acute promyelocytic leukemia
(b) Acute myelomonocytic leukemia
(c) CML
(d) Autoimmune haemolytic anemia
129. Intravascular hemolysis occurs in:
(a) Hereditary spherocytosis
(b) Autoimmune haemolytic anemia
(c) Paroxysmal nocturnal hemoglobinuria
(d) Thalassemia
130. G6PD help in maintaining the integrity of RBC by:
(a) Controlling reduction stress on RBC
(b) Controlling oxidative stress on RBC
(c) Maintaining flexibility of cell membrane
(d) Component of electron transport chain
131. Cold agglutinin is:
(a) IgG
(b) IgM
(c) IgA
(d) IgD
132. Helmet cells are characteristic of anemia of:
(a) Hemolytic uremic syndrome
(b) Polysplenia
(c) Spherocytosis
(d) Acanthocytosis
133. Schistocyte is/are found in:
(a) TTP
(b) March hemoglobinuria
(c) Severe iron deficiency
(d) All of the above
134. In DIC, following are seen except:
(a) Fibrinogen decreased
(b) Thrombocytopenia
(c) Normal APTT
(d) PT elevation
135. All are the features of hemolytic anemia except:
(a) Hemoglobinuria
(b) Jaundice
(c) Increased haptoglobin
(d) Hemosiderinuria
136. Bite cells are seen in:
(a) G6PD deficiency
(b) Sickle cell anemia
(c) Hereditary spherocytosis
(d) Trauma
137. A 56-year-old female presents in the month of December
with chronic fatigue and cyanosis of nose with blue lips. A
peripheral blood smear showed the following image. What is
the most likely cause of the findings seen?
(a) Clumps of RBCs due to IgM mediated cold autoimmune
haemolytic anemia
(b) Clumps of RBCs due to IgG mediated warm autoimmune
haemolytic anemia
(c) RBC lysis due to hemoglobinopathy
(d) Clumps of RBCs due to IgG mediated cold autoimmune
haemolytic anemia
138. All of these are seen in hemolytic anaemia except:
(AIIMS Nov 2016)
(a) Yellowing of eyes and sclera
(b) Increased LDH
(c) Decrease in haptoglobin
(d) Low reticulocyte count
139. Direct Coombs test is positive in all of the following except:
(a) Drug induced AIHA
(b) Aplastic anemia
(c) Hemolytic anemia due to transfusion
(d) Hemolytic disease of newborn
140. A 26-year-old female presents with pallor with a hemoglobin
of 9.5 mg/dL, PCV 30 mm Hg and RBC count of 2 million per
cubic millimeters. What is the likely diagnosis?
(AIIMS Nov 2016)
(a) Sideroblastic anemia
(b) Iron deficiency anemia
(c) Thalassemia
(d) Folic acid deficiency
141. Peripheral smear with small pale red cells, anisocytosis and
poikilocytosis is suggestive of:
(a) Aplastic anemia
(b) Iron deficiency anemia
(c) Hereditary spherocytosis
(d) Megaloblastic anemia
142. Hemolytic uremic syndrome is associated with which of the
following:
(a) Most commonly caused by verocytogenic E.coli
(b) Causes mild to severe coombs positive hemolytic anemia
(c) Recurrences are rare
(d) Transient thrombocytopenia
143. Intrinsic causes of hemolytic anemia are all except:
(a) G6PD deficiency
(b) Hereditary spherocytosis
(c) Hypersplenism
(d) Pyruvate kinase deficiency
144. Which of the following is the nature of the warm antibody in
autoimmune hemolytic anemia?
(a) IgE (b) IgG
(c)IgM (d)IgD

HEMOGLOBINOPATHIES: SICKLE CELL ANEMIA,


THALASSEMIA

145. Person having heterozygous sickle cell trait are protected


from infection of:
(AIIMS Nov 2012)
(a) P. falciparum
(b) P. vivax
(c) Pneumococcous
(d) Salmonella
146. A 6-years-old child belonging to Punjabi family with past
history of blood transfusions presented with hemoglobin 3.5
g/dl, MCV – 30 fl, MCHC – 20. Peripheral smear findings of
microcytic hypochromic anemia with target cell and reduced
osmotic fragility. What is the he probable diagnosis of
patient?
(AIIMS Nov 2012)
(a) Alpha thalassemia
(b) Beta thalassemia
(c) Sickle cell anemia
(d) G6PD deficiency
147. NESTROF test is a screening test for which of the following
conditions?
(AIIMS Nov 2011)
(a) β- thalassemia
(b) Hereditary spherocytosis
(c) Autoimmune haemolytic anaemia
(d) Megaloblastic anaemia
148. Which of the following is the cause of alpha thalassemia?
(AIIMS May 2011)
(a) Deletion of alpha genes
(b) Deletion of beta genes
(c) Excess of alpha genes
(d) Single amino acid substitution in alpha chain
149. HbH is formed due to which of the following?
(a) Deletion of 4 alpha chains
(AI 2011)
(b) Deletion of 3 alpha chains
(c) Deletion of 2 alpha chains
(d) Deletion of 1 alpha chain
150. Mutation causing sickle cell anemia is protective for which
of the following?
(AI 2010)
(a) Malaria
(b) Filaria
(c) Leishmania
(d) None of the above
151. What is affected in HbS (Hemoglobin S)?
(AI 2009)
(a) Stability
(b) Function
(c) Affinity
(d) Solubility
152. A couple, with a family history of beta thalassemia major in
a distant relative, has come for counseling. The husband has
HbA2 of 4.8% and the wife has HbA2 of 2.3%. The risk of
having a child with beta thalassemia major is:
(AI 2003)
(a) 50%
(b) 25%
(c) 5%
(d) 0%
153. The primary defect which leads to sickle cell anemia is:
(AI 2003)
(a) An abnormality in porphyrin part of hemoglobin
(b) Replacement of glutamate by valine in b-chain of HbA
(c) A nonsense mutation in the b-chain of HbA
(d) Substitution of valine by glutamate in the a-chain of HbA
154. Which one of the following statements about hemoglobin S
(HbS) is not true?
(AIIMS Nov 2004)
(a) Hemoglobin HbS differs from hemoglobin HbA by the
substitution of Val for Glu in position 6 of the beta chain
(b) One altered peptide of HbS migrates faster towards the
cathode (–) than the corresponding peptide of HbA
(c) Binding of HbS to the deoxygenated HbA can extend the
polymer and cause sickling of the red blood cells
(d) Lowering the concentration of deoxygenated HbS can
prevent sickling
155. Sickle cell trait patient do not have manifestations of sickle
cell disease, because:
(AIIMS Nov 2001)
(a) 50% HbS is required for occurrence of sickling
(b) HbA prevents sickling
(c) 50% sickles
(d) HbA prevents polymerization of HbS
156. True about Sickle cell anemia are all except:
(a) Commonly seen in blacks
(PGI June 2004)
(b) RBC size is altered
(c) Valine is substituted for glutamic acid in beta chain of globin.
(d) Deletion of gene
(e) Target cell are present
157. True about beta-thalassemia trait is:
(PGI June 2004)
(a) Increased HbF
(b) Increased HbA2
(c) Microcytosis
(d) Severe anemia
(e) Target cell

MOST RECENT QUESTIONS


158. Sickle cell anemia is the clinical manifestation of
homozygous genes for an abnormal haemoglobin molecule.
The event responsible for the mutation in the b chain is:
(a) Insertion
(b) Deletion
(c) Non-disjunction
(d) Point mutation
159. Hemoglobin H disease is caused by deletion of:
(a) A single a globin gene
(b) Two a globin genes
(c) Three a globin genes
(d) All four a globin genes
160. All of the following aggravate sickling phenomenon in sickle
cell disease except:
(a) Higher concentration of HbS
(b) Higher concentration of HbF
(c) Lower concentration of HbC
(d) A fall in blood pH
161. Sickle cell anemia is due to:
(a) Presence of a structurally abnormal Hb
(b) Red cell enzyme deficiency
(c) Unknown multiple mechanisms
(d) Disturbance of proliferation and differentiation of stem cells
162. In sickle cell disease, the defect is in:
(a) a-chain
(b) b-chain
(c) g-chain
(d) Hb formation
163. Which of the following manifestations is common to sickle
cell anemia and thalassemia major?
(a) Autosplenectomy
(b) Bone marrow expansion in the calvarium
(c) Ineffective erythropoiesis
(d) Predisposition to Hemophilus influenzae infections
164. One of the common variants of sickle cell anemia frequently
marked by lesser degree of haemolytic anemia and greater
propensity for the development of retinopathy and aseptic
necrosis of bones is:
(a) Sickle cell trait
(b) Haemoglobin SC disease
(c) Sickle thalassaemia
(d) Sickle –Hb E disease
165. In α-thalassemia, which of the following is a finding?
(a) No β-chain
(b) Excess α-chain
(c) No α-chain
(d) Relative excess of β, γ, and δ chains
166. Ideally children with thalassemia should be transfused with:
(a) Packed RBC
(b) Platelet rich plasma
(c) Saline washed packed RBC
(d) Whole blood
167. All are seen in Thalasemia major except:
(a) Transfusion dependency
(b) Splenoheptatomegaly
(c) Ineffective erythropoiesis
(d) Macrocytic anaemia
168. Bone infarcts are seen in:
(a) Iron deficiency anemia
(b) Thalassemia
(c) Sickle cell anemia
(d) Hereditary spherocytosis
169. In sickle cell anemia defect is in which chain:
(a) Alpha chain
(b) Beta chain
(c) Both the chains
(d) None of these
170. A 18-year-old Afro American boy presenting with a non
healing ulcer of the foot, with recurrent pneumonia and
chronic hemolytic anemia. The peripheral blood erythrocytes
showed some peculiar appearance. Most likely cause is:
(a) Trinucleotide repeat
(b) Genomic imprinting
(c) Single amino acid base substitution
(d) Antibody to red cell membrane
171. Molecular pathogenesis of α thalassemia involves:
(a) Mutation in transcription sequence
(b) Gene deletion
(c) Codon termination mutation
(d) mRNA splicing defect
172. The primary defect which lead to stickle cell anemia is?
(a) An abnormality in porphyrin part of haemoglobin
(b) Substitution of valine by glutamate in the β-chain of HbA
(c) Replacement of glutamate by valine in β-chain of HbA
(d) A nonsense mutation in the β-chain of HbA
173. Sickle cell red blood cells have:
(a) Altered stability
(b) Altered functions
(c) Decreased oxygen carrying capacity
(d) Protective action against adult malaria
174. In a case of Plasmodium falciparum malaria, the peripheral
blood smear does not demonstrate trophozoites and
schizonts. The reason for this is:
(AIIMS Nov 2016)
(a) Apoptosis of red cells because of hemozoin pigments
(b) Lysis of red cells with malarial parasite
(c) Infested cells are trapped in the spleen
(d) Infested red blood cells stick to the capillaries
175. Which of the following is true about sickle cells?
(a) Sickling occurs both in heterozygous and homozygous
(b) Fetal hemoglobin facilitates sickling
(c) Sickling is reversible with oxygenation
(d) Sickling is associated with reduced MCHC
176. Which of the following is true about sickle anemia?
(a) Leucopenia
(b) Decreased ESR
(c) Microcardia (d) Ringed sideroblast
1. Ans. (b) Hereditary spherocytosis (Ref: Robbins 9/e p632)
Regarding Hereditary spherocytosis;
Direct quote from Robbins....‘this inherited disorder is caused by
intrinsic defects in the red cell membrane that render red cells
spheroid, less deformable, and vulnerab le to splenic
sequestration and destruction’
Other disorders with intrinsic defect in red cell membrane are
hereditary elliptocytosis and abetalipoproteinemia.
2. Ans. (c) Myoblast (Ref: Robbins 8th/85, 9/e p580)
Bone marrow cells include

• Hematopoietic stem cells include lymphoblast, myeloblast and


normoblast.
• Marrow stromal cell/multipotent stem cells (MSC) including myoblast,
osteoblasts, chondrocytes, adipocytes and endothelial cell
precursors. Myoblast is an example of MSC giving rise to muscle cells or
myocytes.

3. Ans. (d) CD 34 (Ref: Robbins 7th/670, 9th/590)

CD34 is expressed on pluripotent hematopoietic stem cells and progenitor


cells of many lineages

4. Ans. (c) Nutritional anemia


(Ref: Harrison 18th/454, 17th/359-361 and Ghai 6th/306) ...see
Table 7.1

Reticulocytes are nonnucleated spherical cells bigger than normal RBCs and
are polychromatic (having a blue color) due to the presence of free ribosomes
and RNA.

5. Ans. (b) Basophilic stippling; (d) Howell-Jolly bodies; (e)


Cabot ring (Ref: PJ Mehta 16th/372, T. Singh 1st/34)
• Romanowsky dyes are used for staining blood films. They
are made up of combination of acid and basic dyes. The
nucleus and neutrophilic granules are basophilic and stains
blue. Hemoglobin is acidophilic and stains red.
• Various modifications available are Leishman’s stain,
Wright’s stain, Giemsa and Jenner’s stain.
• Basophilic stippling, Howell-Jolly body and Cabot rings are
seen by Romanowsky stain.

• Basophilic stippling: These are small blue or black granules in red cells
seen in megaloblastic anemia, heavy metal poisonings, etc.
• Howell-Jolly Body: These are remnants of the nucleus seen as small,
round dark blue particles near the periphery of the cells; found in
postsplenectomy, asplenia and severe hemolytic anemia.

• Cabot ring: These are pale staining nuclear remnants in the form of rings
or figure of eight seen in hemolytic anemia, megaloblastic anemia,
leukemia and after splenectomy. These are arginine rich and acidophilic.
• Heinz bodies are denatured hemoglobin which does not stained with
Romanowsky stain. It is demonstrated by supravital stains such as crystal
violets. Reticulocytes also require Supravital staining.

6. Ans. (d) CD 34 (Ref: Robbins 7th/670, 9/e p590)


7. Ans. (d) Benign liver tumor (Ref: Robbins 9/e p331)
8. Ans. (a) MCV (Ref: Robbins 9/e p630)
9. Ans. (c) Megaloblastic (Ref: Robbins 9/e p645)
10. Ans. (a) Acute glomerulonephritis (Ref: Robbins 9/e p898)
11. Ans. (b) Spherocytosis (Ref: Robbins 9/e p633)
12. Ans. (d) Erythropoietin (Ref: Robbins 9/e p656)
13. Ans. (d) All of the above
(Ref: Robbins 8th/665; 7th/960, Harrison 18 Table 280 (5))
14. Ans. (a) Uremia (Ref: Tejinder Singh 1st/38)
15. Ans. (a) Abetalipoproteinemia
(Ref: Tejinder Singh 1st/38)
16. Ans. (b) Brilliant Cresyl blue (Ref: Robbins, 9/e 635)
• Reticulocytes are stained in living state in vitro so staining
with dyes like brilliant cresyl blue and new methylene blue
(Best stain) is referred to as supravital staining.
17. Ans. (a) Ferritin (Ref: Robbins, 9/e 650)
18. Ans. (a) Histidine residues
Harper mentions …‘Hemoglobin also functions in CO2 and proton
transport from tissues to lungs. Release of O2 from oxy Hb at the
tissues is accompanied by uptake of protons due to lowering of
the pKa of histidine residues.’
19. Ans. (b) Henoch Schonlein purpura
(Ref: Robbins, 9/e p526-527 8/e p666; 7/e p986-987)
All other options have decreased platelet count except Henoch
Schonlein purpura. Though it has the name purpura, but the
platelet count in this condition is normal. The skin
manifestations in HSP are due to small vessel vasculitis.
20. Ans. (c) 900... About 900 gQ of hemoglobin is present in the
circulating blood of an adult man.
21. Ans. (a) Lymphocyte (Read below)

• Lymphocyte is the longest livingQ white blood cell.


• Neutrophil is the most numerousQ white blood cell.

22. Ans. (c) –0.54° C (See below)


• The freezing point of normal human plasma averages -0.54 °CQ,
which corresponds to an osmolal concentration in plasma of 290
mOsm/LQ.
• The term tonicity is used to describe the osmolality of a solution
relative to plasma
• A 0.9% saline solution and 5% glucose solution is isotonic when
initially infused intravenously.

Also know how to calculate osmolality: medicine link!

Osmolality (mOsm/L) = 2[Na+] (mEq/L) + 0.055 [Glucose]


(mg/dL) + 0.36[BUN] (mg/dL)

23. Ans. (b) 2:1 (Read below)


• Normal albumin-globulin ratio is 1.8 : 1 to 2 : 1Q.
• Synthesis of albumin exclusively occurs in liver but many
globulins (immunoglobulins) are synthesized by B-
lymphocyte.
Conditions with altered albumin globulin ratio
High Albumin Globulin Ratio Low Albumin Globulin Ratio
• Hypothyroidism • Overproduction of globulins in
• Hypogammaglobulinemia conditions like multiple myeloma,
• Leukemia chronic infections and in some
• Glucocorticoid excess autoimmune diseases.
• Under production of albumin in
conditions like liver cirrhosis,
malnutrition and nephrotic
syndrome.

24. Ans. (b) Contractile protein


(Ref: Harsh Mohan 5th/ 177, The Circulating Platelet/ 215-8)
Thrombosthenin is a contractile protein in platelets that is active
in the formation of blood clots.
25. Ans. (c) Cyanmethemoglobin method (Read below)
Hemoglobin estimation
Visual Colorimetric Sahli’s (acid hematin Most popular
method method) method
Alkali hematin method
Photocolorimetric Cyanmethemoglobin method Most accurate
method and currently
used

26. Ans. (c) 4


Hemoglobin is a globular molecule made up of four subunits. Each
subunit contains a heme moiety conjugated to a polypeptide.
Heme is an iron-containing porphyrin derivative. So, hemoglobin
= 4 globins + 4 heme groups
Since each heme molecule contains an iron, so total iron atoms
present in hemoglobin are 4 in number.
70% of the iron in the body is in hemoglobin, 3% in myoglobin, and
the rest in ferritin, which is present not only in enterocytes, but
also in many other cells.
27. Ans. (d) ESR (Ref: A Textbook of Hematology p133)
28. Ans. (b) Labile factor
If whole blood is allowed to clot and the clot is removed, the remaining
fluid is called serum. Serum has essentially the same
composition as plasma, except that:
• Fibrinogen (factor I) and clotting factors II, V, and VIII have been
removed and.
• Has higher serotonin content because of the breakdown of platelets
during clotting.

Also know that factor 5 is called as labile factorQ.


29. Ans. (a) Bone marrow (Ref: Robbins 8/e p590, 9/e p580)
By birth, marrow throughout the skeleton is hematopoietically active
and hepatic hematopoiesis dwindles to a trickle, persisting only in
widely scattered foci that become inactive soon after birth.
30. Ans. (c) RBCs to whole blood (Ref: Robbins, 9/e 631)
Haematocrit (also known as packed cell volume) is the volume
percentage (%) of red blood cells in blood.
31. Ans. (c) Myelophthisic type (Ref: Robbins 9/e p655)
• Myelophthisic anemia describes a form of marrow failure in which
space-occupying lesions replace normal marrow elements. The
commonest cause is metastatic cancer, most often carcinomas
arising in the breast, lung, and prostate.

32. Ans. (b) No nucleus (Ref: Wintrobe 11th/200)


33. Ans. (b) 60-80 days (Ref: Oski’s Hematology /28)
• Premature infant: 35-50 days
• Term infant: 60-70 days
• Adult: 100-120 days

34. Ans. (d) Hereditary spherocytosis (Ref: Robbins 9th/633 )


35. Ans. (b) <34
(Ref: Park’s PSM textbook/Nutrition and health)
At all ages the normal MCHC should be 34; values below that
indicate that red cells are hypochromic, which occurs in iron
deficiency anaemia.
36. Ans. (a) Increased reticulocyte count
(Ref: Robbins 9/e 645, 8th/655, Wintrobe’s 12th/1151-3)
Direct quote from Robbins … ‘The reticulocyte count is low’.
Please be clear of the concept friends that the reticulocyte count is
increased when the megaloblastic anemia is being treated with
vitamin B12 and folate supplementation i.e. after the initiation
of the treatment in these patients.
• Nucleated red cell progenitors occasionally appear in the circulating
blood when anemia is severe.
• The derangement in DNA synthesis causes most precursors to
undergo apoptosis in the marrow (ineffective hematopoiesis) and leads
to pancytopenia. The anemia is further exacerbated by a mild degree of
red cell hemolysis. This leads to raised bilirubin.
• Wintrobe mentions … ‘Mild reversible splenomegaly is present in
megaloblastic anemia’.

37. Ans. (d) Myelofibrosis (Ref: Harrison 17th/661, 674, 646)


The findings in the given question are:
Anemia (Hb = 6g)
Reduced leukocyte count (TLC= 1200)
Reduced platelet count (60000)
Reduced MCV (12fL)
• Normal value of MCV is 80-100 fL. It is the measure of size
of RBC. Reduced MCV means microcytic and increased
MCV means macrocytic RBCs. Normal TLC is 4000-11000,
normal platelet count is 1,50,000 to 4,50,000 and normal Hb
is above 12 g/dL.
• Now, considering the options one by one:
• Aplastic anemia has reduced RBC, WBC as well as platelet counts.
Anemia is normocytic normochromic, thus it can be easily ruled out.
• Paroxysomal nocturnal hemoglobinuria (PNH) typically presents
with anemia which is usually normomacrocytic. If MCV is high, it is
due to reticulocytosis. Neutropenia and thrombocytopenia may or
may not be present. Therefore, this option also cannot be the answer.
• Megaloblastic anemia presents with raised MCV. There may be
leukopenia as well as thrombocytopenia. Severity of these changes
parallels the degree of anemia.
• Myelofibrosis usually presents with anemia, leukopenia and
thrombocytopenia. Mostly anemia is normocytic but in 30% cases
microcytic anemia can be present.

Based on the above discussion, the most probable answer is


‘Myelofibrosis’.
However, the value of MCV given is 12 fL which is practically not
possible. Normal value of MCV is 80-100 fL. MCV < 50 fL is
considered to be extremely low. Therefore, one possibility is that
it may be 112 fL and due to typographical error in the question,
written as 12 fL. If this is the case, then the answer will become
megaloblastic anemia.
As we cannot say for sure that it is a printing mistake, so we will go for
‘Myelofibrosis’.
38. Ans. (a) ↑MCV.
(Ref: T. Singh 1st/35, Harrison 17th/357, Robbins 9/e 629-630)
MCV > 100fL indicates macrocytosis.
39. Ans. (a) Megaloblastic precursors are present in bone
marrow; (b) Mean corpuscular volume is increased; (c)
Serum LDH is increased
(Ref: Robbins 9/e 645, Harrison’ 17th/645-646)
• Megaloblastic anemia is a maturation disorder of red cells
• Cells are macrocytic and hyperchromic. Anisocytosis
and hypersegmented neutrophils are also seen.
• Bone marrow shows hypercellularity, erythrocyte precursors
at different stages of development is found. Increased
megaloblast causes ineffective erythropoiesis.
• In severe anemia as many as 90% of RBC precursors may
be destroyed before their release into the circulation
(Normal 10-15%). Thereby increased unconjugated
bilirubin and lactic acid dehydrogenase.
• Target cells result due to increased ratio of RBC surface
area to volume, seen in hemoglobin disorders, thalassemia,
liver disease. Reticulocytes, platelet count and leukocyte
count decreased.
40. Ans. (b) Copper deficiency (Ref: Harrison 17th/449)
Thiamine, pyridoxine and Bl2 deficiency leads to megaloblastic
anemia.

Copper deficiency is manifested by hypochromic normocytic anemia,


osteopenia, depigmentation, mental retardation and psychomotor
abnormalities.

Classification of megaloblastic anemia


Drugs Diseases Unknown etiology

Cobalamin Purine and Hereditary Refractory megaloblastic


deficiency Pyrimidine orotic aciduria anemia.
Folic acid antagonists Lesch-Nyhan Di-Gueglielmo syndrome
deficiency Others-procarbazine, syndrome Congenital
zidovudine dyserythropoietic anemia
hydroxyurea,
acyclovir
• Causes of anemia in Liver diseases are gastrointestinal
bleeding, folate deficiency and hypersplenism.
41. Ans. (a) Fish tapeworm infestation; (c) Gastrectomy; (d) Ileal
resection
(Ref: Robbins 7th/640, 9/e 645)
The other two cause megaloblastic anemia by interfering with folic acid
metabolism.
42. Ans. (c) Pure red cell aplasia
(Ref: T. Singh 1st/140; Robbins 7th/648, 636, 9/e 642)
PNH is the only hemolytic anemia caused by an acquired intrinsic
defect in the cell membrane. PNH arises in the setting of aplastic
anemia and these patients are at increased risk of developing
acute myelogenous leukemia.
Aplastic anemia may progress to PNH and MDS.
43. Ans. (d) Cholestatic jaundice (Ref: Robbins 9/e 645)
44. Ans. (d) Thalassemia (Ref: Robbins 9/e 639)
45. Ans. (a) Myelofibrosis (Ref: Robbins 9/e 620-621)
46. Ans. (b) Folic acid deficiency (Ref: Robbins 9/e 648)
47. Ans. (d) Hypothyroidism
(Ref: Tejinder Singh 1st/66, Robbins 9/e 645)
48. Ans. (a) Thymoma (Ref: Robbins 9/e 627)
49. Ans. (b) Diphyllobothrium latum (Ref: Robbins 9/e 648)
50. Ans. (c) Anemia occurs after 3-4 months of poor absorption
(Ref: Robbins 9/e 648, 8th/657; 7th/682)
51. Ans. (b) Folic acid deficiency
(Ref: Harsh Mohan 6th/308, Robbins 9/e 647)
52. Ans. (c) Megaloblastic (Ref: Robbins 9/e 645)
53. Ans. (c) Megaloblastic anemia (Ref: Robbins 9/e 645)
54. Ans. (d) Postsplenectomy
(Ref: Robbins 9/e 627, 633, 8th/646, 7th/627, Harrison 17th/375)
55. Ans. (c) Diphyllobothrium latum infestation
(Ref: Robbins 9/e 648, 8th/655)
56. Ans. (d) Thymoma (Ref: Robbins 9/e 627, 8th/664)
In the rare pure red cell aplasia, the erythroid marrow elements are
absent or nearly absent, while granulopoiesis and thrombopoiesis
remain unaltered. This condition occurs in both primary and
secondary forms, both of which are thought to be related to
autoimmune destruction of erythroid precursors. There is a
specific association between thymic tumors (thymoma) and
autoimmune hematologic diseases, specifically including pure red
cell aplasia.
57. Ans. (a) Aplastic anemia (Ref: Robbin 9/e p653)
Aplastic anemia refers to a syndrome of chronic primary hematopoietic
failure and attendant pancytopenia (anemia, neutropenia, and
thrombocytopenia).
58. Ans. (b) Human parvo virus B19 (Ref: Robbin 9/e p460)
Parvovirus B19 causes erythema infectiosum or “fifth disease of
childhood” in immunocompetent older children. Parvovirus B19
has a particular tropism for erythroid cells, and diagnostic viral
inclusions can be seen in early erythroid progenitors in infected
infants.

Parvovirus infection in pregnant women is associated with hydrops fetalis due


to severe fetal anemia, sometimes leading to miscarriage or stillbirth.

59. Ans. (c) Increased reticulocyte count (Ref: Robbin 8/e p655,
Wintrobe’s 12/e p1151-3)
Repeat from AIIMS Nov 12 see earlier explanation of answer 32
60. Ans. (a) Beta thalassemia trait (Ref: Robbins 9/e 641)
Hemoglobin electrophoresis usually reveals an increase in HbA2
(α2δ2) to 4% to 8% of the total hemoglobin (normal, 2.5% ±
0.3%), which is a reflection of an elevated ratio of δ-chain to β-
chain synthesis. HbF levels are generally normal or occasionally
slightly increased.
61. Ans. (c) Chronic renal failure….. read below
The same question was asked in different sets with different choices.
Choices a,b,d are examples of hemolytic anemias and hence the
answer by exclusion is chronic renal failure. CRF has low
erythropoietin levels due to less production and has normocytic
normochromic anaemia.
62. Ans. (c) Vitamin B12 absorption (Ref: Robbins 9/e 648)
The Schilling test is performed to determine the cause for cobalamin
malabsorption. Since cobalamin absorption requires multiple
steps, including gastric, pancreatic, and ileal processes, the
Schilling test also can be used to assess the integrity of those
other organs.
Differential Results of Schilling Test in Several Diseases with
Cobalamin (Cbl) Malabsorption
With With
Intrinsic Pancreatic After 5 Days of
58Co-Cbl Factor Enzymes Antibiotics
Pernicious Reduced Normal Reduced Reduced
anemia
Chronic Reduced Reduced Normal Reduced
pancreatitis
Bacterial Reduced Reduced Reduced Normal
overgrowth
Ileal disease Reduced Reduced Reduced Reduced

63. Ans. (a) Anaemia of chronic disease


(Ref: Robbins 9/e 652-653, Wintrobe 12th/1221-2)
Looking at the data one by one friends, we infer that:
• We have an old patient with chronic inflammatory condition.
In addition,
Inference in our
Parameter with value in question Normal range patient
Hemoglobin 4.5gm/dl 13-17g/dl Decreased
Platelet count 2 lakh/ml 1.5-4.5lakh/ml Normal
TLC 6000/mm3 4000-11000/mm3 Normal
Serum ferritin 200 µg/L 15-300 µg/L Normal
Serum iron 30 mg/L 50-150 µg/L Reduced
TIBC 280 ng/L 300-400 mg/L Reduced

Final conclusion, decreased serum iron, increased storage iron


i.e. serum ferritin, decreased serum transferring and decreased
total iron binding capacity suggest the diagnosis of anaemia of
chronic disease.
64. Ans. (b) Thalassemia minor
(Ref: Hematology by Renu Saxena 1st/174)
Thalassemia major patient presents with severe anemia and cannot
survive without blood transfusion, so this option can be easily
ruled out. For other options, Mentzer index is useful for
differentiating between thalassemia minor and iron deficiency
anemia.

Mentzer index is calculated as MCV/RBC count. Its value is >13 in iron


deficiency anemia and <13 in thalassemia minor. For our given question, the
value on calculation comes out to be 55/4.5 = 12.22.

As the value is < 13, so it is a case of thalassemia minor.


65. Ans. (a) Iron deficiency anemia
(Ref: Robbins 8th/651, 9/e 652, T.Singh 1st/34)
• Decreased hemoglobin with the clinical features of fatigue
and weakness is diagnostic of anemia
• MCV is 70 fl, so, microcytosis is present (normal MCV is 82-
96fl)
• MCH is 22pg, so, decreased MCH is suggestive of
hypochromic anemia (normal MCH is 27-33pg)
• Red cell distribution width (RDW) is the coefficient of
variation of size of RBCs. Normal value is 11.5-14.5. It is an
indicator of anisocytosis which may present in IDA as well
as hemolytic anemias.
• In early iron deficiency anemia, RDW increases along with
low MCV while in beta thalassemia trait, RDW is normal with
low MCV, thus distinguishing from each other.

Increased reticulocytosis is a feature of treatment with iron therapy in a


patient of iron deficiency anemia.

66. Ans. (b) Myelodysplastic syndrome


(Ref: Robbins 9/e 615, Wintrobe’s 10th/1022-25)
Sideroblastic anemia can be hereditary (due to decreased ALA
synthase activity) or acquired (secondary to leukemias,
myelodysplastic syndrome, alcoholism, copper deficiency,
pyridoxine deficiency or lead poisoning).
67. Ans. (c) Thalassemia minor (Ref: Robbins 9/e 641)
• Thalassemia major patient presents with severe anemia and
cannot survive without blood transfusion, so this option can
be easily ruled out.
• This cannot be a case of megaloblastic anemia because in
megaloblastic anemia, M.C.V. is increased (> 100 fL)
• So, we are left with two options, i.e. Iron deficiency anemia
and thalassemia minor. Both of these presents with
microcytic hypochromic anemia.
The key point in the differential diagnosis of these two conditions is the
RBC count.
• In thalassemia minor, the RBC count is near normal and
only the hemoglobin is reduced. In this condition the R.B.C.
count is not reduced as much as the hemoglobin and
hematocrit in fact it is usually normal. This due to the fact
that the marrow can keep on producing the cell at normal
rate but it cannot fill them with hemoglobin. Hence, the
hemoglobin is low and the empty cells occupy less space
thus lowering the hematocrit relative to the erythrocyte
count.
• On the other hand, in Iron deficiency anemia, the RBC
production is also impaired.
There are many indices to differentiate between iron
deficiency anemia (IDA) and beta-thalassemia (BT) Applying
mentzer index as discussed in a previous question, the
value is < 13, so it is a case of thalassemia minor:
68. Ans. (a) Inhibition of enzymes involved in heme
biosynthesis (Ref: Robbins 9/e 411, Tejinder Singh 1st/147)
• Lead inhibits the enzymes δ aminolevulinic acid
dehydrase, red cell pyrimidine 5’ nucleotidase and
ferrochelatase which are involved in the synthesis of heme.
• Deficiency of heme causes microcytic hypochromic anemia
because heme is an integral part of hemoglobin and
hemoglobin deficiency causes microcytic hypochromic
anemia.
69. Ans. (b) Sideroblastic anemia
(Ref: Harrison 17th/631-32 (t) 98-4)
The hematological findings suggest the diagnosis of sideroblastic
anemia.
70. Ans. (a) ↓ Erythropoietin; (b) ↓ RBC survival; (c) ↓ folate; (d)
Bone marrow hypoplasia; (e) Iron deficiency
(Ref: Harrison 17th/633-634, 18th/e (t) 280 (5))
71. Ans. All
(Ref: Robbins 9th/651, Harrison’ 19th/Table 335-3)
72. Ans. (a) Thalassemi(a); (c) Anemia in chronic disease; (e)
Megaloblastic anemi(a)
(Ref: Harsh Mohan 6th/302, 307)
• Increased bone marrow iron is seen in:
– Sideroblastic anemia
– Anemia of chronic disease
– Megaloblastic anemia
– Pernicious anemia
– Thalassemia.
73. Ans. (a) Thalassemia; (b) Hb Lepore; (c) Hb Barts.
(Ref: Harsh Mohan 6th/323-324)
Causes of microcytosis (MCV < 80fl)
• Iron deficiency anemia
• Thalassemia
• Sideroblastic anemia
• Anemia of chronic disease
In Hb Barts, all the four a chain genes are deleted resulting in
formation of Barts Hb.

In Hb Lepore, there is nonhomologus fusion of b and d genes and forms an


abnormal hemoglobin with total absence of normal b chain. It is one of the
form of b thalassemia minor.

74. Ans. (a) Microcytic hypochromic anemia; (d) Bone marrow


iron decreased earlier than serum iron
(Ref: Robbins 7th/645, de Gruchy’s 5th/42)
75. Ans. (d) S. ferritin ↑ (Ref:
Harrison 17th/632-4, Robbins 9/e 625-653, 8th/662)
76. Ans. (d) Persistently raised HbF
(Ref: Wintrobes 11th/1326; Nelson 17th/1630)
It is a case of persistently raised HbF.
Characteristics
• There is persistence of fetal Hb in adult life so that almost
whole of the Hb of patient is HbF.
• Patient remains asymptomatic even without blood
transfusion.
• No anemia or splenomegaly seen.
Beta-globin genes HbA HbA2 HbF
Normal Homozygous b 97-99% 1-3% < 1%
Homozygous b0 0% 4-10% 90-96%
Thalassemia major Homozygous b+ 0-30% 0-10% 60-100%
(mild)
Thalassemia Homozygous b+ 0-30% 0-10% 60-100%
intermedia (mild)
Homozygous b0 80-95% 4-8% 1-5%
Thalassemia minor
Homozygous b+ 80-95% 4-89% 1-5%

• From the values given in question it can be thalassemia


major or thalassemia intermedia.
• In thalassemia major patient presents with severe hemolytic
anemia at the age of 6 months and cannot survive without
blood transfusions.
• In thalassemia intermedia patient can survive without
transfusion but they are not asymptomatic.
77. Ans. (c) Absence of intrinsic factor (Ref: Robbins 9/e 645)
Absence of IF causes Pernicious anemia which is an example of
megaloblastic anemia.
78. Ans. (c) Rheumatoid arthritis (Ref: Robbins 7th/639-640)

Note: In anemia of chronic disease (of Rheumatoid Arthritis, TB, UTI, etc), the
red cells are mainly normocytic; normochromic red cells. In some cases, red
cells may be hypochromic. So, we would go with RA as the best answer in
this question.

79. Ans. (a) Microcytic hypochromic anemia


(Ref: Hematology by Tejinder Singh/83)
Explained in text.
80. Ans. (a) Congenital spherocytosis (Ref: Robbins 7th/625)
Hereditary spherocytosis – Small RBCs are seen without central
pallor (normal RBCs have central 1/3rd pallor). It is also seen
conditions where spherocytes are present in peripheral blood
like:
• Hereditary spherocytosis • Burns
• Autoimmune hemolytic anemia • G6PD deficiency
• Cirrhosis • ABO incompatibility
• Clostridial sepsis

81. Ans. (d) Megaloblastic anemia


(Ref: Robbins 9/e 645, 7th/639/643/626/629)
• Megaloblastic anemia: Two principal types
• Pathology: Defective DNA synthesis and diminished
erythropoiesis
• Morphology - Anisocytosis (various size + shape),
Macrocytosis (MCV > 100 fl), MCHC is normal and,
Macropolymorphonuclear (hyper segmented) neutrophils
82. Ans. (c) ↓ Serum ferritin level(Ref: Robbins 9/e 652-653)
83. Ans. (a) Iron deficiency (Ref: Robbins 9/e 652-653)
84. Ans. (c) TIBC is decreased
(Ref: Robbins 9/e 652-653, 8th/662; Harrison 17th/632)
85. Ans. (b) Transferrin (Ref: Robbins 9/e 650)
86. Ans. (d) Increased mean corpuscular volume
(Ref: Robbins 9/e 652, 8th/660-661, 7th/645)
87. Ans. (c) Serum ferritin
(Ref: Robbins 9/e 652)
88. Ans. (d) Salivary gland (Ref: Robbins 9/e 848-849)
89. Ans. (a) Melanin (Ref: Robbins 9/e 849)
90. Ans. (a) Ferritin (Ref: Robbins 9/e 650)
91. Ans. (c) Iron deficiency anemia (Ref: Robbins 9/e 652)
92. Ans. (a) Reticulocytosis…explained earlier
(Ref: Robbins 9/e 652, 8/e p641, 7/e p624)
93. Ans. (d) Macrocytic anemia (Ref: Robbins 9/e 411)
• Lead inhibits the activity of enzymes involved in heme synthesis; δ-
aminolevulinic acid dehydrataseQ and ferrochelataseQ (BIOCHEMISTRY
NEET QUESTION INFO)
• Ferrochelatase catalyzes the incorporation of iron into protoporphyrin, and
its inhibition causes a rise in protoporphyrin levels as well as
appearance of scattered ringed sideroblastsQ. The elevated levels of
protoporphyrin may appear in the urine of an individual.
• There is a distinctive punctate basophilic stipplingQ of the red cells and
the presence of microcytic, hypochromic anemiaQ.
• Also know that in lead poisoning is associated with reduction in uric acid
excretion which can lead to gout (“saturnine gout”Q)

94. Ans. (a) Anaemia of chronic disease.... see text for details
95. Ans. (b) Hookworm (Ref: Robbins 9/e 651)
Hookworm infestation can cause chronic blood loss and therefore
may cause iron deficiency anemia.
96. Ans. (a) Lead (Ref: Robbins 9/e 411)
Lead is associated with sideroblastic anemia…..details are discussed
in a separate question.
97. Ans. (a) RBCs (Ref: Harrison 17/e p77)
98. Ans. (b) Reticulocytosis (Ref: Robbins 9/e 652)
In uncomplicated cases, oral iron supplementation produces an
increase in reticulocytosis in about 5-7 days that is followed
by a steady increase in blood counts and normalization of
red cell indices.
99. Ans. (d) Cutaneous porphyria
(Ref: Hematology: Diagnosis and Treatment p467)
Sideroblastic anemia is associated with the following
Hereditary: X linked, autosomal recessive, autosomal dominant
Acquired: previous chemotherapy, irradiation, myelodysplasia,
myelproliferative disorders
Drugs: alcohol isoanizid, choramphenicol, pyridoxine deficiency, lead
poisoning
Rare causes: copper deficiency, zinc overload, hypothermia,
erythropoetic porphyria
Hereditary syndromic: Pearson syndrome, thiamine responsive
megaloblastic anemia,

100. Ans. (a) Mucosal cell iron stores (Ref: Robbins 9/e 650)
Rate of iron uptake is dependent on the levels of a protein called
hepcidin. This protein functions to regulate (inhibit) iron transport
across the gut mucosa, thereby preventing excess iron
absorption and maintaining normal iron levels within the body.
Hepcidin also inhibits transport of iron out of macrophages
(where iron is stored).
• Mutation of the gene coding for hepcidin is implicated in the
causation of hemochromatosis.
101. Ans. (a) Duffy blood group (Ref: Robbins 9/e 391)
Conditions providing protection against malaria with the
reasons

• Sickle cell disease: P falciparum can not multiply properly in the presence
of HbS
• α and β thalassemia:
• Absence of duffy blood group: duffy antigen is required for parasite to
enter the RBCs
• G6 PD deficiency: G6PD is required for respiration of plasmodium

102. Ans. (b) DMT1 (Ref: Robbins 9th/650)


Iron absorption is regulated by hepcidin, a small circulating peptide that
is synthesized and released from the liver in response to
increases in intrahepatic iron levels. Hepcidin inhibits iron transfer
from the enterocyte to plasma by binding to ferroportin and
causing it to be endocytosed and degraded.
Fe2+ iron is then transported across the apical membrane by divalent
metal transporter 1 (DMT1). However, as DMT-1 also transports
other divalent metal ions like Cu, Zn, Cd, it is non specific. This is
the answer of choice here.
103. Ans. (c) Coombs test (Ref: Robbins 9/e 643, 8th/653)
The presence of spherocytes can be seen in the following
conditions:

• Hereditary spherocytosis
• Autoimmune hemlolytic anemia
• G6PD deficiency
• Infections
• Burns
• Hemolytic disease of new born
• PNH is not a cause for the presence of spherocytes; so, no
test for this condition is required.
• Osmotic fragility is increased with spherocytes. So, it does
not add anything to our existing information about the
disease causing spherocyte formation.
• Reticulocyte count is expected to be elevated in the setting
of haemolytic anemia (suggested by jaundice and pallor).

Coombs test going to be positive in autoimmune hemolytic anemia whereas


negative in hereditary spherocytosis. Thus, a differentiation between these
conditions can be done.

104. Ans. (c) Defective GPI anchor


(Ref: Robbins 9/e 642, 8th/65, Harrison 17th/660)
Ham’s acidified serum test is used for the diagnosis of paroxysmal
nocturnal hemoglobinuria (PNH). In this condition, defective GPI
anchor prevents the attachment of CD55, CD59 and C8 binding
protein.
105. Ans. (d) Paroxysmal nocturnal hemoglobinuria (Ref
Wintrobe’s 12th/1007, 1044-5, Nathan and Oski’s Hematology
of infancy and childhood 7th/45, Guha’s NEONATOLOGY
Principles and Practice vol 2, 3rd/910)
• G6PD and Hereditary spherocytosis can cause anemia and
jaundice since birth and are causes of haemolytic anemias.
So, they are easily ruled out.
• Friends I got some worthy information after spending few
days and multiple book searching which should bring a
smile on your face.
• These are the lines from Oski’s Hematology “β chain
mutations generally produce no clinical symptomatology in
the newborn period. This does not mean that chain variants
are never a problem in the neonate. Sickle cell
hemoglobinopathies are the most commmonly
encountered β chain variants in the newborn period.
Several cases of homozygous sickle cell disease have
been seen in neoanates. In infants in whom sickle cell
anemia has been diagnosed in first days of life because of
some specific symptoms specifcally jaundice, fever, pallor,
respiratory distress and abdominal distension”.
• ‘Hyperbilirubinemia appears to be more common in
newborns with sickle cell anemia.’.. Nathan and Oski
7th/45
• ‘β chain defects do not manifest in newborn. An important
exception is sickle cell disease which can manifest in newborn as up
to 30% of hemoglobin at birth may be adult hemoglobin. Sickling and
hemolytic jaundice may result and is best treated with exchange
transfusion.’… Guha 3rd/910, Jaypee
Therefore the answer of exclusion is option ‘d’ .. paroxysmal nocturnal
hemogloninuria. Its mean age of presentation is in the 30’s. Even
after extensive search, I could not get hold of any material which
supports presence of symptomatic PNH in infancy.
106. Ans. (c) Idiopathic thrombocytopenic purpura
(Ref: Robbins 9/e 658-659, 8th/668, Harrison 17th/367)
Primary or idiopathic ITP has two clinical subtypes: acute and chronic.
Both of them are autoimmune disorders in which platelet
destruction results from the formation of antiplatelet
autoantibodies. The opsonized platelets are rendered susceptible
to phagocytosis by the cells of the mononuclear phagocyte
system.
Option ‘a’: The triad of hemolysis, pancytopenia and
thrombosis is unique to this condition. Thrombosis is the
leading cause of death in individuals with PNH….Robbins 8th/653
Option ‘b’: DIC is an acute, subacute, or chronic
thrombohemorrhagic disorder occurring as a secondary
complication in a variety of diseases. It is characterized by
activation of the coagulation sequence that leads to the formation
of microthrombi throughout the microcirculation of the body..
Robbins 8th/673
Option ‘d’: Heparin Induced Thrombocytopenia can be two
types; type I thrombocytopenia which occurs rapidly after onset of
therapy, is moderately severe, clinically insignificant and may
resolve despite continuation of heparin therapy. Type II
thrombocytopenia is more severe and occurs 5 to 14 days after
initiation of therapy. It can, paradoxically, lead to life-threatening
venous and arterial thrombosis.
Why do we have paradoxical thrombosis in HIT type II?
It is caused by an immune reaction against a complex of heparin and platelet factor
4 (a normal component of platelet granules). The attachment of antibody to
platelet factor 4 produces immune complexes that activate platelets,
promoting thrombosis even in the setting of marked thrombocytopenia.

Additional important features of Heparin Induced


Thrombocytopenia

• Platelet count < 100,000/μL or decreased by > 50%.


• Starts 5-10 days after starting heparin.
• More common with unfractionated heparin (than LMW heparin), Surgical
patients (than medical patients) and females (than males)
• Venous thrombosis is more common than arterial.

107. Ans. (c) Glycosylphosphatidylinositol (GPI)


(Ref: Robbins 9/e 642, 8th/65, Harrison 17th/660)
108. Ans. (d) Donath-Landsteiner antibody (Ref:
Robbins 9/e 644, 7th/407-408; Harrison 16th/611-614)
Cold hemagglutinin is associated with more commonly IgM or rarely
IgG antibodies. These are not to be confused with anti-IgM or
anti-IgG antibodies given as other options in the question. An
important example of cold hemagglutinin disease is paroxysmal
cold hemoglobinuria. For details, see text
109. Ans. (c) Glycophorin C
(Ref: Harrison 17th/653-655, Robbins 9/e 632, 8th/642-644, 7th/625)
• Normally, RBC membrane consists of a protein spectrin, ion
transporter called band 3 of membrane with the help of
ankyrin and band 4.2.
• Mutations in glycophorin A has not been reported.
110. Ans. (c) CLL (Ref: Harrison 17th/693, Robbins 9/e 643)
Leukemias especially of the CLL type are associated with warm
autoimmune hemolytic anemia.
111. Ans. (d) Metallic cardiac valves
(Ref: Rubin’s pathology 5th/878, Nathan and Oski hematology
7th/643, Goljan pathology edn 2013/ 314)
Microangiopathic hemolytic anemia is associated with
fragmentation of the red cells in the microcirculation. It is
associated with antiphospholopid antibody syndrome... NMS
Medicine 5th/348

Pathologically, macroangiopathic hemolytic anemia is associated with


prosthetic cardiac valves..

Macroangiopathic hemolytic anemia may be caused by:


• Direct red cell trauma from abnormal valvular surface: prosthetic valve grafts,
tight aortic stenosis and synthetic vascular grafts.
• Large vessel disorders that may cause shearing of red cells: cavernous
hemangioma (Kasabach Merrit syndrome),
• Other causes include coarctation of aorta, ruptured sinus of valsalva, ruptured
chordae tendinae and aortic aneurysm.

Causes of Microangiopathic hemolytic anemia

1. Vasculitis like microscopic 5. Scleroderma


polyangiitis 6. Thrombotic thrombocytopenic purpura
2. Malignant hypertension (TTP) and Hemolytic uremic Syndrome
3. Eclampsia (HUS)
4. Renal graft rejection 7. DIC
8. March hemoglobinuria

112. Ans. (b) Conjugated and unconjugated bilirubin


(Ref: Robbins illustrated, 9/e 853, 8th/840-841, 7th/886-887)
• It is a case of erythroblastosis fetalis. (type II Hypersensitivity
reaction)
• In erythroblastosis fetalis there is excessive breakdown of
RBC’s leading to increased production of bilirubin in the blood.
This increased bilirubin is predominantly unconjugated but
the level of conjugated bilirubin will also increase because of
compensatory increase in bilirubin conjugation process by the
liver. So, both unconjugated bilirubin and bilirubin glucuronides
may accumulate systemically and deposit in tissues, giving
rise to the yellow discoloration of jaundice. This is particularly
seen in yellowing of the sclera (because of presence of
elastin fibers).
113. Ans. (a) Teardrop and Burr cells (Ref: Robbins 9/e 620)
• Teardrop cells also known as dacryocytes are seen in
myelofibrosis. Rest of the features are seen in hemolytic
anemia.
114. Ans. (b) Acute G6PD deficiency; (e) PNH
(Ref: Robbins’ 7th/624, 625, 9/e 631-632)
Intravascular hemolysis occurs due to disruption of red cell
membrane in circulation. The RBCs are damaged mechanically,
by complement fixation, malaria, toxins and drugs. It is seen in:
Acute G6PD deficiency and PNH
• In Sickle cell disease and hereditary spherocytosis, RBC
destruction occurs in spleen (extravascular hemolysis)
• Thalassemia is a hemoglobinopathy.
115. Ans. (a) HUS; (c) Malignant hypertension; (e) TTP

(Ref: Robbins’ 7th/638, 9/e 630)


Prosthetic valves cause Macroangiopathic hemolytic
anemia.

116. Ans. (a) Hemoglobin C; (c) Hereditary spherocytosis


(Ref: de Gruchy’s 5th/184, Robbins 9/e 632)
117. Ans. (a) Microangiopathic hemolytic anemia; (b) DIC; (d)
Malignant hypertension; (e) HELLP syndrome
(Ref: de Gruchy’s 5th/209 , Wintrobes 11th/1236, Robbins 9/e 630)
Schistocytosis or fragmented RBCs are found in
Thalassemia Severe burn DIC

Hereditary elliptocytosis Microangiopathic Malignant


hemolytic anemia hypertension

Megaloblastic anemia Iron deficiency anemia HUS

HELLP syndrome

118. Ans. (a) Glucose-6 phosphate dehydrogenase deficiency


(Ref: Robbins 7th/624, 628, 9/e 631)
119. Ans. (b) Reticulocytosis seen; (c) Smaller size; (e) Always
associated with ↑ MCHC (Ref: Robbins 9/e 633)
Hereditary spherocytosis is an usually autosomal dominant condition in
which the ratio of surface area to volume decreases and hence
the RBC becomes rounded. Spherocytes are small densely
staining RBC without central pallor. Mean cell Hb concentration
(MCHC) is always increased in this disease because the size is
decreased. Reticulocytosis would be seen because HS is an
important cause of hemolytic anemia.
RBCs are macrocytic; hyperchromic and show anisocytosis.
Hypersegmented neutrophils are also seen.
120. Ans. (a) Thrombotic Thrombocytopenic purpura; (b)
Hemolytic uremic syndrome; (d) DIC
(Ref: Robbins 7th/638, 9/e 630)
• In Henoch-Schonlein purpura, there is hematuria and
palpable purpura.

DIC is the commonest cause of Microangiopathic hemolytic anemia.

121. Ans. (b) Cold agglutination


(Ref: Harrison 17th/660, Robbins 7th/657, Robbins 9/e 644)
Paroxysomal cold hemoglobinuria is characterized by the
presence of Donath-Landsteiner antibody. It has anti-P
specificity and bind to red cells only at low temperature
(Optimally at 4°C)
122. Ans. (d) Spectrin (Ref: Robbins 9/e 632, 8th/642)
Ideal answer is ankyrin (most commonly) but in the given options,
spectrin is the best option.
123. Ans. (b) Ankyrin deficiency (Ref: Robbins 9/e 632)
124. Ans. (d) Increased platelets (Ref: Robbins 9/e 642)
125. Ans. (a) SLE (Ref: Robbins 9/e 642-644)
126. Ans. (a) Mycoplasma infection (Ref: Robbins 9/e 643-644)
127. Ans. (a) CD 59 (Ref: Robbins 9/e 642) ...see text
128. Ans. (a) Acute promyelocytic leukemia
(Ref: Robbins 9/e 664, 8/e p673-4, Harrison 18/e p)
The most common causes are bacterial sepsis, malignant disorders
such as solid tumors or acute promyelocytic leukemiaQ, and
obstetric causes (pregnant women with abruptio placentae, or
with amniotic fluid embolism).
129. Ans. (c) Paroxysmal nocturnal hemoglobinuria
(Ref: Robbins 9/e 642, 8/e p652)
130. Ans. (b) Controlling oxidative stress on RBC
(Ref: Robbins 9/e 634)
G6PD helps in neutralizing the effect of oxidative stress on the RBC.
Oxidative stress is induced by drugs like primaquine and hence in
patients of G6PD there is accelerated hemolysis (intravascular
during the hemolytic episode) resulting in hemoglobinuria and
passage of shockingly black urine by the patient.
131. Ans. (b) IgM (Ref: Robbins 9/e 644, 8/e p653)
Cold agglutinins are monoclonal IgM antibodies that react at 4 to
6°C. They are called agglutinins because the IgM directed
against the ‘I’ antigen present on the RBCs can agglutinate red
cells due to its large size (pentamer).
132. Ans. (a) Hemolytic uremic syndrome
(Ref: Robbins 9/e 660, 8/e p952)
Schistocytes are typically irregularly shaped, jagged, and have two
pointed ends. A true schistocyte does not have central pallor.
Helmet cells are also known as schistocytes/triangle cells/burr
cells are a feature of microangiopathic diseasesincluding
disseminated intravascular coagulation (DIC), thrombotic
microangiopathies (TTP), mechanical artificial heart valves
and hemolytic uremic syndrome (HUS).
133. Ans. (d) All of the above
(Ref: Robbins 8/e p952)
Yes friends, only additional important thing that you need to be aware
of is that schistocytes can also be seen in severe iron deficiency
anemia.
Echinocytes/Burr • Regular spine-like • Megaloblastic
cells projections on cell surface; anemia/ hemolyti
Acanthocytes/Spur • irregular thorn-like anemia /burns
cells projections; • in liver disease,
Stomatocytes • Slit-like (mouth like) area of abetalipoproteine
Schistocytes pallor
Leptocytes • Fragmented RBCs; • in liver disease,
Codocytes triangular, comma-shaped anemia of chronic
Dacrocytes or helmet shaped disease)
Drepanocytes • Thin flat cells
Elliptocytes • Mexican hat cells
Keratocytes • Tear drop cells or Target
Knizocytes cells (red cells with central
dark area;
• Sickle cells
• Pencil cells or cigar cells
• Helmet cells

134. Ans. (c) Normal APTT


(Ref: Robbins 8/e p673-4, 9/e 665, Harrison 18/e p)
DIC is an acute, subacute, or chronic thrombohemorrhagic disorder
characterized by the excessive activation of coagulation, which
leads to the formation of thrombi in the microvasculature of the
body.
Harrison 18/e pmentions… “Common findings include the
prolongation of PT and/or aPTT; platelet counts ≤100,000/
µL3, or a rapid decline in platelet numbers; the presence of
schistocytes (fragmented red cells) in the blood smear; and
elevated levels of FDP.
The D-dimer test is more specific for detection of fibrin—but not
fibrinogen—degradation products and indicates that the cross-
linked fibrin has been digested by plasmin. Because fibrinogen
has a prolonged half-life, plasma levels diminish acutely
only in severe cases of DIC.

The most sensitive test for DIC is the FDP levelQ. DIC is an unlikely
diagnosis in the presence of normal levels of FDP.

135. Ans. (c) Increased haptoglobin (Ref: Robbins 9/e 631)


136. Ans. (a) G6PD deficiency(Ref: Robbins 9/e 634, 8/e p645)
137. Ans. (a) Clumps of RBCs due to IgM mediated cold
autoimmune haemolytic anemia (Ref: Robbins 9th/643-4)
The image shows agglutinated red cells. Since the history mentions
that cyanosis is seen affecting fingers and tips of nose in the
month of December, it suggests that the red cells agglutinated
with cold antibody associated autoimmune haemolytic anemia.
138. Ans. (d) Low reticulocyte count (Ref: Robbins 9th/631)
Haemolytic anaemia is characterised by the increased reticulocyte
count, splenomegaly (especially extravascular hemolysis) and
increased concentration of bilirubin and urobilinogen.
139. Ans. (b) Aplastic anemia (Ref: Robbins 9th/653)
Direct coombs test is positive in conditions associated with presence of
antibodies on the surface of the cells. It is therefore Negative in a
patient of aplastic anemia. Some of the important conditions with
positive Coombs test include:
• Autoimmune haemolytic anaemia
• Haemolytic disease of new born or Rh incompatibility
• Haemolytic anaemia due to mismatched blood transfusion
140. Ans. (d) Folic acid deficiency
Looking at the parameters being mentioned in the question, we
analyse and understand the type of anemia the patient is having:
MCV = haematocrit ×10
RBC count in million
30 × 10 = 150
2
Hence, the patient is having macrocytic anemia. The answer is folic
acid deficiency anemia. All other option lead to microcytic
anemia.
141. Ans. (b) Iron deficiency anemia (Ref: Robbins 9th/650)
142. Ans. (b) Causes mild to severe coombs positive hemolytic
anemia (Ref: Robbins 9/e 643, 660)
Coombs test is done for immune hemolytic anemia whereas HUS is
non immune hemolytic anemia.
143. Ans. (c) Hypersplenism (Ref: Robbins 9/e 631)
Hypersplenism is an extracorpuscular cause of hemolytic anemia. All
other options are intracorpuscular/intrinsic causes of hemolytic
anemia.
144. Ans. (b) IgG (Ref: Robbins 9/e 643)
145. Ans. (a) P. Falciparum (Ref: Robbins 9/e 391)
• People who are heterozygous for the sickle cell trait (HbS)
become infected with P. falciparum, but they are less likely
to die from infection. The HbS trait causes the parasites to
grow poorly or die because of the low oxygen
concentrations.
• HbC, another common hemoglobin mutation, also protects
against severe malaria by reducing parasite proliferation.
• People can also be resistant to malaria due to the absence
of proteins to which the parasites bind. P. vivax enters red
cells by binding to the Duffy blood group antigen. Many
individuals (usually Africans), are not susceptible to infection
by P. vivax because they do not have the Duffy antigen.
146. Ans. (b) Beta thalassemia
(Ref: Robbind 9/e 640-641, 8th/648-652, Textbook of Hematology
1st/89)
Friends, let’s get the answer of this question in a methodical
manner. The clues in the question:
• Reduced values of MCV and MCHC: microcytic hypochromic
anemia (G6PD deficiency is ruled out)
• Age of presentation and ethnicity: 6 year old and Punjabi
ethnicity
• History of repeated blood transfusion: in favour of
thalassemia
• Osmotic fragility is reduced: in favour of thalassemia again
though it may be seen in sickle cell also
If we compare the above mentioned points, we can deduce that the
stem talks about a patient suffering from thalassemia. Now
comparing the incidence of alpha and beta thalassemia, it is clear
that beta thalassemia is far more common than alpha
thalassemia. Hence, it is a better option than alpha thalassemia.
Also, the hemoglobin is the question is more suggestive of severe
anemia with positive history of multiple blood transfusions
both being important pointers towards thalassemia major.
Please note that apart from Punjabis, other ethnic groups having high
prevalence of thalassemia are Sindhis, Gujaratis, Parsis, Begalis
and Lohanas.
147. Ans. (a) β- thalassemia
(Ref Recent Advances in Hematology -3, 1st/173)
148. Ans. (a) Deletion of alpha genes
(Ref: Robbins 8th/651, T. Singh 2nd/95)
149. Ans. (b) Deletion of 3 alpha chains
(Ref: Robbins 8th/652, 9/e 642)
150. Ans. (a) Malaria
(Ref: Robbins 9/e 638, 8th/645, T. singh 1st/270)
151. Ans. (d) Solubility (Ref: Robbins 9/e 635-636)
152. Ans. (d) 0% (Ref: Robbins 7e/632-33, Harrison – 17th/641)
Normal percentage of HbA2 ranges from 1.5 to 3%.
• In thalassemia trait (β thalassemia minor), HbA2 level may
be elevated (3.5-7.5%).
• Thus, wife in this question has normal genotype (bb)
whereas husband has thalassemia – trait (β+β).
• β-Thalassemia is an autosomal recessive disease

• None of the offsprings thus will have thalassemia major (β+


β+), thus, the risk of having a child with thalassemia major is
therefore 0%.
• 50% of the offspring (b+b) will be carriers like father.
153. Ans. (b) Replacement of glutamate by valine in b-chain of
HbA
(Ref: Robbins 9/e 635, Harrison 17th/637)
154. Ans. (c) Binding of HbS to the deoxygenated HbA can
extend the polymer and cause sickling of the red blood cells
(Ref: Harper 25th/71, Robbins 7th/628, 630, 8th/645-646, 9/e 636)
• Sickle cell anemia is caused by a point mutation is which
there is replacement of glutamine by valine at position 6 of
the beta chain generating a sticky patch on the surface of
HbS.
• The sticky patch on the surface of adjacent HbS molecules
cause their polymerization resulting in formation of long
fibrous precipitates.
HbA molecule can complementarily bind with the sticky patch by HbS but
cannot promote the formation of long fibers because it does not have a patch
itself. So, the option c is false in the given question.

Other hemoglobins which also weakly interact with HbS and prevent its
polymerization include HbF and HbC.
155. Ans. (a) 50% HbS is required for occurrence of sickling
(Ref: Robbins 7th/628-629, 9/e 635-636)
• Sickle cell disease is a hemoglobinopathy in which HbS due
to point mutation. If an individual is homozygous for sickle
mutation almost all the Hb in erythrocyte is HbS, if he is
heterozygote only 40% is HbS the remainder being normal.

In addition, Nelson also mentions that ‘persons with sickle cell trait have
totally benign clinical course because the low level of HbS present in
them (35-40% of total) is insufficient to produce sickling manifestation’.
So, option ‘a’ is a better answer than option ‘b’...

156. Ans. (d) Deletion of gene (Ref: Robbin 9/e 635)


157. Ans. (a) Increased HbF; (b) Increased HbA2; (c)
Microcytosis; (e) Target cell
(Ref: Harrison 17th/641, Robbins 7th/634-635, 9/e 641)
158. Ans. (d) Point mutation
(Ref: Harrison 17th/637 Robbins 7th/628, 9/e 635)
159. Ans. (c) Three α globin genes
(Ref: Robbins 9/e 642)
160. Ans. (b) Higher concentration of HbF
(Ref: Robbins 7th/629, 9/e 642)
161. Ans. (a) Presence of a structurally abnormal Hb
(Ref: Robbins 8th/645; 7th/628)
162. Ans. (b) b-chain
(Ref: Robbins 8th/645; 7 th/628, 9/e 635)
163. Ans. (b) Bone marrow expansion in the calvarium
(Ref: Robbins 8th/646-651, 9/e 636-638)
164. Ans. (c) Sickle thalassaemia
(Ref: Robbin 9/e and Harrison chapter 104, disorders of
hemoglobin synthesis)
Condition Clinical Hemogblobin MCV. Hemoglobin
Abnormalities Level g/l (g/dL) fL Electrophoresis
Sickle cell None: rare Normal Normal Hb S/A: 40/60
trait painless
hematunia
Sickle cell Vasooclusive 70-100 (7-10) 80-100 Hb S/A: 100/0
anemia crises; aseptic Hb F:2-25%
necrosis of bone
S/ B0 Vasoocclusive 70-100 (7-10) 60-80 Hb S/A: 100/0
thalassemia crises: aseptic Hb F;1-10%
necrosis of bone
S/B+ Rare crises and 100-140 (10-14) 70-80 Hb S/A: 60/40
thalassemia aseptic necrosis
Hemoglobin Rare crises and 100-140 (10-14) 80-100 Hb S/A:50/0
SC aseptic necrosis;
painless
hematuria

165. Ans. (d) Relative excess of β, γ, and δ chains


(Ref: Robbins 8/e p651, 9/e 635-636)
• The α-thalassemias are caused by inherited deletions that
result in reduced or absent synthesis of α-globin chains and
relative excess of other chains.
• The options which created confusion were options “c” and
“d”.
Total absence of α-chains is a feature of most severe form of α-
thalassemia resulting in hydrops fetalis.
Every patient having α-thalassemia would not be having total absence
of α- chains but all patients would be having relative excess of β,
γ, and δ chains as per their age of presentation. Hence, we prefer
option “d” as the answer for this question.
166. Ans. (c) Saline washed packed RBC Ref: Choudhary p80

Saline washed RBCs are specially indicated in conditions requiring repeated


transfusion when the chances of urticarial reactions due to plasma is high.
These have negligible plasma proteins and just 10% leucocytes. It is also
preferred in the management of babies suffering from thalassemia and
paroxysmal nocturnal hemoglobinuria.

167. Ans. (d) Macrocytic anaemia (Ref: Robbin 9/e)


Thalassemia major presents before 1 yr of age with severe anemia
which necessitates packed RBC transfusions every 2-3 months.
The child is said to be transfusion dependent as survival is
decided by RBC being transfused.
The ineffective erythropoiesis in bone marrow results in shift of
hematopoiesis to liver and the bone marrow. Hence the liver and
spleen enlarge in size. But the net result is defective microcytes
being produced. The type of anaemia is microcytic hypochromic
anaemia.
Mnemonic to remember causes of macrocytic anemia: ABCDEF

• Alcohol + liver disease


• B12 deficiency
• Compensatory reticulocytosis (blood loss and hemolysis)
• Drug (cytotoxic and AZT)/Dysplasia (marrow problems)
• Endocrine (hypothyroidism)
• Folate deficieny/ Fetus (pregnancy)

168. Ans. (c) Sickle cell anemia (Ref: Robbins 9/e 635)
Sickled cells can cause microvascular occlusion affecting bones,
brain, kidney, liver, retina and pulmonary vessels.
169. Ans. (b) Beta chain….too obvious friends
(Ref: Robbins 9/e 635, 8/e p645, 7/e p628)
170. Ans. (c) Single amino acid base substitution
(Ref: Robbin 8/e p645-6)
Afro American male presenting with the mentioned features is
suggestive of sickle cell anemia is due to vaso-occlusion caused
by sickled cells. Sickle cell anemia can cause chronic hemolytic
anemia, recurrent pneumonia and non haling painful ulcer.
“Sickle cell anemia is caused by a point mutation in the sixth codon of
β-globin that leads to the replacement of a glutamate residue with
a valine residue”… (Ref: Robbind 8/e p645)
171. Ans. (b) Gene deletion (Ref: Robbin 9/e p641)
172. Ans. (c) Replacement of glutamate by valine in β-chain of
HbA (Ref: Robbins 8/e p645, 9/e 635)9
173. Ans. (d) Protective action against adult malaria
(Ref: Robbins 9/e 638, 8/e p645-648, 7/e p629)
174. Ans. (d) Infested red blood cells stick to the capillaries
(Ref: Robbins 9th/391)
Plasmodium falciparum is associated with infected red cells
expressing PfEMP (Plasmodium falciparum erythrocyte
membrane protein) leading to their attachment to the endothelial
cells. This leads to sequestration of infected red cells in the
capillaries. That’s the reason for the non-appearance of
trophozoites and schizonts in the peripheral blood smear.
175. Ans. (c) Sickling is reversible with oxygenation
(Ref: Robbins 9/e 635 )
176. Ans. (b) Decreased ESR (Ref: Robbins 9th/636)
Sickle cell is associated with decreased ESR (only important exception
amongst anemias since all other anemias are associated with
increased ESR).

ANNEXURE
You must know: Different type of red blood cells are:

• Echinocytes/Burr cells : Regular spine-like projections on cell surface; in


Megaloblastic anemia/burns/hemolytic anemia
• Acanthocytes/Spur cells : irregular thorn-like projections; in liver disease,
abetalipoproteinemia
• Stomatocytes : Slit-like (mouth like) area of pallor
• Schistocytes : Fragmented RBCs; triangular, comma-shaped or helmet
shaped
• Leptocytes : Thin flat cells
• Codocytes : Mexican hat cells
• Dacrocytes : Tear drop cells or Target cells (red cells with central dark
area; in liver disease, anemia of chronic disease)
• Drepanocytes : Sickle cells
• Elliptocytes : Pencil cells or cigar cells
• Keratocytes : Helmet cells
• Knizocytes : Cells with more than one concavities
1. A boy after playing football complaining fatigue and
abdominal pain. He also had a history of hand swelling in
past. On ultrasonography, he has shrunken spleen. What is
the likely diagnosis of this patient?
NEET 2020 like pattern)
(a) Sickle cell anemia
(b) Iron deficincy anemia
(c) Acute pancreatitis
(d) Intermittent porphyria
Ans. (a) Sickle cell anemia (Ref: Robbins 9th/636-7)
Presence of fatigue is likely to be due to anemia. Shrunken spleen is
likely to be due to autosplenectomy and swelling of the hand can
be attributed to dactylitis or ‘hand foot syndrome’. So, the clinical
picture given in the stem of the question is likely to be associated
with sickle cell anemia.
Shrunken spleen and hand swelling is not seen in other options like
iron deficiency anemia, pancreatitis and intermittent porphyria.
2. A young male presented with history of fatigue and tiredness.
On investigating, he had a hemoglobin of 8 g/dL, MCV of 101
fL, hematocrit of 33% with the peripheral smear showing
macrocytes and hypersegmented neutrophils. Which of the
following is the most likely cause of the findings in this
patient?
(NEET 2020 like pattern)
(a) Lead poisoning
(b) Chronic alcohol
(c) Chronic renal failure
(d) Hemolytic anemia
Ans. (b) Chronic alcohol > (c) Chronic renal failure
(Ref: Robbins 9th/ 649)
The clinical picture given in the question is suggestive of anemia with
high MCV and hypersegmented neutrophils which means
megaloblastic anemia. Out of the given options in the questions,
alcohol is a commoner cause of folate deficiency. So, it can be
associated with development of megaloblastic anemia.
3. A 30-year-old women came with complaints of easy
fatigability, exertional dyspnea and weight loss. She also has
a complaint of frequent fall. Physical examination revealed
there was bilateral decrease in vibration sense. Her
hemoglobin levels were 8.2 g%. She was treated with folate.
Her anemia improved but neurological symptoms worsened.
Which of the following is the most probable reason for her
condition? (NEET 2020 like pattern)
(a) Folate not absorbed
(b) Folic acid deficiency unmasked pyridoxine deficiency
(c) Deficiency of folate reductase in CNS
(d) Folate therapy caused rapid use of B12 stores aggravating
symptoms
Ans. (d) Folate therapy caused rapid use of B12 stores
aggravating symptoms (Ref: Robbins 9th/ 649)
The clinical is suggestive of anemia (features of fatigability and
exertional dyspnea with low hemoglobin). However, it is important
to know that megaloblastic anemia with neurological symptoms
respond to vitamin B12 administration. If by mistake, the patient
is administered folate first, the neurological complications are
likely to worsen as had happened in this case.
4. Hepcidin inhibits which of the following?
(AIIMS Nov 2019 like pattern)
(a) Hephaestin
(b) DMT-1
(c) Ceruloplasmin
(d) Ferroportin
Ans. (d) Ferroportin (Ref: Robbins 9th/ 650)
• Iron absorption is regulated by hepcidin, a peptide that is
synthesized and released from the liver in response to
increases in intrahepatic iron levels.
• Hepcidin inhibits iron transfer from the enterocyte to plasma
by binding to ferroportin and causing it to be endocytosed
and degraded.
• When the body is replete with iron, high hepcidin levels
inhibit its absorption into the blood.
5. Iron entry into enterocytes is done by which of the
following? (AIIMS Nov 2019 like pattern)
(a) DMT-1
(b) Ferroportin
(c) Transferrin
(d) Hepcidin
Ans. (a) DMT-1 (Ref: Robbins 9th/ 650)
2+
• Fe iron is then transported across the apical membrane of
the enterocytes in the duodenum by divalent metal
transporter 1 (DMT 1).
• Iron that enters the duodenal cells and is destined for the
circulation is transported from the cytoplasm across the
basolateral enterocyte membrane by ferroportin.
• Transferrin is involved in the transport of iron in the blood.
6. Which of the following findings are there in iron deficiency
anemia? (AIIMS Nov 2019 like pattern)
(a) ↑ TIBC, ↑ Ferritin, ↓ Transferrin saturation
(b) ↑ TIBC, ↓ Ferritin, ↓ Transferrin saturation
(c) ↓ TIBC, ↓ Ferritin, ↓ Transferrin saturation
(d) ↓ TIBC, ↓ Ferritin, ↑ Transferrin saturation
Ans. (b) ↑ TIBC, ↓ Ferritin, ↓ Transferrin saturation
(Ref: Robbins 9th/ 652)
As discussed in the text in detail, the patients with iron deficiency
anemia have the following changes:
• Serum iron: reduced
• Serum ferritin: reduced
• Transferrin saturation: reduced
• Total iron binding capacity: increased

7. Which of the following is downregulated by hepcidin?


(AIIMS May 2019 like pattern)
(a) Ferroportin
(b) Transferrin
(c) DMT 1
(d) Hephaestin
Ans. (a) Ferroportin (Ref: Robbins 9th/650)
• Hepcidin inhibits iron transfer from the enterocyte to plasma by binding to
ferroportin and causing it to be endocytosed and degraded….it is important in
the pathogenesis of anemia of chronic disease.
• This process is coupled to the oxidation of Fe2+ iron to Fe3+ iron, which is
carried out by the iron oxidases hephaestin and ceruloplasmin.
8. Which of the following is a cause of intravascular
hemolysis? (NEET 2019 like pattern)
(a) Warm type autoimmune hemolytic anaemia
(b) PNH
(c) Cold agglutinin disease
(d) Hereditary spherocytosis
Ans. (b) PNH (Ref: Robbins 9th e/p 642 )
Paroxysmal nocturnal hemoglobinuria (PNH) is a disease that results
from acquired mutations in the phosphatidylinositol glycan
complementation group A gene (PIGA), an enzyme that is
essential for the synthesis of certain membrane-associated
complement regulatory proteins.
Red cells deficient in these GPI-linked factors are abnormally
susceptible to lysis or injury by complement. This manifests as
intravascular hemolysis, which is caused by the C5b-C9
membrane attack complex.
9. Which of the following is the best test for diagnosis of
paroxysmal nocturnal hemoglobinuria?
(AI 2018 Pattern)
(a) Sucrose lysis test
(b) Ham test
(c) Flow cytometry
(d) Bone marrow aspiration
Ans. (c) Flow cytometry
(Ref: Robbins 9/e p642, CMDT 2018/518)
FLAER Flow cytometry is the gold standard investigation for the
diagnosis of PNH.
10. CD59 deficiency leads to: (AI 2018 Pattern)
(a) Chediak Higashi disease
(b) TTP
(c) Paroxysmal nocturnal hemoglobinuria (PNH)
(d) Burkitt’s lymphoma
Ans. (c) PNH (Ref: Robbins 9/e p642)
11. Which of the following hematological problems can be
precipitated by parvovirus? (AI 2018 Pattern)
(a) Haemolytic crisis
(b) Aplastic crisis
(c) Pure red cell aplasia
(d) Pancytopenia
Ans. (b) Aplastic crisis (Ref: Robbins 9/e p633-635)
12. All of the following decrease in iron deficiency anemia
except: (AIIMS Nov 2017 Pattern)
(a) Ferritin
(b) TIBC
(c) Serum iron
(d) Transferrin saturation
Ans. (b) TIBC (Ref: Robbins 9/e p652, Harrison 19/e p627)
Iron deficiency anemia is characterized by reduced serum iron, %
transferrin saturation and serum ferritin. The only parameter
increased in these patients is total iron binding capacity.
13. A 6-year-old patient with anemia, on electrophoresis shows
HbFof 90% and HbA2 of 3%. Which of the following will be
seen on peripheral smear:
(AIIMS Nov 2017 Pattern)
(a) A,B
(b) A,C
(c) B,C
(d) ABC
Ans. (d) ABC (Ref: Robbins 9/e p655)
The presence of HbFof 90% and HbA2 of 3% is suggestive of
thalassemia major. Now, looking at the images, they represent A
(target cells), B (Howell Jolly bodies) and C (microcytic cells with
tear drop cells suggestive of poikilocytosis). Since, all the three
can be seen I a patient with thalassemia major, so, the answer for
the question is A,B,C.
14. Which of there is not involved in iron metabolism?
(AIIMS Nov 2017 Pattern)
(a) Hepcidin
(b) Ferroportin
(c) Transthyretin
(d) Ceruloplasmin
Ans. (c) Transthyretin (Ref: Robbins 9/e p642)

• Hepcidin is an inhibitor of iron absorption in the body


• Ferroportin increases intake of iron from the duodenal cells
• Ceruloplasmin is involved in the oxidation of ferrous iron into ferric iron.
• Transthyretin is involved in the transport of thyroid hormones and retinol

15. Which of these is true about intracellular iron homeostasis in


iron deficiency anemia?
(AIIMS May 2017 Pattern)
(a) Ferritin mRNA concentration decreases and ferritin synthesis
increases
(b) Transferrin receptor 1mRNA upregulation and increased
receptor expression
(c) Ferritin mRNA concentration increases and ferritin synthesis
decreases
(d) Transferrin receptor 1mRNAdownregulation and decreased
receptor expression
Ans. (b) Transferrin receptor 1 mRNA upregulation and increased
receptor expression
Iron deficiency anemia is characterized by decreased ferritin,
increased transferrin and increased transferrin receptors. So,
option “b” is the best answer here.
16. What does the red cell distribution width represents?
(AIIMS May 2017 Pattern)
(a) Anisocytosis
(b) Level of hypochromia
(c) Poikilocytosis
(d) Anisochromia
Ans. (a) Anisocytosis (Ref: Robbins 9/e p630)
Red cell distribution width is the coefficient of variation of red cell size
and volume. This is indicated by anisocytosis.
17. What will be the corrected reticulocyte count in a patient with
a hemoglobin of 5 and absolute reticulocyte count of
9%? (AIIMS May 2017 Pattern)
(a) 4.5%
(b) 6%
(c) 1%
(d) 3%
Ans. (d) 3%
Corrected reticulocyte count = reticulocyte% ×

Applying the same in our question,

Disclaimer
Any resemblance to an actual question is purely coincidental.
• Alkaline phosphatase is specific for Neutrophils and is called as
NAP.
• High LAP score: Infection, pregnancy, myeloproliferative
disorder except CML, drugs (oral contraceptive pills, growth
factors, lithium, Corticosteroids etc)
• Low LAP score: CML, PNH and myelodysplastic syndrome.
• Dohle bodies are rough ER remnants in neutrophils which are
seen in infections and Chediak-Higashi syndrome.
• Downey cells are seen in infectious mononucleosis.
• Phenytoin causes Pseudolymphoma as it causes Paracortical
hyperplasia.
• Leukamoid reaction is differentiated from leukemia by LAP score
(increased in leukamoid reaction).
• Important causes of leukamoid reaction: Infections, hemorrhage,
drugs (glucocorticoids), malignancies, Down syndrome.
• Important markers of myeloid lineage: CD 13, CD 33, CD 11b, Cd
15, CD4, Cd 117, cMPO.
• Important marker of B-cells: CD 19, CD 20, CD 22, CD 79a, cCD
22, cCD 79a.
• Most of acute lymphoblastic leukemia is arising from immature
precursor B-cells.
• T cell ALL presents as a mediastinal mass whereas ALL-L3 has
morphology identical to Burkitt’s lymphoma cells.
• Lymphoblasts stain positive for PAS (Periodic acid schiff) and tdT
whereas myeloblasts stain for myeloperoxidase, Sudan Black-B
and Non-specific esterase.
• Gingival hypertrophy, hepatomegaly, splenomegaly, and
infiltration of skin (leukemia cutis) is seen with M4/M5 AML
whereas chloroma formation is seen with M2 AML.
• Biphenotypic acute leukemia is acute leukemia with a single
blast cells population showing markers of two different
lineage.
• Bilinear acute leukemia is acute leukemia with two different
blast cells population
• Sezary syndrome and Mycosis fungoides are seen with
Cutaneous T-cell leukemias.
• Myelodysplastic syndrome shows presence of ringed
sideroblasts, Pseudo-Pelger-Huet cells and pawn ball
megakaryocytes. It is associated with monosomy 7 (in children)
and deletion 5 (adults).
• Peripheral smear with neutrophilia, basophilia, eosinophilia and
increased platelets is seen with CML. It is also having the
presence of Philadelphia chromosome.
• Features of Juvenile CML (CMML): Raised HbF; Ph chromosome
negative; monocytosis (>1×109L) and thrombocytopenia; it is
associated with NF-1.
• Most common myeloproliferative disorder is Polycythemia vera.
• Characteristic bone marrow aspiration finding in myelofibrosis is
Dry tap and so, diagnosis is made with bone marrow biopsy.
• Hodgkin lymphoma is a lymphoma characterized by involvement
of painless lymphadenopathy and Pel Ebstein fever. It has
characteristically presence of Reed-sternberg cells (RS cells).
• RS like cells are seen in infectious mononucleosis,
immunoblastic lymphoma, carcinoma ans sarcoma.
• Classification of HD proposed by international lymphoma study
group: REAL.
• Non Hodgkin lymphoma is more common lymphoma and has
several subtypes.
• Low grade NHL: Small lymphocytic, follicular small cleaved cells,
follicutar mixed.
• Intermediate grade NHL: Follicular large cells, Diffuse small
cleaved/mixed/large cells.
• Diffuse large B cell lymphoma is the commonest NHL in the
world.
• Most common extranodal site for NHL is Stomach whereas M/C
extranodal site for NHL in HIV infected patients is CNS.
• Post transplant lymphoma arises from B-cells (as T cells are
destroyed by therapeutic drugs).
• Multiple myeloma is a plasma cell cancer secreting excessive
immunoglobulin light or heavy chains. It has NORMAL alkaline
phosphatase.
• Most important prognostic factor of multiple myeloma is serum
β2-microglobulin
• Histiocytosis-X (Langerhans cell histiocytosis) includes Letterer-
siwe disease, Hand-Schuller- Christian disease and Eosinophilic
granuloma. The characteristic of Langerhans cell histiocytosis is
Birbeck granules (have Tennis racket appearance).
• Multiple permeating (osteolytic) lesions in a child: Histiocytosis-X.
• Generalized necrotizing lymphadenopathy is a feature of
Kikuchi disease whereas eosinophilic abscess in lymph nodes
is seen with Kimura’s disease.

HEMATOGENOUS NEOPLASMS

Leukemia is a term used to describe the widespread involvement of the


bone marrow accompanied with large number of cancer cells in the
peripheral blood whereas lymphoma is a term used for proliferation of
lymphoid cells arising as discrete tissue masses.

WHO Classification of the Lymphoid Neoplasms


I. Precursor III. Precursor
B-Cell II. Peripheral B-Cell T-Cell IV. Peripheral T-Cell
Neoplasms Neoplasms Neoplasms Neoplasms
* Precursor-B Chronic lymphocytic *Precursor-T Angioimmunoblastic T-
lymphoblastic leukemia/small lymphocytic lymphoblastic cell lymphoma
leukemia or lymphoma leukemia or
lymphoma lymphoma
B-cell prolymphocytic Large granular
leukemia lymphocytic leukemia
Lymphoplasmacytic T-cell prolymphocytic
lymphoma leukemia
Splenic and nodal marginal Peripheral T-cell
zone lymphomas lymphoma, unspecified
Extranodal marginal zone Anaplastic large cell
lymphoma lymphoma
Mantle cell lymphoma Mycosis
fungoides/Sezary
syndrome
Follicular lymphoma Enteropathy-associated
T-cell lymphoma
Marginal zone lymphoma Panniculitis-like T-cell
lymphoma
Hairy cell leukemia Hepatosplenic γδ T-cell
lymphoma
Plasmacytoma/plasma cell Adult T-cell
myeloma leukemia/lymphoma
Diffuse large B-cell NK/T-cell lymphoma,
lymphoma nasal type
Burkitt lymphoma NK-cell leukemia

V. HODGKIN LYMPHOMA

Classic subtypes Nonclassical

• Nodular sclerosis • Lymphocyte predominant


• Mixed cellularity
• Lymphocyte-rich
• Lymphocyte depletion

• The FAB (French-American-British Classification) diagnostic criteria for acute


leukemia is the presence of >30% blasts in the bone marrow (normally, they are
<5%) and increased number of cells in the blood.
• Acute leukemias have a high rate of proliferation without differentiation and
their clinical course is rapid.
• Chronic leukemias have a low rate of proliferation of tumor cells with good
differentiation and their clinical course is slow.

For broad understanding, WBC neoplasms can be classified as:


LEUKEMIAS

1. Acute Lymphoblastic Leukemia (ALL)

It is the commonest leukemia seen in childhoodQ having slight


predilection for males. The etiological agents include exposure to ionizing
radiations as X rays, chemical like benzene, genetic disorders like Down
syndrome, ataxia telangiectasia and acquire disorders like paroxysmal
nocturnal hemoglobinuria and aplastic anemia.

Fig. 1: Lymphoblast having agranular cytoplasm and minimal nucleoli.


The predominant cell seen in this leukemia is lymphoblastQ
characterized by coarse nuclear chromatin, 1-2 nucleoli, high N:C
(nuclear:cytoplasmic) ratio and staining positively with PAS (block
positivity) and terminal deoxynucleotidyltransferase (TdT).
FAB (French-American-British) classification of ALL (Older
classification)
L1 ALL L2 ALL L3 ALL

CommonestQ type of ALL Next common type having Rarest type of ALL with the
having the best worse prognosis. worstQ prognosis.
prognosis.

Small homogenous blast, Large, heterogenous blast, Large homogenous blast,


scanty cytoplasm, indented nuclei, one or more abundant basophilic
indistinct nucleoli nucleoli, moderately cytoplasm with prominent
abundant cytoplasm, minimal cytoplasmic vacoulation
cytoplasmic vacoulation staining positive with Oil
Red ‘O’.

Clinical features
• ALL is characterized by sudden onset of symptoms that arise due to
replacement of the normal bone marrow cells with blast cells, thereby
causing features/symptoms due to decreased number of RBC, WBC
and platelets (anemia, infections and increased bleeding tendency
respectively). The leukemic cells also infiltrate the organs of the body
like spleen, liver and lymph nodes causing splenomegaly,
hepatomegaly and lymphadenopathy.

• Bone marrow expansion is responsible for bone pain and tenderness


(usually sternal tenderness) in these patients. Testicular involvement
and CNS features like headache, vomiting and nerve palsies are also
seen in these patients. This leukemia can either be a precursor B-cell
or T-cell type (these are the two predominant types of lymphocytes). In
the pre-T-cell type, there is presence of mediastinal mass due to
thymus involvement which can compress either the vessels or airways
in the region.
Investigations

Blood findings

They include markedly elevated WBC count. Uncommonly, some patients may show
pancytopenia with few or no blast cells in peripheral blood which is called as aleukemic
leukemia. However, diagnosis is made in this condition by the presence of >20% blasts
in the bone marrow. Blast cells with Periodic Acid Schiff (PAS) positivity are seen. There
is presence of anemia, neutropenia and thrombocytopenia.

Bone marrow

It is hypercellular with blast cells >20% of the marrow cells.

Biochemical investigations

It includes elevated serum uric acid and phosphate levels accompanied by


hypocalcemia (because of hyperphoshatemia). Serum LDH is also increased as a result
of increased turnover of the cancer cells.

Prognostic factors in ALL


Good Prognosis Bad Prognosis
• Age 2-10 years • Age <1 year or > 10 years
• Female sex • Male sex
• L1 cell • L2 or L3 cell
• Peripheral blast count <1,00,000 • Peripheral blast count >1,00,000
• Pre B-cell phenotype • Pre T-cell phenotype
• Absence of mediastinal mass • Mediastinal mass/CNS/testicular
• Hyperdiploidy (>50 involvement
chromosomes) or t(12;21) • Pseudodiploidy or t (9;22) (Philadelphia
• Trisomy 4,7 and 10 chromosome) or t (8;14) or t (4;11)

Genetic associations of ALL


• T cell ALL is associated with gain of function mutation in NOTCH 1 gene (normally
required for T cell development)
• B cell ALL is associated with loss of function mutation in PAX5, E2A and EBF or
balanced t (12; 21) affecting TEL and AML1 genes (normally required for B cell
development).

2. Acute Myelogenous Leukemia (AML)

It is the leukemia affecting adults seen most commonly between the ages
of 15-39 years. The etiological agents include exposure to ionizing
radiations such as X-rays, chemicals like benzene, secondary to
myelodysplastic syndrome, drugs like anti-cancer drugs and genetic
disorders like Down’s syndrome and Fanconi’s anemia.

The predominant cell seen in this leukemia is myeloblast


characterized by fine nuclear chromatin, 3-5 nucleoli, high N: C (nuclear:
cytoplasmic) ratio, presence of Auer rods (these are abnormal azurophilic
granules) and staining positively with Sudan black B, myeloperoxidase
(MPO) and Non Specific Esterase (NSE).
Fig. 2: Myeloblast having granular cytoplasm and 4-5 nucleoli.

FAB (French-American-British) classification of AML


Class Salient Features

M0: Minimally differentiated AML Myeloid lineage blasts


M1: AM L without maturation Myeloblasts without maturation (> 3% blasts MPO
M2: AML with maturation or SBB positive)
M3: Acute (Hypergranular) t (8;21)Q is present, maximum incidence of
promyelocytic leukemia chloromaQ, Auer rods are seen
M4: Acute myelomonocytic t (15;17)Q seen, Associated with DICQ, Auer rods
leukemia (Naegli type) are seen
Inversion 16Q present, Presence of both
M5: Acute monocytic leukemia
myeloblasts and monoblasts (blasts > 20%;
(Schilling type)
neutrophil and its precursors > 20%; monocyte
M6: Acute erythroleukemia (Di and precursors > 20%)
Gugliemo disease) t (9;11) seen, Highest incidence of tissue
M7: Acute megakaryocytic infiltrationQ, organomegaly, and
leukemia lymphadenopathy
Abnormal erythroid precursors are seen
Least common type of AML, associated with
myelofibrosis and Down syndrome

Additional Salient Points

Clinical features
They are similar to ALL i.e. fatigue due to anemia, bleeding and infections
in oral cavity, lungs etc. Patients may develop bleeding diathesis due to
DIC which results from release of thromboplastic substances in the
granules (most common with M3 AML). Infiltration of these cells into the
organs is relatively less common as compared to ALL resulting in only mild
hepatosplenomegaly and lymphadenopathy.

However, gum hypertrophy and infiltration in the skin (called as


leukemia cutis) is common with particularly M5 AML. Less frequently,
patients may present with localized masses in absence of marrow or
peripheral blood involvement called myeloblastoma, granulocytic
sarcoma or chloroma (so named as they turn green in presence of dilute
acid due to the presence of MPO). Lysozyme, CD43, CD45, CD117 and
MPO are positive markers of granulocytic sarcoma. These manifest as
proptosisQ (due to orbital involvement) most commonly or may present
as bone or periosteal masses.

Investigations

Blood findings

It includes markedly elevated WBC count. Findings are similar to that in ALL except that
the blast cells show positivity with MPO, NSE or Sudan black. There is presence of
anemia, neutropenia and thrombocytopenia.

Bone marrow

It is hypercellular with blast cells >20% of the marrow cells.

Biochemical investigations

These show elevated serum uric acid and phosphate levels accompanied by
hypocalcemia (because of hyperphosphatemia). Serum Muramidase levels is also
increased in M4 and M5 AML. The fibrin degradation products (FDPs) are elevated in
M3 AML due to DIC.

Prognostic factors in AML

Good prognosis Bad prognosis


• Age <40 years • Age <2 years or >55 years
• M2,M3,M4 forms of AML • M0,M6,M7 forms of AML
• Blast cell with Auer rods • Complex karyotypes
• TLC < 25 X 109/L • TLC > 100 X109/L
• t(15;17), t (8;21), inv 16 • Deletions 5q, 7q
• Rapid response to therapy • Delayed response to therapy
• Leukemia without preceding MDS • AML with preceding MDS or
anticancer drug exposure
Flowchart 1: Diagnostic approach to leukemias.

LEUKEMOID REACTION

It is defined as the presence of elevated leucocyte count (>50,000 cells/ml)


in the peripheral blood resembling leukemia in an individual who actually
does not have leukemia.

Myeloid leukemoid reaction Lymphoid leukemoid reaction


Seen in conditions like: Seen in conditions like:
• Infections (sepsis, TB, endocarditis) • Infections (usually viral like measles,
• Severe hemorrhage and hemolysis chicken pox, CMV, infectious
• Malignancies (Hodgkin’s, multiple mononucleosis or bacterial like TB,
myeloma, metastasis, myelofibrosis) pertussis)
• Miscellaneous (burns, eclampsia, • Rare with malignancies.
mercury poisoning).

MYELODYSPLASTIC SYNDROMES (MDS)

Myelodysplastic syndromes refer to a clonal stem cell disorder resulting in


ineffective hematopoiesis and increased risk of development into acute
myelogenous leukemia (AML). The bone marrow is replaced with
multipotent stem cells which can differentiate but in an unorganized and
ineffective manner only.
It has been linked to the exposure to radiation, benzene, alkylating
agents and some chromosomal abnormalities. MDS can be classified into:
1. Primary MDS – Develops slowly usually after 50 years of age.
2. Secondary or Therapy related MDS (t-MDS) – Usually 2 to 8 years
after toxic drug or radiation exposure. The secondary MDS gets
transformed to AML most frequently and so has a poorer
prognosis.

The bone marrow is usually hypercellular in this condition but the


myelodysplastic precursor cells undergo apoptosis at a fast rate resulting
in ineffective hematopoiesis. MDS is frequently associated with
chromosomal abnormalities including monosomy 5 and 7, deletion of 5q
and 7q, trisomy 8 and deletion of 20 q.
Bone marrow findings
Cells affected Features seen
Erythroid cells Ringed sideroblastsQ (Iron laden mitochondria in
erythroblasts) with increased iron stores
Megaloblasts, nuclear budding, intranuclear bridging,
irregular nuclei

Megakaryocytes Pawn ball megakaryocytesQ (Megakaryocytes with


multiple separate nuclei)

Neutrophils Dohle bodiesQ (toxic granulations) are seen,


Pseudo-Pelger-Huet cellsQ (Neutrophils with two
nuclear lobes) are also seen
Peripheral blood shows the presence of Pseudo-Pelger-Huet cells,
giant platelets, macrocytes, poikilocytes and monocytosis.
Clinical features are seen in only 50% patients including weakness,
infection and hemorrhage due to pancytopenia. Usually patients are of an
old age (mean age of onset is >60 years). The prognosis is poor.

3. Chronic Myelogenous Leukemia (CML)

It is a type of myeloproliferative disorder characterized by the increased


number of immature leukocytes, basophilia and splenomegaly seen in
adults between the ages of 25-60 years. An increased risk of CML is seen
in people exposed to ionizing radiation (survivors of nuclear bombs). There
is presence of ABL gene (a proto-oncogene) on chromosome 9 and BCR
(break point cluster) gene on chromosome 22. A reciprocal translocation
between these two chromosomes causes formation of BCR-ABL fusion
gene on chromosome 22 (called Philadelphia chromosomeQ or Ph). This
fused gene causes synthesis of a 210 kDa fusion protein.
Clinical features

CML has a gradual onset with fatigue, anorexia and weight loss as the
initial complaints. Characteristically, there is presence of splenomegalyQ
caused by infiltration of leukemic cells as well as extramedullary
hematopoiesis. Hepatomegaly is also seen but lymphadenopathy is
uncommon in these patients. Leukocytic infiltration and hypercellularity can
cause sternal tenderness whereas leukostasis can cause priapism, venous
thrombosis and visual disturbances.
Fig. 3: CML showing. B: Basophils; E: Eosinophil; N: Neutrophil; My:
Myeloblast; Band: Band cells.
Bone marrow
It is 100% cellular in these patients (in normal individuals, the marrow is
50% cellular and 50% fat is present). The erythroid precursors are
decreased (due to replacement by myeloid precursors) whereas abnormal
megakaryocytes are commonly seen. The presence of scattered
histiocytes with blue granules (sea-blue histiocytes or pseudo-Gaucher
cellsQ) is characteristically seen. There is also increased deposition of
reticulin fibres.

PERIPHERAL SMEAR
It shows the presence of thrombocytosis and marked leukocytosis with
presence of immature white cells, eosinophilia and basophilia. The
Neutrophil Alkaline Phosphate (NAP or LAP) is decreased (in chronic
phase) in these patients.

BIOCHEMICALLY
There are increased levels of uric acid, serum B12 levels (due to increased
transcobalamin) serum LDH and serum alkaline phosphatase.

PHASES OF CML
1. Chronic phase

• Lasting for about 3-6 years having <10% blasts in the blood or bone marrow.

2. Accelerated phase

• Aggressive phase lasting for few months showing increased anemia and
thrombocytopenia.

• Number of blasts are >10% but <20%.

• Cytogenetic abnormalities like trisomy 8, isochromosome 17q, duplication of Ph


chromosome may develop.

3. Blast phase

• Resembles AML

• Characterized by the presence of >20% blasts in the blood/bone marrow.

• Two third of the blasts are of myeloid lineage whereas the remaining 1/3rd are of
lymphoid lineage (expressing CD10 & CD19; TdT).

4. Chronic Lymphocytic Leukemia (CLL)/Small Lymphocytic


Lymphoma (SLL)

It is a proliferation of mature lymphocytes seen in the old patients (mean


age is 60 years) having a chronic course of onset which means that in this
cancer, replacement of the bone marrow hematopoietic cells occurs after a
period of few years. So, anemia, thrombocytopenia and granulocytopenia
occur late in this disease. TNF-a and TGF-b have a role in this cancer.

Clinical features
The cancer is more commonly seen in males and is asymptomatic in a
large number of cases. FatigueQ is the commonest presenting complaint
associated with lymphadenopathy (initially, cervical followed by a
generalized lymphadenopathy). There is also presence of pallor, mild
hepatosplenomegaly, skin rash and petechiae. However, sternal
tenderness is absent (it is seen in acute leukemia). These cells are not
able to produce normal immunoglobulins resulting in the increased
susceptibility to infections. As already discussed above, the presence of
anemia, thrombocytopenia and granulocytopenia signify the late stage of
the disease.

Investigations
The diagnostic criteria for CLL are:

• Peripheral blood lymphocyte count >5000 cells/mm3with <55% cells being


atypical.
• Bone marrow aspirate showing >30 % lymphocytes.
Fig. 4: CLL showing smudge cells (S) and numerous lymphocytes (L).

Blood investigation
It reveals low Hb, elevated TLC with lymphocytosis being the hallmark
of the disease. Peripheral smear shows increased number of lymphocytes
with scanty cytoplasm. These cells are fragile, so they get disrupted while
making a smear and are called as ‘smudge’ cells or ‘basket’ cells or
‘parachute’ cells.
Bone marrow
It is hypercellular with >30% of the nucleated cells being lymphocytes as
the diagnostic feature of the leukemia. The aggregation of small
lymphocytes and larger cells called ‘prolymphocytes’ is called
proliferation center which is a characteristic finding of CLL.

Immunophenotyping
The cancer cells are positive for CD19, CD20, CD23 and CD5. There is
also low level expression of surface immunoglobulin heavy and light
chains.

Additional point

• The distinguishing feature of CLL and SLL is that in the former blood involvement
is predominant presenting feature whereas in SLL the patients usually have
lymph node findings.
Flowchart 2: Concept of origin of Non-Hodgkin lymphoma.

NON-HODGKIN LYMPHOMA (NHL)

Many subtypes of NHL are there. However, diffuse large B cell lymphoma
(DLBCL) is the commonest NHL.

1. Diffuse large B cell lymphoma

Diffuse large B cell lymphoma arises from a Germinal-center or


postgerminal center B cell. It is associated with diverse chromosomal
rearrangements, most often of BCL6 (30%), BCL2 (10%), or MYC (5%). It
affects patients of all ages, but is most common in older adults (mean age
is 60 years). It often appears as a rapidly growing mass and affects
Waldeyer ring commonly. 30% of the tumors are extranodal affecting liver
and spleen.
Morphologically: there is presence of a relatively large
cell size (usually four to five times the diameter of a small
lymphocyte) and a diffuse pattern of growth in the
tumor.
Immunophenotype. These mature B-cell tumors express CD19 and
CD20 and show variable expression of germinal center B-cell markers
such as CD10 and BCL6. Most have surface Ig.
It is an aggressive tumor is subdivided using immunophenotyping in
the following subtypes
Germinal centre DLBCL has better prognosis whereas
• ABC- DLBCL has worse prognosis.

2. Follicular Lymphoma

It is the commonest NHL in the US (otherwise the commonest NHL is


Diffuse large B cell lymphoma) derived from the B-lymphocytes usually
presenting in the middle age. It shows the presence of translocation
t(14;18)Q. Normally chromosome 14 has immunoglobulin heavy chain
gene whereas the chromosome 18 has bcl-2 gene. The translocation
results in the increased expression of bcl-2Q. The bcl-2 being the inhibitor
of apoptosis causes promotion of the follicular lymphoma cells resulting in
the cancer.
Clinical features
The cancer presents usually as painless generalized lymphadenopathy
with less commonly the involvement of CNS, GIT or testes. The median
survival is for 7-9 years. In almost 50% of patients, this cancer gets
transformed to diffuse large B-cell lymphoma.

Fig. 5: Follicular Lymphoma.


Investigations

Immunophenotyping

The cells expressing bcl-2 protein, surface Ig, CD19, CD20 and CD10 (CALLA). CD5 is
negative in these cells (differentiating feature from mantle cell lymphoma and CLL).

Lymph node biopsy

There is presence of centrocytes (small cell with cleaved nucleus and scant cytoplasm)
and centroblasts (large cell with open nuclear chromatin and multiple nucleoli).

Peripheral blood

Presence of lymphocytosis.

Bone marrow

It shows the presence of characteristic para-trabecular lymphoid aggregates.


3. Mantle Cell Lymphoma

It is a neoplasm in which the tumor cells resemble the normal mantle zone
B-cells which surround germinal centers. These have the translocation
t(11; 14)Q leading to in the increased expression of cyclin D1Q and
subsequently neoplasia.

Clinical features
The cancer usually presents as painless generalized lymphadenopathy,
splenomegaly or involvement of the GIT. Uncommonly, multifocal mucosal
involvement of the small bowel and colon produces lymphomatoid
polyposis.

Investigations
Immunophenotyping reveals the cells expressing cyclin D1, surface Ig
and CD 5. CD23 is negative in these cells. Lymph node biopsy reveals
typically the presence of small cleaved cells with diffuse effacement of
lymph nodes.
• Centroblasts are absentQ (differentiating feature from mantle cell lymphoma and
CLL).
• CD23 is negativeQ in these cells (differentiating feature from CLL)
4. Burkitt’s Lymphoma/Small Non Cleaved Lymphoma

It is a cancer of the germinal center B cell origin characterized by the


presence of hallmark translocation t(8;14)Q. The other translocations which
may be present include t (2;8) or t (8;22). It has the following 3 categories:

Investigations
Immunophenotyping reveals the cells expressing bcl-6Q protein, surface
Ig, CD19, CD20 and CD10 (CALLA).
Fig. 6: Burkitt Lymphoma (Starry sky appearance).

Lymph node biopsy reveals typically the presence of a high mitotic


index of lymphoid cells associated with apoptotic cell death. The presence
of tissue macrophages with clear cytoplasm distributed with tumor cells
creates the typical starry skyQ pattern.

Fig. 7: Burkitt Lymphoma.(Circled area shows dead cells taken by


macrophages).

5. Marginal Zone Lymphoma (MALToma)

It is a group of B-cell tumors arising within the lymph nodes, spleen or


extranodal tissues in which the tumor cells resemble the normal marginal
zone B cells. They are associated with mucosa associated lymphoid
tissue, so, are called maltoma. Their salient features include:
• Begin as polyclonal activation associated with autoimmune disorders,
chronic inflammatory conditions or having infectious etiology.
• Remain localized for a long duration of time.
• May regress if causative agent is removed.
• Splenic marginal zone lymphomas are TRAP +ve (like hairy cell
leukemia)
The extranodal lymphomas can occur in stomach (H. pylori), orbit
(Chlamydia), skin (Borrelia), lung, salivary gland, intestine, etc. Tumors
may respond to antibiotic therapy. If they have cytogenetic abnormality t
(11;18)Q as in extranodal marginal zone lymphoma, they are refractory to
antibiotic therapy.

6. Hairy Cell Leukemia

It is a misnomer because it is not a leukemia but a B cell NHL of the old


age predominantly affecting males (M:F ratio is 4:1) characterized by the
presence of hairy cells in the peripheral blood, splenomegaly and
pancytopenia. The exact cause of this cancer is unknown but the role of
TNF- α is postulated which is responsible for proliferation of hairy cells and
are responsible for the suppression of the proliferation of the normal bone
marrow cells resulting in pancytopenia. The chromosomal abnormalities
associated with this leukemia like trisomy 5 etc have been detected.

Investigations

Blood
There is presence of pancytopenia with the presence of atypical lymphoid cells despite
the presence of neutropenia. Characteristic cells are hairy cells which are leukemic cells
having hair-like projections due to fine cytoplasmic processes seen best under phase
contrast microscope. Electron microscope shows the presence of ribosomal lamellar
complexes in the cytoplasm.
Peripheral blood shows hairy cells with nuclei of different shapes.
Bone marrow aspirate
There is presence of dry tapQ due to presence of reticulin fibrils along with the leukemic
cells.
Bone marrow biopsy
It reveals infiltration by the cancer cells called as honeycomb appearanceQ and
leukemic cells have nucleus surrounded by cytoplasmic halo called as fried egg
appearanceQ which is diagnostic of hairy cell leukemia.
Clinical features are massive splenomegaly and less commonly
there is presence of hepatomegaly (note that lymphadenopathy is distinctly
rare in this disorder). Marrow failure contributes to pancytopenia resulting
in increased chances of infection, fatigue and easy bruisability in these
patients.
HODGKIN’S LYMPHOMA (HL)

It is a group of lymphoid neoplasms arising in a single node and spreads


from the nodes to spleen, then liver and finally bone marrow. Clinical
importance of this predictable route of spread is highlighted by the
importance of staging which determines prognosis as well guides the
choice of therapy of HL.

It is differentiated from non- Hodgkin’s lymphoma by the following


features:
Hodgkin’s lymphoma Non-Hodgkin’s lymphoma
• More often localized to a single axial • More frequent involvement of
group of nodes (cervical, mediastinal, multiple peripheral nodes
para-aortic)
• Orderly spread by contiguity • Non contiguous spread
• Mesenteric nodes and Waldeyer ring • Waldeyer ring and mesenteric
rarely involved nodes commonly involved
• Extra nodal involvement uncommon • Extranodal involvement common

There is presence of neoplastic giant cell called Reed-Sternberg cell


(derived from the germinal center B cell) which induces the accumulation
of reactive lymphocytes, macrophages and granulocytes.

The cause for the development of HL is inappropriate activation of


NF-κB usually induced by the latent membrane protein-1 of Ebstein Barr
virus (EBV) in majority of the cases.
The malignant cell is Reed Sternberg (RS) cell which is having an
“owl-eye” appearance due to the presence of symmetric (mirror image)
bilobed nucleus with prominent nucleoli surrounded by clear space. The
RS cells are positive for CD15 and CD30 for most subtypes except in
lymphocyte predominant HL in which the neoplastic cells stain for CD20
and BCL-6 and are negative for CD15 and CD30.
Fig. 8: Hodgkin lymphoma: Reed Stermberg cells having owl eye
appearance.

Clinical Features
Presence of painless enlargement of lymph nodes is the common
presenting symptom and is associated with fever (Pel Ebstein fever) and
night sweats in disseminated disease. A strange paraneoplastic syndrome
in HL is pain in the affected lymph nodes on consumption of alcohol. The
prognosis is directly related to the number of RS cells present.

CHRONIC MYELOPROLIFERATIVE DISORDERS

The common pathogenic feature of the myeloproliferative disorders is the


presence of mutated, constitutively activated tyrosine kinases which lead
to growth factor independent proliferation and survival of marrow
progenitor cells. These arise from the clonal proliferation of multipotent
stem cells which proliferate along the three cell lines (erythroid,
megakaryocytic and granulocytic) except in CML in which with the
pluripotent cell gives rise to myeloid cells.
Myeloproliferative disorders
• Chronic myelogenous leukemia
• Polycythemia vera
• Primary myelofibrosis
• Essential thrombocythemia
• Chronic eosinophilic leukemia
• Systemic Mastocytosis
• Stem cell leukemia
Depending on the predominant cell in the myeloproliferative disorder,
the following more frequently asked disorders are:
1. Chronic Myelogenous Leukemia (CML)

It has been discussed earlier with other leukemias.

2. Polycythemia Vera (PV)

This is a myeloproliferative disorder characterized by the increased


number of erythroid, granulocytic and megakaryocytic cells. Polycythemia
vera progenitor cells have markedly decreased requirements for
erythropoietin and other hematopoietic growth factors.

Clinical features
1. Increase in hematocrit and red cell mass contributing to sluggish blood
flow and even increased chances of thrombosis. These manifest in the
form of dusky cyanosis, visual disturbances, headache, dizziness, venous
thrombosis (causes Budd-Chiari syndrome due to hepatic vein
thrombosis), bowel infarction and stroke.
2. Increased basophils release histamine causing intense itching and
increased incidence of peptic ulcer
3. Hyperuricemia is seen due to increased cell turnover.
4. The patients also have splenomegaly due to extramedullary
hematopoiesis.
Investigations
• Blood shows elevated hemoglobin (Hb > 18 g %) and red cell count
(> 6 million/mm3; normal is 3.5-5.0 million/mm3), increased
hematocrit with decreased levels of erythropoietin. The last
differentiate it from secondary polycythemia in which serum
erythropoietin is elevated.
• Peripheral blood shows increased basophils and abnormal
platelets.
• Bone marrow is hypercellular having increased number of erythroid,
granulocytic and megakaryocytic cells. In later stage, there is
presence of myelofibrosis.

Recent info in exams


Proposed Revised WHO Criteria for Diagnosis of Polycythemia Vera*
Major criteria 1. Hemoglobin > 18.5 g/dL in men or > 16.5 g/dL in women
or evidence of increased red cell volume
2. Presence of JAK2 mutation
Minor criteria 1. Hypercellular bone marrow biopsy with panmyelosis
with prominent erythroid, granulocytic, and
megakaryocytic hyperplasia
2. Low serum erythropoietin level
3. Endogenous erythroid colony formation in vitro

Polycythemia vera is the most common of the chronic


myeloproliferative disorders. Its diagnosis requires presence of both the
major criteria and one minor criterion or the presence of the first major
criterion plus 2 minor criteria.
3. Essential Thrombocytosis

It is associated with activating mutation in JAK2 (more commonly) or


MPL, a receptor tyrosine kinase normally activated by thrombopoietin. This
is the stem cell disorder having increased proliferation of the
megakaryocytes in bone marrow and high platelet count in the blood (> 4.5
lakh/mm3; normal level is 1.5-4.5 lakh/mm3). It is usually a diagnosis of
exclusion.

It is strongly associated with activating point mutation in the tyrosine kinase JAK2
or MPL, the latter is receptor tyrosine kinase activated by thrombopoietin.

Clinical Features arise because of:


1. Non-functioning platelets: bleeding (mucosal, skin, post trauma).
2. Dysfunctional platelets: thrombosis (arterial thrombosis more common
than venous)resulting in headache, dizziness or ischemia of the digits.

Investigations

• Bone marrow is hypercellular with increase in number of giant megakaryocytes


along with dysmegakaryopoeisis. Erythroid and myeloid cell show only mild
hyperplasia, if at all.
• Blood shows elevated platelet count in the blood (diagnostic criteria is > 6
lakh/mm3), normal Hb levels and elevated LAP scores.
4. Idiopathic or Primary Myelofibrosis

It is characterized by the presence of marrow fibrosis associated with


extramedullary hemopoeisis in the spleen in old patients (usually more
than 60 years of age). It is associated with activating mutation in JAK2
(more commonly) or MPL. There is presence of neoplastic
megakaryocytes which are responsible for the release of fibrogenic factors
like PDGF (platelet derived growth factor) and TGF-β. These factors cause
progression of marrow fibrosis. This results in:

• Replacement of normal hematopoietic stem cells by fibrous tissue.


• Movement of hematopoietic stem cells to spleen and liver (responsible
for hepatosplenomegaly).
Both the above contribute to development of decreased cell count
resulting in the symptoms of fatigue, weight loss or bleeding episodes
along with hepatosplenomegaly. Hyperuricemia is seen due to increased
cell turnover.

Bone marrow aspiration reveals dry tap due to fibrosis of the bone marrow.

So, bone marrow biopsy is the investigation of choice for the


diagnosis which shows the presence of hypocellularity with
increased deposition of reticulin inside the marrow, abnormal
megakaryocytes and the characteristic finding of dilated marrow
sinusoids.
Peripheral blood findings include presence of nucleated red cells and immature white
cells (called as leukoerythroblastosis).
There is also presence of abnormal red cells called dacryocytes (or
tear drop RBC) formed due to damage to red cell membrane caused by
fibrous tissue in the marrow. Blood shows mild leukocytosis, decreased Hb
and hematocrit levels and elevated LAP scores.

Fig. 9: Tear Shaped Cell (T) in Myelofibrosis.

PLASMA CELL DYSCRASIAS

These are characterized by proliferation of B-cell clone which synthesizes


and secretes a single homogenous immunoglobulin or its fragments. The
entity includes the following conditions:
• Multiple myeloma (Plasma cell myeloma) – presents as multiple
masses in the skeletal system. Smoldering myeloma is an
asymptomatic subtype with high plasma M component.
• Waldenstrom’s macroglobulinemia – Caused by blood
hyperviscosity due to high level of IgM. It is seen in adults with
lymphoplasmacytic lymphoma.
• Heavy chain disease – Characterized by synthesis and secretion of
free heavy chain fragments and is seen in association with CLL/SLL,
Mediterranean lymphoma, lymphoplasmacytic lymphoma.
• Primary or immunocyte associated amyloidosis – Results from a
monoclonal proliferation of plasma cell secreting free light chains (most
commonly α isotype).
• Monoclonal Gammopathy of Undetermined Significance (MGUS) –
It is the most common symptomatic monoclonal gammopathy.Usually
asymptomatic disease seen in elderly patients. Rarely, it may progress
to symptomatic monoclonal gammopathy (most often multiple
myeloma).

1. Multiple Myeloma

It is a plasma cell cancer having skeletal involvement at multiple sites. The


most common karyotypic abnormalities in this condition are deletions of
13q and translocation involving Ig heavy chain locus on 14q.

Pathogenesis and clinical features in multiple myeloma


Diagnosis
In 2017 WHO diagnostic criteria for diagnosis of plasma cell myeloma are
Symptomatic plasma cell myeloma
Clonal bone marrow plasma cell percentage ≥10% or biopsy-proven plasmacytoma and
≥1 of the following myeloma-defining events:
a. Evidence of End-organ damage (CRAB)
– HyperCalcaemia: serum calcium > 11 mg/dL
– Renal insufficiency: Creatinine clearance < 40 ml/minute or senum creatinine > 2
mg/dL
– Anaemia: (Hb< 10g/dl or >2g/dl lower than normal)
– Bone lesions: ≥1 osteolytic lesion on skeletal radiography, CT, or PET/CT
b. >1 of the following biomarkers of malignancy:
Clonal bone marrow plasma cell percentage > 60%
– An involved-to-uninvolved serum free light chain ratio >100
– >1 focal lesion on MRI

The diagnosis can be made on the basis of blood, bone marrow and
urine findings as described the following flowchart:

2. Monoclonal Gammopathy of Undetermined Significance (MGUS)

It is characterized by the presence of M spike without associated disease


of the B cells. MGUS is the commonest cause of monoclonal gammopathy.
Around 1% of the patients with MGUS progress to develop multiple
myeloma per year. It is usually a diagnosis of exclusion. Patients of MGUS
have less than 3 g/dL of monoclonal protein in the serum and no Bence
Jones proteinuria.

3. Waldenstrom Macroglobulinemia/Lymphoplasmacytic Lymphoma

It is a B cell neoplasm presenting in 6th or 7th decade of life having features


similar to CLL/SLL and multiple myeloma.

Clinical features
These include non specific symptoms like fatigue, weakness, weight loss,
hepatosplenomegaly and cervical lymphadenopathy. The immunoglobulin
increases viscosity of the blood resulting in hyperviscosity syndrome
affecting CNS and retina characterized by the headache, dizziness, visual
disturbances etc. Abnormal globulins may interfere with platelet function
resulting in bleeding and cryoglobulins may lead to acrocyanosis and cold
urticaria.

Investigations

• Bone marrow reveals the presence of plasmacytoid lymphocytes’ infiltration. PAS


+ inclusions containing immunoglobulins are seen in the cytoplasm (called Russell
bodies) or in the nucleus (called Dutcher bodies) of the plasmacytoid cells.
• Blood investigations show anemia with atypical plasmacytoid lymphocytes. ESR is
elevated and rouleaux formation is seen. Immunoelectrophoresis reveals the
presence of ‘M’ spike composed of IgM.
• Immunophenotyping reveals the lymphocytic cells expressing B-cell markers like
CD20. These cells are negative for CD5 and CD10. The plasma cell secretes a
monoclonal immunoglobulin.

Langerhans Cell Histiocytosis (LCH)


The term histiocytosis is a broad term for a variety of proliferative disorders
of dendritic cells (DCs) or macrophages. Langerhans cell is a special type
of dendritic cell in the skin functioning as antigen presenting cell.
Langerhans cell histiocytosis (LCH) has the following entities:

1. Letterer-Siwe syndrome (multifocal multisystem LCH)


2. Pulmonary Langerhans’ cell histiocytosis: seen in adult smokers and may
regress on cessation of smoking.
3. Eosinophilic granuloma.
Fig. 10: Tennis-racket appearance of Langerhans cell.

• These three conditions are now considered different expressions of the


same basic disorder. The tumor cells in each are derived from dendritic
cells and express HLA-DR, S-100, and CD1a. They have abundant,
often vacuolated cytoplasm and vesicular nuclei containing linear
grooves or folds.
• The presence of Birbeck’s granules in the cytoplasm is characteristic.
These granules, under the electron microscope, have a pentalaminar,
rod like, tubular appearance and a dilated terminal end (tennis-racket
appearance) which contains the protein langerin.

LEUKEMIA: ALL, AML, CLL, CML

1. Flow cytometry is done in:


(AIIMS Nov 2012)
(a) Polycythemia
(b) Thrombocytosis
(c) Neutrophilia
(d) Lymphocytosis
2. Myelofibrosis leading to a dry tap on bone marrow aspiration is
seen with which of the following conditions?
(AIIMS Nov 2012)
(a) Burkitt’s lymphoma
(b) Acute erythroblastic leukemia
(c) Acute megakaryocytic leukemia
(d) Acute undifferentiated leukemia
3. Marker of myeloid cancers:
(AIIMS Nov 2012)
(a) S100
(b) HMB45
(c) Common leukocyte antigen
(d) Cyto-keratin
4. Adult patient presents with generalized lymphadenopathy and
blood film shows 70% immature looking lymphocytes. What
should be the next best investigation?
(a) Genotyping/karyotyping
(AIIMS Nov 2012)
(b) Immunophenotyping
(c) Bone marrow
(d) Peripheral smear study
5. Which of the following statements in context of leukemias is true?
(AIIMS May 2012)
(a) Chronic myeloid leukemia occurs in less than 50 years of age
(b) Hairy cell leukemic in less than 50 years has a good prognosis
(c) Acute lymphoid leukemic in less than 1 year has a poor
prognosis
(d) Chronic lymphocytic leukemia occurs in less than 50 years of
age
6. In an ablated animal, myeloid series cells are injected. Which of
following is seen after incubation period?
(AIIMS May 2012)
(a) Fibroblast
(b) T lymphocytes
(c) RBC
(d) Hematopoetic stem cell
7. A young boy came with dyspnea and was found to have a
mediastinal mass. Which of the following is known to produce
mediastinal lymphadenopathy?
(a) Diffuse large B cell Lymphoma
(AI 2012)
(b) B cell rich T cell lymphoma
(c) Mediastinal rich B cell lymphoma
(d) T cell Lymphoblastic ALL
8. Which of the following is the least likely to be a pre-leukaemic
condition?
(AIIMS Nov 2011)
(a) Paroxysmal nocturnal haemoglobinuria
(b) Aplastic anaemia
(c) Paroxysmal cold haemoglobinuria
(d) Myelodysplastic syndrome
9. A 6-year-old child presents with pallor that required two blood
transfusions previously. He has now developed fever and
petechial haemorrhages. His hemoglobin is is 9 g/dL, platelet
count is 20,000/mm3 and TLC is 60,000/mm3. Flow cytometry
reveals the cells to be CD10+ve, MPO+ ve, CD 19 + ve, CD 33 –
ve, CD 117 + ve, and CD3 – ve. Which of the following is the
most likely diagnosis?
(AIIMS Nov 2011)
(a) ALL
(b) AML
(c) Mixed phenotypic leukaemia
(d) Undifferentiated leukaemia
10. In a patient with acute leukemia, immunophenotype pattern is CD
19+ve, CD 10+ve, CD33+ve, CD 13+ve. He may probably have:
(AIIMS May 2004)
(a) Biphenotypic leukemia
(b) ALL
(c) AML-M2
(d) AML-M0
11. All the following are poor prognostic indicators in AML except:
(AIIMS Nov. 2010)
(a) Inv 16
(b) Complex karyotype
(c) AML M7
(d) Deletion 7q
12. Most specific marker for myeloid series is:
(a) CD 34
(b) CD 45
(AIIMS May 2010)
(c) CD 99
(d) CD 117
13. t (2,8) is associated with:
(AI 2010)
(a) T cell ALL
(b) B cell ALL
(c) CML
(d) CLL

14. ALL L3 morphology is a malignancy arising from which cell


lineage?
(AI 2007)
(a) Mature B-cell
(b) Precursor B-cell
(c) Immature T-cell
(d) Mixed B cell and T-cell
15. Non-specific esterase is positive in all the categories of AML
except:
(AI 2007)
(a) M3
(b) M4
(c) M5
(d) M6
16. Which of the following statements pertaining to leukemia is
correct?
(AI 2005)
(a) Blasts of acute myeloid leukemia are typically Sudan black
negative
(b) Blasts of acute lymphoblastic leukemia are typically
myeloperoxidase positive
(c) Low leucocyte alkaline phosphatase score is characteristically
seen in blastic phase of chronic myeloid leukemia
(d) Tartarate resistant acid phosphatase positivity is typically seen in
hairy cell leukemia
17. Which is the most common cytogenetic abnormality in adult
myelodysplastic syndrome (MDS)?
(AI 2004)
(a) Trisomy 8
(b) 20q–
(c) 5 q–
(d) Monosomy 7
18. Which of the following is a pan-T lymphocyte marker?
(a) CD2
(b) CD3
(AI 2003)
(c) CD19
(d) CD25
19. B cell marker are all except: (AIIMS Nov 09)
(a) CD 19
(b) CD 20
(c) CD 10
(d) CD135
20. Which of the following is having poor prognosis in ALL:
(AIIMS Nov 09)
(a) TLC 4000-10000
(b) Age < 2 yrs
(c) Presence of testicular involvement at presentation
(d) Presence of blasts in peripheral smear
21. All of the following are good prognostic factors for acute
lymphoblastic leukemia except:
(AIIMS Nov 2008)
(a) Age of onset between 2-8 years
(b) Initial WBC count less than 50000
(c) Hyperdiploidy
(d) t(9:22), t(8:14), t(4:11)
22. AML with gum infiltration, hepato- splenomegaly is most likely to
be:
(AIIMS May 2008)
(a) ALL
(b) M3
(c) M2
(d) M4
23. Marker for granulocytic sarcoma:
(AIIMS May 2008)
(a) CD33
(b) CD38
(c) CD117
(d) CD137
24. All are B-cell marker except:
(AIIMS May 2007)
(a) CD-15
(b) CD-19
(c) CD-21
(d) CD-24
25. Acid phosphatase is specific to which of the following cells:
(AIIMS Nov 2006)
(a) Monocyte
(b) T-lymphocyte
(c) B-lymphocyte
(d) Myelocytes
26. A peripheral smear with increased neutrophils, basophils,
eosinophils, and platelets is highly suggestive of:
(AIIMS May 2006)
(a) Acute myeloid leukemia
(b) Acute lymphoblastic leukemia
(c) Chronic myelogenous leukemia
(d) Myelodysplastic syndrome
27. A 17-year-old boy presented with TLC of 138 × 109/L with 80%
blasts on the peripheral smear. Chest X-ray demonstrated a
large mediastinal mass. Immunophenotyping of this patient’s
blasts would most likely demonstrate:
(AIIMS May 2006)
(a) No surface antigens (null phenotype)
(b) An immature T-cell phenotype (Tdt/CD34/CD7 positive)
(c) Myeloid markers, such as CD13, CD33 and CD15
(d) B cell markers, such as CD19, CD20 and CD22
28. A 15-year-old boy presented with one day history of bleeding
gums, subconjunctival bleed and purpuric rash. Investigations
revealed the following results: Hb-6.4 gm/dL; TLC-26,500/mm3
Platelet 35,000 mm3; prothrombin time-20 sec with a control of
13 sec; partial thromboplastin time-50 sec; and Fibrinogen
10mg/dL. Peripheral smear was suggestive of acute
myeloblastic leukemia. Which of the following is the most
likely?
(AIIMS May 2006)
(a) Myeloblastic leukemia without maturation
(b) Myeloblastic leukemia with maturation
(c) Promyelocytic leukemia
(d) Myelomonocytic leukemia
29. Poor prognostic factor for ALL is:
(AI 2011)
(a) Hyperdiploidy
(b) t(9;22) t(4;11)
(c) Age at presentation is 2-8 yrs
(d) Total Leucocyte count <50000
30. Which of the following is not compatible with a diagnosis of
chronic myelomonocytic leukemia?
(AIIMS Nov 2003)
9
(a) Peripheral blood monocytosis more than 1×10 /L
(b) Absence of Philadelphia chromosome
(c) More than 20% blasts in blood or bone marrow
(d) Absent or minimal dysplasia in myeloid lineages
31. A 60-year-old man presented with fatigue, weight loss and
heaviness in left hypochondrium for 6 months. The hemogram
3
showed Hb, 10gm/dL, TLC 5 lakhs/mm , platelet count 4
3
lakhs/mm , DLC, neutrophil 55%, lymphocytes 4%, monocytes
2%, basophils 6%, metamyelocytes 10%, myelocytes 18%,
promyelocytes 2% and blasts 3%. The most likely cytogenetic
abnormality in this case is:
(AIIMS May 2003)
(a) t (1:21)
(b) t (9:22)
(c) t (15, 17)
(d) Trisomy 21
32. A 42-year old man was referred with a 2 week history of fever
weakness and bleeding gum. Peripheral smear showed
pancytopenia. The bone marrow examination revealed 26%
blasts frequency exhibiting Auer rods and mature myeloid cells.
An occasional neutrophil with pseudo Pelger-Huet anomaly was
also detected. Which of the following cytochemical stains is
most likely to be positive?
(a) Acid phosphatase
(AIIMS Nov 2002)
(b) Non-specific esterase
(c) Myeloperoxidase
(d) Toluidine blue
33. AML with worst prognosis:
(AIIMS May 2007)
(a) 8/21 translocation
(b) Inversion 16
(c) Normal cytogenetics
(d) Monosomy 7
34. Pancytopenia with cellular marrow is seen in all except:
(a) Megaloblastic anemia
(AIIMS Nov 2006)
(b) Myelodysplasia
(c) Paroxysmal nocturnal hemoglobinuria
(d) G6PD deficiency
35. Highest LAP score is seen in:
(PGI Dec 2001)
(a) CML
(b) Polycythemia vera
(c) PNH
(d) Pregnancy
(e) Lymphoma
36. Causes of eosinophilia are:
(PGI June 2004)
(a) Hodgkin’s disease
(b) Filariasis
(c) MI
(d) HIV infection
37. Sideroblasts are seen in:
(PGI Dec 2005)
(a) Thalassemia
(b) Myelofibrosis
(c) Alcoholism
(d) Iron overload
38. Absolute monocytosis is seen in:
(PGI Dec 2006)
(a) Infectious mononucleosis
(b) Kala-azar
(c) TB
(d) Brucellosis
39. Aplastic anemia can progress to: (PGI Dec 01)
(a) AML
(b) Myelodysplastic syndrome
(c) CLL
(d) PNH
(e) Iron deficiency anemia
40. True about aplastic anemia:
(PGI June 2005)
(a) Splenomegaly
(b) Nucleated RBC in peripheral blood.
(c) Reticulocytopenia
(d) Thrombocytopenia
(e) Neutropenia
41. Leucocytosis is seen in all except:
(Delhi 2010)
(a) Brucellosis
(b) Acute MI
(c) Typhoid
(d) Diphtheria

MOST RECENT QUESTIONS


42. The blast cells of acute lymphocytic leukemia in childhood
contain:
(a) Surface antigen
(b) CALLA Ag
(c) Antibodies to WBC
(d) Thrombocytosis
43. “Smudge cells” in the peripheral smear are characteristic of:
(a) Chronic myelogenous leukemia
(b) Chronic lymphocytic leukemia
(c) Acute myelogenous leukemia
(d) Acute lymphoblastic leukemia
44. Chromosomal translocation characteristic in acute
promyelocytic leukemia is:
(a) t ( 15: 17)
(b) t ( 22: 9)
(c) t ( 21: 17)
(d) t ( 8: 21)
45. TRUE abut acute myelogenous leukemia
(a) Philadelphia chromosome is seen
(b) Auer bodies are seen
(c) Common in childhood
(d) Peroxidase negative granules
46. In CML, serum vitamin B12 level is
(a) Slightly decreased
(b) Normal
(c) Markedly decreased
(d) Increased
47. BCR-ABL hybrid gene is present in
(a) Burkitt’s lymphoma
(b) Retinoblastoma
(c) Breast carcinoma
(d) CML
48. The difference between leukemia and leukemoid reaction is done
by:
(a) Total leukocyte count
(b) Leucocyte alkaline phosphatase
(c) Erythrocyte sedimentation rate
(d) Immature cells
49. Neutropenia is caused by all except:
(a) Typhoid fever
(b) Viral infection
(c) Brucellosis
(d) Glucocorticoids
50. Basophilic leucocytosis occurs in:
(a) AML
(b) ALL
(c) CML
(d) CLL
51. All are causes of splenomegaly except:
(a) Malaria
(b) Kala azar
(c) Hemolytic anemia
(d) Aplastic anemia
52. In myelodysplastic syndrome, the following statement is
incorrect:
(a) Platelet counts are normal or elevated
(b) Leucocyte counts are normal or elevated
(c) Hypocellular bone marrow
(d) Refractory anemia
53. Leukoerythroblastic picture is seen in all except:
(a) Myelofibrosis
(b) Secondary malignancy of bone marrow
(c) Thalassemia
(d) Gaucher disease
54. A round cell having, fine nuclear chromatin, prominent nucleoli
and fine azurophillic granule, cell is:
(a) Myeloblast
(b) Lymphoblast
(c) Monoblast
(d) None
55. Autoimmune hemolytic anemia is seen in:
(a) ALL
(b) AML
(c) CLL
(d) CML
56. The presence of the Philadelphia chromosome is associated with
a worse prognosis in patients with which of the following
diseases?
(a) Acute lymphoblastic leukemia
(b) Acute myelogenous leukemia
(c) Chronic lymphocytic leukemia
(d) Chronic myelogenous leukemia
57. Examination of a peripheral blood smear demonstrates leukemia
composed of small mature lymphocytes without blast forms.
Which of the following is the most likely age of this patient?
(a) 1 year
(b) 20 years
(c) 45 years
(d) 65 years
58. Which of the following is associated with good prognosis in
ALL?
(a) T cell line
(b) Philadelphia chromosome
(c) Hyperdiploidy
(d) Hypodiploidy
59. B cell ALL is due to which of the following?
(a) T cells
(b) Immature B cells
(c) Immature T cells
(d) Both T and B cells
60. CD-10 is seen in:
(a) ALL
(b) CLL
(c) GCL
(d) CML
61. Dohle bodies in neutrophils are comprising of:
(a) Mitochondria
(b) Golgi apparatus
(c) Lysosomes
(d) Dilated endoplasmic reticulum
62. What is the chromosomal translocation in AML M3:
(a) t (18, 21)
(b) t (15, 17)
(c) t (8, 21)
(d) t (9, 11)
63. Reed Sternberg cells are found in:
(a) Hodkin’s disease
(b) Sickle cell anaemia
(c) Thalassemia
(d) CML
64. Specific stain for myeloblasts is:
(a) Sudan black
(b) PAS
(c) Myeloperoxidase (MPO)
(d) LAP
65. Dohle bodies are seen in which of the following?
(a) Multiple myeloma
(b) May-Heggline anomaly
(c) Waldenstorm Macroglobulinemia
(d) Lymphoma
66. Auer rods are seen in:
(a) Lymphoblast
(b) Myeloblast
(c) Erythroblast
(d) Megakaryoblast
67. The peripheral blood eosinophil count in Eosinophilia myalgia
syndrome is usually:
(a) Between 500 to 2000 cells microilter
(b) 2000 to 5000 cells/microliter
(c) Less than 500 cells/microliter
(d) More than 5000 cells/microliter
68. Most common ALL subtype?
(a) Pre B cell
(b) Pre T cell
(c) T cell
(d) B cell
69. Which of these is the most important prognostic factor in ALL?
(a) Hyperploidy
(b) Total leucocyte count greater than 50,000
(c) Age
(d) Response to steroids
70.ALL-L3 resembles:
(a) Mantle cell lymphoma
(b) MDS
(c) Burkitt’s lymphoma
(d) AML
71. AML – bad prognostic factor is:
(a) Preceding MDS
(b) Inv 16
(c) Auer rods
(d) Type M4
72. Which variety of AML is associated with good prognosis?
(a) M0
(b) M3
(c) M6
(d) M7

NON HODGKIN LYMPHOMA

73. Progressive transformation of germinal centres (PTGC) is a


precursor lesion of:
(DPG 2011)
(a) Hodgkin’s Lymphoma, nodular sclerosis
(b) Hodgkin’s Lymphoma, mixed cellularity
(c) Anaplastic large cell Lymphoma
(d) Peripheral T cell Lymphoma
74. Eosinophilic Abscess in lymph node is characteristically seen in:
(DPG 2011)
(a) Kimura’s disease
(b) Hodgkin’s Lymphoma
(c) Tuberculosis
(d) Sarcoidosis
75. A 50 years old male presents with massive splenomegaly. His
differential diagnosis will include all, except:
(a) Chronic myeloid leukemia
(DPG 2011)
(b) Polycythemia rubra vera
(c) Hairy cell leukemia
(d) Aplastic anemia
76. Burkitt’s lymphoma is associated with:
(AI 2010)
(a) t (8:14)
(b) t (9:22)
(c) t (11; 14)
(d) t (8:21)
77. All of the following immunohistochemical markers are positive in
the neoplastic cells of granulocytic sarcoma, except:
(AI 2006)
(a) CD 45 RO
(b) CD 43
(c) Myeloperoxidase
(d) Lysozyme
78. Mantle cell lymphomas are positive for all of the following,
except:
(AI 2006)
(a) CD 23
(b) CD 20
(c) CD 5
(d) CD 43
79. The classification proposed by the International Lymphoma
Study Group for non-Hodgkin lymphoma is:
(a) Kiel classification
(AI 2005)
(b) REAL classification
(c) WHO classification
(d) Rappaport classification
80. A 48-year-old woman was admitted with a history of weakness
for two months. On examination, cervical lymph nodes were
found enlarged and spleen was palpable 2 cm below the costal
margin. Her hemoglobin was 10.5 g/dl, platelet count 2.7×109/L
and total leukocyte count 40×109/L, which included 80% mature
lymphoid cells with coarse clumped chromatin. Bone marrow
revealed a nodular lymphoid infiltrate. The peripheral blood
lymphoid cells were positive for CD 19, CD 20 and CD 23 and
were negative for CD 79B and FMC-7.
The histopathological examination of the lymph node in this patient
will most likely exhibit effacement of lymph node architecture
by:
(AI 2005)
(a) A pseudofollicular pattern with proliferation centers
(b) A monomorphic lymphoid proliferation with a nodular pattern
(c) A predominantly follicular pattern
(d) A diffuse proliferation of medium to large lymphoid cells with high
mitotic rate
81. A four-year-old boy was admitted with a history of abdominal
pain and fever for two months, maculopapular rash for ten days,
and dry cough, dyspnea and wheezing for three days. On
examination, liver and spleen were enlarged 4 cm and 3 cm
respectively below the costal margins. His hemoglobin was 10.0
g/dl, platelet count 37×109/L and total leukocyte count 70×109/L,
which included 80% eosinophils. Bone marrow examination
revealed a cellular marrow comprising 45% blasts and 34%
eosinophils and eosinophilic precursors. The blasts stained
negative for myeloperoxidase and nonspecific esterase and
were positive for CD 19, CD 10, CD 22 and CD 20. Which one of
the following statements is not true about this disease?
(AI 2005)
(a) Eosinophils are not part of the neoplastic clone
(b) t (5:14) rearrangement may be detected in blasts
(c) Peripheral blood eosinophilia may normalize with chemotherapy
(d) Inv (16) is often detected in the blasts and the eosinophil
82. All of the following statements about hairy cell leukemia are true
except:
(AI 2004)
(a) Splenomegaly is conspicuous
(b) Results from an expansion of neoplastic T-lymphocytes
(c) Cells are positive for Tartarate Resistant Acid phosphatase
(d) The cells express CD25 consistently
83. True about Burkitt’s lymphoma: (AIIMS Nov 09)
(a) CD 34 and surface Ig both +ve
(b) CD 34 negative but surface Ig+
(c) CD 34 positive but surface Ig -
(d) CD 34 and surface Ig both (–) ve
84. Which of the following is false? (AIIMS Nov 09)
(a) Bcl-6 is associated with Burkitts lymphoma
(b) Bcl-2 is associated with follicular lymphoma
(c) CD-10 is associated with mantle cell lymphoma
(d) CD 34 is associated with Diffuse large B Cell Lymphoma
85. Post transplant lymphoma occurs due to proliferation of which of
the following cells:
(AIIMS Nov 2006)
(a) T-cell
(b) B-cell
(c) NK cell
(d) Monocyte
86. Which of the following statements on lymphoma is not true?
(AIIMS May 2006)
(a) A single classification system for Hodgkin’s disease (HD) is
almost universally accepted
(b) HD more often tends to remain localized to a single group of
lymph nodes and spreads by contiguity
(c) Several types of non Hodgkin’s lymphoma (NHL) may have a
leukemic phase
(d) In general follicular (nodular) NHL has worse prognosis
compared to diffuse NHL

MOST RECENT QUESTIONS

87. Mantle cell lymphomas are positive for all of the following
except:
(a) CD23
(b) CD20
(c) CD5
(d) Cyclin D1
88. Over-expression of BCL-2 proteins occurs in:
(a) Burkitt’s lymphoma
(b) Follicular lymphoma
(c) Diffuse large B-cell lymphoma
(d) Small lymphocytic lymphoma
89. ‘Starry sky’ appearance is seen in:
(a) Burkitt’s lymphoma
(b) Mantle cell lymphoma
(c) Extra nodal marginal zone B-cell lymphoma of MALT type
(d) Chronic myeloid leukemia
90. All are B cell lymphomas except:
(a) Burkitt’s lymphoma
(b) Mycosis fungoides
(c) Mantle cell lymphoma
(d) Follicular cell lymphoma
91. True statement regarding non Hodgkin’s lymphoma of follicular
type is:
(a) Increased incidence in adolescents
(b) Predominantly in males
(c) Prognosis is better than in diffuse type
(d) Affects T cells only
92. MALToma is:
(a) B-cell lymphoma
(b) APUDoma
(c) NK cell tumor
(d) T cell lymphoma
93. Which of the following is the most common site for extranodal
lymphoma?
(a) Esophagus
(b) Stomach
(c) Intestine
(d) Skin
94. Cell of origin of hairy cell leukemia is:
(a) T cell
(b) B cell
(c) NK cell
(d) Dendritic cell
95. Which one of the following Non-Hodgkin Lymphomas is
aggressive?
(a) Follicular Lymphoma
(b) Burkitt Lymphoma
(c) Small lymphocytic lymphoma
(d) Lymphoplasmacytic lymphoma
96. The low grade non- Hodgkin’s lymphoma is:
(a) Follicular small cleaved lymphoma
(b) Follicular large cell lymphoma
(c) Diffuse large cell lymphoma
(d) Lymphoblastic lymphoma
97. Which of the following is the most common non Hodgkin
lymphoma?
(a) Follicular lymphoma
(b) Anaplastic large cell lymphoma
(c) Diffuse large B cell lymphoma
(d) Marginal zone lymphoma
98. Most common Non-Hodgkin’s lymphoma of orbit:
(a) B cell
(b) T cell
(c) NK cell
(d) Plasma cell
99. Marginal lymphoma is type of:
(a) B cell lymphoma
(b) T cell lymphoma
(c) NK cell lymphoma
(d) Hodgkin lymphoma
100. Which of the following is the marker of mantle cell cancer?
(a) CD5 +, CD25 –
(b) CD 5 +, CD 10 +
(c) CD 5 +, CD 23 +
(d) CD 5 +, CD 23 –
101. Mycosis fungoides is:
(a) Fungal infections of skin
(b) Leukemia
(c) Exfoliative erythroderma
(d) Cutaneous lymphoma
102. Histological presence of “Hallmark Cells” with horse shoe-like
or embryoid like nuclei and voluminous cytoplasm are seen in:
(a) Anaplastic large cell lymphoma (ALK positive)
(b) Familial Medullary Carcinoma
(c) Familial Neuroblastoma
(d) Lymphocyte predominance type Hodgkin’s lymphoma
103.Prevalence of burkitt lymphoma is highest in?
(a) Australia
(b) Africa
(c) Asia
(d) America
104. Cells seen in cutaneous T cell lymphoma are called as
(a) Councilman body
(b) Barr body
(c) Sezary cells
(d) Dohle body
105. Not a B cell lymphoma
(a) Mycosis fungoides
(b) CLL
(c) Hairy cell leukemia
(d) Mantle cell lymphoma
106. Sezary cells show which type of nucleus?
(a) Pleomorphic
(b) Round
(c) Eosinophilic
(d) Cerebriform

HODGKIN LYMPHOMA

107. Classical markers for Hodgkin’s disease are:


(a) CD 15 and CD 30
(AI 2008)
(b) CD 15 and CD 22
(c) CD 15 and CD 20
(d) CD 20 and CD 30
108. All of the following are the good prognostic features for
Hodgkin’s disease except:
(AI 2004)
(a) Hemoglobin > 10 gm/dl
(b) WBC count < 15000/mm3
(c) Absolute lymphocyte count < 600/µl
(d) Age < 45 years
109. The lymphocytic and histiocytic variant of Reed-Sternberg cell
is seen in:
(AIIMS Nov 2005)
(a) Follicular center lymphoma
(b) Lymphocyte depleted Hodgkin’s disease
(c) Nodular sclerosis Hodgkin’s disease
(d) Lymphocyte predominant Hodgkin’s disease
MOST RECENT QUESTIONS
110. Which cell is not seen in Hodgkins lymphoma:
(a) Reed Sternberg cell
(b) Lacunar cell
(c) L and H cell
(d) Langerhan’s cell
(e) Hodgkin cell
111. The sub-type of Hodgkin’s lymphoma characteri zed by L and H
cells is:
(a) Nodular sclerosis
(b) Mixed cellularity
(c) Lymphocyte depletion
(d) Lymphocyte predominant
112. An elderly patient presented with hypercellular bone marrow,
peripheral blood smear shows pancytopenia, and 15%
myeloblast cells. Most likely diagnosis is:
(a) Myelodysplastic syndrome
(b) Blast crisis in CML
(c) AML
(d) Polycythemia vera
113. The subtype of Hodgkin’s disease, which is histogenetically
distinct from all the other subtypes, is:
(a) Lymphocyte predominant
(b) Nodular sclerosis
(c) Mixed cellularity
(d) Lymphocyte depleted
114. ‘Popcorn cells’ are seen in which type of Hodgkin’s disease?
(a) Lymphocyte dominant
(b) Lymphocyte depleted
(c) Nodular sclerosis
(d) Mixed type
115. In a 45-year-old female presenting with painless supraclavicular
lymphadenopathy, biopsy was taken. It revealed the presence of
binucleated acidophlic owl eye appearance with clear
vacuolated space. The cell was CD15 and CD30 positive. Which
is the most likely diagnosis?
(a) Lymphocyte predominant Hodgkin lymphoma
(b) Nodular sclerosis Hodgkin lymphoma
(c) Mixed cellularity Hodgkin lymphoma
(d) Lymphocyte depleted Hodgkin lymphoma
116. Which Hodgkin’s disease is associated with best prognosis:
(a) Lymphocyte depletion
(b) Mixed cellularity
(c) Lymphocytic predominance
(d) Nodular sclerosis
117. Lacunar cells are seen in which type of Hodgkin’s lymphoma:
(a) Lymphocyte predominance
(b) Lyphocyte depletion
(c) Nodular sclerosis
(d) Mixed cellularity

MYELOPROLIFERATIVE DISORDERS

118. Splenomegaly is associated with all except:


(AI 2012)
(a) CML
(b) Polycythemia vera
(c) Essential thrombocythemia
(d) Primary myelofibrosis
119. Essential criteria for polycythemia vera according to WHO is:
(AI 2010)
(a) Low EPO
(b) JAK 2 mutation
(c) Bone marrow showing panmyelosis
(d) MPL point mutation

MOST RECENT QUESTIONS


120. Which of the following is not a chronic myeloproliferative
disorder?
(a) Polycythemia vera
(b) Myeloid metaplasia
(c) CML
(d) Essential thrombocytopenia
121. Leucocyte alkaline phosphatase (LAP) is raised in all conditions
except:
(a) Myelofibrosis
(b) Essential thrombocythemia
(c) Chronic myeloid leukemia
(d) Polycythemia
122. Leukoerythroblastic reaction is seen in the following except:
(a) Secondaries in bone
(b) Multiple myeloma
(c) Hemolytic anemia
(d) Lymphoma
123. Increase in alkaline phosphatase is seen in:
(a) Chronic myeloid leukemia; CML
(b) Leukemoid reaction
(c) Eosinophilia
(d) Malaria
124. Polycythemia is absolute venous haematocrit of more than:
(a) 45%
(b) 55%
(c) 65%
(d) 70%
125. CD marker of histiocytosis is:
(a) CD 1a
(b) CD 1b
(c) CD 1c
(d) CD 1d
126. Shape of Birbeck granules is which of the following?
(a) Hockey stick
(b) Bat
(c) Ball
(d) Tennis racket
127. One of the following is not a myelo-proliferative disorder?
(a) Essential thrombocytosis
(b) Myelofibrosis with myeloid metaplasia
(c) Acute myeloblastic leukemia
(d) Chronic myeloid leukemia
128. Isolated deletion of which chromosome is associated with
myelodysplastic syndrome?
(a) 2q
(b) 5q
(c) 8q
(d) 11q

PLASMA CELL DYSCRASIAS


129. Which of the following is the least common presentation of
multiple myeloma?
(AI 2012)
(a) Anemia
(b) Hyperviscosity
(c) Bone pains
(d) Infection
130. Which of the following metabolic abnormality is seen in multiple
myeloma?
(DPG 2011)
(a) Hypernatremia
(b) Hypokalemia
(c) Hypercalcemia
(d) Hyperphosphatemia
131. Lymphoplasmacytoid lymphoma is associated with:
(AI 2010)
(a) IgG
(b) IgA
(c) IgD
(d) IgM
132. Which of the following statement is not true?
(AI 2005)
(a) Patients with IgD myeloma may present with no evident M-spike
on serum electrophoresis.
(b) A diagnosis of plasma cell leukemia can be made if circulating
peripheral blood plasma blasts comprise 14% of peripheral blood
9
white cells in a patient with white blood cell count of 1 × 10 /L
9
and platelet count of 88 × 10 /L
(c) In smoldering myeloma plasma cells constitute 10-30% of total
bone marrow cellularity
(d) In a patient with multiple myeloma, a monoclonal light chain may
be detected in both serum and urine
133. Which of the following is not a minor diagnostic criterion for
multiple myeloma? (AIIMS Nov 08, 10)
(a) Lytic bone lesions
(b) Plasmacytosis greater than 20%
(c) Plasmacytoma on biopsy
(d) Monoclonal globulin spike on serum electrophoresis of < 2.5 g/dl
for IgG, < 1.5 g/dl for IgA)
134. A-3-year old female child presented with skin papules. Which of
the following is a marker of Langerhan’s cell histiocytosis?
(AIIMS Nov 2007)
(a) CD 1a
(b) CD 3
(c) CD 68
(d) CD 57

135. A 70-year-old male has a pathologic fracture of femur. The


lesion appears a lytic on X-rays film with a circumscribed
punched out appearance. The curetting from fracture site is
most likely to show which of the following?
(AIIMS May 2006)
(a) Diminished and thinned trabecular bone
(b) Sheets of atypical plasma cells
(c) Metastatic prostatic adenocarcinoma
(d) Malignant cells forming osteoid bone
136. Hyperviscosity is seen in:
(PGI Dec 2003)
(a) Cryoglobulinemia
(b) Multiple myeloma
(c) MGUS
(d) Lymphoma
(e) Macroglobulinemia
137. Hyperviscosity syndrome is seen in:
(a) NHL
(PGI Dec 2004)
(b) Waldenstrom’s macroglobulinemia
(c) Multiple myeloma
(d) Hodgkin’s lymphoma
(e) Acute promyelocytic leukemia
138. True about Langerhan’s cell histiocytosis is:
(a) CD 68+
(PGI Dec 2005)
(b) CD 1+
(c) Birbeck’s granules are pathognomic
(d) Proliferation of antigen presenting cells
(e) Resembles dendritic cells

MOST RECENT QUESTIONS


139. Finding of multiple myeloma in kidney are all except:
(a) Tubular casts
(b) Amyloidosis
(c) Wire loop lesions
(d) Renal tubular necrosis
140. Proliferation and survival of myeloma cells are dependent on
which of the following cytokines?
(a) IL-1
(b) IL-6
(c) IL-2
(d) IL-5
141. Plasma cell dyscrasias include all of the following except:
(a) Waldenstorm’s macroglobulinemia
(b) Heavy chain disease
(c) Monoclonal gammopathy of uncertain significance
(d) Systemic lupus erythematosus
142. Histiocytosis is NOT associated with:
(a) Spontaneous fractures
(b) Cutaneous eruptions
(c) Bone marrow suppression
(d) No lymphadenopathy

143. M-spike in multiple myeloma is due to?


(a) IgM
(b) IgA
(c) IgG
(d) None of these

144. Birbeck’s granule is found in:


(a) Langerhans cell
(b) Langhans giant cell
(c) Lepra cell
(d) Clue cell
145. Which of the following is not a characteristic feature of multiple
myeloma?
(a) Increased Ig levels in serum
(b) Positive ANA
(c) Plasmacytosis
(d) M spike on electrophoresis
146. Which histiocytosis involves the bones:
(a) Malignant
(b) Langherhans
(c) Sinus histiocytosis
(d) Option not recalled
147. Beta-2 microglobulin is a tumor marker for:
(a) Multiple myeloma
(b) Lung cancer
(c) Colonic neoplasm
(d) Choriocarcinoma
148. Russell bodies are found in which of the following conditions?
(a) Multiple Myeloma
(b) Gonadal tumor
(c) Parkinsonism
(d) Intracranial neoplasms
149. In Langerhans cell histiocytosis, the characteristic abnormality
seen on microscopy is:
(a) Birbecks granules
(b) Foamy macrophages
(c) Giant cells
(d) Plasma Cells
150. For the diagnosis of hypereosinophilia syndrome, after
exclusion of all parasitic infections and in the presence of end
organ damage, the absolute eosinophil count needed for the
diagnosis is:
(AIIMS Nov 2016)
(a) 500
(b) 1000
(c) 1500
(d) 2000

1. Ans. (d) Lymphocytosis
(Ref: Robbins 8th/324)
Flow cytometry can
rapidly and quantitatively
measure several
individual cell
characteristics, such as
membrane antigens and
the DNA content of
tumor cells.
Flow cytometry has also
proved useful in the
identification and
classification of tumors
arising from T and B
lymphocytes and from
mononuclear-phagocytic
cells.
In flow cytometry, it is
observed that large
objects will refract more
light than small objects
leading to high forward
scatter signals and vice
versa. So, forward
scatter in flow
cytometry denotes cell
size….. AIIMS May 15
question

2. Ans. (c) Acute megakaryocytic leukemia. (Ref:


Robbins 8th/622, 9/e p612, Wintrobe’s 12th/1857-8)
Direct quote from Robbins… ‘In some AMLs, blasts show
megakaryocytic differentiation, which is often accompanied by
marrow fibrosis caused by the release of fibrogenic cytokines’

Acute megakaryocytic leukemia is the most common variant of AML associated


with Down syndrome. The release of PDGF (platelet derived growth factor) is
responsible for marrow fibrosis.

3. Ans. (c) Common leukocyte antigen


(Ref: Robbins 8th/600, 9/e p590)
• CD45 is present on all the leukocytes; it is also known as
leukocyte common antigen (LCA).
4.Ans. (b) Immunophenotyping
(Ref: Robbins 9/e p593)
Adult patient presenting with generalized lymphadenopathy and blood film
shows 70% immature looking lymphocytes is highly suggestive of
chronic lymphocytic leukemia.
Immnophenotyping can be one of the best ways to differentiate
between CLL and other B cell neoplasms.
Important points about CLL
• Most of the patients are often asymptomatic at diagnosis. When
symptoms appear, they are nonspecific and include easy
fatigability, weight loss, and anorexia. Hepatosplenomegaly and
generalized lymphadenopathy are present in 50% to 60% of
symptomatic patients.
• The immunophenotype of CLL is distinct. The tumor cells
express the pan-B cell markers CD19 and CD20, as well as
CD23 and CD5, the latter a marker that is found on a small
subset of normal B cells. Low-level expression of surface Ig
(usually IgM or IgM and IgD) is also typical.
5. Ans. (c) Acute lymphoid leukemic in less than 1 year has a poor
prognosis
(Ref: Robbins 9/e p592, Wintrobe 12th)
Prognostic factors in ALL have been discussed in text
Explaining other options,
• Chronic myeloid leukemia occurs beyond 50 years of age…
Robbins 8th/
• Hairy cell leukemic is present in median age of 55 years and has
M:F ratio of 5:1. HCL tends to follow an indolent course. For
unclear reasons, the tumor cells are exceptionally sensitive to
particular chemotherapeutic regimens, which produce long-
lasting remissions. The overall prognosis is excellent. So, the
condition is not having additional increase in improvement with
age less than 50 years.
• The median age of diagnosis of Chronic lymphocytic leukemia is
60 years and there is a 2:1 male predominance.…Robbins
8th/603
6. Ans. (c) RBC
(Ref: Robbins 8th/592-593, 9/e p580)
The following flowchart is self explanatory for this question:
So, on the injection of a myeloid stem cell, it can give rise to a cell of
its lineage which can be either of the RBCs, monocytes, neutrophils,
eosinophils, basophils and platelets. The answer is therefore, red
blood cells.
7. Ans. (d) T cell Lymphoblastic ALL
(Ref: Robbins 8th/601-3, 9/e p592)
8. Ans. (c) Paroxysmal cold haemoglobinuria (Ref:
Robbins 8th/625, 653,664, 9/e p544, Wintrobes 12th/965)
Analyzing all the options one by one;
Option ‘a’…direct quote.. ‘about 5% to 10% of patients eventually develop
acute myeloid leukemia or a myelodysplastic syndrome, possibly
because hematopoietic stem cell have suffered some type of genetic
damage’. (Robbins 8th/653)
Even PNH is associated with aplastic anemia as both the disorders have
an autoimmune basis.
Option ‘b’…direct quote.., aplastic anemia results from a fundamental stem
cell defect supported by the presence of karyotypic aberrations in
many cases; the occasional transformation of aplasias into myeloid
neoplasms, typically myelodysplasia or acute myeloid leukemia; and
the association with abnormally short telomeres. (Robbins 8th/664)
Option ‘d’…direct quote.. The term “myelodysplastic syndrome” (MDS)
refers to a group of clonal stem cell disorders characterized by
maturation defects that are associated with ineffective hematopoiesis
and a high risk of transformation to AML. (Robbins 8th/625).

So the answer of exclusion is PCH (paroxysmal cold hemoglobinuria). It is an


autoimmune haemolytic anemia due to IgG autoantibodies which bind to P blood
group antigen and cause intravascular hemolysis and hemoglobinuria. Most cases
are seen in children and have recovery within a month.....Wintrobes.

9. Ans. (c) Mixed phenotypic leukaemia


(Ref: Wintrobes 12th/1814-8, Dacie and Lewis hematology 10th/344-6)
Markers on different cells
• B lymphoid markers: CD10, CD19 and CD79
• T lineage markers: CD2, CD3 and CD7
• Myeloid markers; CD13, CD33, CD117 and myeloperoxidase (MPO).
• Non lineage specific markers which are expressed in hematopoietic progenitor
cells: CD34, HLA-DR and TdT

Comparing this with the information provided in the stem of our question, it
is easy to decipher that the cells mentioned are CD10+ve, MPO +ve,
CD19+ve, CD33-ve, CD117 +ve and CD3-ve which is showing both
lymphoid (CD10, CD19;B lymphoid lineage) and myeloid (CD117
and MPO+-) markers in the same cell. So, the answer is Mixed
phenotypic leukemia
10. Ans. (a) Biphenotypic leukemia
(Ref: Robbins illustrated 8th/600)
• Refering to the flowchart in the previous explanation, we
understand that CD 10 +ve and CD 19+ve are the markers
for B-cell lineage, whereas CD 33 and CD 13 are associated
with monocyte and macrophages. So, the patient is having
acute leukemia with immunophenotype pattern with
coexpression of more than one cell lineage. The answer is
therefore Biphenotypic leukemia.
11. Ans. (a) Inv 16
(Ref: Wintrobe 12th/1859, Robbins 8th/624, 9/th 614, T. Singh 2nd/168)
...see text
12. Ans. (d) CD 117
(Ref: Wintrobes 11th/4145)
As per Wintrobe’s the markers for myeloid series are CD13, CD33, CD
11b, CD15, CD117 and cMPO.
c MPO is the most lineage specific marker amongst these.

Regarding other options, • CD 34 - Myeloid and lymphoid blasts, stem


cells, • CD 45 - Leukocyte common antigen (nonerythroid
hematopoietic cells), • CD 99 - Ewing’s sarcoma/primitive
neuroectodermal cells.
13. Ans. (b) B cell ALL
(Ref: Robbins 8th/608, 9/th/597, Harrison 17th/696)
t (2;8) is causing translocation between immunoglobulin κ chain on
chromosome 2 and the myc gene present on chromosome 8 and is
seen in Burkitt’s lymphoma/leukemia. The translocation results in
the increased expression of c-MYC resulting in development of
neoplasia.
14. Ans. (a) Mature B-cell (Ref: Robbins 7th/677, 9th/590)
• Acute Lymphoblastic Leukemias (ALL) of the L3 (FAB) subtype
are tumors of Mature B-cells (e.g. Burkitt’s lymphoma)
15. Ans. (d) M6
(Ref: Robbins 7th/759, Hematology Basic Principles and Practice,
Hoffman, Benz et al.4th/1080)
Nonspecific esterase (NSE) is characteristic of M4 (Acute myelomonocytic) and
M5 (Acute monocytic) leukemia only. NSE positivity is not a characteristic
feature of other subclasses of AML.

• However, NSE positivity may also be seen in 15-20% of cases of


M3 and in some cases of M7.
• NSE positivity is not a feature of M0, M1, M2 and M6 classes
of AML.
16. Ans. (d) Tartarate resistant acid phosphatase positivity is
typically seen in hairy cell leukemia
(Ref: Wintrobe’s 11th/2468, 2470, 2471, Robbin 9/e p603-604)
• Tartarate resistant acid phosphatase (TRAP) is an important
tool in differential diagnosis of hairy cell leukemia (HCL). The
test is positive in 95% of cases of HCL and usually negative or
weakly positive in other disorders.
• TRAP is also positive in some cases of splenic marginal zone
lymphoma.
• LAP (leukocyte alkaline phosphatase) score is decreased in
CML and PNH (paroxysmal nocturnal hemoglobinuria).
However, LAP score often increases when CML transforms to a
blast crisis or accelerated phase.
17. Ans. (c) 5q—
(Ref: Wintrobe’s 12th/1959-64, Ann Hematol. 2008 July; 87(7): 515–
526)
The article in the Annals of hematology ‘Cytogenetic features in
myelodysplastic syndromes’ by Detlef Haase gives the different
causes as percentage of myelopdysplastic syndromes. This is a
typical example of question where the changed data may lead to the
question being repeated in the future exam friends.
‘Myelodysplastic syndromes’ are a group of clonal hematopoietic stem cell
diseases characterized by dysplasia and ineffective hematopoiesis in
one or more of the major myeloid stem lines.
Direct quote from the Haase’s paper…..’Deletions within the long arm of
chromosome 5 are the most frequent cytogenetic changes in
MDSQ accounting for roughly 30% of abnormal cases.’
The cytogenetic abnormalities in adult myelodysplastic syndrome are:
5q 30%
Monosomy 7 20%

Complex chromosome abnormalities are defined by the simultaneous


occurrence of at least three independent abnormalities within one cell clone.
They are also seen in almost 30% cases.

Direct quote from Williams Hematology 8th/edn “The most common


abnormalities are 5q–, -7/7q–, +8, –18/18q–, and 20q–.
Monosomy 7 is the second most frequent cytogenetic
abnormality in the marrow cells of patients with
myelodysplasia.”
So, we would prefer to go with (c) deletion of 5q as the preferred answer.
18. Ans. (b) CD3
(Ref: Harrison 17th/2020, 2032, Robbins 9/e 190)
When a cluster of monoclonal antibodies were found to react with
particular antigen it was defined as a separate marker and given a
CD (cluster of differentiation) number.
• CD3 is used as a pan T-cell marker (present on all stages of T-
cells [Pro–T, Pre-T, Immature and mature T-cells]
• CD19 is a Pan B-cell marker.
CD-1 Thymocytes and Langerhans’
associated
CD - 1, 2, 3, 4, 5, 7, 8 T-cell markers
CD - 10, 19, 20, 21, 22, 23 B-cell markers
CD - 10 CALLA antigen
CD - 13, 14, 15, 33 Monocyte macrophage associated
CD - 16, 56 NK- associated
CD - 41 Platelet marker
CD - 21 EBV receptors
19. Ans. (d) CD135
(Ref: Robbins 9/e 590, 8th/600, 7th/670, www.wikipedia.com)
• B cell associated markers are CD10 (CALLA), CD19, CD20,
CD21 (EBV receptor), CD22 and CD23.
Concept of CD135
• CD135 is a proto-oncogene. It is also the receptor for the cytokine Flt3
ligand (Flt3L) and has the presence of tyrosine kinase activity. Its mutation
can lead to acute myelogenous leukemia (AML) and is associated with a
poor prognosis.

20. Ans. (c) Presence of testicular involvement at presentation


(Ref: Robbins 9/e 592, 8th/603)
Prognostic factors in ALL
GOOD PROGNOSIS BAD PROGNOSIS
• Age 1-10 years • Age <1 year or > 10 years
• Female sex • Male sex
• L1 cell • L2 or L3 cell
• Peripheral blast count <1,00,000 • Peripheral blast count >1,00,000
• Pre B cell phenotype • Pre T cell phenotype
• Absence of mediastinal mass • Mediastinal mass
• Hyperdiploidy (>50 chromosomes) • Pseudodiploidy or t (9;22) or
or t(12;21) presence of Philadelphia
• Trisomy of chromosomes 4,7,10 chromosome, t (8;14), t (4;11)

Friends, please remember age is less than 2 years is given as


bad prognosis in Robbins whereas in NELSON it is mentioned
that the age for bad prognosis is less than 1 year. Nelson also
mentions the testicular involvement to be a bad prognostic
factor. Since in the given question, both (less than 2 years as
well as testicular involvement) are mentioned we would go for
testicular involvement as the better answer here.
21. Ans. (d) t(9:22), t(8:14), t(4:11) explained earlier.
(Ref: Robbins 9/e 592, 8th/603)
22. Ans. (d) M4
(Ref: Robbin 7th/693, Harrison 17th/680, Wintrobe’s 12th/1857)
Signs and symptoms related to infiltration of tissues are usually less
striking in AML than in ALL. Mild lymphadenopathy and
organomegaly can occur. In tumors with monocytic differentiation M4
and M5 (more commonly), infiltration of the skin (leukemia cutis)
and the gingiva leading to gum hypertrophy can be observed, likely
reflecting the normal tendency of non-neoplastic monocytes to
extravasate into tissues.
• M5 AML is the commonest AML associated with extramedullary
diseaseQ (skin lesions/gum infiltration/CNS disease/testicular
involvement).
• After M3 AML, M5 AMLQ is the commonest AML associated with the
development of DIC.

23. Ans. (c) CD117

(Ref: Devita 6th/503, Robbins 7th/826, 9/e 614)


Explained in an earlier question
24. Ans. (a) CD-15
(Ref: Robbins 7th/670, 9/e 590, Harrison 17th/1908, 689, Wintrobe’s
12th/2507)
Direct lines from Wintrobes hematology…..’CD-15 is expressed on
neutrophils, eosinophils and monocytes but not on platelets,
lymphocytes and erythrocytes’.
About option (d), CD 24 is expressed on B cells but it decreased with B cell
activation and differentiation and is lost at the plasma cell stage. It is
also present ion granulocytes and thymocytes but not on mature T
cells. Data shows its increased levels are associated with
colon/breast and pancreatic cancer.
25. Ans. (a) Monocytes
(Ref: Wintrobe’s 12th/15)
• Acid phosphatase is found in all hematopoietic cells, but the
highest levels are found in macrophages and osteoclasts. A dot
like pattern is seen in many T lymphoblasts.
• Tartarate resistant acid phosphatase (TRAP) is seen in
osteoclasts and Hairy cell leukemia. Positive TRAP staining may
be seen in activated T lymphocytes, macrophages, Gaucher
cells, mast cells and some marginal zone lymphomas.
Conditions associated with increased TRAP staining are:
– Hairy cell leukemia
– Gaucher’s diease
– HIV-induced encephalopathy
– Osteoclastoma
– Osteoporosis
– Metabolic bone diseases
26. Ans. (c) Chronic myelogenous leukemia
(Ref: Harrison 17th/683-684; Robbins 7th/697, 698, 9/e 617-618)
The peripheral blood picture of this patient is quite characteristic of
chronic myeloid leukemia.
Chronic myeloid leukemia is a stem cell disease that is characterized by
leukocytosis with granulocytic immaturities, basophilia, splenomegaly and
distinct chromosomal abnormality Philadelphia chromosome.

27. Ans. (b) An immature T cell phenotype [Tdt/CD34/CD7 positive]


(Ref: Robbins 7th/670-673, 9/e 590-593)
• Increased leukocyte count in the range of 138 × 109/L and on
peripheral blood examination 80% of them constituting blast
cells indicate acute leukemia.
• The age group (adolescent) and the mediastinal mass suggests
that this leukemia is likely to be a T-cell leukemia.
• “T-cell ALL tends to present in adolescent males as
lymphomas often with thymic involvement”
So, the diagnosis is T-cell ALL.
Immunophenotypic classification of acute lymphoblastic leukemia
Pre-T-cell ALL TdT, CD2, CD3, CD4, CD5, CD7, CD8, CD34
Early pre-B TdT, DR, CD10, CD19, CD24
Pre-B-cell TdT, DR, CD10, CD19, CD20, CD24, Surface Ig
B-cell DR, CD19, CD20, CD24, Surface Ig

28. Ans. (c) Promyelocytic leukemia


(Ref: Robbins 7th/693, 656-658)
The child presented with acute onset of bleeding, along with the following
laboratory findings:
• Thrombocytopenia (35000/mm3 as compared to normal value of
1, 50,000/mm3)
• Increased prothrombin time (20s as compared to control of 13s)
• Increased partial thromboplastin time (50s vs. normal 26-32s)
• Decreased fibrinogen (10 mg/dL vs. normal of 233-496 mg/dl)
These hematological abnormalities indicate disseminated intravascular
coagulation.
Most common form of AML associated with DIC is M3-AML (Acute
promyelocytic leukemia).
29. Ans. (b) t(9;22) t(4;11)
(Ref: Robbins 9/e p592-593)
Prognostic factors in ALL
GOOD PROGNOSIS BAD PROGNOSIS
• Age 2-10 years • Age <1 year or > 10 years
• Female sex • Male sex
• L1 cell • L2 or L3 cell
• Peripheral blast count <1,00,000 • Peripheral blast count >1,00,000
• Pre B cell phenotype • Pre T cell phenotype
• Absence of mediastinal mass • Mediastinal mass
• Hyperdiploidy (>50 chromosomes) • Pseudodiploidy or t (9;22) or
or t(12;21) presence of Philadelphia
• Trisomy of chromosomes 4,7,10 chromosome, t (8;14), t (4;11)

30. Ans. (c) More than 20% of blasts in blood or bone marrow.
(Ref: Wintrobe’s 12th/1999-2000)
WHO criteria for chronic myelomonocytic leukemia (CMML)
1. Absolute monocytosis > 1 × 109/L in the peripheral blood
2. Blasts + monocytes < 20% in blood and bone marrow
3. Absence of philadelphia chromosome or BCR/ABL fusion gene
4. Dysplasia in one or more of myeloid lineages
• If bone marrow blasts + monocytes > 20%, it is diagnosed as acute
myeloid leukemia
• Unlike classic CML, chronic myelomonocytic leukemia has absence
of basophilia and eosinophilia and more monocytes. Also CML does
not have granulocyte dysplasia (present in CMML).

31. Ans. (b) t ( 9;22)


(Ref: Robbins 9/e 614, 8th/627-628, T. Singh 1st/186-190, Harrison
17th/683 – 84)
• An old man having fatigue and weight loss (due to anemia and
cancer) and heaviness in left hypochondrium (most likely due to
splenomegaly). He also has elevated TLC (most likely due to
leukemia). But the no. of blast cells is 3%, so it cannot be acute
leukemia. Old man with leukemia and splenomegaly is
suggestive of CML which is associated with t (9; 22).
Other options
• t (15, 17) is associated with acute myeloid leukemia. For the diagnosis of
acute leukemia, the number of blasts in the blood should be >20%.

32. Ans. (c) Myeloperoxidase


(Ref: Robbin’s illustrated 7th/692-693, 9/e p613)
Patient here gives a short history (acute onset) of development of
pancytopenia (fever; weakness and gum bleeding suggest
leucopenia; anemia and thrombocytopenia respectively).
The presence of 26% blasts in the bone narrow suggests the development
of acute leukemia and the presence of Auer rods means that the
diagnosis is most likely AML.
Pseudo Pelger Huet cells are neutrophils having greater than 2 nuclear
lobes and are usually seen in myelodysplastic syndrome. In 10%
patients, MDS can give rise to AML. So, the AML in question may
have developed from MDS.
The chief cell in AML is myeloblast for which the staining is positive for
myeloperoxidase.

Note: Acid phosphatase is useful for lymphoblasts which are seen in ALL

33. Ans. (d) Monosomy 7


(Ref: T. singh 1st/175, Robbins 9/e p612) ...see text
34. Ans. (d) G6PD deficiency
(Ref: Harrison 17th/663)
35. Ans. (b) Polycythemia vera
(Ref: Wintrobe’s Clinical Haematology 12th/15)
High levels of LAP score are found in:
• Infection
• Inflammatory disorder
• Growth factor therapy
• Pregnancy
• OCP
• Stress
• Myeloproliferative disorders (except CML)
• Drugs (e.g. Lithium, Corticosteroid, Estrogen)
*Abnormally high values of LAP is seen in myeloproliferative disorder e.g.
polycythemia vera and myelofibrosis
*Decreased LAP score is seen in CML (chronic phase) and PNH (paroxysmal
nocturnal hemoglobinuria).

36. Ans. (a) Hodgkin’s disease; (b) Filariasis; (d) HIV


(Ref: Harrison 17th/383)
Conditions producing allergic reactions and resulting eosinophilia
are:
Drugs: Iodides, Aspirin, Sulfonamides, Nitrofurantoin, Penicillins,
Cephalosporins.
Disease conditions: Hay fever, Asthma, Eczema, Serum sickness,
Allergic vasculitis, Pemphigus, All types of parasitic infections.
Collagen vascular diseases: RA, Eosinophilic fasciitis, Allergic angiitis,
Polyarteritis nodosa.
Malignancy: Hodgkin’s disease, Mycosis fungoides, CML, Carcinoma of
stomach, ovary, lung, Pancreas and uterus.
Other diseases: Job’s syndrome, Sarcoidosis, Skin disease.
Viral infection like HIV and human T-cell Iymphotropic virus (HTLV-l).

In MI polymorphonuclear leukocytosis seen.

37. Ans. (b) Myelofibrosis; (c) Alcoholism; (d) Iron Overload;


(Ref. de Gruchy’s 5th/56)
Sideroblasts are erythroblasts with Prussian blue positive iron granules in
their cytoplasm. They can be found in circulation in the following
diseases:
*Drugs and chemicals: *Hematological disorders: *Inflammatory
Antitubercular drugs (INH, Myelofibrosis disorders:
cycloserine) Polycythemia vera Rheumatoid arthritis
Lead Myeloma SLE
Ethanol Acute leukemia Carcinoma
Hodgkin’s disease Myxedema
Hemolytic anemia Malabsorption
Iron overload.

38. Ans. (b) Kala-azar; (c) TB; (d) Brucellosis


(Ref: Harrison 17th/342, Harsh Mohan 6th/350)
CAUSES OF MONOCYTOSIS:
• Bacterial infections: TB, sub acute bacterial endocarditis,
syphilis, brucellosis.
• Viral infections
• Protozoa and Rickettsial infections: Malaria, typhus,
trypanosomiasis, kala-azar, RMSF.
• Hematopoietic disorder: Monocytic leukemia, lymphoma,
myeloproliferative disorder, multiple myeloma, lipid storage
disorder.
• Malignancies: Ca ovary, stomach and breast.
• Granulomatous diseases e.g. sarcoidosis, IBD.
• Collagen vascular diseases.
39. Ans. (a) AML; (b) Myelodysplastic syndrome; (d) Paroxysmal
nocturnal hemoglobinuria
(Ref: DeGruchy’s 5th/127, Harrison 17th/663)
• Aplastic anemia is a pancytopenia with bone marrow
hypocellularity. It can progress to –
– Paroxysmal Nocturnal hemoglobinuria
– Myelodysplastic anemia
– Rarely acute leukemia

Pure red cell aplasia is a selective disease of absence of erythrocyte progenitor


cells. In contrast to aplastic anemia and MDS, the unaffected lineage (WBC and
platelets) appear quantitatively and qualitatively normal.

• Myelofibrosis is a clonal disorder of a multipotent hematopoietic


progenitor cell of unknown origin and is characterized by
– Marrow fibrosis
– Myeloid metaplasia with extramedullary erythropoiesis
– Splenomegaly
40. Ans. (c) Reticulocytopenia; (d) Thrombocytopenia; (e)
Neutropenia
(Ref: Robbins 7th/647, 9/e p653)
Aplastic anemia is a disorder of marrow failure which stems from
suppression or disappearance of multipotent myeloid stem cells. It is
characterized by: Anemia, Neutropenia , Thrombocytopenia, and
Reticulocytopenia. Splenomegaly is characteristically absent; if
present, the diagnosis of aplastic anemia is almost ruled out. Bone
marrow shows hypocellular marrow largely devoid of hematopoietic
cells, often only fat cells, fibrous stroma, and scattered or clustered
foci of lymphocytes and plasma cells.
Nucleated RBCs in peripheral smear (leukoerythroblastic picture) are
found in cases of marrow fibrosis.
41. Ans. (c) Typhoid (Ref: Harsh Mohan 6th/348)
42. Ans. (b) CALLA Ag (Ref: Robbins 7th/330)

• CALLA (common acute lymphoblastic leukemia antigen) is CD 10


• CALLA positive acute leukemias have best prognosis.
• CD45 is known as common leukocyte antigen.

43. Ans. (b) Chronic lymphocytic leukemia


(Ref: Robbins 7th/673, 9/e 593)
In CLL, the peripheral blood contains increased numbers of small, round
lymphocytes with scant cytoplasm. These cells are fragile and are
frequently disrupted in the process of making smears, producing so-
called smudge cells.
44. Ans. (a) t (15: 17) (Ref: Robbins 9/e 612, 8th/624; 7th/692)
45. Ans. (b) Auer bodies are seen (Ref: Robbins 9/e 613)
46. Ans. (d) Increased
(Ref: Robbins 7th/697-698, Wintrobes 12th/205, Williams CMDT
2010/458)
Wintrobe’s hematology mentiones….. ‘granulocytes contain and release
B12 binding proteins. Markedly elevated transcobalmin I level are
seen in chronic myelocytic leukemia and myeloid metaplasia
whereas low levels are seen in chronic leucopenia and aplastic
anemia’.
Williams hematology 8th/adds that ‘the increase is proportional to the total
leukocyte count in untreated patients and falls with treatment’.
Conditions having elevated levels of cobalamin
Hematological conditions Non-hematological conditions
• Chronic myelogenous leukemia • Acute hepatitis
• Promyelocytic leukemia • Cirrhosis
• Polycythemia vera • Hepatocellular carcinoma
• Hypereosinophilic syndrome • Metastatic liver disease

47. Ans. (d) CML (Ref: Robbins 8th/627; 7th/697-698)


48. Ans. (b) Leucocyte alkaline phosphatase
(Ref: Robbins 8th/595, 9/e 584)
49. Ans. (d) Glucocorticoids (Ref: Harsh Mohan 6th/348)
50. Ans. (c) CML (Ref: Robbins 8th/627; 7th/438, 9/e 584)
51. Ans. (d) Aplastic anemia (Ref: Robbins 9/e p654)
52. Ans. (c) Hypocellular bone marrow
(Ref: Robbins 9/e 615, 8th/625; 7th/695)
53. Ans. (c) Thalassemia (Ref: Robbins 9/e p582, 620, 653)
54. Ans. (c) Myeloblast (Ref: Robbins 8th/602, 9/e p613)
55. Ans. (c) CLL (Ref: Robbins 8th/605, 9/e p594)
56. Ans. (a) Acute lymphoblastic leukemia
(Ref: Robbins 8th/603, 9/e p593)
The presence of the Philadelphia chromosome, a translocation from the
long arm of chromosome 22 to chromosome 9 [t(9;22)], is associated
with a more favorable prognosis in patients with chronic
myelogenous leukemia but it associated with an unfavorable
outcome in Acute lymphoblastic leukemia (ALL).
57. Ans. (d) 65 years (Ref: Robbins 8th/604, 9/e p593)
Different leukemias tend to affect populations of different ages. The
disease described is chronic lymphocytic leukemia (CLL), which is a
disease of older adults.
The 1 year-old (choice A) would be most likely to have acute lymphocytic
leukemia (ALL).
The 20 year-old (choice B) would be most likely to have acute myelocytic
leukemia (AML).
The 45 year-old (choice C) would be likely to have either AML or chronic
myelogenous leukemia (CML).

58. Ans. (c) Hyperdiploidy

(Ref: Robbins 8/e p602-3, 9/e p592-593) ...see text


59. Ans. (b) Immature B cells (Ref: Robbins 8/e p600)
As per the table given in Robbins, B cell ALL is due to immature B cells.
60. Ans. (a) ALL (Ref: Robbins 8/e p602, 7/e p670)
CD 10 is called as common acute lymphoblastic leukemia antigen or
CALLA. It is seen in acute lymphoid leukemia (ALL).
61. Ans. (d) Dilated endoplasmic reticulum … explained earlier
62. Ans. (b) t (15,17) (Ref: Robbins 9/e p612, 8/e p624)
Commonly asked information in NEET EXAM!
• t(9;22) (Philadelphia chromosome): CML (bcr-abl hybrid)

• t( 8;14) Burkitt’s lymphoma (c-myc activation)


• t( ll ; l4): Mantle cell lymphoma (cyclin D1 activation)
• t( l4; 18 ): Follicular lymphomas (bcl-2 activation)
• t( l5;17): M3 type of AML (responsive to all-trans retinoic acid)

63. Ans. (a) Hodkin’s disease


(Ref: Robbins 8/e p618-9) ...see text
64. Ans. (c) Myeloperoxidase (Ref:
Robbins 8/e p602 Pathology and Genetics of Tumours of
Haematopoietic and Lymphoid Tissues p79)
This is what we have knowledge of:
• Lymphoblast: PAS positive
• Myeloblast: Sudan black, myeloperoxidase and non-specific
esterase
• Please know that:

WHO manual pg 69 writes that.. “MPO is specific for myeloid differentiation.


Sudan Black B reactivity is similar to MPO in myeloblasts and monoblasts. The
specificity of Sudan Black is less than MPO.”

65. Ans. (b) May-Heggline anomaly


(Ref: A Color Atlas and Instruction Manual of Peripheral Blood Cell
Morphology pg 221)
Döhle bodies are basophilic leukocyte inclusions located in the peripheral
cytoplasm of neutrophils. They are said to be remnants of the rough
endoplasmic reticulum.
Conditions associated with Dohle bodies
• Burns
• Infections
• Physical trauma
• Neoplastic diseases
• Wissler’s disease
• May-Hegglin anomaly (seen in neutrophil, monocyte,
lymphocyte)
• Chédiak-Steinbrinck-Higashi’s syndrome
66. Ans (b) Myeloblast (Ref: Robbins 9th/612-3)
67. Ans (d) More than 5000 cells/microliter
(Ref: Washington Manual 2013, table 11-4)
68. Ans: (a) Pre B cell (Ref: Robbins 9th/590)
Most common subtype of A.L.L is L1 according to older classification and
Pre B cell variety by the latest WHO classification. The recent WHO
International panel on ALL recommends that the FAB classification
be abandoned, since the morphological classification has no clinical
or prognostic relevance. It instead advocates the use of the
immunophenotypic classification.
69. Ans. (d) Response to steroids (Ref: Robbins 9th/590-3)
Response to steroids is the most consistent marker in the patients of ALL.
For other factors, see text.
70. Ans. (c) Burkitt’s lymphoma (Ref: Robbins 9th/590)
As per the old classification, L3 ALL had cells resembling Burkitt’s
lymphoma like cells morphologically as well in nature.
71. Ans. (a) Preceding MDS (Ref: Robbins 9th/612)
72. Ans (b) M3 (Ref: Robbins 9th/612)
73. Ans. (a) Hodgkin’s Lymphoma, nodular sclerosis

(Ref: Loachim’s lymph node pathology/186)


Direct quote. ‘Progressive transformation of germinal centers (PTGC)
is a benign reaction pattern in lymph nodes. It is most often
associated with reactive follicular hyperplasia. It is also associated
with nodular sclerosis and lymphocyte predominant Hodgkin’s
lymphoma’.
74. Ans. (a) Kimura’s disease
(Ref: Loachim’s lymph
node pathology/190)
Kimura Disease is a chronic inflammatory disorder prevalent in Asians. It
involves subcutaneous tissues and lymph nodes predominantly in the
head and neck region and is characterized by angiolymphoid
proliferation and eosinophilia.
Histopathology
• Lymphoid infiltrates with formation of follicles and germinal centers
accompanied by plasma cells, mast cells and particularly large amount of
eosinophils are present in subcutis.
• Lymph nodes are enlarged and show markedly hyperplastic follicles with
reactive germinal centers and a well-defined peripheral mantle.
• Diffuse eosinophilia, eosinophilic abcesses and infiltration of germinal
centres, sometimes resulting in folliculolysis, are part of the process.
• Polykaryocytes of the Warthin Finkeldey type, characterized by the
overlapping, grape-like arrangement of nuclei, are common, often within
the germinal centers.

75. Ans. (d) Aplastic anemia


(Ref: Harrison
17th/374, Robbins 9/e p653)
Massive splenomegaly is labeled when spleen extends greater than 8 cm
below left costal margin and/or weighs more than 1000 g.
Diseases Associated with Massive Splenomegaly
Chronic myelogenous leukemia Gaucher’s disease
Lymphomas Chronic lymphocytic leukemia
Hairy cell leukemia Sarcoidosis
Myelofibrosis with myeloid metaplasia Autoimmune hemolytic anemia
Polycythemia vera Diffuse splenic hemangiomatosis

76. Ans. (a) t (8:14) (Ref: Robbins 8th/608, 9/e p597)


77. Ans. (a) CD 45 RO
(Ref: Flow Cytometry and Immunohistochemistry for Hematologic
Neoplasms, Lippincott Williams and Wilkins, Tsieh Sun 1st/133;
Neoplastic Hematology Daniel Knowles 2nd/1342
Direct quote from Tsieh Sun .....“myeloid sarcoma usually expresses
CD45 but rare cases may demonstrate T cell markers, such as
CD45RO, CD3 and CD7”. So, the answer of choice is CD45RO.
Salient features of Lab diagnosis of Granulocytic sarcoma
1. Screening panel for CD45, CD19 and CD20.
2. Standard flowcytometry panel includes CD13, 14, 15 , 33 and
myeloperoxidase
3. Immunohistochemistry panel may include chloroacetate esterase (Leder
stain), lysozyme, CD15, CD43 and CD 68
4. LysozymeQ and CD43Q are the most sensitive markers
5. Two new markers CD99Q and CD117Q can be added in equivocal cases.
6. Common cytogenetic abnormalities include t(8;21), inv (16) and t(9;11) Q

Additionally, the percentage of these molecules can be derived from the


data given in Neoplastic Hematology….
Immunohistochemistry Molecule %
CD 45 90%
CD 43 50%
Lysozyme 75%
Leder stain (chloroacetate esterase) >75%

No mention of staining with CD45RO is there, so, we prefer CD45 RO as


the answer of choice.
78. Ans. (a) CD 23
(Ref: Robbins 7th/683; http://www.emedicine.com/med/topic1358.htm,
Robbins 9/e p603) ...see text
Mantle cells lymphomas are usually CD23 negative. They are positive for
CD5, CD20 and CD43.
79. Ans. (b) REAL Classification
(Ref: Robbins 7th/688; AJC cancer staging handbook)
In 1994, a group of hematopathologists, oncologists and molecular
biologists came together (International Lymphoma Study Group) and
introduced a new classification, called the ‘Revised European-
American Classification of Lymphoid Neoplasms (REAL).
WHO has reviewed and updated the REAL classification in 1999 resulting
in inclusion of additional rare entities.
Non-Hodgkin’s lymphoma (NHL) has been classification by many groups.
Major classification systems are:
• Rappaport classification (Developed in 1966)
• Working formation classification
• REAL classification (Developed in 1994)
• WHO classification (modified REAL classification, 1999)
1. Rappaport classification: It is based on microscopic
appearance of tumor cells.
– Size: Lymphocytic or histiocytic
– Growth pattern: Diffuse or nodular
2. Working classification: According to this system, NHL is
divided according to prognostic criteria into:
Low grade Intermediate grade High grade
• Small lymphocytic • Follicular large cell NHL • Small non-cleaved
NHL • Diffuse small cleaved NHL NHL (Burkitt’s)
• Follicular small • Diffuse small cleaved NHL • Immunoblastic
cleaved NHL • Diffuse mixed small cleaved lymphoma
• Follicular mixed small and large-cell NHL • Lymphoblastic
cleaved and large-cell • Diffuse large-cell NHL lymphoma
NHL

3. Revised European-American classification of lymphoma


(REAL) It includes:
– Precursor B-cell neoplasm
– Peripheral B-cell neoplasm
– Precursor T-cell neoplasm
– Peripheral T-cell neoplasm
– Hodgkin’s lymphoma
4. WHO classification: It takes into account morphologic, clinical,
immunological and genetic information. For its details, see text.
80. Ans. (d) A diffuse proliferation of medium to large lymphoid cells
with high mitotic rate
(Ref: Robbins: 7th/673-674, 9th/p 593-594, several journals through
internet)
• This is a case of chronic lymphocytic leukemia (CLL) as
indicated by the characteristic clinical picture and
immunophenotypic characteristics. (Typically, CLL cells
express CD5, CD19, CD23 and show absence of CD79B,
CD22 and FMC7)
• Histopathological examination in a case of typical CLL shows
diffuse effacement of lymphocyte architecture by small to
medium sized lymphocytes with clumped chromatin, indistinct or
absent nucleoli and scanty cytoplasm.
• The round lymphocytes may give way focally to paler areas
consisting of larger round cells (prolymphocytes). These paler
areas are often referred to as proliferation centers and when
present are pathognomic for CLL/SLL. They contain relatively
large number of mitotically active cells.
• Follicular lymphoma is positive for CD10, CD79b and FMC7 but
negative for CD25 and CD43.
• CLL and prolymphocytic leukemia are CD23 positive.
Mantle cell lymphoma is also CD5 positive but here the cells are CD23
negative and CD 79b and FMC7 positive.

81. Ans. (d) Inv (16) is often detected in the blasts and the
eosinophils
(Ref: Annals of Hematology: 2000 May: 79(5): 272-4)
This is a case of ALL with hypereosinophllic syndrome. Inv (16) is
associated with AML and not ALL, and therefore represents the
incorrect statement amongst the option. About other options, the
relevant points:
Eosinophils are not a part of this neoplasm differentiating it from eosinophilic
leukemia.
t(5;14) may be observed in about half of such patients. The symptoms may
resolve after dug therapy.

For details, see the journal with the relevant article.


82. Ans. (b) Results from an expansion of neoplastic T-lymphocytes
(Ref: Harrison’s 17th/697; Robbins 7th/683, 9/e p588)
Hairy cell leukemia is a type of B-cell leukemia. For details, see text.
83. Ans. (b) CD 34 negative but surface Ig+
(Ref: Robbins 9/e p597-598, 8th/608, 7th/677-678, Harrison 17th/, 696)
• Burkitt’s lymphoma is a cancer characterized by the presence
of hallmark translocation t (8;14)Q.
• The translocation results in the increased expression of c-
MYCQ resulting in development of neoplasia.
• Immunophenotyping reveals the tumor cells expressing bcl-6
protein, surface Ig, CD19, CD20 and CD10 (CALLAQ.
84. Ans. (d) CD 34 is associated with Diffuse large B Cell Lymphoma
(Ref: Robbins 9/e p603 8th/605-608, 612-613, 7th/675-678, Wintrobe’s
12th/2223)
Burkitt’s Follicular Mantle cell lymphoma
lymphoma lymphoma
Hallmark t (8;14) t (14;18) t (11; 14)
translocation
Over expression of bcl-6 bcl-2 bcl-1
gene
Immunophenotyping sIgM+, CD5–, sIg, CD5–, CD10+, sIgM+, sIgD+, CD5+,
CD10+, CD19+, CD19+, bright CD10–, CD19+,
CD20+, CD23–, CD20+, CD23+/–, CD20+, CD23–, Cyclin
CD45+ CD38+, CD45+ D1+, FMC-7+

Though Williams hematology mentiones (Table 92.1) and even we


normally read that the Mantle cell lymphoma is CD10- but WHO
manual writes that ‘mantle cell lymphoma may show expression
of CD10 molecule rarely’. This is also supported by Wintrobe’s
12th/pg 2223 where the table clearly mentions that Mantle cell
lymphoma may be CD10+/-. So, option C may be assumed to be true
(after all AIIMS questions can be nerve wrecking friends. This
question has been altered to suit the easy goals in other MCQ
books).
The answer of exclusion is therefore ‘D’ as CD 34 is the marker for
hematopoetic stem cell. Diffuse large B cell lymphoma has the
phenotype of sIgM+, sIgD+/–, CD5–/+, CD10–/+, CD19+, CD20+,
CD45+, PAX5+. The tumor cells can be BCL-6 positive in 40%
cases when associated with t (3;14) or bcl-2 positive in 20% with
t(14;18).
85. Ans. (b) B-cell
(Ref: Harrison 17th/845, 847, Robbins 9/e p1313)
• ‘Post-transplant Lymphoproliferative Disorders’ (PTLDs) are
lymphomas developing after solid organ transplantation e.g.
kidney, liver, heart or lung transplants.
• PTLDs are almost always related to infection by the Epstein-Barr
virus (EBV) which causes a cancerous transformation of B-cells.
In normal individuals immune cells can tackle the EBV infection,
but in organ transplants, the high doses of drugs used suppress
the immune system and the the chances of developing
lymphomas increase.

Difference between Post transplant lymphomas and non Hodgkin’s


lymphomas. PTLDS have the following:
• Extranodal involvement (brain, lungs and the intestines)
• Poorer prognosis

86. Ans. (d) In general follicular (nodular) NHL has worse prognosis
compared to diffuse NHL

(Ref: Robbins 7th/674- 6, 667-8)


• Hodgkin’s lymphoma is clinically and histologically distinct from
the non-Hodgkin’s lymphoma. While non-Hodgkin lymphomas
frequently occur at extranodal sites and spreads in an
unpredictable fashion, Hodgkin’s lymphoma arises in a
single node or chain of nodes and spreads first to
anatomically contiguous nodes. Several ways of classifying
Hodgkin’s lymphoma exist Rappaport, REAL classification
and now WHO (modified REAL classification).
The prognosis of non Hodgkin’s lymphoma varies markedly with various
histological types of non Hodgkin’s lymphoma, “In general, lymphomas
with a follicular histological pattern are of lower grade (longer
survival) than those of diffuse pattern”.

87. Ans. (b) CD 23


(Ref: Harrison 17th/695, Robbins 7th/683, 9/e p602-603)
Mantle cell lymphomas are positive for CD43, CD20, BCl-1 Protein (Cyclin
D1) and CD5.
88. Ans. (b) Follicular lymphoma (Ref: Robbins 9/e p594)
89. Ans. (a) Burkitt’s lymphoma (Ref: Robbins 9/e p597)
90. Ans. (b) Mycosis fungoides
(Ref: Robbins 9/e p605, 8th/1184-1185; 7th/685)
91. Ans. (c) Prognosis is better than in diffuse type
(Ref: Robbins 9/e p594-595, 8th/619)
92. Ans. (a) B-cell lymphoma
(Ref: Robbins 9/e p603, 8th/613, 7th/826)
93. Ans. (b) Stomach (Ref: Robbin 9th/ 773)
Although extranodal lymphomas can arise in virtually any tissue, they do
so most commonly in the GI tract, particularly the stomach.
94. Ans. (b) B cell (Ref: Robbin 9/e p603)
95. Ans. (b) Burkitt Lymphoma (Ref: Robbin 9/e p597)
“Burkitt lymphoma is believed to be the fastest growing human
tumor”…direct line
96. Ans. (a) Follicular small cleaved lymphoma
(Ref: Robbins 8/e p605, 7/e p675)
This was based on the working classification of NHL:
Low grade Intermediate grade High grade
• Small lymphocytic • Follicular large cell NHL • Small non-cleaved NHL
NHL • Diffuse small cleaved NHL (Burkitt’s)
• Follicular small • Diffuse small cleaved NHL • Immunoblastic
cleaved NHL • Diffuse mixed small lymphoma
• Follicular mixed cleaved and large-cell NHL • Lymphoblastic
small cleaved and • Diffuse large-cell NHL lymphoma
large-cell NHL

97. Ans. (c) Diffuse large B cell lymphoma


(Ref: Robbin 8/e p606)
“Diffuse large B-cell lymphoma (DLBCL) is the most common form of
NHL”… (Ref: Robbind 8/e p606)
98. Ans. (a) B cell
(Ref: Robbin 8/e p1348, Eyelid, Conjunctival, and Orbital Tumors 2/e
p746 )
Direct quote... “Non Hodgkin Lymphoma of the B cell lineage is the
most common type in the orbit”....... Eyelid, Conjunctival, and
Orbital Tumors
Also know: for a future question

• The most frequently encountered primary neoplasms of the orbit are vascular
in origin like the capillary hemangioma, the lymphangioma and the
encapsulated cavernous hemangioma.
• MC intraocular tumour in children: Retinoblastoma
• MC intraocular tumour in adults: choroidal malignant melanoma

99. Ans. (a) B cell lymphoma


(Ref: Robbins 9/e p603, 8/e p613)
Marginal zone lymphoma is a B cellQ tumour has the presence of
translocation t(11;18) Q. It is associated with H. pyloriQ infection.
They have the following exceptional characteristics:
• Often arise within tissues involved by chronic inflammatory
disorders of autoimmune or infectious etiology
• Remain localized for prolonged periods, spreading systemically
only late in their course
• May regress if the inciting agent (e.g., Helicobacter pylori) is
eradicated
100. Ans. (d) CD 5 +, CD 23 –
(Ref: Robbins 9/e p602-603, 8/e p612-3) ...see text
101. Ans. (d) Cutaneous lymphoma
(Ref: Robbins 9/e p605, 8/e p1184-1185, 7/e p1685)

• Mycosis fungoides is a T cell lymphoma affecting skin which can evolve into
generalized lymphoma.
• Histological hallmark: Sezary Lutzner cellsQ which are helper T cells forming
band like aggregates in superficial dermis and have cerebriform contourQ.
• May invade epidermis as single cells and small clusters called as Pautrier
microabscessesQ.

102. Ans. (a) Anaplastic large cell lymphoma (ALK positive)


(Ref: Robbin 8/e p605)
• Anaplastic Large-Cell Lymphoma (ALK Positive) is an uncommon entity which is
defined by the presence of rearrangements in the ALK gene on chromosome 2p23.
• This tumor is typically composed of large anaplastic cells, some containing
horseshoe-shaped nuclei and voluminous cytoplasm (so-called hallmark cells).

103. Ans. (b) Africa…….Indian exam asking global question


104. Ans (c) Sezary cells (Ref: Robbins 9/e 606)
105. Ans (a) Mycosis fungoides
(Ref: Robbins 9th/ 1159)
Mycosis fungoides is a lymphoma of skin-homing CD4+ T helper
cells that presents in the skin.
106.Ans (d) Cerebriform
(Ref: Robbins 9/e 606)
107. Ans. (a) CD 15 and CD 30
(Ref: Robbins 9/e p608, 7th/422-423)
In classical HL, CD15 and CD30 are the surface markers. In non
classical HL, CD20 and BCL/6 are the markers.
108. Ans. (c) Absolute lymphocyte count < 600/µl (Ref:
Hodgkin’s lymphoma: A Comprehensive Update on Diagnostics
and Clinics By Andreas Engert/2010/104)
Seven adverse prognostic factors described for advanced Hodgkin’s
disease are
1. Male gender
2. Age > 45 years
3. Stage IV disease
4. Hemoglobin < 10.5 g/dl
5. Leukocytosis with WBC > 15,000/µl
6. A serum albumin level <4 g/dl
7. Lymphocytopenia with either Absolute lymphocyte count < 600/µl or
lymphocyte being < 8% of WBCs

109. Ans. (d) Lymphocyte predominant Hodgkin’s disease


(Ref: Robbins 7th/686, 9/e p609) ...see text

110. Ans. (d) Langerhans’ cell.


(Ref: Robbins 9/e p 608-609, 7th/686, 688)
Langerhans’ cells are epidermal dendritic cells that take up and process
antigenic signals and communicate the information to lymphoid cells.
111. Ans. (d) Lymphocyte predominant
(Ref: Robbins 7th/688, 9/e p609)
112. Ans. (a) Myelodysplastic syndrome
(Ref: Robbins 9/e p614-615, 8th/625-626; 7th/695-696)
113. Ans. (a) Lymphocyte predominant
(Ref: Robbins 9/e p609, 7th/668, 686, 689) ...see text
114. Ans. (a) Lymphocyte dominant
(Ref: Robbins 9/e p609, 8th/619; 7th/689)
115. Ans. (b) Nodular sclerosis Hodgkin lymphoma
(Ref: Robbin 8/e p618-9)
Presence of binucleated acidophlic owl eye appearance with CD15 and
CD 30 is suggestive of Reed Sternberg cell.
• Lymphocyte depletion HL occurs predominantly in the elderly and in
HIV+ individuals

• In lymphocyte predominant HL, the Reed Sternberg cells are positive


for CD20 and BCL6, and are usually negative for CD15 and CD30.

• Mixed-cellularity HL is more common in males, in elderly with presence


of constitutional symptoms. Involved lymph nodes are diffusely effaced
by a heterogeneous cellular infiltrate, which includes T cells, eosinophils,
plasma cells, and benign macrophages admixed with Reed-Sternberg cells

116. Ans. (c) Lymphocytic predominance


(Ref: Robbins 9/e p608-609, 8/e p618-9) ...see text
117. Ans (c) Nodular sclerosis...See earlier explanation (Ref:
Robbins 9/e p608)
118. Ans. (c) Essential thrombocythemia
(Ref: Robbins 8th/629, 9/e p620 Harrison 17th/374)
This. is a modified version of DPG2011 question. Please see the
table for causes of Massive Splenomegaly given earlier.
In CML … ‘first symptom of CML is a dragging sensation in the abdomen
caused by splenomegaly’ Robbins 8th/627-8
Polycythemia vera Robbins 8th/629, 9/e p619.. spent phase of
polycythemia has extensive extramedullary hematopoeisis principally
in the spleen which enlarges greatly ‘’
Primary myelofibrosis page 631, 9/e p621… ‘it comes to attention because
of progressive anemia and splenomegaly’
119. Ans. (b) JAK 2 mutation
(Ref: Robbins 9/e p618, 8th/626-8, Wintrobe’s hematology 12th/1991-
2; Journal Blood 110:1092, 2007)
Polycythemia vera is the most common of the chronic myeloproliferative
disorders. As discussed is text: JAK 2 mutation is a major criteria.
However this mutation is not diagnostic of PV as it is also seen in
essential thrombocytosis, chronic idiopathic myelofibrosis (CIMF) and
atypical myeloproliferative disorders.
Other options, options “a” and “c” are minor criteria whereas Option
‘d’, remember friends that
• MPL point mutation is seen in 5-10% patients with essential
thrombocytosis and primary myelofibrosis.
120. Ans. (d) Essential thrombocytopenia
(Ref: Robbins 7th/696, 9/e p616)
Essential thrombocytosis (and not Essential thrombocytopenia) is a
myeloproliferative disorder.
121. Ans. (c) Chronic myeloid leukemia
(Ref: Robbins 8/e p628 , 9/e p618)
Leukocyte alkaline phosphatase (LAP) is found within the white blood
cells. Revise the following condition s asked in the exam:
High LAP score Low LAP score

• Leukemoid reaction • Chronic myelogenous leukemia


• Polycythemia vera (PV) (CML)
• Paroxysmal nocturnal
• Essential thrombocytosis (ET)
hemoglobinuria (PNH)
• Primary myelofibrosis (PM)
• Acute myelogenous leukaemia
(AML).
• Sideroblastic anemia

122. Ans. (c) Hemolytic anemia


(Ref: Robbins 8/e p595, Blood 2/e p255, T. Singh 1/e p198)
Leukoerythroblastosis is a term used for “an anemia characterized by
the presence in the peripheral blood of immature red cells and a
few immature white cells of the myeloid series” that is
erythroblasts and leukoblasts. The following are the causes of
leukoerythroblastosis:
Causes of leukoerythroblastosis
Marrow invasion Tumors (lymphoma, Hodgkin disease,
leukemia, multiple myeloma, bony
metastasis)
Infections (sepsis, TB, osteomyelitis)
Miscellaneous (osteopetrosis,
histiocytosis, storage disease, vasculitis
including rheumatoid arthritis)

Myeloproliferative disorders Polycythemia vera


Myelofibrosis
CML
Erythroleukemia
Thrombocythemia
Down syndrome

Hematological disease Erythroblastosis fetalis


Pernicious anemia
Thalassemia major
Severe hemolytic anemia

Hypoxia Congestive heart failure


Cyanotic congenital heart disease
Respiratory disease
Please remember friends that severe hemolytic anemias may be
associated with a similar picture but routinely, leukoerythroblastosis is
not observed with hemolytic anemia.

123.Ans. (b) Leukemoid reaction.


(Ref: Robbins 8/e p595, 9/e p584, Hematology 3/e p402)

LAP is found in the membranes of secondary granules of neutrophils.


Its activity is measured in mature neutrophils and band cells
onlyQ.
Eosinophils do notQ show alkaline phosphatase activity and must not
be mistaken with mature neutrophils with a score of zero. Malaria is
characterized by monocytosis.

124. Ans. (c) 65%

(Ref: Textbook of Clinical Pediatrics 2/e p356)


Direct lines…. “Polycythemia is defined as venous hematocrit exceeding
65%”.
Just be cautious not to confuse polycythemia with the polycythemia vera.
125. Ans. (a) CD 1a (Ref: Robbins 9/e p622, 8/e p631)
• In Langerhans cell histiocytosis, the presence of Birbeck granules in the
cytoplasm is characteristic.
• In this condition, Birbeck granulesQ are pentalaminar tubules, often with
a dilated terminal end producing a tennis racket–like appearanceQ, which
contain the protein langerin.
• In addition, the tumor cells also typically express HLA-DR, S-100, and
CD1aQ.

126. Ans. (d) Tennis racket (Ref: Robbins 8/e p631, 9/e p622)
127. Ans. (c) Acute myeloblastic leukemia
(Ref: Robbins 9th/ 616)
Myeloproliferative disorders are charcterised by an increased
production of one or more types of blood cells. The common
pathogenic feature is the presence of mutated, constitutively
activated tyrosine kinases or other acquired mutations resulting
in growth factor independence. The examples include:

• Chronic myelogenous leukemia


• Polycythemia vera
• Primary myelofibrosis
• Essential thrombocythemia
• Chronic eosinophilic leukemia
• Systemic Mastocytosis
• Stem cell leukemia

128. Ans. (b) 5q (Ref: Robbins 9/e p615)


129. Ans. (b) Hyperviscosity
(Ref: Robbins 8th/610-1, 9/e p600, Harrison 18th/938-940, Wintrobes
hematology 12th/2374-8)
According to Harrison,
• Bone pain is seen in 70% patients
• Infections is the next common (>75% have serious infection at some time in their
course)
• Normocytic normochromic anemia is seen in 80% patients

130. Ans. (c) Hypercalcemia


(Ref: Harrison 17th/702, 9/e p600)
131. Ans. (d) IgM (Ref: Robbins 8th/65, 9/e p601)
Lymphoplasmacytoid lymphoma (or Waldenstrom’s macroglobulinemia)
is a B cell neoplasm presenting in 6th or 7th decade of life having
features similar to CLL/SLL and multiple myeloma (MM). Like MM,
there is presence of a ‘M’ or monoclonal spike (caused due to IgM
whereas in MM, it is caused by IgG).
Also Remember
• Deletion involving chromosome 6q is the commonest abnormality in
Waldenstrom’s macroglobulinemia.

132. Ans. (b) A diagnosis of plasma cell leukemia……


(Ref: Wintrobe’s haematology 11th/2620, 2593)
Plasma cell leukemia by definition is characterized by more than 20%
plasma cells in the peripheral blood.
The patient in question has 14% plasma blasts in the peripheral blood and
thus does not fit into category of plasma cell leukemia.
133. Ans. (c) Plasmacytoma on biopsy
(Ref: T. Singh 1st/210-211, Robbins 9/e p600-601)
The commonly used diagnostic criteria of multiple myeloma are:
MULTIPLE MYELOMA
Major criteria
1. Plasmacytoma on tissue biopsy
2. Bone marrow plasmacytosis with > 30% plasma cells
3. Monoclonal globulin spike on serum electrophoresis (> 3.5g/dL for IgG, > 2
g/dL for IgA) or on urine (> 1g/24 h of Bence-Jones protein)
Minor criteria
1. Bone marrow plasmacytosis 10 to 30% plasma cells
2. Monoclonal globulin spike less than the level defined above
3. Lytic bone lesions
4. Reduced normal immunoglobulin (<50% of normal); IgM <0.05 g/dL, IgA <
0.1g/dL, IgG < 0.6 g/dL

The diagnosis of multiple myeloma requires a minimum of two major


criteria or one major criteria + one minor criteria, or three minor
criteria.
134. Ans. (a) CD 1a (Ref: Robbins 9/e p622, 7th/701, 702)
135. Ans. (b) Sheets of atypical plasma cells
(Ref: Robbins 9/e p599, 7th/679)
• Old patient along with lytic circumscribed punched out X-ray
appearance suggests multiple myeloma
• Multiple myeloma most often presents as multifocal destructive
bone tumors composed of plasma cells throughout the skeletal
system.
136. Ans. (a) Cryoglobulinemia; (b) Multiple myeloma; (d)
Lymphoma; (e) Macroglobulinemia
(Ref: William’s Haemotology 6/1268)
Hyperviscosity is seen in

• Multiple myeloma
• Waldenstrom’s macroglobulinemia
• Cryoglobulinemia
• Myeloproliferative disorders

MGUS (Monoclonal Gammopathy of uncertain significance): Here M


Protein can be identified in the Serum of 1% of healthy individual >50
years. age and 3% in older than 70 years of it is the most common
form of monoclonal gammopathy. In MGUS less than 3g/dL of
monoclonal protein is present in serum and there is no Bence Jones
proteinuria.
Hyperviscosity is defined on the basis of the relative viscosity of serum as
compared with water. Normal relative viscosity of serum is 1.8

137. Ans. (b) Waldenstrom’s macroglobulinemia; (c) Multiple


myeloma.
(Ref: William’s Hematology 6th-1268)
138. Ans. (b) CD l+ ; (c) Birbeck’s granules are pathognomonic; (d)
Proliferation of antigen presenting cells; (e) Resembles
Dendritic cells; (Ref: Robbins 9/e p622)
139. Ans. (c) Wire loop lesions
(Ref: Harrison 17th/573, Robbins 7th/232 9/e p223)
Wire loop lesions are characteristic of SLE and are not seen in multiple
myeloma.
140. Ans. (b) IL-6 (Ref: Robbins 7th/679, 9/e p599)
141. Ans. (d) Systemic lupus erythematosus
(Ref: Robbins 7th/678-679)
Plasma cell dyscrasias are characterized by proliferation of B-cell clone
which synthesizes and secretes a single homogenous
immunoglobulin or its fragments. This entity includes Multiple
myeloma, Waldenstrom’s macroglobulinemia, Heavy chain
diseases, Primary or immunocyte associated amyloidosis and
Monoclonal Gammopathy of Undetermined Significance
(MGUS).
142. Ans. (d) No lymphadenopathy (Ref: Robbins 9/e p622)
143. Ans. (c) lgG (Ref: Robbins 9/e p600)
144. Ans. (a) Langerhans cell (Ref: Robbins 9/e p622)
145. Ans. (b) Positive ANA (Robbins 9th/598-601)
Increased serum levels if immunoglobulins, plasmacytosis and M
spike on electrophoresis are all seen in multiple myeloma. For
details, see text of chapter 8.
146. Ans. (b) Langherhans (Ref: Robbins 9/e p622, 8/e p631-2)
LCH has the following presentations:
• Multifocal multisystem Langerhans cell histiocytosis
(Letterer-Siwe disease)
– Occursmost frequently before 2 years of age
– Dominant clinical feature is the development of cutaneous
lesions resembling a seborrheic eruption over the front
and back of the trunk and on the scalp.
– Presence of concurrent hepatosplenomegaly,
lymphadenopathy, pulmonary lesions, and (eventually)
destructive osteolytic bone lesions.
– Unifocal and multifocal unisystem Langerhans cell
histiocytosis (eosinophilic granuloma)
– Characterized by proliferations of Langerhans cells
admixed with variable numbers of eosinophils,
lymphocytes, plasma cells, and neutrophils.
– Typically arises within the medullary cavities of bones,
most commonly the calvarium, ribs, and femur. Less
commonly, unisystem lesions of identical histology arise in
the skin, lungs, or stomach.
– Unifocal lesions most commonly affect the skeletal system
in older children or adults. Unifocal disease is indolent and
may heal spontaneously or be cured by local excision or
irradiation.
– Multifocal unisystem disease usually affects young
children, who present with multiple erosive bony masses
that sometimes expand into adjacent soft tissue.
Hand-Schuller-Christian triad: calvarial bone defects + diabetes ins
exophthalmos
Pulmonary Langerhans cell histiocytosis
– Seen in adult smokers
– Regress spontaneously upon cessation of smoking.
147. Ans. (a) Multiple myeloma
(Ref: Harsh Mohan 6th/383, Robbins 9th/)
Increased levels of microglobulin are seen in the urine and serum of
patients with multiple myeloma.
Harrison 18th/941…. Serum β 2 -microglobulin is the single most powerful
predictor of survival and can substitute for staging.
148. Ans. (a) Multiple Myeloma (Ref: Robbins 9/e p599)
In multiple myeloma, cytologic variants stem from the dysregulated
synthesis and secretion of immunoglobulins, which often leads to
intracellular accumulation of intact or partially degraded protein. Such
variants include:
• Flame cells: with fiery red cytoplasm,
• Mott cells: with multiple grapelike cytoplasmic droplets
The globular inclusions are referred to as Russell bodies (if cytoplasmic)
or Dutcher bodies (if nuclear).
149. Ans. (a) Birbecks granules (Ref: Robbins 9th/622)
Langerhans cell histiocytosis is characterized by the presence of “Birbeck
granules” in the cytoplasm
150. Ans. (c) 1500 (Ref: Harrison 18th/135-e-7)
Idiopathic hypereosinophilia syndrome is characterized by eosinophilia
(>1500 cells/mm3) persisting for at least 6 months for which no
underlying cause is found.

ANNEXURE
Distinguishing Features In Various Types of Plasma Cell Disorders:
1. Plasma cell leukemia
• More than 20% plasma cells in the peripheral blood
• Absolute plasma cell count of more than 2 X 109/L
2. IgD Myeloma
• Presence of Monoclonal IgD in the serum usually indicates IgD myeloma
• No evident M– spike on serum protein electrophoresis
• Higher incidence of renal insufficiency, amyloidosis and proteinuria than
IgG/IgA myeloma.
• Higher incidence of extramedullary involvement and inferior survival rates.
3. Monoclonal gammopathy of undetermined significance (MGUS)
• Serum monoclonal protein <3g/dl
• Clonal bone marrow plasma cells < 10%
• Absence of end organ damage such as hypercalcemia, renal failure, anemia
and bone lesions (CRAB)
• No lytic bone lesions
• No evidence of other B-cell proliferative lesion
4. Smoldering multiple myeloma (Asymptomatic multiple myeloma)
Both criteria must be met
• Serum monoclonal protein ( > 3 g/dl) and/or 10-60% clonal bone marrow
plasma cells or both
• Absence of myeloma defining events or amyloidosis
5. Multiple myeloma
Diagnostic criteria for plasma cell myeloma
Clonal bone marrow plasma cell percentage ≥10% or biopsy-proven plasmacytoma and
≥1 of the following myeloma-defining events:
a. Evidence of End-organ damage (CRAB)
– HyperCalcaemia: serum calcium > 11 mg/dL
– Renal insufficiency: Creatinine clearance < 40 ml/minute or senum creatinine >
2 mg/dL
– Anaemia: (Hb < 10g/dl or >2g/dl lower than normal)
– Bone lesions: ≥1 osteolytic lesion on skeletal radiography, CT, or PET/CT
b. >1 of the following biomarkers of malignancy:
Clonal bone marrow plasma cell percentage > 60%
– An involved-to-uninvolved serum free light chain ratio >100
– >1 focal lesion on MRI

Characteristic Immunophenotypes of Major Subtypes of Lymphoma

Lymphoma Immunophenotype
Follicular CD20+, CD3–, CD10+, CD5–
Small lymphocytic CD20+, CD3–, CD10–, CD5+, CD23+
Marginal zone/MALT CD20+, CD3–, CD10–, CD5–, CD23–
Mantle cell CD20+, CD3–, CD10–, CD5+, CD23–,
CD43+, PRADI+
Diffuse large B-cell CD20+, CD3–, CD5–, CD45+
Burkitt CD20+, CD3–, CD10+, CD5–; Tdt–
Lymphoblastic CD20–, CD3+, Tdt+
Anaplastic large cell CD20–, CD3+, CD30+, CD15–, EMA+, ALK+
Peripheral T-cell CD20–, CD3+
Hodgkin CD30+, CD15+

1. All trans-retinoic acid is used in the treatment of tumor
associated with: (NEET 2020 like pattern)
(a) BCR-ABL
(b) PML - RARA
(c) cMYC
(d) CEBPA
Ans. (b) PML-RARA (Ref: Robbins 9th/317
Direct lines from Robbins…
• When given in pharmacologic doses, all-trans retinoic acid binds to PML-RARα
and causes a conformational change causing the neoplastic myeloid progenitors to
differentiate and allowing for recovery of normal hematopoiesis.
• This highly effective therapy is the first example of differentiation therapy, in
which immortal tumor cells are induced to differentiate into their mature progeny,
which have limited life spans.

2. Match the following: (AIIMS May 2019 like pattern)


(a) Dohle bodies 1. Dilated endoplasmic reticulum
(b) Flame cells 2. Azurophilic granules
(c) Howell Jolly bodies 3. Immunoglobulin
(d) Auer rods 4. Nuclear remnant

Ans. is a–1, b–3, c–4, d–2


• Dohle bodies are patches of dilated ER… appear as sky-blue cytoplasmic “puddles.”
• Plasma cells with Ig inclusions FIERY RED cytoplasm (flame cells) and Mott
cells with multiple grapelike cytoplasmic droplets, and cells containing a variety
of other inclusions,
• Howell Jolly body: basophilic nuclear remnant………postsplenectomy, megaloblastic
anemia, MDS
• Auer rods: azurophilic granule……………AML…AML-M3
3. Person had severe pruritus while taking hot shower and presents
with deep vein thrombosis. He also had JAK 2 mutation and
hypertension. Which of the following is the most likely
diagnosis of this patient? (AIIMS May
2019 like pattern)
(a) Myelofibrosis
(b) Polycythemia vera
(c) CML
(d) AML
Ans. (b) Polycythemia vera (Ref: Robbins 9th/618-9)
• Polycythemia vera is strongly associated with activating point mutations in the
tyrosine kinase JAK2.
• Serum erythropoietin levels in these patients are low, whereas secondary forms of
polycythemia have high erythropoietin levels.
• The elevated hematocrit and thrombocytosis make the patients prone to both
thrombosis and bleeding.
• Intense pruritus and peptic ulceration may occur due to the release of histamine
from basophils.
• JAK mutation could also be seen in patient with Primary
myelofibrosis but the clinical presentation is that of an elderly patient
with progressive anemia and splenomegaly. He may also have
nonspecific symptoms such as fatigue, weight loss, and night sweats
as well as hyperuricemia.
• CML and AML do not have JAK 2 mutation.
4. Which of the following is positive in Follicular
lymphoma? (NEET 2019 like pattern)
(a) Bcl 2
(b) Bcl 6
(c) Bcl 1
(d) Bcl 10
Ans. (a) Bcl 2 (Ref: Robbins 9th e/p 594)
• Follicular lymphoma likely arises from germinal center B cells and is strongly
associated with chromosomal translocations involving BCL2.
• Its hallmark is a (14;18) translocation.
• The t(14;18) is seen in up to 90% of follicular lymphomas, and leads to
overexpression of BCL2. BCL2 antagonizes apoptosis and promotes the survival
of folicular lymphoma cells.

Looking at other options:


• Bcl 2: follicular lymphoma
• Bcl 6: diffuse large cell lymphoma
• Bcl 1: mantle cell lymphoma
• Bcl 10: extranodal marginal zone lymphoma

5. How will you differentiate a mediastinal mass being a thymoma or


ALL? (AIIMS Nov 2018 like pattern)
(a) Cytokeratin
(b) CD1a
(c) CD3
(d) Tdt
Ans. (a) Cytokeratin
(Ref: Differential Diagnosis in Surgical Pathology3rd e/p 270)
Direct lines…. most important stain to differentiate a mediastinal mass
being a thymoma or acute lymphoblastic lymphoma is cytokeratin;
shows scattered keratin positive thymic epithelial cells admixed with
the immature lymphoid cell population in lymphocyte rich lymphoma.
Rest all 3 options can be common to both these conditions.
Significance of markers in thymoma
• CD1a: Highlights immature T lymphocytes
• CD3: Highlights T cell population
• Cytokeratin: Highlights epithelial cells
• P63 and PAX8: nuclear positivity in thymic epithelial cells
Also know for future AIIMS questions
• Foxn1 is a sensitive and specific marker for thymoma and thymic
carcinoma, and it appears to be superior to CD5 and CD117 for the
diagnosis of thymic carcinoma.
• CD205 is a sensitive and specific marker for thymoma but its sensitivity to
thymic carcinoma is lower than CD5 and CD117.

6. A patient presented with intermittent fever, no weight loss, no


anorexia, but with a retroperitoneal mass. Peripheral smear
findings were normal. Gross and microscopy of the mass is
given. What’s the diagnosis?
(AIIMS Nov 2018 like pattern)

(a) NHL
(b) Castleman disease
(c) Angiolymphoid hyperplasia
(d) Ig G4 disease
Ans. (b) Castleman disease
(Ref: Harrison 20th e/p 771, 1451)
• Castleman’s disease is a lymphoproliferative disorder but is
not a true malignancy. It may be unicentric or multicentric.
• There are two variants of the unicentric disease:
a. Hyaline vascular variant: It is more common, is usually
asymptomatic and mostly an incidental mediastinal mass is the
presentation. In classical HV variant, lymph node is preserved
though distorted. Histologically, lymphoid follicles may contain
1. Interfollicular vascular hypeplasia
2. More than 2 germinal centres (“twinning”)
3. Penetration of sclerotic blood vessel in the germinal
centres (lollipop lesions)
Lollipop lesions
4. Thickened mantle zones with rings of small lymphocytes
(“onion skin” pattern)…given image in the exam
b. Plasma cell variant: It is less common and presents as
fever, night sweats, weight loss and malaise.
Histologically, there are sheets of plasma cells in
interfollicular areas and hyperplastic germinal centres in
the follicles.
• Multicentric Castleman disease is a symptomatic disorder
presenting as lymphadenopathy, B symptoms (fever, weight
loss and night sweats), hepatosplenomegaly.
Micrsocopically, it has mixed features and the boundary
between mantle zone and interfollicular regions is blurred.
7. Which among the following laboratory investigation is best to
reveal bleeding in Disseminated Intravascular
Coagulation? (AIIMS May 2018 like pattern)
(a) Increased PT
(b) Increased aPTT
(c) Decreased fibrinogen
(d) Increased FDPs
Ans. (d) Increased FDP
(Ref: Robbins 9th e/ p 665, Harrison 20th e/p 835)
The diagnosis of DIC is based on clinical observation and laboratory
studies, including measurement of fibrinogen levels, platelets, the PT
and PTT, and fibrin degradation products.
• The most sensitive test for DIC is the FDP level as DIC is an unlikely diagnosis in
the presence of normal levels of FDP.
• The D-dimer test is more specific for detection of fibrin—but not fibrinogen—
degradation products and indicates that the crosslinked fibrin has been digested
by plasmin.
• Because fibrinogen has a prolonged half-life, plasma levels diminish acutely only
in severe cases of DIC. High-grade DIC is also associated with levels of
antithrombin III or plasminogen activity <60% of normal.
8. Which of the following is not a provisional entity as per WHO 2016
classification of acute leukemia?
(AIIMS May 2018 like pattern)
(a) AML with hyperploidy
(b) B-ALL with BCR-ABL like mutation
(c) AML with BCR-ABL
(d) Early T-cell precursor leukemia/Lymphoma
Ans. (a) AML with hyperploidv Ref : WHO 2016 classification
I have no idea about the expectation of the examiner but all I can say
friends is that have a look at the WHO 2016 classification and
then it is easy to answer that the AML with hyperploidy is not a
provisional entry.
List of provisional entries in WHO 2016 classification of acute leukemia
In Myeloid/lymphoid neoplasms with eosinophilia and rearrangement of PDGFRA,
PDGFRB, or FGFR1, or with PCM1-JAK2
• Provisional entity: Myeloid/lymphoid neoplasms with PCM1-JAK2
In MDS
• Provisional entity: Refractory cytopenia of childhood
In AML and related neoplasms,
• Provisional entity: AML with BCR-ABL1
• Provisional entity: AML with mutated RUNX1
In B cell lymphoblastic leukemia
• Provisional entity: B-lymphoblastic leukemia/lymphoma, BCR-ABL1–like
• Provisional entity: B-lymphoblastic leukemia/lymphoma with iAMP21
In T cell lymphoblastic leukemia
• Provisional entity: Early T-cell precursor lymphoblastic leukemia
• Provisional entity: Natural killer (NK) cell lymphoblastic leukemia/lymphoma

9. Lymphohistiocytic variant of Reed Sternberg cell is seen in


which of the following? (AI 2018 Pattern)
(a) Lymphocyte rich HL
(b) Lymphocyte predominant HL
(c) Nodular sclerosis HL
(d) Mixed cellularity HL
Ans. (b) Lymphocyte predominant HL
(Ref: Robbins 9/e p608)
10. Biopsy of the bone marrow of a patient is shown below. What is
the likely hematological disease associated with it?
(AI 2018 Pattern)
(a) Myelofibrosis
(b) Metastatic cancer
(c) Multiple myeloma
(d) Myelodysplastic syndrome
Ans. (c) Multiple myeloma (Ref: Robbins 9/e p600)
The histology shows a plasma cell with a cart wheel appearance and so,
is suggestive of multiple myeloma.
11. Tumor cells in chronic lymphocytic leukemia or small
lymphoblastic lymphoma (CLL/SLL)arisefrom which of the
following?
(AIIMS Nov 2017 Pattern)
(a) Mature B cell
(b) Naive B cell
(c) Centrocytes of germinal center
(d) Progenitor B-cell
Ans. (b) Naive B cell (Ref: Robbins 9/e p593)
Direct lines…. “DNA sequencing has revealed that theIg genes of some
CLL/SLL are somatically hypermutated whereas others are not,
suggesting that the cell of originmay be either a postgerminal
center memory B cell or a naive B cell.”
12. A patient presented with headache and fever. His investigations
revealed hemoglobin of 16g/dl, TLC of 21,000/ uL, platelet count
of 3,75,000. His DLC showed Neutrophils (25%), Lymphocytes
(20%), Metamyelocytes and myelocytes 40%and eosinophils5%.
Which of the following is the next best investigation in this
patient? (AIIMS Nov 2017 Pattern)
(a) JAK 2 mutation
(b) EPO level
(c) Philadelphia chromosome
(d) Bone marrow biopsy
Ans. (c) Philadelphia chromosome
Presence of the clinical picture given is suggestive of a left shift (TLC
with myeloid bulge) which could be due to a myeloproliferative
disorder like CML or leukemoid reaction. So, to confirm the
diagnosis, a Ph chromosome study should be done.
13. A new marker for mantle cell lymphoma especially useful in
Cyclin D1 negative cases is:
(AIIMS May 2017 Pattern)
(a) SOX11
(b) MYD88
(c) Annexin V
(d) ITRA 1
Ans. (a) SOX11
(Ref: WHO update on Lymphoid Malignancies)
SOX11 expression is detected in most cyclin D1 negative mantle cell
lymphoma.

SOX11 expression is detected in most cyclin D1 negative mantle cell


lymphoma.

14. On histopathological examination of lymph node as shown


below, what zone is represented by the marked area?
(AIIMS May 2017 Pattern)

(a) Germinal center


(b) Mantle zone
(c) Marginal zone
(d) Paracortical area
Ans. (b) Mantle zone
In lymph nodes the B cells are concentrated in discrete structures, called
follicles, located around the periphery, or cortex, of each node. If the
B cells in a follicle have recently responded to an antigen, this follicle
may contain a central region called a germinal center. The T
lymphocytes are concentrated in the paracortex, adjacent to the
follicles.
15. All the following markers are expressed on the surface of T-cells
at some stage of development except:
(AIIMS May 2017 Pattern)
(a) CD1a
(b) CD34
(c) PAXS
(d) Tdt
Ans. c. PAX5 (Ref: Robbins 9/e p590)
This is a modified version of previous question.
This question should be attempted by ruling out the options.
CD1a: though a specific marker of Langerhans cell but CD1 to CD8 are
present on T cell surface.
• CD34: hematopoetic stem cell marker, so would be expressed in
early stage of T cell development.
• Tdt: known T cell marker useful for diagnosis of T cell ALL.

PAX5 (and genes like E2A, and EBF) are required for B-cell
development…Robbins 9th/590

16. A patient presented with complaints of fever and right sided neck
swelling. A biopsy from the swelling revealed normal lympho-
mononuclear cells with interspersed Reed Sternberg cells. On
immune-histochemistry, these cells were found to be CD2O
positive, while they were negative for CD3O, CDI5 and EBV
latent membrane protein. What is the diagnosis?
(AIIMS May 2017 Pattern)
(a)Lymphocyte rich Hodgkin’s lymphoma
(b) Diffuse large B-cell lymphoma
(c) Nodular lymphocyte predominant Hodgkin’s lymphoma
(d) Small cell lymphoma
Ans. (c) Nodular lymphocyte predominant Hodgkin’s lymphoma
(Ref: Robbins 9/e p609)
• Nodular lymphocyte predominant Hodgkin’s lymphoma is
characterized by the presence of L-H subtype of Reed
Sternberg cells.
• In contrast to the Reed-Sternberg cells found in classical forms
of HL, L&H variants express B-cell markers typical of germinal-
center B cells, such as CD20 and BCL6, and are usually
negative for CD15 and CD30. EBV is not associated with this
subtype.
17. A 70-year-old male presented with severe intractable diarrhea.
His bone marrow and renal biopsy was done which is as shown
below. What is the most appropriate diagnosis?
(AIIMS May 2017 Pattern)

(a) Multiple myeloma


(b) Urate nephropathy
(c) Amyloidosis
(d) Hodgkin lymphoma
Ans. (a) Multiple myeloma (Ref: Robbins 9/e p595)
The interpretation of the images is as follows:
• Image A: amyloid deposition in the glomeruli
• Image B: presence of large number of plasma cells in the bone
marrow
• Image C: electron microscopic appearance of a plasma cell
showing inclusions
The combination of the three images suggests the presence of multiple
myeloma in these patients which is characterised by increased
number of plasma cells in the bone marrow, high chances of renal
amyloidosis and presence of intracellular inclusions in the electron
microscopic examination.
Disclaimer
Any resemblance to an actual question is purely coincidental.
• Bleeding time is the test for platelet defect.
• Hess test is done for capillary fragility.
• Prothrombin time is used to assess extrinsic pathway whereas aPTT is used to
assess intrinsic pathway. These should be measured within two hours once blood
is drawn.
• Bernard-soulier syndrome is due to congenital defect in platelet adhesion due to
deficiency/dysfunction of Gp lb-IX. In this disease, platelet aggregation is normal
with collagen/ADP but abnormal with ristocetin.
• Glanzmann thrombasthenia is due to congenital defect in platelet aggregation due
to deficiency/ dysfunction of Gp IIb-IIIa. It presents as bleeding from umbilical stump
in newborn. In this disease, the platelet aggregation is normal with ristocetin but
abnormal with collagen/ADP/ thrombin.
• Small sized platelets in peripheral smear are characteristic of Wiskot-Aldrich
syndrome.
• Aplastic anemia causes amegakaryocytic thrombocytopenia.
• Immune thrombocytopenia is caused by autoantibodies (IgG) against platelet
glycoproteins IIb-IIIa or Ib-IX.
• Triad of hemolytic uremic syndrome (HUS): Microangiopathic hemolytic anemia,
renal failure and thrombocytopenia.
• Pentad of thrombotic thrombocytopenic purpura (TTP): Triad of HUS plus fever
and neurological symptoms.
• Blood banking using certain additives for the blood to be transfused later. Shelf life of
blood with CPD-A is 5 weeks (35 days).
• Platelets are stored at room temperature around 20–24°C. So, they have maximal
chances of transmission of infections.
• Cryoprecipitate is used in deficiency of factors I (Hypofibrinogenemia), VIII
(Hemophilia A), von Willebrand factor (vW disease).
• Disease transmitted by all components of blood is malaria.


PLATELETS

Platelets are enucleated cells in the circulation released from the


megakaryocyte, likely under the influence of flow in the capillary sinuses. The
normal blood platelet count is 1.5- 4.5 lakhs/mm3. The production of the
platelets is regulated by the hormone thrombopoietin produced in the liver.

The platelet synthesis is also specifically increased by interleukin 6. The


average life span of the platelets is 7–10 days.
Bleeding time represents the time taken for a standardized skin puncture
to stop bleeding and it gives an in vivo assessment of platelet response to
limited vascular injury. The value varies from 2 to 9 minutes. It is abnormal
when there is a defect in platelet numbers or function. Currently, quantitative
measures of platelet function are being introduced by using an electronic
particle counter.

Thrombocytopenia is characterized by spontaneous bleeding a prolonged bleeding time and


a normal PT and PTT.

Platelet disorders can be of the following two types:


1. Functional platelet disorders: The platelet count is normal but there is
a qualitatitve defect in the platelets
2. Quantitative platelet disorders: The platelet count is reduced but there is
no qualitative defect in the platelets
Classification of functional platelet disorders
Classification of functional platelet disorders
Disorders of adhesion
Inherited Acquired
Bernard-Soulier syndrome Uremia
von Willebrand disease Acquired vWD
Disorders of aggregation
Inherited Acquired
Glanzmann thrombasthenia FDP inhibition
Afibrinogenemia Dysproteinemias
Drugs-Ticlopidine, GpIIb/IIIa inhibitors
Disorders of granule release
Inherited Acquired
Oculocutaneous albinism Cardiopulmonary bypass
Chediak-Higashi syndrome Myeloproliferative disease
Isolated dense granule deficiency Drugs- NSAIDs
Gray platelet syndrome (combined a and b
granule deficiency)
Contd...
Contd...

Immune Thrombocytopenic Purpura (ITP)

ITP can be either primary (idiopathic) or secondary (SLE, AIDS, viral infections
and drug induced). The primary ITP can be dependent on the duration of the
disease, acute (less than 6 months) or chronic (> 6 months). The platelet
destruction in both of them results from the formation of antiplatelet
autoantibodies (type II hypersensitivity reaction).

Pathogenesis

Chronic ITP is caused by the formation of autoantibodies mostly of the IgG


class against platelet membrane glycoproteins, most often IIb-IIIa or Ib-IX. The
opsonized platelets are rendered susceptible to phagocytosis by the cells of
the mononuclear phagocyte system. The spleen is the major site of removal of
sensitized platelets.
Clinical features

Chronic ITP occurs most commonly in adult women younger than age 40 years. The
female-to-male ratio is 3:1. This disorder is often insidious in onset and is characterized
by bleeding into the skin (pinpoint hemorrhages called petechiae, especially in the
dependent areas where the capillary pressure is higher or ecchymoses), mucosal
surfaces (nose bleed, post brushing gum bleeds and hematuria), menorrhagia
(menstrual bleeding in females) and intracranial bleeds. Splenomegaly and
lymphadenopathy are uncommon in primary ITP, and their presence should make one
consider other possible diagnoses.

Findings: The blood smear shows abnormally large platelets


(megathrombocytes), Bone marrow is hypercellular and shows
megakaryocytic hyperplasia. The bleeding time is prolonged, but PT and PTT
are normal.

Acute Immune Thrombocytopenic Purpura

It is similar to chronic ITP (caused by anti platelet antibodies) but is


differentiated from the same by the following:
• Seen in children
• Less duration (2-6 weeks)
• Occurs with equal frequency in both sexes
• Abrupt onset of thrombocytopenia
• Preceded by a viral illness and is
• Self-limiting (resolving spontaneously within 6 months). Steroids may be required in
few cases only.

The diagnosis of Idiopathic Thrombocytopenic Purpura should be made only after exclusion
of other known causes of thrombocytopenia.

Thrombotic Thrombocytopenic Purpura (TTP)/Moschcowitz Disease

It is a rare disorder of the blood-coagulation system, causing extensive


microscopic blood clots to form in the small blood vessels throughout the body.
It arises from deficiency of the vWF metalloprotease enzyme ADAMTS13
which is responsible for cleaving large multimers of von Willebrand factor
(vWF). In the absence of this enzyme, very high molecular weight multimers of
vWF accumulate in plasma and promote platelet aggregation. This is also
associated with activation of coagulation in the small blood vessels. Platelets
are consumed in the coagulation process, and bind fibrin, the end product of
the coagulation pathway. These platelet-fibrin complexes form microthrombi
which circulate in the vasculature and cause shearing of red blood cells,
resulting in hemolysis. Any additional endothelial cell injury further increases
microaggregate formation. Reduced blood flow and cellular injury result in end
organ damage.

Classically, the following five features (“pentad”) are indicative of TTP:

• Fluctuating neurological symptoms, such as bizarre behavior, altered mental status,


stroke or headaches
• Kidney failure
• Fever
• Thrombocytopenia (low platelet count), leading to bruising or purpura
• Microangiopathic hemolytic anemia (anemia, jaundice and a characteristic blood film
see Figure 11)
Fig. 1: Microangiopathic hemolytic anemia showing (H: Helmet Cells; B: Burr Cells).
If coagulation tests indicate a major consumption of procoagulants, the
diagnosis of TTP is doubtful. The management of these patients is done with
plasmapheresis (plasma exchange) and sometimes additional
immunosuppressive therapy.

Hemolytic Uremic Syndrome (HUS)

HUS is also associated with microangiopathic hemolytic anemia and


thrombocytopenia but is distinguished from TTP by:

• Normal ADAMTS13 levels


• Absence of neurological symptoms
• Dominance of acute renal failure
• Childhood onset of disease
It can be of the following types:
1. Epidemic or typical HUS: It is associated with infectious gastroenteritis
caused by E. coli strain O157: H7. This bacterium releases a Shiga-like
toxin damaging endothelial cells followed by platelet activation and
aggregation. The patients presents with bloody diarrhea followed by HUS
after few days.
2. Non-epidemic or atypical HUS: It is associated with mutations in the
gene encoding complement regulatory proteins like factor H, factor I or
membrane cofactor protein CD46. These proteins normally prevent
excessive activation of alternate pathway of the complement system. So,
their deficiency is associated leads to uncontrolled complement activation
after minor endothelial injury, resulting in thrombosis. The patients have a
relapsing remitting course.
HUS can also be seen due to other factors (e.g., certain drugs, radiation
therapy) that damage endothelial cells.

Von Willebrand Disease (vWD)

The von-Willebrand factor (vWF) is a heterogenous multimeric plasma


glycoprotein produced by endothelial cells (Weibel Palade bodies) and
megakaryocytes (can be shown inside platelet α-granules). It helps in platelet
adhesion and factor VIII carrier in plasma.

The disease can have the following variants:


1. Type 1 and type 3 von Willebrand disease are associated with a reduced
quantity of circulating vWF. Type 1 (commonest variant) is an autosomal
dominant disorder and is mild clinically. Type 3 (an autosomal recessive
disorder) is associated with extremely low levels of functional vWF, and
severe clinical manifestations. Type 1 disease is associated with missense
mutations whereas Type 3 disease is associated with deletions or
frameshift mutations
2. Type 2 von Willebrand disease is characterized by qualitative defects in
vWF. The type 2A variant is the most common subtype of type 2 vWD. It is
inherited as an autosomal dominant disorder and is associated with
missense mutations. It is associated with mild to moderate bleeding.

Clinical features in von Willebrand disease are due to


Platelet Adhesion defects

Deficiency of vWF results in defect in the adhesion of platelets to collagen preventing the
formation of haemostatic plug. It leads to mucus and cutaneous bleeding in the form of
epistaxis, menorrhagia and GI bleeding.

Coagulation defect

There is reduced half life of factor VIII leading to its deficiency resulting in hemorrhages and
intramuscular hematoma.

LABORATORY FINDINGS
1. A prolonged bleeding time in the presence of a normal platelet count.
2. The defective platelet adhesion also results in a positive tourniquet test
(Hess test).
3. In deficiency of vWF, ristocetin induced platelet aggregation does not take
place. So, ristocetin induced aggregation is defective and is diagnostic
of this disease. However, platelet aggregation with ADP, collagen and
thrombin is normal.
4. Though the synthesis of factor VIII remains normal but half life of VIII in
plasma decreases due to reduced vWF (carrier) levels. This leads to
secondary VIIIC deficiency in plasma. So, intrinsic pathway of coagulation
is affected and thus, aPTT is increased in these patients.
Hemophilia A (Factor VIII Deficiency)

Hemophilia A is the most common hereditary disease associated with serious


bleeding. This X linked disorder is caused by a reduction in the amount or
activity of factor VIII which is a cofactor for factor IX in the activation of factor X
in the coagulation cascade. The disease can have the following variants:
1. Mild disease – levels of factor VIII activity between 6% and 50% of normal
2. Moderate disease - levels between 2% and 5% of normal
3. Severe disease- the levels less than 1% of normal activity.

Clinical features: it includes easy bruising and massive hemorrhage after


trauma or operative procedures. The disease is evident early in life when there
is bleeding after circumcision or when the child begins to walk or crawl. The
hemorrhages occur frequently in the joints (hemarthroses) and recurrent
bleeding may lead to progressive deformities. Acute hemarthroses is painful
and to avoid pain, the patient may adopt a fixed position leading to muscle
contractures. It mainly affect knees, elbows, ankles, shoulders, and hips
Petechiae are characteristically absent. Muscle hematoma can also be seen
leading to a compartment syndrome. Fascial hemorrhages can result in the
formation of blood filled cysts with calcification and proliferation of fibroblasts
giving the appearance of a tumor (pseudotumor syndrome). Self limiting
episodes of hematuria in the absence of genitourinary pathology are frequent
in the patients.

LABORATORY FINDINGS

1. Patients with hemophilia A typically have a normal bleeding time,


platelet count, and PT
2. There is a prolongation of aPTT due to an abnormality of the intrinsic
coagulation pathway.
3. Factor VIII-specific assays are required for diagnosis.

Hemophilia B (Christmas disease, Factor IX Deficiency)

The factor IX deficiency produces a disorder similar to hemophilia A. It is


inherited as an X-linked recessive trait and the PTT is prolonged and the PT is
normal, as is the bleeding time. The diagnosis is done only by assay of the
factor IX levels.

Recent Exam Topic

FACTOR INHIBITORS

A relatively rare cause of prolonged aPTT is presence of antibodies against coagulation


plasma proteins called inhibitors. It can be seen due to the following reasons:

• Hemophilia A and B patients receiving clotting factors to control their bleeding episodes
• Pregnancy
• Autoimmune diseases
• Malignancies (lymphoma, prostate cancer)
• Dermatologic conditions

Clinical manifestations include bleeding episodes in soft tissues, skin, GIT and
genitourinary tract.

The diagnosis is made with a prolonged aPTT with normal PT and TT which is not
corrected with mixing the test plasma with normal pooled plasma for 2hrs at 370C.

Treatment is done with high dose i.v. immunoglobulins and anti CD20 monoclonal
antibody.
To differentiate between different causes of isolated prolongation of aPTT,
the following flowchart is useful.

HIGH YIELDS TOPICS FOR EXAMS

1. Blood Grouping

The red blood cells have many antigens expressed on their surface which is
the basis of multiple subtypes of blood groups (ABO, Rh, Kell, Duffy, P
antigen). In transfusion medicine, the most important blood groupings that are
practiced are the ABO and the Rh grouping.
ABO is most important for the following reasons:
When the ABO antigen is not expressed on the red cell, individuals always
have ABO antibodies in their plasma
• The ABO antibodies formed are frequently mixtures of immunoglobulins
(IgM >>>IgG) antibodies, both having thermal reactivity at 37°C and both
capable of activating complement.

Genetics of ABO Antigens

THE GENES DETERMINING THE A AND B PHENOTYPES ARE FOUND ON


CHROMOSOME 9P AND ARE EXPRESSED IN A MENDELIAN
CODOMINANT MANNER.
• H gene is present on chromosome 19 and is inherited in an autosomal
recessive manner.
• The Rh antigen genes are present on chromosome 1. These antigens are
proteins. The antigens which may represent the Rhesus antigen are C, D
(most important) and E. The D antigen is a potent alloantigen which is
present in 85% individuals.
• Formation of Abo Antigens
ABO antigens are glycoproteins. On the red blood cell surface, H substance
is formed by the addition of fucose to the glycolipid or glycoprotein backbone.
The subsequent addition of N-acetyl galactosamine (NAG) creates the A
antigen, whereas the addition of galactose (Gal) produces the B antigen.
People lacking both transferases are phenotypically type ‘O’ whereas those
who inherit both transferases are type ‘AB’.

Bombay Blood Group

Discovered by Dr YM Bhende, it is a very rare blood group. If an individual lacks the


H gene coding for the fucosyl transferase, he cannot form H substance. These
individuals are homozygous for the silent h allele (hh) and have Bombay phenotype
(Oh). These people have anti H, anti A and anti B antibodies in their serum. They
are detected by reverse blood grouping (explained below). The only blood which is safe
for transfusion in these individuals is Bombay blood group.
Detection of blood grouping is done by forward and reverse grouping

Forward grouping is done with anti A or anti B antisera. This detects the
presence of antigens on the RBCs of the individual.
• Reverse grouping is done with serum taken from the person and known
cells of A/B/O subtype.
Forward grouping Reverse grouping
Blood group Anti A Anti B A cell B cell O cell
O - - + + -
A + - - + -
B - + + - -
AB + + - - -
Bombay blood - - + + +

Secretors and Nonsecretors


Some people secrete the A, B and H antigens in the body fluids (such as
plasma, gastric juice, saliva, sweat, tears, semen, milk, etc. with an exception
being the cerebrospinal fluid). This ability is dependent on presence of a
dominant secretor gene (Se). About 80% of individuals are secretors. Both
secretors and non-secretors express ABO antigens on red cells.

Blood group antigens and disease association

1. Gastric cancer with blood group A


2. Peptic ulcer is more often in group O individuals.
3. von Willebrand factor (vWF) antigen level varies among individuals with
different ABO blood groups. Individuals with blood group O have the lowest
vWF antigen level, followed by group A, then group B, and, last, group AB.
4. P. vivax infection is increased by presence of Duffy antigen.
2. Blood Transfusion
A. Donor selection

• The donor should be between 18–65 years of age.


• Weight should >45 kg
• Blood pressure should be controlled
• No skin disease at the phlebotomy site
• Time interval between 2 blood donations: 3 months
• Whole blood donation to be deferred for 3 days after platelet/plasmaphersis
• Donor should have eaten something in the last 3 hours

B. Routine transfusion
• The first choice is the donor blood of the same ABO group as that of the recipient.
• If blood of the same ABO group may occasionally be not available and if blood transfusion
is likely to be potentially lifesaving, blood of an alternate but compatible group may be
transfused as per the table mentioned below.
Donor blood group
Recipient blood group First choice Alternative
A A O
B B O
AB AB A, B, O (in this order)
O O Nil

• To reduce the risk of haemolysis in a case of non-identical but compatible ABO


transfusion, packed red cells instead of whole blood should be transfused (i.e.
most of the plasma which contains anti-A and/or anti-B should be removed).

C. Emergency transfusion

• For patients in hemorrhagic shock, it is necessary to transfuse blood immediately.


The risk of transfusing group O “uncrossmatched” red cells is extremely low and
is certainly much lower than the risk of the patient’s death if blood transfusion is
delayed.
• For AB group recipients, if red cells of group AB are not available, group A donor
blood is preferred over other alternatives since anti-B in group A is weaker than
anti-A in group B.
• In the Rh system, individuals with Rh-negative blood group should be
transfused only with Rh-negative blood, especially Rh-negative females of
childbearing age and young girls (to prevent Rh immunisation and future haemolytic
disease of newborn).
• In an emergency, Rh-positive blood may be transfused to older females and
males of unknown blood group (if Rh-negative blood is not available). Persons
with Rh-positive group should be transfused with Rh-positive blood preferably.
• If the patient’s blood group cannot be determined, then he should be transfused with
O group red cells which are Rh-negative with colloids/crystalloids.

Salient Features about Blood Transfusion

BLOOD SHOULD BE TRANSFUSED THROUGH A STERILE, DISPOSABLE


ADMINISTRATION SET INCORPORATING A STANDARD FILTER (170 ΜM
PORE SIZE Q). THIS FILTER RETAINS SMALL CLOTS OR CELLULAR
AGGREGATES BUT PERMITS PASSAGE OF SINGLE CELLS AND
MICROAGGREGATES.
• The usual needle size is 18-or 19-gaugeQ.
• The solution in the blood bag usually contains citrate phosphate dextrose
adenine (CPDA)-1 (49Q ml for 350 ml of blood and 63 ml for 450 ml of
blood). Function of each of the constituents is mentioned below:

i. Citrate: Anticoagulation by binding of calcium in plasma


ii. Phosphate: Acts as a bufferQ to minimize the effects of decreasing pH in blood
iii. Dextrose: Maintenance of red cell membrane and metabolism
iv. Adenine: Generation of ATP (energy source).

Components of Blood
1. Time frame for transfusion of different components of blood
Blood component Initiation of transfusion Completion of transfusion
Whole blood Within 30 minutesQ Within 4 hoursQ
Cryoprecipitate / FFP As early as possible Within 20 minutesQ

2. Shelf life and storage of different blood components


Storage
Blood component temperature Shelf life Indications
Whole blood 2-6°C 42 days • Acute blood loss
• Exchange transfusion
Packed red cells 2-6°C 42 days • Severe anemia
• Exchange transfusion
Platelets 20-24°C 5 days • Thrombocytopenia
(platelet
agitator)
Cryoprecipitate -24°C 1 year • Hemophilia A
• vW disease
• Factor 13 deficiency
• Fibrinogen deficiency
Fresh frozen plasma -24°C 1 year • DIC
• Hemophilia B
• Liver disease

Complications of Blood Transfusion


Blood products are often responsible for saving lives of individuals but may be
associated with the development of the complications. These include:

• Febrile Non Hemolytic Transfusion Reaction (FNHTR): this is the most


common complication leading to fever and chills, within 6 hours of a
transfusion of red cells or platelets. It is caused by the release of
inflammatory chemicals from the donor leukocytes. It is treated
symptomatically with antipyretics.
• Allergic Reactions: Severe, potentially fatal allergic reactions may occur
when blood products containing certain antigens are given to
previously sensitized recipients. These occur more commonly in
patients with IgA deficiency. Urticarial allergic reactions may occur more
commonly due to an allergen in the donated blood product. The condition
is managed with antihistaminic drugs.
• Hemolytic Reactions: Acute hemolytic reactions are usually caused by
preformed IgM antibodies against donor red cells that fix complement.
They occur due to improper labeling in the blood bank (ABO
incompatibility). Clinical features include fever with chills, flank pain,
intravascular hemolysis, and hemoglobinuria. It is diagnosed with a
positive direct Coombs test. It may be fatal in rare cases.
• Delayed hemolytic reactions: they are caused by antibodies that
recognize red cell antigens that the recipient was sensitized to
previously, for example, through a prior blood transfusion. These are
typically caused by IgG antibodies to foreign protein antigens and are
associated with a positive direct Coombs test.
• Transfusion-Related Acute Lung Injury (TRALI)

• Reason for TRALI is presence of antibodies in the donor’s plasma against


recipient’s HLA II antigens located on the neutrophils which leads to
neutrophilic aggregation in the pulmonary vasculature and endothelial
damage there. This leads to the clinical symptoms in the patient.
• Infectious Complications: They are more common with platelet
preparations. These can be prevented by Donor screening.

Anticoagulants
Agents Trisodium 3.2% Heparin Potassium
EDTA Trisodium Oxalate
Citrate
Mechanism of Remove Remove Activation of Binds calcium
action calcium calcium antithrombin III
Preferred • Blood cell • Platelet • Osmotic fragility • Anticoagulant
Uses counts and studiesQ testQ (not preferred
morphology • Coagulation • WBC Functional or because
Q labile factors
studiesQ immunophenotyping
• ESR Q Q are unstable
• Red cell testingQ in oxalate)
• Arterial blood gas
analysis
Advantages Complete Preserves the Does not affect shape Cheap
anticoagulation labile and size Easily available
with minimal coagulation
morphologic factors
and physical
effects on the
cell
Disadvantages • Not suitable for blood • Distorts cell
counts because it morphology
cannot inhibit platelet • Shrinks red
and leucocyte cell size, so,
clumping not used for
• Bluish discoloration hematocrit
to blood smear slide estimation
on applying Wright
Giemsa stain.

• Anticoagulated blood may be stored at 4°CQ for 24 hour period without


significantly altering cell counts or cellular morphology but hematologic analysis
should be done as soon as possible.
• Any anticoagulantQ can be used for collecting blood for Flow cytometryQ.
PLATLETS AND BLEEDING DISORDERS

1. A newborn baby presented with profuse bleeding from the umbilical


stump after birth. Rest of the examination and PT, APTT are within
normal limits. Most likely diagnosis is which of the following?
(a) Factor X deficiency
(b) Glanzmann’s thrombasthenia
(c) von Willebrand disease
(d) Bernard Soulier disease
2. A 25 years old asymptomatic pregnant female underwent a pre-op
coagulation test. Her bleeding time is 3minutes, PT is 15/14sec, a
PTT is 45/35 sec, platelet count is 2.5 lac/mm3 and factor VIII levels
were 60IU/dL. What is her most likely diagnosis?
(AIIMS Nov 2011)
(a) Factor IX deficiency
(b) Lupus anticoagulant
(c) Factor VIII inhibitors
(d) VWD – Type III
3. True about prothrombin time to:
(AIIMS Nov 2011)
(a) Immediate refrigeration to preserve factor viability
(b) Platelet-rich plasma is essential
(c) Done within 2 hours
(d) Activated with kaolin
4. A 22 year old female having a family history of autoimmune disease
presents with the complaints of recurrent joint pains after
pregnancy. She has now developed petechial hemorrhages. She is
most likely to have which of the following disorders?
(AIIMS Nov 2011)
(a) Megakaryocytic thrombocytopenia
(b) Amegakaryocytic thrombocytopenia
(c) Platelet function defects/Functional platelet defect
(d) Acquired Factor VIII inhibitors
5. Patient with bleeding due to platelet function defects has which of the
following features?
(AI 2011)
(a) Normal platelet count and normal bleeding time
(b) Normal platelet count and increased bleeding time
(c) Decreased platelet count and increased bleeding time
(d) Normal platelet count and decreased bleeding time
6. A 9-year-old boy presents with elevation in both PT and aPTT. What is
the diagnosis?
(AIIMS Nov. 2010)
(a) Defect in extrinsic pathway
(b) Defect in intrinsic pathway
(c) Platelet function defect
(d) Defect in common pathway
7. All are true about thrombotic thrombocytopenic purpura except?
(AIIMS Nov 2008)
(a) Microangiopathic hemolytic anemia
(b) Thrombocytopenia
(c) Normal complement level
(d) Grossly abnormal coagulation tests
8. D.I.C. is seen in:
(AIIMS May 2007)
(a) Acute promyelocytic leukemia
(b) Acute myelomonocytic leukemia
(c) CMC
(d) Autoimmune hemolytic anemia
9. All of the following can cause megakaryocytic thrombocytopenia,
except:
(AIIMS Nov 2004)
(a) Idiopathic thrombocytopenia purpura
(b) Systemic lupus erythematosus
(c) Aplastic anemia
(d) Disseminated intravascular coagulation (DIC)
10. A patient with cirrhosis of liver has the following coagulation
parameters, Platelet count 2,00,000, Prothrombin time 25s/12s,
Activated partial throm-boplastin time 60s/35s, thrombin time
15s/15s. In this patient:
(a) D-dimer will be normal
(AIIMS May 2004)
(b) Fibrinogen will be < 100 mg
(c) ATIII will be high
(d) Protein C will be elevated
11. The presence of small sized platelets on the peripheral smear is
characteristic of:
(AIIMS Nov 2003)
(a) Idiopathic thrombocytopenia purpura (ITP)
(b) Bernard Soulier syndrome
(c) Disseminated intravascular coagulation
(d) Wiskott Aldrich syndrome
12. Platelet aggregation in vivo is mediated by:
(a) Serotonin
(PGI Dec 2003)
(b) Ig mediators.
(c) Interaction among the leukocytes
(d) Interaction among the platelets
(e) Macromolecules.
13. Conditions associated with incoagulable state:
(a) Abruption placentae
(PGI Dec 2004)
(b) Acute promyelocytic leukemia
(c) Severe falciparum malaria
(d) Snake envenomation
(e) Heparin overdose
14. In DIC, which is/are seen:
(PGI June 2005)
(a) Normal aPTT
(b) Increased PT
(c) Increased factor VIII
(d) Decreased fibrinogen
(e) Decreased platelets
15. Causes for DIC are:
(PGI Dec 2005)
(a) Anaerobic sepsis
(b) Malignancy
(c) Lymphoma
(d) Leukemia
(e) Massive blood transfusion
16. Platelet function defect is seen in: (PGI June 03)
(a) Glanzmann syndrome.
(b) Bernard Soulier syndrome
(c) Wiskott Aldrich syndrome
(d) Von-Willebrand disease
(e) Weber Christian disease
17. VWF factor deficiency causes:
(Delhi PG 2008)
(a) ↓ Platelet aggregation
(b) ↓ Factor VIII in plasma
(c) Defective platelet adhesion
(d) All of the above
18. Thrombospondin is:
(Delhi PG 2008)
(a) Coagulation protein
(b) Coagulation promoting protein
(c) Contractile protein
(d) Angiogenesis inhibitory protein
19. Which is must for prothrombins time (PT)?
(a) Thromboplastin
(Delhi PG 2007)
(b) Prothrombin
(c) Fibrin
(d) Fibrinogen
20. Thrombocytosis is seen in:
(Delhi PG 2005)
(a) Myelofibrosis
(b) SLE
(c) Azidothymidine therapy
(d) Myelodysplastic syndrome
21. All of the following are true about Willebrand factor except:
(Delhi PG-2005)
(a) Synthesized by hepatocytes
(b) Its deficiency can cause factor 8 defect also
(c) Its deficiency may cause problem with platelet adhesion
(d) It serves as carrier for the factor eight
22. All of the following clotting factors are completely synthesized from
liver except:
(UP 2002)
(a) II
(b) V
(c) VII
(d) VIII

MOST RECENT QUESTIONS

23. Cryoprecipitate contain all except:


(a) Fibrinogen
(b) Factor VIII
(c) von Willebrand factor
(d) Antithrombin
24. Bleeding time is abnormal in:
(a) Hemophilia
(b) Christmas disease
(c) von Willebrand disease
(d) Vitamin K-deficiency
25. The chromosomal translocation involving bcl-2 in B-cell lymphoma
is:
(a) t (8: 14)
(b) t (8: 12)
(c) t (14: 18)
(d) t (14: 22)
26. Agranulocytosis means:
(a) Decrease in neutrophil count
(b) Decrease in platelet count
(c) Increase in RBC count
(d) Decrease in RBC count
27. Thrombocytopenia syndrome is caused by decrease in platelet
counts below:
(Bihar 2004)
(a) 50,000/cmm
(b) 1,00,000/cmm
(c) 1.2 lac/cmm
(d) 20, 000/cmm
28. Glycoprotein IIb-IIa complex is deficient in
(a)Bernard Soulier syndrome
(b) Glanzmann disease
(c) Von willebrand disease
(d) Gray platelet syndrome
29. All the following statements are correct about treatment in chronic
immune thrombocytopenic purpura except:
(a)Most of the patients respond to immunosuppressive doses of
glucocorticoids
(b) Relapse is rare
(c) Splenectomy is the treatment of choice for relapse
(d) Minority have refractory forms of ITP and difficult to treat
30. Splenectomy is useful in which of the following?
(a) Chronic ITP
(b) Sickle cell anemia
(c) Tuberculosis
(d) Good pasture syndrome
31. Glanzmann disease is characterised by which of the following?
(a) Congenital defect of RBCs
(b) Defect of neutrophils
(c) Congenital defect of platelets
(d) Clotting factor deficiency
32. All of the following are true about DIC except:
(a)Platelet aggregation
(b) Fibrin deposition in microcirculation
(c) Decreased fibrin degradation products
(d) Release of tissue factor
33. All are true regarding thrombotic thrombocytopenic purpura except:
(a)Normal ADAMTS levels
(b) Microangiopathic hemolytic anemia
(c) Thrombocytopenia
(d) Thrombosis
34. Pancreatic insufficiency and cyclic neutropenia is a part of which
syndrome:
(a) Young syndrome
(b) Colts syndrome
(c) Shwachman syndrome
(d) Roots syndrome
35. Best blood product to be given in a patient of multiple clotting factor
deficiency with active bleeding:
(a) Fresh frozen plasma
(b) Whole blood
(c) Packed RBCs
(d) Cryoprecipitate
36. Which of the following is false about Transfusion-Related Acute Lung
Injury?
(a) Develops within 24 hours
(b) Mostly seen after sepsis and cardiac surgeries
(c) It’s a cause of non cardiogenic pulmonary edema
(d) Plasma is more likely to cause it than whole blood
37. Cyclin D1/Ig H gene is associated with?
(a) Diffuse large B-cell lymphoma
(b) Hairy cell leukemia
(c) Follicular lymphoma
(d) Mantle cell lymphoma
38. Ristocetin test in von Willebrand disease shows?
(a) Increased agglutination
(b) Decreased agglutination
(c) Normal agglutination
(d) No agglutination
39. Glanzmann’s thrombasthenia is characterized by defective
(a) Gp IB/IIIA
(b) Gp IB/IX
(c) Gp IIB/IIIA
(d) Gp IIB/IX
40. Shelf life of platelets is?
(a) 24 hours
(b) 5 days
(c) 9 days
(d) 3 days
41. Mega platelets are seen in which of the following disease?
(a) Glanzmann thrombasthenia
(b) Von Willebrand disease
(c) Wiskot-Aldrich Syndrome (WAS)
(d) Bernard-Soulier syndrome
42. Hemolytic uremic syndrome is associated with which of the
following:
(a) Most commonly caused by verocytogenic E.coli
(b) Causes mild to severe coombs positive hemolytic anemia
(c) Recurrences are rare
(d) Transient thrombocytopenia

BLOOD GROUPING, BLOOD TRANSFUSION, ANTI COAGULANTS

43. Which of the following regarding Bombay blood group is false?


(AIIMS May 2012)
(a) Lack of H, A and B antigen on RBCs
(b) Lack of H, A and B substance in saliva
(c) Lack of antigens of several blood group systems
(d) H, A and B antibody will always be present in serum
44. Which of the following is the genotype of a person with blood group
A?
(AI 2012)
(a) BO
(b) AO
(c) AB
(d) OO
45. Secondary hemochromatosis is associated with all except:
(AI 2012)
(a) Thalassemia
(b) Sideroblastic anemia
(c) Multiple drug transfusions
(d) Paroxysmal nocturnal hemoglobinuria
46. The anticoagulant of choice for anticoagulation testing:
(a) Heparin
(AIIMS Nov 2011)
(b) EDTA
(c) Sodium oxalate
(d) 3.2% trisodium citrate
47.You are working in a PHC and have to send a sample for blood
glucose estimation. Which of the following anticoagulant will you use
for sending your sample?
(a) EDTA
(AIIMS Nov 2011)
(b) Heparin
(c) Potassium oxalate + sodium fluoride
(d) Tri Sodium citrate
48. A newborn with ABO incompatibility, the characteristic feature on
peripheral smear is the presence of:
(a) Microspherocytes
(AIIMS Nov. 2010)
(b) Fragmented RBC
(c) Polychromasia
(d) Elliptocytosis
49. Most common blood transfusion reaction is:
(a) Febrile non-hemolytic transfusion reaction
(b) Hemolysis
(AI 2008)
(c) Transmission of infections
(d) Electrolyte imbalance
50. Rh antigen is a/an:
(AI 2008)
(a) Antibody
(b) Mucopolysaccharide
(c) Protein
(d) Fatty acid
51. A 40 years old male had undergone splenectomy 20 years ago.
Peripheral blood smear examination would show the presence of:
(AI 2003)
(a) Dohle bodies
(b) Hypersegmented neutrophils
(c) Spherocytes
(d) Howell-Jolly bodies
52. Which of the following complications is likely to result after several
units of blood have been transferred?
(a) Metabolic alkalosis
(AI 2001)
(b) Metabolic acidosis
(c) Respiratory alkalosis
(d) Respiratory acidosis
53. ABO incompatibility not seen with:
(a) Fresh frozen plasma
(AIIMS Nov 2009)
(b) Platelet rich plasma
(c) Single donor platelets
(d) Cryoprecipitate
54. A 55 years old male accident victim in casualty urgently needs blood.
The blood bank is unable to determine his ABO group, as his red
cell group and plasma group do not match. Emergency transfusion
of the patient should be with:
(AIIMS Nov 2002)
(a) RBC corresponding to his red cell group and colloids/crystalloid
(b) Whole blood corresponding to his plasma group
(c) O positive RBC and colloids/crystalloid
(d) AB negative whole blood
55. Although more than 400 blood groups have been identified, the ABO
blood system re-mains the most important in clinical medicine
because:
(AIIMS Nov 2002)
(a) It was the first blood group system to be discovered
(b) It has four different blood group A, B, AB, O (H)
(c) ABO (H) antigens are present in most body tissues and fluids
(d) ABO (H) antibodies are invariably present in plasma when persons
RBC lack the corresponding antigen
56. Which of the following is seen in peripheral smear of a patient who
has underwent splenectomy:
(a) Howell-Jolly bodies
(PGI Dec 2001)
(b) Eosinophilia
(c) Macrocytosis
(d) Thrombocytopenia
(e) Neutrophilia
57. Blood component products are all except:
(a) Whole blood
(PGI Dec 2005)
(b) Platelets
(c) Fresh frozen plasma
(d) Packed red blood cells
(e) Leukocyte reduced RBC

MOST RECENT QUESTIONS


58. The anticoagulant of choice for performing coagulation studies is:
(a) EDTA
(b) Heparin
(c) Trisodium citrate
(d) Double oxalate
59. Howell Jolly bodies are seen in:
(a) Liver disease
(b) Postsplenectomy
(c) Hemolysis
(d) DIC
60. Spur cell anemia is caused by:
(a) Chronic liver disease
(b) Acute blood loss
(c) Chronic blood loss
(d) None
61. Hypersplenism is characterized by all except:
(a) Leukemoid reaction
(b) Thrombocytopenia
(c) Splenomegaly
(d) Responds to splenectomy
62. The antigen lacking in Rh negative person is:
(a) C
(b) D
(c) d
(d) E
63. Platelets growth factor are synthesized by:
(a) Glial cells
(b) Endothelium
(c) Fibroblasts
(d) All of the above
64. Hemophilia B is due to deficiency of:
(a) Factor VIII
(b) Factor VII
(c) Factor IX
(d) Factor X
65. All are true about polycythemia vera except:
(a) Increased LAP score
(b) Increased erythropoietin level
(c) Splenomegaly
(d) May cause Budd-Chiari syndrome
66. Elevated ESR is seen in following conditions except:
(a) Polymyositis rheumatica
(b) Multiple myeloma
(c) Temporal arteritis
(d) Polycythemia rubra
67. The major hemoglobin present in an adult is:
(a) HbA2
(b) HbA1
(c) HbA1c
(d) HbA1b
68. Patient with hemophilia A have bleeding disorder because of:
(a) Lack of platelet aggregation
(b) Lack of reaction accelerator during activation of factor X in
coagulation cascade
(c) Neutralization of antithrombin III
(d) Release of Thromboxane A2
69. Stored plasma is deficient in:
(a) Factors 7 and 8
(b) Factors 5 and 7
(c) Factors 5 and 8
(d) Factors 5, 7 and 8
70. All cause pseudohyperkalemia, except:
(a) Thrombocytopenia
(b) Leucocytosis
(c) Clenching of fists
(d) Hemolysis
71. True about hemophilia is:
(a) If the male is affected, it will transmit to male
(b) Normal PT
(c) Low PT
(d) Low aPTT
72. True about von Willebrand factor are all except:
(a) Component of factor VIII
(b) Synthesized by spleen
(c) Facilitate the adhesion of platelets
(d) Leads to increased APTT
73. Bence Jones protein in urine are due to the presence of:
(a) Light chain of monoclonal immunoglobulins
(b) Heavy chain of monoclonal immunoglobulins
(c) Light chain of polyclonal immunoglobulins
(d) Heavy chain of polyclonal immunoglobulins
74. “Starry Sky” pattern is seen in all of these except:
(a) Burkitt’s lymphoma
(b) Large B cell lymphoma
(c) Small cleaved cell lymphoma
(d) Lymphoblast lymphoma
75. Hemophilia is associated with:
(a) X chromosome
(b) Y Chromosome
(c) Chromosome 3
(d) Chromosome 16
76. Blood group antigens are:
(a) Carried by sex chromosomes
(b) Attached to plasma proteins
(c) Attached to hemoglobin molecule
(d) Sometimes found in saliva
77. A 7-year-old girl presents with bleeding in joints. She has prolonged
aPTT, normal PT and platelet counts. What could be the deficiency?
(a) Factor IX
(b) Factor VIII
(c) Factor VII
(d) von Willebrand Factor
78. Carbohydrate present in blood group substance is:
(a) Fucose
(b) Deoxyribose
(c) Ribulose
(d) Ribose
79. Hemophilia A is characterized by:
(a)Prolonged PTT
(b) Prolonged PT
(c) Low platelet count
(d) Abnormal BT
80. RBCs are stored at what temperature:
(AIIMS May 2016)
(a) 2–6 °C
(b) –2 to–4 °C
(c) 37 °C
(d) 20–25°C
81. RBC should be transfused with a needle having which of the
following size?
(AIIMS May 2016)
(a) With a 16–18 G needle
(b) With a 18–20 G needle
(c) With a 20–22 G needle
(d) With a 22–24 G needle
82. Blood group antigens chemically are made up of ?
(a) Carbohydrate
(b) Glycoprotein
(c) Phospholipids
(d) Polysaccharide
83.Routine Rh typing includes testing?
(a) A antigen
(b) B antigen
(c) C antigen
(d) D antigen
84. Which of the following is true about h/h blood group?
(a) Lacks H-antigen
(b) Lacks A-antigen
(c) Lacks H-antigen
(d) All of the above
85. Which of the following is the chromosomal location of Rh antigen?
(a) 1
(b) 9
(c) 19
(d) 13
86. Anticoagulant used in coagulation study is?
(a) Calcium citrate
(b) EDTA
(c) Sodium bromide
(d) Trisodium citrate
87. Anticoagulant used for chelating calcium-
(a) EDTA
(b) Oxalate
(c) Sodium citrate (d) All of the above
1. Ans. (b) Glanzmann’s thrombasthenia > (d) Bernard Soulier disease
(Ref: Robbins 8th/, Wintrobe’s 12th/1365-1370)
Presence of normal PT and aPTT rules out the presence of any clotting
factor deficiency. So, options like factor X deficiency and von Willebrand
disease are ruled out. Clinically, both Glanzmann thrombasthenia and
Bernard Soulier syndrome are indistinguishable. However, most of the
haematologists agreed on placing Glanzmann thrombasthenia in
preference to Bernard Soulier syndrome as the answer. We got an article
supporting the increased prevalence of Glanzmann in comparison to
Bernard Soulier in Western India as well.
Name of Bernard Soulier disease Glanzmann’s thrombasthenia
disease
Cause of Defect in the platelet GpIb-IX Defect in platelet Gp IIb/IIIa
disease complex
Lab findings ↑BT, mild thrombocytopenia, ↑BT, deficient clot retraction time,
deficient or low levels of platelet deficient platelet aggregation with ADP,
GpIb-IX complex by flowcytometry collagen, thrombin, adrenaline
Ristocetin aggregation test is Ristocetin aggregation test and
defective coagulation tests are normal
Platelet Giant platelets under normal smear Platelets are normal under microscope
morphology

2. Ans. (c) Factor VIII inhibitors


(Ref: Robbins 8th/672-3, Wintrobe 12th/1447-1453, Harrison 18th/982)
This is a case of 25 year old asymptomatic female whose parametric inference
is as follows:
• BT is 3 min (normal)
• PT is 15 sec/14 sec (normal)
• aPTT is 45 sec/35 sec (raised)
• Platelet count is 2.5lac/mm3 (normal)
• Factor VIII levels were 60IU/dL (normal i.e. between 50-150IU/dL).

Analyzing all the options one by one


Option ‘a’… Factor IX deficiency would have resulted in increased aPTT.
However females are less likely to suffer from hemophilia B because it is
an X-linked disease. Additionally, the lady is pregnant. This favours an
autoimmune etiology.
Option ‘b’….. Lupus anticoagulant (Wintrobe 12th/1452, Harrison 18th/982)
clearly says that these patients have thrombocytopenia (here platelet
count is normal), prolonged PTT and hypoprothrombinemia due to
antibodies against prothrombin (normal in this case)
Option ‘c’….Factor VIII inhibitors (Wintrobe 12th/1441-1444, Harrison 18th/982);
Tejender Singh hematology page no 308)
• Acquired coagulant inhibitor is immune-mediated condition
characterized by autoantibody against a specific clotting factor and
factor VIII is most common target.
• In 50% of patients no underlying disease is indentified at the time of
diagnosis. In remaining the causes are autoimmune disease,
malignancies, dermatologic diseases, pregnancy and post partum.
• Diagnosed by isolated prolongation of PTT with normal PT
(coagulation profile is similar to haemophilia A). Platelet count and
bleeding time is normal (matches with the data given in our
question).
Option ‘d’…. VWD – type III (Robbins 8th/672-3): Bleeding time will be
prolonged (in this patient BT is normal) with abnormal platelet count. So,
option‘d’ is excluded.
So the likely diagnosis for the patient in the stem of the question is presence of
acquired coagulation inhibitors against Factor VIII.
3. Ans. (c) Done within 2 hours
(Ref: Dacie and Lewis practical hematology 10th/392, 398)

Be aware!
Platelet poor plasma is for coagulation studies whereas platelet rich plasma is
used for platelet disorder studies.

4. Ans. (d) Acquired Factor VIII inhibitors


(Ref: Wintrobes 12th/1442-4, 1274)
The clinical presentation in a young female of recurrent joint pains with
petechial hemorrhage is suggestive of an autoimmune disease. Options
‘a’, ‘b’ and ‘c’ are rare because these would present with some additional
symptoms apart from the ones mentioned in the question. They would
also be present since birth.
Talking about option ‘d’,
A female patient is unlike to have hemophilia as it is an X linked disorder.
However, she can have autoantibodies against factor VIII. This could be
due to other conditions like pregnancy or coexisting autoimmune
disease.
5. Ans. (b) Normal platelet count and increased BT
(Ref: Harrison 17th/723, Robbins 8th/670, Robbins 9/e 656)
The stem of the question clearly mention the fact that there is platelet
function defect. It means that the platelet count is normal with a
problem in the functioning of platelets. see text for details.
6. Ans. (d) Defect in common pathway
(Ref: Robbins 9th/119)

• Defect in the extrinsic pathway causes elevation of PT


• Defect in the intrinsic pathway causes elevation of aPTT.
• Defect in common pathway cause elevation of both PT and aPTT.
• Platelet function defect causes elevation of BT.

7. Ans. (d) Grossly abnormal coagulation tests


(Ref: Harrison 17th/722, Robbins 9/e p659-660)
If coagulation tests indicate a major consumption of procoagulants, the
diagnosis of TTP is doubtful.
8. Ans. (a) Acute promyelocytic leukemia
(Ref: Harrison 17th/679, Robbins 9/e p612)
9. Ans. (c) Aplastic anemia
(Ref: Harrison 17th/719,720, 667-668 & 16th/674, 622)
• Megakaryocyte is a precursor of platelet. Any peripheral destruction
of platelets causes increased activity of bone marrow resulting in
megakaryocytic thrombocytopenia because of compensatory
increase in megakaryocytes.
• However, if thrombocytopenia occurs due to any defect in the bone
marrow itself, the compensatory increase in megakaryocytes will not
occur. This is known as amegakaryocytic or hypoplastic
thrombocytopenia.
Aplastic anemia is also an example of amegakaryocytic thrombocytopenia.
10. Ans. (a) D-dimer will be normal
(Ref: Wintrobe’s Hematology 11th/1672)
• Coagulation defects in severe liver disease include elevated
thrombin time, prothrombin time and activated partial thromboplastin
time.
• All factors procoagulant as well as anti clotting factors (antithrombin
III, protein C and protein S) being synthesized in the liver are
reduced in liver dysfunction.
• Fibrin degradation products are increased in patients with severe
liver disease and DIC because endogenous plasminogen activators
are removed by the normal liver. In severe liver disease, they
circulate for long time and cause activation of the fibrinolytic system.
D-dimer is usually normal in liver disease patients. It is increased with DIC.
Wintrobe’s mentions page 1396-7 that ..Normal fibrinogen level is 150-350
mg/dl and the levels between 50-100mg/dl are required for normal
hemostasis. However, I could not find any level specifically in cirrhosis
patients friends.
11. Ans. (d) Wiskott Aldrich syndrome
(Ref: Robbins 7th/244, 8th/235, 9/e p242)
Wiskott Aldrich syndrome is characterized by the triad of:
• Severe eczema
• Thrombocytopenia
• Recurrent infections
The platelets are small and are reduced in number in Wiskott Aldrich syndrome.
About other options, in ITP, Bernard Soulier syndrome and Myelofibrosis, there
is increase in size of platelets.
12. Ans. (a) Serotonin; (b) Ig mediators; (d) Interaction among the
platelets; (e) Macromolecules;
(Ref: William’s Haematology 61th/1366, Harrison 17th/363-364, Robbins
9/e p117-118)
Agonists of platelet aggregation:
Adhesins vWF Plasmin Serotonin
Thrombospondin Fibrinogen Immunocomplex Vasopressin
TxA2 ADP Epinephrine
P-selectin mediates the interaction between WBC and platelets.
13. Ans. (a) Abruptio placentae; (b) Acute promyelocytic leukemia; (c)
Severe falciparum malaria; (d) ‘Snake envenomation; (e) Heparin
overdose.
(Ref: Robbins 7th/657, 9/e 663-664, KDT 5th- 562)
14. Ans. (b) Increased PT; (d) Decreased fibrinogen; (e) Decreased
platelets
(Ref: Robbins 9/e 664-665)
Laboratory findings in DIC:
*Low platelet count.
*Microangiopathic hemolytic anemia
*Elevated PT, PTTK, TT
*Plasma fibrinogen level decreased
*Fibrin degradation products (FDP) are raised.
*Factor V and factor VIII decreased.

15. Ans. (a) Anaerobic sepsis; (b) Malignancy; (c) Leukemia.


(Ref: Robbins 7th/657, 9/e 663-664)
16. Ans. (a) Glanzmann’s syndrome; (b) Bernard-Soulier syndrome; (c)
Wiskott Aldrich syndrome; (d) von Willebrand disease
(Ref: Harrison 17th/723, Robbins 9th/660)
For details, See text
Leucocytosis is seen in all of the given options i.e. brucellosis, acute MI, and
diphtheria but in typhoid, there is leucopenia.
17. Ans. (d) All of the above
(Ref: Harrison 17th/723-724; Robbins 8th/118, 9/e p662)
18. Ans. (d) Angiogenesis inhibitory protein
(Ref: Robbins 9/e p306, 8th/96, 7th/105)
• Thrombospondin, is a family of large multi-functional proteins, some
of which, similar to SPARC (secreted protein acidic and rich in
cysteine), inhibit angiogenesis.
• The production of thrombospondin-I has been shown to be inversely
related to the ability of a cell line to produce a tumor and vessels in
vivo; loss of thrombospondin-I production allowed non-tumorigenic
cells to become tumorigenic.
• Thrombospondin-I is regulated by wild-type p53.
19. Ans. (a) Thromboplastin (Ref: Robbins, 7th/649, 9/e 656)
20. Ans. (a) Myelofibrosis (Ref: Harrison 17th/723)
Causes of Thrombocytosis
• Iron deficiency anemia • Myelodysplasia
• Hyposplenism • Post surgery
• Postsplenectomy • Infection
• Malignancy • Polycythemia vera
• Collagen vascular disorder • Hemolysis
• Idiopathic myelofibrosis • Hemorrhage
• Essential thrombocytosis • Idiopathic sideroblastic anemia
• CML • Rebound (cessation of ethanol intake,
correction of B12 and folate
deficiency).

21. Ans. (a) Synthesized by hepatocytes


(Ref: Robbins 8th/670-671, 9/e 661)
In the given options, (a) seems to be the best answer though even the liver
cells can produce a small amount of von-Willebrand factor. For
clarification of other options, see text
22. Ans. (d) VIII (Ref: Robbins 8th/835, 9/e 661)
23. Ans. (d) Antithrombin (Ref: Harsh Mohan 6th/340)
24. Ans. (c) von Willebrand disease (Ref: Robbins 9/e 662)
25. Ans. (c) t (14: 18) (Ref: Robbins 9/e 591)
26. Ans. (a) Decrease in neutrophil count
(Ref: Robbins 8th/592-593, 9/e p582)
27. Ans. (b) 1,00,000/cmm (Ref: Robbins 9/e p657)
28. Ans (b) Glanzmann disease (Ref: Robbins 9th/118)
Inherited deficiency of GpIIb-IIIa results in a bleeding disorder called
Glanzmann thrombasthenia.
29. Ans (b) Relapse is rare (Robbins 9th/658)
Direct quote…. “Almost all patients respond to glucocorticoids (which inhibit
phagocyte function), but many eventually relapse.”
• In individuals with severe thrombocytopenia, splenectomy normalizes
the platelet counts.
• Immunomodulatory agents such as intravenous immunoglobulin or
anti-CD20 antibody (rituximab) are often effective in patients who
relapse after splenectomy or for whom splenectomy is
contraindicated.
• Peptides that mimic the effects of thrombopoietin (so-called TPO-
mimetics) are also useful for improving the platelet counts of the
patients.
30. Ans: (a) Chronic ITP…see earlier explanation
31. Ans. (c) Congenital defect of platelets
(Ref: Robbins 8/e p670, 9/e p660)

• Glanzmann thrombastheniais a defect in platelet aggregation (both have ‘g’ in


them).
• Bernard Soulier syndrome is a defect in platelet adhesion (both have‘d’ in them).

Glanzmann thrombasthenia, which is also transmitted as an autosomal


recessive trait. Thrombasthenic platelets fail to aggregate in response to
adenosine diphosphate (ADP), collagen, epinephrine, or thrombin
because of dysfunction of glycoprotein IIb-IIIa, an integrin that
participates in “bridge formation” between platelets by binding fibrinogen
32. Ans. (c) Decreased fibrin degradation products…discussed earlier
(Ref: Robbins 9/e 662-663)
DIC is associated with increased fibrin degradation products.
33. Ans. (a) Normal ADAMTS levels
(Ref: Robbins 9/e 659-660, 8/e p669-670, 7/e p652-653) ...see text for
details
34. Ans (c) Shwachman syndrome…………….
(Ref: Harrison 18th/Ch 107)
Shwachman-Diamond syndrome is associated with pancreatic insufficiency, marrow
failure and malabsorption.

35. Ans. (a) Fresh frozen plasma (Ref: Robbins 9th/664)


The following flowchart helps us to understand that fresh frozen plasma is the
best blood product to be given in a patient of multiple clotting factor
deficiency with active bleeding.

36. Ans. (a) Develops within 24 hours


(Ref: Critical Care Study Guide 2nd/1102)
Salient features of TRALI
• TRALI develops within 6 hours of transfusion.
• It is a cause of non cardiogenic pulmonary edema
• Fresh frozen plasma is the commonest blood product to cause it.
• Mostly seen after sepsis and cardiac surgeries
• Clinically, it is characterised by acute onset of respiratory distress, associated
with oxygen desaturation (hypoxemia) and bilateral lung infiltrates.

37. Ans (d) Mantle cell lymphoma (Ref: Robbins 9/e p602)
38. Ans (b) decreased agglutination (Ref: Robbins 9/e p662)
39. Ans (c) Gp IIB/IIIA (Ref: Robbins 9/e p118)
40. Ans (b) 5 days (Essentials of Hematology 2/e p492)
Maximum shelf life of platelets is 5 days.
41. Ans (d) Bernard soulier syndrome
(Ref: Wintrobe 12/e p1277, CMDT2018/563)
42. Ans (b) Causes mild to severe coombs positive hemolytic
anemia (Ref: Robbins 9/e p643-660
Coombs test is done for immune hemolytic anemia whereas HUS is non
immune hemolytic anemia.
43. Ans. (c) Lack of antigens of several blood group systems
(Ref: Wintrobe’s hematology 12th/635-6; Harrison 18th/951)
• Red cells of group O individuals lack A and B antigens but carry H
substance
• The enzyme in group A individuals is N-acetylgalactosaminosyl
transferase
• The enzyme in group B individuals is D-galactosyltransferase

• People with Bombay phenotype (rarest blood group in the world) express no A, B or
H antigens on the red blood cells. These are homozygous for the silent h allele
(being represented hh). So, these antigens are not present in the saliva also. As the
antigens are not expressed, so, the H, A and B antibody will always be present in
serum.
• ABO antigens are present not only on the red blood cells abut also on the other blood
cells, in most body fluids (except CSF), cell membrane of tissues such as intestine,
urothelium and vascular endothelium.

• Clinical significance of knowing about Bombay blood group is that in


case of requirement of blood transfusion, these people would be
compatible only with Bombay blood from another individual.
44. Ans. (b) AO ...Too obvious to explain friends.
45. Ans. (d) Paroxysmal nocturnal hemoglobinuria
(Ref: Robbins 8th/861 (table 18-6, 9/e 847)
I. Hereditary Hemochromatosis
• Mutation in the gene coding for HFE, transferring receptor 2 or hepcidin
• Mutation in the gene coding for hemojuvilin:juvenile hemochromatosis

II. Secondary Hemochromatosis


A.Parenteral iron overload: Transfusions, Long-term hemodialysis, Aplastic
anemia, Sickle cell disease, Leukemias, Myelodysplastic syndromes Iron-dextran
injections
B.Ineffective erythropoiesis with increased erythroid activity
β-Thalassemia, Sideroblastic anemia and Pyruvate kinase deficiency

C.Increased oral intake of iron: African iron overload (Bantu siderosis)


D. Congenital atransferrinemia
E. Chronic liver disease: Chronic alcoholic liver disease and Porphyria cutanea
tarda.

46. Ans. (d) 3.2% trisodium citrate


(Ref: Wintrobe 12th/1, Dacie and Lewis practical hematology 10th/7,
391) ...See text
47. Ans. (c) Potassium oxalate + sodium fluoride
(Ref: Henry’s Clinical Diagnosis and Management by Laboratory
Methods 21st/188, Clinical Chemistry Theory, Analysis and
Correlation (Mosby) 4th/23-4, 71, Harper 27th/152)
Direct quote from Harper…. ‘enolase enzyme in the glycolysis is inhibited by
fluoride, and when blood samples are taken for measurement of glucose,
it is collected in tubes containing fluoride to inhibit glycolysis.’
48. Ans. (a) Microspherocytes
(Ref: Wintrobe’s 12th/982-3, T. Singh Hematology 2nd/32-3, Handbook of
pediatric transfusion medicine by Hillyer 1st/198)

• Wintrobe’s mentions ‘spherocytes predominate in the peripheral blood smear of


infants with ABO hemolytic disease of newborn.’ Peripheral blood smear shows
numerous spherocytes, occasional nucleated red cells, anisocytosis and
polychromasia.
• The blood film in ABO hemolytic disease of the newborn (ABO HDN) is marked by
the presence of microspherocytes (a feature not usually seen in Rh hemolytic
disease of the newborn). The spherocytosis is attributed to loss of membrane
surface area when the spleen removes antigen-antibody complexes from the
affected cell.

Other options
Option B…Fragmented RBC or schistocytes are feature of
microangiopathic hemolytic anemia, DIC and cardiac hemolytic anemia.
Option C…polychromasia is the term used for red cells staining bluish red with
Roamnowsky stains. These cells are larger than normal and show fine
reticulin network in supravital staining. They are commonly observed in
response to therapy in deficiency anemias and hemolytic anemia. So, is
not specific for ABO incompatibility.
Option D…Elliptocytosis is a feature of hereditary elliptocytosis and macrocytic
anemias.
49. Ans. (a) Febrile non-hemolytic transfusion reaction
(Ref: Harrison 16th/665-666, Robbin 9/e 665)
The most frequent reaction associated with the transfusion of cellular blood
components is a febrile non-hemolytic transfusion reaction.

FNHTR is characterized by chills and rigor and > 1°C rise in temperature.

50. Ans. (c) Protein


(Ref: Wintrobe’s hematology, 11th/797; Harrison 17th/708)
• Unlike other red cell antigens, Rh antigens do not contain any
sugar. The Rh proteins are multipass membrane proteins that
traverse the RBC membrane 12 times. The Rh proteins form a
complex with other membrane glycoproteins.

Please remember friends that the Rh antigen should not be confused with Rh factor
which is an antibody (Ig) against the Fc portion of IgG seen in patients of rheumatoid
arthritis..

51. Ans. (d) Howell-Jolly bodies


(Ref: Harrison’s 17th/374-375, 9/e 623, 636)
Howell-Jolly bodies are spherical or ovoid eccentrically located granules in
stroma of erythrocytes in stained preparations. These represent nuclear
remnants and these occur most frequently after:

1. Splenectomy 2. Megaloblastic anemia 3. Severe hemolytic anemia

Acute manifestations of splenectomy include leukocytosis (up to 25000/


µl) and thrombocytosis (up to 1 × 106/µl) but these return back to
baseline levels within 2-3 weeks.
Chronic Manifestations of splenectomy include:
• Anisocytosis and poikilocytosis
• Howell-Jolly bodies (nuclear remnants)
• Heinz bodies (denatured hemoglobin)
• Basophilic stippling
• Occasional nucleated erythrocyte in peripheral blood
Other options
When such erythrocyte abnormalities are seen without splenectomy, splenic infiltration by
tumor should be suspected.
• Dohle bodies are discrete round or oval bodies. These represent
rough ER and glycogen granules and are found in neutrophils.
These may be seen in patients with infections, burns, trauma,
pregnancy or cancer.
• Hypersegmented neutrophills are seen in megaloblastic anemia.
• Spherocyte may be seen in hypersplenism and not after
splenectomy.
52. Ans. (a) Metabolic alkalosis
(Ref: Harrison 17th/293, 9/e 666)
Massive Transfusion is defined as the need to transfuse from one to two
times the patient’s normal blood volume. In a normal adult, this is
equivalent to 10-20 units. Most common abnormality is metabolic
alkalosis. It results from conversion of citrate (present in stored blood)
and lactate (accumulated due to hypoperfusion) to bicarbonate
53. Ans. (d) Cryoprecipitate
(Ref: Harrison 17th/708-710, Wintrobes 11th/846-8)
Wintrobe’s clearly mentions that ‘ABO incompatible plasma carries high risk of
transfusion reactions, therefore, plasma transfusions should always be
ABO incompatible’. So, option ‘a’ and ‘b’ are ruled out.
The platelets bear the intrinsic ABO antigens. So, platelet rich plasma and
single donor platelets also should be preferably ABO compatible. So,
answer of exclusion is cryoprecipitate. It is not carrying ABO antigens
but should preferably be ABO compatible to avoid even the minimal risk
of hemolytic reaction.
Apheresis technology is used for the collection of multiple units of platelets
from a single donor. These single-donor apheresis platelets (SDAP)
contain the equivalent of at least six units of random donor (RD) platelets
and have fewer contaminating leukocytes than pooled RD platelets. Still
the risk of severe hemolytic reactions is much more with single
donor incompatible platelets than pooled plasma because the dose
of incompatible plasma is also increased.

FFP is an acellular component and does not transmit intracellular infections, e.g. CMV

54. Ans. (c) O positive RBC and colloids/crystalloid


(Ref: Wintrobes Clinical Hematology vol.I, 10th/833, CMDT 2010/477)
Selection of blood for emergency transfusion

• If patient’s blood group is known, unmatched blood group of the same group
may be used.
• If the patient’s blood cannot be determined, Group O red blood cells should be
chosen. The use of such unmatched blood should be Rh (–ve) when used in
woman of child-bearing age in whom we do not want sensitization to Rh antigen. As
Rh negative blood is often in limited supply, Rh positive blood is used in the
emergency transfusion of older females and males of unknown blood group. In
such cases sensitization may occur but the risk of an immediate hemolytic reaction
is low. O blood group is the universal donor and therefore, should be given to this
patient.

55. Ans. (d) ABO (H) antibodies are invariably present in plasma when
persons RBC lacks the corresponding antigen (Ref: Harrison
17th/708, CMDT 2010/477)
• In clinical transfusion practice the ABO blood groups are the most
important and can never be ignored in red cell transfusion, because
individuals, who genetically lack any antigen, have antibodies
against the red cell types that they have not inherited. These
antibodies can destroy red cells rapidly in circulation.
• The same is not the case with other blood groups where antibodies
are formed only after exposure to the sensitive antigen (Preformed
antibodies are absent).
56. Ans. (a) Howell-Jolly bodies; (c) Macrocytosis
(Ref: Harrison’ 17th/374-375, 9/e 623-636)
Chronic manifestations of splenectomy (Postsplenectomy hematological
features) are:
• Red cells: Marked variation in size and shape (anisocytosis,
poikilocytosis)
• Macrocytosis
• Presence of Howell-Jolly bodies (nuclear remnants)
• Heinz bodies (denatured hemoglobin)
• Basophilic stippling
• Occasional nucleated red cells in the peripheral blood
• Target cells
• Pappenheimer bodies (contain sideroblastic granules)
• Irregular contracted red cells.
– WBC count usually normal but there may be mild
lymphocytosis and monocytosis.
– Thrombocytosis persists in about 30% of cases.
57. Ans. (a) Whole blood (Ref: Harrison 17th/709)
Whole blood is processed into its components intended for transfusion. The
blood component products are:
Packed RBC FFP
Platelets Cryoprecipitate
Plasma derivatives, e.g. albumin, antithrombin, Leukocyte reduced RBC
coagulation factors
58. Ans. (c) Trisodium citrate
(Ref: Wintrobe’s Clinical Hematology 11th/4)
59. Ans. (b) Postsplenectomy (c) Hemolysis
(Ref: Tejinder singh’s 1st/38-39, internet)
Friends, in hemolytic anemia Howell Jolly body is seen only if anemia is very
severe. So, the preferred answer is post splenectomyQ
Howell-Jolly bodies are nuclear remnants seen in red cells, intermediate or
late normoblasts. They are seen in:

• Normally in neonates (spleen is immature)


• Megaloblastic anemia is due to dyserythropoiesis
• Post splenectomy due to absence of pitting function of the spleen
• Acute severe hemolytic anemias
• Hyposplenia (radiation exposure, splenic trauma, autosplenectomy due to sickle
cells disease)
• Myelodysplastic syndrome

60. Ans. (a) Chronic liver disease


(Ref: Harrison 17th/359, T. Singh 1st/86)
61. Ans. (a) Leukemoid reaction (Ref: Robbins 9/e 719)
62. Ans. (b) D (Ref: Robbins 8th/460, 7th/485)
63. Ans. (b) Endothelium (Ref: Robbins 8/e p434, 9/e p19-20)
Platelet-derived growth factor (PDGF) is present in the following:
• The alpha granules of the platelets
• Macrophages
• Endothelial cells
• Keratinocytes and
• Smooth muscle cells.
64. Ans. (c) Factor IX (Ref: Robbins 8/e p672, 9/e 662-663)
Disorder Deficiency
• Hemophilia A • Factor 8
• Hemophilia B Hemophilia C • Factor 9 (Christmas factor)
• Parahemophilia • Factor 11
• Factor 5 (labile factor)

65. Ans. (b) Increased erythropoietin level


(Ref: Robbins 9/e 656, 8/e p628, 7/e p699)
Polycythemia vera progenitor cells have markedly decreased
requirements for erythropoietin and other hematopoietic growth
factors. Accordingly, serum erythropoietin levels in polycythemia
vera are very low, whereas almost all other forms of absolute
polycythemia are caused by elevated erythropoietin levels.
66. Ans. (d) Polycythemia rubra (Ref: Robbins 9/e 619)
Lab manifestations in polycythemia rubra (CMDT)
• Elevated hemoglobin level and hematocrit: due to increased number
of red blood cells
• Platelet count or white blood cell count may also be increased.
• Erythrocyte sedimentation rate (ESR) is decreased due to an
increase in zeta potential.
• Low erythropoietin (EPO) levels.
67. Ans. (b) HbA1 (Read below)
68. Ans. (b) Lack of reaction accelerator during activation of factor X in
coagulation cascade. (Ref: Robbins 9/e 662)
Hemophilia A is caused by the deficiency of clotting factor 8.
• The chief role of the extrinsic pathway in hemostasis is to initiate a
limited burst of thrombin activation upon tissue injury. This initial
procoagulant stimulus is reinforced and amplified by a critical
feedback loop in which thrombin activates factors XI and IX of the
intrinsic pathway. In the absence of factor VIII, this feedback loop
is inactive and insufficient thrombin (and fibrin) is generated to
create a stable clot.
69. Ans. (c) Factors 5 and 8 (Ref: Internet)
FFP contains an average of 1 IU/mL of each coagulation factor, including
the labile factors V and VIII. In the question it was asked regarding stored
plasma.
By day 5 of storage the amount of factor VIII (8) is reduced by up to 40% and
factors V (5) and VII (7) may be reduced by up to 20.
70. Ans. (d) Hemolysis (Ref: Hematology manual)
Conditions with pseudohyperkalemia
• Cessive muscle activity during venipuncture (fist clenching),
• Thrombocytosis, leukocytosis, and/or erythrocytosis
• Acute anxiety
• Cooling of blood after venipuncture
• Gene defects leading to hereditary pseudohyperkalemia
Hemolysis causes real hyperkalemia.
71. Ans. (b) Normal PT (Ref: Robbins 9/e 662)
As discussed earlier, hemophilia A is an X linked disorder, so, if a male is
affected, he cannot transmit it to his son (option ‘a’ is false). In this
sdiorder, there is an increase in the activated partial thrombolastin
time and normal values of prothrombin time.
72. Ans. (b) Synthesized by spleen (Ref: Robbins 9/e 661)
vWF is produced by endothelial cells, megakaryocytes and liver.
73. Ans. (a) Light chain of monoclonal immunoglobulins
(Ref: Robbins 9/e p600)
Bence Jones proteins are made up of light chains of the immunoglobulin and
are monoclonal in nature.
74. Ans. (c) Small cleaved cell lymphoma
(Ref : Pattern Approach to Lymph Node Diagnosis by Anthony S-Y
Leong)
Friends, I know people would require reference for this question for
believing the answer! JJ The tumor cells, which are large with minimal
cytoplasm, are closely apposed to each other, forming a dark blue
background (the “sky”). These cells have a very high turnover rate, so
the macrophages that happen to be hanging around get stuffed with
cellular debris (they are at this point called “tingible body macrophages”),
and upon fixation, the cytoplasm falls away, leaving round white spaces
filled with debris (the “stars”). This pattern can be seen on both bone
marrow or lymph node sections. It is seen with:
• Burkitt’s lymphoma (earlier called as small non cleaved lymphoma)
• Mantle cell lymphoma
• Large B cell lymphoma (including plasmablastic lymphoma)
• T lymphoblastic lymphoma
75. Ans. (a) X chromosome (Ref: Robbins 8/e p672, 9/e 662)
Hemophilia A is caused by mutations in factor VIII, which is an essential
cofactor for factor IX in the coagulation cascade. It is inherited as an X-
linked recessive traitQ and thus affects mainly males and homozygous
females.
Read the following lines carefully for a future NEET question!

• Hemophilia A is the most common hereditary disease associated with life-


threatening bleeding.
• Von Willebrand disease is the most common inherited bleeding disorder of
humans.

76. Ans. (d) Sometimes found in saliva (Read below)


• The A and B antigens are inherited as mendelian allelomorphs, A
and B being dominants and they are located on chromosome 9.
• They are located on the membranes of human red cells
• Antigens very similar to A and B are common in intestinal bacteria
and possibly in foods to which newborn individuals are exposed.
Therefore, infants rapidly develop antibodies against the antigens
not present in their own cells. Thus, type A individuals develop anti-
B antibodies, type B individuals develop anti-A antibodies, type O
individuals develop both, and type AB individuals develop neither
• Secretors are individuals who secrete ABH antigens in body fluids
like saliva and plasma.
77. Ans. (d) von Willebrand Factor(Ref: Robbins 9/e 661-662)
The clues given in the stem of the question are:
• Female patient
• Bleeding tendency
• Prolonged aPTT
• Normal PT
• Normal platelet count
• As platelet count is normal, thrombocytopenia as a cause of bleeding
can be easily ruled out.
– PT is prolonged in defects of extrinsic pathway of coagulation
whereas aPTT increases in defective intrinsic pathway.
Therefore, deficiency of factor VII can be ruled out, because it
is involved in extrinsic pathway and its deficiency will prolong
PT.
– Factor VIII and IX are involved in intrinsic coagulation pathway
and vWF stabilizes factor VIII. Therefore, deficiency of any of
these will prolong aPTT with PT remaining normal. However,
both Hemophilia A (factor VIII deficiency) and Hemophilia B
(Christmas disease; factor IX deficiency) are X-linked recessive
diseases and commonly affect males. Females are affected
only in homozygous state, which is rare.
– So, the answer is vWF deficiency which is mostly inherited as
autosomal dominant disorder.
78. Ans (a) Fucose (Ref: Harrison 18th/ 951)
The first blood group antigen system was ABO and is the most important
in transfusion medicine. The major blood groups of this system are
A, B, AB, and O. H substance is the immediate precursor on which
the A and B antigens are added. This H substance is formed by the
addition of fucose to the glycolipid or glycoprotein backbone. The
subsequent addition of N-acetylgalactosamine creates the A
antigen, while the addition of galactose produces the B antigen.
79. Ans. (a) Prolonged PTT
(Ref: Robbins 9/e 662)

• aPTT is prolonged by deficiency of factors XII, XI, IX, III, X, V, prothrombin and
fibrinogenQ and drugs like heparinQ
• Hemophilia A is characterized by the deficiency of factor 8 and decreased activity of
intrinsic pathway. This is associated with prolongation of partial thromboplastin
timeQ.

80. Ans. (d) 20–25°C


(Ref: Essentials of Hematology 2nd/500)
Red cells are stored at a temperature of 22–26°C.
81. Ans. (b) With a 18–20 G needle
(Ref: Essentials of Hematology 2nd/500)
The Concept of using a big sized needle is to reduce the risk of hemolysis and
speed up the blood flow rate. As per the norms, the needle to be used in
the adults for blood transfusion is 18-19 guage.
The transfusion has to be started within 4hrs of issue from the blood bank.
82. Ans. (b) Glycoprotein
(Ref: Harrison 19th/ 138 e1)
83. Ans. (d) D antigen
(Ref: Robbins 9th/ 462, Harrison 19th/ 138e-1)
84. Ans. (d) All of the above
(Ref: Harrison 19th/420)
h/h blood group is the other name of Bombay blood group. It is characterized
by absence of H, A and B antigen expression on the surface of red cells.
85. Ans. (a) 1
(Ref: Harrison 19th/ 138e-1)
The three Rh genes, E/e, D, and C/c, are arranged in tandem on chromosome
1.
86. Ans. (d) Trisodium citrate
(Ref: Wintrobe 12th/ chapter 1)
Trisodium citrate is the preferred anticoagulant for platelet and coagulation
studies.
87. Ans. (d) All of the above
(Ref: Wintrobe 12th/ chapter 1)

ANNEXURE

SUMMARY OF LABORATORY FINDINGS IN HEMOSTATIC DISORDERS

Platelet Bleeding
Diseases count time PT APTT FDP

Hemophilia A N N N ↑ Absent

Hemophilia B N N N ↑ Absent

vWD N ↑ N ↑ Absent

Liver failure ↓ ↑ ↑ ↑ Absent

DIC ↓ ↑ ↑ ↑ Present

Vascular purpura N N N N Absent

Aspirin N ↑ N N Absent
Warfarin N N ↑ (Even in ↑ (In high Absent
low dose) dose)
1. Why is citrate phosphate dextrose (CPD) is better than acid citrate
dextrose (ACD) for storage of blood?
(NEET 2020 like pattern)
(a) Because it is less acidic
(b) Improves oxygen transport
(c) Hypertonicity of blood
(d) More citrate ions
Ans. (b) Improves oxygen transport
(Ref: Blood Banking and Transfusion Medicine:Basic Principles and
Practice, 2006 ed 208)
Compared to ACD anticoagulant, CPD has the following advantages for blood
preservation:
• Isotonicity for red blood cells, thus minimizing the lesion of collection and resulting in
improved red blood cell survival,
• More physiological pH,
• 15% less citrate ion, and
• Improved red cell oxygen transport.
For these reasons, many blood banks use CPD in blood collections.
2. Match the following: (AIIMS Nov 2019 like pattern)
1. Burkitt’s lymphoma2. Mantle cell A. t(11,18)…..
lymphoma3. Marginal Zone lymphoma4. B. t(14,18)
Follicular lymphoma C. t(8,14)
D. t(11,14)

Answer key
• 1…C
• 2…D
• 3….A
• 4….B
(Ref: Robbins 9th /591)
The following are the important translocations associated with non Hodgkin
lymphoma:
(a) Burkitt’s lymphoma: t(8,14)
(b) Mantle cell lymphoma: t(11,14)
(c) Marginal zone lymphoma: t(11,18)
(d) Follicular lymphoma: t(14,18)
3. Which of the following is the complication of massive blood
transfusion? (AIIMS Nov 2019 like pattern)
(a) Metabolic acidosis
(b) Metabolic alkalosis
(c) Respiratory alkalosis
(d) Respiratory acidosis
Ans. (b) Metabolic alkalosis (Ref: Harrison 20th/795)
Massive transfusion is defined as the need to transfuse from one to two times
the patient’s normal blood volume. Most common abnormality is
metabolic alkalosis which results from conversion of citrate (present in
stored blood) and lactate (accumulated due to hypoperfusion) to
bicarbonate.
4. von Willebrand factor is secreted by which of the following
cells? (AIIMS Nov 2019 like pattern)
(a) Platelets
(b) Macrophages
(c) Endothelial cells
(d) Neutrophils
Ans. (c) Endothelial cells
Ref: Robbins 9th/662
Von Willebrand factor (vWF) which is produced by endothelial cells and, to a
lesser degree, by megakaryocytes. Inside the endothelial cells, vWF is
being stored in Weibel Palade body.
5. Allergy to transfusion what processing should be done of the blood
before transfusion to avoid it?
(AIIMS May 2019 like pattern)
(a) Irradiation
(b) Washing
(c) Leucocyte reduction
(d) Glycosylation
Ans. (b) Washing Ref: Harrison 20th/813
Patients with a history of allergic transfusion reaction should be
premedicated with an antihistamine and the cellular components
can be washed to remove residual plasma for the extremely
sensitized patient.
Also note that….Anaphylaxis if present is associated with IgA deficiency
• Irradiation is done for graft versus host disease as it destroys the donor T lymphocytes.
• Leucocyte reduction is done for febrile non hemolytic transfusion reaction as (FNHTR) as
it is due to antibodies against donor leukocytes
• Glycosylation is done for autologous blood transfusion.

6. What among the following is the use of this?


(AIIMS May 2019 like pattern)
(a) Prevent viral infections
(b) Prevent transfusion related reactions
(c) Prevent bacterial contamination
(d) Prevent blood mismatch
Ans. (b) Prevent transfusion related reactions
(Ref: Harrison 20th/813)
The given image in the question is that of a leukocyte filter which has been
shown to reduce the risk of a variety of adverse reactions, including
alloimmunization, febrile non-hemolytic transfusion reactions,
immunosuppression and transmission of infections like CMV.
7. Glanzmann thrombasthenia is due to defect in:
(NEET 2019 like pattern)
(a) Gp IIb/IIIa
(b) Gp Ib-IX
(c) CD68
(d) Von Willebrand factor
Ans. (a) Gp IIb/IIIa Ref: Robbins 9th e/p 118
Inherited deficiency of Gp IIb-IIIa results in a bleeding disorder called
Glanzmann thrombasthenia.
8. What is the treatment of Choice for Idiopathic Thrombocytopenic
Purpura? (NEET 2019 like pattern)
(a) Blood transfusion
(b) Steroids
(c) IV immunoglobin
(d) Splenectomy
Ans. (b) Steroids
(Ref: Robbins 9th e/p 658, CMDT 2019/559-560)
The mainstay of initial treatment of new-onset primary ITP is a short course of
corticosteroids with or without intravenous immunoglobulin (IVIG) or
anti-D.
9. Which of the following tubes contain Sodium fluoride as
anticoagulant?
(AIIMS Nov 2018 like pattern)

Ans. (c) Gray colored tube


This is likely to be repeated friends. As I had discussed the same in my
Pathology Mobile ApplicationHematology Video, I reproduce the following
list of color coding for the tubes commonly used:
Cap Color Specification
Green Heparin sodium
Light Green Heparin lithium
Blue Sodium citrate 1:9
Black Sodium citrate 1:4
Red No additive
Yellow Coagulant separating gel
Orange Coagulant
Grey Potassium oxalate monohydrate and
sodium fluoride
Purple EDTA anticoagulant tube
Purple EDTAK2, EDTAK3
Color of Tube Additive
Sodium polyethanol sulfonate

Trisodium citrate

No additive

Heparin

EDTA

Sodium fluoride

10. A trauma patient presents at emergency department. There is no time


for cross matching. FFP of which group can be transfused safely:
(AIIMS Nov 2018 like pattern)
(a) O Rh D +
(b) O Rh D -
(c) AB Rh D +
(d) AB Rh D –
Ans. (c) AB Rh D + (Ref: Essential of Hematology 2nd e/p 499)
Read the question carefully friends,
In case of emergency blood transfusion
• If the patient’s blood group cannot be determined, then he should be transfused with O
group red cells which are Rh-negative with colloids/crystalloids.
In case of emergency plasma transfusion
• If the patient’s blood group cannot be determined, then he should be transfused with AB
Rh D +. The reason attributed is that since group AB plasma has no ABO antibodies, it
has been the standard choice when a recipient’s blood type is not known.

11. Patient who is known case of thalassemia major already on repeated


blood transfusions with history of iron overload previously treated
with chelating agents. She also has a history of cardiac arrhythmia.
She came for blood transfusion now. During blood transfusion
patient complained of backache and looks extremely anxious. What
is next management?
(AIIMS Nov 2018 like pattern)
(a) Observe for a change in colour of the urine
(b) Continue blood transfusion, do ECG
(c) Stop blood transfusion and wait for patient to get normal and start
(d) Stop blood transfusion and do clerical check
Ans. (d) Stop blood transfusion and do clerical check
(Ref: Harrison 20th e/p 812)
During blood transfusion, complaint of backache is suggestive of a mismatched
blood transfusion reaction. When acute hemolysis is suspected, the
transfusion must be stopped immediately, intravenous access
maintained, and the reaction reported to the blood bank. A correctly
labeled post transfusion blood sample and any untransfused blood should
be sent to the blood bank for analysis.
Errors at the patient’s bedside, such as mislabeling the sample or
transfusing the wrong patient, are responsible for the majority of these
reactions. The blood bank investigation of these reactions includes
• Examination of the pre- and post transfusion samples for hemolysis and
repeat typing of the patient samples
• Direct antiglobulin test (DAT), also known as direct Coombs test, of the
post transfusion sample
• Repeating the cross-matching of the blood component, and
• Checking all clerical records for errors.
12. Which of the following is true regarding blood transfusion of packed
RBC?
(AIIMS May 2018 like pattern)
(a) Should be started within 4 hours of receiving it from blood bank
(b) Should be completed within 4 hours of receiving from blood bank
(c) Wait till the patient is stable then transfuse, irrespective of any
timing.
(d) Should be completed within 6 hours of receiving from blood bank.
Ans. (b) Should be completed within 4 hours of receiving from blood
bank (Ref: Wintrobes 13th e/p 565)
Blood transfusion of packed RBC should be started within 30 minutes of
receiving it from blood bank and should be completed within 4 hours.
13. Storage temperature of RBC, Platelet, and Fresh Frozen Plasma
(FFP) are: (AIIMS May 2018 like pattern)
(a) RBC 2-6°C, Platelet 20-22°C, FFP -30°C
(b) RBC -30°C, FFP 2-6°C, Platelet 20-22°C
(c) RBC 20-22°C, Platelet 2-6°C, FFP -30°C
(d) RBC 20-22°C, FFP -30°C, Platelet 2-6°C
Ans. (a) RBC 2-6 °C, Platelet 20-22 °C, FFP-30 °C
(Ref: Essentials of Hematology 2nd e/p 500)
The storage temperature of different blood components are:
• RBC 2-6°C
• Platelet 20-22°C
• FFP -30°C
14. Which of the following is not true regarding von Willebrand
disease?
(AI 2018 Pattern)
(a) Normal platelet count
(b) Quantitative defects are seen in subtypes 1 and 3 von Willebrand
disease
(c) Hemarthrosis is the usual presentation
(d) Produced by endothelial cells
Ans. (c) Hemarthrosis is the usual presentation
(Ref: Robbins 9/e p662)

15. A voluntary donorunderwent apheresis for platelet donation for the


first time after which he developed perioral tingling and numbness.
This is seen because

(AIIMS Nov 2017 Pattern)


(a) His platelet count was low for donation
(b) He underwent apheresis for the first time
(c) Due to fluid depletion
(d) Due to citrate based anticoagulant
Ans. (d) Due to citrate based anticoagulant
(Ref: Wintrobe’s hematology 13/e p674-5)
Apheresismeans to separate or to take away. There are now continuous-flow
devices in which incoming blood is continuously subjected to a centrifugal
force. A standing cell gradient is established. The fraction(s) to be
removed are pumped into a bag, and the rest is reinfused continuously.
An increasing proportion of blood components is being collected using
automated cell separation.

Apheresis donors face the additional potential complication of transient


hypocalcemiafrom the citrate infused when anticoagulated blood
components are returned to them from the apheresis device.
Symptoms consist of tingling or muscle cramps. Citrate symptoms are
treated by slowing the flow of the device or giving the donor oral
calcium supplements.

16. Which of the following anticoagulant preservative can be used to


store blood, so that it can be kept for 35 days?
(AIIMS Nov 2017 Pattern)
(a) Acid citrate dextrose (ACD)
(b) CPD Citrate phosphate dextrose
(c) Citrate phosphate dextrose-adenine (CPD-A)
(d) CP2D – citrate phosphate double dextrose
Ans. (d) Citrate phosphate dextrose-adenine (CPD-A)
(Ref: Wintrobe’s hematology 13/e p674-5)

Preservative Shelf life

Acid citrate dextrose 21 days


Citrate phosphate dextrose 21 days
Citratephosphate double dextrose 21 days
Citrate phosphate dextrose-adenine 35 days

17. Which is the best anticoagulant to send sample for serum electrolyte
measurement?
(AIIMS Nov 2017 Pattern)
(a) EDTA
(b) Lithium heparin
(c) Sodium fluoride
(d) Citrate
Ans. (b) Lithium heparin
(Ref: WHO Guidelines)
WHO Guidelines for Use of anticoagulants in diagnostic laboratory
investigations
Lithium heparin can be used for the assessment of serum electrolytes.
18. In a platelet poor plasma sample calcium and tissue thromboplastin
is added. This is used to assess which of the following pathway?
(AIIMS Nov 2017 Pattern)
(a) Extrinsic
(b) Intrinsic
(c) Fibrinolytic
(d) Common
Ans. (a) Extrinsic
(Ref: Robbins 9/e p656)
For clotting pathway studies, a platelet poor plasma sample is taken.

• Prothrombin time (PT): This test assesses the extrinsic and common coagulation
pathways. The clotting of plasma after addition of an exogenous source of tissue
thromboplastin (e.g., brain extract) and Ca2+ ions is measured in seconds.
• Partial thromboplastin time (PTT). This test assesses the intrinsic and common
clotting pathways. The clotting of plasma after addition of kaolin, cephalin, and
Ca2+ ions is measured in seconds.

Disclaimer
Any resemblance to an actual question is purely coincidental.
• Concentric hypertrophy is seen in conditions with pressure
overload like hypertension and aortic stenosis.
• Eccentric hypertrophy is seen in conditions with volume
overload like aortic regurgitation.
• Heart failure cells are hemosiderin laden alveolar
macrophages seen in the lungs.
• Commonest cause of right heart failure is Left heart failure.
• Critical narrowing of coronary vessels to cause angina
>75%.
• Most common coronary artery involved in atherosclerosis
and MI: Left anterior descending (LAD) artery and so, most
common type of MI is anterior (antero-lateral) wall MI. of left
ventricle, anterior 2/3rd of ventricular septum.
• Inferior (posterior) wall MI is caused by occlusion of
Posterior inter-ventricular artery.
• Earliest light microscopy change in MI: Waviness of
fibres.
• Rheumatic fever is due to molecular mimicry (Cross
reactivity of streptococcal antigen to endogenous human
antigen). Its characteristic pathological finding is Aschoff
bodies.
• Infective endocarditis vegetations: Large, bulky, friable, non-
sterile, on upper surface of cusps, less commonly on mural
endocardium.
• Libman Sack enodocarditis vegetation: Small/medium, flat,
verrucous, sterile, affects both surfaces of valve.
• Vegetations of NBTE: Small, friable, sterile, along the line of
closure of valves.
• Rheumatic fever vegetation: Small, firm, sterile, along the
line of closure of valves.
• True about hypertrophic obstructive cardiomyopathy:
Myocardial hypertrophy without ventricular dilatation,
Asymmetrical septal hypertrophy, outflow obstruction and
dilatation of atria.
• Carcinoid heart disease is characterized by fibrous
endocardial thickening of right ventricle and tricuspid
valve.
• Tigered effect in myocardium is due to fat deposition.
• Cardiac polyp is a post-mortem fibrinous clot in heart.
• Mitral valve prolapse microscopically shows Myxomatous
degeneration.
• Weibel-Palade bodies of endothelial cells have von
Wlllebrand factor and P selectin.
• Neointimal hyperplasia in vascular graft is due to
hypertrophy of smooth muscles.
• Medial calcification is seen in :Monckebergs sclerosis.
• Important features of atheromatous plaque: thick or thin
fibrous cap, macrophages, smooth muscle cells (undergo
apoptosis in later stages), from cells cell debris.
• Common sites of atherosclerosis (in decreasing order):
Abdominal aorta > coronary arteries > popliteal artery >
internal carotid artery.
• Vessels spared in atherosclerosis include vessels of upper
extremities, mesenteric and renal vessels (except at their
ostia).
• Tree bark calcification is seen in : Syphilitic aneurysm.
• Most common cause of aortic dissection : Hypertension.
• Vascular changes in benign hypertension : Hyaline
arteriosclerosis.
• Vascular changes in malignant hypertension : Hyperplastic
arteriosclerosis, onion skin appearance and fibrinoid
necrosis (necrotizing arteriolitis).
• Hyperplastic arteriosclerosis affects : Kidney (necrotizing
glomerulonephritis), small intestine, gall bladder
peripancreatic fat, periadrenal fat.
• cANCA is produced Proteinase-3 and is seen in Wegner’s
granulomatosis.
• pANCA is produced against Myeloperoxidase (MPO) and is
seen in Microscopic polyangitis, Churg-strauss syndrome,
idiopathic crescentic glomerulonephritis, Good-pasture
syndrome, renal-limited vasculitis.
• Conditions with granulomatous vasculitis: Giant cell
arteritis, Takayasu’s disease, Wegner’s granulomatosis and
Churg Strauss syndrome.
• Characteristic tetrad of Henoch Schonlein purpura: Palpable
purpura, arthritis/arthralgia, glomerulonephritis and
abdominal pain.
• There is no structural abnormality or change in vessel
wall in Raynaud’s disease.
• Most common benign vascular tumor : Hemangioma.
• Vascular tumor with spontaneous regression is a Strawberry
angioma (a type of capillary hemanioma).
• Kaposi sarcoma is caused HHV 8 infection and arises from
vessels.
• Most common cause of SVC syndrome: Extrinsic
compression by malignant tumors.

HEART

The human heart is a muscular pump responsible for maintaining the


circulation of the blood and perfusion of different organs of the body.
The thickness of the left ventricle is almost three times that of the right
ventricle. The cardiac output is about 5 liters per minute. The heart is
supplied by the right and the left coronary artery (through left anterior
descending artery; LAD and the left circumflex artery; LCX).

Out of the three layers of the heart (pericardium, myocardium


and endocardium), the least collateral perfusion is present in the
endocardium and so, it is most prone to ischemic injury The coronary
artery supplying the posterior 1/3rd of the ventricular septum (by giving
rise to the posterior descending branch) is called dominant.
Therefore, most people have right dominant circulation. Since,
cardiac myocytes cannot undergo division, so, they only undergo
hypertrophy which can be of the following types:
• Concentric hypertrophy/pressure overload hypertrophy: It is due to
deposition of the sarcomeres in parallel to the long axis of the cells. It is
associated with hypertension and aortic stenosis.
• Volume overload hypertrophy: In this, dilatation with increased ventricular
diameter is present. It is seen with valvular regurgitation (mitral or aortic
regurgitation), thyrotoxicosis and severe anemia.

HEART FAILURE

It is a clinical condition characterized by the inability of the heart to


pump blood in proportion with the requirements of the metabolic
tissues of the body or being able to do so at increased filling
pressures. It may be divided into systolic or diastolic failure. We
can also classify heart failure as:

LEFT VENTRICULAR FAILURE/LEFT SIDED HEART FAILURE


(LVF)
It is most commonly caused by ischemic heart disease, hypertension,
aortic or mitral valvular disease and myocardial disease (non-
ischemic). The features include hypertrophy and fibrosis in the
myocardium associated with secondary involvement of the atria which
shows enlargement. Atrial involvement results in the development of
the atrial fibrillation which is responsible for thrombus formation or
embolic stroke. The other organs affected include:
Lungs

• It is the most commonly affected organ. The increased pressure in the


pulmonary vein causes pulmonary edema (heavy wet lungs). The edema fluid
accumulates in the alveolar space.
• There is leakage of hemosiderin and other iron containing particles which are
phagocytosed by macrophages. The iron gets converted to hemosiderin
leading to the formation of siderophages or heart failure cells (hemosiderin
containing macrophages).
• The alveolar fluid impairs gaseous exchange giving rise to breathlessness or
dyspnea (earliest feature of LVF), orthopnea (dyspnea on lying down) and
paroxysmal nocturnal dyspnea (dyspnea at night).
• There is presence of Kerley B lines on chest X-ray due to transudate in the
interlobular septa.

Kidneys

In the early stages, decreased renal perfusion causes activation of the renin-
angiotensin-aldosterone system whereas in the later stages, continued reduced
renal perfusion may precipitate prerenal azotemia.

Brain

It may suffer from hypoxic ischemic encephalopathy.

RIGHT-SIDED HEART FAILURE


Left ventricular failure is the most common cause of the right sided
heart failure whereas pure right sided failure is seen in chronic severe
pulmonary hypertension and is called as cor pulmonale.
The features seen as a result of the inability of the right heart to
pump blood in different organs include:
Liver
There is presence of congestive hepatomegaly. In addition, if associated LVF is
present, the centrilobular necrosis is also seen which is replaced by fibrotic tissue
in longstanding cases and is known as cardiac sclerosis or cardiac cirrhosis.
Spleen
Congestive splenomegaly is seen.
Kidney
These show the congestion resulting in severe azotemia. The congestion is more
prominent in RVF than in LVF.

• Pleural effusion, pericardial effusion and ascites are also seen.


• The hallmark of the right sided heart failure is peripheral edema of
the dependent parts of the body particularly pedal and pretibial
edema.
• Anasarca is the generalized massive edema seen in heart
failure.

RHEUMATIC FEVER AND RHEUMATIC HEART DISEASE (RHD)

Rheumatic fever is an acute immunologically mediated multisystem


inflammatory disease that occurs few weeks after an attack of group
A b- hemolytic streptococcal pharyngitis. It is not an infective disease.
The most commonly affected age group is children between the ages
of 5-15 yearsQ. Only 3%Q of patients with group A streptococcal
pharyngitis develop acute rheumatic fever.

The disease is a type II hypersensitivity reaction in which


antibodies against ‘M’ protein of some streptococcal strains (1, 3, 5, 6,
and 18) cross-react with the glycoprotein antigens in the heart, joints
and other tissues (molecular mimicry).

WHO criteria for diagnosis of RF and RHD [Based on revised


(1992) Jones criteria]
1. Major manifestations:
J
Joint involvement (Polyarthritis)
O
N - Nodules (Subcutaneous)
E - Erythema marginatum
S - Sydenham’s chorea
Criteria - Carditis
2. Minor manifestations:
Clinical: Fever polyarthralgia
Laboratory: Increased ESR or C-RP
ECG: Prolonged PR interval
3. Supporting evidence of a preceding streptococcal infection within last
45 days:

Elevated or rising ASO or other Ab titers

Positive throat culture

Rapid antigen test for group A streptococcus.


Note:
1. Two major or one major and two minor manifestations plus any of the
evidence of preceding group A streptococcal infection is required for
diagnosis of primary episode of rheumatic fever.
2. 1992 revised Jones criteria do not include elevated TLC (total
leukcocyte count) as a laboratory minor manifestation [instead, it
includes elevated C-reactive protein] and do not include recent scarlet
fever as supporting evidence of recent streptococcal infection.

Fig. 1: Aschoff Body (A) in Rheumatic Myocarditis. ...(All India


Image)

SALIENT FEATURES OF THE MAJOR CRITERIA


Carditis
All the layers of the heart namely pericardium, myocardium and endocardium are
involved, so this is called pancarditis. The pericarditis is associated with
fibrinous/serofibrinous exudate and is called as ‘bread and butter’ pericarditis.
Valvular involvement is common in rheumatic heart disease. The most common
valve to be affected is the mitral valve and least commonly affected is pulmonary
valve. In acute rheumatic heart disease, the most common valvular lesion is
mitral regurgitation and in chronic rheumatic heart disease, it is mitral
stenosis.
Migratory polyarthritis
There is involvement of the large joints of the body. It is more commonly seen in
the adults as compared to children. The arthritis involves one joint after the other
(migratory) and subsides spontaneously without any residual deformability in the
joints (non-erosive arthritis). Clinically, this is the most commonly seen
manifestation and the joint pain shows dramatic response to salicylates like
aspirin.
Subcutaneous nodules
These are painless subcutaneous lesions found on the extensor surface of the
elbows, shin and the occiput.
Erythema marginatum
There is presence of red macular rash more easily appreciated in fair skinned
individuals sparing the face and without residual scarring.
Sydenham’s chorea
It is a late manifestation of the disease characterized by presence of involuntary,
purposeless movements associated with emotional lability of the patient.

Microscopically, the characteristic feature of rheumatic heart


disease is Aschoff’s body (Fig. 1). The latter consist of foci of
swollen eosinophilic collagen surrounded by T-lymphocytes, few
plasma cells and plump macrophages called Anitschkow cells
(pathognomonic for RF). These distinctive cells have abundant
cytoplasm and central round-to-ovoid nuclei in which the chromatin is
disposed in a central, slender, wavy ribbon (hence, they are also
called as “caterpillar cells”).
The myocardium has Aschoff’s bodies in the perivascular
location. The involvement of the endocardium results in fibrinoid
necrosis within the cusps or along the tendinous cords which also
have small vegetations called verrucae present along the lines of
closure. The presence of mitral regurgitation also induces irregular
thickening in the left atrial wall called as MacCallum plaques.
Chronic RHD is characterized by organization of the acute
inflammation and subsequent fibrosis. The valves show leaflet
thickening, commissural fusion and shortening, and thickening and
fusion of the tendinous cords. There is mitral stenosis called as ‘fish-
mouth’ or ‘button-hole’ stenosis. Mitral stenosis may also lead to
atrial fibrillation and thromboembolic phenomenon in these patients.

INFECTIVE ENDOCARDITIS (IE)

It is colonization or invasion of heart valves and mural endocardium


by microbiologic agent leading to formation of bulky, friable
vegetations composed of thrombotic debris and organisms with
destruction of underlying cardiac tissues. It can be
• Acute
• Subacute
Acute endocarditis Subacute endocarditis
• Necrotizing, ulcerative, invasive • Insidious infection following a
valvular infection on a previously protracted course on a previously
normal valve. damaged valve
• Highly virulent organisms • Low virulence organisms
• Death of the patient within days to • Recover after antibiotic therapy
weeks • MC caused by a-hemolytic
• MC caused by Staph. aureus (viridans) Streptococcus

MORPHOLOGY
The friable, bulky destructive vegetations containing fibrin, bacteria
and inflammatory cells are found on the valve cusps. These can also
extend on to chordae. The aortic valve and the mitral valve are
most commonly infected whereas the right side of heart is affected
in intravenous drug abusers. When the vegetations erode into
myocardium, they can form an abscess called Ring abscess. The
systemic embolisation can result in septic infarcts.

CLINICAL FEATURES
Fever is the most consistent sign of IE. The other features include
weight loss, flu-like syndrome, cardiac murmur, systemic emboli,
Roth spots (due to retinal emboli), Osler nodes (painful,
subcutaneous nodules on the fingers and toes) and Janeway lesions
(red painless lesions on the palms and soles).

• The disease is diagnosed by Dukes criteria.

COMPLICATIONS
Cardiac complications
• Valvular insufficiency or stenosis
• Myocardial ring abscess
• Suppurative pericarditis
• Valvular dehiscence
Embolic complications
• With left sided lesion – Brain, spleen, kidney
• With right sided lesion – Lung infarct, lung abscess
Renal complications
• Embolic infarct
• Focal (more common) or diffuse glomerulonephritis (less common)

MARANTIC ENDOCARDITIS/NON BACTERIAL THROMBOTIC


NON BACTERIAL THROMBOTIC ENDOCARDITIS (NBTE)

• It is seen in patients suffering from debilitating diseases like


malignancy (carcinoma pancreas and acute promyelocytic
leukemia) and hypercoagulable states (DIC)
• Vegetations on heart valves are sterile (do not contain
microorganisms). These are usually present along the line of
closure, single or multiple.
LIBMAN-SACKS ENDOCARDITIS (SLE)

• Seen in patients of SLE


• Vegetations are small or medium sized, sterile, granular and
pink on either or both sides of valve leaflets.
• Mitral and tricuspid valves are involved and show fibrinoid
necrosis.
Table 1: Summary of salient features of vegetations in different
endocarditis
Non Bacterial
Thrombotic
Rheumatic (Marantic Libman-Sacks Infective
Fever Endocarditis) Endocarditis Endocarditis

• Small, warty • Small, warty • Medium sized • Large


• Firm • Friable (small) • Bulky
• Friable • Flat, Verrucous • Irregular
• Irregular

• Along lines • Along lines of • On surface of • Vegetations on


of closure closure cusps the valve cusps
• (both surfaces • Less often on
may be involved mural
but the endocardium
undersurface is
more likely
affected, less
commonly mural
endocardium is
involved
• In pockets of
valves

• Sterile (no • Sterile • Sterile • Non-sterile


organism) (bacteria)

• Embolisation • Embolisation is • Embolisation is • Embolisation is


is uncommon common uncommon very common
(max chances)

• In rheumatic • In cancers (like • In SLE • In infective


heart M3-AML, endocarditis
disease pancreatic
cancer), deep
vein
thrombosis,
Trosseau
syndrome

Valvular lesions MC cause


Aortic stenosis Congenitally bicuspid aortic valves
Aortic regurgitation Hypertension
Mitral stenosis RHD
Mitral regurgitation Mitral valve prolapse

BARLOW SYNDROME (MITRAL VALVE PROLAPSE; MVP)

• Valvular abnormality seen predominantly in females where one


or both mitral leaflets are “floppy” and prolapse, or balloon back
into the left atrium during systole. This gives rise to the mid
systolic click.
• Most of the patients are usually asymptomaticQ
• The condition is discovered only on routine examinationQ by the
presence of a midsystolic click Q as an incidental finding on
physical examination.
• Some patients may present with chest pain mimicking angina,
dyspnea, and fatigue or, psychiatric manifestations, such as
depression, anxiety reactions, and personality disorders
• The risk of complications like Infective endocarditis, Mitral
insufficiency, Stroke or other systemic infarct and Arrhythmias is
higher in men, older patients, and those with either arrhythmias
or some mitral regurgitation, as evidenced by holosystolic
murmurs and left-sided chamber enlargement.
ISCHEMIC HEART DISEASE

Ischemia of the heart is a result of imbalance between the perfusion


and demand of the heart for oxygenated blood. Atherosclerotic
narrowing resulting in coronary arterial obstruction is the cause of
ischemic heart disease in almost 90% of the patients.

Stable Angina

Stable angina occurs when the myocardial oxygen demand is more


than the supply. It takes place when the coronary artery is occluded
>75%. Stable angina is characterized by pain on exertion which is
relieved on taking rest or taking vasodilators like nitrates. There is
neither any plaque disruption nor any plaque associated thrombus.
Prinzmetal or Variant or Vasospastic Angina
It is an episodic angina due to coronary artery spasm resulting in
pain at rest. It is characterized by ST segment elevation on the ECG
(due to transmural ischemia).

Unstable or Crescendo Angina

It is induced by atherosclerotic plaque disruption with superimposed


partial thrombosis or vasospasm or both of them. The pain occurs
with increasing frequency and for a longer duration and is
characteristically precipitated by progressively less exertion.

Myocardial Infarction (MI)

Myocardial infarction
Subendocardial MI Transmural MI

• Ischemic necrosis limited to 1/3rd • Ischemic necrosis involves full


of ventricular wall thickness of ventricular wall
• Caused by incomplete coronary • Caused by severe coronary
artery occlusion. atherosclerosis, with acute plaque
rupture and superimoposed occlusive
thrombus.

Pathogenesis of MI
Changes in atheromatous plaque (hemorrhage/ulceration/rupture)

Exposure of underlying collagen and platelet aggregation

Platelets release mediators which cause vasospasm

Activation of extrinsic clotting pathway and increased thrombus formation

Complete occulsion of coronary vessel by thrombus

Fig. 2: Coagulative necrosis with eosinophilic cytoplasm*.


Fig. 3: Day 2 Neutrophilic Infiltration.
Fig. 4: Macrophage (M) and Granulation Tissue.
Fig. 5: Fibrous tissue (F) replaces Normal cardiac tissue.

Myocardial Response

Feature Time
Cessation of aerobic respiration or onset of ATP depletion Seconds
Loss of contractility <2 min
ATP reduced to 50% of normal 10 min
ATP reduced to 10% of normal 40 min
Irreversible cell injury 20-40 min
Microvascular injury >1 hr

Table 2: Evolution of Morphological Changes in MI


Time Gross Light Microscopy
Reversible injury
0-30 min. None None
Time Gross Light Microscopy
Irreversible injury
30 min to 4 None Waviness of fibers at border
hr. (earliest change)
4-12 hr. Occasional dark mottling Beginning of coagulative necrosis,
edema and hemorrhage
12-24 hr. Dark mottling Ongoing coagulative necrosis,
marginal contraction band necrosis,
beginning of neutrophilic infiltration
1-3 days Mottling with yellow tan Coagulation necrosis, interstitial
infarct center. neutrophilic infiltrate

3-7 days Hyperemic borders, central Beginning of disintegration with dying


yellow tan softening neutrophils, early phagocytosis by
macrophages

7-10 days Maximum yellow tan and Early formation of fibrovascular


soft depressed red-tan granulation tissue at margins
margin

10-14 days Red gray depressed infarct Well established granulation tissue
borders and collagen deposition

2-8 weeks Gray-white scar Collagen deposition, ↓ Cellularity


progressive from border
towards infarct core

> 2 months Scarring complete Dense collagenous scar

Diagnosis of MI

MI should be suspected in any patient developing severe chest pain,


rapid weak pulse, sweating, dyspnea and edema. Infact, rapid pulse
is the first sign and dyspnea is the first symptom of acute MI. The
ECG shows the ST segment elevation in acute MI whereas ‘Q’
wave indicates old MI.
Laboratory investigations show nonspecific markers like
increased ESR, leukcocytosis and elevated C-reactive protein. The
specific markers include:

Enzyme Initiation of rise Peak Return to baseline


CK-MB 2-4 hours 24 hours 48-72 hours

Troponin T and I 2-4 hours 48 hours 7-10 days


(TnT, TnI)

AST/SGOT In 12 hours 48 hours 4-5 days

LDH 24 hours 3-6 days 2 weeks

IMPORTANT POINTS ABOUT THE CARDIAC ENZYMES

Troponin T and Troponin I

These are the proteins that mediate calcium mediated contraction of the cardiac
and the skeletal muscles. They are very specific for MI. Troponin I is more
important than troponin T (remember, I for Important). If the patient has another
MI (due to reinfarction within 1 week), these enzymes cannot be used for
diagnosis of reinfarction because their levels remain elevated for a long time from
the first attack. In that condition, we prefer an enzyme elevated for a short
duration. This is the enzyme of choice for diagnosing reinfarction.

Creatine kinase (CK)

It is an alternative to troponin measurement. It has got 3 isoforms:


• CK-MM—Present in the skeletal muscle and heart
• CK-MB—Present in the myocardium and a small amount in skeletal muscle
• CK-BB—Present in the brain, lung and other tissues.
Elevation of the CK-MB isoforms is seen in MI. Any absence of elevation of CK-
MB in the first-two days excludes the diagnosis of MI.

Myoglobin

It is a small monomer with a rapid rise and fall in serum (has a narrow window). It
is the earliest enzyme to increase after MI.
LDH

Normally, serum LDH2 is greater than LDH1 but in MI, LDH1 is more than LDH2.
This is called “flipping of LDH ratio”.
Fig. 6: Reperfusion Injury with Contraction Band Necrosis.

COMPLICATIONS OF MI
• Contractile dysfunction resulting in cardiogenic shock.
• Arrhythmia: Ventricular fibrillation is the most common arrhythmia within one
hour whereas supraventricular tachycardia is the most common arrhythmia
after one hour of MI.
• Cardiac rupture syndrome: Rupture of ventricular free wall is the most
common cardiac rupture syndrome. It results in cardiac tamponade. The
anterolateral wall at the midventricular level is the most common site for
postinfarction free wall rupture. It is most frequent 3 to 7 days after MI. The
rupture of ventricular septum leads to formation of left to right shunt. The
rupture of papillary muscles can cause mitral regurgitation.
• Pericarditis: It is the epicardial manifestation of the underlying myocardial
injury and is also known as Dressler syndrome or post MI syndrome. It is an
autoimmune reaction, which takes place around 2-3 weeks after a transmural
MI. though it has been reported to occur even after 48 hrs. It is associated
with pleural effusion, pleuritic chest pain and pericardial effusion.
• Right ventricular infarction.
• Ventricular aneurysm: This may contribute to thromboembolism also
• Papillary muscle dysfunction: This leads to post infarct mitral regurgitation.

CARDIAC TUMORS

Myxoma

Myxomas are the most common primary tumor of the heart in adults.
Though they may arise in any cavity of the heart but nearly 90% are
located in the atria, with a left-to-right ratio of approximately 4:1 (atrial
myxomas). The major clinical manifestations are due to valvular “ball-
valve” obstruction, embolization, or a syndrome of constitutional
symptoms, such as fever and malaise the latter most commonly due
to the effect of interleukin-6.
The tumors are almost always single. The region of the fossa
ovalis in the atrial septum is the favored site of origin.
Histologically, myxomas are composed of stellate or globular myxoma
(“lepidic”) cells, endothelial cells, smooth muscle cells, and
undifferentiated cells embedded within an abundant acid
mucopolysaccharide ground substance and covered on the surface
by endothelium.
Rhabdomyoma

Rhabdomyomas are the most frequent primary benign tumor of the


heart in infants and children. They are actually hamartomas or
malformations rather than true neoplasms. Cardiac rhabdomyoma is
associated with tuberous sclerosis due to defect in the TSCI or
TSC2 tumor suppressor gene. The TSC proteins stimulate the cell
growth and are involved in myocyte overgrowth.

Vessel Property
Arteriole Resistance vessels
Capillaries maximum cross-sectional surface area
Venules Most important vessel in inflammation
Vein Maximum blood volume
Rhabdomyomas are generally small, gray-white myocardial
masses protruding into the ventricular chambers. Histologically they
are composed of large, rounded, or polygonal cells containing
numerous glycogen-laden vacuoles separated by strands of
cytoplasm running from the plasma membrane to the more or less
centrally located nucleus, the so-called spider cells.

BLOOD VESSELS

The blood vessels are responsible for the transport of blood in the
circulation from the heart to the various organs and back to the heart.
The histological layers which are seen in a blood vessel
(particularly arteries) are:
1. Tunica intima (Innermost layer)
2. Internal elastic lamina
3. Tunica media (Middle layer)
4. External elastic lamina
5. Tunica adventitia (Outermost layer)
The outer half of the tunica media and the whole of tunica
adventitia are supplied by vasa vasorum whereas the other inner
layers of the blood vessel get their nourishment by diffusion.
Any injury/denudation of endothelial cells stimulate thrombosis
and smooth muscle cell proliferation. ‘Sclerosis” means loss of
elasticity of vessels commonly associated with thickening. It may be
of the following types:

1. Arteriolosclerosis - It affects small arteries and arterioles, it can


be of the following types:
Hyaline arteriolosclerosis

• Pink, hyaline thickening of arteriolar walls.


• Seen in elderly, more commonly in benign hypertension, diabetes mellitus
(DM) and benign nephrosclerosis.
Hyperplastic arteriolosclerosis

• ‘Onion skinning’ or concentric thickening of the arteriolar wall seen in


malignant hypertension.
• Fibrinoid necrosis/necrotizing arteriolitis (inflammatory cells in vessel wall
particularly in kidney)

2. Monckeberg’s medial calcific stenosis


– Seen is muscular arteries of people > 50 years of age.
– Associated with dystrophic calcificationQ and is
asymptomatic.
3. Atherosclerosis
It is characterized by deposition of atheroma/fibrofatty plaque
consisting of raised focal lesion. Plaque is present within the
intima, has a core of lipid (cholesterol and cholesterol esters)
and a covering of fibrous cap.
The histopathology shows:
Atherosclerosis: risk factors

A- Age (↑ with age)

T- Type ‘A’ personality

H- Hyperhomocysteinemia

E- Extra lipids (Hyperlipidemia); Extra BP (hypertension); Extra sugar


(DM)

R- Reduced physical activity

O- Obesity

S- Sex (males >> females)

C- CMV; Chlamydia infection; Cigarette smoking

L- Lipoprotein ‘A’; Lp(a)

1. Fibrous cap - Consists of smooth muscle cells, macrophages and foam cells.
2. ‘Shoulder’ - Cellular area around cap having macrophages, smooth muscle
cells and T lymphocytes.
3. Necrotic core - Debris of dead cells, foam cells and cholesterol clefts.
Pathogenesis

It is best explained by the ‘Response to Injury Hypothesis’Q

According to this hypothesis, chronic endothelial injury results in


increased permeability, leukocyte adhesion and thrombotic potential.
This is associated with accumulation of lipoproteins (mainly LDL)Q
followed by oxidation of lipoproteins in the vessel wall. The blood
monocytes initially adhere to the endothelium followed by their
transformation into macrophages and foam cells inside the intima
along with adhesion of platelets. The activated platelets release
factors causing migration of smooth muscle cells from the media to
intima and their proliferation along with release of proteoglycans and
collagen. This results in enhanced accumulation of lipids. In advanced
atheroma, the smooth muscle cells may undergo apoptosis and so,
smooth muscle cell paucity may be observed.
Foam cells are formed because oxidized LDL is ingested by the
scavenger receptors present on the macrophages and smooth
muscle cells both intracellularly as well as extracellularly.

Natural History of Atherosclerosis


American Heart Association classification of human atherosclerosis:
Type Gross Microscopy
Clinically silent
Type I Fatty dot (initial lesion) Isolated macrophage; foam cell
Type II Fatty streak Intracellular lipid accumulation
Type III Intermediate lesion Type II change + small extracellular
lipid pool
Clinically silent or overt
Type IV Atheroma lesion Type II + core of extracellular lipids
Type V Fibroatheroma Lipid core and fibrotic layer
Type VI Complicated lesion Surface defect, Hemorrhage and
thrombus

FATTY STREAK
• It is the earliest lesion of atherosclerosis and is composed of lipid
filled foam cells. It begins as yellow flat spots less than 1 mm
which gradually progress to atheroma formation.

Significance of involved blood vessels in atherosclerosis


Abdominal Aorta - Most common site of atherosclerotic
aneurysm in body

Coronary Arteries - Left Anterior Descending is MC coronary


artery involvedQ

Poplitial Artery - MC peripheral vessel showing aneurysm


formationQ

Descending Thoracic Aorta

Internal carotid artery

Circle of Willis
ANEURYSM

A localized abnormal dilation of a blood vessel or the wall of the heart


is called aneurysm. It is of two types:
1. True aneurysm: Involves intact attenuated arterial wall or
thinned ventricular wall of the heart. The common causes include
Atherosclerosis, syphilis and post MI ventricular aneurysms.
2. False/Pseudo- aneurysm: It is characterised by a breach in the
vascular wall leading to extravascular hematoma communicating
with intravascular space. The two most common causes of
pseudoaneurysm are post MI rupture and leakage at the site of
vascular anastomosis.

Causes of True Aneurysm in Aorta


1. Atherosclerosis *It is the most common cause of true aneurysm in
aorta
*The most commonly affected vessel is the
abdominal aorta (below the origin of renal artery and
above bifurcation into common iliac artery).
2. Syphilis *The thoracic aorta is involved in tertiary stage of
syphilis
*Endarteritis of vasa vasorum results in patchy
ischemia of tunica media. This is responsible for the
often seen “tree barking” appearance of the
thoracic aorta.
*Aortic valve insufficiency can also occur which may
result in cardiac hypertrophy. The increase in the size
of heart is called as cor bovinum/cow heart.
3. Other causes Trauma; infection (mycotic aneurysm; mostly due to
Salmonella gastroenteritis) and systemic disease
(vasculitis)

Inherited Causes of Aneurysm


• Marfan’s syndrome: defective synthesis of the protein fibrillin.
• Ehlers-Danlos syndrome: defect in collagen type III
• Loeys Dietz syndrome: defect in elastin and collagen types I and III due to
mutation in TGF-β receptor.

• Marfan’s syndrome: defective synthesis of the protein fibrillin.


• Ehlers-Danlos syndrome: defect in collagen type III
• Loeys Dietz syndrome: defect in elastin and collagen types I and III due to
mutation in TGF-β receptor.
AORTIC DISSECTION

• It occurs when blood splays apart the laminar planes of the


media with the formation of blood-filled channel within the aortic
wall.
• It is mostly seen in men due to hypertension (age group of 40-60
years) or Marfan’s syndrome (young patients)
• Medial degeneration is a characteristic pre-existing lesion in most
of the patients.

Dissection is classified into two types:


1. Type A - Involves ascending aorta with/without descending aorta. It is more
common and is more dangerous.
2. Type B - Does not involve ascending aorta but lesion begins distal to
subclavian artery.

VASCULITIS

The inflammation of the vessel wall is called vasculitis. It may be


classified on the basis of pathogenesis or on the basis of size of the
involved vessel.
ANTI-NEUTROPHIL CYTOPLASMIC ANTIBODIES (ANCAs)

ANCAs are autoantibodies against enzymes inside the neutrophils c-


ANCA is formed against a neutrophil azurophilic granule constituent
proteinase 3 (PR3) whereas p-ANCA is formed against a lysosomal
granul constituent myeloperoxidase (MPO). These can be of the
following types:

ANCA

PR3 ANCA MPO ANCA

• Earlier called as C-ANCA • Earlier called as P-ANCA


• Antigen is proteinase 3 • Antigen is myeloperoxidase
• Cytoplasmic staining • Perinuclear staining
• Seen in conditions like • Seen in conditions like
– Wegener’s granulomatosis
– Microscopic polyangiitis
– Churg-Strauss polyangiitis
– Goodpastures syndrome
– Creascentic glomeruelonephritis
– Ulcerative colitis
– Primary sclerosing cholangitis
– Drugs (hydralazine, propythiouracil)

LARGE VESSEL VASCULITIS

1. Giant cell (Temporal) arteritis/Cranial arteritis


• It is the most common type of vasculitis in adultsQ (usually >50Q years)
• This vasculitis is characterized by granulomatous arteritis of the aorta
and its major branches particularly the extracranial branches of the
carotid artery. Since the superficial temporal arteryQ is the most
commonly involved vessel, the giant cell arteritis is called as temporal
arteritis.
• Clinical features include constitutional symptoms like fever, fatigue,
weight loss, jaw painQ facial pain, localized headacheQ (most intense
along the anatomical course of the superficial temporal artery) and
sudden onset of blindness (due to involvement of ophthalmic artery).
• Biopsy of temporal arteryQ is the investigation of choice.
• Microscopically, there is presence of granulomatous inflammation with
multinucleated giant cells and fragmentation of internal elastic lamina.
Fig. 7: Giant cell (temporal) arteritis.

2. Takayasu arteritis/Aortoarteritis /Aortic Arch syndrome


– It is seen in adult females < 50 years of age.
– This condition is characterized by granulomatous vasculitis
followed by thickening of the aortic arch and decreased
lumen of the vessels arising from the aortic arch. The
pulmonary, renal and coronary arteries may also be involved.
– Clinical features include weak pulses in the upper limbs (so,
the disease is also called as pulseless diseaseQ), ocular
disturbances, hypertension and neurological defects.
MEDIUM VESSEL VASCULITIS

1. Classic Polyarteritis Nodosa (PAN)


• It is a systemic vasculitis of medium sized muscular arteries (no
involvement of arterioles/capillaries/venulesQ).
• The most frequently involved vessels are those of the kidney and other
viscera vessels. The vessels of the pulmonary circulationQ are
typically NOT involved.
• Characteristic feature of this disease is sharp segmental lesions
showing transmural inflammation of vessel wall accompanied by
fibrinoid necrosis and existence of all stages of inflammation in the
same vessel.
• 30% patients have association with Hepatitis B antigenQ in their serum.
• No glomerulonephritisQ is seen.
• It is the commonest cause of mononeuritis multiplexQ.
Fig. 8: Transmural Inflammation with fibrinoid necrosis in Polyarteritis
Nodosa.
2. Kawasaki’s disease (Mucocutaneous Lymph Node
Syndrome)

It is the vasculitis affecting children < 5 yearsQ of age. It


is characterized by fever, conjunctivitis and oral
erythema, skin rash often with desquamation,
erythema of palms and soles and cervical
lymphadenopathy.
– For the diagnosis of Kawasaki disease, there must be
presence of feverQ (most important constitutional symtpom)
for greater than 5 days plus any 4 of the following:
Fig. 9: Kawasaki’s disease with coronary artery narrowing.
– It may present with myocardial infarctionQ in children. It is
having the presence of anti-endothelial cell antibodiesQ.
There is typically intimal proliferation and mononuclear
infiltration of vessel wall. The patients also have elevated
platelet count in this condition.

SMALL VESSEL VASCULITIS

1. Microscopic Polyarteritis/Microscopic
Polyangiitis/Leukocytoclastic Vasculitis
– Necrotizing vasculitis affecting arterioles/capillaries/venules
in which all lesions are of the same age.
– Granulomatous inflammation is absentQ
– Necrotizing glomerulonephritis and capillaritis are common.
– Fibrinoid necrosis associated with infiltration of neutrophils
which become fragmented (leukocytoclasia).

2. Henoch-Schönlein purpura (HSP)/Anaphylactoid purpuraQ


– It is the commonest vasculitis in childenQ.
– This is a vasculitis with IgAQ deposits affecting small vessels
like arterioles, capillaries and venules of the skin, gut and
glomeruli and commonly associated with arthralgia.
– Clinical features include palpable purpura (due to
vasculitis and not reduced platelet count)Q, colicky
abdominal painQ, arthralgia in multiple joints and
glomerulonephritis.
– It is caused due to immune complex deposition but
complement levels are usually normalQ.

3. Hypersensitivity vasculitis/Cutaneous vasculitis


• Defined as inflammation of the blood vessels of the dermis.
• Also called as hypersensitivity vasculitis/cutaneous
leukocytoclastic angiitis.
• Microscopic features include presence of vasculitis of small vessels
characterized by a leukocytoclasisQ (refers to the nuclear debris
remaining from the neutrophils that have infiltrated in and around the
vessels during the acute stages).
• Hallmark clinical feature is skin involvement typically appearing as
palpable purpuraQ appearing on most commonly lower limbs
• Diagnosis is best made with biopsy showing vasculitis.
• Removal of offending agent (if any) and steroids help most of the
patients.

4. Churg-Strauss syndrome (Allergic granulomatosis and


angiitis)
– Characteristically have necrotizing vasculitis accompanied
by granulomas with eosinophilic necrosis.
– p-ANCA present in 50% of patients.
–Strong association with allergic rhinitis, bronchial asthma and
eosinophilia.
– Principal cause of death includes coronary arteritis and
myocarditis.
5. Wegener’s granulomatosis (Granulomatosis with
polyangitis)

Necrotizing vasculitis which is characterized by triad of


1. Acute necrotizing granulomas of either upper (more commonly) or
lower respiratory tract or both.
2. Focal necrotizing or granulomatous vasculitis most commonly affecting
lungs and upper airways.
3. Renal involvement in the form of focal necrotizing, often crescentic
glomerulonephritis.
• Clinical features include fever, weight loss, otitis media, nasal septal
perforationQ, strawberry gumsQ, cough, hemoptyis, palpable purpuraQ,
joint pain and ocular features (uveitis, conjunctivitis)
• Investigations show serum c-ANCAQ positivity, cavitatory lesionsQ in
the chest X ray and red cell casts (indicative of glomerulonephritis) in
the urine.

RAYNAUD’S PHENOMENON

Raynaud’s disease or Primary Raynaud’s phenomenon is seen in


young females. It is characterized by intense vasospasm of small
vessels in the digits of hands and feet induced by cold and emotional
stimuli (so, pulses are NOT affected).
• Characteristic sequence of color change is

• Structural changes are absent except in later course when


thickening of intima is seen.
Secondary Raynaud’s phenomenon is associated with
conditions like systemic sclerosisQ (commonest cause), SLE,
atherosclerosis and Buerger’s disease. It usually affects people of
age >30 years. The index and middle fingers are more sensitive to
attacks.

Buerger’s disease (Thromboangiitis obliterans)


• It is usually seen among heavy cigarette smokers.
• Onset is before age 35
• It is associated with hypersensitivity to intradermal injections of tobacco
extracts.
• Microscopic examination demonstrates segmental thrombosing vasculitis
often extends into contiguous veins and nerves (a feature rarely seen in
other types of vasculitis), encasing them in fibrous tissue.
• Thrombus contains microabscess with granulomatous inflammation.Q
• This patient has distal lower extremity vascular insufficiency which may
present as Calf, foot or hand intermittent claudication, superficial nodular
phlebitis and cold sensitivity (Raynaud’s phenomenon). Severe distal pain
even at rest can result and may be due to neural involvement. Later
complications include ulcerations and gangrene of the toes feet or fingers.
• Treatment includes smoking cessation, drugs (peripheral vasodilators) and
may even require surgery.

VASCULAR TUMORS
Benign Tumors
Hemangioma
1. Capillary hemangioma
• It is the most common type of vascular tumor which occurs in skin,
mucus membrane and viscera.
• “Strawberry” type of capillary hemangioma (also called as juvenile
hemangioma) is very common, growing rapidly in the first few
monthsQ and regresses by age 7Q in newborns. The child is normalQ
at birth in almost 90% of cases.
• Histologically, they are lobulated unencapsulated aggregates of closely
packed, thin walled capillaries which are blood filled and lined by a
flattened endothelium.

2. Cavernous hemangioma
– It is less common than capillary hemangioma with same age
and anatomic distribution. It more frequently involves deep
structures as it shows no tendency to regress. So, it usually
requires surgery.
– Morphologically, Cavernous hemangiomas are made up of
large, cavernous vascular spaces in which intravascular
thrombosis and dystrophic calcification is common.
– They may be life-threatening as in von Hippel Lindau
disease where they occur in cerebellum, brainstem and the
eye.

3. Pyogenic granuloma
It is a polypoid form of capillary hemangioma seen attached by a stalk
to skin or oral mucosa. It is associated with edema and
inflammatory cells.
Granuloma gravidarum is present in the gingiva of pregnant
women and it regresses after delivery.

LYMPHANGIOMA

1. Cavernous lymphangioma (also called as cystic hygroma)


• This is present in the neck region of the children.
• Made up of dilated, cystic lymphatic spaces lined by endothelial cells.
• Lesions are non-encapsulated, so, removal is difficult.

2. Capillary lymphangioma
It is a lesion composed of small lymphatic channels occurring
subcutaneously in the head and neck region and in the axilla.

GLOMUS TUMOR (GLOMANGIOMA)


• Benign tumor arising from the smooth muscle cells of the glomus
body which is an arteriovenous anastomosis involved in
thermoregulation.
• Most commonly present in the distal portion of the digits (under
fingernails).
• Histologically, there is presence of branching vascular channels
and stroma containing nests/aggregates of glomus cells arranged
around vessels.

INTERMEDIATE/BORDERLINE TUMORS

Kaposi’s Sarcoma (KS)

It is caused by KS Herpes virus or Human herpes virus 8 (HHV8)


and has the following 4 forms:

Association Chief affected


Type of Kaposi sarcoma with HIV organs
Classic/Chronic/European KS Absent Skin plaques and
nodules
African/Endemic KS Absent No skin lesions;
lymphadenopathy
present
Transplant Absent Lymph nodes, mucosa
associated/Immunosuppression- and visceral organs
associated KS
Epidemic/AIDS associated KS PresentQ Lymph nodes and
viscera involved
Fig. 10: Kaposi Sarcoma with Spindle-Shaped Cells.
The morphology is characterized by three stages:
1. Patch stage
2. Plaque stage
3. Nodular stage – Often accompanied by involvement of lymph
nodes and of viscera particularly in African and AIDS-associated
disease.

MALIGNANT TUMORS

Angiosarcoma

• Malignant endothelial cell neoplasm most commonly seen in skin, soft tissue,
breast and liver.
• May also arise from dilated lymphatic vessels (lymphangiosarcoma).
• Endothelial cell origin is demonstrated by staining for CD31, CD34 or vWF.
Hemangiopericytoma

• Tumor derived from pericytes which are the cells present along
the capillaries and venules.
• These tumors most commonly arise from pelvic retroperitoneum or
the lower limbs (particularly thighs).
• Capillaries are arranged in ‘fish-hook pattern’ and silver stain is
used for diagnosing this condition.

Quick review of superior vena cava and inferior vena cava


syndromes
Syndromes Associated Cancers Clinical Features

Superior Vena • Bronchogenic cancerQ • Dilation of the veins of the


Cava (SVC) • Mediastinal head, neck and arms
syndrome lymphomaQ • Cyanosis
• Respiratory distress
Inferior Vena • Renal cell carcinomaQ • Lower limb edema
Cava (IVC) • Hepatocellular • Dilation of the superficial
syndrome carcinomaQ collateral veins of the lower
abdomen
• Massive proteinuria (if renal
vein is involved)

HEART FAILURE, RHEUMATIC HEART DISEASE,


ENDOCARDITIS

1. A young female patient came for routine examination. On


examination a mid systolic click was found. There is no
history of rheumatic heart disease. The histopathological
examination is most likely to show which of the following?
(AIIMS May 2012)
(a) Myxomatous degeneration and prolapse of the mitral valve
(b) Fibrinous deposition on the tip of papillary muscle
(c) Rupture of chordae tendinae
(d) Aschoff nodule on the mitral valve
2. Least chances of infective endocarditis is associated with
(AI India 2012)
(a) Mild MS
(b) Mild MR
(c) Small ASD
(d) Small VSD
3. A 45 years old male had severe chest pain and was admitted
to the hospital with a diagnosis of acute myocardial
infarction. Four days later he died and autopsy showed
transmural coagulative necrosis. Which of the following
microscopic features will be seen on further examination?
(AIIMS May 2011)
(a) Fibroblasts and collagen
(b) Granulation tissue
(c) Neutrophilic infiltration surrounding coagulative necrosis
(d) Granulomatous inflammation
4. Which one of the following is not included as major criteria in
Jones criteria?
(AIIMS Nov. 2010)
(a) Pancarditis
(b) Arthritis
(c) Subcutaneous nodules
(d) Elevated ESR
5. The mechanism of the development of Acute Rheumatic
Fever is which of the following?
(AIIMS May 2010)
(a) Cross reactivity with exogenous antigen
(b) Innocent bystander effect
(c) Due to toxin secretion by streptococci
(d) Release of pyrogenic cytokines
6. Cardiac involvement in carcinoid syndrome is characterized
by:
(AI 2010)
(a) Calcification tricuspid valve
(b) Intimal fibrosis of right ventricle, tricuspid and pulmonary
valve.
(c) Involvement of the major blood vessels is commonly seen
(d) Equal involvement of both the sides of the heart
7. Most friable vegetation is seen in:
(AI 2010)
(a) Infective endocarditis
(b) Libman Sacks endocarditis
(c) Rheumatic heart disease
(d) Rheumatoid heart disease
8. Aschoff’s nodules are seen in:
(AI 2005)
(a) Subacute bacterial endocarditis
(b) Libman-Sacks endocarditis
(c) Rheumatic carditis
(d) Non-bacterial thrombotic endocarditis
9. A 10-year-old boy, Pappu, died of acute rheumatic fever. All
the following can be expected at autopsy except:
(a) Aschoff’s nodules
(AI 2002)
(b) Rupture of Chordae tendinae
(c) McCallum patch
(d) Fibrinous pericarditis
10. NOT true about ASO titer:
(AIIMS Nov 2009)
(a) May be positive in normal people
(b) Major Jones’ criteria
(c) May be negative in post streptococcal glomerulonephritis
(d) May not be elevated even in presence of Carditis
11. In mitral valve prolapse syndrome, histopathology of mitral
valve shows:
(AIIMS Nov 2007)
(a) Hyaline degeneration
(b) Elastic degeneration
(c) Myxomatous degeneration
(d) Fibrinoid necrosis
12. Which of the following is not a complication of infective
endocarditis?
(AIIMS Nov 2003)
(a) Myocardial ring abscess
(b) Suppurative pericarditis
(c) Myocardial infarction
(d) Focal and diffuse glomerulonephritis
13. Aschoff bodies in Rheumatic heart disease show all of the
following features, except:
(AIIMS Nov 2002)
(a) Anitschkow cells
(b) Epithelioid cells
(c) Giant cells
(d) Fibrinoid necrosis
14. Rheumatic heart disease can be diagnosed on the basis of:
(PGI Dec 2001)
(a) Aschoff bodies
(b) Vegetation along the line of closure of valves
(c) Endocardial involvement only
(d) Follows skin and throat infection
15. Pathognomic feature of acute rheumatic fever is:
(a) Pericarditis
(Delhi 2009 RP)
(b) Myocarditis
(c) Mitral stenosis
(d) Aschoff’s nodules
16. Vegetations on under surface of cusps are found in:
(a) Infective endocarditis
(Delhi PG-2008, P 2006)
(b) Libman-Sacks endocarditis
(c) SABE
(d) Rheumatic fever
17. Aschoff’s nodules are seen in:
(Delhi PG-2007)
(a) Acute rheumatic fever
(b) Bacterial endocarditis
(c) Pneumoconiosis
(d) Asbestosis
18. Anitschkow cells are pathognomonic for:
(a) Acute rheumatic fever
(Delhi PG-2006)
(b) Yellow fever
(c) Malarial spleen
(d) ITP
19. All are the causes of myocarditis except:
(a) Trichinosis
(Karnataka 2005)
(b) Mycobacterium tuberculosis
(c) Corynebacterium diphtheriae
(d) Systemic lupus erythematosus
20. Disarrangement of myofibrils is found in:
(a) Dilated cardiomyopathy
(UP 2001)
(b) Constrictive cardiomyopathy
(c) Fibroelastic cardiomyopathy
(d) Hypertrophic cardiomyopathy

MOST RECENT QUESTIONS

21. Most common cause of mitral stenosis is:


(a) Rheumatic heart disease
(b) Infective-endocarditis
(c) Diabetes mellitus
(d) Congenital
22. Calcification of aortic valve is seen in:
(a) Hurler’s syndrome
(b) Marfan’s syndrome
(c) Syphilis
(d) None
23. Most common cause of left sided cardiac failure is:
(a) Myocardial infarction
(b) Systemic hypertension
(c) Rheumatic heart disease
(d) Infective endocarditis
24. Libman-Sacks endocarditis is found in:
(a) Rheumatoid arthritis
(b) SLE
(c) Syphilis
(d) Lymphoma
25. Chronic constrictive pericarditis is most commonly caused
by:
(a) Staphylococcus
(b) TB
(c) Viral
(d) Autoimmune
26. Aschoff’s bodies are seen in:
(a) Acute rheumatic fever
(b) SLE
(c) SABE
(d) TB
27. Diagnostic feature of rheumatic fever is:
(a) Antischkow cells
(b) Aschoff’s nodule
(c) MacCallum’s patch
(d) Epithelioid cells
28. Rheumatoid factor is:
(a) lgM directed against lgG
(b) lgE directed against lgM
(c) lgG directed against lgM
(d) None
29. Major criteria for rheumatic fever, consists of all except:
(a) Pancarditis
(b) Arthritis
(c) Subcutaneous nodule
(d) Erythema nodosum
30. Which type of endocarditis has vegetation on both sides of
the valves ?
(a) Infective endocarditis
(b) Libman-Sacks endocarditis
(c) Rheumatic fever
(d) Non bacterial thrombotic enodcarditis
31. Which of the following is the feature of vegetations in
Libmann Sacks endocarditis?
(a) Large and fragile
(b) Small warty along the line of closure of valve
(c) Small or medium sized on either or both sides of valve
(d) Small bland vegetations
32. Heart failure cells are seen in which of the following organs?
(a) Kidney
(b) Heart
(c) Lungs
(d) Brain
33. Mc Callum’s patch is diagnostic of:
(a) Infective endocarditis
(b) Rheumatic endocarditis
(c) Myocardial infarction
(d) Tetralogy of Fallot (ToF)
34. Tigered effect in myocardium is due to:
(a) Malignant change
(b) Fat deposition
(c) Seen in rheumatic fever
(d) Associated with myocarditis
35. Mitral valve vegetations do not embolise usually to:
(a) Brain
(b) Liver
(c) Spleen
(d) Lung
36. ASLO titers are used in the diagnosis of:
(a) Acute rheumatoid arthritis
(b) Acute rheumatic fever
(c) Ankylosing spondylitis
(d) Osteoarthritis
37. Most common heart valve involved in IV drug user is
(a) Mitral valve
(b) Aortic valve
(c) Pulmonary valve
(d) Tricuspid valve
38. Angina, dyspnea and syncope is seen in:
(a) Pulmonary stenosis
(b) Atrial septal defect
(c) Ventricular septal defect
(d) Aortic stenosis
39. Which of the following cardiac valves is not commonly
involved in rheumatic fever?
(a) Mitral
(b) Aortic
(c) Pulmonary
(d) Tricuspid
40. In a patient with mitral valve vegetations, the vegetations are
present along lines of closure along with fusion of
commisures. Which of the following is the most likely
diagnosis?
(AIIMS May 2016)
(a) Infective endocarditis
(b) Marantic endocarditis
(c) Rheumatic endocarditis
(d) Libman sacks endocarditis
41. Level of which of the following is not elevated in heart
disease?
(a) LDH
(b) 5’-nucleotidase
(c) SGOT
(d) ALP
42. Which of the following is not a feature of rheumatic heart
disease?
(a) Chorea
(b) Arthritis
(c) Janeways lesion
(d) Carditis
43. Which protein is defective in dilated cardiomyopathy?
(a) Myosin
(b) Tropomyosin
(c) Dystropin
(d) Troponin

ISCHEMIC HEART DISEASE

44. A 70-year-old male Rohan with advanced visceral cancer


dies of extensive myocardial infarction. Autopsy also
reveals sterile non-destructive vegetations along the mitral
leaflet edges. The pathogenesis of this patient’s
vegetations is most similar to that of:
(a) Hypercalcemia of malignancy
(b) Distant metastases
(c) Trousseau syndrome
(d) Raynaud’s phenomenon
45. A 56-year-old male presented with sudden substernal pain,
impending doom and died 4 days after. On autopsy, there
was a large transmural anterior wall infarction. It would be
associated with:
(a) Presence of collagen and fibroblasts
(AI 2009)
(b) Presence of neutrophils
(c) Granulomatous inflammation
(d) Granulation tissue

46. All of the following statements regarding subendocardial


infarction are true, except:
(a) These are multifocal in nature
(AI 2006)
(b) These often result from hypotension or shock
(c) Epicarditis is not seen
(d) These may result in aneurysm
47. A 60-year-old male presented with acute chest pain of 4
hours duration. Electrocardiographic examination revealed
new Q wave with ST segment depression. He succumbed to
his illness within 24 hours of admission. The heart revealed
presence of a transmural hemorrhagic area over the
septum and anterior wall of the left ventricle. Light
microscopic examination is most likely to reveal:
(AI 2004)
(a) Edema in between normal myofibers
(b) Necrotic myofibers with presence of neutrophils
(c) Coagulative necrosis of the myocytes with presence of
granulation tissue
(d) Infiltration by histiocytes with hemosiderin laden
macrophages
48. Which of the following increases the susceptibility to
coronary artery disease?
(AI 2003)
(a) Type V hyperlipoproteinemia
(b) von Willebrand’s disease
(c) Nephrotic syndrome
(d) Systemic lupus erythematosus
49. A myocardial infarct showing early granulation tissue has
most likely occurred:
(AI 2002)
(a) Less than 1 hour
(b) Within 24 hours
(c) Within 1 week
(d) Within 1 month
50. Troponin-T is a marker of:
(AIIMS May 2004)
(a) Renal disease
(b) Muscular dystrophy
(c) Cirrhosis of liver
(d) Myocardial infarction
51. Autopsy finding after 12 hrs in a case of death due to M.I. is
(Delhi 2010)
(a) Caseous pecrosis
(b) Coagulative necrosis
(c) Fat necrosis
(d) Liquefactive necrosis
52. In MI with hypothyroidism, what is the marker of choice?
(Delhi PG-2008)
(a) LDH
(b) CPK-MB
(c) Aldolase
(d) Troponin-I
53. What is the investigation for second MI after 1 week of
previous MI?
(Delhi PG-2008)
(a) Troponin I
(b) Myoglobin
(c) CPK-MB
(d) LDH
54. Earliest light microscopic change in myocardial infarction
is: (UP 04, Bihar 03)
(a) Waviness of the fibers
(b) Neutrophilic infiltration
(c) Phagocytic infiltration
(d) Coagulative necrosis

MOST RECENT QUESTIONS

55. Most common artery involved in myocardial infarction is:


(a) Right coronary artery
(b) Left coronary artery
(c) Left anterior descending coronary artery
(d) Left circumflex coronary artery
56. In myocardial infarction scarring completes by:
(a) 1 day
(b) 1 week
(c) 1 month
(d) 3 months
57. Dressler’s syndrome is:
(a) Viral
(b) Bacterial
(c) Fungal
(d) Autoimmune
58. Heart muscle contains the isoenzymes:
(a) MM
(b) MB
(c) MM and MB
(d) BB
59. Enzyme elevated in first 2 hours of MI is:
(a) CPK MB
(b) LDH
(c) SGPT
(d) Acid phosphatase
60. Post MI day 10 which enzyme is raised:
(a) CPK
(b) Troponin
(c) LDH
(d) Myoglobin
61. 7 day old MI the most sensitive biochemical marker:
(a) Troponin T
(b) CPK MB
(c) LDH
(d) Myoglobin
62. In myocardial reperfusion injury, the maximum effect is
caused due to which of the following?
(a) Neutrophil
(b) Monocytes
(c) Eosinophils
(d) Free radicals
63. The cells seen after 72 hours in the infarcted area in MI are:
(a) Neutrophils
(b) Lymphocytes
(c) Macrophages
(d) Monocytes
64. Myocarditis is most commonly caused by
(a)Influenza
(b) Measles virus
(c) Coxsackie virus
(d) Epstein barr virus
65. In myocardial infarctions, microscopes picture of
coagulation necrosis with neutrophilic infiltration is seen
after:
(a) 4-12 hr
(b) 12-24 hr
(c) 1-3 days
(d) 3-7 days
66. During autopsy of a patient died due to suspected
myocardial infarction, the heart was stained with triphenyl
tetrazolium tetra-chloride dye. What wil be the color of the
viable myocardium?
(AIIMS Nov 2016)
(a) Blue
(b) White
(c) Yellow
(d) Brick red
67. The following is the Hematoxylin and Eosin stained section
from the heart of a patient after Myocardial Infarction. What
can you say about the age of the infarction?
(AIIMS May 2016)

(a) 1–2 days


(b) 3 weeks
(c) 6 hours
(d) 1 week

CARDIAC TUMOUR

68. Which malignancy metastasizes to heart?


(AP 2007)
(a) Bronchial carcinoma
(b) Prostate carcinoma
(c) Breast carcinoma
(d) Wilms’ tumor

MOST RECENT QUESTIONS


69. Most common tumour of heart is:
(a) Myxoma
(b) Rhabdomyosarcoma
(c) Fibroma
(d) Leiomyosarcoma
70. Atrial myxoma commonly arises from:
(a) Left ventricle
(b) Left atrium
(c) Right ventricle
(d) Right atrium
71. Most common benign heart tumor is:
(a) Rhabdomyoma
(b) Hemangioma
(c) Lipoma
(d) Myxoma

HTN, ATHEROSCLEROSIS, ANEURYSM

72. In 2 patients with atherosclerosis, one is diabetic and the


other is non diabetic. In relation to the non diabetic, the
diabetic patient has 100 times risk of which of the
following?
(AIIMS May 2012)
(a) MI
(b) Stroke
(c) Lower limb ischemia
(d) Vertebrobasilar insufficiency
73. ALL of the following statements regarding athero-sclerosis
are true except:
(AIIMS Nov 2012)
(a) Omega-3 fatty acid (abundant in fish oil) decrease LDL
(b) Atherosclerosis is less important in age more than 45 years
age
(c) Cigarette smoking is independent risk factor for M.I
(d) C reactive protein is independent risk factor for M.I
74. Which of the following is the commonest histological finding
in benign hypertension?
(AIIMS May 2011)
(a) Proliferative end arteritits
(b) Necrotizing arteriolitis
(c) Hyaline arteriosclerosis
(d) Cystic medial necrosis
75. The presence of stroke, peripheral vascular disease and
atherosclerosis is associated with which hormone?
(a) Insulin deficiency
(AI 2010)
(b) Hyperestrogenemia
(c) Hypothyroidism
(d) Progesterone
76.Most common cause of abdominal aortic aneurysm is:
(a) Atherosclerosis
(b) Syphilis
(AI 2010)
(c) Trauma
(d) Congenital
77. Hallmark feature of benign HTN is:
(AI 2009)
(a) Hyaline arteriosclerosis
(b) Cystic medial necrosis
(c) Fibrinoid necrosis
(d) Hyperplastic arteriosclerosis
78. All are seen in malignant hypertension, except:
(a) Fibrinoid necrosis
(AI 2008)
(b) Hyaline arteriosclerosis
(c) Necrotizing glomerulonephritis
(d) Hyperplastic arteriosclerosis
79. Recurrent ischemic events following thrombolysis has been
pathophysiologically linked to which of the following
factors:
(AI 2003)
(a) Antibodies to thrombolytic agents
(b) Fibrinopeptide A
(c) Lipoprotein A
(d) Triglycerides
80. 70-year-old man has abdominal pain with mass in abdomen.
Angiography reveals aneurysm of aorta. Most likely cause
is:
(AIIMS Nov 2001)
(a) Trauma
(b) Atherosclerosis
(c) Syphilis
(d) Congenital
81. In malignant hypertension hyperplastic arteriosclerosis is
seen in all except:
(AIIMS May 2001)
(a) Heart
(b) Kidney
(c) Pericardial fat
(d) Peripancreatic fat
82. CAD predisposing factors:
(PGI Dec 2002)
(a) Homocysteinemia
(b) Increased lipoprotein B
(c) Increased fibrinogen
(d) Increased HDL
(e) Increased plasminogen activator inhibitors
83. Features of essential hypertension:
(PGI Dec 2002)
(a) Concentric hypertrophy of LV
(b) Increased heart size
(c) Increased size of the heart muscles
(d) Myohypertrophy
(e) Myohyperplasia

84. In atherosclerosis, increased LDL in monocyte macrophage


is due to:
(Delhi 2010)
(a) LDL receptors on macrophage
(b) LDL receptors on endothelium
(c) Lipids in LDL get auto-oxidized
(d) All of the above
85. Which of the following is the least common site of
atherosclerotic lesions?
(Delhi 2009 RP)
(a) Aortic bifurcation
(b) Pulmonary arterial trunk
(c) Common carotid artery
(d) Middle cerebral artery

86. Vascular pathology of benign hypertension includes:


(a) Segmental fibrinoid necrosis
(Delhi 2009 RP)
(b) Hyaline arteriosclerosis
(c) Periarteritis
(d) Loss of internal elastic lamina

MOST RECENT QUESTIONS

87. Accelerated phase of hypertension is characterized


microscopically by:
(a) Fibrinoid necrosis of arteriolar wall
(b) Hyaline arteriosclerosis
(c) Elastosis of the intima
(d) Marked calcification of the media
88. Most common site of atherosclerotic aneurysm is:
(a) Coronary artery
(b) Renal artery
(c) Arch of aorta
(d) Abdominal aorta
89. Monckeberg’s calcific sclerosis affects the medium sized
muscular arteries by involving the structure of:
(a) Intima
(b) Media
(c) Adventitia
(d) All of the above
90. Malignant hypertension causes which of the following
changes in the kidney?
(a) No change in kidney
(b) Flea bitten kidney
(c) Irregular granular contracted kidney
(d) Large white kidney
91. Most common site of artery of atherosclerosis:
(a) Right coronary artery
(b) Left anterior descending coronary artery
(c) Left circumflex artery
(d) Diagonal branch of LAD
92. Most common cause of dissecting hematoma is because of:
(a) Hypertension
(b) Marfan syndrome
(c) Iatrogenic
(d) Kawasaki
93. Cystic medical necrosis is seen in:
(a) Marfan syndrome
(b) Friedrichs ataxia
(c) Down syndrome
(d) Kawasaki disease
94. Visceral aneurysm is most commonly seen in:
(a) Splenic
(b) Renal
(c) Hepatic
(d) Coronary
95. Medial calcification is seen in:
(a) Atherosclerosis
(b) Arteriolosclerosis
(c) Monckeberg’s sclerosis
(d) Dissecting aneurysm
96. Atheromatous changes of blood vessels affects early in ?
(a) Kidney
(b) Heart
(c) Liver
(d) Spleen
97. Most common cause of aortic aneurysm is:
(a) Syphilis
(b) Marfan’s syndrome
(c) Atherosclerosis
(d) Congenital
98. Ascending aorta involvement is the commonest site of
which aneurysm:
(a) Syphilitic
(b) Atherosclerotic
(c) Berry aneurysm
(d) Traumatic
99. Atherosclerosis is seen with which bacteria:
(a) Staph aureus
(b) Streptococcus pneumonia
(c) Chlamydia pneumoniae
(d) Chlamydia trachomatis
100.Commonest histological finding in benign hypertension is:
(a) Proliferating endarteritis
(b) Necrotising arteriolitis
(c) Hyaline arteriosclerosis
(d) Cystic medial necrosis
101. Onion skin thickening of arteriolar wall is seen in:
(a)Atherosclerosis
(b) Median calcific sclerosis
(c) Hyaline arteriosclerosis
(d) Hyperplastic arteriosclerosis
102. HDL receptors:
(a) SR-B1
(b) HDLR
(c) LDLR
(d) SR-B2
103. Obliterative endarteritis is vasa vasorum is seen in:
(a) hypertension
(b) TB
(c) SLE
(d) Syphilis
vasculitis, raynaud disease
104. Which of the following is a feature of temporal arteritis?
(a) Giant cell arteritis
(AIIMS Nov 2012)
(b) Granulomatous vasculitis
(c) Necrotizing vasculitis
(d) Leucocytoclastic vasculitis
105. Small vessels vasculitis seen in
(AI India 2012)
(a) Giant cell arteritis
(b) Takayasu arteritis
(c) PAN
(d) Microscopic polyangiitis
106. A 5-year-old child presents with perivascular IgA
deposition and neutrophilic collection. There is
erythematous rash on the lower limbs and non-blanching
purpura. The likely diagnosis in the child is:
(a) Henoch-Schonlein Purpura
(AIIMS Nov 2011)
(b) Wegner’s granulomatosis
(c) Giant cell Vasculitis
(d) Kawasaki’s disease
107. Pathogenesis of all of the following is granulomatous,
except:
(AI 2010)
(a) Wegener’s granulomatosis
(b) Buerger’s disease
(c) Takayasu’s arteritis
(d) Microscopic polyangiitis
108. ANCA is associated with:
(AIIMS Nov 2009)
(a) Henoch-Schonlein Purpura
(b) Goodpasture syndrome
(c) Rheumatoid arthritis
(d) Wegener’s granulomatosis
109. Which of the following is not a common cause of Vasculitis
in adults?
(AIIMS Nov 2009)
(a) Giant Cell Arteritis
(b) Polyarteritis nodosa
(c) Kawasaki’s disease
(d) Henoch-Schonlein Purpura
110. Hypersensitivity vasculitis most commonly involves:
(a) Arterioles
(AIIMS May 09, Nov 08, DNB 2008)
(b) Post-capillary venules
(c) Capillaries
(d) Medium sized arteries
111. A patient presents with respiratory symptoms, i.e. cough,
hemoptysis and glomerulonephritis. His c-ANCA levels in
serum were found to be raised. The most likely diagnosis
is:
(AIIMS Nov 2002)
(a) Goodpasture’s syndrome
(b) Classic polyarteritis nodosa
(c) Wegener’s granulomatosis
(d) Kawasaki’s syndrome
112. Vasculitis is seen in:
(PGI Dec 2002)
(a) Buerger’s disease
(b) HSP
(c) Gout
(d) Reiter’s disease
(e) Behcet’s syndrome
113. Wegener’s granulomatosis involve:
(PGI Dec 2003)
(a) Lung
(b) Liver
(c) Kidney
(d) Upper respiratory tract
(e) Heart
114.Wegener’s granulomatosis:
(PGI Dec 2006)
(a) Involves lungs
(b) Involves nose
(c) Involves kidney
(d) Treated with cytotoxic agent and/or steroids
115. All are true about ANCA associated crescentic
glomerulonephritis, except: (Delhi 2009
RP)
(a) Seen in Wegener’s granulomatosis
(b) Seen in microscopic polyangitis
(c) Seen in Henoch-Schönlein purpura
(d) Is pauci immune in nature
116. All of the following are small vessel vasculitis except:
(a) Kawasaki’s disease
(Delhi PG-2006)
(b) Churg-Strauss syndrome
(c) Wegener granulomatosis
(d) None of the above
117. Polyarteritis nodosa can occur in association with which of
the following:
(Delhi PG-2005)
(a) Hypertension
(b) Trauma
(c) Drugs
(d) Bronchial asthma

MOST RECENT QUESTIONS

118. The term infantile polyarteritis nodosa was formerly used


for:
(a) Goodpasture’s disease
(b) Henoch-Schönlein purpura
(c) Kawasaki disease
(d) Takayasu’s arteritis
119. Most common organs involved in Wegener’s
granulomatosis are:
(a) Skin and nose
(b) Lung and kidney
(c) Heart and kidney
(d) Kidney and nervous system
120. Polyarteritis nodosa does not involve:
(a) Pulmonary artery
(b) Bronchial artery
(c) Renal artery
(d) Cerebral artery
121. C-ANCA antibodies are characteristic of:
(a) Sjögren’s syndrome
(b) Giant cell arteritis
(c) Wegener’s granulomatosis
(d) Kawasaki’s disease
122. ANCA is seen in all except:
(a) Wegener’s granulomatosis
(b) Henoch-Schönlein purpura
(c) Microscopic polyangiitis
(d) Churg-Strauss disease
(e) Tuberculosis
123. Which is associated with vasculitis of medium size
vessels:
(a) Temporal arteritis
(b) Wegener’s granulomatosis
(c) Polyarteritis nodosa
(d) Henoch-Schönlein purpura
124. All is true about Giant cell arteritis except:
(a) Involves large to small sized arteries
(b) Granulomatous inflammation
(c) Most commonly involved artery is abdominal aorta
(d) Segmental nature of the involvement
125. In PAN, the lesions are seen in all except:
(a) Lung
(b) Pancreas
(c) Liver
(d) Heart
126. Which of the following is not a characteristic of Wegener’s
granulomatosis?
(a) Granuloma is vessel wall
(b) Focal necrotising glomerulonephritis
(c) Positive for cANCA
(d) Involves large vessels
127. Raynaud’s phenomenon what change is seen in vessels
initial stage:
(a) No change
(b) Thrombosis
(c) Fibrinoid necrosis
(d) Hyaline sclerosis
128. Frequency of renal involvement in Henoch Schonlein
Purpura (HSP) is ?
(a) 20-40%
(b) >80%
(c) 40-60%
(d) 10%
129. Which of the following is abdominal angiitis?
(a)Giant cell arteritis
(b) Takayasu arteritis
(c) Kawasaki disease
(d) Polyarteritis nodosa
130. Glomus cells are found in which of the following
conditions?
(a) Carotid body tumour
(b) Thyroid carcinoma
(c) Liver carcinoma
(d) Glomus tumor
131.Sturge weber syndrome is associated with:
(a) Port wine stain
(b) Cavernous hemangioma
(c) Lymphangioma
(d) Hemangiosarcoma
132. All of the following are true about pyogenic granuloma
except
(a) Bacterial infection
(b) Bleeding
(c) Benign tumour
(d) Capillary hemangioma
133. Glomus tumor is seen in:
(a) Retroperitoneum
(b) Soft tissue
(c) Distal portion of digits
(d) Proximal portion of digits
134. Anti-neutrophil cytoplasmic antibodies (ANCA) is seen in:
(a) Wegener’s granulomatosis
(b) Diabetes mellitus
(c) Rheumatoid arthritis
(d) Churg-Strauss syndrome
135. A 6-year old girl presents with fever for the past 5 days,
generalized erythematous rash, strawberry tongue and
cervical lymphadenopathy. The most likely diagnosis is:
(a) Kimura disease
(b) Kawasaki disease
(c) Scarlet fever
(d) Rosie-Dorfman syndrome
136. Neutrophilic infiltration with fibrinoid necrosis in walls of
vessels is seen in:
(a) Giant cell arteritis
(b) Takayasu arteritis
(c) Churg-Strauss syndrome
(d) Polyarteritis nodosa
137. Most dreadful complication of Kawasaki’s disease is:
(a) Rash
(b) Lymph node
(c) Cardiac involvement
(d) Thrombrocytosis
138. Bilateral pulselessness in hand which of the following
conditions?
(a) Giant cell arteritis
(b) Takayasu arteritis
(c) Kawasaki disease
(d) Polyartertis nodosa
139. False regarding cavernous hemangioma is:
(a) More infiltrative than capillary hemangioma
(b) Undergo spontaneous regression
(c) Intravascular thrombosis and dystrophic calcification seen
commonly
(d) Associated with VHL disease
140. Kawasaki disease not true:
(a) Erythema
(b) Posterior cervical lymphadenopathy
(c) Thrombocytopenia
(d) Conjuctivitis
1. Ans. (a) Myxomatous degeneration and prolapse of the mitral
valve
(Ref: Robbins 9/e p556, 8th/563-565)
The important clues given in the question;
• Female patient
• Presenting for Routine examination (means she was
asymptomatic)
• Presence of mid systolic click on physical examination
• Absence of history of rheumatic heart disease
• All these are significant pointers towards a diagnosis of
mitral valve prolapse or Barlow syndrome. The other name
of the same condition is Myxomatous degeneration of the
mitral valve. So, the answer is option ‘a’
Direct lines from Robbins ‘Most patients with mitral valve prolapse
are asymptomatic, and the condition is discovered only on
routine examination by the presence of a midsystolic click
as an incidental finding on physical examination’
Concept

• Commissural fusion that typifies rheumatic heart disease is absent in


mitral valve prolapse.

2. Ans. (c) Small ASD


(Ref: Ghai 7th/390,403, Adult congenital heart disease: a
practical guide page/36-37)
Direct quote Ghai… ‘Infective endocarditis is very rare in patients of
ostium secundum atrial septal defect, unless floppy mitral
valve is present’.
Risk of infective endocarditis in various lesions
High Risk Moderate Risk Low Risk
• Prosthetic heart value • MVP + M.R. • ASD
• Tetralogy of Fallot • Tricuspid stenosis • MVP without
• PDA • Tricuspid MR
• Aortic regurgitation regurgitation
• Aortic stenosis • Pulmonary stenosis
• Coarctation of Aorta • Mitral stenosis
• VSD
• Mitral regurgitation
3. Ans. (c) Neutrophilic infiltration surrounding coagulative
necrosis
(Ref: Robbins 8th/550; 7th/579)
4. Ans. (d) Elevated ESR (Ref: Robbins 9/e 559, 8th/566)
5. Ans. (a) Cross reactivity with exogenous antigen
(Ref: Robbins 8th/566, 9/e 558)
Acute rheumatic fever results from immune response to group A
streptococci (Strep. pyogenes) which cross-reacts with host
tissues. The antibodies directed against the M proteins of
streptococci cross react with the self antigens in the heart. In
addition, CD4+ T cells specific for streptococcal peptides also
react with self proteins in the heart and produce macrophage
activating cytokines. So, the damage to the heart tissue is a
combination of antibody and T-cell mediated reactions.
6. Ans. (b) Intimal fibrosis of right ventricle, tricuspid and
pulmonary valve.
(Ref: Robbins 8th/569, 9/e 562)
• Cardiac lesions are present in 50% of the patients with the carcinoid
syndrome.
• These are largely right-sided due to inactivation of both serotonin and
bradykinin in the blood during passage through the lungs by the
monoamine oxidase present in the pulmonary vascular endothelium.

7. Ans. (a) Infective endocarditis


(Ref: Robbins 9/e 560)
The hallmark of infective endocarditis is the presence of friable, bulky
and potentially destructive vegetations containing fibrin,
inflammatory cells and bacteria or other organisms on the heart
valves. Do refer to the table comparing different vegetations in
differrent condtions in the text.
8. Ans. (c) Rheumatic carditis
(Ref: Robbins 7th/593, 9/e 558, Harrison 17th/2095)
• Aschoff’s bodies are characteristic focal inflammatory lesion of
acute rheumatic fever found in any of the three layers of the heart.

9. Ans. (b) Rupture of Chordae tendinae


(Ref: Robbins 9/e 558-559, 7th/593-94, Harrison’s 17th/2092)
Rupture of chordae tendinae is a feature of chronic rheumatic heart
disease.
10. Ans. (b) Major Jones’ criteria
(Ref: Robbins 9/e 559, 8th/566, Harrison 17th/2095)
ASO titre may be positive due to streptococcal infection even in
normal people. In some individuals with rheumatic carditis, ASO
titre may not be elevated. In PSGN, the titre of anti DNA se B
antibody is elevated more commonly than ASLO.
11. Ans. (c) Myxomatous degeneration
(Ref: Robbins 7th/591-2,8th/563, 9/e 556, Harrison 17th/1472)

MVP/Myxomatous degeneration/Barlow’s syndrome/Floppy-valve


syndrome

• Characteristic anatomic change in myxomatous degenerationQ is


interchordial ballooning of the mitral leaflets. The affected leaflets are
often enlarged, redundant, thick, and rubbery.
• Annular dilationQ is a characteristic finding. (it is rare in other causes
of mitral insufficiency).
• There is reduction in the production of type III collagen and electron
microscopy has revealed fragmentation of collagen fibrils.

12. Ans. (c) Myocardial infarction


(Ref: Harrison 17th/791, Harsh Mohan 6th/448, Robbins7th 596-7,
8th/567, 9/e 561)
Cardiac complications Extra cardiac complications
• Valvular stenosis or • Systemic emboli from left side of
insufficiency heart affect spleen, brain and
• Abscess on the myocardium kidneys whereas those from right
(ring abscess) heart affect pulmonary abscess
• Myocardial abscess formation.
• Suppurative pericarditis • Antigen -antibody complexes
• Perforation, rupture and cause focal (more commonly) and
aneurysm of valve leaflets diffuse (less commonly)
• Cardiac failure glomerulonephritis.

Harrison writes that “Emboli to a coronary artery may result in


myocardial infarction; nevertheless embolic transmural infarcts are rare.”

13. Ans. (b) Epithelioid cell

(Ref: Robbins 7th/593-4, 8th/565-6, 9/e 558)


Aschoff bodies consist of foci of swollen eosinophilic collagen
surrounded by lymphocytes (primarily T cells), occasional
plasma cells, and plump macrophages called Anitschkow cells
(pathognomonic for rheumatic fever). These cells are also
called “caterpillar cells”. Some of the larger macrophages
become multinucleated to form Aschoff giant cells.
14. Ans. (a) Aschoff bodies; (b) Vegetation along the line of
closure of valves
(Ref: Robbins 7th/593-94, 9/e 558)
15. Ans. (d) Aschoff’s nodule
(Ref: Robbins 8th/565-566, 9/e 558)
16. Ans. (b) Libman-Sacks endocarditis
(Ref: Robbins 7th/597, 89, 9/e 562)
Vegetations in Libman Sack’s endocarditis occur on surfaces of
cusps. Both surfaces may be involved but, more commonly, the
undersurface is affected.
17. Ans. (a) Acute rheumatic fever
(Ref: Robbin 7th/593, 9/e 558)
18. Ans. (a) Acute rheumatic fever

(Ref: Robbin 8th/565, 7th/593, 9/e 558)


19. Ans. (b) Mycobacterium tuberculosis
(Ref: Robbins 7th/607-609, 9/e 571)
• Mycobacterium tuberculosis causes involvement of pericardium
(Caseous pericarditis). It is the commonest cause of chronic
constrictive pericarditisQ.
• Primary pericarditis is unusual and is almost always viralQ in origin.

20. Ans. (d) Hypertrophic cardiomyopathy


(Ref: Robbins 9/e 569, 8th/576; 7th/604, 607)
21. Ans. (a) Rheumatic heart disease
(Ref: Robbins 9/e 554)
22. Ans. (c) Syphilis
(Ref: Robbins 9/e p554-555)
23. Ans. (a) Myocardial infarction
(Ref: Robbins 9/e p529)
24. Ans. (b) SLE
(Ref: Robbins 9/e p562, 8th/220; 7th/598)
25. Ans. (b) TB
(Ref: Harrison 17th/1493, Robbins 9/e p575)
26. Ans. (a) Acute rheumatic fever
(Ref: Robbins 9/e p558)
27. Ans. (b) Aschoff’s nodule
(Ref: Robbins 9/e p558)
28. Ans. (a) lgM directed against lgG
(Ref: Robbins 9/e p1210, 8th/1238)
29. Ans. (d) Erythema nodosum
(Ref: Robbins 9/e p559)
30. Ans. (b) Libman-Sacks endocarditis (Ref:
Robbins 8/e p567, 9/e p562) ...Refer to the earlier question
31. Ans. (c) Small or medium sized on either or both sides of
valve
(Ref: Robbins 9/e p560) ...see Table 9.1 in text
32. Ans. (c) Lungs
(Ref: Robbins 8/e p 535, 9/e p529)
33. Ans. (b) Rheumatic endocarditis
(Ref: Robbins 9/e p558)
34. Ans. (b) Fat deposition
(Ref: Robbins 8/e p34)
In a pattern of lipid deposition seen with prolonged moderate hypoxia,
such as that produced by profound anemia, there is intracellular
deposits of fat, which create grossly apparent bands of yellowed
myocardium alternating with bands of darker, red-brown,
uninvolved myocardium (tigered effect).
35. Ans. (d) Lung
(Ref: Robbins 9/e p561, 8/e p566, 7/e p597)
• Mitral valve vegetations are associated with systemic
embolisation which can affect brain, liver, spleen and
kidney.
• Embolisation of the lung is associated with involvement of
right heart (tricuspid valve vegetation) involvement.
36. Ans. (b) Acute rheumatic fever… discussed in detail earlier
(Ref: Robbins 8/e p565-566, 9/e 559)
37. Ans. (d) Tricuspid valve
(Ref: Robbins 9/e p560)
38. Ans. (d) Aortic stenosis
(Ref: Robbins 9th/555)
2
The valve area is approximately 0.5 to 1 cm in severe aortic stenosis
whereas it is normally approximately 4 cm2. The aortic stenosis
leads to concentric left ventricular (pressure overload)
hypertrophy. If untreated, most patients with aortic stenosis will
die within 5 years of developing angina, within 3 years of
developing syncope, and within 2 years of CHF onset.
39. Ans. (c) Pulmonary
(Ref: Robbins 9/e p559)
40. Ans. (c) Rheumatic endocarditis
(Ref: Robbins 9th/ 560)
Presence of vegetation along lines of closure of the valve is
highly suggestive of this being a case of rheumatic heart
disease. Readers are requested to see the figure given in
Robbins.
41. Ans. (b) 5’-nucleotidase
(Ref: Robbins 9/e p547, Harrison 19/e p1997)
• Alkaline phosphatase may be increased in congestive heart failure.

42. Ans. (c) Janeways lesion


(Ref: Robbins 9/e p561)
Janeway lesions are a feature of infective endocarditis.
43. Ans. (c) Dystropin
(Ref: Robbins 9/e p565)
44. Ans. (c) Trousseau syndrome
(Ref: Robbins 9/e p562)
The pathogenesis of nonbacterial thrombotic endocarditis (NBTE) often
involves a hypercoagulable state. When the hypercoagulability is the result
of the procoagulant effects of circulating products of cancers the resulting
cardiac valve vegetations may also be called marantic endocarditis. The
pathophysiology of NBTE is similar to that of Trousseau’s syndrome
(migratory thrombophlebitis) which may also be induced by disseminated
cancers like mucinous adenocarcinoma of the pancreas and
adenocarcinoma of the lung which may relate to procoagulant effects of
circulating mucin.
(Choice B) Cancer metastases to the heart usually involve the
pericardium or myocardium. Valve metastases are less frequent
and would probably have shown invasive characteristics on
histological examination.
45. Ans. (b) Presence of neutrophils
(Ref: Robbins 9/e p544)
As discussed in the text, granulation tissue appears between 7-10
days and collagen appears after 2 months. Between 3-7 dyas,
neutrophils start disintegrating with early phagocytosis caused
by macrophages. Presence of macrophages would have been a
better answer but in the given question, presence of neutrophils
is the best option.
46. Ans. (d) These may result in aneurysm
(Ref: Robbins 9/e p543, 7th/575)
Ventricular aneurysms result from transmural infarcts, which involve
the whole thickness of myocardium from epicardium to
endocardium. Subendocardial infarcts being limited to only
the inner one-third or at most one half of the ventricular wall
do not cause ventricular aneurysms.
Aneurysm of the ventricular wall most commonly results from a
large transmural anteroseptal infarct.
47. Ans. (b) Necrotic myofibers with presence of neutrophils
(Ref: Robbins 7th/578-581, 9/e p544)
The patient in the question succumbed to myocardial infarction after
about 28 hours of the attack. After twenty-four hours of the
attack light microscopy shows coagulative necrosis of
myofibrils with loss of nuclei and striations along with an
interstitial infiltrate of neutrophils. For details, see the table
of morphological changes of MI in text.
48. Ans. (c) Nephrotic syndrome
(Ref: Harrison 17th/272, Robbins 9/e p914)
• Nephrotic syndrome is a clinical complex characterized by
proteinuria (> 3.5 g/day), hypoalbuminemia, edema and
hyperlipidemia.
• A hypercoagulable state frequently accompanies severe
nephrotic syndrome due to urinary loss of AT-III, reduced
serum levels of protein C and S, hyperfibrinogenimia and
enhanced platelet aggregation.
• Due to increased coagulation state, predisposition to CAD
is present in patients with nephrotic syndrome.
Potential future questions
Among hyperlipoproteinemias type II, III and IV are associated with
increased risk of CAD whereas Type I and V are not associated with CAD.

49. Ans. (c) Within 1 week


(Ref: Robbins 7th/579, 9/e 544, Chandrasoma Taylor 3rd/364)

50. Ans. (d) Myocardial infarction

(Ref: Robbins 9/e 547, 8th/555, 7th/583, Harrison 17th/1534)


The preferred biomarkers for myocardial damage are cardiac-specific
proteins, particularly Troponin-I (TnI) and Troponin-T.
51. Ans. (b) Coagulative necrosis
(Ref: Robbins 9/e 545)
52. Ans. (d) Troponin-I
(Ref: Cardiovascular Imaging ; Vol. 22, No. 2, April 2006, Robbins
9/th p547)
Hypothyroid patients have increased concentration of CPK that is
mostly due to increased CPK MM. However CPK-MB has also
been reported to increase above reference value in hypothyroid
patients without myocardial damage. This may create confusion
during the evaluation of myocardial injury in a hypothyroid
patient presenting with chest pain.
Troponin I is considered superior, marker for the diagnosis of
myocardial infarction in hypothyroid patient.
53. Ans. (a) Troponin
(Ref: Harrison 17th/1535, Robbins 9/e 547)
• Now Troponin is a better marker for diagnosis of
reinfarction.
54. Ans. (a) Waviness of the fibers
(Ref: Robbins 9/e p544)
55. Ans. (c) Left anterior descending coronary artery
(Ref: Robbins 9/e p542, 8th/551; 7th/577)
56. Ans. (d) 3 months
(Ref: Robbins 9/e p544)
57. Ans. (d) Autoimmune
(Ref: Robbins 9/e p549)
58. Ans. (c) MM and MB
(Ref: Robbins 8th/555, 9/e p547)
59. Ans. (a) CPK MB
(Ref: Robbins 9/e p547)
60. Ans. (c) LDH
(Ref: Robbins 8/e p555, 9/e p547)
61. Ans. (a) Troponin T
(Ref: Robbins 8/e p555, 9/e p547)
Initiation of Return to
Enzyme rise Peak baseline
CK-MB 2-4 hours 24 hours 48-72 hours
Troponin T and l 2-4 hours 48 hours 7-10 days
(TnT, Tnl)
AST/SGOT In 12 hours 48 hours 4-5 days
LDH 24 hours 3-6 days 2 weeks

62. Ans. (d) Free radicals


(Ref: Robbins 8/e p553, 9/e p546)
Restoration of blood flow to ischemic tissues can promote recovery of
cells if they are reversibly injured. However, under certain
circumstances, when blood flow is restored to cells that have
been ischemic but have not died, injury is paradoxically
exacerbated and proceeds at an accelerated pace. This process
is called ischemia-reperfusion injury. The following mechanisms
have been proposed for the reperfusion injury:
• New damage may be initiated during reoxygenation by
increased generation of reactive oxygen and nitrogen
species from parenchymal and endothelial cells and from
infiltrating leukocytes.
• Ischemic injury is associated with inflammation as a result
of the production of cytokines and increased expression of
adhesion molecules by hypoxic parenchymal and
endothelial cells, which recruit circulating neutrophils to
reperfused tissue.
• Activation of the complement system may contribute to
ischemia-reperfusion injury. Some IgM antibodies have a
propensity to deposit in ischemic tissues, for unknown
reasons, and when blood flow is resumed, complement
proteins bind to the deposited antibodies, are activated,
and cause more cell injury and inflammation.
63. Ans. (c) Macrophages
(Ref: Robbins 8/e p550, 9/e p544) ...see Table 9.2
64. Ans. (c) Coxsackie virus
(Ref: Robbins 7/e 610, 9/e p570)
65. Ans. (c) 1-3 days…. See the table in text or details
(Ref: Robbins 9/e p544, 8/e p550, 7/e p579)
66. Ans. (d) Brick red
(Ref: Robbins 9th / 544)
When tissue slices are dipped in a solution of triphenyltetrazolium
chloride, the stain imparts a brick-red color to intact,
noninfarcted myocardium where lactate dehydrogenase
activity is preserved. Because dehydrogenases leak out through
the damaged membranes of dead cells, an infarct appears as
an unstained pale zone.
67. Ans. (c) 6 hours
(Ref: Robbins 9th/544)

Presence of a coagulative necrotic pattern along with widened space


between dead fibers and wavy fiber appearance suggests this to a pattern of
injury taking place around 6 hours.

68. Ans. (a) Bronchial carcinoma


(Ref: Robbins 9/e p574)
69. Ans. (a) Myxoma
(Ref: Robbind 8/e p583, 9/e 575)
• Myxomas are the most common primary tumor of the heart in adults.
• Familial cardiac myxoma syndrome (known as Carney syndrome) is
characterized by autosomal dominant transmission, multiple cardiac and
often extracardiac (e.g. skin) myxomas, spotty pigmentation, and
endocrine overactivity.

70. Ans. (b) Left atrium


(Ref: Robbins 9/e 575)
Atrial myxoma is most commonly located on the left atrium.
Other important points have discussed earlier.
71. Ans. (d) Myxoma
(Ref: Robbins 9/e p575, 8/e p583-4)
72. Ans. (c) Lower limb ischemia
(Ref: Robbins 8th/499, 1140)
Direct quote from Robbins... ‘Gangrene of the lower extremities as a
result of advanced vascular disease is about 100 times more
common in diabetics than in general population’.

The risk of myocardial infarction (MI) is twice in a diabetic as compared to a


non diabetic individual.

73. Ans. (b) Atherosclerosis is less important in age more than


45 years age
(Ref: Robbins 9/e p493, 8th/497)
Risk factors for atherosclerosis
Modifiable Non modifiable
• Hyperlipidemia • Increasing age
• Hypertension • Gender
• Cigarette smoking • Family history
• Diabetes • Genetics
• C reactive protein

According to Robbins,
• Age is a dominant influence on atherosclerosis. Between ages 40 and 60,
the incidence of myocardial infarction increases fivefold.
• Omega-3 fatty acids (abundant in fish oils) are beneficial, whereas (trans)
unsaturated fats produced by artificial hydrogenation of polyunsaturated
oils (used in baked goods and margarine) adversely affect cholesterol
profiles.
• Cigarette smoking is a well-established risk factor in both men and
women. Prolonged (years) smoking of one pack of cigarettes or more
daily increases the death rate from ischemic heart disease by 200%.
Smoking cessation reduces that risk substantially.
• C-reactive protein (CRP) is an acute-phase reactant synthesized primarily
by the liver. When locally synthesized within atherosclerotic intima, it can
also regulate local endothelial adhesion and thrombotic states. Most
importantly, it strongly and independently predicts the risk of myocardial
infarction, stroke, peripheral arterial disease, and sudden cardiac death,
even among apparently healthy individuals

74. Ans. (c) Hyaline arteriosclerosis


(Ref: Robbin’s 9/e p490)
Hyaline arteriosclerosis is characterized by the following:

• Pink, hyaline thickening of arteriolar walls (due to leaking of plasma


proteins) associated with luminal narrowing.
• Seen in elderlyQ, more commonly in benign hypertensionQ, diabetes
mellitusQ (DM) and benign nephrosclerosisQ.

75. Ans. (a) Insulin deficiency


(Ref: Robbins 9/e p499)
All the mentioned features in the question are the macrovascular
complications of diabetes mellitus which is caused due to insulin
deficiency.
76. Ans. (a) Atherosclerosis
(Ref: Robbins 8th/507, Harrison 18th/2060-1)
Direct quote from Harrsion….’atleast 90% of all abdominal aortic
aneurysms >4.0 cm are related to atherosclerotic disease
and most are present just below the renal arteries’.
77. Ans. (a) Hyaline arteriosclerosis
(Ref: Robbins 9/e p490)
Hyaline arteriosclerosis is characterized by the following:
• Pink, hyaline thickening of arteriolar walls (due to leaking of plasma
proteins) associated with luminal narrowing.
• Seen in elderly, more commonly in benign hypertension, diabetes
mellitus (DM) and benign nephrosclerosis.
Hyaline arteriolosclerosis affecting glomeruli (G)
78. Ans. (b) Hyaline arteriolosclerosis
(Ref: Robbins 7th/534, 8th/496, 949, 9/e p490)
Features of malignant hypertension
The characteristic features include:
1. Fibrinoid necrosisQ of the small vessels particularly the arterioles
2. Necrotizing arteriolitisQ: Presence of inflammatory cells in the vessel
wall
3. Hyperplastic arteriosclerosisQ: Proliferation of the internal smooth
muscle cells seen in the interlobular arteries giving onion skin like
appearanceQ

Renal involvement is called malignant nephrosclerosis having the features


like
a. Kidneys have a typical “flea bitten “appearanceQ (due to petechial
hemorrhage on cortical surface)
b. Necrotizing glomerulonephritis: Necrotizing arteriolitis may involve the
glomeruli also.

79. Ans. (c) Lipoprotein A


(Ref: Robbins 7th/520-521)
Lp (a) has structural similarity with plasminogen. It interferes with
formation of plasmin and hence, fibrinolysis. This contributes to
atherosclerosis.

80. Ans. (b) Atherosclerosis


(Ref: Harrison 17th/1563, 18th/2060-1, Robbins 7th/531, 9/e p503)
81. Ans. (a) Heart
(Ref: Ultrastructural pathology/374, Robbins 8th/495-6)
Friends, trust me this seemed to be an easy one but as i found out to
my surprise, the reference pages mentioned by all MCQ books
including our competitors (whom we expect to copy this info as
well as has happened so many times earlier !J) give page
number from Robbins which don’t have this info! But I managed
to find something worthy as follows:
Ultrastructural pathology by Cheville NF page 374 mentions that
…. ‘hyperplastic arteriosclerosis of the kidney has the most
serious effects but this lesion is also found in the
artereioles of the intestine , gall bladder and pancreas. ” I
was not able to found anything relevant about pericardial fat. But
after discussion with many senior faculty members, the answer
of consensus is option ‘a’ i.e. Heart.
82. Ans. (a) Homocysteinemia; (c) Increased fibrinogen; (e)
Increased plasminogen activator inhibitors.
(Ref: Robbins 7th/520, 8th/498, 9/e p492)
83. Ans. (a) Concentric hypertrophy of LV; (b) Increased heart
size; (c) Increased size of the heart muscles; (d)
Myohypertrophy:
(Ref: Robbins 7th/587, Harrison 17th/1552)
Features of essential hypertensions are:
• Concentric hypertrophy of the left ventricles due to pressure
overload of the heart.
• Increase in weight of heart (>500 gm) is disproportionate to
the increase in overall size of heart.
• Thickening of the left ventricular wall increased the ratio of
its wall thickness to radius.
• Microscopically the earliest changes of systemic
hypertensive heart disease is an increased transverse
myocytes diameter. In advanced stage the cellular and
nuclear enlargement is prominent. Electron microscopy
reveals increase in number of myofilaments comprising
myofibrils, mitochondrial changes and multiple intercalated
disks.
• Increase total RNA and ratio of RNA to DNA contents.
84. Ans. (c) Lipids in LDL get auto-oxidized
(Ref: Harrison 17th/1502 Robbins 7th/523, 9/e p496)
Scavenger receptors and not LDL receptors on macrophages result in
uptake of oxidized LDL to form foam cells contributing to atherosclerosis.

85. Ans. (b) Pulmonary arterial trunk


(Ref: Robbins 9/e p498)
86. Ans. (b) Hyaline arteriosclerosis
(Ref: Robbins 9/e p490)
87. Ans. (a) Fibrinoid necrosis of arteriolar wall
(Ref: Robbins 7th/1007, 9/e p490)
Accelerated phase of hypertension is the other name for malignant
hypertension.
88. Ans. (d) Abdominal aorta
(Ref: Robbins 9/e p502)
89. Ans. (b) Media
(Ref: Robbins 9/e p491)
90. Ans. (b) Flea bitten kidney
(Ref: Robbins 9/e p490)
In malignant hypertension, on gross inspection the kidney size
depends on the duration and severity of the hypertensive
disease. Small, pinpoint petechial hemorrhages may appear on
the cortical surface from rupture of arterioles or glomerular
capillaries, giving the kidney a peculiar “flea-bitten”
appearance.
Causes of contracted kidney
Symmetric Asymmetric
• Chronic glomerulonephritis • Chronic pyelonephritis
• Benign nephrosclerosis
Causes of enlarged kidneys
• Amyloidosis • Diabetic renal disease [Kimmelstiel
• Rapidly progressive Wilson nodules are pathognomic]
glomerulonephritis (RPGN) • Polycystic kidney disease
• Myeloma kidney • Bilateral obstruction (hydronephrosis)

91. Ans. (b) Left anterior descending coronary artery


(Ref: Robbins 8/e p551, 9/e p542)
The frequencies of involvement of each of the three main arterial
trunks and the corresponding sites of myocardial lesions
resulting in infarction (in the typical right dominant heart) are as
follows: LAD >> RCA >> LCX.
92. Ans. (a) Hypertension
(Ref: Robbins 9/e p504) ...see text
93. Ans. (a) Marfan syndrome
(Ref: Robbins 9/e p502)
The vascular wall is weakened through loss of smooth muscle cells or
the inappropriate synthesis of noncollagenous or nonelastic
ECM. Atherosclerosis and hypertension induced ischemia is
reflected in “degenerative changes” of the aorta, whereby
smooth muscle cell loss leads to scarring (and loss of elastic
fibers), inadequate ECM synthesis, and production of increasing
amounts of amorphous ground substance (glycosaminoglycan).
Histologically these changes are collectively called cystic medial
degeneration. Though these are nonspecific, they can be seen
with Marfan diseaseQ and scurvyQ
94. Ans. (a) Splenic
(Ref: Peripheral Vascular Interventions chapter 15, pg 263)
• The visceral arteries include the three main unpaired
branches of the abdominal aorta namely celiac artery,
superior mesenteric and inferior meseneteric arteries.
• The most common visceral vessel showing aneurysm formation is
splenic arteryQ followed by the hepatic artery.

95. Ans. (c) Monckeberg’s sclerosis


(Ref: Robbins 9/e p491)
• Mönckeberg medial sclerosis is characterized by calcific
deposits in muscular arteries in persons typically older
than age 50.
• The deposits may undergo metaplastic change into bone.
• Nevertheless, the lesions do not encroach on the vessel
lumen and are usually not clinically significant.
96. Ans. (b) Heart
(Ref: Robbins 8/e p502, 9/e p498)
Coronary arteries supply the heart. They are the second most
common vessel (after abdominal aorta) to be affected in
atherosclerosis.
97. Ans. (c) Atherosclerosis
(Ref: Robbins 9/e 502) ...see text
98. Ans. (a) Syphilitic
(Ref: Robbins 8 /e p507-508, 7/e p532)
Direct lines… “The obliterative endarteritis characteristic of late-
stage syphilis shows a predilection for small vessels,
including those of the vasa vasorum of the thoracic aorta.”
Atherosclerosis affects abdominal aorta most commonly.
Traumatic aneurysm can affect any site of the aorta. Berry
aneurysm affects the circle of Willis.
99. Ans (c) Chlamydia pneumoniae
(Ref: Robbins 9/e 496)
Robbins.. “Herpes virus, cytomegalovirus and Chlamydia
pneumoniae have all been detected in atherosclerotic plaques
but not in normal arteries, and seroepidemiologic studies find
increased antibody titers to C. pneumoniae in patients with more
severe atherosclerosis”.
100. Ans (c) Hyaline arteriosclerosis
(Ref: Robbins 9/e p490)
101. Ans. (d) Hyperplastic arteriosclerosis
(Ref: Robbins 9/e p490, 8/e p496, 7/e p530)
Hyperplastic arteriosclerosis is associated with malignant
hypertension in which concentric thickening of the walls
and luminal narrowing leads to “onion skin like lesions”.
102. Ans (a) SR-B1
(Ref: Robbins 9/e p291-e-2)
HDL cholesterol can also be taken up directly by hepatocytes
via the scavenger receptor class B1 (SR-B1), a cell surface
receptor that mediates the selective transfer of lipids to cells.
Recent info… SR-B1 is also useful for facilitating the entry of
HCV in the host cells.
103. Ans (d) syphilis
(Ref: Robbins 9/e p380)
104. Ans. (a) Giant cell arteritis
(Ref: Robbins 9/e p508)
105. Ans. (d) Microscopic polyangiitis
(Ref: Robbins 9/e p506, 8th/512, 515) ...see Table 9.3
106. Ans. (a) Henoch-Schonlein Purpura (Ref:
Robbins 8th/934, 9/e p926, Harrison 17th/2128, Wintrobes
12th/1343)
107. Ans. (d) Microscopic polyangiitis
(Ref: Robbins 9/e p510)
Robbins clearly mentions that “microscopic polyangiitis is
characterized by segmental fibrinoid necrosis of the media with
focal transmural necrotizing lesions; granulomatous
inflammation is absent”.
108. Ans. (d) Wegener’s granulomatosis
(Ref: Robbins 8th/516-517, 9/e p507)
109. Ans. (c) Kawasaki’s disease (Ref:
Robbins 9/e p510, 8th/515, 7th/539, Harrison 17th/2130.
18th/2800)
Kawasaki’s disease (mucocutaneous lymph node syndrome) is an
arteritis that often involves the coronary arteries, usually in
young children and infants (majority of the cases are seen in
<5 years old). It is treated with intravenous immunoglobulin and
aspirin. (For details, see text.)
110. Ans. (b) Post-capillary venules (Ref:
Harrison 17th/2128, 18th/2798, Robbins 8th/515, 9/e p510)
Direct quote from Harrison 18th/2798… ‘Post capillary venules are
the most commonly involved vessels; capillaries and
arterioles may be involved less frequently’.
111. Ans. (c) Wegener’s granulomatosis
(Ref: Robbins illustrated 7th/541, 8th/516, 9/e p511-512)
112. Ans. (a) Buerger’s disease; (b) HSP; (d) Reiter’s disease; (e)
Behcet’s syndrome.
(Ref: Harrison’ 17th/2119, 1485, Robbins’ 7th/535, 537)

Vasculitis syndromes
Primary vasculitis Secondary vasculitis syndrome
• Wegener’s granulomatosis • Drug induced vasculitis
• Churg-Strauss syndrome • Serum sickness
• PAN, HSP • Infection
• Malignancy
• Microscopic polyangitis
• Rheumatic disease
• Giant cell arteritis, Takayasu’s
arteritis
• Idiopathic cutaneous vasculitis
• Essential mixed cryoglobulinemia
• Behcet’s syndrome, Cogan’s
syndrome
• Kawasaki disease

113. Ans. (a) Lung; (c) Kidney; (d) Upper respiratory tract; (e)
Heart
(Ref: Robbins 7th/541, 9/e p511)
114. Ans. (a) Involves lungs; (b) Involves nose; (c) Involves
kidney; (d) Treated with cytotoxic agent and/or steroid.
(Ref: Robbins 7th/541, 9/e p511, Harrison 17th/2121)
Wegener’s granulomatosis is treated with steroids and
cyclophosphamide. They dramatically ameliorate glomerular
injury in pauci-immune glomerulonephritis.
115. Ans. (c) Seen in Henoch-Schonlein purpura
(Ref: Robbins 8th/920-921, 9/e p926)
116. Ans. (a) Kawasaki’s disease
(Ref: Robbin 7th/537, 9/e p506) ...see text
117. Ans. (a) Hypertension
(Ref: Robbins 9/e p509-510)
Polyarteritis Nodosa
• It is a systemic vasculitis manifested by transmural necrotizing
inflammation of small or medium sized muscular arteries.
• Renal artery is most commonly involved whereas Pulmonary circulation
is spared.
• Most common manifestations are malaise, fever of unknown cause,
weight loss, hypertension (rapidly developing), abdominal pain,
melena, diffuse muscular pains, and peripheral neuritis (predominantly
motor).

118. Ans. (c) Kawasaki disease


(Ref: http://emedicine.medscape.com/article/1006838, Robbins
9/e p510)
Clinically, infantile polyarteritis nodosa (IPAN) often is part of the
spectrum of Kawasaki disease (KD).
119. Ans. (b) Lung and kidney
(Ref: Robbins 9/e p511-512)
120. Ans. (a) Pulmonary artery
(Ref: Robbins 9/e p 509-510)
121. Ans. (c) Wegener’s granulomatosis
(Ref: Robbins 9/e p511-512, 8th/516-517; 7 th/541)
122. Ans. (b) Henoch-Schönlein purpura
(Ref: Robbins 8th/920-921, 9/e p926)
123. Ans. (c) Polyarteritis nodosa
(Ref: Robbins 8/e p512, 9/e p506) ...see Table 9.3
124. Ans. (c) Most commonly involved artery is abdominal aorta
(Ref: Robins 8/e p512-3, 9/e p507-508) ...see text
125. Ans. (a) Lung
(Ref: Robins 8/e p514-5, 9/e p509-510)
Polyarteritis nodosa (PAN) is characterised by necrotizing
inflammation typically involving renal arteries but sparing
pulmonary vessels.
126. Ans. (d) Involves large vessels
(Ref: Robbins 9/e p511)
Wegener’s granulomatosis is a small vessel necrotizing vasculitis.
...for details see text
127. Ans. (a) No change
(Ref: Robbins 8/e p518, 9/e p513)
Structural changes in the arterial walls are absent except late in
the course, when intimal thickening can appear.
128. Ans. None or (c) 40-60%
(Ref: Robbins 8/e p Heptinstall’s Pathology of the Kidney,
Volume 1 p463.)
I don’t know why they frame these type of questions.
Heptinstall writes…“In different series, the degree if renal involvement
in children with HSP (defined by the presence of hematuria) is
20-56% (overall 32%) whereas in adults, it is 49-78% (overall
59%)”.
You may choose your answer friends.
129. Ans. (b)Takayasu arteritis
(Ref: Internet, Robins 9/e p508)
Takayasu arteritis is the choosen answer as it is associated with
involvement of superior mesenteric artery. So, it may be
associated with abdominal angiitis.
130. Ans. (a)
Carotid body tumour
(Ref: Robbins 8/e p522)
Before we discuss the answer friends, a simple clarification that is to
be kept in mind;
Glomus tumor (also known as a glomangioma) is a rare benign
neoplasm arising from the glomus body and mainly found
under the nail, on the fingertip or in the foot. It DOES
NOTcontain glomus cells.
• A glomus cell (type I) is a peripheral chemoreceptor mainly located in
the carotid bodies and aortic bodies helping in regulation of the
breathing.
• Neoplasms of glomus cells are known as paraganglioma.
• The most common location of these tumors is within the adrenal
medulla, where they give rise to pheochromocytomas,
• Approximately 70% of extra-adrenal paragangliomas occur in the head
and neck region. Paragangliomas typically develop in two locations:
• Paravertebral paraganglia (e.g., organs of Zuckerkandl and, rarely,
bladder). Such tumors have sympathetic connections and are
chromaffin-positive, a stain that detects catecholamines.
• Paraganglia related to the great vessels of the head and neck, the so-
called aorticopulmonary chain, including the carotid bodies (most
common); aortic bodies; jugulotympanic ganglia; ganglion nodosum of
the vagus nerve; and clusters located about the oral cavity, nose,
nasopharynx, larynx, and orbit. These are innervated by the
parasympathetic nervous system and infrequently release
catecholamines
Important points about carotid body tumour
The carotid body tumor is a prototype of a parasympathetic
paraganglioma.
Arisesclose to or envelops the bifurcation of the common carotid artery.
The microscopic features include presence of nests (Zellballen) of round
to oval chief cells (neuroectodermal in origin) that are surrounded by
delicate vascular septae.
The chief cells stain strongly for neuroendocrine markers such as
chromogranin, synaptophysin, neuron-specific enolase, CD56, and CD57.
In addition, the supporting spindle-shaped stromal cells called
sustentacular cells are positive for S-100 protein.

131. Ans (a) Port wine stain


(Ref: Robbins 8/e p522, 9/e p516)
Sturge–Weber syndrome is usually manifested at birth by a port-wine
stain on the forehead and upper eyelid of one side of the face.
132. Ans (a) Bacterial infection
(Ref: Robbins 8/e p521, 9/e p516)
Pyogenic granuloma: Key points

• It is a benignQ tumor
• Type of capillary hemangiomaQ which it bleedsQ easily and is often
ulcerated.
• Is a rapidly growing pedunculated red nodule on the skin, or gingival or
oral mucosa;
• 1/3rd of the lesions develop after trauma

Also know that;


• Pregnancy tumor (granuloma gravidarum) is a pyogenic granuloma that
occurs infrequently (1% of patients) in the gingiva of pregnant women.
• These lesions may spontaneously regress post pregnancy) or undergo fibrosis;
• Some cases require surgical excision
• Recurrence is rare.

133. Ans. (c) Distal portion of digits


(Ref: Robbins 9/e p517)
• Glomus tumor is a biologically benign tumorQ
that arises from the modified
smooth muscle cells of the glomus body
• Glomus tumor is a biologically benign tumorQ that arises from the modified
smooth muscle cells of the glomus body
• Glomus body is a specialized arteriovenous anastomosis that is involved in
thermoregulationQ.
• It is an often exquisitely painful tumor
• Glomus tumors are most commonly found in the distal portion of the
digitsQ, especially under the fingernails.
• ExcisionQ is curative.

134. Ans. (a) Wegener’s granulomatosis


(Robbins 9th/511-2)
135. Ans. (b) Kawasaki disease
(Robbins 9th/510)
136. Ans. (d) Polyarteritis nodosa
(Robbins 9th/509)
137. Ans. (c) Cardiac involvement
(Robbins 9th/510)
138. Ans. (b) Takayasu arteritis
(Robbins 9th/508)
139. Ans. (b) Undergo spontaneous regression
(Robbins 9th/516)
Juvenile hemangiomas (“strawberry hemangiomas”) regress
abut cavernous hemangiomas do not regress spontaneously.
140. Ans. (c) Thrombocytopenia
(Robbins 9th/510)

ANNEXURE
Myocardial vessel spasm
1. Cardiac Raynaud

Excessive constriction of coronary arteries or myocardial arterioles may cause


ischemia, and persistent vasospasm can even cause myocardial infarction. High
levels of vasoactive mediators like endogenous epinephrine (in
pheochromocytoma) or exogenous chemicals (cocaine or phenylephrine) can
precipitate prolonged myocardial vessel contraction.

2. Takotsubo cardiomyopathy (broken heart syndrome)


Takotsubo cardiomyopathy is an ischemic dilated cardiomyopathy caused by
emotional stress.

3. Multifocal microinfarction
Small intramural vessel involvement is seen with microembolization, vasculitis,
or vascular spasm, for example, due to endogenous catechols (epinephrine) or
drugs (cocaine or ephedrine). This is called as Multifocal microinfarction. This
may even lead to takotsubo cardiomyopathy.

1. Which of the following is true about polyarteritis nodosa?


(NEET 2020 like pattern)
(a) It shows fibrinoid necrosis in large blood vessels
(b) It has ANCA positivity
(c) HBsAg is positive in 30% patients
(d) Affected individuals have involvement of pulmonary
circulation
Ans. (c) HBsAg is positive in 30% patients
(Ref: Robbins 9th/509-10
• Polyarteritis nodosa (PAN) is a systemic vasculitis of small- or medium-sized
muscular arteries, typically involving renal and visceral vessels but sparing
the pulmonary circulation.
• There is no association with ANCA.
• About 30% of patients with PAN have chronic hepatitis B and deposits
containing HBsAg-HBsAb complexes in affected vessels, indicating an
immune complex–mediated etiology in that subset.
• Histologically, there is transmural inflammation of the arterial wall with a mixed
infiltrate of neutrophils, eosinophils, and mononuclear cells, frequently
accompanied by fibrinoid necrosis.
• Characteristically, all stages of activity (from early to late) may coexist in
different vessels or even within the same vessel.

2. Irregular, bulky and friable vegetation in cardiac valve are a


finding of which of the following disorders?
(NEET 2020 like pattern)
(a) Infective endocarditis
(b) Rheumatic endocarditis
(c) Libman sack endocarditis
(d) Nonbacterial endocarditis
Ans. (a) Infective endocarditis
(Ref: Robbins 9th/560)
Direct lines…. “Vegetations on heart valves are the classic hallmark of
infective endocarditis (IE); these are friable, bulky, potentially
destructive lesions containing fibrin, inflammatory cells, and
bacteria or other organisms”.
3. Microscopic examination of the reperfused myocardium is
likely to have which of the following findings?
(NEET 2020 like pattern)
(a) Neutrophil swelling
(b) Waviness of fibres
(c) Contraction band necrosis
(d) Neutrophilic infiltration
Ans. (c) Contraction band necrosis
(Ref: Robbins 9th/546)
Reperfused infarcts are usually hemorrhagic because the vasculature
is injured during ischemia and there is bleeding after flow is
restored. Microscopic examination reveals that irreversibly
injured myocytes exhibit contraction bands, intensely
eosinophilic intracellular “stripes” composed of closely packed
sarcomeres.

Reperfusion injury with contraction band necrosis (arrows)


4. Fish mouth stenosis in rheumatic heart disease is due to
which of the following mechanisms?
(NEET 2019 like pattern)
(a) Calcification and fibrosis bridging across valvular
commissures
(b) Fibrinoid necrosis
(c) Acute inflammation leading to valvular damage
(d) Myxomatous degeneration of the valve
Ans. (a) Calcification and fibrosis bridging across valvular
commissures
(Ref: Robbins 9th e/p 559)
In rheumatic mitral stenosis, calcification and fibrous bridging across
the valvular commissures create “fish mouth” or “buttonhole”
stenoses.
5. Which of the following is included in essential major blood
culture criteria for infective endocarditis?
(NEET 2019 like pattern)
(a) Single positive culture of HACEK
(b) Single positive culture of coxiella
(c) Single positive culture of corynebacterium
(d) Both a & b
Ans. (a) Single positive culture of HACEK
(Ref: Robbins 9th e/p 561)
Major criteria for infective endocarditis include:
• Blood culture(s) positive for a characteristic organism or persistently positive
for an unusual organism
• Echocardiographic identification of a valve-related or implant-related mass or
abscess, or partial separation of artificial valve
• New valvular regurgitation
The causal organisms differ among the major high-risk groups.
• Endocarditis of native but previously damaged or otherwise
abnormal valves is caused most commonly (50% to 60% of cases) by
Streptococcus viridians
• Intravenous drug abusers and overall for 20% to 30% of
cases: S. aureus is the major offender
• Other bacterial causes include enterococci and the so-called
HACEK group (Haemophilus, Actinobacillus,
Cardiobacterium, Eikenella, and Kingella), all commensals in
the oral cavity.
• Prosthetic valve endocarditis is caused most commonly by
coagulase-negative staphylococci (e.g., S. epidermidis).
6. Which wall of heart is enlarged first in a patient with
McCallum patch?
(NEET 2019 like pattern)
(a) Left atrium
(b) Right atrium
(c) Left ventricle
(d) Right ventricle
Ans. (a) Left atrium
(Ref: Robbins 9th e/p 558)
Subendocardial lesions, perhaps exacerbated by regurgitant jets, can
induce irregular thickenings called MacCallum plaques, usually
in the left atrium. This is seen in patients with rheumatic heart
disease.
7. Characteristic feature of hypererophic obstructive
cadiomyopathy is:
(NEET 2019 like pattern)
(a) Asymmetric septal hypertrophy of septum
(b) Increased size of atria
(c) Increased size of ventricle
(d) Normal myofiber arrangement
Ans. (a) Asymmetric septal hypertrophy of septum
(Ref: Robbins 9th e/p 569)
• The essential feature of HCM is massive myocardial hypertrophy, usually
without ventricular dilation.
• The classic pattern involves disproportionate thickening of the ventricular
septum relative to the left ventricle free wall termed asymmetric septal
hypertrophy.
• The most important histologic features of HCM myocardium are massive
myocyte hypertrophy, myofiber disarray and interstitial and replacement
fibrosis.

8. A 50-year male presented with cutaneous vasculitis,


glomerulonephritis and peripheral neuropathy. Which of the
following investigations will help in establishing the
diagnosis?
(AI 2018 Pattern)
(a) P ANCA
(b) C ANCA
(c) Anti HCV antibody
(d) Anti HAV antibody
Ans. (c) Anti HCV antibody
(Ref: Robbins 9/e p927)
Essential mixed cryoglobulinemia is a systemic condition in
which deposits of cryoglobulins composed principally of IgG-IgM
complexes induce cutaneous vasculitis, synovitis, and a
proliferative glomerulonephritis, typically MPGN.

Most cases of essential mixed cryoglobulinemia have been


associated with infection with hepatitis C virus, and with
MPGN type I glomerulonephritis.

9. Stunning of myocardium without any acute coronary


syndrome is:
(AI 2018 Pattern)
(a) Subendocardial infarction
(b) Restrictive cardiomyopathy
(c) Takotsubo cardiomyopathy
(d) Transmural infarction
Ans. (c) Takotsubo cardiomyopathy
(Ref: Robbins 9/e p543)
When there is pathology involving only smaller intramural vessels, the
elevated levels of catechols exacerbate ischemia caused by the
vasospasm. This may lead to sudden cardiac death or an
ischemic dilated cardiomyopathy, so-called takotsubo
cardiomyopathy.
10. A teenager
has a progressively increasing soft reddish swelling on his
chest. Its histological appearance is given below. Which of the
following is the likely diagnosis?
(AI 2018 Pattern)
(a) Lipoma
(b) Hemangioma
(c) Fibroadenoma
(d) Fibroma
Ans. (b) Hemangioma
(Ref: Robbins 9/e p517)
Microscopic examination is showing increased number of blood
vessels which are filled with blood. This is seen with
hemangioma.
11. Which of the following indicates ‘flipping effect’?
(AI 2018 Pattern)
(a) LDH2>LDH1
(b) LDH1>LDH2
(c) LDH2>LDH3
(d) LDH2>LDH4
Ans. (b) LDH1>LDH2
(Ref: Robbins 9/e p544)
LDH1>LDH2 is seen with myocardial infarction as more amount of
LDH1 is released due to damage to myocardial cells.
12. A 30-year-old male presented with severe dyspnea. His
investigations showed mitral stenosis with left atrial
enlargement. The histopathology report from his mitral
valve is shown below. What is the likely diagnosis of these
patients?
(AIIMS Nov 2017 Pattern)
(a) Sarcoidosis
(b) Fungal granuloma
(c) Tuberculosis
(d) Rheumatic heart disease
Ans. (d) Rheumatic heart disease
(Ref: Robbins 9/e p558)

• The image given in the question is suggestive of Aschoff body. Aschoff


bodies, consisting of foci of T lymphocytes,occasional plasma cells, and
plump activated macrophages called Anitschkow cells (pathognomonic
for RF).
• In patient with mitral stenosis we would suspect rheumatic heart disease
as a suspected etiology.
• So, the diagnosis with both the history presented and the histological
finding is rheumatic heart disease.

13. An old man had severe chest pain. The patient died on the
way to the hospital. In the hospital, at autopsy tetrazolium
chloride staining of the heart was done. What will be the
color of viable myocardium?
(AIIMS May 2017 Pattern)
(a) Blue
(b) Dark brown
(c) Red
(d) Pink
Ans. (c) Red
(Ref: Robbins 9/e p544-5)
Triphenyltetrazolium chloride (TTC) is a gross histochemical
stain which imparts a brick-red color to intact, noninfarcted
myocardium where lactate dehydrogenase activity is
preserved. Since dehydrogenases leak out through the
damaged membranes of dead cells, an infarct appears as an
unstained pale zone.
Disclaimer
Any resemblance to an actual question is purely coincidental.
• Acinus is the functional unit of the lung.
• Clara cells are seen in Terminal bronchioles.
• Bronchogenic (pulomonary) sequestration is seen in posterior basal segment of
lower lobe of left lung. The sequestrated lobe of lung is supplied by aorta or its
branches.
• Characteristic histological finding of ARDS (shock lung) is Diffuse alveolar
damage.
• Part of airway involved in emphysema is distal to Terminal bronchioles.
• Smoking is the commonest cause of emphysema.
• Important points about asthma: hyper-responsive airways having Charcot-
Leyden crystals, curschmann’s spirals and creola bodies.
• Bronchiectasis (permanent dilation of airways) affects commonly lower lobe
bronchi of left lung. It does not lead to lung cancer.
• Hypersensitivity pneumonitis is type IV >>> type III hypersensitivity
reactions.
• Sarcoidosis is characterised by bilateral hilar lymphadenopathy (Potato nodes)
with non-caseating granuloma.
• Good pasture syndrome is characterized by focal necrotizing hemorrhagic
interstitial pneumonitis.
• Bacterial pneumonia is characterized by presence of alveolar exudates.
• Most common cause of lung abscess: Aspiration of oropharyngeal secretion.
• Most common lung carcinoma in India: Squamous cell carcinoma (associated
with p53 gene mutation). Its histological marker is cytokeratin.
• Lung to lung metastasis is seen in adenocarcinoma in situ (Bronchiolo-alveolar
carcinoma)
• Most common site of metastasis of lung carcinoma: Brain.
• Most common presentation of bronchial adenoma is recurrent hemoptysis.
• Pleural fibroma (benign mesothelioma) is characterized by: Intense fibrosis,
CD34 positive, keratin negative.
• Malignant mesothelioma is associated with exposure to asbestos. Its
characteristic microscopic finding includes long-branching microvilli.
• Most common mediastinal mass: Neurogenic tumors.
• “Airway remodeling” is associated with bronchial asthma.
• Pulmonary hypertension is defined as a mean pulmonary artery pressure
greater than or equal to 25 mm Hg at rest.
• Cough is the most common symptom of the bronchogenic carcinoma.
• The lung is the most common site of metastatic neoplasms.
• The NAB2- STAT6 fusion gene is virtually unique to solitary fibrous tumor.
• In about 80% of mesotheliomas, the most common is homozygous deletion of
the tumor suppressor gene CDKN2A/INK4a. The demonstration of this deletion
(usually by FISH) involving chromosome 9p can be very helpful in
distinguishing mesothelioma from reactive mesothelial proliferations.

ANATOMY OF RESPIRATORY TRACT

• Acinus is the functional unit of lung whereas alveoli are the chief sites of gaseous
exchange.
• Lobule is composed of 3-5 terminal bronchioles with their acini.

• Alveoli are lined by type I pneumocytes (forming 95% of alveolar surface) and type II
pneumocytes (responsible for secretion of surfactant and repair of alveoli after type I
pneumocyte destruction). The alveolar wall has the presence of pores of KohnQ for
allowing the passage of bacteria and exudate between adjacent alveoli.
• The entire respiratory tract is lined by pseudostratified, tall, ciliated columnar
epithelial cells except vocal cords (these have stratified squamous epithelium).
Broadly, the diseases of lung may be divided into infectious, obstructive, restrictive,
vascular and neoplastic etiologies.

INFECTIVE LUNG DISEASES


1. Pneumonia

Infection of the lung parenchyma is called pneumonia. It can be of two types

Pneumonia
Typical pneumonia Atypical pneumonia
• Infection caused by extracellular organisms • Infection caused by intracellular organisms like
mainly bacteria Mycoplasma, Chlamydia pneumoniae and
• Characterized by neutrophilic infiltration and viruses like RSV, influenza virus, rhinovirus.
presence of Intra-alveolar exudates (leading • Characterized by lymphocytic infiltration and
to consolidationQ). presence of alveolar septal and interstitial
• Clinical features include acute onset of high inflammation with absence of alveolar
grade fever and mucopurulent cough which exudatesQ.
may also be associated with pleuritic pain. • Clinical features include fever, headache, dry
cough and myalgia. Productive Cough and
pleural involvement is uncommonQ.

Viral pneumonia result in interstitial infiltrates (therefore called interstitial pneumonia) and may
result in variety of cytopathic effects. e.g. RSV shows bronchiolitis and multinucleate giant cells and
CMV and herpes show inclusion bodies.

Furthermore, typical pneumonia can be of two types:


Typical pneumonia
Lobar pneumonia Bronchopneumonia
• Consolidation of entire lobe usually caused by • Patchy consolidation in the lobe of lung.
Streptococcus pneumoniae. • Usually bilateral basalQ in location due to
• Following 4 stages of inflammation are present. gravitation of secretions.
1. Congestion: It is due to vasodilation. There is • Affects extremes of ageQ (infants or old).
bacteria rich intra-alveolar fluid. • Chest X-ray shows patchy opacification of
2. Red hepatization: Exudate is rich in RBC, the lobeQ.
neutrophils and fibrinQ.
3. Gray hepatization: Degradation of RBC and
fibrinosuppurative exudates
4. Resolution: Enzymatic degradation of
exudate and healing
• Chest X-ray show opacification of the entire lobe.

2. Lung Abscess
• Local suppurative process within the lung associated with necrosis of the lung tissue
is called lung abscess.
• It is most commonly caused by aspiration of infective material.
• Commonest etiological agent is Anaerobic bacteria of the oral cavity.Q

Causes of lung abscess


Septic
Aspiration Post pneumonic infection Post obstructive emboli Miscellaneous
• Most common • Infection caused by • Due to • Direct
causeQ. Staph aureus, klebsiella primary or hematogenous
• Right lower or type 3 secondary spread to lung
lobe is the most Pneumococcus cancer. from infection in
frequently • Usually basal, multiple esophagus or
Q
affected . and diffusely scattered. pleural cavity.

Clinical features: Fever, productive cough with large amount of sputum, chest pain,
weight loss and presence of clubbing of the fingers and toes.

Characteristic histologic feature: Suppurative destruction of lung parenchyma


within the central area of cavitation.
Complications include empyema, brain abscess or meningitis, pulmonary
hemorrhage and secondary amyloidosisQ.

3. Tuberculosis (Koch’s Disease)


Pulmonary tuberculosis is caused by droplet infection (coughing, sneezing etc) due to
Mycobacterium tuberculosis. It is a strict aerobic bacteria for which the reservoir of
infection is a human being with active tuberculosis. However, certain clinical conditions
can increase the risk of tuberculosis like diabetes mellitus, Hodgkin’s lymphoma, chronic
lung disease (particularly silicosis), chronic renal failure, malnutrition, alcoholism, and
immunosuppression.
False-negative Mantoux test False-positive Mantoux test
• Sarcoidosis • Infection by atypical mycobacteria
• Malnutrition • Previous vaccination with BCG
• Hodgkin disease
• Immunosuppression
• Fulminant tuberculosis

Pathogenesis

The bacteria enter macrophages by endocytosis and multiply there. The bacterial cell
wall glycolipid lipoarabinomannan blocks the fusion of the phagosome and
lysosome. After about 3 weeks of infection, the TH1 cells are stimulated by
mycobacterial antigens and these cells differentiate into mature TH1cells by the action of
IL-12.
The mature TH1 cells in the lymph nodes and lung produce IFN-g which activates
macrophages leading to, oxidative damage to the mycobacteria. Activated macrophages
produce TNF and recruit monocytes which then differentiate into the “epithelioid
histiocytes”, a characteristic feature of granulomatous inflammation. The immune
response is usually accompanied by hypersensitivity and tissue destruction.
Clinical Features
Primary tuberculosis

• It develops in a previously unexposed and unsensitized individual. The source of the organism is
usually exogenous. Most patients with primary tuberculosis develop latent disease while a minority
develops progressive infection.
• Primary tuberculosis almost always begins in the lungs leading to formation of a subpleural lesion
called Ghon’s foccus. During the first few weeks, there is also lymphatic and hematogenous
dissemination to other parts of the body.
• At times, occult hematogenous spread occurs in primary TB where the focus is then called Simon
focus.
• In majority of the people, development of cell-mediated immunity controls the infection. The Ghon’s
complex undergoes progressive fibrosis and calcification (detected radiologically and called as
Ranke complexQ).
Histologically
The sites of active disease show a characteristic granulomatous inflammatory reaction having the
presence of both caseating and non-caseating tubercles. There is also presence of Langhans giant cells
and lymphocytes.

Fig. 1: Pulmonary TB with caseous foci.


Secondary tuberculosis
• It is the pattern of disease that arises in a previously sensitized host. It usually results from a
reactivation of latent primary lesions after many years of an initial infection, particularly when host
immunity is decreased or uncommonly may follow primary tuberculosis. The preexistence of
hypersensitivity contributes to localization of the infection and increased incidence of cavitation. This
can lead to erosion into an airway (leading to spread of bacilli during coughing).

Progressive pulmonary tuberculosis

It is seen in the elderly and the immunosuppressed individuals. The apical lesion enlarges with increase
in the area of caseation. The erosion of blood vessels (particularly bronchial arteryQ) results in
hemoptysis. The pleural cavity is associated with pleural effusion or empyema. If the treatment is
adequate, the disease may be controlled but if it is inadequate, the infection may disseminate through
airways, lymphatics or the vascular system.

Miliary disease

It occurs when organisms drain through lymphatics and blood vessels to the different organs of the body
resulting in small yellow-white consolidated lesions. Miliary tuberculosis is most prominent in the liver,
bone marrow, spleen, adrenals, meninges, kidneys, fallopian tubes, and epididymis.

Important radiological info


• Early infraclavicular lesion - Assman Redeker SimonQ
• In post primary stage (late dissemination), coarse granular dissemination is called Aschoff Puhl
focus

The patients present with insidious onset of symptoms like low grade remittent fever
usually associated with night sweats, productive cough, weight loss, hemoptysis,
dyspnea and pleural effusion. The investigations usually reveal lymphocytosis and
increased ESR (on hemogram), hilar lymphadenopathy and pleural effusion (on chest X
ray), presence of acid fast bacilli with Ziehl Nielson staining. The treatment is provided
with multiple drugs (for details, refer to Review of Pharmacology by the same authors).

Pulmonary diseases
Obstructive lung disease Restrictive lung disease
• Characterized by increased resistance to • Characterized by decreased expansion of the lung.
airflow due to airway obstruction. • Spirometry reveals reduced total lung capacity
• Spirometry reveals ratio is and vital capacityQ.
decreasedQ. • Examples:
• Examples: Asthma, Emphysema, Chronic 1. Chest wall disorder-polio, obesity kyphoscoliosis.
bronchitis, Bronchiectasis. 2. Interstitial/infiltrative disease: Pneumoconiosis,
ARDS, Pulmonary fibrosis.

OBSTRUCTIVE LUNG DISEASES

1. Chronic Bronchitis

It is defined clinically as the presence of productive cough for at least 3 months in at


least 2 consecutive years in absence of any other identifiable cause.
The most important initiating agent is smokingQ resulting in airway irritation leading
to mucus hyper secretion; the latter may cause airway obstruction. Infection plays a
secondary role particularly in maintaining chronic bronchitis and is also responsible for
the acute exacerbations.
• Histologic features include chronic lymphocytic infiltration of the airways and submucosal
gland hypertrophy. There is also increase in Reid indexQ. The bronchial epithelium may also
have squamous metaplasia and dysplasia.

Clinical features: Late onset of dyspnea with productive cough (copious sputum),
recurrent infections, hypoxemia and mild cyanosis (BLUE BLOATERS). Long standing
chronic bronchitis can cause cor pulmonale (right sided heart failure due to pulmonary
hypertension).

2. Emphysema

It is abnormal permanent enlargement of the airspace distal to terminal bronchioles and


is associated with destruction of their walls. Characteristically, there is loss or reduction
of elastic recoil of the lung.
• Most important etiological agent for emphysema is smokingQ which causes
inflammation in airways resulting in increased neutrophils and macrophages. These
inflammatory cells release elastase responsible for destruction of lung tissue
resulting in emphysema.
• Normally, the pulmonary tissue destruction by elastase is prevented by the presence
of anti-elastase activity which is primarily due to α1-antitrypsin; a1-AT (mainly) and
serum a1- macroglobulin). So, any increase in neutrophils (usually in smokers) or
deficiency of a1–AT would contribute to development of emphysema.

Emphysema
Centriacinar Panacinar Distal acinar/Paraseptal Irregular

• Involvement of central • Involvement of the • Distal part of acinus is • Irregular acinar


part of acinus with entire acinusQ. affected with sparing of involvement
sparing of alveollQ. • Seen with a1-AT proximal part of associated with
deficiency. acinus Q. fibrosis/scarringQ.
Q
• Seen in smokers .
• Seen in smokersQ.
• Usually more severe • Occurs more • Involvement of lung • Most common type of
in upper lobesQ (due severely in lower adjacent to pleura. emphysema
to relative deficiency lobes at base of histologicallyQ.
of serum a1-AT to this lung (due to lower • Associated with the
Q

less perfused region). lung distribution of development of


neutrophils because spontaneous
• MC type of
of more perfusion of pneumothorax.
emphysema seen
clinically. this region).

Clinical features: Progressively increasing dyspnea, weight loss, late onset of


cough with scanty sputum. The patient is non-cyanotic, uses accessory muscle of
respiration and shows pursed lip breathing. (PINK PUFFERS).
Management: Cessation of smoking and use of bronchodilators is the mainstay of
the management.

3. Asthma
Hyperactivity of the airways resulting in reversible bronchoconstriction and air flow
obstruction on exposure to some external stimuli is called asthma.

Pathogenesis: Primary exposure of an allergen causes TH2 cell dominated


inflammatory response resulting in IgE production and eosinophil recruitment (called
sensitization). Exposure to the same allergen causes cross linking of IgE bound to IgE
receptors on mast cells in the airways which cause opening up of epithelial cells due to
released mediators. Antigens then cause activation of mucosal mast cells and
eosinophils and this along with neuronal reflexes (subepithelial vagal receptors) cause
bronchospasm, increased vascular permeability and mucus production (Acute or
Immediate response). Later on, leukocytic infiltration causes release of more mediators
and damage to the epithelium (Late Phase Reaction). Eosinophils in airways release
major basic protein which causes epithelial damage and more airway constriction.

Leukotrienes C4, D4, E4 and acetylcholine have definite role in bronchoconstriction


whereas agents like histamine, PGD2 and platelet activating factor (PAF) may also have
role in the features of the disease. The following are the two variants of asthma.
Features Extrinsic asthma Intrinsic asthma
Pathogenesis Type I hyper-sensitivity reaction due Initiated by non-immune
to exposure to an extrinsic antigen mechanisms with intrinsic
body stimuli
Age on presentation Child Adult
Family history PresentQ AbsentQ
Prior allergic reaction/allergen Positive history of rhinitis, urticaria, Absent
exposure eczema
Serum IgE levels IncreasedQ NormalQ
Skin test Positive Negative
Associated COPD Rare Usually present
Examples Atopic/allergic asthma, Occupational Drugs (most commonly
asthma, Allergic bronchopulmonary aspirin), viral infections, cold
aspergillosis exposure, exercise induced
asthma

IMPORTANT FACT
• IL-13 gene polymorphism is strongly associated with bronchial asthma.
• ADAM-33 is another gene causing proliferation of smooth muscle cells and fibroblasts in bronchi
resulting in bronchial hyper-reactivity and subepithelial fibrosis.
• ↑ serum YKL-40 is co-related with airway remodeling, and disease severity.

Clinical features: Acute asthmatic attack is characterized by wheezing, cough and


severe dyspnea.
Morphology: The most stroking macroscopic finding is occlusion of the bronchi and
bronchioles by mucus plugs. Histologically, there are numerous eosinophils, Charcot
leyden crystalsQ (composed of eosinophil membrane protein called as galectin-10)
and Curschmann spiralsQ (whorls of shed airway epithelium). Structural changes in the
bronchial wall called “airway remodelling” is characterized by presence of eosinophilic
inflammation and edema of bronchial walls, increased size of submucosal glands,
hypertrophy of bronchial wall smooth muscle and deposition of subepithelial collagen in
the bronchial wall. Individual epithelial cells present in the sputum of the patients are
called Creola bodies.
Management is done with bronchodilators and corticosteroids (for details, refer to
the Review of Pharmacology by the same authors).
4. Bronchiectasis

Abnormal permanent airway dilation resulting from chronic necrotizing infections is called
bronchiectasis.
Causes
Bronchial Congenital Necrotizing Miscellaneous
obstruction pneumonias
• Tumor • Cystic fibrosis • Mycobacterium • SLE
• Foreign body • Kartagener • Staph aureus • Rheumatoid arthritis
syndromeQ. • Aspergillus • Post
• Influenza virus. transplantation.

Obstruction and infection are the chief contributors to the damage of airway wall
associated with destruction of smooth muscle and elastic tissue fibrosis and further
dilatation of bronchi.
Clinical features: Chronic cough, fever, foul smelling sputum production, recurrent
pulmonary infections, sinusitis and immune deficiencies.
It usually affects vertical air passages of the lower lobes bilaterally with involvement
of left sideQ more frequent than right. The dilated bronchi can be followed directly out to
the pleural surfaces. There is usually presence of inflammatory cells in the walls of
bronchi and bronchioles which may also exhibit squamous metaplasia.
Fig. 2: Bronchiectasis: dilated bronchi reach pleural surface.

RESTRICTIVE LUNG DISEASE

1. Interstitial Lung Disease

Interstitial lung disease (ILD) is a group of heterogeneous diseases characterized by


chronic inflammation and fibrosis of the interstitium and lung parenchymaQ. The
interstitium of the lung (supporting structure) is the area in and around the small blood
vessels and alveoli where the exchange of oxygen and carbon dioxide takes place.
Inflammation and scarring of the interstitium (and eventually extension into the alveoli)
will disrupt normal gas exchange. Usually, patients present with exertional dyspnea and
non-productive cough.

The chest X-ray shows reticular or reticulonodular pattern (“ground glass”


appearance). Pulmonary function tests show evidence of intrapulmonary
restrictive pattern.

Earliest manifestation of the disease is inflammation in the alveolar wall called


Alveolitis. The inflammatory cells release chemical mediators resulting in injury to
parenchymal cells and stimulation of fibrosis. In advanced stages of the disease, the
gross destruction and scarring of the lung results in end stage disease or honeycomb
lung.Q
ILD
Fibrosis Granulomatous Associated with Miscellaneous
smoking
• Idiopathic pulmonary fibrosis • Sarcoidosis • Desquamative • Pulmonary
• Collagen vascular disease • Hypersensitivity interstitial alveolar
associated pulmonary fibrosis pneumonitis. pneumonia proteinosis.
(SLE, RA, Scleroderma)
• Pneumoconiosis
• Drug reaction
• Radiation pneumonitis
• Fumes and gases (SO2 etc.)

We shall focus on each of the chief subtypes of interstitial lung disease:

A. PNEUMOCONIOSIS
Non-neoplastic lung reaction (usually fibrosis) to inhalation of mineral dust, organic and
inorganic particles and chemicals and vapors is called pneumoconiosis.

These particles overwhelm the normal phagocytosis by alveolar macrophages to


evoke fibroblast proliferation and collagen deposition. Some of the important
pneumoconiosis includes:

(1) Coal worker’s pneumoconiosis (Anthracosis)


It is due to inhalation of coal dust in coal miners and is usually present adjacent to
respiratory bronchiole. It is categorized into:
Simple CWP:

(i) Coal macules (carbon-laden macrophages)


(ii) Coal nodules (upper lobes more heavily involved)
(iii) Centrilobular emphysema

Complicated CWP: Develops after many years

(i) Intense blackened scars larger than 2 cm in diameter


(ii) Center of lesion is often necrotic

(2) Silicosis (Grinder’ disease)

• Nodular fibrosing diseaseQ due to inhalation of silica in workers engaged is sandblasting, hard
rock mining, foundry work, glass and pottery making.
• Most common chronic occupational disease in the worldQ.
• Crystalline form of silica called quartz is most commonly implicated in silicosis.
• Co-inhalation of other mineral particles reducesQ the fibrogenic effect of silicosis.
• Involvement of upper lobes of the lung.Q
• Association with increased susceptibility to tuberculosisQ.
• Chest X-ray shows presence of Eggshell calcificationQ (thin sheet of calcification in the lymph
node surrounding a zone lacking calcification).
• Polarized microscopy reveals presence of birefringent silica particlesQ.

(3) Asbestosis

Fig. 3: Ferruginous Body (F) and foreign body giant cell (G) in Asbestosis.

• Diffuse interstitial fibrotic disease due to inhalation of asbestos particles in workers


engaged in mining, pipes, brakes, insulation and boilers.
• Initially, involvement of lower lobes of the lung pleurallyQ.
• In contrast to other dusts, can also act as a tumor initiator and tumor promoterQ.
Two types of fibers are:
• Serpentine (curly and flexible fibres, chrysotile): These account for most of the
asbestos used in industry.
• Amphibole (straight, stiff and brittle fibres, crocidolite, amosite, actinolyte): These are
more pathogenic than chrysotiles, particularly with respect to induction of malignant
pleural tumors (mesotheliomas).
Lesions with asbestosis

Pleural plaqueQ:
It is the most common manifestation of asbestos exposure and is composed of well circumscribed
plaques of dense collagen containing calcium. They are usually asymptomatic and develop on anterior
and posterolateral parts of parietal pleura and over the diaphragm.
Interstitial fibrosis

Pulmonary fibrosis with the presence of asbestos bodyQ (iron containing proteinaceous material coating
asbestos fiber) and ferruginous bodyQ (iron protein complex coating other inorganic particles like talc,
mica, fibre, glass and other less common materials in the lung). True asbestos bodies are clear whereas
the core of these particles is dark).
Bronchogenic cancer (latent period is 10-30 years)

Most common cancer associated with asbestosQ whose risk is increased with concomitant smoking.
Mesothelioma (latent period is 25-45 years)
Localization of asbestos fibres in the lung close to the mesothelial layers increases the risk of
development of pleural and peritoneal mesothelioma. Concomitant smoking does not increase the risk
of mesothelioma. It is the most specific cancer associated with asbestos inhalation.
Pleural effusion, laryngeal and colon cancers.

Clinical symptoms of pneumoconiosis include progressively increasing dyspnea


(First symptom), productive cough, reduced exercise tolerance which may progress to
respiratory failure, cor pulmonale and ultimately death.

B. SARCOIDOSIS
It is a systematic disease of unknown etiology characterized by the presence of non-
caseating granulomas in many organs. It is seen more commonly in females of 20-40
years of age. It is associated with HLA-A1 and HLA-B8.
Fig. 4: Sarcoidosis with Non Caseating Granuloma (G); Inset shows Schaumann body.

Immunological abnormalities associated with sarcoidosis

• Intra-alveolar and interstitial accumulation of CD4 + T cells resulting in CD4:CD8 T cell ratio
ranging from 5:1 to 15:1Q.
• Increased levels of IL-2 and IFN-g causing T-cell expansion and macrophage activation
respectivelyQ.
• Polyclonal hypergammaglobulinemiaQ.
• Anergy to skin antigens like purified protein derivative (PPD)Q.

Histologically, there is characteristically presence of non-caseating granulomaQ


composed of aggregates of epithelioid cells and giant cells. There is also presence of
Schaumann bodiesQ (laminated concretions of calcium and protein) and asteroid bodiesQ
(stellate or star shaped inclusions in giant cells).

Organs Affected

Lungs
• Most common site of organ involvement
• There is presence of non-caseating granuloma in the bronchial submucosa.
• Bronchoalveolar lavageQ shows CD4:CD8 T- lymphocytes ratio of > 2.5 is seen.

Lymph nodes

• Involvement of hilar and mediastinal nodes is seen in almost all the cases.
Liver and spleen

• Splenomegaly may be seen with sparing of capsule. Scattered granulomas are seen more in portal
triads as compared to globular parenchyma.

Bone marrow

• Favored site of localization having tendency to involve phalangeal bones of hands and feet showing
small areas of bone resorption, bony shaft widening and new bone formation.

Skin

• Lesions include erythema nodosum, subcutaneous nodules, erythematous plaques and lupus
pernioQ.
Eye, lacrimal glands and salivary glands

• Unilateral or bilateral ocular involvement resulting in iritis, glaucoma or corneal opacity may occur.
• Causes lacrimal gland inflammation (causing dry eyesQ) and salivary gland involvement (dry
mouthQ).

Clinical features include shortness of breath, cough, chest pain, hemoptysis,


constitutional signs and symptoms (fever, fatigue, weight loss) or it may be discovered
on routine X-ray as bilateral hilar lymphadenopathy. It can also manifest as Lofgren
Syndrome and Heerfordt Waldenstrom Syndrome.
• Chest X-ray shows bilateral hilar and left paratracheal lymphadenopathy. There is
hypercalcemiaQ due to increased circulation of vitamin D by macrophages.
Elevated levels of ACEQ (Angiotensin Converting Enzyme) are seen in the disease.
These patients also demonstrate skin anergy and a restrictive pattern on
pulmonary function tests.
• Management: It is done usually with corticosteroids.

C. PULMONARY ALVEOLAR PROTEINOSIS


Contd...
Contd...
It is a rare disease in which phospholipids accumulate within alveolar spaces. This is a
condition of unknown cause characterized by auto-antibodies against granulocyte
macrophage-colony stimulating factor (GM-CSF).
Some (10%) PAP cases are congenital in nature secondary to mutations in the GM-CSF
gene leading to reduced or absent SP-B and intra alveolar accumulation of SP-A and
SP-C.
Secondary PAP is associated with immunodeficiency, hematopoietic disorders,
malignancies, acute silicosis and other inhalational syndromes.
Clinical features: Progressive dyspnea is the usual presenting symptom. Chest
X-ray shows bilateral alveolar infiltrates suggestive of pulmonary edema.
Bronchoalveolar lavage shows characteristic milky appearance and PAS-positive
lipoproteinaceous material. Lung biopsy reveals amorphous intra-alveolar
phospholipids.

2. Adult Respiratory Distress Syndrome (ARDS)

• Also called as Shock lungQ, Diffuse alveolar damageQ or Acute lung injuryQ
• Characterized by damage to alveolar cells and blood vessels resulting in oxygen
refractory progressive respiratory insufficiency.
Conditions associated with ARDS
Infections Physical factors Chemical factors Miscellaneous
• Septicemia • Drowning • Drugs like aspirin • Pancreatitis
• Aspiration • Head injury • Heroin/Methadone • Uremia
• Pulmonary infections • Radiation • Overdose of • Multiple transfusion
(TB, viral, exposure barbiturates • DIC.
mycoplasma, etc.) • Fat embolism • Hypersensitivity by
• Smoking organic solvents.
• Irritant gases.
Morphological features include Interstitial and intra-alveolar edema with lining of
alveoli with hyaline membrane composed of fibrin rich edema fluid and lipid remnants of
epithelial cells. Later on, there is intra-alveolar fibrosis.
Clinical features include Progressively increasing dyspnea (difficulty in breathing),
tachypnea resulting in respiratory failure, cyanosis and hypoxemia, chest -ray shows
bilateral diffuse infiltration (“white out” lungQ). Management is done primarily with
treatment of underlying cause and mechanical ventilation.
Neonatal RDS/Hyaline membrane disease (HMD)
Prematurity Infants of diabetic mothers Cesarean section

• Results from surfactant deficiency which is chemically composed of lecithin


(dipalmitoylphosphatidylcholine; DPPCQ). Due to reduced surfactant, surface tension
of alveoli increases. Increased alveolar surface tension causes atelectasis resulting
in hypoxemia responsible for damage to endothelial and epithelial cells. The latter
contributes to formation of hyaline membraneQ (fibrin + necrotic cells).

• Clinical features include normal infant at birth but within 30 minutes, there is development of
progressively increasing respiratory effort and cyanosis. Chest X-ray demonstrates multiple
reticulogranular densities (“ground glass” appearanceQ).
VASCULAR LUNG DISEASES

1. Pulmonary Emboli and Pulmonary Infarction

Most (90-95%) of the pulmonary emboli arise from deep vein thrombosis (DVT) in the
leg and only 10% of pulmonary emboli cause infarction. The infarcts occur in patients
with underlying cardiopulmonary disease. It is a wedge shaped hemorrhagic
infarction. The diagnosis of pulmonary embolism is made on ventilation/perfusion scan
(V/Q lung scan) which shows a mismatch. The complications associated with this
condition include pulmonary hypertension, cor pulmonale, pulmonary abscess and even
sudden death.

2. Pulmonary Hypertension

It is defined as increased pulmonary artery pressure, usually due to increased vascular


resistance or blood flow. It is seen in association with:
• COPD and interstitial disease (hypoxia induced vasoconstriction is seen).
• Multiple ongoing pulmonary emboli.
• Mitral stenosis and left sided heart failure.
• Congenital heart disease with left to right shunts (ASD, VSD, PDA).
• Primary (idiopathic).

Pulmonary hypertension is characterised by increased medial hypertrophy and intimal fibrosis in


small arteries.

The presence of plexogenic pulmonary arteriopathy may also be seen in primary


pulmonary hypertension or congenital heart disease with left to right shunts.

3. Pulmonary Edema

It is defined as the fluid accumulation within the lungs usually due to disruption of starling
forces or endothelial injury.
So, it can be due to:
1. Increased hydrostatic pressure as in left-sided heart failure, mitral valve
stenosis, fluid overload, etc.
2. Decreased oncotic pressure as in nephrotic syndrome, or liver disease.
3. Increased capillary permeability as in infections, drugs (bleomycin, heroin),
shock, radiation.

The lungs are wet and heavy and the fluid accumulation is more in the lower lobes.
Microscopically, there is presence of intra-alveolar fluid, engorged capillaries and
hemosiderin-laden macrophages (heart-failure cells).

NEOPLASTIC LUNG DISEASES

1. Bronchogenic Cancer

The lung is a common site of metastatic neoplasms. However, bronchogenic cancer is


the most common primary malignant tumor of the lung. It is most frequently diagnosed
major cancer in the world.

Risk factors
Non-genetic Genetic
• Tobacco smoking (contain chemicals like Mutations affecting
benzopyrene and polycyclic • Tumor suppressor genes p53 and Rb gene
• Air pollution • Oncogenes
• Occupational exposure (asbestosis, uranium • K-ras – Adenocarcinoma
mining, radiation, etc.) • L-myc – Small cell carcinoma.

Histological variants of lung cancer


Squamous cell Small cell cancer or oat
Adenocarcinoma Large cell cancer
carcinoma cancer

• MC type of lung • Overall MC type of • Associated with smoking


cancer in smokersQ. cancerQ.

• MC type in malesQ. • MC type in non- • Commoner in smokersQ.


smokers and
femalesQ.
• Usually central in • Usually peripheral in • Central in location • Peripheral in
location (arise from location (arise from location.
terminal bronchiole)
the segmental
bronchi)
• Shows highest • Associated with K-ras • Immunohistochemistry
frequency of p53 mutationQ. shows high expression
Q
mutation . of Bcl-2 geneQ in
majority of tumors.
• Intercellular bridges • Glandular pattern of • Cells have scanty • Cells have large
or junction is very growth of the tumor is cytoplasm, small nuclei, prominent
specific. seen. nucleoli, granular nucleoli and a
• Histologically, the • Cells are positive for chromatin (salt and moderate amount of
tumor has presence mucin, thyroid pepper patternQ). cytoplasm.
Q
of keratinization . transcription factor-1 • Azzopardl effect
(TTF-1)Q and (Basophilic staining of
Napsin-A (sensitive vascular walls) is
as well as specific frequently present.
marker) • Electron microscopy
shows presence of
neurosecretory granules
chromogranin,
synaptophysis and leu-
7Q.
• Highest association • Has best response to
with cavitation chemotherapy and
• Has the best radiotherapy.
prognosis • Has the worst
prognosisQ.
• Most aggressive lung
cancerQ.

• HypercalcemiaQ • Associated with • Has gynecomastia


due to PTHrP is the maximum as paraneoplastic
MC paraneoplastic paraneoplastic syndromeQ.
syndrome. syndromeQ (particularly
SIADH and Cushing
syndrome).

Fig. 5: Squamous Cell Cancer with Keratin Pearls.


Horner syndrome

Horner syndromeQ is caused due to compression of sympathetic nerve plexus by an apical tumor
called as Pancoast tumorQ. It is usually an adenocarcinoma. Its components are remembered by
the mnemonic: Punjabi MEAL (see box on the side).

Clinical features: Cough is the most common symptom in these patients which is
followed by weight loss and dyspnea. They also have anorexia, fatigue, hemoptysis,
and chest pain.
Metastasis of the cancer causes involvement of adrenal (most commonlyQ)
followed by liver, brain and bone. Intrathoracic spread of the cancer causes enlargement
of lymph nodes (hilar, mediastinal, bronchial and tracheal), pleural involvement,
hoarseness (recurrent laryngeal nerve invasion), dysphagia (esophageal obstruction),
diaphragmatic paralysis (phrenic nerve paralysis), Horner syndrome and superior vena
cava (SVC) syndrome.
Important paraneoplastic syndromes associated with bronchogenic cancer
include:
1. Endocrinological syndrome
a. Cushing syndrome (Due to ACTH)
b. Syndrome of inappropriate ADH secretion (SIADH) [Due to anti-diuretic
hormone]
c. Hypercalcemia: Due to parathyroid hormone related peptide (PTH related
peptide).
d. Hypocalcemia: Due to calcitonin
e. Gynecomastia: Due to gonadotropins
2. Lambert eaton syndrome: Due to autoantibodies against neuronal calcium channel
3. Acanthosis nigricans: Hyperpigmentation of axillary region.
4. Hypertrophic pulmonary osteoarthropathy having clubbing and periosteal new
born formation.

2. Solitary (Localized) Fibrous Tumor

It was earlier referred to as ‘benign mesothelioma’. It has got no relationship with


asbestosisQ. Microscopically, there is presence of whorls of reticulin and collagen fibers
with interspersed spindle cells resembling fibroblasts. The tumor cells characteristically
show immunostaining pattern of CD34(+) and keratin-negativeQ.
3. Malignant Mesothelioma

It is a tumor arising from visceral or parietal pleura which is seen after prolonged duration
(after a latent period of 25-45 years) of asbestos inhalation. It is not associated with
smoking. Unlike bronchogenic cancer, it is less commonly associated with p53 mutation.
It is associated with extensive pleural effusion and local invasion of thoracic structures.
Microscopically, the tumor can have the following patterns:
1. Sarcomatoid type: Mesenchymal stromal cell type
2. Epithelioid type: Consists of cuboidal, columnar or flattened cell forming tubular or
papillary structure resembling adenocarcinoma.
3. Mixed type: It contains both epithelioid and sarcomatoid patterns.

Difference between mesothelioma and adenocarcinoma


Epithelioid mesothelioma Pulmonary adenocarcinoma
• Acid mucopolysaccharide staining (+) • Acid mucopolysaccharide staining (–)
• Keratin (+) • Keratin (–)
• CEA (–) • CEA (+)
• Electron microscopy shows long, slender, • Electron microscopy shows short microvilli
numerous microvilli and tonofilaments and secretory rough endoplasmic reticulum

Clinical features include chest pain, dyspnea and recurrent pleural effusions. Right
lung is more commonly affected than left lung. It is usually unilateral at presentation.
The lung is invaded directly, and there is often metastatic spread to the hilar lymph
nodes and, eventually to the liver and other distant organs. It is associated with very poor
prognosis.

ATELECTASIS

Incomplete expansion of the lungs or the collapse of previously inflated lung is known as
atelectasis.
1. Resorption Atelectasis:
– Due to airway obstruction leading to resorption of oxygen trapped in the alveoli.
– Causes mediastinal shift towards affected lung.
– Associated with chronic bronchitis/asthma/aspiration of foreign body/secretions.
2. Compression Atelectasis:
– Due to presence of fluid, blood, air or tumor in pleural space.
– Causes mediastinal shift away from the affected lung.
– Most commonly associated with cardiac failureQ.
3. Contraction Atelectasis:
– Fibrosis in the lung or pleura preventing full expansion of pulmonary tissue.
– Only irreversible cause of atelectasisQ.

Mediastinal tumors
Anterior Mediastinum Middle Mediastinum Posterior Mediastinum

• Thymoma (commonest) • Bronchogenic cyst • Neurogenic tumors like


(commonest) schwannoma, neurofibroma
(commonest)

• Teratoma • Pericardial cyst • Lymphoma

• Lymphoma • Lymphoma • Metastatic tumor (most are


from the lung)

• Thyroid lesions • Bronchogenic cyst

• Parathyroid tumors • Gastroenteric hernia

• Metastatic carcinoma
INFECTIVE LUNG DISEASE: PNEUMONIA, TB, LUNG ABSCESS

1. Infraclavicular lesion of tuberculosis is known as:


(a) Gohn’s focus
(AIIMS May 2011)
(b) Puhl’s focus
(c) Assman’s focus
(d) Simmon’s focus
2. Pulmonary tuberculosis is more common in following associated diseases,
except:
(DPG 2011)
(a) Acquired immune deficiency syndrome
(b) Diabetes
(c) Chronic renal failure
(d) Mitral stenosis
3. All of the following features are seen in the viral pneumonia except:
(AI 2005)
(a) Presence of interstitial inflammation
(b) Predominance of alveolar exudate
(c) Bronchiolitis
(d) Multinucleate giant cells in the bronchiolar wall
4. Atypical pneumonia can be caused by the following microbial agents except?
(AI 2005)
(a) Mycoplasma pneumoniae
(b) Legionella pneumophila
(c) Human corona virus
(d) Klebsiella pneumoniae
5. In primary tuberculosis, all of the following may be seen except:
(AI 2002)
(a) Cavitation
(b) Caseation
(c) Calcification
(d) Langhans giant cell
6. Lung granuloma with necrosis is seen in:
(a) PAN
(PGI June 01)
(b) TB
(c) Histoplasmosis
(d) Cryptococcosis
(e) Wegener’s granulomatosis
7. Predisposing factors of lung abscess are:
(a) Altered sensorium
(PGI Dec 2003)
(b) Dental sepsis
(c) Aggressive treatment of pneumonia
(d) Subpulmonic effusion
(e) Endobronchial obstruction
8. Pulmonary, renal syndrome is seen in:
(a) Goodpasture syndrome
(PGI Dec 2003)
(b) Leptospirosis
(c) Legionella
(d) Wegener’s granulomatosis
(e) Hantan virus infection
9. Lung granuloma found in A/E:
(PGI June 2004)
(a) Berylliosis
(b) Asbestosis
(c) SLE
(d) Sarcoidosis
10. True about Ghon’s focus:
(PGI Dec 2004)
(a) Left apical parenchymal lesion
(b) Right apical parenchymal lesion
(c) Subpleural caseous lesion in right upper lobe
(d) Subpleural caseous lesion just above or below interlobar fissure
(e) Caseous hilar lymphadenopathy
11. Which of these is seen in primary tuberculosis:
(a) Ghon’s focus
(PGI Dec 2006)
(b) Pleural effusion
(c) Miliary mottling
(d) Fibrosis
(e) Cavity
12. Characteristic histopathological feature of pneumocystis carinii pneumonia:
(PGI Dec 2000)
(a) Interstitial pneumonitis
(b) Increased eosinophils
(c) Foamy vacuolated exudates
(d) Mononuclear cell in bronchoalveolar lavage
(e) Neutrophil infiltration
13. A 9-year-old girl Bandhini developed a 10 mm area of induration on the left
forearm 72 hours after intradermal injection of 0.1 ml of purified protein
derivative (PPD). Which of the following is most likely to be seen on the X-ray
of this patient?
(a) Marked hilar adenopathy
(b) Upper lobe calcifications
(c) No abnormal findings
(d) Reticulo-nodular densities

MOST RECENT QUESTIONS

14. In the stage of Grey hepatisation, which of the following is a finding?


(a) WBC’s fill the alveoli
(b) RBC’s fill the alveoli
(c) Organisms fill the alveoli
(d) Accumulation of fibrin
15. Gray hepatization of lungs is seen on day:
(a) 1
(b) 2-3
(c) 3-4
(d) 5-7
16. Collapse of lung is called:
(a) Emphysema
(b) Bronchiactasis
(c) Atelectasis
(d) Bronchitis
17. Reactivated TB is most commonly located near:
(a) Apex
(b) Near bronchus
(c) Subpleurally
(d) Base
18. Maximum smooth muscle relative to wall thickness is seen in
(a) Terminal bronchiole
(b) Trachea
(c) Bronchi
(d) Respiratory bronchioles
19. The earliest feature of tuberculosis is:
(a)Caseation
(b) Recruitment of lymphocytes
(c) Formation of giant cells (Langhans)
(d) Granuloma formation
20. ESR is a very critical investigation is the diagnosis of TB. Which of the
following is true about ESR in TB?
(a) No change is ESR
(b) Confirms recovery from TB
(c) ESR is raised because of increased RBC aggregate
(d) ESR is raised due to decreased RBC size
21. The alveoli are filled with exudates the air is displaced converting the lung
into a solid organ this description suggests:
(a) Chronic bronchitis
(b) Bronchial asthma
(c) Bronchiectasis
(d) Lobar pneumonia
22. Azoospermia seen in which syndrome-
(a) Kartagener syndrome
(b) Young syndrome
(c) Churg strauss syndrome
(d) Both a and b
OBSTRUCTIVE LUNG DISEASE: BRONCHITIS, ASTHMA, BRONCHIECTASIS,
EMPHYSEMA

23. Increased Reid’s index is increased in which of the following?


(AI 2012)
(a) Bronchiectasis
(b) Bronchial asthma
(c) Chronic bronchitis
(d) Emphysema
24. True about alpha-1 antitrypsin deficiency, is/are:
(a) Autosomal dominant
(PGI June 01)
(b) Pulmonary emphysema
(c) Diastase resistant hepatic cells
(d) Hepatic cells are orcein stain positive
(e) Associated with berry aneurysm
25. Late response in bronchial asthma is due to:
(a) Mast cells
(UP 2003)
(b) Eosinophils
(c) Neutrophils
(d) Macrophages
26. Charcot-Leyden crystals and Curschmann’s spirals are seen in:
(UP 2006)
(a) Bronchial asthma
(b) Chronic bronchitis
(c) Bronchiectasis
(d) Emphysema
27. Most common type of emphysema clinically is:
(a) Panacinar
(RJ 2006)
(b) Centriacinar
(c) Paraseptal
(d) Segmental
28. In a heavy smoker with chronic bronchiolitis, which of the following is likely
to be seen:
(Kolkata 2003)
(a) Centrilobular emphysema
(b) Panacinar emphysema
(c) Paraseptal emphysema
(d) None of the above
29. A 30-year-old woman Chinamma has had increasing dyspnea with cough for
the past week. Over the past 2 days she is having productive cough with
copious sputum. On examination, she is afebrile but has extensive dullness
to percussion over all the lung fields. Her chest X ray has B/L diffuse
opacification. Electron microscopic examination of the biopsy tissue shows
many lamellar bodies. The antibody is directed against which of the following
substances in the pathogenesis of the above described condition?
(a) CFTR
(b) Granulocyte – macrophage colony stimulating factor
(c) DNA topoisomerase 1
(d) Glomerular basement membrane
30. A 50-year-old man Shahid K. John has had increasing dyspnea for the past 3
years with associated occasional cough but little sputum production.
Auscultation reveals that his lungs are hyper-resonant and is associated with
expiratory wheeze. Pulmonary function tests reveal increased total lung
capacity (TLC) and slightly increased FVC. There is decreased FEV1 and
FEV1/FVC ratio also. ABG analysis reveals pH of 7.35, pO2 of 60 mm Hg and
pCO2 of 48 mm Hg. What is the most likely diagnosis?
(a) Sarcoidosis
(b) Centriacinar emphysema
(c) Diffuse alveolar damage
(d) Chronic pulmonary embolism
31. A 65-year-old smoker Sutta Ram with hemoptysis and weight loss undergoes
a left upper lobectomy for squamous cell carcinoma. The uninvolved lung
tissue shows destruction of the alveolar septae around the respiratory
bronchioles, with marked enlargement of the airspaces. Anthracotic
pigments deposited heavily in the walls of these tissues. These findings are
most compatible with:
(a) Asthma
(b) Chronic bronchitis
(c) Emphysema
(d) Pulmonary hypertension

MOST RECENT QUESTIONS

32. Alpha-1-antitrypsin deficiency occurs in:


(a) Emphysema
(b) Bronchiectasis
(c) Empyema
(d) Bronchogenic carcinoma
33. Thickening of pulmonary membrane is seen in:
(a) Asthma
(b) Emphysema
(c) Bronchitis
(d) Bronchiectasis
34.Creola bodies are seen in:
(a) Bronchial asthma
(b) Chronic bronchitis
(c) Emphysema
(d) Bronchiectatsis
35. Distension of distant alveoli is seen in:
(a) Irregular emphysema
(b) Paraseptal emphysema
(c) Panacinar emphysema
(d) Centriacinar emphysema
RESTRICTIVE LUNG DISEASE: ILD, ARDS, PNEUMOCONIOSIS

36. Which of the following is the characteristic feature of adult respiratory


distress syndrome?
(AI 2012)
(a) Diffuse alveolar damage
(b) Interstitial tissue inflammation
(c) Alveolar exudates
(d) Interstitial fibrosis
37. All are true about phagocytosis except:
(AI 2011)
(a) Size of the particle ingested is less than 0.5 micrometer
(b) Size of the particle ingested is more than 0.5 micrometer
(c) Combines with lysosome forming phagolysosome
(d) Amoeba and other unicellular organisms make their living out of it
38. The following does not occur with asbestosis:
(a) Methaemoglobinemia
(DPG 2011)
(b) Pneumoconiosis
(c) Pleural mesothelioma
(d) Pleural calcification
39. Ferruginous bodies are seen in:
(AI 2008)
(a) Silicosis
(b) Byssinosis
(c) Asbestosis
(d) Bagassosis
40. All of the following are seen in asbestosis except:
(a) Diffuse alveolar damage
(AI 2002)
(b) Calcify pleural plaques
(c) Diffuse pulmonary interstitial fibrosis
(d) Mesothelioma
41. Which of the following is characteristically not associated with the
development of interstitial lung disease?
(AIIMS May 2006)
(a) Organic dusts
(b) Inorganic dusts
(c) Toxic gases, e.g. chlorine, sulphur dioxide
(d) Inhalation of tobacco smoke
42. All of the following features are seen in asbestosis except:
(AIIMS Nov 2002)
(a) Diffuse pulmonary interstitial fibrosis
(b) Fibrous pleural thickening
(c) Emphysema
(d) Calcific pleural plaques
43. Asbestosis of the lung is associated with all of the following except:
(AIIMS May 2002)
(a) Mesothelioma
(b) Progression of lesion even after stopping exposure to asbestos
(c) Nodular lesions involving upper lobe
(d) Asbestos bodies in sputum
44. Which of the following is associated with hypersensitive pneumonitis?
(AIIMS May 2002)
(a) Silicosis
(b) Asbestosis
(c) Byssinosis
(d) Berylliosis
45. End stage lung disease is seen in:
(PGI June 2004)
(a) Sarcoidosis
(b) Interstitial lung disease
(c) Langerhan’s cell histiocytosis
(d) Aspergillosis
(e) Asbestosis
46. Features seen in bronchiolitis obliterans with organizing pneumonia include:
(PGI Dec 2001)
(a) Polypoid plugs in bronchioles
(b) Ulceration and exudation of epithelium into the lumen
(c) Exudation of proteinaceous material in terminal airways
(d) Bronchoconstriction
(e) Response to steroids
47. Which of the following inhaled occupational pollutant produces extensive
nodular pulmonary fibrosis?
(a) Silica
(Delhi 2009 RP)
(b) Asbestos
(c) Wood dust
(d) Carbon
48. Earliest lesion seen in asbestosis is:
(a) Pleural plaques
(Delhi PG-2007)
(b) Hilar lymphadenopathy
(c) Adenoma lung
(d) Mesothelioma
49. Most dangerous particles causing pneumoconiosis are of size:
(Delhi PG-2006)
(a) 1-5 micron
(b) < 1 micron
(c) 5-10 micron
(d) 10-20 micron
50. Asbestos exposure can cause all except:
(a) Arthralgia
(Delhi PG-2006)
(b) Mesothelioma
(c) Carcinoma larynx
(d) Bronchogenic carcinoma
51. Predominant constituent of Hyaline membrane is:
(a) Albumi
(Delhi PG-2005)
(b) Anthracotic pigment
(c) Fibrin rich exudates
(d) None of the above

MOST RECENT QUESTIONS


52. “Egg-shell calcifications” are seen in:
(a) Silicosis
(b) Berylliosis
(c) Asbestosis
(d) Bronchial asthma
53. Hyaline membrane disease is associated with:
(a) Respiratory distress syndrome
(b) Bronchopulmonary dysplasia
(c) Sudden infant death syndrome
(d) Bronchiolitis obliterans
54. Baggasosis is caused by:
(a) Cotton dust
(b) Sugarcane
(c) Asbestosis
(d) None
55. Which interstitial lung disease is caused by organic dust:
(a) Silicosis
(b) Asbestosis
(c) Byssinosis
(d) Anthracosis
56. Lower lung involvement is common in:
(a) TB
(b) Asbestosis
(c) Silicosis
(d) All
57. Pleural calcification is found in all of the following except:
(a) Asbestosis
(b) Hemothorax
(c) Tuberculous pleural effusion
(d) Coal worker pneumoconiosis
58. Caplan’s syndrome is seen in:
(a) COPD
(b) Pneumoconiosis
(c) Pulmonary edema
(d) Bronchial asthma
59. Acute pulmonary sarcoidosis is least likely to be associated with:
(a) Uveitis
(b) Pleural effusion
(c) Erythema nodosum
(d) Lymphadenopathy
60. Which of the following would most likely be observed in the lung during an
autopsy of a 2-week-old infant who died of neonatal respiratory distress
syndrome?
(a) Alveoli filled with neutrophils
(b) Dense fibrosis of the alveolar walls
(c) Enlarged air space
(d) Hyaline membranes and collapsed alveoli
61. Pneumoconiosis is seen with which particle size?
(a) 0.5-3 microns
(b) 3.5-6 microns
(c) 6.5-8 microns
(d) 10-20 microns
62. Which of the following increases tuberculosis?
(a) Asbestosis
(b) Sarcoidosis
(c) Silicosis
(d) Berylliosis
63. The commonest cause of death in ARDS is:
(a) Hypoxemia
(b) Hypotension
(c) Nonpulmonary organ failure
(d) Respiratory failure
64. ARDS is due to a defect/damage in:
(a) Type 1 pneumocytes
(b) Type 2 pneumocytes
(c) Clara cells
(d) Endothelial cells
65. Which of the following is true regarding Non-specific interstitial pneumonia?
(a) Honey combing on CT
(b) Predominant in males
(c) Affects elderly age group
(d) Good prognosis
66. All are recognized causes of Adult Respiratory Distress Syndrome (ARDS),
except:
(a) Smoke inhalation
(b) Malignant hypertension
(c) Gastric aspiration
(d) Viral pneumonias
67. Interstitial lung disease is seen in
(a) Rheumatoid arthritis
(b) Sjogren syndrome
(c) SLE
(d) None
68. Radiotherapy induced radiation pneumonitis is mediate by all of the following
except?
(a) TNF alpha
(b) PAF
(c) TGF-beta
(d) NF-kappa beta

VASCULAR LUNG DISEASE: INFARCT, PULMONARY EDEMA, PULMONARY HTN

69. The percentage of pulmonary emboli, that proceed to infarction, is


approximately:
(AI 2006)
(a) 0-5%
(b) 5-15%
(c) 20-30%
(d) 30-40%
70. On sectioning of an organ at the time of autopsy, a focal, wedge-shaped firm
area is seen accompanied by extensive hemorrhage, with a red appearance.
The lesion has a base on the surface of the organ. This findings is typically
of:
(AIIMS May 2003)
(a) Lung with pulmonary thromboembolism
(b) Heart with coronary thrombosis
(c) Liver with hypovolemic shock
(d) Kidney with septic embolus
71. All are the histological features of pulmonary hypertension:
(PGI June 2004)
(a) Capillaritis of alveolar septa
(b) Saddle thrombi in pulmonary trunk
(c) Thrombi in pulmonary vasculature
(d) Veno-occlusive disease
(e) Thickened arterial wall
72. Bilateral exudative pleural effusion is seen in:
(a) SLE
(PGI Dec 2006)
(b) Lymphoma
(c) CCF
(d) Nephrotic syndrome
(e) Ascites
73. “Sudden cardio pulmonary collapse” occurring in pulmonary embolism is
due to:
(UP 2005)
(a) Peripheral embolism of the vessels
(b) 60% pulmonary circulation is obstructed by emboli
(c) Multiple small thrombi causes impaction
(d) Organization of the clot
74. Dr. Sushant Verma conducted a study in MAMC which included admitted
patients hospitalized for more than 10 days and are bedridden for more than
7 days. On investigations, Dr. Verma finds that a small number of patients
have abnormal ultrasound (suggestive of thrombosis in lower limbs), low pO2
and pulmonary perfusion defects. Which of the following symptoms is most
likely associated with these patients?
(a) Cor pulmonale
(b) Hemoptysis
(c) Dyspnea
(d) No symptoms

MOST RECENT QUESTIONS

75. Bronchogenic sequestration is seen in which lobe:


(a) Left lower lobe
(b) Right upper lobe
(c) Left middle lobe
(d) Left upper lobe
76. Which of the folloowing is not true about pulmonary embolus?
(a) Saddle embolus may cause sudden death
(b) Most lesions affect are in the lower lobes
(c) Small arterioles are blocked
(d) Most of the emboli cause infarction
77. Sequestrated lobe of lung is commonly supplied by which of the following
vessels?
(a) Pulmonary artery
(b) Intercostal artery
(c) Descending aorta
(d) Bronchial artery

LUNG MALIGNANCIES

78. All are true regarding mesothelioma except:


(a) Bilaterally symmetrical
(AIIMS May 2011)
(b) Associated with asbestos exposure
(c) Histopathalogy shows biphasic pattern
(d) Occurs in late middle age
79. A 67 yr male with history of chronic smoking hemoptysis with cough.
Bronchoscopic biopsy from centrally located mass shows undifferentiated
tumor histopathologically. Most useful I.H.C. (immunohistochemical) marker
to make a proper diagnosis would be:
(AIIMS Nov 2009)
(a) Cytokeratin
(b) Parvalbumin
(c) HMB-45
(d) Hep-par1
80. Which of the following is a finding in biopsy of mesothelioma of pleura
- (PGI Dec 01)
(a) Myelin figures
(b) Desmosomes
(c) Weibel-Palade bodies
(d) Microvilli invasion
(e) Intense fibrosis
81. Neuroendocrine lesions of lung are:
(PGI June 2004)
(a) Carcinoid hamartoma
(b) Alveolar carcinoma
(c) Hamartoma
(d) Asthma
82. Hypersecretory granules are seen in which carcinoma of lung?:
(PGI Dec 2006)
(a) Adenocarcinoma
(b) Small cell carcinoma
(c) Large cell carcinoma
(d) Bronchoalveolar carcinoma
(e) Squamo us cell carcinoma
83. Most common site of metastasis in lung carcinoma is:
(a) Brain
(b) Kidney
(RJ 2000)
(c) Adrenal
(d) Testes
84. True about oat cell carcinoma of lung is:
(a) Secrete ectopic hormone
(RJ 2001)
(b) Variant of small cell carcinoma
(c) Cause SIADH
(d) All
85. A medical examination of a student reveals absence of cardiac sounds on
left side of the chest but surprisingly the normal heart beat on the right side
of the chest. The liver edge can be palpated on the left but not the right side
of the abdomen. He also gives history of bronchiectasis and sinusitis. Which
of the following should be suspected?
(a) Down syndrome
(b) Kartagener syndrome
(c) Kawasaki disease
(d) Marfan syndrome
86. Pleural mesothelioma is associated with:
(a) Asbestosis
(PGI Dec 2005)
(b) Berylliosis
(c) Silicosis
(d) Berylliosis
(e) Baggasosis

MOST RECENT QUESTIONS


87. Which of the following can develop into lung cancer?
(a) Asbestosis
(b) Silicosis
(c) Byssinosis
(d) Anthracosis
88. Scar in lung tissue may get transformed into:
(a) Adenocarcinoma
(b) Oat cell carcinoma
(c) Squamous cell carcinoma
(d) Columnar cell carcinoma
89. APUD cells are seen in:
(a) Bronchial adenoma
(b) Bronchial carcinoid
(c) Hepatic adenoma
(d) Villous adenoma
90. All give rise to malignancy except:
(a) Cholelithiasis
(b) Bronchiectasis
(c) Ulcerative colitis
(d) Paget’s disease
91. Indoor air pollution does not lead to:
(a) Chronic lung disease
(b) Impaired neurological development
(c) Pneumonia in child
(d) Adverse pregnancy outcome
92. Cavity formation is observed in one of the following bronchogenic carcinoma:
(a) Squamous cell
(b) Oat cell
(c) Adenocarcinoma
(d) Bronchoalveolar
93. Which of the following is having the minimal chances of causing a
mesothelioma?
(a) Amphibole
(b) Crysolite
(c) Amesolite
(d) Tremolite
94. The most common lesions in the anterior mediastinum are all except:
(a) Thymomas
(b) Lymphomas
(c) Lymph node enlargement from metastasis
(d) Teratomatous neoplasms
95. In a 70-year-old man who was working in asbestos factory for 10-15 years. On
routine X ray, a mass was seen in the right apical region of the lung. Biopsy
was taken from the mass. Which of the following is seen on electron
microscopic examination?
(AIIMS Nov 2013)
(a) Numerous long slender microvilli
(b) Melanosomes
(c) Desmososmes with secretory endoplasmic reticulum
(d) Neurosecretory granules in the cytoplasm
96. Lung cancer most commonly associated with?
(a) Asbestosis
(b) Silicosis
(c) Berylliosis
(d) Coal worker pneumoconiosis
97. Least common cause of clubbing is:
(a) Adenocarcinoma
(b) Squamous cell cancer
(c) Small cell cancer
(d) Mesothelioma
98. Most common benign tumor of the lung is?
(a) Leiomyoma
(b) Hamartoma
(c) Papilloma
(d) Adenoma
1. Ans. (c) Assman’s focus
(Ref: Robbins 8th/370; OP Ghai 5th/2001-1, Radiology of chest diseases 2nd/77
Thieme)
Recent Advances in Pediatrics pulmonology by Suraj Gupte volume 10 page/165)

• Most common lesion of chronic pulmonary TB is called as Puhl lesionQ.


• The infraclavicular lesion is called Assman Redeker SimonQ focus.
• In post primary stage (late dissemination), coarse granular dissemination is called Aschoff Puhl
focus.

2. Ans. (d) Mitral stenosis


(Ref: Robbins 8th/367-72)
Disease states that increase the risk of tuberculosis are:
Diabetes mellitus Chronic lung disease Malnutrition
(particularly silicosis)
Hodgkin’s lymphoma Chronic renal failure Alcoholism
Immunosuppression (e.g. seen in
AIDS)

3. Ans. (b) Predominance of alveolar exudate


(Ref: Robbins 7th/751, 8th/713-4, 9th/704; Harrison’s 17th/1620)
4. Ans. (d) Klebsiella pneumonia
(Ref: Harrison 17th/838, Robbins 7th/751, 9/e p703-705)
Klebsiella pneumonia presents as typical air space pneumonia with cough productive of
purulent sputum.
Causes of atypical pneumonias
• Mycoplasma
• Chlamydia pneumonia
• Viral infections (Influenza, RSV, Adenovirus),
• Legionella
• Coxiella burnetti
• Pneumocystis carinii
• MycoplasmaQ is the commonest cause of atypical pneumonia.
5. Ans. (a) Cavitation
(Ref: Robbins 7th/384–5, 9/e p373; Harrison 17th/1010)
• Cavitation is seen when there has been a previous sensitization of the host
resulting in caseous necrotic material being present which is discharged
through the cavities. So, it is associated with secondary tuberculosis more
frequently.

Caseous granulomas with multinuclear giant cells are present in both primary and secondary
tuberculosis.
6. Ans. (b) TB; (c) Histoplasmosis; (d) Cryptococcosis; (e) Wegener’s
granulomatosis
(Ref: Robbins’ 7th/399, 754, 9/e p98,709)
Granuloma with necrosis is seen in following conditions
• Tuberculosis
• Histoplasmosis
• Wegener’s granulomatosis
• Cryptococcosis
– Classical PAN does not involve the pulmonary arteryQ.
7. Ans. (a) Altered Sensorium; (b) Dental sepsis; (d) Subpulmonic effusion; (e)
Endobronchial obstruction.
(Ref: Robbins 7th/753, 8th/716-7, 9/e p708)
Predisposing factors of lung abscess
• Aspiration of infective material: Seen in alcoholics, during general anesthesia,
sinusitis, gingivodental sepsis, coma, gastroesophageal reflux diseases.
• Antecedent primary bacterial infection: Post pneumonic abscess.
• Septic embolism: Thrombophlebitis, bacterial endocarditis, IV drug abusers.
• Carcinoma bronchus: ausing obstruction to bronchopulmonary segment.
• Miscellaneous: Direct spread of infection from suppuration of subphrenic
space, pleural cavity, esophagus, spine, etc.
8. Ans. (a) Goodpasture syndrome; (b) Leptospirosis; (d) Wegener’s
granulomatosis; (e) Hantana virus infection;
(Ref: Harrison 17th 1793)
Pulmonary renal syndrome is seen in:

• Goodpasture’s syndrome: Pulmonary hemorrhage and renal failure


• Leptospirosis: Renal and hepatic dysfunction, hemorrhagic pneumonia, bleeding diathesis
• Hantan virus also cause pulmonary renal syndrome.
• Wegener’s granulomatosis: Lung and kidney involvement is common.
Note: Legionella does not affect kidneys. It causes atypical pneumonia, diarrhea and hyponatremia.

9. Ans. (b) Asbestosis; (c) SLE

(Ref: Crofton and Douglas 5th/979,1043, 579,1437,611, Robbins 9/e p693)


Causes of Granulomatous lung response
Known cause Unknown cause
• Hypersensitivity pneumonitis • Sarcoidosis
• Inorganic dust: beryllium, silica • Langerhan’s cell granulomatosis
• TB • Granulomatous vasculitis
• Coccidiodomycosis • Wegener’s granulomatosis
• Schistosomiasis • Churg-Strauss syndrome
• Bronchocentric granulomatosis
• Lymphomatoid granulomatosis

10. Ans. (d) Subpleural caseous lesion just above or below the interlobar fissure.
(Ref: Robbins 8th/370, 9/e p374)
• Inhaled tubercle bacilli implanted in the distal air spaces of the lower part of
upper lobe or upper part of the lower lobe, close to the pleura lead to
formation of Ghon’s focus.
• Primary complex or Ghon’s complex of tuberculosis consists of 3 components
– Pulmonary compound or Ghon’s focus
– Draining lymphatics
– Caseating hilar lymph node
• Caseous hilar lymphadenopathy is associated with Ghon complex and not
Ghon focus.
11. Ans. (a) Ghon’s focus, (b) Pleural efffusion (d) Fibrosis
(Ref: Robbins 7th/384, 9/e p374-375)
12. Ans. (a) Interstitial pneumonitis; (c) Foamy vacuolated exudates; (d)
Mononuclear cell in bronchoalveolar lavage; (e) Neutrophil infiltration:
(Ref: Harrison’ 17th/1267-8, 18th/1671)
Pneumocystis carinii pneumonia

• On lung sections stained with H and E, the alveoli are filled with typical foamy, vacuolated
exudates.
• Severe disease may include interstitial edema, fibrosis and hyaline membrane formation.
• The host inflammatory to lung injury results in increasing neutrophil count in bronchoalveolar
lavage fluid, hypertrophy of alveolar type II cells and a mild mononuclear cell infiltrate.
• Malnourished infants display an intense plasma cell infiltrate.

13. Ans. (c) No abnormal findings


(Ref: Robbins 9/e p373)
Most Mycobacterium tuberculosis infections are asymptomatic and subclinical
infections.

• Calcifications and cavitation are more frequent after re-infection or reactivation of tuberculosis
infections in adults.
• Lymphadenopathy or subpleural granuloma formation is more frequent in primary tuberculosis
infections.
• A diffuse reticulo-nodular pattern is suggestive of miliary tuberculosis.

14. Ans. (d) Accumulation of fibrin


(Ref: Robbins 8/e p713, 9/e p704) ...see text
15. Ans. (d) 5-7
(Ref: Harsh Mohan 6/e p469) ...see text
16. Ans. (c) Atelectasis
(Ref: Robins 9/e 670-671)
Atelectasis refers either to incomplete expansion of the lungs (neonatal atelectasis)
or to the collapse of previously inflated lung, producing areas of relatively airless
pulmonary parenchyma.
Resorption Atelectasis Compression Contraction
Atelectasis Atelectasis
Resorption Atelectasis Compression Contraction
Atelectasis Atelectasis
• Due to airway obstruction leading to • Due to presence of • Fibrosis in the lung
resorption of oxygen trapped in the alveoli. fluid, blood, air or or pleura
tumor in pleural preventing full
space. expansion of
pulmonary tissue.
• Causes mediastinal shift towards affected • Causes • Only irreversible
lung. mediastinal shift cause of
away from the atelectasisQ.
affected lung.
• Associated with chronic • Most commonly
bronchitis/asthma/aspiration of foreign associated with
body/ secretions cardiac failureQ.

17. Ans. (a) Apex


(Ref: Robbins 9/e p373, 8/e p370, 7/e p383)
Secondary pulmonary tuberculosis classically involves the apex of the upper
lobes of one or both lungs
18. Ans. (a) Terminal bronchiole
(Ref: Robbins 9/e p373)
19. Ans. (b) Recruitment of lymphocytes
(Ref: Robbins 9/e p371-374, 8/e p74, 7/e p382)
Macrophages are the primary cells infected by M. tuberculosis. Early in infection,
tuberculosis bacilli replicate essentially unchecked, while later in infection,
the cell response stimulates macrophages to contain the proliferation of the
bacteria.
About 3 weeks after infection, a T-helper 1 (TH1) response is mounted that
activates macrophages to become bactericidal.
• The TH1 response orchestrates the formation of granulomas and caseous
necrosis. Macrophages activated by IFN-γ differentiate into the “epithelioid
histiocytes” that characterize the granulomatous response, and may fuse to
form giant cells.
20. Ans. (c) ESR is raised because of increased RBC aggregate
(Ref: Wintrobe’s hematology 13/e p16-7)
ESR is a non specific test which is used as an indicator of active disease. It increases in
some disease states due to increase in plasma fibrinogen, immunoglobulins and
other acute phase reactants. In addition, change in red cell shape and number
affect ESR.
Decreased ESR: Sickle cell disease, polycythemia, and hereditary spherocytosis.
Increased ESR: Infections, anemia, liver disease, cancer, pregnancy, collagen
vascular disease
So, in a patient with TB, ESR will increase due to aggregation of red cells called rouleaux
whose formation is facilitated by fibrinogen. Since, it is non specific; it cannot
conform recovery from TB.
Additonal info for AIIMS
• The ESR may be measured by Wintrobe or Westergren’s tube but the readings need to be
corrected for the patient anemia.
• However, a variant of ESR called zeta sedimentation rate is developed which produced
reproducible results and doesnot require correction for anemia.
21. Ans. (d) Lobar pneumonia
(Ref: Robbins 9/e p704)
• ‘Bacterial invasion of the lung parenchyma causes the alveoli to be filled
with an inflammatory exudate, thus causing consolidation
(“solidification”) of the pulmonary tissue’…..Robbins definition of
pneumonia
22. Ans. (b) Young syndrome
(Ref: Robbins 9/e p683)

• Young syndrome is characterized by bronchiectasis, rhinosinusitis and reduced fertility.


• Kartagener syndrome, marked by situs inversus, bronchiectasis and sinusitis.

23. Ans. (c) Chronic bronchitis


(Ref: Robbins 9/e p679)
24. Ans. (b) Pulmonary emphysema; (c) Diastase resistant hepatic cells
(Ref: Robbins 9/e p675-676)
• Alpha 1-anti-trypsin deficiency is an autosomal recessive disease marked by
abnormally low serum levels of α1 AT enzyme resulting in panacinar
emphysema.
• It is characterized by presence of round to oval, PAS positive and diastase
resistant cytoplasmic inclusions in hepatocytes which on H and E stain
acidophilic and indistinctly demarcated from surrounding cytoplasm.
25. Ans. (b) Eosinophils
(Ref: Robbins 9/e p680)
26. Ans. (a) Bronchial asthma
(Ref: Robbins 9/e p682)
27. Ans. (b) Centriacinar
(Ref: Robbins 9/e p675)
28. Ans. (a) Centrilobular emphysema
(Ref: Robbins 9/e p675, 8th/684, 7th/719)
29. Ans. (b) Granulocyte–macrophage colony stimulating factor
(Ref: Robbins 8th/705, 9/e p696)

The patient in the stem of the question has the acquired from of pulmonary alveolar proteinosis
(PAP).

CFTR gene mutations lead to cystic fibrosis and widespread bronchiectasis.


• Anti –DNA topoisomerase I antibodies are seen in diffuse scleroderma, which
produces interstitial fibrosis.
• Anti – glomerular basement membrane antibody is present in Goodpasture’s
syndrome with extensive alveolar hemorrhage.

• PAP is associated with impaired surfactant clearance by alveolar macrophages.


• Microscopically there is accumulation of acellular surfactant in intra-alveolar and bronchiolar
spaces.
30. Ans. (b) Centriacinar emphysema
(Ref: Robbins 8th/684-5, 9/e p675)
The findings of Mr. John point to an obstructive lung disease like emphysema which
occurs due to airway narrowing or even from loss of elastic recoil. So, emphysema
is the most likely diagnosis.

• Sarcoidosis is a chronic restrictive lung disease with all lung volumes decreased, low FVC,
and normal FEV1/FVC ratio.
• Diffuse alveolar damage is an acute restrictive lung disease.
• Chronic pulmonary embolism does not affect FVC because the airways are not affected. It is
however associated with a ventilation/perfusion mismatch.

31. Ans. (c) Emphysema


(Ref: Robbins 9/e p675-676)
Emphysema is a pulmonary disease characterized by enlargement of the alveolar
airspaces due to destruction of the septae without consequent fibrosis. Pulmonary
hypertension (choice D) affects neither the airways nor the alveoli. It causes
involvement of pulmonary blood vessels.
32. Ans. (a) Emphysema
(Ref: Robbins 9/e p675)
Alpha 1 anti trypsin deficiency is associated with panacinar emphysema.
33. Ans (a) Asthma
(Ref: Robbins 9th/ 682)
Thickening of the airway wall is a feature of airway remodeling and is seen in asthma.
34. Ans. (a) Bronchial asthma
(Ref: Robbins 9/e p682)
35. Ans (b) Paraseptal emphysema
(Ref: Robbins 9th/ 675)
Paraseptal emphysema is synonymous as distal acinar emphysema. It is characterized
by distension of the distal alveoli.
36. Ans. (a) Diffuse alveolar damage
(Ref: Robbins 9/e p672)
37. Ans. (a) Size of the particle ingested is less than 0.5 micrometer
(Ref: Robbins 8th/53, Pharmaceutical Research, Vol. 25, No. 8, August 2008)
The article writes….Particles possessing diameters of 2-3 microns exhibits maximum
phagocytosis and attachment.
38. Ans. (a) Methaemoglobinemia
(Ref: Harrison 17th/1612-3, Robbins 9/e p691)
39. Ans. (c) Asbestosis
(Ref: Robbins 8th/700, 9/e p691)
Robbins direct quote… ‘inorganic particles may become coated with iron protein
complexes and are called ferruginous bodies’.
Asbestosis in its classical form is a diffuse fibrotic disease of the lung tissue. Typically,
the fibrotic changes of asbestosis are focal and are most prominent in the lower
lung lobes. The presence of asbestos bodies in or adjacent to the walls of fibrotic
respiratory bronchioles is the hallmark of the disease. A characteristic of asbestos
bodies is a core of asbestos coated by ferroprotein. (So, ‘c’ is the answer)
Asbestos bodies are mimicked by “ferruginous bodies” which are formed on particles of talc, mica,
fibre, glass and other less common materials in the lung. True asbestos bodies are clear whereas
the core of these particles is dark.

40. Ans. (a) Diffuse alveolar damage (Ref: Robbins 9/e p672)
Diffuse alveolar damage is a characterstic feature of ARDS. As explained in the text,
asbestos inhalation is associated with pleural plaque, interstitial fibrosis,
bronchogenic cancer and mesothelioma.
41. Ans. NONE
(Ref: Harrison 17th/1643, Robbins 8th/694, 704, 9/e p685)
All the mentioned options are associated with interstitial lung disease. Following is
a table adapted from Robbins and Harrison for a quick reference.
Causes of Interstitial Lung Disease (ILD)
Fibrosing Granulomatous Smoking related Miscellaneous
Usual interstitial pneumonia Sarcoidosis Desquamative interstitial Eosinophilic
(idiopathic pulmonary fibrosis) pneumonia
Associated with collagen Hypersensitivity Respiratory bronchiolitis- Pulmonary alveolar
vascular diseases, drugs and pneumonitis associated interstitial lung proteinosis
radiation disease
Cryptogenic organizing
pneumonia
Nonspecific interstitial
pneumonia
Pneumoconiosis

Directly quoting Pneumoconiosis as is given on page 696 of Robbins …. ‘the


pneumoconiosis was originally coined to describe the non-neoplastic lung
reaction to inhalation of mineral dusts encountered in the workplace. Now it
also includes diseases induced by organic as well as inorganic particulates
and chemical fumes and vapors’. Presented underneath is an adapted
classification from Robbins table 15-6;
Causes of pneumoconiosis
Mineral dusts Coal dust, silica, asbestos, iron oxide, barium
sulfate
Organic dusts inducing hypersensitivity Moldy hay, bagasse, bird droppings
pneumonitis
Chemical fumes and vapours Sulfur dioxide, ammonia, benzene, insecticides
Organic dusts inducing asthma Cotton, flax, hemp

As can be concluded from both the above tables, the answer should be none in the
options provided. If the question would have been containing pneumoconiosis and
NOT ILD, then smoking would have been the answer of choice.
42. Ans. (c) Emphysema
(Ref: Robbins 9/e p691)
43. Ans. (c) Nodular lesions involving upper lobes
(Ref: Robbins 7th/734-6, 8th/700, 9/e p688)
• In asbestosis, there is presence of lesions affecting lower lobes or base of the
lungs
• Nodular lesions involving upper lobes’ is a feature of silicosis. The lesions
continue to progress even after exposure to asbestos has stopped.
44. Ans. (c) Byssinosis
(Ref: Robbins 7/e p733, 9/3 p688)
• Hypersensitivity pneumonitis (also called allergic alveolitis) describes a
spectrum of immunologically mediated, predominantly interstitial lung
disorders caused by intense, often prolonged exposure to inhaled organic
antigensQ. It is a type III + IV hypersensitivity reaction.
• Table 15-6 on page 697/8th Robbins mentions- Byssinosis is an organic
dust causing asthma; rest of the options silicosis, asbestosis and berylliosis
are given as examples of mineral dusts. So, they can be easily excluded. The
best answer would therefore be option ‘c’ i.e. Byssinosis.
Other examples associated with hypersensitivity pneumonitis
Farmer’s lung: Thermopohilic actionomycete or mouldy hay or grain dust*.
Pigeon breeder’s lung: Proteins from serum, excreta or feathers of the birds.
Air conditioner lung (or Humidifier lung): Thermopohilic bacteria in heated water reservoirs.

45. Ans. (a) Sarcoidosis; (b) Interstitial lung disease; (c) Langerhan’s cell
histiocytosis (e) Asbestosis.
• Parenchymal causes of end stage lung • Pulmonary Langerhan’s cell histiocytosis
disease: is a progressive disease, and can lead
• Emphysema to end stage lung disease.
• Pneumoconiosis • Sarcoidosis of the lung is an interstitial
lung disease; which may lead to
• Bronchitis progressive fibrosis and end stage lung
• ARDS disease.
• Asbestosis • Aspergillosis causes extrinsic allergic
• Interstitial lung disease. alveolitis or hypersensitivity pneumonitis.

46. Ans. (a) Polypoid plugs in bronchioles; (c) Exudation of proteinaceous


material in terminal airways; (d) Bronchoconstriction; (e) Response to
steroids

(Ref: Robbins’ 7th/731, 8th/696, 9/e p687; CMDT’ 2010 243)


Cryptogenic organizing pneumonia (earlier called as Bronchiolitis obliterans with
organizing pneumonia)

• Affects men and woman equally, around 50-70 years


• Etiology is unknownQ
• Clinical features- Dry cough and dyspnea
• Chest X-ray shows subpleural and peribronchialQ patchy area of airspace consolidation.
• Histopathology: there is presence of polypoid plugs of loose organizing connective tissue
(called as Masson bodiesQ) within alveolar ducts, alveoli and often bronchiolesQ (all are of same
age)
• There is no interstitial fibrosis or honey comb lungQ.
• Treatment is done with steroidsQ.

47. Ans. (a) Silica


(Ref: Robbins 8th/698-9, 9/e p690)
48. Ans. (a) Pleural plaques
(Ref: Robbins 9/e p691)

Pleural plaqueQ:
It is the most common manifestation of asbestos exposure composed of plaques of dense collagen
containing calcium. They are usually asymptomatic and develop on anterior and posterolateral parts
of parietal pleura and over the diaphragm.

49. Ans. (a) 1-5 micron


(Ref: Robbins 9/e p688)
In pneumoconiosis, the most dangerous particles range from 1-5 micron in diameter,
because they may reach the terminal small airways and air sacs and settle in their
linings.

Note:
• The solubility and cytotoxicity of particles, modify the nature of pulmonary response.
• In general, the smaller the particle, the higher the surface area-to-mass ratio, and the more likely
and more rapidly toxic levels will appear in the pulmonary fluids.
• Larger particles resist dissolution and so may persist within lung parenchyma for years.
• Larger particles tend to evoke fibrosing collagenous pneumoconiosis, such as characteristic of
silicosis.

50. Ans. (a) Arthralgia


(Ref: Robbins 9/e p691)
51. Ans. (c) Fibrin rich exudates
(Ref: Robbins 9/e p457)
• The membranes (in hyaline membrane disease) are largely made up of
fibrinogen and fibrin admixed with cell debris derived chiefly from necrotic
type-II pneumocytes.
• There is a remarkable paucity of neutrophilic inflammatory reaction associated
with these membranes.
• The lesions of hyaline membrane disease are never seen in still born infants or
in live-born infants who die within a few hours of birth.
52. Ans. (a) Silicosis (Ref: Robbins 9/e p690, 8th/699, 7th/734)
53. Ans. (a) Respiratory distress syndrome
(Ref: Robbins 9/e p457, 8th/680, 7th/715)
54. Ans. (b) Sugarcane
(Ref: Robbins 9/e 688)
55. Ans. (c) Byssinosis
(Ref: Robbins 9/e 688)
56. Ans. (b) Asbestosis
(Ref: Robbins 9/e 691)
57. Ans. (b) Hemothorax
(Ref: Robbins 8th/732, 9/e 722)
58. Ans. (b) Pneumoconiosis
(Ref: Harrison 17th/1625,)
59. Ans. (b) Pleural effusion
(Ref. Robbins 9/e 693)
60. Ans. (d) Hyaline membranes and collapsed alveoli
(Ref: Robbins 8th/680, 9/e 457)
Neonatal respiratory distress syndrome is a disease of immaturity. The immature
lung is not able to produce sufficient surfactant to prevent collapse of many alveoli.
Severe diffuse damage to alveoli causes precipitation of protein (“hyaline
membranes”) adjacent to many alveolar walls.
Abundant neutrophils (choice A) are seen in pneumonia.
Fibrosis (choice B) is a late, not early, feature of respiratory distress syndrome whereas
the air spaces are collapsed, not enlarged (choice C), in this condition.
61. Ans (a) 0.5-3 microns
(Ref: Robbins 9th/ 688)
62. Ans (c) Silicosis
(Ref: Robbins 9/e p 690)
63. Ans (c) Nonpulmonary organ failure
(Ref: Robbins 9th/ 674)
In ARDS, most of the deaths are attributable to sepsis or multiorgan failure and, in
some cases, direct lung injury.
64. Ans. (d) Endothelial cells
(Robbins 9th/672)
ALI/ARDS is initiated by injury of pneumocytes and pulmonary endothelium, setting in
motion a viscous cycle of increasing inflammation and pulmonary damage.
Endothelial activation is an important early event……Robbins 9th/672
Also revise
The histologic manifestation of this disease is diffuse alveolar damage (DAD).

65. Ans. (d) Good prognosis


(Ref: Robbins 9th/686)
Nonspecific interstitial pneumonia
• Nonspecific interstitial pneumonia may be idiopathic or associated with connective tissue disease.
• Clinical features
• Patients present with dyspnea and cough of several months’ duration.
• More likely to be female nonsmokers in their sixth decade of life.

• High-resolution computed tomography scan: B/L, symmetric, predominantly lower lobe reticular
opacities (honeycomb pattern is absent).
• Patients have a much better prognosis than those with usual interstitial pneumonia.
• Having 2 patterns: cellular and fibrosing patterns. Those having the cellular pattern are somewhat
younger than those with the fibrosing pattern and have a better prognosis
• In cellular pattern, there is mild to moderate chronic interstitial inflammation, in a uniform or patchy
distribution.
• In fibrosing pattern, there is diffuse or patchy interstitial fibrotic lesions of roughly the same stage of
development (an important distinction from usual interstitial pneumonia)
• Fibroblastic foci, honeycombing, hyaline membranes and granulomas are absent.
66. Ans (b) Malignant hypertension
(Ref: Robbins 9th/ 672)
See the table of causes of acute respiratory distress syndrome. ARDS is associated with
non cardiogenic pulmonary edema. Malignant hypertension will cause development
of cardiogenic pulmonary edema.
The four most important causes of ARDS:

Sepsis, diffuse pulmonary infections, gastric aspiration and head injuries.

67. Ans (a) Rheumatoid arthritis


(Ref: Robbins 9e/ p687
Interstitial lung disease is associated with rheumatoid arthritis.)
68. Ans (b) PAF
(Ref: Robbins 9th and multiple sources)
69. Ans. (b) 5-15%
(Ref: Robbins 7th/742, 8th/706, 9/e 698)
Robbins direct quote. ‘Only about 10% of pulmonary artery emboli actually cause
infarction’.
70. Ans. (a) Lung with pulmonary thromboembolism
(Ref: Robbins 9/e p129-130, 8th/128, 7th/138)
• The morphology is characteristically present as red (hemorrhagic) infarct. The
red infarcts are seen in lung, liver and intestine.
• White infarcts are seen in brain, spleen, kidney and heart.
71. Ans. (c) Thrombi in pulmonary vasculature; (d) Vaso-occlusive disease; (e)
Thickened arterial wall
(Ref: Robbins 8th/708, 9/e p699-700, Harsh Mohan 6th/466)
In pulmonary hypertension pathological changes are seen from main pulmonary
arteries to arterioles. They are:
Arterioles and
small
pulmonary Medium
arteries (most sized Large
prominently pulmonary pulmonary
affected) arteries arteries

(i) Medial (i) Medial (i)


hypertrophy hypertrophy; Atheromatous
which is not deposits
marked in
secondary
pulmonary
hypertension

(ii) Thickening (ii)


and Concentric
reduplication of intimal
elastic lamina. thickening

(iii) Plexiform (iii)


pulmonary Adventitial
arteriopathy in fibrosis
which
intraluminal tuft
of capillary
formation
occurs in
dilated thin
walled
arteriolar
branches.

(iv)
Thickening
and
reduplication
of elastic
lamina.

The presence of many organizing or


recanalizing thrombi favors recurrent
pulmonary emboli as the cause and the
coexistence of diffuse pulmonary fibrosis or
severe emphysema and chronic bronchitis
points to chronic hypoxia as the initiating
event.

72. Ans. (a) SLE;


(Ref: Harrison 17th/1660)
Transudative and exudative pleural effusion (PE) are differentiated by measuring the
LDH and protein level in the pleural fluid.
Exudative PE meets at least one of the following criteria:

1. Pleural fluid protein/serum protein > 0.5.


2. Pleural fluid LDH by serum LDH > 0.6.
3. Pleural fluid LDH more than 2/3rd of the normal upper limit of serum.

Transudative PE is caused by:


• CCF, cirrhosis, pulmonary embolism
• Nephrotic syndrome, peritoneal dialysis, superior vena cava obstruction
• Myxoedema
Causes of exudative pleural effusion

Neoplastic disease Pericardial disease Asbestos Yellow nail syndrome


exposure

Hemothorax Chylothorax Uremia Radiation therapy

Pulmonary emboli Sarcoidosis Meig’s syndrome Ovarian hyperstimulation


syndrome

GI disease Collagen vascular Drugs Infections


Esophageal disease Nitrofurantoin Bacterial infections
perforation Rheumatoid pleuritis Dantrolene TB
Pancreatic disease SLE Methysergide Fungal infections
Intra-abdominal Drug induced lupus Bromocriptine Viral infections
abscess Sjogren’s syndrome Amiodarone Parasitic infections
Diaphragmatic Wegener’s Procarbazine
hernia granulomatosis
Churg Strauss
syndrome

73. Ans. (b) 60% pulmonary circulation is obstructed by emboli


(Ref: Robbins 9/e p127, 8th/126; 7th/136)
74. Ans. (d) No symptoms
(Ref: Robbins 8th/698, 706-7)
The stem of the question suggests that Dr. Verma’s clinical study is being done on
patients with pulmonary thromboembolism, and most pulmonary emboli are
small and clinically silent. Cor pulmonale may result from repeated embolization
which is associated with reduction in the pulmonary vascular bed. Hemoptysis is a
rare manifestation of pulmonary embolism. It occurs usually with hemorrhagic
infarction of the lung. Dyspnea occurs with medium to large emboli.
75. Ans. (a) Left lower lobe (Ref: Fetal and Neonatal
Physiology 4/e p872; Robbins 9/e p670 8/e p679; www.uptodate.com)
Pulmonary sequestrationcan be either:
• Extralobar sequestrations are external to the lung and may be located
anywhere in the thorax or mediastinum. They are seen in infants. They occur
in the lower left side of the thorax between the left lower lobe ad the
diaphragm.
• Intralobar sequestrations occur within the lung substance usually in older
children and are often associated with recurrent localized infection or
bronchiectasis. These are seen most commonly in the posterior basal
segment of left lower lobeQ (Ref. Fetal and Neonatal Physiology).
76. Ans. (d) Most of the emboli cause infarction
(Ref: Robbins 8/e p706, 9/e p127)
• Large emboli lodge in the main pulmonary artery or its major branches or at
the bifurcation as a saddle embolus. It may lead to sudden death.
• Smaller emboli travel out into the more peripheral vessels, where they may
cause hemorrhage or infarction. In patients with adequate cardiovascular
function, the bronchial arterial supply can sustain the lung parenchyma.
Hemorrhages may occur, but there is no infarction. The underlying pulmonary
architecture is preserved, and resorption of the blood permits reconstitution of
the preexisting architecture.
Also know: NEET points
• Only about 10%Q of emboli actually cause infarction, which occurs when the circulation
is already inadequate, as in patients with heart or lung disease.
• Pulmonaryinfarcts tend to be uncommon in the young.
• About 3/4thof all infarcts affect the lower lobes Q
• Inmore than half, multiple lesions occur.
• Typically, they extend to the periphery of the lung substance as a wedge with the apex
pointing toward the hilusQ of the lung.

77. Ans. (c) Descending aorta


(Ref: Robbins 8/e p679, 9/e 670)

• Pulmonary sequestration refers to the presence of a discrete mass of lung tissue without normal
connection to the airway system.
• The blood supply to the sequestered area arises not from the pulmonary arteries but from the
aorta or its branches

78. Ans. (a) Bilaterally symmetrical (Ref: Robbins 8th/733-4,


Malignant Pleural Mesothelioma 1st/65, DeVita’s Cancer 8th/1840, 9/e p723-
724)
Mesothelioma is an asbestos exposure releated tumor having Mean age of presentation
as 50-70 years. Microscopically it may have both epithelioid and sarcomatoid
patterns (biphasic pattern).
Option ‘a’…. ‘Pleural mesotheliomas are more commonly right sided (R:L ratio is 3:2)
may be because of greater size of right sided pleural cavity. Although usually
unilateral at presentation, it is not infrequent to find histological evidence of
mesothelioma in the contralateral pleura. Macroscopic evidence of synchronous
bilateral pleural tumors is rare’…. Malignant Pleural Mesothelioma 1st/65.
79. Ans. (a) Cytokeratin
(Robbins 9/e p716)
• The presence of chronic smoking, cough and hemoptysis in old man is a
pointer towards a diagnosis of bronchogenic cancer. The central location
suggests the possibility of a squamous cell cancer.
• Robbins writes “Histologically, this tumor is characterized by the presence
of keratinization and/or intercellular bridges”.
Other options: HMB (melanoma), Hep par1 (liver cancer) and parvalbumin
(schizophrenia).

80. Ans. (d) Microvilli invasion, (e) Intense fibrosis

(Ref: Robbins’ 7th/768-9, 8th/734, 9/e p723-724)


Benign mesothelioma Malignant mesothelioma
Nature Solitary fibrous tumor Diffuse thick and fleshy tumor
Prior asbestos NoQ relationship Definitive associationQ is present
exposure
Microscopic feature Whorls of reticulin and collagen Epithelioid type: cuboidal/columnar cells
fibers among which interspersed form tubular/papillary structures
spindle cells resembling fibroblasts Sarcomatoid type: appear as spindle cell
are present carcinoma
Mixed type
Immunotyping Cells are CD34(+) and keratin (-)Q Cells are CD34(-) and keratin (+)Q

81. Ans. (a) Carcinoid hamartoma


(Ref: Robbin 9/e p719)
Neoplasms of neuroendocrine cells in the lung include:
• Benign tumorlets (small tumors in areas of scarring and inflammation)
• Carcinoid
• Small cell carcinoma
• Large cell neuroendocrine carcinoma of lung.

82. Ans. (b) Small cell carcinoma


(Ref: Robbins 9/e p717)
• Small cell carcinoma of lung shows dense core neurosecretory granules.
The granules are similar to those found in neuroendocrine argentaffin cells
present along the bronchial epithelium.
• Some of these tumors secrete polypeptide hormones. Presence of
neuroendocrine markers such as chromogranin, synaptophysin and Leu-7 is
seen.
• It also secretes PTH like substance. They are most common pattern
associated with ectopic hormone production.
83. Ans. (c) Adrenal
(Ref: Robbins 9/e p717, 8th/723, 7th/763)
84. Ans. (d) All
(Ref: Robbins 9/e p717, 8th/728, 7th/762)
85. Ans. (b) Kartagener syndrome
(Ref: Robbins 9/e p683)
• Isolated inversion of the heart (dextrocardia) is almost always associated
with cardiac defects that may include transposition of the atria and
transposition of the great arteries.
• However, dextrocardia as part of situs inversus totalis, with reversal of the
thoracic and abdominal organs, is usually associated with a
physiologically normal heart. The cluster of situs inversus, sinusitis, and
bronchiectasis is called Kartagener syndrome, which is caused by defective
ciliary function.
Association of cardiac defects with syndromes

• Down syndrome - ostium primum type of atrial septal defect.


• Kawasaki disease- coronary artery aneurysms.
• Marfan syndrome - aortic dissection.
• Turner syndrome - coarctation of the aorta.

86. Ans. (a) Asbestosis


(Ref: Robbins 9/e p723)
87. Ans. (a) Asbestosis
(Ref: Robbins 9/e p691)
Direct lines.. “In contrast to other inorganic dusts, asbestos an also act as a
tumour initiator and promoter”.
88. Ans. (a) Adenocarcinoma
(Ref: Robbins 9/e p714-715, 8/e p724, 7/e p760-761)
“Peripheral adenocarcinomas with a small central invasive component associated
with scarring and a predominantly peripheral bronchioloalveolar growth
pattern may have a better outcome than invasive carcinomas of the same
size.”..Robbins
89. Ans. (b) Bronchial carcinoid
(Ref: Robbins 9/e p719)
Amine Precursor Uptake and Decarboxylation (APUD) cells are also known as
neuroendocrine cells. These cells are the cell of origin of the carcinoid
tumour.
90. Ans. (b). Bronchiectasis
(Ref: Robbins 9/e p684)
• Bronchiectasis cannot cause malignancy. The complications which may be
associated with bronchiectasis are: Cor pulmonale, metastatic brain
abscesses, and amyloidosis.
• To be considered bronchiectasis, the dilation should be permanent;
reversible bronchial dilation often accompanies viral and bacterial pneumonia.
91. Ans. (b) Impaired neurological development
(Ref: Robbin 8/e p 403)

Concept: Please note friends that impaired neurological development can result from lead
exposure but lead is an outdoor air pollutant.

Impaired lung function, lung inflammation, reduced exercise capacity; increased


respiratory symptoms are associated with air pollution….
Indoor air pollution contributes to acute respiratory infections in young children,
chronic lung disease and cancer in adults, and adverse pregnancy outcomes
(such as stillbirths) for women exposed during pregnancy. Acute respiratory
infections, principally pneumonia, are the chief killers of young children….Park
92. Ans. (a) Squamous cell
(Ref: Robbins 9/e p716-717)
Important points about Squamous cell carcinoma of lung

• MC type of lung cancer in smokersQ


• MC type in malesQ
• Usually central in location (arise from the segmental bronchi)
• Intercellular bridges or junction is very specific.
• HypercalcemiaQ due to PTHrP is the MC paraneoplastic syndrome

93. Ans (b) Crysolite


(Ref: Robbins 9/e p 691)
The serpentine chrysotile form accounts for 90% of the asbestos used in industry.
Amphiboles even though less prevalent, are more pathogenic than chrysotiles with
respect to induction of mesothelioma.
94. Ans (c) Lymph node enlargement from metastasis
(Ref: Robbins 9/e p 721)
95. Ans. (c) Desmososmes with secretory endoplasmic reticulum
(Ref: Robbins 8/e p 669, Thurlbeck’s Pathology of the Lung 3/e p428-435)
Tough one friends….AIIMS guys laid the trap beautifully! Lets analyze the
question…
The exposure to asbestos brings the first malignancy to our mind which is
mesothelioma. However, this is not the answer for the current question because:
• Period of exposure is 10-15 years in question. However, as per Robbins..
“there is a long latent period of 25 to 45 years for the development of
asbestos-related mesothelioma”.
• More importantly, mesothelioma is a pleural tumor whereas the question
clearly mentioned that the mass was seen in the right apical region.
Considering that there is no history of smoking, this is more likely to an
adenocarcinoma (and not squamous cancer) which is also the most common
type of lung cancer associated with asbestos exposure.
• Desmososmes with secretory endoplasmic reticulum is a feature of
electrom microscopic finding of adenocarcinoma….. Thurlbeck’s
Pathology table 17-7
Neuro-
Desmosomes/Tonofilament Tight Secretory secretory
Type Bundles Microvilli Junctions Lumina RER Granules
Squamous Many, wellformed + – – – –
carcinomas
Small cell Small, poorly formed Rare – – – +
carcinoma
Adenocarcinoma Many, small well formed + + + Present –
For future AIIMS question!

• Napsin A is a more sensitive and specific marker than TTF-1 for adenocarcinoma of the
lung.

Features of mesothelioma

• On electron microscopy, the presence of long microvilli and abundant tonofilaments but absent
microvillous rootlets and lamellar bodies.
• Lack of staining for carcinoembryonic antigen (it is positive in adenocarcinoma)
• Positive staining for calretinin, Wilms tumor 1 (WT-1), cytokeratin 5/6, and D2–40

96. Ans: (a) Asbestosis


(Ref: Robbins 9/e p713)
97. Ans (c) Small cell cancer.
(Ref: Principles and practice of lung cancer page 348)
Clubbing is most common with adenocarcinoma and is least common with small
cell lung cancer….direct quote
98. Ans: (b) Hamartoma
(Ref: Robbins 9/e p720)
Lung hamartoma is a relatively common lesion that is usually discovered as an
incidental, rounded radio-opacity (coin lesion) on a routine chest film.

ANNEXURE
Acute lung injury (ALI)
• Acute lung injury (ALI) (also called noncardiogenic pulmonary edema) is
characterized by the abrupt onset of significant hypoxemia and bilateral pulmonary
infiltrates in the absence of cardiac failure. Acute respiratory distress syndrome
(ARDS) is a manifestation of severe ALI.
• It is associated with sepsis, severe trauma, or diffuse pulmonary infection.
• The histologic manifestation of these diseases is diffuse alveolar damage (DAD).
There is also the presence of hyaline membranes lining alveolar walls.
• Pulmonary Langerhans Cell Histiocytosis
• Pulmonary Langerhans cell histiocytosis is a rare reactive inflammatory disease characterized by
focal collections of Langerhans cells (often accompanied by eosinophils). It results in scarring and
the appearance of irregular cystic spaces.
• Langerhans cells are immature dendritic cells with grooved, indented nuclei and abundant cytoplasm.
They are positive for S100, CD1a, and CD207 (langerin) and are negative for CD68.
• Most of the affected patients are relatively young adult smokers. It is associated with acquired
activating mutations in the serine/ threonine kinase BRAF.

Lymphangioleiomyomatosis
Lymphangioleiomyomatosis is a pulmonary disorder that primarily affects young woman
of childbearing age. It is characterized by a proliferation of perivascular epithelioid cells
that express markers of both melanocytes and smooth muscle cells. The proliferation
distorts the involved lung, leading to cystic, emphysema-like dilation of terminal
airspaces, thickening of the interstitium, and obstruction of lymphatic vessels. The
condition is characterized by TSC2 mutations. The condition affects young women
mainly and the presenting features include dyspnea or spontaneous pneumothorax. The
condition is treated with lung transplantation.

Inflammatory myofibroblastic tumor (Recent AIIMS Topic)

• More common in children


• Equal male-to-female ratio
• Presenting symptoms include fever, cough, chest pain, and hemoptysis.
• Grossly, the lesion is firm, 3 to 10 cm in diameter, and grayish white.
• Microscopically, there is proliferation of spindle-shaped fibroblasts and myofibroblasts,
lymphocytes, plasma cells, and peripheral fibrosis.
• Some of these tumors have activating rearrangements of the anaplastic lymphoma kinase
(ALK) gene located on 2p23.
1. A middle-aged female with a history of dyspnea, chronic productive cough and
wheezing died of respiratory failure following a bout of pneumonia. Her lungs
on autopsy had the appearance as shown in the image below. What is the
most likely associated finding for this patient?
(NEET 2020 like pattern)

(a) Defect in dynein arm of cilia


(b) Deficiency of alpha 1 antitrypsin
(c) Defect in chloride channels of airway epithelium
(d) Antibodies against basement membrane
Ans. (b) Deficiency of alpha 1 antitrypsin
(Ref: Robbins 9th/675-6)
History of dyspnea, chronic productive cough and wheezing and death due to respiratory
failure indicted pulmonary pathology. The given image is associated with presence
of large alveoli which can be seen on the cut surface of fixed lungs. This is
suggestive of emphysema.
Also know that even though 80% of COPD is due to smoking, there are other risk factors
include environmental and occupational pollutants, airway hyperresponsiveness
and genetic polymorphisms.
Option “a” and “b” would have lead to bronchiectasis but mostly would have their
presentation at an earlier age.
2. Which of the following is true about alpha 1 antitrypsin?
(NEET 2019 like pattern)
(a) Inhibits elastase
(b) Inhibits trypsinogen activation in pancreas
(c) Inhibits trypsin activating protease
(d) Inhibits chymotrypsin
Ans. (a) Inhibits elastase
(Ref: Robbins 9th e/p 676)
α1-antitrypsin, normally present in serum, tissue fluids, and macrophages, is a major
inhibitor of proteases (particularly elastase) secreted by neutrophils during
inflammation.
3. Infertility in Kartagener syndrome is due to which of the following defects?
(AIIMS Nov 2018 like pattern)
(a) Oligospermia
(b) Blockage of epididymis
(c) Asthenozoospermia
(d) Undescended testes
Ans. (c) Asthenozoospermia
(Ref: Robbins 9th e/p 683)
Kartagener syndrome is marked by situs inversus, bronchiectasis and sinusitis. The lack
of ciliary activity interferes with bacterial clearance, predisposes the sinuses and
bronchi to infection, and affects cell motility during embryogenesis, resulting in the
situs inversus. Males with this condition tend to be infertile, as a result of sperm
dysmotility.
Asthenozoospermia (or asthenospermia) is the term used for reduced sperm motility.
4. A worker was working in a factory from the past 20 years, and now presented
with pleural thickening and fibrosis. Histopathology of the lesion is shown in
below image. Most likely diagnosis is which of the following?
(AIIMS May 2018 like pattern)

(a) Asbestosis
(b) Cotton fiber
(c) Coal Worker Pneumoconiosis
(d) Silicosis
Ans. (a) Asbestosis
Decoding the question
• Factory worker is suggestive of an occupational disorder.
• Presence of pleural thickening and fibrosis is likely to be associated with
pneumoconiosis.
• Looking at the histopathology, there is a presence of asbestos body (golden brown,
fusiform or beaded rods with a translucent center and consist of asbestos fibers
coated with an iron-containing proteinaceous material). This is confirmatory for
making the diagnosis as asbestosis.
5. Patient with history of long standing depressive illness comes to the
emergency with acute breathlessness. The X-ray shows diffuse infiltrates
with predominance in right middle lobe and right lower lobe. The patient did
not survive and the following picture in the lungs was seen on autopsy. It is
suggestive of?
(AIIMS Nov 2017 Pattern)

(a) Severe necrosis with fungal hyphae, severe fungal pneumonia


(b) Coagulation necrosis, Tuberculosis
(c) Vegetable matter; Aspiration pneumonia
(d) Severe necrosis; severe necrotizing pneumonia
Ans. (c) Vegetable matter; Aspiration pneumonia
Presence of long standing depression is a risk factor for depressive illness. The
presence of refractive material with a granuloma is seen with a foreign body (like
vegetable matter). Infiltrates within the right middle and right lower lobe also
suggests a foreign body. The presence of vegetable matter in the lungs is possible
only due to aspiration. Hence, the answer (c).
6. Anaplastic Lymphoma Kinase (ALK) gene expression is seen in which of the
following tumor?
(AIIMS May 2017 Pattern)
(a) Inflammatory myofibroblastic tumor
(b) Synovial sarcoma
(c) Fibromatosis
(d) Ewing sarcoma
Ans. (a) Inflammatory myofibroblastic tumor
(Ref: Robbins 9/e p894)
Inflammatory myofibroblastic tumor

• More common in children


• Equal male-to-female ratio
• Presenting symptoms include fever, cough, chest pain, and hemoptysis.
• Grossly, the lesion is firm, 3 to 10 cm in diameter, and grayish white.
• Microscopically, there is proliferation of spindle-shaped fibroblasts and myofibroblasts,
lymphocytes, plasma cells, and peripheral fibrosis.
• Some of these tumors have activating rearrangements of the anaplastic lymphoma kinase
(ALK) gene located on 2p23.

Disclaimer
Any resemblance to an actual question is purely coincidental.
• Albumin does not appear in the urine because of having a negative charge (and not size)
• Proteoglycan in the glomerular basement membrane responsible for charge dependent filtration.
• Inheritance of adult polycystic kidney disease is Autosomal dominant affected by genes like PKD 1
(polycystin-1) and PKD 2 (polycystin) whereas inheritance of childhood polycystic kidney disease is
Autosomal recessive.
• Hematuria of non-glomerular origin as in renal stones have isomorphic RBC. Dysmorphic red cells in urine
are seen with glomerular diseases.
• Tamm Horsfall protein is produced by cells of the ascending limb of Loop of Henle.
• Commonest cause of pediatric glomerulonephritis is post strepotocccal glomerulonephritis. It is a type III
hypersensitivity reaction.
• Crescent formation (Fibrin + parietal epithelial cells + WBCs) is characteristic of RPGN. Number of
crescents also decide the prognosis of the patient.
• Hallmark of IgA nephropathy (commonest glomerulonephritis in adults) is recurrent hematuria after 1–2 days
of an upper respiratory tract infection and presence of IgA mesangial deposits.
• Nephrotic syndrome is characterised by massive proteinuria (> 3·5 gm/day), hypoalbuminemia, edema,
hyperlipidemia and lipiduria. All protein are reduced in nephritic syndrome except fibrinogen.
• Most common cause of nephrotic syndrome in children (Minimal change disease), adults (focal segmental
glomerulosclerosis) and elderly (Membranous glomerulopahy).
• Heymann rat glomerulonephritis true is Immune complex against Heymann antigen (megalin) and is a model
for membranous glomerulopathy.
• Collapsing glomerulopathy (a variant of FSGS) is seen in AIDS patients.
• Subepithelial deposits are seen in: PSGN, membranous GN and RPGN.
• Subendothelial deposits are seen in: MPGN-type I, SLE (lupus nephritis class III & IV).
• Finnish type’ of congenital nephrotic syndrome: NPHS-1 with mutation in ‘Nephrin’.
• Steroid resistant congenital nephrotic syndrome: NPHS-2 with mutation in ‘podocin’. ‘
• Alport syndrome is best diagnosed by electron microscopy (basket weave appearance)
• Renal lesions in diabetes mellitus: basement membrane thickening (commonest lesion), nodular
glomerulosclerosis (KW lesion; most characteristic lesion). Renal changes are dependent mostly on duration
of the disease.
• Commonest pathological renal finding in benign hypertension is: Hyaline arteriosclerosis.
• Renal lesions is malignant hypertension include fibrinoid necrosis (necrotizing arteriolitis), hyperplastic
arteriolitis (Onion skin lesion) and flea bitten kidney.
• Causes of renal papillary necrosis: Diabetes (most common), obstructive uropathy, analgesic nephropathy
and sickle cell disease.
• Pyelonephritis is most commonly caused by E. coli and its most predisposing factor is vesico-ureteric reflux
(in chronic pyelonephritis)
• Xanthogranulomatous pyelonephritis is seen with Proteus (most common) and is characterised by the
presence of foam cells (Xanthoma cells).
• Fibronectin nephropathy is inherited as an Autosomal dominant disease.
• Most common type of renal all carcinoma is Clear cell carcinoma having chromosome 3 defects.
• Sarcomatoid change in the renal cancer has poor prognosis.
• Chromophobe variant of RCC is associated with Hypodiploidy
• Bilateral renal cell carcinoma is associated with: Von-Hippel Lindau syndrome.
• Michaelis Gutmann bodies are seen in Malakoplakia.
• Most common type of bladder cancer is the transitional cell carcinoma (TCC).
• Most common cause of painless hematuria: urinary bladder cancer
• Most common cause of painful hematuria: renal stones.

Kidney is the organ of the body responsible for the removal of nitrogenous products from the blood.

SOME IMPORTANT DEFINITIONS:


1. Azotemia – It refers to an elevation of Blood Urea Nitrogen (BUN) and creatinine levels due to
reduced glomerular filtration rate. It can be due to:
a. Pre-renal cause – Associated with decreased perfusion as in shock, hemorrhage and heart
failure.
b. Renal cause – Due to intrinsic defect in the kidney.
c. Post renal cause – Associated with obstruction to urine outflow.
Patients do not require dialysis at this stage.
2. Uremia – Azotemia + Clinical signs and symptoms + Biochemical abnormalities
– There is secondary presence of uremic gastroenteritis, peripheral neuropathy and uremic
pericarditis.
Patients require dialysis at this stage.

Fig. 1: Polycystic Kidney Disease.

CYSTIC DISEASES OF THE KIDNEY


Polycystic Kidney Disease (PKD)
Adult PKD Childhood PKD
• Autosomal dominantQ inheritance • Autosomal recessiveQ inheritance.
• Mutations in PKD1, PKD2 and PKD3 genes (PKD1 produces • Mutation in PKHD1 gene which produces
Polycystin proteinQ). fibrocystin proteinQ.
• Asymptomatic till middle age • Presents in infancy with renal insufficiency.
• Clinical features include hematuria, hypertension, UTI and renal • Associated with multiple hepatic cysts and
stones. congenital hepatic fibrosisQ.
• Extrarenal manifestations • Grossly, bilaterally enlarged kidney with
1. Cysts in other organs like liverQ (most commonly), small cysts in cortex and medulla having
pancreas, spleen and ovary. their long axis at right angle to capsule.
2. Berry aneurysmQ.
3. Colonic diverticulaQ.
4. Mitral valve prolapse and aortic regurgitationQ.
• Grossly, bilaterally enlarged kidneys with multiple cysts
containing serous or hemorrhagic fluidQ.
MNEMONIC FOR APKD:
1. Clinical Features: (11 B’s)
Signs: Bloody urine
– Bilateral pain [vs. stones, which are usually unilateral pain]
– Blood pressure up
– Bigger kidneys
Complications
– Berry aneurysm
– Biliary cysts
– Bicuspid valve [prolapse and other problems]
Accelerators:
– Boys
– Blacks
– Blood pressure high.

GLOMERULAR DISEASES

PATHOGENESIS OF GLOMERULAR INJURY


a. In Situ immune complex disease
1. Anti-GBM antibody induced nephritis
The antibodies are directed against intrinsic fixed antigens that are normal components of the
glomerular basement membrane (GBM) proper resulting in a diffuse linear pattern of
staining for the antibodies by immunofluorescence techniques. This is the model of anti-
GBM disease (Goodpasture syndrome) which is caused by antibodies against non-
collagenous domain of the alpha 3 chain of collagen type IV.
2. Heymann nephritis
The antibodies are directed against intrinsic fixed antigen called Heymann antigen or ‘megalin’
located on visceral epithelial cells resulting in complement activation and the formation of
subepithelial deposits and a granular pattern of staining for the antibodies by
immunofluorescence techniques.
3. Antibodies against planted antigens
Antibodies can react with antigens that are not normally present in the glomerulus but are
“planted” there. These antigens include cationic molecules like DNA, nuclear proteins,
bacterial, viral and parasitic products and drugs. A granular pattern of staining is observed
by immunofluorescence techniques.

b. Circulating immune complex disease


The glomerular injury is caused by entrapment of circulating antigen-antibody complex within
the glomeruli. This results in complement activation, leukocytic infiltration and proliferation of
glomerular and mesangial cells. The endogenous antigens include DNA and tumor antigens
whereas the exogenous antigens include infectious products. Electron microscopy reveals the
presence of subendothelial deposits or subepithelial deposits. By immunofluorescence
microscopy, the immune complexes can be seen as granular deposits along the basement
membrane, in the mesangium or both.
Nephritic syndrome Nephrotic syndrome

1. Proteinuria (1-2g/d) 1. Severe proteinuria (>3.5g/d)


2. Hematuria 2. Hypoalbuminemia (<3g/dl)
3. Hypertension 3. Edema
4. Azotemia, Oliguria 4. Hyperlipidemia, lipiduria

Thrombotic and thromboembolic complications are common in nephrotic syndrome due to loss
of anticoagulant factor (e.g. antithrombin III, protein C and S) combined with increased platelet
activation. Renal vein thrombosis is most often a consequence of this hypercoagulative state
specially in case of nephrotic syndrome associated with membranous nephropathy in adults. There
is also increased synthesis of fibrinogen in the liver.

TERMINOLOGY IN GLOMERULAR DISEASES


Each region of a renal biopsy is assessed separately. By light microscopy, glomeruli (at least 10
and ideally 20) are reviewed individually for discrete lesions.

According to percentage of glomeruli affected


• Focal (Less than 50%)
• Diffuse (>50%)
Injury in each glomerular tuft can be
• Segmental (involving a portion of the tuft)
• Global (involving most of the glomerulus)
According to characteristic of lesion
• Proliferative when showing increased cellularity.
• Endocapillary (Proliferation of cells in the capillary tuft)
• Extracapillary (Extension of proliferation into Bowman’s space)
• Synechiae (Epithelial podocytes attach to Bowman’s capsule)
• Crescents (when fibrocellular/fibrin collections fill all or part of Bowman’s space)
• Sclerosis (Acellular, amorphous accumulations of proteinaceous material throughout the tuft with loss of functional
capillaries and normal mesangium)
NEPHRITIC SYNDROME

Acute Proliferative (Poststreptococcal) Glomerulonephritis


It is seen 1-4 weeks after a skin or pharyngeal infection caused by group A β hemolytic
streptococci (particularly strains 12, 4 and 1) usually in children. Activation of complement system
results in consumption of complement proteins leading to transiently low complement levels (for 6-8
weeks). The antigen responsible for the development of this condition is a cytoplasmic antigen
called endostreptosin and a cationic proteinase antigen called nephritis strain associated protein or
NSAP Q.

Fig. 2: Post Streptococcal Glomerulonephritis.


Contd...
Contd...
Clinical features
Malaise, fever, nausea, oliguria and hematuria leading to smoky or cocoa colored urine, periorbital edema and mild to
moderate hypertension.
Microscopic findings
Presence of hypercellular glomeruli due to leukocytic infiltration, proliferation of endothelial and mesangial cells.
Immunofluorescence microscopy shows the presence of IgG, IgM and C3 deposits in the mesangium and along the
basement membrane giving ‘starry sky’ appearance.
Investigations
There is elevated levels of antistreptolysin O or ASO and anti DNAase antibodiesQ (indicative of streptococcal
infection) and transiently reduced levels of serum C3.
Management
It is done with fluid restriction. The majority of the patients recover and only a small fraction may progress to chronic
glomerulonephritis.

RAPIDLY PROGRESSIVE (CRESCENTIC GLOMERULONEPHRITIS)

It is characterized by rapid and progressive loss of renal function associated with rapid development
of renal failure in weeks or months.
It is of the following three types with the common feature of severe glomerular injury.
Crescentic Glomerulonephritis

Type I RPGN (Anti-GBM antibody) Type II RPGN (Immune complex) Type III RPGN (Pauci immune)
• Idiopathic • Idiopathic • Idiopathic.
• Goodpasture’s syndrome. • Post infectious • ANCA associated.
• SLE, Henoch Schonlein Purpura. • Wegener’s granulomatosis.
• Microscopic polyangiitis.
Immunofluorescence finding Immunofluorescence finding Immunofluorescence finding
• Linear GBM deposits of IgG and • “Lumpy bumpy” granular pattern of • No immunoglobulin or complement
C3 staining. deposits in GBM.

The characteristic histologic feature is the presence of glomerular crescents (in > 50% of
glomeruli seen on biopsy) which are composed of proliferation of parietal cells, leukocytic
infiltration, and monocyte and macrophage movement in the urinary space. The fibrin is prominent
within the cellular layers of the crescents. Electron microscopy shows the presence of ruptures in
the glomerular basement membrane and subepithelial deposits.
Clinical features include hematuria with RBC casts in the urine, subnephrotic proteinuria,
hypertension and edema. The prognostic features are mentioned as follows:

Fig. 3: Crescentic GN.

Prognosis of RPGN

Poor prognostic factors Good prognostic factors

• Oliguria and azotemia at presentation • Pauci immune RPGN has best prognosis
• More than 80% circumferential crescents have • Non- circumferential crescents in less than 50%
poor response to therapy glomeruli have indolent course.
• Glomerular tuft necrosis, global glomerular sclerosis, • Associated endocapillary proliferation is a good
gaps in Bowman capsule and interstitial fibrosis prognostic factor

NEPHROTIC SYNDROME

Membranoproliferative (Mesangiocapillary) Glomerulonephritis (MPGN)


It is characterized by the presence of basement membrane alterations, proliferation of glomerular
cells (particularly in the mesangium) and leukocytic infiltration. These patients may have a nephritic
or nephrotic picture.

• In type II MPGN, the serum of these patients contain C3 nephritic factor (C3NeF) which causes stabilization
of alternate C3 convertase thereby causing persistent degradation of C3 and resulting
hypocomplementemia.

Light microscopy shows the lobular appearance of glomeruli which are hypercellular (due to
leukocytic infiltration and proliferation of capillary endothelial cells and mesangial cells). The GBM is
thickened and the synthesis of new basement membrane causes ‘tram-track’ or ‘double contour’
appearance appreciated with silver or PAS stains.
Electron microscopically, type I MPGN is having the presence of subendothelial electron dense
deposits. Immunofluorescence studies demonstrate the deposition of C3, IgG and early complement
proteins (C1q and C4) in the glomeruli. In type II disease, there is presence of dense deposits within
the GBM, so it is also called as dense deposit disease. Immunofluorescence studies demonstrate
the linear or granular deposition of C3 whereas IgG and early complement proteins are absent. C3
may also be present in mesangial rings.
Clinical presentation of the patient is nephrotic syndrome with the nephritic component of
hematuria. There is high incidence of recurrence in transplant patients.
Secondary MPGN is invariably type I but the exact mechanism is unknown.
LIPOID NEPHROSIS (MINIMAL CHANGE DISEASE)

It is the commonest cause of nephrotic syndrome in the children (peak incidence in 2-6 years)
characterized by the diffuse effacement of the foot processes of epithelial cells of the glomeruli
which appear normal by light microscopy. So, the other name of the disease is minimal change
disease.
There is absence of immune deposits but presence of visceral epithelial injury due to abnormal
secretion of lymphokines by T cells resulting in the loss of glomerular polyanions responsible for low
molecular weight proteinuria (selective proteinuria). Mutation of the protein nephrin causes a
hereditary form of congenital nephrotic syndrome (Finnish typeQ).
Microscopy
Light microscopy shows the normal glomeruli with lipid accumulation in proximal tubular cells (lipoid nephrosis)
whereas the electron microscope reveals the presence of effacement of foot processes of podocytes.

Clinical features

There is massive proteinuria particularly loss of albumin (highly selective proteinuriaQ) in the absence of hypertension
or hematuria.
MEMBRANOUS GLOMERULOPATHY (MGN)
It is a common cause of nephritoc syndrome in the adults characterized by the diffuse thickening of
the glomerular capillary wall and accumulation of electron dense, immunoglobulin-containing
deposits along the subepithelial side of the basement membrane. Its causes include:

Idiopathic Secondary
• Seen in most of the patients (in • Drugs (penicillamine, captopril, NSAIDs)
85% patients) • Malignancies like carcinoma of colon and lung, melanoma
• Infections like hepatitis B and C, syphilis, malaria, schistosomiasis
• Systemic diseases like SLE, diabetes mellitus, thyroiditis

The disease has resemblance to the Heymann nephritis modelQ of glomerular injury mediated
by immune complex formation against a visceral epithelial antigen called Heymann antigen or
megalin. The immune complex mediated formation of membrane attack complex C5b-C9 causes
activation of glomerular epithelial and mesangial cells which release oxidants and proteases that
cause vessel wall injury and protein leakage.
Microscopy
Light microscopy shows the diffuse membrane-like thickening of the glomerular capillary wall. Basement membrane
projections as ‘spikes’Q are seen on silver stains. Electron microscopy reveals effacement of the foot process of
podocytes and presence of subepithelial deposits.
Immunofluorescence
It demonstrates the linear and granular deposition of C3 and IgG.
Clinical presentation
It is nephrotic syndrome with the excretion of higher weight globulins along with albumin (non selective proteinuria)
which is poorly responsive to steroids.

FOCAL SEGMENTAL GLOMERULOSCLEROSIS (FSGS)

This is now the commonest cause of nephrotic syndrome and is characterized by the presence of
focal (only some glomeruli are affected) and segmental (only a part of the glomerulus is affected)
sclerosis of the glomeruli.
Causes
1. Idiopathic
2. Secondary:
– Associated with loss of renal tissue as unilateral renal agenesis or advanced stages of
reflux nephropathy or hypertensive nephropathy.
– Associated with conditions like Sickle cell anemia, HIV infection, Heroin abuse, Obesity.
– Inherited due to mutations in genes like nephrin, podocin and a-actinin 4.
Degeneration and focal disruption of the visceral epithelial cells is the hallmark feature of
focal segmental glomerulosclerosis. The hyalinosis and sclerosis are due to protein entrapment.
Mutation of the protein podocin and α-actinin 4 results in the development of autosomal recessive
and autosomal dominant forms of focal segmental glomerulosclerosis respectively.

Microscopy
Light microscopy reveals the focal segmental sclerosis and hyalinization of the glomeruli. Electron microscopy
demonstrates the diffuse effacement of the podocytes, focal detachment of the epithelial cells and increased
mesangial matrix in the sclerotic areas. Five mutually exclusive variants of FSGS may be distinguished by the
pathological findings seen on renal biopsy
• Collapsing variant
• Glomerular tip lesion variant
• Cellular variant
• Perihilar variant
• Not otherwise specified variant (NOS) (Commonest)
Immunofluorescence
It demonstrates C3 and IgM deposits in the sclerotic areas and the mesangium.

It affects all age groups and is characterized by the clinical features of non-selective
proteinuria, reduction in GFR and presence of hypertension and poor response to corticosteroids.
There is high incidence of recurrence in transplant patients. Children have better prognosis than
adults.

IgA NEPHROPATHY (BERGER DISEASE)

This is the commonest glomerulonephritis in the world. Q It is characterized by the presence of


prominent IgA deposits in the mesangial regions and clinically by gross or microscopic hematuria. It
can occur either alone or secondary to liver or intestinal disease.
Pathogenesis: The patient usually develops an initial respiratory or gastrointestinal infection
resulting in increased synthesis of IgA1 which gets trapped in the mesangium. Here, these
immunoglobulins cause activation of alternate pathway of complement system resulting in
glomerular injury. Any liver disease causes reduced clearance of IgA whereas intestinal disease
causes increased mucosal production of IgA.
Fig. 4: Berger disease (IgA deposits in mesagium shown by arrows with immunofluorescence microscopy).

Microscopic examination reveals the mesangial widening and proliferation which on immunofluorescence reveals
the presence of the mesangial deposition of IgA usually with C3 and properdin. Mesangioproliferative
glomerulonephritis is seen more commonly than focal proliferative and crescentic (least) glomerulonephritis.

Clinical features: Any age group of the patients (more in children and young adults) may be
affected who present with gross hematuria 3-4 days after an infection of the respiratory, GI or
urinary tract. Almost 1/3rd of the patients have microscopic hematuria. This hematuria lasts for
some days to subside and recur every few months. Recurrence of the disease in the transplanted
kidneys is frequent.

Alport Syndrome

This is a nephritic disorder characterized by the involvement of the triad of kidney, ear and eye.
There is a fundamental defect in the a5-chain of collagen type IV resulting in defective GBM
synthesis responsible for the involvement of the organs where this collagen is found.
Microscopically
• Light microscopy glomerular involvement in the form of the presence of foam cells in the interstitial cells.
Electron microscopy (Diagnostic for this disorderQ)
• It shows the presence of irregular foci of thickening and thinning in the GBM with splitting and lamination of
lamina densa called as “basket weave appearance”. Absence of a5 staining is seen even on skin biopsy apart
from glomerular and tubular basement membrane.
Clinical features
• Males in the age group of 5-20 years are more frequently affected presenting with gross or microscopic
hematuria, nerve deafness and ocular features (posterior cataract, lens dislocation and corneal dystrophy).
Fig. 5: Alport syndrome (basket weave appearance).

Thin Membrane Disease (Benign Familial Hematuria)

This is a disease characterized by the presence of familial asymptomatic hematuria and thinning of
the basement membrane to 150-250 nm (normal GBM thickness is 300-400 nm). There is a defect
in the α3 and α4 chains of collagen type IV.

Goodpasture Syndrome

It is an autoimmune disorder (type II hypersensitivity reaction) having the presence of antibodies


against non-collagenous domain of a3 chain of collagen IV causing destruction of basement
membrane in renal glomeruli and pulmonary alveoli.
The disease affects young men more commonly and is associated with smoking, viral infections
or exposure to hydrocarbon solvents (workers in dry cleaning industry).
Histologically, the alveoli show focal necrosis, intra-alveolar hemorrhage, presence of
hemosiderin laden macrophages and hypertrophy of type II Pneumocytes. The renal involvement is
in the form of either focal proliferative or crescentic glomerulonephritis.
Immunofluorescence studies show linear deposits of immunoglobulins along alveolar septa
and GBM.
Clinical features: Interstitial hemorrhagic inflammation leads to the predominant symptoms of
hemoptysis and hematuria. Management is done by plasmapheresis.
Goodpasture syndrome Goodpasture disease

• Affects young males • Affects elderly females


• Lungs are affected • Lungs are not affected
• Poor prognosis • Good prognosis

SUMMARY OF GLOMERULAR DEPOSITS


Subepithelial Subendothelial Basement membrane Mesangial
*Acute GN (like PSGN) *MPGN (Type I) *MPGN (Type II) *IgA nephropathy
*Membranous GN *SLE *Membranous Glomerulopathy *HSP
*Heymann GN *Acute GN *Anti-GBM diseases like
*RPGN (some cases) RPGN and Goodpasture
*MPGN (Type I) rarely syndrome

Concept: Proliferative Glomerulonephritis


Glomerular lesions with increased cells in the tufts are often known as proliferative glomerulonephritis. It is seen in the
following conditions:
• SLE (particularly class II, mesangial hypercellularity defined as >3 cells in mesangial regions)
• HIV
• Membranoproliferative glomerulonephritis
• Neoplasia (particularly CLL and MALT lymphoma)
• Post streptococcal glomerulonephritis.

CHRONIC GLOMERULONEPHRITIS

It is the final stage of many forms of glomerular disease and is characterized by progressive renal
failure, uremia and ultimately death.
Clinical features include anemia, anorexia, malaise, proteinuria, hypertension and azotemia.
Grossly there is presence of small, shrunken kidneys. There is fine and symmetrical scars.
Microscopically, there is hyalinization of glomeruli, interstitial fibrosis, atrophy of tubules, and a
lymphocytic infiltrate. Management is done by dialysis and renal transplantation.

Diabetic Nephropathy

It is a disorder characterized by hyperglycemia resulting in formation of advanced glycosylation end


products responsible for GBM thickening and increased mesangial matrix. There is also concomitant
presence of hemodynamic changes resulting in glomerular hypertrophy with increased glomerular
filtration area. Both of these contribute to the development of proteinuria.

Morphological features include:


Capillary basement membrane thickening
It is the earliest morphological abnormality which is seen in virtually all diabetics irrespective of proteinuria. It is best
detected with electron microscopy and is associated with thickening of the tubular basement membrane.
Diffuse mesangiosclerosis
There is diffuse increase in the mesangial matrix usually consisting of PAS positive material associated with GBM
thickening.
Nodular glomerulosclerosis
It is a highly specific lesion of diabetes and is also known as intercapillary glomerulosclerosis or Kimmelsteil Wilson
lesion. It consists of PAS positive nodules of matrix situated in the periphery of the glomeruli. It is associated with
prominent accumulation of hyaline material in capillary loops and Bowman’s capsule known as “fibrin caps” and
“capsular drops” respectively.

There is also presence of hyalinizing arteriolar sclerosis (affects characteristically bothQ


afferent and efferent arterioles), pyelonephritis and papillary necrosis.
Clinical features: The increased GFR is associated with microalbuminuria.It is very important
clinical predictor of development of diabetic nephropathy later on. The protein excretion then
reaches subnephrotic proteinuria followed by nephrotic proteinuria. Patients of type I diabetes may
also have hypertension which further aggravates the renal disease.

Differences between benign nephrosclerosis and malignant nephrosclerosis


Features Benign nephrosclerosis Malignant nephrosclerosis
Condition Benign hypertension, DM, elderly age Malignant hypertension
Gross appearance of kidneys Leather grain appearance Flea bitten appearance due to tiny
petechial hemorrhage
Small or reduced kidneys Variable size of the kidney
Microscopy 1. Narrowing of the lumens of arterioles (i) Hyperplastic arteriolitis (onion skinning)
caused by thickening and hyalinization due to proliferation and elongation of
of the walls (hyaline arteriolosclerosis) smooth muscle cells.
2. Fibroelastic hyperplasia of arteries (ii) Necrotizing glomerulitis (neutrophils
and arterioles infiltration and thrombosed capillaries)
(iii) Fibrinoid necrosis of arterioles
(iv) Necrotizing arteriolitis

Activity of renin angiotensin Normal ↑ed


system

Proteinuria Mild Marked


Sign of malignant Absent Present such as retinopathy
hypertension encephalopathy
Renal failure Rare More common
Fig. 6: Nodular Glomerulosclerosis in diabetes mellitus.

TUBULAR DISEASES

Acute Tubular Necrosis (ATN)/Acute Kidney Injury (AKI)

• It is a disorder characterized by destruction of tubular epithelial cells resulting in loss of renal


function. It is the most common cause of acute renal failure (ARF).

Contd...
Contd...

ATN

Ischemic ATN Nephrotoxic ATN


• Most common cause of ATNQ. • Due to drugs (gentamicin, methicillin, radio contrast
• Due to decreased blood flow (in shock, hemorrhage, agents, organic solvents, ethylene glycol, phenol,
hypotension or dehydration). pesticides, myoglobin).
• Presence of focal tubular necrosis affecting proximal • Diffuse necrosis of proximal convoluted tubular
straight tubule and ascending limb of loop of HenleQ. segments and ascending Henle’s loopQ occur.
• In both ischemic and nephrotoxic ATN, there is rupture of tubular basement membranes
(tubulorrhexis) and occlusions of lumen by a cast mostly seen in distal convoluted tubule and
collecting ducts.

Pathogenesis of ATN

CLINICAL FEATURES OF ATN


1. Initiation phase–Lasts for about 36 hours and is characterized by slight decline in urine output
with a rise in blood urea nitrogen (BUN).
2. Maintenance phase–Oliguria, salt and water overload, hyperkalemia, metabolic acidosis and
rising BUN concentration.
3. Recovery phase – Steady increase in urine volume (up to 3L/d), hypokalemia and increased
vulnerability to infections.

PYELONEPHRITIS

It is the infection involving the renal pelvis, tubules and interstitium. It can be of two types:
• Acute Pyelonephritis is the renal lesion associated with bacterial urinary tract infection (UTI)
• Chronic pyelonephritis, bacterial infection is associated with other factors including
vesicoureteral reflex and obstruction.
There is initial colonization of the distal urethra followed by movement in to the bladder due to
frequent instrumentation or catheterization. Bladder dysfunction or outflow obstruction causes stasis
of urine promoting the bacterial multiplication. From the bladder, it is the incompetence of the
vesicoureteral valve allowing retrograde urine flow in to the ureters (vesicoureteral reflux). The
infected urine enters the renal pelvis more commonly in the upper and lower poles of the kidney
(intrarenal reflux).
Predisposing factors include Vesicourethral reflux, urethral instrumentation, diabetes mellitus,
pregnancy, urinary obstruction, benign prostatic hypertrophy and other renal pathology.
Clinical features: Females more commonly affected than males (because of shorter urethra,
hormonal changes favoring bacterial adhesion to mucosa, absence of antibacterial property as in
prostatic fluid and urethral trauma during sexual intercourse). There is presence of fever with chills,
dysuria (painful micturition), increased frequency and urgency along with costovertebral angle
tenderness.
Urinalysis reveals presence of leukocytes particularly neutrophils (pyuria) and WBC castsQ
suggestive of renal involvement (because casts are formed only in tubules).
Complications include papillary necrosis (usually bilateral), pyonephrosis and perinephric
abscess.
Fig. 7: Chronic glomerulonephritis having hyalinised glomeruli.

Differences between chronic glomerulonephritis and chronic pyelonephritis


Traits Chronic glomerulonephritis Chronic pyelonephritis
Causes Various glomerulonephritis Reflux nephropathy or chronic
obstructive pyelonephritis
Pathogenesis End stage glomerular disease due to specific Chronic tubulo-interstitial
glomerulonephritis inflammation and scarring
associated with renal disorder
Gross appearance of surface Diffusely, granular, cortical surfaces Depressed area on dilated and
blunted calyx
Scar Fine and Symmetrical Coarse and Asymmetrical
Glomeruli Reduced in number with obliteration Normal; may show periglomerular
fibrosis
Tubules Atrophied Atrophy in some and hypertrophy
in others filled with colloid casts
(thyroidisation)
Renal pelvis and calyx Normal Dilated
Interstitial and periglomerular Mild More marked
fibrosis
Clinical features Insidious in onset May be asymptomatic or present
Proteinuria, azotemia, hypertension, edema with back pain, fever, polyuria,
nocturia, pyuria, bacteriuria with
gradual onset of hypertension and
renal insufficiency

Fig. 8: Chronic pyelonephritis having thyroidisation of tubules.

Xanthogranulomatous pyelonephritis (XPN)

Unusual variant of chronic pyelonephritis. Most cases occur in the setting of obstruction due to infected renal stones.

Grossly

• Yellow, lobulated masses diffusely replace the renal architecture.

Microscopically

• There is massive destruction of the kidney due to granulomatous tissue containing lipid-laden macrophages; the
appearance may be confused with renal malignancy.

Clinical features

• It most often occurs in middle-aged women with a history of recurrent urinary tract infections.
• Typical presenting symptoms include flank pain, fever, malaise, anorexia and weight loss.
• A unilateral renal mass can usually be palpated on physical examination.
Diagnosis

• Examination of the urine confirms the presence of urinary tract infection. Urine culture typically demonstrates
Enterobacteriaceae. The most common organisms associated with XPN are E. coli, Proteus mirabilis,
Pseudomonas, Streptococcus faecalis and Klebsiella.
Contd...
Contd...
Contd...
Contd...

Renal Stones/Urinary Calculi

In the urinary tract, the most common site of origin of stone is the kidney. Males between the age
group of 20-30 years are most commonly affected. Renal stones are formed either due to
supersaturation of urine (constituent concentration exceeding the solubility) or deficiency of crystal
formation inhibitors like pyrophosphate, diphosphonate, citrate, osteopontin and nephrocalcin.

Fig. 9: Struvite stones or Triple stones.


Renal stones
Calcium oxalate Uric acid Magnesium ammonium Cystine
phosphate
Square envelope shape Diamond / barrel shape Coffin lid shape Hexagonal shape

• Most common stoneQ. • Seen with • Also called “struvite • Due to genetic defec
• Idiopathic hyperuricemia (gout, stones” or “triple the absorption of cy
hypercalciuria is the leukemias)Q. stones”Q. resulting in cystinuriaQ
commonest causeQ. • Seen in acidic urine (pH • Formed in alkaline urine • Formed in acidic urine
• Seen in acidic urine. < 5.5)Q. particularty in infection
• Also associated with • Radiolucent stoneQ. with proteusQ. • Change color from i
hypocitraturiaQ. • Occupy large part of renal yellow to green on
• Radiopaque stoneQ. pelvis, so, called as exposureQ.
“staghom calculi”Q.

Note: Uncommon renal stones can be composed of xanthine (due to xanthine oxidase deficiency), indinavir (in
AIDS patients taking this drug) or triamterene (Patients on this antihypertensive medication). All these are
radiolucent stonesQ.

Stones are usually unilateral (80% of patients) and are deposited in renal pelvis and bladder. If
the developing stone takes the shape of the pelvicalyceal system, it is called staghorn calculi.
Clinical symptoms include hematuria, urinary obstruction, renal colic (if they pass into the
ureters) and increased chances of infection. Most of the renal stones are managed surgically.

URINARY CASTS

Hyaline casts
• This is a normal constituent of urine and has no attached significance.
• Tamm Horsfall protein is a protein secreted by epithelial cells of loop of HenleQ.
• This protein may be exerted as Hyaline cast.
• May be seen in concentrated urine, febrile disease, after heavy exercise

RBC cast

Is suggestive of glomerular injury.Q


White cell casts
Are suggestive of interstitial injury and may be seen in interstitial nephritis. WBC cast with bacteria indicate
Pyelonephritis.Q
Broad granular casts
Arise in the dilated tubules of enlarged nephrons that have undergone compensatory hypertrophy in response to
reduced renal mass i.e. chronic renal failure.Q
Pigmented muddy brown granular casts

Are suggestive of ischemic or nephrotoxic injury i.e. Tubular NecrosisQ.


Lipid cast
• Seen when there is fatty degeneration of the tubular epithelium.
• Also seen in nephrotic syndrome, lupus and toxic renal poisoningQ.

RENAL TUMORS

Benign Tumors
• Angiomyolipoma – It is a hamartoma composed of fat, smooth muscle and blood vessels and is
associated with tuberous sclerosis.
• Oncocytoma Tumor arising from intercalated cells of collecting ducts having large, eosinophilic
cells which have numerous mitochondria. The cells have expression of carbonic anhydrase C
and band 3 protein.

Wilms’ Tumor (Nephroblastoma)

Wilms’ tumor is the most common primary renal tumor of childhood in USA. This tumor’s peak
age is 2-5 years. The risk of Wilms’ tumor is increased in association with at least three
recognizable groups of congenital malformations:

Fig. 10: Wilm Tumor: triphasic combination of blastemal (B), stromal (S), and epithelial (E) cells.

WAGR syndrome
It is characterized by 33% chance of developing Wilms’ tumor, Aniridia, Genital anomalies, and mental Retardation.
Patients with WAGR syndrome carry constitutional (germline) deletions of two genes WT1 and PAX6 both located at
chromosome 11p13Q.
Denys-Drash syndrome
It is characterized by gonadal dysgenesis (male pseudohermaphroditism) and early-onset nephropathy leading to
renal failure. The characteristic glomerular lesion in these patients is a diffuse mesangial sclerosis. These patients
also have germline abnormalities in WT1. In addition to Wilms’ tumors these individuals are also at increased risk for
developing germ-cell tumors called gonadoblastomas.

Beckwith-Wiedemann syndrome
It characterized by enlargement of body organs (organomegaly), macroglossia, hemihypertrophy, omphalocele and
abnormal large cells in adrenal cortex (adrenal cytomegaly). The genetic locus involved in these patients is in band
p15.5 of chromosome 11 called “WT2”. In addition to Wilms’ tumors patients with Beckwith-Wiedemann syndrome are
also at increased risk for developing hepatoblastoma, adrenocortical tumors, rhabdomyosarcomas and pancreatic
tumors.

b catenin mutations are synergestically acting with WT1 mutations to cause cancer. Nephrogenic
rest are precursor lesions of Wilms tumor.

MORPHOLOGY
Grossly, Wilms tumor tends to present as a large, solitary, well-circumscribed mass and on cut
section, the tumor is soft, homogeneous, and tan to grey with occasional foci of hemorrhage, cyst
formation, and necrosis.

Microscopically, Wilms’ tumors are characterized by the classic triphasic combination of blastemal, stromal,
and epithelial cell types (immature glomeruli and tubules) seen in majority of lesions.

The tumor usually presents as a large abdominal mass, which may extend across the midline
and down into the pelvis. The patient may also present with fever and abdominal pain, with
hematuria, or rarely, with intestinal obstruction as a result of pressure from the tumor.
The prognosis for Wilms tumor is generally very good, and excellent results are obtained with a
combination of nephrectomy and chemotherapy.

Renal Cell Carcinoma (Hypernephroma/Grawitz Tumor)

It is the most common malignant cancer of the kidney affecting the poles of the kidney (more
commonly upper pole). Males are more frequently affected (M:F ratio is 2 to 3:1) in the age group
of 6-7th decade.

Fig. 11: Renal cell carcinoma: Clear cell subtype.


Fig. 12: Renal cell carcinoma: Papillary subtype.

Risk factors

Nongenetic factors

• Tobacco (most important risk factor)Q


• HypertensionQ
• ObesityQ
• Tuberous sclerosisQ
• Estrogen therapyQ
• Asbestos exposureQ
• Chronic renal failure and acquired cystic diseaseQ
• Von Hippel Lindau syndrome
Genetic factors
• Trisomy 7 shows increased expression of MET which is a proto-oncogene having tyrosine kinase receptor activity
thereby resulting in development of papillary type of renal cancer.
• t(X; 1) causes translocation of PRCC gene to fuse with TFE-3 gene on X chromosome and fusion gene increases
the risk of papillary renal cell carcinoma (PRCC) particularly in children.

• Hereditary leiomyomatosis and renal cell cancer syndrome: This autosomal dominant disease is caused by
mutations of the FH gene, which expresses fumarate hydratase, and is characterized by cutaneous and uterine
leiomyomata and an aggressive type of papillary carcinoma with increased propensity for metastatic spread.
• Birt-Hogg-Dubé syndrome: The autosomal dominant inheritance pattern of this disease is due to mutations
involving the BHD gene, which expresses folliculin. The syndrome features a constellation of skin (fibrofolliculomas,
trichodiscomas, and acrochordons), pulmonary (cysts or blebs), and renal tumors with varing histologies.
• Xp11 translocation carcinoma is a genetically distinct subtype of renal cell carcinoma. It often occurs in young
patients and is defined by translocations of the TFE3 gene located at Xp11.2 with other genes resulting in over
expression of TFE3. The neoplastic cells consist of clear cytoplasm with a papillary architecture.
Fig. 13: Renal Cancer.

Clinical features include the classical triadQ of hematuria (earliest and most common
symptomQ; usually intermittent), palpable mass and flank pain.

Paraneoplastic syndromes associated with RCC


• Elevation of erythrocyte sedimentation rate
• Polycythemia (Due to erythropoietin)
• Stauffer syndrome (non metastatic hepatic dysfunction)
• Hypertension (Due to renin)
• Anemia and fever
• Hypercalcemia (Due to PTH related peptide)
• Cushing syndrome (Due to corticosteroid synthesis)
• Feminization or masculinization (Due to gonadotropin release)
• May also cause amyloidosis, eosinophilia or leukemoid reactionQ

Metastasis is usually by the hemotogenous route and the organs affected include lungsQ
(most common), bones, regional lymph nodes, liver, adrenals and brain.
Management: Partial/total nephrectomy is the treatment of choice.
URINARY BLADDER

It is the organ (lined by transitional epithelium) responsible for the collection of urine formed by the
kidney and its removal by intermittent voiding.
Inflammation of urinary bladder is called cystitis and it is more common in females as
compared to males. It is usually due to:
1. Bacterial cause: E. coli, Proteus, Klebsiella, Mycobacterium tuberculosis
2. Fungal cause: Candida albicans, seen with immunosuppression
3. Hemorrhagic cystitis: Due to cytotoxic antitumor drugs like cyclophosphamide and Adenovirus.
4. Radiation cystitis: Due to radiation exposure.

Clinical features:
1. Frequency – Requirement of urination every 15-20 minutes
2. Suprapubic pain – Pain in anatomical location of the bladder
3. Dysuria - Painful or burning sensation or urination
This triad may be associated with fever and malaise.
Special cystitis
Hunner ulcer Malacoplakia
• Painful chronic cystitis associated with hemorrhagic • Chronic bacterial cystitis having presence of soft,
inflammation and fibrosis of the layers of bladder yellow mucosal plaques.
wallQ. • Most commonly associated with E. coli or
• Seen frequently in females. uncommonly proteus.
• Usually idiopathic but may be associated with SLE. • Microscopically, there is presence of infiltration with
• Mast cellsQ are characteristically present. lymphocytes and abundant epithelioid histiocytes (Von
Hansemann Histiocytes) having PAS positive
granules and characteristics 3-10m rounded
intracytoplasmic inclusions (called Michaelis-
Gutmann bodies)Q that contain iron (demonstrated by
Prussian blue stain) and calcium (demonstrated by
Von Kossa stain).
• Similar lesions are also seen in colon, lungs, bones,
kidneys and prostate, etc.

URINARY BLADDER NEOPLASMS

The urinary bladder cancers usually are of epithelial origin. The commonest histological variant is
the transitional cell tumors (urothelial tumors).
Risk factors of urinary bladder cancers
Transitional cell cancers Squamous cell cancer Adenocarcinoma
• CigarettesmokingQ. • Infection with Schistosoma • Usually arises from urachal
• Industrial exposure to arylamines as 2- haematobiumQ. remnantsQ or in association
napthylamine, benzidine, aniline in • Chronic bladder infection and with intestinal metaplasiaQ.
textile workers, dye workers and irradiationQ.
leather workersQ. • Diverticula in the bladderQ.
• Pelvic irradiation for other pelvic
cancerQ.
• Long term use of analgesics.
• Exposure to drugs like
cyclophosphamideQ.

Genetic risk factors include:

• 9 p gene deletions- Present in superficial papillary tumors.


• 17p gene deletion- Invasive urothelial cancers.
9p and 9q have tumor suppressor gene, 17 p has the location of p53 (again tumor suppressor
gene). So, any deletion of these genes increases the risk of bladder cancers.

CLINICAL FEATURES
Painless hematuriaQ (most common symptom), features of bladder irritability (frequency, urgency
and dysuria) or uncommonly, hydronephrosis and pyelonephritis may also be seen.
The prognostic markers include grade of tumor, presence of lamina propria invasion and
associated carcinoma in situ.
The worst prognosis is associated with tumor invading the muscularis mucosa (detrusor
muscle)Q.

INVESTIGATIONS:
• Cystoscopy and biopsyQ (Best investigation)
• Urine cytology of urine markers like telomerase, human complement factor H related protein,
mucins, CEA, hyaluronic acid, hyaluronidase, fibrin-fibrinogen degradation products, nuclear
matrix proteins and DNA content.
ISUP (International Society of Urological Pathology) Classification of Transitional Cell
Tumors

• Urothelial Papilloma – Seen in young patients, finger like papillae covered with normal looking urothelium.
• Urothelial neoplasm of low malignant potential – Similar to papilloma but with thicker urothelium with diffuse
nuclear enlargement.
• Papillary urothelial carcinoma, low grade – Almost always papillary having limited cell/nuclear pleomorphism
and limited chromosomal/gene abnormalities.
• Papillary urothelial carcinoma; high grade – May be papillary/nodular or both having considerable anaplasia
and high frequency of chromosomal/gene abnormalities.

MANAGEMENT:
1. Intravesical BCGQ
– Presence of lamina propria invasion
– Carcinoma in situ (CIS)
2. Radical cystectomy
CIS refractory to BCG
– Invasion of muscularis propria
– CIS extending to prostatic urethra or down
3. Chemotherapy (Mitomycin, thiotepa, etc.)
– Advanced bladder cancer
KIDNEY: GENERAL ASPECTS, POLYCYSTIC KIDNEY DISEASE

1. A 28-year-old man has lenticonus and end stage renal disease now. His maternal uncle
also died of the same illness. What is the most likely diagnosis?
(a) Autosomal dominant polycystic kidney disease
(b) Autosomal recessive polycystic kidney disease
(c) Oxalosis
(AIIMS Nov 2012)
(d) Alport syndrome
2. Which one of the following is not associated with adult polycystic kidney disease?
(DPG 2011)
(a) Autosomal dominant inheritance
(b) Mutations involving gene affecting cell-cell matrix interactions
(c) Intracranial berry aneurysm may be present
(d) Tricuspid valve prolapse
3. Which of the following is associated with adult polycystic kidney disease?
(a) Berry aneurysms of Circle of Willis
(b) Saccular aneurysms of aorta
(c) Fusiform aneurysms of aorta
(d) Leutic aneurysms
4. True about adult polycystic kidney disease is all, except:
(AIIMS Nov 2001)
(a) Autosomal dominant inheritance
(b) Hypertension is rare
(c) Can be associated with cysts in liver, lungs and pancreas
(d) Pyelonephritis is common
5. True about autosomal dominant type of APKD:
(a) Small kidney
(PGI June 2004)
(b) Bilateral medullary cysts
(c) Mutation of polycystin 1and 2 gene
(d) Renal transplantation is contraindicated
(e) Pathogenesis starts early and renal failure seen in middle life.
6. Chromosomes involved in adult polycystic kidney disease (APKD)
are: (PGI June 01)
(a) 6 and 11
(b) 4 and 16
(c) 7 and 17
(d) 4 and 12
(e) 4 and 17

MOST RECENT QUESTIONS


7. Acquired cystic disease of kidney is associated with:
(a) Xanthogranulomatous pyelonephritis
(b) Dialysis
(c) Renal stones
(d) Renal dysplasia
8. Adult polycystic kidney disease is inherited by:
(a) Autosomal dominant
(b) Autosomal recessive
(c) X-linked
(d) Mitochondrial
9. Major cause of death in End Stage Renal Disease patients on display is which one of the
following?
(a) Cardiovascular disease
(b) Infections
(c) Uremia
(d) Respiratory Failure
10. Podocytes are seen in:
(a) Proximal convoluted tubule
(b) Distal convoluted tubule
(c) Collecting tubule of the kidney
(d) Bowman’s capsule
11. What is the minimum number of red blood cells per microliter of urine required for
diagnosis of hematuria?
(a) 3
(b) 5
(c) 8
(d) 10
12. Polycystic kidney may be associated with cysts in all the sites, except:
(a) Lung
(b) Liver
(c) Pancreas
(d) Brain
13. What is oliguria
(a) Excretion of less than 300 ml in 24 hrs
(b) Excretion of less than 500 ml in 24 hrs
(c) Excretion of less than 300 ml in 12 hrs
(d) Excretion of less than 100 ml in 24 hrs
14. A postrenal transplant patient presented with decreased urine output. A urinalysis was
done and the microscopic image is shown below. Identify the structure marked with an
arrow:
(AIIMS Nov 2016)

(a) Decoy cells


(b) Renal tubular epithelial cells
(c) Hyaline casts
(d) Clue cells

GLOMERULAR DISEASE: NEPHRITIC SYNDROME, NEPHROTIC SYNDROME,


GLOMERULONEPHRITIS
15. In a specimen of kidney, fibrinoid necrosis is seen and onion peel appearance is also
present. Most probable pathology is:
(AIIMS Nov 2012)
(a) Hyaline degeneration
(b) Hyperplastic arteriosclerosis
(c) Glomerulosclerosis
(d) Fibrillary glomerulonephritis
16. Which of the following is the diagnosis for a condition having mutation in COL4A5
chain?
(AIIMS Nov 2012)
(a) Alport’s syndrome
(b) Good pasture’s syndrome
(c) Thin membrane disease
(d) Nodular glomerulosclerosis
17. The most common gene defect in idiopathic steroid resistance nephrotic syndrome
(AIIMS Nov 2011)
(a) ACE
(b) NPHS 2
(c) HOX11
(d) PAX
18. A person with radiologically confirmed reflux nephropathy develops nephritic range
proteinuria. Which of the following would be the most likely histological finding in this
patient?
(AIIMS Nov 2011)
(a) Focal segmental glomerulosclerosis
(b) Nodular glomerulosclerosis
(c) Membranous glomerulopathy
(d) Proliferative glomerulonephritis with crescents
19. A 7 yrs old girl is brought with complaints of generalised swelling of the body. Urinary
examination reveals grade 3 proteinuria and the presence of hyaline and fatty casts.
She has no history of hematuria. Which of the following statements about her condition
is true?
(AIIMS May 2011)
(a) IgA nephropathy is likely diagnosis
(b) Her C3 levels will be low
(c) No IgG deposits or C3 deposition on renal biopsy
(d) Alport syndrome is likely diagnosis
20. The pathological feature in Wegener’s granulomatosis on renal biopsy is
(AIIMS Nov 2010)
(a) Nodular glomerulosclerosis
(b) Focal necrotizing glomerulonephritis
(c) Granulomas in the vascular wall
(d) Granuloma of parenchyma of kidney
21. Fibronectin nephropathy has all of the following features except
(AIIMS Nov 2010)
(a) Autosomal recessive inheritance
(b) Associated with mesangial expansion
(c) Glomeruli do not stain for immunoglobulin or complement
(d) PAS- positive amyloid negative deposits.
22. Pathological changes of diabetic nephropathy are all except:
(DPG 2011)
(a) Fibrin caps and capsular drops
(b) Kimmelstein-Wilson lesion
(c) Basement membrane thickening
(d) Focal glomerular sclerosis
23. What is the cause of hypercoagulation in nephrotic syndrome:
(AI 2010)
(a) Loss of antithrombin III (AT III)
(b) Decreased fibrinogen
(c) Decreased metabolism of vitamin K
(d) Increase in Protein C
24. Finnish type of nephrotic syndrome is associated with:
(a) Nephrin
(AI ’09, ‘06)
(b) Podocin
(c) Alpha actinin
(d) CD2 activated protein
25. Pauci-immune crescentic glomerulonephritis is associated with:
(AI 2009)
(a) Microscopic polyangiitis
(b) SLE
(c) H S Purpura
(d) PAN
26. Most common mutation seen in congenital nephrotic syndrome is:
(AI 2008)
(a) Nephrin
(b) Podocin
(c) a 4 actinin
(d) Megalin
27. Which of these does not cause crescentic glomerulonephritis?
(AI 2008)
(a) Rapidly progressive glomerulonephritis
(b) Alport syndrome
(c) Goodpasture’s syndrome
(d) Henoch-Schönlein purpura
28. All are non-proliferative glomerulonephritis, except:
(a) Membranous glomerulonephritis
(AI 2008)
(b) Mesangiocapillary glomerulonephritis
(c) Diabetic glomerulosclerosis
(d) Amyloidosis
29. Kidney biopsy from a child with hemolytic uremic syndrome characteristically most
likely presents features of:
(AI 2005)
(a) Thrombotic microangiopathy
(b) Proliferative glomerulonephritis
(c) Focal segmental glomerulosclerosis
(d) Minimal change disease
30. Serum C3 is persistently low in the following except:
(a) Post streptococcal glomerulonephritis
(b) Membranoproliferative glomerulonephritis
(c) Lupus nephritis
(AI ‘04, DNB ‘07)
(d) Glomerulonephritis related to bacterial endocarditis
31. All of the following are associated with low complement levels except:
(AI 2004)
(a) Lupus nephritis
(b) Mesangiocapillary glomerulonephritis
(c) Diarrhea-associated hemolytic uremic syndrome
(d) Post-infections glomerulonephritis
32. All of the following are associated with low C3 level except:
(AI 2003)
(a) Post streptococcal glomerulonephritis
(b) Membranoproliferative Glomerulonephritis
(c) Goodpasture’s disease
(d) Systemic lupus erythematosus
33. Which of the following is not true about Berger’s disease?
(AI 2003)
(a) The pathological changes are proliferative and usually confined to mesangial cells; usually
focal and segmental
(b) Hematuria may be gross or microscopic
(c) On immunofluorescence deposits contain with IgA and IgG
(d) Absence of associated proteinuria is pathognomic
34. Crescent formation is characteristic of which of the following glomerular disease:
(AI 2002)
(a) Minimal change disease
(b) Rapidly progressive glomerulonephritis
(c) Focal and segmental glomerulosclerosis
(d) Rapidly non progressive glomerulonephritis
35. In Wegener’s granulomatosis, kidney has which of the following lesions?
(AIIMS Nov 2009)
(a) Glomerular granuloma
(b) Interstitial granuloma
(c) Crescentic glomerulonephritis
(d) Glomerulosclerosis
36. The Electron Microscopy is virtually diagnostic in renal biopsy study of:
(AIIMS May 2008)
(a) Goodpasture’s syndrome
(b) Churg-Strauss syndrome
(c) Alport syndrome
(d) Wegner’s granulomatosis
37. Which type of FSGS has worst prognosis?
(AIIMS May 2008)
(a) Tip variant
(b) Collapsing
(c) NOS
(d) Perihilar
38. Steroid resistant nephrotic syndrome is caused due to mutation in the gene encoding
for?
(a) Nephrin
(AIIMS May 2008, Nov 2006)
(b) Alpha-actinin-4
(c) Podocin
(d) Transient Receptor Potential 6
39. Which of the following is a feature of Collapsing glomerulopathy?
(AIIMS Nov ‘07)
(a)Tuft necrosis
(b) Mesangiolysis
(c) Parietal epithelial proliferation
(d) Hypertrophy and necrosis of visceral epithelium
40. The most common gene defect in idiopathic steroid resistant nephrotic syndrome
(AIIMS May 2007)
(a) ACE
(b) NPHS 2
(c) HOX 11
(d) PAX
41. HIV associated nephropathy is a type of:
(a) Membranous glomerulonephritis
(AIIMS Nov 2004)
(b) Immunotactoid glomerulopathy
(c) Collapsing glomerulopathy
(d) Fibrillary glomerulopathy

42. Mesangial deposits of monoclonal kappa/Lambda light chains in indicative of


(AIIMS May 2004)
(a) Mesangioproliferative glomerulonephritis
(b) Focal and segmental glomerulosclerosis
(c) Kimmelstiel-Wilson lesions
(d) Amyloidosis
43. In renal disease, Albumin is first to appear in urine because
(AIIMS May 2004)
(a) Of its high concentration in plasma
(b) Has molecular weight slightly greater than the molecules normally getting filtered
(c) High Albumin Globulin ratio
(d) Tubular epithelial cells are sensitive to albumin
44. A 7 year old boy presented with generalized edema. Urine examination revealed marked
albuminuria. Serum biochemical examinations showed hypoalbuminemia with
hyperlipidemia. Kidney biopsy was undertaken. On light microscopic examination, the
kidney appeared normal. Electron microscopic examination is most likely to reveal:
(AIIMS Nov 2003)
(a) Fusion of foot processes of the glomerular epithelial cells
(b) Rarefaction of glomerular basement membrane
(c) Deposition of electron dense material in the basement membrane
(d) Thin basement membrane

45. The prognosis of rapidly proliferating glomerulonephritis (Crescentic GN) depends


upon
(a) Number of crescents
(AIIMS Nov 2001)
(b) Size of crescents
(c) Shape of crescents
(d) Cellularity of crescents
46. True about light microscopy in minimal change disease is:
(AIIMS Nov 2001)
(a) Loss of foot process seen
(b) Anti-GBM antibodies are seen
(c) IgA deposits seen
(d) No change seen
47. A child presented with edema, massive proteinuria and hyperlipidemia. True statment
about this condition is:
(a) A type of focal segmental GN
(AIIMS Nov 2001)
(b) IgA deposition on basement membrane
(c) Foot process of glomerular membrane normal
(d) Glomerular function is lost due to loss of polyanionic charge on both sites of glomerular
foot process
48. Persistent low C3 complement level is not found in:
(AIIMS Nov 2006)
(a) Post streptococcal glomerulonephritis
(b) Mesangiocapillary glomerulonephritis
(c) Cryoglobulinemia
(d) SLE
49. Low complement levels are seen in:
(a) PSGN
(PGI Dec 2006)
(b) MPGN
(c) Goodpasture’s syndrome
(d) Wegener’s granulomatosis
(e) Infective endocarditis.
50. Subepithelial deposits in kidney are seen in:
(a) MPGN-1
(PGI June 2001)
(b) Goodpasture’s syndrome
(c) PSGN
(d) Membranous GN
(e) RPGN
51. Nephrotic syndrome is characterized by:
(PGI Dec 2002)
(a) Proteinuria
(b) Hyperlipidemia
(c) Edema
(d) Haematuria
(e) Lipiduria
52. In glomerular disease which of the following is mainly excreted in urine:
(PGI Dec 2003)
(a) Albumin
(b) Globulin
(c) Light chain
(d) Heavy chain
(e) Tamm-Horsfall protein
53. Which of the following is included in definition of Nephrotic syndrome?
(PGI June 2004)
(a) Microalbuminuria
(b) Massive Proteinuria
(c) Microscopic hematuria
(d) Edema
(e) Hyperlipidemia.
54. True about Heymann rat glomerulonephritis is:
(a) Heymann antigen is called megalin
(PGI Dec 2004)
(b) Electron dense deposits in subendothelial space
(c) Electron dense deposits in mesangium
(d) Subepithelial aspect of basement membrane have deposits
(e) Antigen against bacterial and viral proteins
55. Malignant hypertension is associated with:
(a) RPGN
(PGI Dec 2004)
(b) Malignant nephrosclerosis
(c) Membranous GN
(d) IgA nephropathy
(e) Acute pyelonephritis
56. Histology of Alport syndrome:
(PGI June 2005)
(a) Foamy cells in interstitium
(b) Foamy cells in tubular epithelial cells
(c) Thickening of GBM > 100 nm
(d) Thinning of GBM < 100 nm
(e) Intimal proliferation.
57. RPGN is caused by:
(PGI June 2005)
(a) FSGS
(b) Wegener’s granulomatosis
(c) Goodpasture’s syndrome
(d) PAN
(e) Microscopic polyangiitis
58. Post streptococcal glomerulonephritis is associated with:
(PGI Dec 2006)
(a) Subepithelial deposit
(b) Nephritis along with acute renal failure
(c) Low complement levels
(d) HTN and proteinuria
(e) Normal complement levels
59. Pauci-immune glomerulonephritis is seen in:
(a) RPGN
(Delhi PG-2007)
(b) IgA nephropathy
(c) Microscopic polyangiitis
(d) FSGS
60. Kimmelstiel Wilson lesions are characteristic of:
(a) Diabetic Nephropathy
(Delhi PG-2007)
(b) Analgesic nephropathy
(c) RPGN
(d) Post streptococcal glomerulonephritis
61. Anti-glomerular basement membrane nephritis is seen in:
(Delhi PG-2005)
(a) Goodpasture’s syndrome
(b) SLE associated glomerulopathy
(c) MGN
(d) MPGN
62. Kimmelstiel Wilson disease is diagnostic of:
(a) Diabetic Glomerulosclerosis
(Delhi PG-2005)
(b) Benign Hypertension
(c) Malignant Hypertension
(d) Amyloidosis
63. The following is not a feature of acute post streptococcal glomerulonephritis:
(Karnataka 2007)
(a) Normal C3
(b) Hypertension
(c) Elevated blood urea and creatinine
(d) Increased ASO titre
64. Visceral leishmaniasis causes
(Karnataka 2005)
(a) Membranous glomerulonephritis
(b) Mesangioproliferative glomerulonephritis
(c) Focal segmental glomerulonephritis
(d) Rapidly progressive glomerulonephritis
65. Histological hallmark of rapidly progressive glomerulonephritis is
(Karnataka 2004)
(a) Crescents in most of the glomeruli
(b) Loss of foot processes of epithelial cells
(c) Subendothelial electron dense deposits
(d) The thickening of glomerular capillary wall
66. ANCA is most specific and sensitive marker for
(a) Idiopathic cresenteric glomerulonephritis
(UP 2000)
(b) Post streptococcal glomerulonephritis
(c) Membranoproliferative glomerulonephritis
(d) Focal segmental glomerulosclerosis
67. True about post streptococcal glomerulonephritis is:
(a) Linear deposition
(UP 2001)
(b) Diffuse involvement
(c) Tram track appearance
(d) Global sclerosis
68. Most common in diabetic nephropathy is:
(a) Diffuse glomerulosclerosis
(UP 2002)
(b) Diffuse cortical sclerosis
(c) Nodular glomerulosclerosis
(d) Renal atherosclerosis
69. Steroid responsive glomerulonephritis are all except:
(a) Post streptococcal glomerulonephritis
(UP 2002)
(b) Minimal change disease
(c) Membranoproliferative glomerulonephritis
(d) Focal segmental glomerulosclerosis
70. All conditions lead to CRF except:
(UP 2002)
(a) Post streptococcal GN
(b) Membranoproliferative GN
(c) Minimal change GN
(d) Focal glomerulosclerosis
71. Most characteristic finding in diabetic nephropathy is:
(a) Diffuse glomerulosclerosis
(UP-2002, 2004)
(b) Nodular glomerulosclerosis
(c) Diffuse cortical sclerosis
(d) Renal atherosclerosis
72. All of the following are seen in Goodpasture’s syndrome except:
(PGI-1999)(UP 2004)
(a) Cresenteric glomerulonephritis
(b) Hemorrhage inflammation
(c) Anti-GBM antibody
(d) Diffuse alveolar involvement
73. All of the following are True about minimal change nephrotic disease except:
(PGI-1999)
(a) Respond to steroids
(UP 2004)
(b) Selective proteinuria
(c) IgG deposition in mesangium
(d) Common in the age group of 2-9 years

MOST RECENT QUESTIONS


74. IgA depositions in mesangial cells are seen in
(a) Goodpasture’s syndrome
(b) Berger disease
(c) Cresenteric glomerulonephritis
(d) Alport syndrome
75. All are causes of rapidly progressive glomerulonephritis except:
(a) SLE
(b) Polyarteritis nodosa
(c) Post streptococcal GN
(d) Rheumatoid arthritis
76. The cause of edema in nephritic syndrome is:
(a) Decrease in plasma protein concentration
(UP 2007)
(b) Increase in plasma protein concentration
(c) Reduced plasma osmotic pressure
(d) Sodium and water retention
77. Hall mark of the IgA nephropathy is:
(a) Oedema
(b) Hematuria
(c) Hypertension
(d) Proteinuria
78. Electron dense deposits in the region of hyalinosis and sclerosis, with diffuse loss of
foot processes, seen in electron microscopy are features of:
(a) Minimal change disease
(b) Membranous glomerulonephritis
(c) Membranoproliferative GN
(d) Focal segmental glomerulosclerosis
79. In hemolytic uremic syndrome there is:
(a) Thrombocytopenia + Renal failure
(b) Normal coagulative profile
(c) Microangiopathic hemolytic anemia
(d) All of the above
80. Subepithelial deposits with ‘M’ spike is seen in:
(a) Membranous glomerulonephritis
(b) Membranoproliferative glomerulonephritis
(c) Minimal change disease
(d) RPGN
81. Crescentic glomerulonephritis with pauci-immune glomerulonephritis is associated
with:
(a) Post infectious glomerulonephritis
(b) Goodpasture’s syndrome
(c) Wegener’s glomerulonephritis
(d) Membranous glomerulonephritis
82. Tram track appearance on histopathology of kidney is seen in:
(a) Membranous nephropathy
(b) Membranoproliferative glomerulonephritis
(c) IgA nephropathy
(d) Crescentic glomerulonephritis
83. On electron microscopy, in most of the cases, characteristic splitting of GBM with
subepithelial deposits in few cases is seen in:
(a) RPGN
(b) Membranous nephropathy
(c) FSGS
(d) Minimal change disease
84. An IV drug abuser Chulbul develops an aggressive form of nephrotic syndrome that
does not respond to steroids. A renal biopsy is performed. Which of the following
histological diagnoses will most likely be made from the biopsy tissue?
(a) Focal segmental glomerulosclerosis
(b) IgA nephropathy
(c) Membranous glomerulonephritis
(d) Membranoproliferative glomerulonephritis
85. Crescents are derived from which of the following?
(a) Epithelial cells + fibrin + macrophage
(b) Mesangial cell + fibrin + macrophage
(c) Tubular cell + mesangial cell + fibrin
(d) Neutrophil + tubular cell + fibrin
86. Flea bitten appearance of the kidney is seen in:
(a) Malignant hypertension
(b) Benign hypertension
(c) Chronic pyelonephritis
(d) Diabetes mellitus
87. All of the following decrease in Nephrotic syndrome except:
(a) Thyroxin
(b) Transferrin
(c) Fibrinogen
(d) Albumin
88. Most common cause of nephrotic syndrome in adults is:
(a) Membranous glomerulonephritis
(b) Minimal change disease
(c) Acute GN
(d) Focal segmental glomerulosclerosis
89. The crescent forming glomerulonephritis is:
(a) Acute GN
(b) Rapidly progressive glomerulonephritis
(c) Membranous GN
(d) Membranoproliferative GN
90. Most common cause of nephritic syndrome in adults is:
(a) Berger disease
(b) Focal segmental glomerulosclerosis
(c) Membranous glomerulonephritis
(d) Minimal change disease
91. Post streptococcal glomerulonephritis in children is diagnosed by:
(a) Heavy protienuria, high cholesterol, high ASO titre
(b) Heavy protienuria, hematuria, low ASO titre
(c) Mild proteinuria, hematuria, high ASO titre
(d) Mild protienuria, high cholesterol, normal ASO titre

92. IgA nephropathy is characterized by all of the following except:


(a) Hypertension
(b) Hematuria
(c) Nephritic syndrome
(d) Renal biopsy having thin basement membrane
93.Microalbuminuria is defined as protein levels of:
(a) 100-150 mg/d
(b) 151-200 mg/d
(c) 30-300 mg/d
(d) 301-600 mg/d
94. Type I membranoproliferative glomerulonephritis is commonly associated with all
except:
(a) SLE
(b) Persistent hepatitis C infections
(c) Partial lipodystrophy
(d) Neoplastic diseases
95. RBC cast seen in:
(a) Minimal change disease
(b) Renal vein thrombosis
(c) Bladder schistomiasis
(d) Rapidly progressive glomerulonephritis
96. Most common glomerulonephritis associated with HIV is which of the following?
(a) Focal segmental glomerulonephritis
(b) Diffuse glomerulosclerosis
(c) Membrano-proliferative glomerulonephritis
(d) Crescentic glomerulonephritis
97. In which one of the primary glomerulonephritis the glomeruli are normal by light
microscopy but shows loss of foot processes of the visceral epithelial cells and no
deposits by electron microscopy
(a) Poststreptococcal glomerulonephritis
(b) Membrano-proliferative glomerulonephritis type I
(c) IgA nephropathy
(d) Minimal change disease
98. A 10-year-old child presented with edema and decreased urine output. On evaluation,
serum albumin is 2.5 g/dL, S.Creatinine is 0.5 mg/dL, Urine protein is 3+ with no RBC or
casts. The expected pathological change is:
(a) Interstitial nephritis
(b) Minimal change disease
(c) IgA nephropathy
(d) Membranous nephropathy
99. Which of the following is the commonest nephropathy seen in a patient with cancer?
(a) Minimal change disease
(b) IgA nephropathy
(c) Membranous glomerulopathy
(d) Focal segmental glomerulosclerosis
100. Most consistent feature of RPGN is:
(AIIMS Nov 2016)
(a) IgA deposition
(b) Mesangial cell proliferation
(c) Crescent formation
(d) Loss of foot processes
101. The following is the FITC for IgG stained kidney specimen. What is this suggestive of?
(AIIMS May 2016)

(a) SLE
(b) Berger’s disease
(c) Membranous glomerulopathy
(d) Good Pasture Syndrome

102. Increased BP, proteinuria, urinary RBC casts are the features of which of the following
type of glomerulonephritis?
(a) Membranous GN
(b) RPGN
(c) Minimal change disease
(d) Focal segmental glomerulosclerosis
103. True about Henoch Schonlein purpura is:
(a) Medium vessels vasculitis
(b) Renal symptoms start late in the disease
(c) IgA deposition in mesangium
(d) Low platelet count
104. Granular IgA deposits are seen in which of the following renal conditions?
(a) Minimal change disease
(b) Chronic pyelonephritis
(c) Haemolytic uremic syndrome
(d) Berger’s nephropathy
105. IgA nephropathy is not associated with:
(a) Focal mesangial proliferation
(b) Gross hematuria within 1-2 days
(c) On immunofluorescence deposits contain both IgA and IgG
(d) Increased complement levels
106. Alport syndrome is characterized by all except:
(a) X-linked
(b) Cardiac hypertrophy
(c) Nerve deafness
(d) Glomerulonephritis
107. Characteristic feature of IgA nephropathy:
(a) More common in old age
(b) It is a type of membranoproliferative glomerulonephritis
(c) Serum complement level is normal
(d) Gross hematuria is present after 10 days
108. Which of the following is not seen in SLE affected kidneys?
(a) Focal glomerulonephritis
(b) Diffuse glomerulonephritis
(c) Membranous glomerulonephritis
(d) Lipoid nephrosis

TUBULAR DISEASE AND RENAL CALCULI

109. A lady presents with complaints of abdominal pain. Contrast enhanced CT scan shows
bilateral papillary necrosis. Which of the following test shall not be done to investigate
the cause of her papillary necrosis?
(a) Culture for bacteria
(AIIMS Nov 2011)
(b) Sickling test
(c) Urine acidification
(d) Urine PCR for TB
110. Urine analysis of a patient with haematuria and hypercalciuria is most likely to reveal
which of the following?
(AIIMS Nov 2011)
(a) Isomorphic RBCs
(b) RBC casts
(c) Nephrotic range proteinuria
(d) Eosinophiluria
111. All of the following about xanthogranulomatous pyelonephritis are true except
(AI 2011, AIIMS May 2010)
(a) On cut section yellowish nodules are seen
(b) Associated with tuberculosis.
(c) Foam cells are seen
(d) Giant cells are seen
112. In which of the following conditions bilateral contracted kidneys are characteristically
seen?
(a) Amyloidosis
(AI 2005)
(b) Diabetes mellitus
(c) Rapidly progressive glomerulonephritis
(d) Benign nephrosclerosis
113. Necrotizing papillitis may be seen in all of the following conditions except:
(AI 2002)
(a) Sickle cell disease
(b) Tuberculous pyelonephritis
(c) Diabetes mellitus
(d) Analgesic nephropathy
114. Mercury affects which part of the kidney?
(a) PCT
(b) DCT
(AIIMS May 2007)
(c) Collecting duct
(d) Loop of Henle
115. Nephrocalcinosis is seen in all except:
(a) Sarcoidosis
(AIIMS May 2007)
(b) Distal RTA
(c) Milk alkali syndrome
(d) Medullary cystic kidney
116. Pulmonary, renal syndrome is seen in:
(a) Goodpasture’s syndrome
(PGI Dec 2003)
(b) Leptospirosis.
(c) Legionella.
(d) Wegener’s granulomatosis.
(e) Hanta virus infection
117. Renal papillary necrosis is seen in:
(a) Thalassemia
(PGI June 2001)
(b) DM
(c) Phenacetin abuse
(d) Alcoholism
(e) Cortical necrosis
118. Causes of Nephrocalcinosis are:
(PGI June 2001)
(a) Hyperparathyroidism
(b) TB Kidney
(c) Hypercalcemia
(d) Glomerulonephritis
(e) MCD
119. Bilaterally enlarged kidneys are seen in:
(a) Chronic glomerulonephritis
(PGI Dec 2002)
(b) Chronic pyelonephritis
(c) Benign nephrosclerosis
(d) Polycystic Kidney disease
(e) Amyloidosis
120. Hereditary nephritis is seen in:
(a) Analgesic nephropathy
(PGI Dec 2003)
(b) Balkan nephropathy
(c) Alport’s syndrome.
(d) Eosinophilic nephritis
121. Nephrocalcinosis is seen in:
(PGI June 2004)
(a) Hypoparathyroidism
(b) Medullary sponge kidney
(c) DM
(d) RTA
122. Histology of acute rejection of renal transplant are:
(a) Arteriolar hyalinosis
(PGI June 2005)
(b) Eosinophilic infiltration
(c) Glomerular vasodilatation
(d) Neutrophilic infiltration
(e) Necrotizing vasculitis.
123. Which of the following is seen in hemolytic uremic syndrome?
(Delhi PG-2007)
(a) Spherocytes
(b) Schistocytes
(c) Target cells
(d) Heinz bodies
124. All are true about nephronophthisis except:
(a) Interstitial fibrosis
(Delhi PG-2006)
(b) Cortical tubular hypertrophy
(c) Cysts in the medulla
(d) 20% cases are non-familial
125. Most common cause of papillary necrosis is:
(a) Diabetes Mellitus
(Delhi PG-2005)
(b) Acute Pyelonephritis
(c) Sickle cell disease
(d) Analgesic Nephropathy
126. Renal calculi are commonly made up of
(a) Calcium oxalate
(Karnataka 2006)
(b) Magnesium ammonium phosphate
(c) Uric acid
(d) Cystine
127. Salt losing nephritis is due to
(UP 2000)
(a) Lupus nephritis
(b) Streptococcal infection
(c) Interstitial nephritis
(d) Goodpasture’s syndrome

MOST RECENT QUESTIONS

128. Most common cause of renal papillary necrosis is:


(a) Analgesic nephropathy
(b) Sickle cell disease
(c) Diabetes mellitus
(d) Chronic pyelonephritis
129. Tubulointerstitial disease are all except:
(a) Hypokalemic nephropathy
(b) Lupus nephritis
(c) Hypercalcemic nephropathy
(d) Analgesic nephropathy
130. Renal pathology in SLE includes all except:
(a) Focal glomerulonephritis
(b) Diffuse glomerulonephritis
(c) Diffuse membranous glomerulonephritis
(d) Lipoid nephrosis
131. Pathologic feature of malignant hypertension is:
(a) Fibrinoid necrosis
(b) Papillary necrosis
(c) Hard exudate
(d) All
132. Contracted kidneys are seen in all of the following diseases except:
(a) Chronic glomerulonephritis
(b) Chronic renal failure
(c) Amyloidosis
(d) Analgesic nephropathy
133. Hematuria with dysmorphic RBC are seen in:
(a) Acute glomerulonephritis
(b) Renal TB
(c) Renal Calculi
(d) Chronic renal failure
134. Birefringent crystals in urine is seen with:
(a) Calcium oxalate stone (b) Uric acid stone
(c) Struvite stones (d) None
135. Anti-GBM antibodies are seen in:
(a) Goodpasture’s syndrome
(b) RPGN
(c) Membranous GN
(d) Minimal change disease
136. A 28 year young female Katrina present to your OPD with complaints of suprapubic
pain, urinary frequency and dysuria. She also passed blood in last voided urine about
30 minutes ago. Her urinanalysis demonstrate the presence of pyuria but no white cell
casts. On physical examination, she has suprapubic tenderness on palpation. Which of
the following is the likely diagnosis in this patient?
(a) Acute pyelonephritis
(b) Chronic pyelonephritis
(c) Cystitis
(d) Fanconi syndrome
137. A 51-year-old man Sonu with a history of recurrent calcium-containing renal stones
presents to the emergency room with excruciating flank pain and blood in the urine.
This patient is likely to have which of the following underlying disorders?
(a) Anemia of chronic disease
(b) Chronic Proteus infection
(c) Hyperparathyroidism
(d) Hyperaldosteronism
138. Renal papillary necrosis is almost always associated with one of the following
conditions:
(a) Diabetes mellitus
(b) Analgesicnephropathy
(c) Chronic pyelonephritis
(d) Post streptococcal GN
139. All are causes of granular contracted kidneys except:
(a) Benign nephrosclerosis
(b) Diabetes mellitus
(c) Chronic Pyelonephritis
(d) Chronic glomerulonephritis
140. Alports syndrome is inherited by al the following inheritances except
(a) X linked
(b) Codominant
(c) AD
(d) AR
141. Nephrocalcinosis is seen in which one of the following?
(a) Hyperparathyroidism
(b) Diabetes mellitus
(c) Amyloidosis kidney
(d) End stage kidney
142. Thimble bladder is typically seen in which one of the following?
(a) Acute bacterial cystitis
(b) Tuberculous cystitis
(c) Bilharziasis
(d) Transitional cell carcinoma
143. Periglomerular fibrosis is considered typical of:
(a)Chronic pyelonephritis
(b) Chronic glomerulonephritis
(c) Arterionephrosclerosis
(d) Malignant hypertension
144. Which of the following increases tuberculosis?
(a) Asbestosis
(b) Sarcoidosis
(c) Silicosis
(d) Berylliosis
145. Which one of the following statements is incorrect regarding uric acid stones?
(a) They are radiolucent
(b) Crystals are hexagonal
(c) Formed in acidic urine
(d) Can be seen with normouricemia
146. Characteristic histopathological feature of kidney in DM?
(a) Nodular glomerulosclerosis
(b) Fibrin cap
(c) Papillary necrosis
(d) Diffuse glomerulosclerosis
147. Marker of acute kidney injury includes all of the following except:
(a) Cystatin C
(b) N GAL
(c) KIM 1
(d) Micro RNA-122
148. The resected specimen of a kidney is seen below. What is the diagnosis?
(AIIMS May 2016)

(a) Amyloidosis
(b) Acute post-streptococcal glomerulonephritis
(c) Flea bitten kidney of malignant hypertension
(d) Chronic glomerulonephritis
149. Histological feature of acute pyelonephritis include all of the following except:
(a) Interstitial suppurative inflammation
(b) Tubular necrosis
(c) Intratubular aggregates of neutrophils
(d) Hypercellular glomeruli
150.Renal papillary necrosis is seen in all except:
(a) Urinary tract infection
(b) Hypertension
(c) Diabetes mellitus
(d) Chronic alcohol use
151. Irregular scarred kidney with pelvic dilation is seen with:
(a) Chronic pyelonephritis
(b) Polycystic kidney
(c) Renal artery stenosis
(d) Tuberculosis of kidney

RENAL TUMOURS: RCC, WILM’S TUMOUR

152. Which of the following is not associated with renal cell carcinoma?
(AIIMS May 2011)
(a) Polycythemia
(b) Amyloidosis
(c) Cushing’s syndrome
(d) Hypertension
153. Wilm’s tumor is associated with all of the following except
(AIIMS May 2010)
(a) Hemihypertrophy
(b) Aniridia
(c) Hypertension
(d) Bilateral polycystic kidney
154. The cytogenetics of chromophilic renal cell carcinoma is characterized by:
(AI 2010)
(a) Mutant VHL gene
(b) Loss of 3p
(c) Trisomy 7/17
(d) Loss of 5q 3
155. The most common histological variant of renal cell carcinoma is
(AIIMS Nov 2005)
(a) Clear cell type
(b) Chromophobe type
(c) Papillary type
(d) Tubular type

MOST RECENT QUESTIONS

156. In which of the following conditions, Aniridia and Hemi-hypertrophy are most likely
present
(a) Neuroblastoma
(b) Wilm’s tumor
(c) Non-Hodgkin’s lymphoma
(d) Germ cell tumor
157. In Wilm’s tumor the following leads to emergence of resistance to chemotherapy:
(a) Nephrogenic rests
(b) Monophasic morphology
(c) Anaplasia
(d) Capsular infiltration
158. True statement regarding Wilm’s tumour is:
(a) Common in adult
(b) Associated with deletion of chromosome 11p13
(c) Associated with MIC-2 genes
(d) Commonest presentation is hematuria
159. Most common histological type of renal cell carcinoma is:
(a) Clear cell
(b) Medullary
(c) Papillary
(d) Mixed type
160. Oncocytic carcinoma arises from:
(a) Perivascular tissue
(b) Glomerulus
(c) Loop of henle
(d) Collecting duct
161. Gene for Wilm’s tumour is located on which of the following?
(a) Chromosome 1
(b) Chromosome 10
(c) Chromosome 11
(d) Chromosome 12
162. Most important prognostic factor of wilms tumour:
(a) Histopathology and ploidy of cells
(b) Tumour stage
(c) Age of patient
(d) Mutation of chromosome 1p
163. Deletion of short arm of chromosome 11 is seen in:
(a) Osteosarcoma
(b) Meningioma
(c) Wilm’s tumor
(d) Colon Carcinoma
164. Percentage of renal vein involvement in renal cell carcinoma is:
(a) 2%
(b) 8%
(c) 16%
(d) 32%
165. All are true about renal cell cancer except:
(a) Invades renal vein
(b) Hematuria may occur
(c) Arises from proximal convoluted tubule
(d) More common in females
166. VHL syndrome is associated most commonly with which carcinoma?
(a) Lung carcinoma
(b) Renal cell carcinoma
(c) Endometrial carcinoma
(d) Hepatocellular carcinoma
167. Pseudohermaphroditism is seen with which tumor?
(a) RCC
(b) Wilms tumor
(c) Carcinoma lung
(d) HCC

URINARY BLADDER DISEASE

168. Michaelis Gutmann bodies are seen in


(a) Xanthogranulomatous pyelonephritis
(b) Malacoplakia
(c) Nail patella syndrome
(d) Tubercular cystitis
169. Chronic urethral obstruction due to benign prostatic hyperplasia can lead to the
following change in kidney parenchyma
(a) Hyperplasia
(b) Hypertrophy
(c) Atrophy
(d) Dysplasia

MOST RECENT QUESTIONS

170. Michaelis Guttmann bodies are present in:


(a) Analgesic nephropathy
(b) Homman’s ulcer
(c) Malacoplakia
(d) Erythroplasia
171. Bence Jones proteins are:
(a) Light chain
(b) Heavy chain
(c) Medium chain
(d) All
172. Transitional cell carcinoma bladder is associated with which of the following?
(a) Schistosomiasis
(b) Ascariasis
(c) Malaria
(d) Any of the above
173. Metabolic complication in CRF include all of the following except
(a) Hyperkalemia
(b) Hypophosphatemia
(c) Hypocalcemia
(d) Hypokalemia
174. Mutation of which gene leads to urinary bladder carcinoma?
(a) p53
(b) p7
(c) n-myc
(d) BRCA1
175. Gene associated with superficial papillary urothelial neoplasm:
(a) p53
(b) p16
(c) p7
(d) KRAS
176. Schistosomiasis causes which type of bladder cancer:
(a) Squamous cell carcinoma
(b) Transitional cell carcinoma
(c) Adenocarcinoma
(d) Both a and b
177. Most common type of rhabdomyosarcoma is:
(a) Pleomorphic
(b) Embryonal
(c) Alveolar
(d) Botryoid
1. Ans. (d) Alport’s syndrome
(Ref: Robbins 8th/931-2)
Presentation of male patient with lenticonus and end stage renal disease with a family history of
renal disease is highly suggestive of Alport syndrome.
AR polycystic kidney is ruled out because the age of presentation in ARPKD is childhood and most
of the affected children do not survive beyond their childhood.
• AD polycystic kidney is ruled out because there is no association of ADPKD with
lenticonus as is mentioned in our question.

– Alport syndrome is manifest by hematuria with progression to chronic renal failure,


accompanied by nerve deafness and various eye disorders, including lens
dislocation, posterior cataracts, and corneal dystrophy.

2. Ans. (d) Tricuspid valve prolapse


(Ref: Robbins 8th/959 9/e p947)
Mitral valve prolapse (and not tricuspid valve prolapse) and other cardiac valvular anomalies occur
in 20% to 25% of patients.
Adult polycystic kidney disease (APKD) is a hereditary disorder characterized by multiple expanding
cysts of both kidneys that ultimately destroy the renal parenchyma and cause renal failure. It is
an autosomal dominant condition caused by mutation in PKD1 gene encoding for polycystin-1.
This protein is present on distal tubular epithelial cells. It is involved in cell-cell and cell-matrix
interactions. The PKD2 gene product polycystin-2 is localized to all segments of the renal
tubules and may act as a Ca2+-permeable cation channel for regulatoring intracellular Ca2+
levels.
3. Ans. (a) Berry Aneurysm in Circle of Willis
(Ref: Robbins 7th/964 9/e p947)
4. Ans. (b) Hypertension is rare
(Ref: Robbins 7th/964, 9/e p 947)
Hypertension is common in patients with autosomal dominant polycystic kidney disease. It is
present in 75% of adult patients and 25% of children.
5. Ans. (e) Pathogenesis starts early and renal failure seen in middle life
(Ref: Robbins 9/e p947)
6. Ans. (b) 4 and 16.
(Ref: Robbins 8th/960; 9/e p946)
7. Ans. (b) Dialysis
(Ref: Robbins 9/e p949, 8th/960; 7th/962, 966)
8. Ans. (a) Autosomal dominant….. Too obvious at this stage friend...For details, read text.
9. Ans. (a) Cardiovascular disease
(Ref: Cambell’s Urology, 8/e p346,349)
Principal causes of death in renal transplant patients (in decreasing order)
• Heart diseaseQ • Infection • Stroke
10. Ans. (d) Bowman’s capsule
(Ref: Robbins 9/e p900)
Podocytes (or visceral epithelial cells) are cells in the Bowman’s capsule in the kidneys that wrap
around the capillaries of the glomerulus.
11. Ans. (a) 3
(Ref: Harrison 18/e p 339; CMDT 2014/e p879)
The minimum number of red blood cells per microliter of urine required for diagnosis of hematuria is
>3 RBC/HPF of centrifuged specimen.
Persistent or significant hematuria (>3 RBCs/ HPF on three urinalyses, a single urinalysis with >100
RBCs, or gross hematuria) is associated with significant renal or urologic lesions…Harrison
12. Ans. (d) Brain
(Ref: Robbbins 9/e p947)
About 40% have one to several cysts in the liver (polycystic liver disease) that are usually
asymptomatic. Cysts occur much less frequently in the spleen, pancreas, and lungs.
13. Ans. (b) Excretion of less than 500 ml in 24 hrs
As per Harrison

• OliguriaQ refers to a 24-h urine output of <500 mLQ, and


• AnuriaQ is the complete absence of urine formation (<50 mL) Q.

14. Ans. (c) Hyaline casts


Hyaline cast are tubular structures made of solidified Tamm Horsfall mucoprotein. That’s what the
pointer has shown in the image.
Decoy cells are virally infected (polyoma and cytomegalovirus) epithelial cells found in the urine.
They are seen in immunocompromised patients typically in transplant patients. These cells
have intracellular inclusion bodies. This option confused most of the people.
15. Ans. (b) Hyperplastic arteriosclerosis
(Ref: Robbins 8th/950-951, 9/e p939)
Histological alterations characterizing blood vessels in malignant hypertension
• Fibrinoid necrosis of arterioles:
• Onion-skinning (due to concentrically arranged smooth muscle with collagen).
• This condition, hyperplastic arteriolitis correlates with renal failure.
16. Ans. (a) Alport’s syndrome
(Ref: Robbins 8th/931-2, 9/e p924)
COL4A5 chain represents the a5 chain of collagen type IV. It is associated with the development of
Alport syndrome.
Genetic Mutation in Chain Disease
Gene encoding a5 chain of collagen type IV Alport syndrome
Gene encoding a4 chain of collagen type IV Benign familial hematuria/Thin membrane
disease
Gene encoding a3 chain of collagen type IV Goodpasture syndrome

17. Ans. (b) NPHS 2


(Ref: Robbins 8th/927, 9/e p918)
Gene Chromosome Protein Location Disease
NPHS 1 19q13 Nephrin Slit diaphragm Steroid sensitive/Finnish type
of nephrotic syndrome

NPHS 2 1q25 Podocin Slit diaphragm Steroid resistant nephrotic


syndrome

18. Ans. (a) Focal segmental glomerulosclerosis


(Ref Robbins 8th/926, 9/e p918)
Direct quote Robbins 8th/926…. ‘Focal segmental glomerulosclerosis occurs as a component of
the adaptive response to loss of renal tissue in advanced stage of other renal disorders such
as reflux nephropathy, hypertensive nephropathy and unilateral renal agenesis.’
19. Ans. (c) No IgG deposits or C3 deposition on renal biopsy
(Ref: Robbin’s 8th/925-6, 9/e 910)
The clinical symptoms and the findings in the stem of the question (child with generalised swelling
because of reduced serum protein secondary to massive proteinuria with urinary lipid casts)
are suggestive of nephrotic syndrome. The commonest cause in a child of nephrotic syndrome
is lipoid nephrosis or minimal change disease. So option ‘a’ and ‘d’ are ruled out. Both cause
hematuria as a finding.
Electron microscopic examination in minimal change disease show the presence of effacement of
foot processes of podocytesQ.
20. Ans. (b) Focal necrotizing glomerulonephritis (Ref: Robbins 9/e p511-512,
8th/516-7, 7th/541, Harrison 17th/2121-3)
• Robbins .....‘The renal lesions range from a mild/early disease, where glomeruli show
acute focal necrosis with thrombosis of isolated glomerular capillary loops (focal and
segmental necrotizing glomerulonephritis) whereas advanced glomerular lesions are
characterized by diffuse necrosis and parietal cell proliferation to form crescents
(crescentic glomerulonephritis)’.
• Direct quote from Harrison…….. “Necrotizing vasculitis of small arteries and veins with
granuloma formation is seen in pulmonary tissue whereas granuloma formation is rarely
seen on renal biopsy”.
21. Ans. (a) Autosomal recessive inheritance
(Ref: Heptinstall’s pathology of kidney vol 1/931)
Fibronectin nephropathy is a disease with the following features:
• Autosomal dominantQ mode of inheritance.
• Presents with proteinuria (MC presentation) and slowly progressive loss of renal function.
• The fibronectin levels in serum are normal.
• Light microscopy: is glomerular enlargement and PAS (⊕), trichrome-positive but amyloid (–)
mesangial deposits.
• Immunofluorescence microscopy: the glomeruli do NOT stain for immunoglobulin or complement
components
• Electron microscopy: large (giant), mesangial and subendothelial electron-dense deposits..

22. Ans. (d) Focal glomerular sclerosis


(Ref: Robbins 9/e p1118-1119, 8th/1141-1142)
Diabetes may cause nodular and diffuse glomerulosclerosis but not focal glomerular sclerosis.
23. Ans. (a) Loss of antithrombin III (AT III)
(Ref: Robbins 9/e p914, 8th/922, 7th/978 CMDT 2010)
Thrombotic and thromboembolic complications are common in nephrotic syndrome due to loss of
anticoagulant factor (e.g. antithrombin III, protein C and S) combined with increased platelet
activation. Renal vein thrombosis is most often a consequence of this hypercoagulative state.
There is also increased synthesis of fibrinogen in the liver.
24. Ans. (a) Nephrin
(Ref: Robbins 9/e p918, 8th/927)
25. Ans. (a) Microscopic polyangiitis
(Ref: Robbins 9/e p912, 8th/920)
Crescentic Glomerulonephritis
Type I RPGN (Anti-GBM antibody) Type II RPGN (Immune complex) Type III RPGN (Pauci immune)
• Idiopathic • Idiopathic • Idiopathic.
• Goodpasture’s syndrome. • Post infectious • ANCA associated.
• SLE, Henoch Schonlein Purpura. • Wegener’s granulomatosis.
• Microscopic polyangiitis.

Immunofluorescence finding Immunofluorescence finding Immunofluorescence finding


• Linear GBM deposits of IgG and • “Lumpy bumpy” granular pattern • No immunoglobulin or
C3 of staining. complement deposits in GBM.

26. Ans. (a) Nephrin


(Ref: Robbins 9/e p912, 8th/918, Nelson 17th/1757)
• Lipoid nephrosis or nephrotic syndrome is also called as minimal change disease.
Congenital nephrotic syndrome occurs as a result of mutation in the nephrin gene.
(Finnish type of nephrotic syndrome)
Also now
• Megalin in involved is the pathogenesis of membranous glomerulonephritis
• Podocin and a4 actinin can also result in congenital nephrotic syndrome.

27. Ans. (b) Alport syndrome


(Ref: Robbins 9/e p912, 8th/920, 7th/523-4)
As discussed earlier, crescentic glomerulonephritis is the alternative name of RPGN.
Alport syndrome is a hereditary nephritis characterized by associated nerve deafness and eye disorders
like lens dislocation, posterior cataracts and corneal dystrophy. Histologically foam cells (interstitial cells
having fats and mucopolysaccharides) are seen.

28. Ans. (b) Mesangiocapillary GN

(Ref: Anderson 10th/2076; Robbins 7th/979)


Glomerular lesions with increased cells in the tufts are often known as proliferative
glomerulonephritis.
Conditions with Proliferative Glomerulonephritis
• SLE (particularly class II, mesangial hypercellularity defined as >3 cells in mesangial regions)
• HIV
• Membranoproliferative/Mesangiocapillary glomerulonepritis
• Neoplasia (particularly CLL and MALT lymphoma)
• Post streptococcal glomerulonephritis

Rest all options are nonproliferative glomerulonephritis.


29. Ans. (a) Thrombotic microangiopathy
(Ref: Robbins 9/e p941, 8th/952, 7th/1009, 612, Harrison 17th/1813-4)
The term thrombotic microangiopathy encompasses a spectrum of clinical syndromes that include
TTP and HUS.
Microthrombi are demonstrated in renal arterioles and capillaries. In HUS, microthrombi mainly
contain fibrin whereas in TTP, these are composed of platelet aggregates, fibrin and VWF.
More than 90% patients with HUS have significant renal failure whereas < 10% cases of
classic TTP have anuria.
30. Ans. (a) Post streptococcal glomerulonephritis
(Ref: Robbins 9/e p918, 8th/919, Harrison’s 17th/1786)
• Although all conditions mentioned in the question are associated with low complement
levels. Persistently depressed levels’ are not seen in post streptococcal
glomerulonephritis.
• In poststreptococcal glomerulonephritis: serum C3 levels are depressed within 2
weeks, however, these usually return to normal levels within 6 to 8 weeks (transient
decrease of complement levels).
• Persistently depressed levels after this period should suggest another cause such
as presence of C3 nephritic factor (Membranoproliferative glomerulonephritis).
31. Ans. (c) Diarrhea associated hemolytic uremic syndrome
(Ref: Harrison 17th/1813-1814, 9/e p942)
Nephritic syndrome associated with low C3
Immune complex glomerulonephritis • Crescenteric glomerulonephritis
• Post streptococcal glomerulonephritis • Idiopathic proliferative glomerulonephritis
• Lupus nephritis • Atheroembolic renal disease
• Cryoglobulinemia • Sepsis
• Bacterial endocarditis • Acute Pancreatitis/advanced liver disease
• Shunt nephritis
• Membranoproliferative glomerulonephritis

*Hemolytic uremic syndrome is associated with normal C3 levels.


32. Ans. (c) Goodpasture’s disease
(Ref: Harrison’s 17th/1788)
33. Ans. (d) Absence of associated proteinuria is pathognomic
(Ref: Robbin’s 7th/986-988, 9/e p923)
Berger’s disease (IgA nephropathy) is a frequent cause of gross or microscopic hematuria. Mild
proteinuria is frequently present and occassionally nephrotic syndrome may develop.
34. Ans. (b) Rapidly progressive glomerulonephritis
(Ref: Robbins 9/e p912, 8/e p920-921)
• Rapidly progressive glomerulonephritis, also known as crescentic glomerulonephritis is
characterized by the presence of crescents in most of the glomeruli.
• Crescents are produced by proliferation of the parietal epithelial cells of Bowman’s
capsule and by infiltration of monocytes and macrophages.
35. Ans. (c) Crescentic glomerulonephritis (Ref: Robbins 9/e p912, 8th/516-7,
7th/541, Harrison 17th/2121-3)
Discussed earlier.
36. Ans. (c) Alport syndrome
(Ref: Robbins 9/e p924, 8th/932, 7th/988, Rubin pathology/361)
For Alport syndrome, Rubins mentions ‘The most diagnostic morphologic lesion is seen only by
electron microscopy as an irregularly thickened GBM, with splitting of the lamina densa into
interlacing lamellae that surround electron-lucent areas’
37. Ans. (b) Collapsing
(Ref: Robbin’s 9th/p 920)
• Five mutually exclusive variants of focal segmental glomerulosclerosis (FSGS) may
be distinguished by the pathological findings seen on renal biopsy
– Collapsing variant
– Glomerular tip lesion variant
– Cellular variant
– Perihilar variant
– Not otherwise specified variant (NOS)
• The NOS variant is the most common subtype and collapsing FSGS has worst prognosis.
38. Ans. (c) Podocin
(Ref: Robbins 9/e p918, 8th/927, 7th/983-4)
39. Ans. (d) Hypertrophy and necrosis of visceral epithelium
(Ref. Robbin’s 9/e p920, 8th/926, 7th/983-4)
Collapsing Glomerulopathy is characterised by
• A characteristic feature is proliferation and hypertrophy of glomerular visceral epithelial cells.
• The minimum diagnostic criteria for defining a collapsing variant of FSGS is the presence by light
microscopy of at least one glomerulus showing segmental or global obliteration of the glomerular
capillary lumen by wrinkling and collapse of glomerular basement membrane in association with
hypertrophy and hyperplasia of overlying visceral epithelial cells.

40. Ans. (b) NPHS-2


(Ref: Robbin 9/e p918)
41. Ans. (c) Collapsing glomerulopathy

(Ref: Harrison 17th/1796, Robbins, 9/e p920, 8th/928, 7th/982- 3)


42. Ans. (d) Amyloidosis

(Ref: Robbins 9/e p926, 8th/252,610-1,927-8)


• Mesangial deposits of monoclonal kappa/lambda light chains suggest the diagnosis of
Amyloidosis. In primary Amyloidosis AL protein is deposited in the organs, which is
made of light chains (usually lambda type) of immunoglobulins. Renal amyloidosis is the
most common cause of death due to amyloidosis (including both primary and
secondary amyloidosis).
Other options:
• Mesangioproliferative glomerulonephritis shows mesangial deposits of IgG, IgA and C3.
• Focal and segmental glomerulosclerosis shows hyaline deposits of IgM, C3 and IgA and fibrinogen in
juxtamedullary capillaries.
• In Kimmelstiel-Wilson disease (or nodular glomerulosclerosis), the hyaline masses are deposited in
the mesangial core of the glomerular lobules consist of lipids and fibrin.

43. Ans. (b) Has molecular weight slightly greater than the molecules normally getting
filtered
(Ref: Robbins 9/e p914, 8th/910, Harrison 18th/2334, Ganong 21st/709-10; Guyton 10th/373)
• There are three main types of plasma proteins which include Albumin, Globulin and
Fibrinogen
• Normally, the glomerular filtration layer does not allow any plasma protein to pass
through it. However, in any renal disease, it allows protein molecules to pass through it
and albumin is the first protein to appear in the urine.
• The filtration of any substance through glomerular filtration layer depends on two factors
as described below:
Size of the substance

The glomerular filtration layer is thick and porous membrane. Any neutral substance < 4A
diameter freely filters through this layer. However, the filterability of any neutral substance with
diameter between 4 nm and 8 nm is inversely proportional to its size. The filterability of
substances of diameter more than 8 nm is zero.

Charge of the substance

The sialoproteins contained in the glomerular filtration layer are negatively charged
causing repulsion of all negatively charged particles including proteins. This explains the
negligible filtration of albumin anion which has diameter of 7 nm

In some renal diseases, the anionic charge of the filtration membrane is lost and this allows
negatively charged particles of diameter 8 nm to pass through it resulting in proteinuria.
Albumin has a diameter of 7 nm and it starts appearing in urine as soon as the negative
charge of filtration layer disappears.
44. Ans. (a) Fusion of foot process of the glomerular epithelial cells
(Ref: Robbins 9/e p917, 8th/925)
• The child is presenting with features likely of Nephrotic syndrome whose most frequent
cause in children is minimal change disease or lipoid nephrosis.
• Light microscopy there is no abnormality whereas on electron microscopy there is fusion
of foot processes of the glomerular epithelial cells with normal glomeruli.
45. Ans. (a) Number of crescents
(Ref: Robbins 6th/453, Essential of nephrology by K visweswaran 2nd/102 )
As discussed in text the prognosis in RPGN is related to the number of crescents. However, this
point is not mentioned in the 7th, 8th and 9th editions of Robbins.
Poor prognostic factors Good prognostic factors
• Oliguria and azotemia at presentation • Pauci immune RPGN has best prognosis
• More than 80% circumferential crescents have • Non- circumferential crescents in less than
poor response to therapy 50% glomeruli have indolent course.
• Glomerular tuft necrosis, global glomerular • Associated endocapillary proliferation is a
sclerosis, gaps in Bowman capsule and interstitial good prognostic factor
fibrosis

Essential of nephrology writes that ‘oliguria is related to crescent formation’.


46. Ans. (d) No change seen
(Ref: Robbins 9/e p917, 7th/981-982)
47. Ans. (d) Glomerular function is lost due to loss of polyanionic charge on both sites of
glomerular foot process.
(Ref: Harrison 17th/1790, Robbins 9/e p900)
As discussed earlier, Glomerular barrier function depends on the molecular size (the larger, the less
permeable) and charge (the more cationic, the more permeable) is done. The anionic moieties
present within the capillary wall including the acidic proteoglycans of the GBM and the
sialoglycoproteins of epithelial and endothelial cells are responsible for the virtually complete
exclusion of albumin (also having anionic charge) from the filtrate. In addition, the visceral
epithelial cell (or podocyte) is important for the maintenance of glomerular barrier function. The
slit diaphragm of the podocyte is composed of nephrin, actin and podocin, so, any defect in
any of these is responsible for increased protein excretion from the kidney. In a patient of
minimal change disease, there is presence of visceral epithelial injury leading to the loss of
glomerular polyanions resulting in low molecular weight proteinuria (selective proteinuria).
Clarifying Option c; Foot process of the podocyte is effaced. However, it appears to be normal on light
microscopy.

48. Ans. (a) Poststreptococcal glomerulonephritis


(Ref: Robbins 9/e p910, Harrison 17th/1786-1787)

49. Ans. (a) PSGN; (b) MPGN; (e) Infective endocarditis:

(Ref: Robbins 9/e p910, Harrison 17th/1788)

50. Ans. (a) MPGN-l; (c) PSGN; (d) Membranous GN; (e) RPGN
(Ref: Robbins 9/e p910, 7th-975)
Summary of Glomerular Deposits
Subepithelial Subendothelial Basement membrane Mesangial
• Acute GN (like PSGN) • MPGN (Type I) • MPGN (Type II) • IgA nephropathy
• Membranous GN • SLE • Membranous • HSP
• Heymann GN • Acute GN • Glomerulopathy • Anti-GBM diseases like
• RPGN (some cases) • RPGN and Goodpasture
• MPGN (Type I) rarely syndrome

51. Ans. (a) Proteinuria; (b) Hyperlipidemia; (c) Edema; (e) Lipiduria
(Ref: Robbins 9/e p914, 8th/907, 7th/978)
52. Ans. (a) Albumin (Ref: Robbins 9/e p914, 7th/958,
Vasudevan-Sreekumari 4th/224)
Albuminuria is seen in nephrotic syndrome.
53. Ans. (b) Massive proteinuria; (d) Edema; (e) Hyperlipidemia.
(Ref: Robbins 7th/978, 9/e p914)
54. Ans. (a) Heymann antigen is called megalin; (d) Subepithelial aspect of basement
membrane has deposit.
(Ref: Robbins 7th/968 – 970, 9/e p903, 915)
• The Heymann model of rat glomerulonephritis is induced by immunizing rat with an
antigen containing preparation of proximal tubular brush border.
• The rats develop antibodies to this antigen and a membranous glomerulopathy,
resembling human membranous glomerulopathy. The antigen is called megalin and has
homology to LDL receptor.
• On electron microscopy, the glomerulopathy is characterized by presence of numerous
electron dense deposits along the subepithelial aspect of basement membrane.
Immunofluorescence microscopy shows granular deposits.
55. Ans. (b) Malignant nephrosclerosis.
(Ref: Robbins 7th/1008, 9/e p939)

56. Ans. (a) Foamy cells in interstitium; (d) Thinning of GBM < 100 nm:
(Ref: Robbins 7th/988, 9/e p924)
Histological characteristics of Alport’s syndrome:
• Diffuse basement membrane thinning
• Vascular sclerosis
• Foam cells in interstitium
• Tubular atrophy
• In advanced stage there is focal or global glomerulosclerosis
• Interstitial fibrosis

On electron microscopy (diagnostic of this disorder), GBM shows


• Irregular foci of thickening alternating with thinning
• Pronounced splitting and lamination of lamina densa (basket weave appearance)Q.
• Info: Immunohistochemistry shows failure to stain a-3, 4, 5 collagen.

57. Ans. (b) Wegener’s granulomatosis (c) Goodpasture’s syndrome (e) Microscopic
polyangiitis
(Ref: Robbins 9/e p912, 7th/977)
58. Ans. (a) Subepithelial deposits; (b) Nephritis along with acute renal failure; (c) Low
complement levels; (d) HTN and proteinuria:
(Ref: Robbins 9/e p910-911)
59. Ans. (c) Microscopic polyangiitis
(Ref: Robbin 7th/540, 9/e p912, Harrison 17th/1789)
• In microscopic polyangiitis, there is a paucity of immunoglobulin demonstrable by immunofluorescence
microscopy (pauci-immune injury). There are few or no immune deposits in this type of vasculitis.
• Pauci immune injury is also seen in Churg Strauss syndrome and Wegener granulomatosis.

60. Ans. (a) Diabetic nephropathy


(Ref: Robbin 9/e p1118)
61. Ans. (a) Goodpasture’s syndrome
(Ref: Robbins 7th/745-746, 9/e p912)
62. Ans. (a) Diabetic glomerulosclerosis
(Ref: Robbins 9/e p1118)
63. Ans. (a) Normal C3
(Ref: Robbins 9/e p910-911)
64. Ans. (b) Mesangioproliferative glomerulonephritis
(Ref: Robbins 7th/403-405 , 9/e p393)
• Visceral leishmaniasis is caused by L. donovani
• Mucocutaneous leishmaniasis is caused by L. braziliensis.
• Visceral leishmaniasis is characterized by the clinical features of hepatosplenomegaly,
lymphadenopathy, pancytopenia, fever and weight loss. There is hyperpigmentation of the skin, so it is
also called as “Kala azar”.
• In the renal involvement in this disease, there is presence of mesangioproliferative glomerulonephritis
and in advanced cases, there is presence of amyloidosis.

65. Ans. (a) Crescents in most of glomeruli


(Ref: Robbins 975 7th/20-6, 9/e p910)
66. Ans. (a) Idiopathic cresenteric glomerulonephritis
(Ref: Robbins 9/e p910-911, 8th/920; 7th/977)
67. Ans. (b) Diffuse involvement
(Ref: Robbins 9/e p911)
68. Ans. (a) Diffuse glomerulosclerosis
(Ref: Robbins 9/e p1118, 8th/934; 7th/990-992)
69. Ans. (c) Membranoproliferative glomerulonephritis
(Ref: Robbins 8th/929; 7th/985)
70. Ans. (a) Poststreptococcal GN
(Ref: Robbins 8th/920)
71. Ans. (b) Nodular glomerulosclerosis
(Ref: Robbins 9/e p1118, 8th/934; 7th/991)
72. Ans. (d) Diffuse alveolar involvement
(Ref: Robbins 8th/709-710, 9/e p913)
73. Ans. (c) IgG deposition in mesangium
(Ref: Robbins 9/e p923, 8th/925-926; 7th/981)
74. Ans. (b) Berger disease
(Ref: Robbins 9/e p923)
75. Ans. (d) Rheumatoid arthritis
(Ref: Robbins 9/e p912)
76. Ans. (d) Sodium and water retention
(Ref: Robbins 9/e p922, 8th/407-408; 7th/522)
77. Ans. (b) Hematuria
(Ref: Robbins 9/e p923)
78. Ans. (d) Focal segmental glomerulosclerosis
(Ref: Robbins 9/e p919, 8th/926, 7th/983)
79. Ans. (d) All of the above
(Ref: Robbins 9/e p942)
80. Ans. (a) Membranous glomerulonephritis
(Ref: Robbins 9/e p915, 8th/918, 923; 7th/984,979-981)
81. Ans. (c) Wegener’s glomerulonephritis
(Ref: Robbins 9/e p903, 8th/920; 7th/993,997)
82. Ans. (b) Membranoproliferative glomerulonephritis
(Ref: Robbins 9/e p920, 8th/929; 7th/984)
83. Ans. (b) Membranous nephropathy
(Ref: Robbins 9/e p915, 8th/918,920; 7th/975, 979)
84. Ans. (a) Focal segmental glomerulosclerosis
(Ref: Robbins 8th/928, 9/e p920)
There is a specific association between focal segmental glomerulosclerosis and both IV drug abuse
and HIV nephropathy. This disorder usually presents as an aggressive form of nephrotic
syndrome with poor prognosis and non responsive to steroid therapy (called as ‘collapsing
variant’).
85. Ans. (a) Epithelial cells + fibrin + macrophage
(Ref: Robbins 8/e p921, 9/e p913)
Crescents are formed by proliferation of parietal cells and by migration of monocytes and
macrophages into the urinary space. Neutrophils and lymphocytes may be present. Fibrin
strands are frequently prominent between the cellular layers in the crescents. The escape
of fibrinogen into Bowman space and its conversion to fibrin are an important contributor to
crescent formation.
Rapidly progressive glomerulonephritis is characterized histologically by domination of distinctive
crescents. It is also known as Crescentic glomerulonephritis. The crescents eventually
obliterate Bowman space and compress the glomerular tuft. So, the prognosis is dependent on
the number of crescents in the kidney biopsy.
86. Ans. (a) Malignant hypertension
(Ref: Robbins 8/e p950, 9/e p939)
Mnemonic for differential diagnosis of flea bitten kidney: We HaTe PSM

We - Wegener’s granulomatosis
HaTe – Henoch Schonlein purpura; HUS;TTP
P- Poststreptococcal glomerulonephritis (PSGN); Polyarteritis nodosa (PAN)
S-Subacute bacterial endocarditis (SABE)
M- Malignant hypertension

87. Ans. (c) Fibrinogen


(Ref: Robbins 8/e p922, Heptinstall’s Pathology of the Kidney 6/e p131)
Direct quote from Heptinstall’s Pathology.. “Many factors increase in nephrotic syndrome usually due
to lowering of serum albumin to which they are usually bound. These factors include
fibrinogen, factors V, VII, VIII and von Willebrand factor. On the other hand, substances like
factors XI and XII, plasminogen an anti-thrombin III.”
Thrombotic and thromboembolic complications are common in nephrotic syndrome due to loss of
anticoagulant factor (e.g. antithrombin III, protein C and S) combined with increased platelet
activation. Renal vein thrombosis is most often a consequence of this hypercoagulative state.
There is also increased synthesis of fibrinogen in the liver.

Figure: Mechanisms of edema formation in the nephrotic syndrome left. The classic view of edema formation, in which
a low blood volume (underfill) serves as the signal for secondary renal sodium retention Right. The mechanism of
edema formation in most patient with the nephrotic syndrome who have normal or slightly elevated blood volumes
(overfill). The blunted response to atrial natriuretic peptide observed in patients with the nephrotic syndrome may be the
stimulus for primary renal sodium retention that plays a central role in edema formation. ANP, atrial natriuretic peptide.

Edema in nephrotic syndrome is attributed to the following:


• Underfill hypothesis/classical hypothesis:

– Seen in patients with low plasma volume
– Hypovolemia is the primary stimulus leading to sodium and water retention leading to edema by alteration
of Starling forces

• Overfill hypothesis:

– Seen in patients with normal or increased plasma volume


– Sodium retention is the primary mechanism leading to increased blood pressure and then alteration in
Starling forces and edema formation.

88. Ans. (d) Focal segmental glomerulosclerosis


(Ref: Robbins 8/e p926, 9/e p 918)
Focal segmental glomerulosclerosis is the commonest cause of nephrotic syndrome in the
adults in the World. It is characterized by
NEET Points to be brushed up!

• Associated with loss of renal tissue as unilateral renal agenesisQ or advanced stages of reflux nephropathy
or hypertensive nephropathy.
• Also seen with with conditions like Sickle cell anemiaQ, HIV infectionQ, Heroin abuse, Obesity

• Degeneration and focal disruption of the visceral epithelial cells is the hallmark feature of focal segmental
glomerulosclerosis.
• Is chief renal lesion in HIV associated nephropathy (especially collapsingQvariant)

89. Ans. (b) Rapidly progressive glomerulonephritis… discussed earlier


(Ref: Robbins 7/e p976-977, 9/e p912)
90. Ans. (a) Berger disease
(Ref: Robbins 9/e p912-913)
91. Ans. (c) Mild proteinuria, hematuria, high ASO titre
(Ref: Robbins 9/e p910-911)
In poststreptococcal glomerulonephritis, there elevated levels of antistreptolysin O or ASO and anti
DNAase antibodies Q (indicative of streptococcal infection) and reduced levels of serum C3
92. Ans. (d) Renal biopsy having thin basement membrane
(Ref: Robbins 9/e p925)
Benign familial hematuria is common hereditary entity manifested clinically by familial asymptomatic
hematuria—usually uncovered on routine urinalysis—and morphologically by diffuse thinning
of the GBM to widths between 150 and 225 nm (compared with 300 to 400 nm in healthy
adults).
93. Ans. (c) 30-300 mg/d
(Ref: Robbins 8/e p1145, 9/e p1120)

• The earliest manifestation of diabetic nephropathy is the appearance of low amounts of albumin in the urine
(>30 mg/day, but <300 mg/day), that is, microalbuminuria.
• Microalbuminuria is also a marker for greatly increased cardiovascular morbidity and mortality for persons with
either type 1 or type 2 diabetes

94. Ans. (c) Partial lipodystrophy


(Ref: Robbins 9/e p 922)
Secondary MPGN (invariably type I) is more common in adults and arises in the following:
• Chronic immune complex disorders, such as SLE; hepatitis B infection; hepatitis C infection, usually with
cryoglobulinemia; endocarditis; infected ventriculoatrial shunts; chronic visceral abscesses; HIV
infection; and schistosomiasis
• α1-Antitrypsin deficiency
• Malignant diseases particularly CLL which have formation of autoantibodies
Robbins 8th mentioned that partial lipodystrophy is associated with C3 nephritic factor (C3NeF). It is
associated with type II membranoproliferative glomerulonephritis.
95. Ans. (d) Rapidly progressive glomerulonephritis
(Ref: Robbins 9/e p912)
RBC casts are a feature of glomerular damage. Normally < 3 RBC/HPF are going to leak. But in
case of glomerular damage the number of RBC in the urine will exceed the limit mentioned
above and these RBC get impinged on Tamm Horsfall protein. The resultant RBC casts can be
seen under microscopic examination of urine.
96. Ans. (a) Focal segmental glomerulonephritis
(Ref: Robbins 9/e p919)
A morphologic variant of FSGS is called collapsing glomerulopathy and is the most characteristic
lesion of HIV-associated nephropathy.
97. Ans. (d) Minimal change disease (Ref: Robbins 9/e p 917)
The glomeruli are normal by light microscopy. By electron microscopy the GBM appears normal,
and no electrondense material is deposited. The principal lesion is in the visceral epithelial
cells, which show a uniform and diffuse effacement of foot processes.

Foot process effacement is also present in other proteinuric states (e.g., membranous glomerulopathy, diabetic
nephropathy); it is only when effacement is associated with normal glomeruli by light microscopy that the
diagnosis of minimal-change disease can be made.

98. Ans. (b) Minimal change disease (Ref: Robbins 9th/917)

• Presence of edema, proteinuria (frothy urine in stem of question), hypoalbuminemia etc. is suggestive of
nephrotic syndrome.

• Minimal change disease is the most frequent cause of nephrotic syndrome in children. There is commonly no
hyper-tension or hematuria. The proteinuria usually is highly selective, most of the protein being albumin.
• No RBC casts in the urine is suggestive of absence of glomerulonephritis. So, options ‘a’ and ‘c’ are ruled out.
Membranous nephropathy causes nephrotic syndrome in adults. The best answer therefore is option ‘b’

99. Ans. (c) Membranous glomerulopathy

(Ref: Nephrol Dial Transplant (2001) 16: 13-14 Robbins 9th/832-3)


Malignancy associated nephropathy is a recognised entity for many years which usually presents as
nephrotic syndrome.
The most common association are:

• Solid tumors cause nephrotic syndrome due to membranous glomerulopathy.


• Hodgkin disease cause nephrotic syndrome due to minimal change disease.

100. Ans. (c) Crescent formation


(Ref: Robbins 9/e p913)
Presence of crescents in the glomeruli is the characteristic microscopic finding in patient of rapidly
progressive glomerulonephritis.
101. Ans. (c) Membranous glomerulopathy
(Ref: Robbins 9/e p916)

Presence of granular IgG deposits along the glomerular basement membrane is associated with membranous
glomerulopathy.
Berger‘s disease (IgA nephropathy) is associated with linear Ig deposits are seen.
102. Ans. (b) RPGN
(Ref: Robbins 9/e p912)
The features are suggestive of nephritic syndrome and the only option associated with
nephritic syndrome is RPGN.
103. Ans. (c) IgA deposition in mesangium
(Ref: Robbins 9/e p926)

104. Ans. (d) Berger’s nephropathy


(Ref: Robbins 9/e p923)
105. Ans. (d) Increased complement levels
(Ref: Robbins 9/e p923)
106. Ans. (b) Cardiac hypertrophy
(Ref: Robbins 9/e p924)
107. Ans. (c) Serum complement level is normal
(Ref: Robbins 9/e p923)
108. Ans. (d) Lipoid nephrosis
(Ref: Robbins 9/e p222-223)
109. Ans. (d) Urine PCR for TB
(Ref: Robbins 9/e p943, 8th/947, Harrisons 18th/2372)
110. Ans. (a) Isomorphic RBC (Ref: Paediatric Nephrology
5th/141, 190 by R.N. Srivastava, Arvind Bagga (senior faculty from AIIMS)
Approach to patient with hematuria to determine source of bleeding

Parenchymal Intrarenal Collecting system Extrarenal

Appearance of urine Tea colored Bright red, blood clots

Pattern of hematuria Total hematuria (throughout the stream) Initial, terminal hematuria

Urinary symptoms Painless Dysuria, urgency, frequency

Associated features Sore throat, HTN, edema Fever, colicky pain

Family history Deafness, renal failure Renal stones, urinary infection

Proteinuria High grade (urine protein: creatinine ratio >1) Low grade

Other urinary findings RBC casts (highly specific less sensitive) Crystals

RBC morphology Dysmorphic Eumorphic

111. Ans. (b) Associated with tuberculosis


(Ref: Robbins 8th/943, 9/e p934)
As discussed in text, Xanthogranulomatous pyelonephritis is associated with E. coli Proteus
mirabilis, Pseudomonas, Streptococcus faecalis and Klebsiella.
112. Ans. (d) Benign Nephrosclerosis (Ref: Robbins 9/e p938, 8th/949,
7th/992, 1006; Chandrasoma taylor 3rd/275)
Direct quote Robbins.. ‘In benign nephrosclerosis, kidneys are either normal or moderately reduced
in size on gross appearance. The loss of mass is due mainly to cortical scarring and shrinking’.
Causes of contracted kidneys
Symmetric Asymmetric
• Chronic glomerulonephritis • Chronic pyelonephritis
• Benign nephrosclerosis

Causes of enlarged kidneys


• Amyloidosis • Diabetic renal disease [Kimmelstiel Wilson
• Rapidly progressive glomerulonephritis (RPGN) nodules are pathognomic]
• Myeloma kidney • Polycystic kidney disease
• Bilateral obstruction (hydronephrosis)

Important info
Contracted kidneys:Less than 8 cm length of kidney is taken as chronic contracted kidney. Normal size
corresponds to 3 times the length of L1 vertebrae or 2/3rd of additive length of T11, T12 and L1 vertebrae.

Note: In some patients of diabetes (especially in late stages), kidney may be reduced in size.

113. Ans. (b) Tuberculous pyelonephritis


(Ref: Robbins 7th/1004, 9/e p936, Harrison 17th/1825 – 1826)
Necrotizing papillitis is the other name of acute papillary necrosis. When infection of renal
pyramids develop in association with vascular diseases of the kidney or with urinary tract
obstruction, renal papillary necrosis is likely to result. In the given options, DM is the
commonest and TB is the rarest cause of papillary necrosis
• Diabetes mellitus
• Sickle cell disease
• Chronic alcoholism
• Vascular disease
• Analgesic abuse nephropathy
114. Ans. (a) PCT
(Ref: Robbins 9/e p928, 8th/937-8, 7th/918)
115. Ans. (d) Medullary cystic kidney (Ref: Harrison 18th/2383,
16th/1807, Robbin’s 9/e p948, 8th/947, 7th/1005)
Nephrocalcinosis is a diffuse deposition of calcium salts in the interstitium of the kidney.
Conditions associated with nephrocalcinosis are:
• Hyperoxaluria • Hyperthyroidism
• Hyperparathyroidism • Hyperuricosuria
• Prolonged immobilization • Renal candidiasis
• Hypervitaminosis D • Excessive calcium intake (milk alkali
• Hypophosphatemic rickets syndrome)
• Excessive bone destruction in metastasis • Sarcoidosis
• Cortical necrosis malignancies (such as multiple • Renal tubular acidosis (distal)
myeloma) • Medullary sponge kidney
• Cushing syndrome

116. Ans. (a) Goodpasture’s syndrome; (b) Leptospirosis; (d) Wegener’s granulomatosis; (e)
Hantan virus infection;
(Ref: Harrison 17th/1793)
Pulmonary renal syndrome is seen in
• Goodpasture’s syndrome: pulmonary hemorrhage and renal failure
• Leptospirosis: Renal and hepatic dysfunction, Hemorrhagic pneumonia, bleeding diathesis
• Hantan virus also cause pulmonary renal syndrome.
• Wegener’s granulomatosis: Lung and kidney involvement common.
• Other causes of pulmonary renal syndrome include Henoch Schonlein purpura, Churg Strauss
vasculitis, microscopic polyangiitis and cryoglobulinemia.

Please note that Legionella does not affect kidneys. It causes atypical pneumonia, diarrhea and
hyponatremia.*
117. Ans. (b) DM, (c) Phenacetin abuse, (d) Alcoholism
(Ref: Harrison 17th/1826, Robbins 9/e p936, 8th/947, 7th-1004)
Causes of papillary necrosis
Diabetes Analgesic Sickle cell Obstruction
mellitus nephropathy disease
M:F 1:3 1:5 1:1 9:1
Time course 10 years 7 years of abuse Variable Variable
Infection 80% 25% + or – 90%
Calcification Rare Frequent Rare Frequent
Number of papillae Several; all of Almost all; all in Few Variable
affected same stage different stages of
necrosis

118. Ans. (a) Hyperparathyroidism; (b) TB Kidney; (c) Hypercalcemia


(Ref: Harsh Mohan 6th/685, Robbins 9/e p937, 7th/l005)
Causes of Nephrocalcinosis
• Distal RTA • Hyperparathyroidism • Multiple myeloma
• Severe hypercalcemia • Hypercalcemia • Vitamin D intoxication
• Medullary sponge kidney • TB kidney • Metastatic bone disease

119. Ans. (d) Polycystic kidney disease; (e) Amyloidosis:


(Ref: Robbins 7th/964, 992, 989, 9/e 947)
Causes of bilaterally enlarged kidneys Causes of small contracted kidney
• Polycystic kidney diseases • Benign nephrosclerosis
• Amyloidosis of kidney • Chronic glomerulonephritis
• DM • Chronic Pyelonephritis

120. Ans. (c) Alport’s syndrome


(Ref: Robbins 7th/988, 9/e p924, Harrison 17th/1794)
Hereditary Nephritis refers to a group of heterogenous hereditary familial diseases associated
primarily with glomerular injury. These are: Alport’s syndrome and Thin membrane disease.
121. Ans. (b) Medullary sponge kidney; (d) RTA
(Ref: Robbins 7th/1005, 9/e p937,948)
122. Ans. (d) Neutrophilic infiltration; (e) Necrotizing vasculitis:
(Ref: Robbins 7th/220)
Acute rejection can be caused by either cellular or humoral mechanisms
The histologic features of acute humoral rejection (within days) in renal transplants are:
• Necrotizing vasculitis
• Endothelial cell necrosis
• Neutrophilic infiltration
• Deposition of immunoglobulins
• Complement and fibrin deposition and thrombosis.
• There is extensive necrosis of renal parenchyma.
In acute cellular rejection
• Extensive interstitial mononuclear cell infiltration
• Edema
• Interstitial hemorrhage is seen.
In hyper acute rejection (within minutes or hours)
• Fibrinoid reactions are seen.
In chronic rejection (over period of 4-6 months)
• Vascular changes consisting of dense, obliterative intimal fibrosis, principally in the cortical arteries
seen. It clinically presents with progressive rise in serum creatinine.

123. Ans. (b) Schistocytes


(Ref: Harrison 17th/723, Robbin 9/e p644)
The presence of a severe hemolytic anemia with schistocytes or fragmented red blood cells in the
peripheral blood smear is characteristic of microangiopathic hemolytic anemia such as in
Hemolytic uremic syndrome and Thrombotic thrombocytopenic purpura.
124. Ans. (b) Cortical tubular hypertrophy
(Ref: Robbins 8th/959, 7th/966)
• Cortical tubulointerstitial damage occurs in nephronophthisis.
Nephronophthisis (Uremic Medullary Cystic Disease Complex)
• It is a group of progressive renal disorders that usually have onset in childhood.
• Common characteristic is the presence of a variable number of cysts in the medulla,
usually concentrated at the corticomedullary junction.
• Cortical tubulointerstitial damage occurs.
• 4 variants of this disease are:
– Sporadic/nonfamilial (20%)
– Familial juvenile nephronophthisis (40-50% autosomal recessive).
– Renal-retinal dysplasia (15% autosomal recessive)
– Adult-onset medullary cystic disease (15% autosomal dominant).
• Affected children present first with polyuria and polydipsia which reflect a marked tubular
defect in concentrating ability.
• Sodium wasting and tubular acidosis are also prominent.
• Renal failure occurs in a period of 5-10 years.
• In gross appearance, the kidneys are small, have contracted granular surfaces and show
cysts in the medulla, most prominently at the corticomedullary junction.
• Small cysts are also seen in the cortices and are lined by flattened or cuboidal epithelium.
• In the cortex, there is widespread atrophy and thickening of the basement membranes of
proximal and distal tubules together with interstitial fibrosis.
• In general, glomerular structure is preserved.
125. Ans. (a) Diabetes mellitus
(Ref. Robbins 9/e p936)
126. Ans. (a) Calcium oxalate
(Ref: Robbins 9/e p951)
127. Ans. (c) Interstitial nephritis
(Ref: Robbins 9/e p930)
128. Ans. (c) Diabetes mellitus
(Ref: Robbins 9/e p936)
129. Ans. (b) Lupus nephritis
(Ref: Robbins 9/e p929)
130. Ans. (d) Lipoid nephrosis
(Ref: Robbins 9/e p222-223)
131. Ans. (a) Fibrinoid necrosis
(Ref: Robbins 9/e p939)
132. Ans. (c) Amyloidosis
(Ref: Robbins 8th/254)
133. Ans. (a) Acute glomerulonephritis
(Ref: Robbins 9/e p911)
134. Ans. (a) Calcium oxalate stone
(Ref: Robbins 9/e p951)
135. Ans. (a) Goodpasture’s syndrome
(Ref: Robbins 9/e p912)
136. Ans. (c) Cystitis
(Ref: Robbins 8th/974-5, 9/e p962)
The clinical presentation of the patient in the stem of the question is of cystitis, which is
characterized by pyuria and hematuria but absence of white cell casts in the urine.
Patients with acute pyelonephritis present with fever, leucocytosis, flank tenderness, urinary white
cells, and white cell casts in the urine. Chronic pyelonephritis is almost always the result of
chornic urinary tract obstruction and repeated bouts of acute inflammation in the kidneys.
137. Ans. (c) Hyperparathyroidism
(Ref: Robbins 9/e p951)
The patient’s history of recurrent urolithiasis with calcium-containing stones implies a disorder in the
regulation of calcium concentration. Hyperparathyroidism is associated with increased
parathormone (PTH) levels, which can produce hypercalcemia, hypercalciuria, and, ultimately,
renal stones.
138. Ans. (a) Diabetes mellitus
(Ref: Robbins 8/e p947, 9/e p 936, Top 3 Differentials in Radiology: A Case Review Thieme pg
121)
The most common cause of papillary necrosis is diabetes mellitusQ…..Top 3 Differentials in
Radiology
Encyclopedia of Imaging, Vol 2; pg 1192.. “diabetes mellitus is the most common condition
associated with renal papillary necrosis accounting for >50% of all cases.”
Clinical Pathology Oxford press pg220.. “acute papillary necrosis is most often a complication of
acute pyelonephritis in diabetes. Chronic papillary necrosis is seen most often in association
with analgesic nephropathy”.
Conditions with papillary necrosis: Mnemonic is POSTCARDS
• Pyelonephritis
• Obstruction of the urinary tract
• Sickle cell hemoglobinopathies, including sickle cell trait
• Tuberculosis
• Cirrhosis of the liver, Chronic alcoholism
• Analgesic abuse
• Renal transplant rejection, Radiation
• Diabetes mellitus
• Systemic vasculitis

139. Ans. (b) Diabetes mellitus

(Ref: Robbins 8/e p934, 7/e p991-992)

Causes of contracted kidneys


Symmetric Asymmetric
• Chronic glomerulonephritis • Chronic pyelonephritis
• Benign nephrosclerosis
Causes of enlarged kidneys
• Amyloidosis • Diabetic renal disease [Kimmelstiel winos
nodules are pathognomonic]
• Rapidly progressive glomerulonephritis (RPGN) • Polycystic kidney disease

• Myeloma kidney • Bilateral obstruction (hydronephrosis)

140. Ans. (b) Codominant


(Ref: Robbins 8/e p931-2, 9/e p924)
Friends, read the following lines from Robbins carefully….
• Alport syndrome is manifested by hematuria with progression to chronic renal failure,
accompanied by nerve deafness and various eye disorders, including lens dislocation,
posterior cataracts, and corneal dystrophy.
• The disease is inherited as an X-linked trait in approximately 85% of cases. In this X-
linked form, males express the full syndrome and females are carriers in whom
manifestations of disease are typically limited to hematuria.
• Autosomal recessive and autosomal dominant pedigrees also exist, in which males
and females are equally susceptible to the full syndrome
141. Ans. (a) Hyperparathyroidism
(Ref: Robbins 9/e p937)
• Disorders characterized by hypercalcemia, such as hyperparathyroidism, multiple
myeloma, vitamin D intoxication, metastatic bone disease, or excess calcium intake (milk-
alkali syndrome), may induce the formation of calcium stones and deposition of calcium in
the kidney (nephrocalcinosis).
142. Ans. (b) Tuberculous cystitis
(Ref: Bailey 25/e p1108)
TB Urinary Bladder
• Bladder tuberculosis is almost always secondary to renal tuberculosisQ.
• The disease starts at the ureteric opening, the earliest evidence being pallor of the mucosa due to
submucosal edema.
• Subsequently tiny white transluscent tubercles develop all over. Gradually these tubercles enlarge and
may ulcerate (but do not cause bladder perforationQ).
• These tubercles lend ‘cobblestone’ appearanceQ on cystoscopy.
• There is considerable suhmucous fibrosis which causes diminished capacity of bladder. Scarred &
ftbrosed, small capacity bladder is known as thimble bladderQ.
• The fibrosis which usually starts around the ureter, contracts to cause a pull at the ureters. This either
leads to a stricture or displaced, dilated and rigid wide mouthed ureter called as golf hole uretersQ
(this almost always leads to ureteral reflux.)

143. Ans. (a) Chronic pyelonephritis


(Ref: Robbins 9/e p934, 8/e p943, 7/e p989)
In the morphology of chronic pyelonephritis, glomeruli may appear normal except for
periglomerular fibrosis.
144. Ans. (c) Silicosis
(Ref: Robbins 9/e p690)
145. Ans. (b) Crystals are hexagonal
(Ref: Robbins 9/e p 952)
• Hexagonal stones are seen in cystine stones whereas uric acid stones are barrel or diamond shaped.
• Uric acid stones are common in individuals with hyperuricemia, such as patients with gout, and diseases
involving rapid cell turnover, such as the leukemias.
• A tendency to excrete urine of pH below 5.5 may predispose to uric acid stones, because uric acid is
insoluble in acidic urine. In contrast to the radiopaque calcium stones, uric acid stones are
radiolucent.

146. Ans. (a) Nodular glomerulosclerosis


(Ref: Harsh Mohan 6/e p678)
Nodular lesions of diabetic glomerulosclerosis are also called as Kimmelstiel-Wilson (KW) lesions or
intercapillary glomerulosclerosis. These lesions are specific for type 1 diabetes.
147. Ans. (d) Micro RNA-122
(Ref: Harrison 18/e p2304-5)
New question friends and highly likely to be repeated… see annexure.
148. Ans. (c) Flea bitten kidney
(Ref: Robbins 9th/947)
Presence of a granular kidney with multiple haemorrhagic spots is suggestive of either of the
following conditions:

• Malignant hypertension
• Wegener’s granulomatosis
• Henoch Schonlein purpura
• Post streptococcal glomerulonephritis
• Polyarteritis nodosa
• Subacute infective endocarditis

149. Ans. (d) Hypercellular glomeruli


(Ref: Robbins 9/e p931)
Hypercellular glomeruli are a feature of acute glomerulonephritis.
150. Ans. (b) Hypertension
(Ref: Robbins 9/e p932)
Diabetes, analgesics, sickle cell disease and urinary tract infection are the main causes of
sickle cell disease.
151. Ans. (a) Chronic pyelonephritis
(Ref: Robbins 9/e p933)

152. Ans. None or ‘c’ Cushing syndrome. (Ref: Robbins 8th/966, Kidney
Cancer: Principles and Practice (2012) pg 71, Springer, Harrison 17th/592: 618)
Friends, ideal answer of this question would be none. Robbins 8th/966…. ‘renal cell carcinomas
produce a number of paraneoplastic syndromes, ascribed to abnormal hormone production,
including polycythemia, hypercalcemia, hypertension, hepatic dysfunction, feminization or
masculinization, Cushing syndrome, eosinophilia, leukemoid reactions, and amyloidosis.’
However, a table from Kidney Cancer: Principles and Practice is given underneath to help you
decide the fact that if we have to compulsorily mark one option as the answer, then it has to be
Cushing syndrome (option ‘c’) because it has the rarest incidence.
Paraneoplastic manifestations of renal cell cancer syndromes with their incidence
Paraneoplastic syndrome Incidence

Endocrinological 13-20%
Hypercalcemia 40%
Hypertension 1-8%
Polycythemia 3-20%
Stauffer syndrome 10%
Elevated Alkaline phosphatase 2%
Cushing syndrome -
Thrombocytosis 30%
Cachexia 3-8%
Non endocrine 20%
Amyloidosis 3%
Anemia -
Neuromyopathy -
Vasculopathy 20%
Nephropathy
Fever

Stauffer syndrome is the name give to nonmetastatic hepatic dysfunction.

153. Ans. (d) Bilateral polycystic kidney

(Ref: Robbins 8th/479-480, 9/e p479-480)


Wilm’s tumor (Nephroblastoma) is the most common primary renal tumor of childhood in
USA. It is associated with the following:
154. Ans. (c) Trisomy 7/17
(Ref: Robbins 8th/964-964, 9/e p953-954, Harrison 17th/592)
Friends, answer to the question is very easy for us to understand provided we are aware of the fact
that chromophilic renal cell cancer is the other name of papillary renal cell cancer because
80% of chromophilic renal cell cancers show a tubulopapillary architecture. (source….British
Journal of Cancer 1996,74; 1605-1614.). As discussed in text, Papillary cancer is associated
with trisomy 7, 16, 17.
155. Ans. (a) Clear cell type
(Ref: Robbins 7th/1016, 1017, 9/e p953)
156. Ans. (b) Wilm’s tumor
(Ref. Robbins 8th/479, 9/e p479)
Wilm’s tumor is increased in WAGR syndrome, Dennys-Drash syndrome and Beckwith- iedemann
syndrome.
The features mentioned in the question point to a diagnosis of WAGR syndrome. The components
of this syndrome are
W – Wilm’s tumor
A – Aniridia
G – Genital anomalies
R – Mental retardation

157. Ans. (c) Anaplasia


(Ref: Robbins 8th/481, 9/e p481)
5% of tumors reveal anaplasia, defined as the presence of cells with large, hyperchromatic,
pleomorphic nuclei and abnormal mitoses and the presence of anaplasia correlates with
underlying p53 mutations and the emergence of resistance to chemotherapy.
158. Ans. (b) Associated with deletion of chromosome 11p13
(Ref: Robbins 9/e p479, 8th/479-481; 7th/504-506)
159. Ans. (a) Clear cell
(Ref: Robbins 9/e p953)
160. Ans. (d) Collecting duct (Ref: Robbins 8/e p964, 9/e p953)
Oncocytoma

• Arises from the intercalated cells of collecting ductsQ.


• Epithelial tumor composed of large eosinophilic cells having small, round, benign-appearing nuclei that have
large nucleoli.
• Ultrastructurally the eosinophilic cells have numerous mitochondriaQ.

161. Ans. (c) Chromosome 11


(Ref: Robbins 9/e p479-480)
Easiest way to remember that info….. count the number of letters in Wilms tumour..yea it is exactly
11…the location of both genes associated with Wilms tumour
So, the two genes associated with Wilms tumour WT1 gene (located on chr 11p13)and WT2 gene
(located on chr 11p15).
162. Ans. (b) Tumour stage
(Ref: Robbins 8/e p, Rudolph Pediatrics; Robbins 9/e p481)
Also revise that anaplsia is an adverse prognostic factor because it increases the resistance to
chemotherapy and increased chances of recurrence.
Good prognostic factors Poor prognostic factors
• Age < 2 years • Age > 2 years
• Early stage disease (stage I,II) • Late stage disease (stage III,IV)
• Favourable histology • Anaplasia (unfavourable histology)
• Tumour < 500 gm • Tumour > 500 gms
• Loss of genetic material on 11q,16q
• Gain of material on 1q
• Renal vessel and capsule invasion

163. Ans. (c) Wilm’s tumor…. Discussed in a separate question


(Ref: Robbins 8/e p479-480, 9/e p479-480)
164. Ans. (b) 8%
(Ref: Internet: multiple sources)
Renal vein involvement is seen 4-9% of the patients of renal cell carcinoma.
165. Ans. (d) More common in females
(Ref: Robbins, 9/e p953)
166. Ans. (b) Renal cell carcinoma
(Ref: Robbins, 9/e p1134)
167. Ans. (b) Wilms tumor
(Ref: Robbins, 9/e p480)
168. Ans. (b) Malacoplakia
(Ref: Robbins, 9/e p963)
Michaelis-Gutmann bodies are laminated mineralized concretions typically present within the
macrophages resulting from deposition of calcium in enlarged lysosomes. It is seen in
malacoplakia. Similar lesions are also seen in the colon, lungs, bones, kidneys, prostate, and
epididymis. In this conditions, there is presence of PAS+ macrophages.
169. Ans. (c) Atrophy
(Ref: Robbins 7th/9, 1012, 9/e p950)
Chronic urethral obstruction because of urinary calculi, prostatic hypertrophy, tumors, normal
pregnancy, uterine prolapse or functional disorders cause hydronephrosis which by definition
is used to describe dilatation of renal pelvis and calyces associated with progressive atrophy of
the kidney due to obstruction to the outflow of urine.
Concept
Atrophy is shrinkage in size of the cell by loss of cell substance.Atrophied cells are only decreased in size;
they are not dead

170. Ans. (c) Malacoplakia


(Ref: Robbins 9/e p963)
171. Ans. (a) Light chain
(Ref: Robbins 9/e p937-938)
172. Ans. (a) Schistosomiasis
(Ref: Robbins 9/e p965)
Direct lines from Robbins...
Schistosoma haematobium infections in endemic areas like Egypt and Sudan are an established
risk. The ova are deposited in the bladder wall and incite a brisk chronic inflammatory
response that induces progressive mucosal squamous metaplasia and dysplasia and, in some
instances, neoplasia.
Most of these cancers are squamous cellQ carcinomas.
173. Ans. (b) Hypophosphatemia
(Ref: Harrison 18/e)
• Hyponatremia
• Hyperkalemia
• Hyperphosphatemia
174. Ans. (a) p53
(Ref: Robbins 9/e p965)
Loss-of-function mutations in the TP53 and RB tumor suppressor genes are almost always
seen in high-grade and, frequently, muscle invasive tumors.
175. Ans. (b) p16
(Ref: Robbins 9/e p965)
Particularly common (occurring in 30% to 60% of tumors) are losses of genetic material on
chromosome 9 (including monosomy or deletions of 9p and 9q). These abnormalities are
often the only chromosomal changes present in superficial noninvasive papillary tumors.
The 9p deletions (9p20) affects tumor suppressor gene CDKN2A, which encodes the cyclin-
dependent kinase inhibitor p16/INK4a and ARF, a protein that augments p53 function.
176. Ans. (d) Both a and b
(Ref: Robbins 9/e p398, 965)
There is an association between urinary schistosomiasis and squamous cell carcinoma (more
commonly) as well as transitional cell cancer (less commonly) of the bladder.
177. Ans. (b) Embryonal
(Ref: Robbins 9/e p968)

• MC sarcoma in infancy or childhood is embryonal rhabdomyosarcoma. In some cases, it manifests as a


polypoid grapelike mass (sarcoma botryoides).
• MC sarcoma in the bladder in adults is leiomyosarcoma

ANNEXURE

Markers of Kidney Injury

Name of marker Significance


Kidney injury • Type-1 cell membrane glycoprotein upregulated in de: differentiated proximal tubule epithelial cells
molecule-1 (KIM- • Elevated urinary levels are highly sensitive and specific for AKI
1) and Clusterin

Cystatin C • Important extracellular inhibitor of cysteineproteases


• Elevated urinary levels reflect tubular dysfunction; high levels may predict poorer outcome

NGAL • Expression upregulated in kidney proximal tubule cells and urine following ischemic or cisplatin
induced renal injury
• Found to be an early indicator of AKI following cardiopulmonary bypass

IL-18 • Constitutively expressed in distal tubules; strong immunoreactivity in proximal tubules with
transplant rejection
• Elevated urinary levels found to be early marker of AKI and independent predictor of mortality
in critically ill patients

Na+/H+ • For discrimination between prerenal azotemia and AKI in ICU patients
exchanger 3
(NHE 3)

L-FABP Biomarker in CKD and diabetic nephropathy

Osteopontin • Correlates with inflammation and tubulointerstitial fibrosis

β2-Microglobulin • Light chain of the MHC I molecule;


• An early marker of tubular dysfunction

α1-Microglobulin • Synthesized by the liver; tubular dysfunction marker

Microalbumin Marker for monitoring progression of chronic kidney disease


NGAL is Neutrophil gelatinase associated lipocalin, Liver fatty acidbinding protein (L-FABP);
In addition, N-Acetyl--(D) glucosaminidase (NAG), Retinol-binding protein, Cysteine-rich protein
CYR 61, Exosomal fetuin-A and enzymes like Alanine aminopeptidase (AAP), alkaline
phosphatase (AP), Glutathione-S-transferase (α-GST) Glutamyl transpeptidase (γ-GT) are
other markers of acute kidney injury (AKI).
1. A patient presents with hematuria to the clinic. He is afebrile with no history of infection,
no pus cells in urine, and no glycosuria. He has pedal edema. The renal biopsy is given
as under. Which of the following is the next investigation to confirm?
(NEET 2020 like pattern)

(a) Antinuclear antibody


(b) Antiglomerular basement membrane antibody
(c) HIV- RNA
(d) Urine electrophoresis
Ans. (b) Antiglomerular basement membrane antibody
(Ref: Robbins 9th/913)
The clinical presentation of a patient with nephritic syndrome is hematuria with red blood cell
casts in the urine, moderate proteinuria and variable hypertension and edema.
The image given in the question is an electron micrograph showing characteristic wrinkling of
glomerular basement membrane with focal disruptions. This is seen in patients with crescentic
glomerulonephritis.
Absence of any fever and pus cells in the urine is helpful in ruling out an infectious condition
• Urine electrophoresis is indicated for the detection of free monoclonal light chain
paraproteins (Bence-Jones Proteins) in patients with suspected myeloma. The clinical
history given in the question does not fit into this condition.
• Anti nuclear antibody (ANA) is helpful for detection of a condition like SLE but absence of
any extra renal systemic manifestations makes it an unlikely cause.
• The etiological factors leading to crescentic glomerulonephritis cause the disease by anti
GBM antibody formation, immune complex mediated damage or pauci immune (ANCA
associated) conditions. Thus, option “b” is the preferred answer for the given question.

Important info:
The histologic picture in RPGN is dominated by distinctive crescents. Crescents are formed by proliferation of parietal
cells, deposition of fibrin and by migration of monocytes and macrophages into the urinary space.

2. Loss of foot process on electron microscopy is classical in:


(NEET 2020 like pattern)
(a) Membranous nephropathy
(b) Minimal change disease
(c) IgA nephropathy
(d) Rapidly progressive glomerulonephritis
Ans. (b) Minimal change disease
(Ref: Robbins 9th / 917)
• In patients with minimal change disease, the glomeruli are normal by light microscopy.
• By electron microscopy the glomerular basement membrane appears normal, and no electron dense material is
deposited. The principal lesion is in the visceral epithelial cells, which shows a uniform and diffuse effacement of
foot processes.

3. Bellini duct cancer is seen in which of the following?(NEET 2019 like pattern)
(a) Liver
(b) Kidney
(c) Heart
(d) Spleen
Ans. (b) Kidney
(Ref: Robbins 9th e/p 954 )
Bellini duct carcinoma represents approximately 1% or less of renal epithelial neoplasms. They
arise from collecting duct cells in the medulla.
4. Immunofluorescene staining pattern from a kidney biopsy from a 35-year-old patient
presenting with proteinuria has been shown below. What is the most probable cause?
(AIIMS Nov 2018 like pattern)

(a) FSGS
(b) PSGN
(c) Lupus Nephritis
(d) Goodpasture syndrome
Ans. (c) Lupus Nephritis
(Ref: Robbins 9th e/p 222)
Let us first revise the fluorescent microscopy finding associated with the different glomerular
disorders in the options:
• Goodpasture syndrome: Linear IgG and C3; fibrin in crescents
• FSGS: Focal; IgM and C3 in many cases
• PSGN: Granular IgG and C3 in GBM and mesangium; Granular IgA in some cases
The given images show positive immunostaining for all antibodies including IgG, IgA, IgM and C3.
This is likely to be lupus nephritis as six patterns of glomerular disease are seen in SLE
namely:
• Minimal mesangial lupus nephritis (class I): least common variant
• Mesangial proliferative lupus nephritis (class II)
• Focal lupus nephritis (class III)
• Diffuse lupus nephritis (class IV) is the most common and severe form
• Membranous lupus nephritis (class V)
• Advanced sclerosing lupus nephritis (class VI) is characterized by sclerosis of more than 90% of the glomeruli,
and represents end-stage renal disease.

5. A 43-year-old male presented with facial puffiness and a history of frothy urine for 4 days.
Acute kidney injury is suspected. A renal biopsy was done and Direct
immunofluorescence and electron microscopic image is as shown below. What is the
likely diagnosis?
(AIIMS May 2017 Pattern)
(a) Membranous glomerulopathy
(b) Membranoproliferative glomerulopathy
(c) Minimal change disease
(d) Focal segmental glomerulosclerosis
Ans (a) Membranous glomerulopathy
(Ref: Robbins 9/e p915-6)
Clinical picture is suggestive of nephrotic syndrome. Biopsy is suggestive of the presence of
subepithelial electron dense deposits with effacement of foot processes overlying deposits
on the electron microscopy. The direct immunofluorescence shows diffuse granular deposits
along the glomerular basement membrane.
Both the above findings are associated with the diagnosis of membranous glomerulopathy.
6. An elderly male patient presented with blurring of vision. Fundus examination revealed
cotton wool spots on retina and systemic examination showed decreased peripheral
sensations and increased urine output. What finding is the following renal biopsy
showing?
(AIIMS May 2017 Pattern)

(a) Kimmelstiel-Wilson lesion


(b) Amyloid deposits
(c) Crescents
(d) Hyaline atherosclerosis
Ans (a) Kimmelstiel-Wilson lesion
(Ref: Robbins 9/e p1118)
The clinical picture (cotton wool spots on fundus, decreased peripheral sensations and increased
urine output) is suggestive of diabetes mellitus. The image depicted in the question is
suggestive of the most characteristic feature of diabetes which is nodular
glomerulosclerosis also known as Kimmelstiel-Wilson lesion.
Disclaimer
Any resemblance to an actual question is purely coincidental.
• Length of esophagus: 25 cm and it begins from lower
border of cricoid cartilage.
• Tear in Mallory Weiss syndrome is located at the gastro-
esophageal junction.
• Pathogenesis of Achalasia cardia: Neurogenic
degeneration (causing absence of nerves).
Pseudoachalasia is seen in cancer of the lower
esophageal sphincter.
• Most common type of eosphageal carcinoma is
squamous cell carcinoma and it affects the middle 1/3rd of
the esophagus.
• Type A gastritis (autoimmune gastritis) affects body of the
stomach.
• Type B gastritis (bacterial cause; H. pylori). affects the
antrum.
• Most common location of duodenal ulcer: Anterior wall of
1st part of duodenum.
• Most common type of gastric polyp: Hyperplastic
(inflammatory) polyp.
• Gastric polyp with no malignant potential: Hyperplastic
polyp.
• Most important risk factor for gastric carcinoma is
intestinal metaplasia and its most common location is
antrum.
• Type of gastric carcinoma with best prognosis:
Superficial spreading.
• Gastric lymphoma is a B-cell non Hodgkin lymphoma
called MALToma associated with H. pylori infection. It has
lymphoepithelial lesion.
• GIST arises from Interstitial cell of Cajal having the most
specific marker as DOG > c-Kit (CD117).
• Paneth cells contain zinc and provide mucosal immunity.
• Cereales to be avoided in Celiac sprue: Wheat, oat,
barley, rye (rice and maize are safe).
• Most important association of celiac sprue (gluten
sensitive enteropathy): HLA-DQ2 followed by HLA-DQ8.
• Maximum malignant potential: Familial adenomatous
polyposis.
• Intestinal polyps with no malignant potential: Hyperplastic
polyp, Juvenile polyp, Peutz Jegher syndrome.
• Protective against colorectal cancers: Vitamin A, C & E;
omega-3 fatty acids (fish fat), aspirin, high fibre diet, folic
acid, calcium, selenium.
• Most common location of colorectal cancer: Rectum
followed by sigmoid colon. The ascending or right
colon cancer presents as a fungating/ ulcerative mass
whereas descending/left colon cancer presents as a
napkin-ring constriction.
• Single most important prognostic factor for colorectal
cancer: Extent of tumor (stage).
• Most common intestinal lymphoma: Diffuse large B-cell
NHL.
• Allgrove (triple A) syndrome, an autosomal recessive
disorder characterized by achalasia, alacrima, and
adrenocorticotrophic hormone-resistant adrenal
insufficiency.
• The most common cause of gastroesophageal reflux is
transient lower esophageal sphincter relaxation.
• Reflux of gastric contents into the lower esophagus is
the most frequent cause of esophagitis.
• The malabsorptive diarrhea of Whipple disease is due to
impaired lymphatic transport.
• Gastritis is a mucosal inflammatory process. When
neutrophils are present, the lesion is referred to as acute
gastritis. When inflammatory cells are rare or absent, it is
termed as gastropathy.
• The most common cause of chronic gastritis is
infection with the bacillus H. pylori.
• Autoimmune gastritis is the most common cause of
diffuse atrophic gastritis. It is the most common form of
chronic gastritis in patients without H. pylori
infection.
• Carney triad, a nonhereditary syndrome of unknown
etiology is seen primarily in young females and includes
gastric GIST, paraganglioma, and pulmonary
chondroma.
• Carney-Stratakis syndrome (different from Carney triad)
is caused by loss of succinate dehydrogenase complex
(SDH) function. It increases the risk of GIST and
paraganglioma.
• Celiac disease is associated with a higher incidence of
other autoimmune diseases like including type 1
diabetes, thyroiditis, and Sjögren syndrome, IgA
nephropathy, and neurologic disorders, such as ataxia,
autism, depression, epilepsy, Down syndrome, and
Turner syndrome.
• Individuals with celiac disease have a higher than normal
rate of malignancy. The most common celiac disease-
associated cancer is enteropathy-associated T-cell
lymphoma. The patients also have increased risk of
small intestinal adenocarcinoma.

The normal layers present in the gastrointestinal tract are:


1. Mucosa consisting of epithelial layer, lamina propria and
muscularis mucosae
2. Submucosa having submucosal glands and Meissner’s plexus
3. Muscularis propria consisting of inner circular layer, outer
longitudinal layer and having Auerbach’s plexus in between
these two layers.
4. Serosa.

ESOPHAGUS

It is a muscular tube almost 25 cm in length in adults (it is about


10 cm in a newborn) taking the food from the oral cavity into the
stomach. The esophagus is having the following four constrictions
in it:
• Cricopharyngeus constriction: present at 15 cm (6 inches) from the
incisor teethQ.
• Aortic arch constriction: present at 22.5 cm (9 inches) from the incisor
teethQ.
• Left bronchus constriction: present at 27.5 cm (11 inches) from the
incisor teeth.Q
• Diaphragmatic and lower esophageal sphincter constriction: present at
40 cm (16 inches) from the incisor teethQ.

There is presence of a functional sphincter at the lower end of


the esophagus (LES) which prevents the reflux of the gastric
contents back into the esophagus.

ACHALASIA CARDIA

It is a disease characterized by loss of ganglion cells in the


Auerbach’s plexus the cause of which may be unknown (Primary
achalasia) or it may be due to secondary cause like Chagas’
disease (caused by T. cruzi) or Varicella zoster infection. This
result in the incomplete relaxation of the LES and its increased
resting tone. There is selective loss of function of inhibitory
neurons like those secreting vasoactive intestinal peptide and nitric
oxide whereas cholinergic innervation is intact. It is characterized
by the triad of incomplete LES relaxation, increased LES tone and
aperistalsis of the esophagus.

Clinical features include progressive dysphagia (difficulty in


swallowing increases with time as the disease progresses) for food
though usually dysphagia is more for the liquid food as compared
to solid food particles.

Screening test
CholecystokininQ (CCK) test: CCK normally causes a fall in the sphincter
pressure (because of the relaxant effect of inhibitory neurotransmitters like VIP
and nitric oxide) but in achalasia cardia it causes paradoxical increase in LES
tone (loss of inhibitory neurons).
Diagnosis
• Barium swallow shows ‘bird beak’Q or ‘rat tail’Q appearance of the
esophagus (due to normal upper esophagus with tapering in the lower
part).
• Manometry is the most confirmatory investigation Q.
Treatment
It is medically managed with botulinum toxin but the treatment of choice is
surgical excision of the muscle of the lower esophagus and cardia (Heller
myotomyQ).
HIATAL HERNIA

It is characterized by the separation of the diaphragmatic crura and


increased space between the muscular crura and the esophageal
wall. It can be of two types:
1. Sliding hernia (95%): Characterized by upward dislocation of
cardioesophageal junction. Esophagitis resulting from the
reflux is commonly seen.
2. Paraesophageal/Rolling hernia (5%): A part of the stomach
enters the thorax without any displacement of the
cardioesophageal junction. Dysphagia is common and chest
pain may also be present (usually relieved by a loud belch).
Treatment is achieved only with surgical repair of the defect.

MALLORY-WEISS TEARS

Mallory-Weiss tears are mucosal tears in the esophagogastric


junction or the gastric cardiac mucosa caused due to vigorous
vomiting usually seen in alcoholics. In most of the cases (90%), the
tear is present immediately below the squamocolumnar junction
at the cardiaQ whereas in 10% cases, it is present in the
esophagus. These tears never involve the muscular layer of the
esophagus whereas, in contrast, in Boerhaave syndrome, rupture
of all the esophageal layers is seen including the muscle layer.
Most common location of the perforation in this syndrome is in left
posterolateral part 3-5 cm above the gastroesophageal junction.

ESOPHAGITIS

Inflammation of the esophageal mucosa is known as esophagitis


and reflux of the gastric contents into the lower esophagus is its
most important cause. Gold standard for the diagnosis of reflux
esophagitis is 24 hours pH study Q. The reflux is associated with
obesity, alcohol intake, smoking, pregnancy and overeating.

BARRETT’S ESOPHAGUS

Barrett’s esophagus is the metaplastic change in the esophageal


lining in which the normal squamous epithelium is changed to
columnar epithelium due to prolonged gastroesophageal reflux
disease (GERD). It is classified as long segment (if >3 cm is
involved) or short segment (if <3 cm is involved). Microscopically,
esophageal squamous epithelium is replaced by columnar
epithelium. Definite diagnosis is made only when columnar
mucosa contains the intestinal goblet cellsQ.

Note: Barrett’s ulcer is the ulcer in the columnar lined portion of Barrett’s
esophagus.
Fig. 1: Barret esopahgus with goblet cells (G) of columnar
epithelium. ...(AIIMS Image)

CARCINOMA OF THE ESOPHAGUS

It is a cancer affecting individuals of mid to late adulthood which is


of two main types: squamous cell cancer and adenocarcinoma.

Risk factors for squamous cell cancer


• Tobacco and alcohol consumption
• Hot beverages or food
• Longstanding esophagitis
• Achalasia
• Plummer Vinson syndrome (also known as Patterson Kelly
syndrome)
• Ingestion of nitrites in diet
• Nutritional deficiency of vitamins A, vitamin C, riboflavin, zinc,
molybdenum
• Tylosis et palmaris (hyperkeratosis and pitting of palms and soles)
• Longstanding celiac disease
• Other conditions like ectodermal dysplasia and epidermolysis bullosa
• Genetic alterations include amplification of cyclin D1, c-MYC and
Epithelial Growth Factor Receptor (EGFR)
Most of the cancers are well differentiated and the
morphological patterns include:

i. Exophytic protruding lesion in the lumen (60%)


ii. Flat, diffuse infiltrative form spreading in the esophageal wall (15%)
iii. Ulcerative lesion (25%)

Fig. 2: Esophageal squamous cancer (N) with keratin pearl (K). ...
(AIIMS Image)
Risk factors for Adenocarcinoma
• Barrett’s esophagusQ (Most important)
• Tobacco exposure
• Obesity
• Genetic alterations include over expression of p53,
amplification of c-ERB-B2 and nuclear translocation of b-
catenin (biomarkers of disease progression).
Microscopically, most of the cancers are mucin producing
glandular tumors exhibiting intestinal type features. Multiple foci of
dysplastic epithelium are present adjacent to the mucosa.

Clinical features include progressive dysphagia (more for


solids as compared to liquids), weight loss, chest pain and
vomiting. The lymph node metastasis is dependent on the
anatomic site of the primary tumor.
• Cancer in the upper 1/3rd of the esophagus: metastasis to
cervical lymph nodes.
• Cancer in the middle 1/3rd of the esophagus: metastasis to
paratracheal, mediastinal and tracheobronchial lymph nodes.
• Cancer in the lower 1/3rd of the esophagus: metastasis to
gastric and celiac lymph nodes.

Treatment: It is mainly surgical with partial or total esophagectomy.

STOMACH

CELLS AND THEIR SECRETIONS


Parietal (Oxyntic) cells • Secrete acid (from the proton pump, H+-K+
ATPase)
• Secrete intrinsic factor (required for vitamin B12
absorption)
Chief (Zymogenic) • Secrete the proenzymes pepsinogen I and II
cells (activated to pepsin)
Endocrine cells • Secrete gastrin in antrum (by G cells),
histamine in the body
Foveolar cells • Secrete mucin layer over the mucosal cells

GASTRITIS
Gastritis is the inflammation of the gastric mucosa. It can either be
acute gastritis or chronic gastritis.
Risk factors of acute gastritis
• Heavy smoking and alcohol consumption
• Excessive NSAID use (particularly aspirin)
• Uremia
• Ischemia and shock
• Stress (trauma, burns, surgery)
• Others (nasogastric intubation, distal gastrectomy,
systemic infections)

Microscopically
Presence of neutrophils above the basement membrane in direct contact with
the epithelial cells is indicative of active inflammationQ.

Risk factors of chronic gastritis


• Chronic infection with H. pylori
• Autoimmune cause (pernicious anemia)
• Alcohol and smoking
• Radiation
• Antrectomy with gastroenterostomy
• Others (amyloidosis, graft-versus-host disease, uremia,
Crohn’s disease).

MICROSCOPICALLY

Chronic gastritis has the presence of lymphocytes and plasma cells


associated with intestinal metaplasia and mucosal atrophy.
Chronic gastritis can be of the following two types:
1. Associated with H. pylori (in 90% patients)
H. pylori is a gram-negative flagellated bacteria producing
enzymes like phospholipase and urease, adhesion molecules
like BabA (responsible for enhanced binding in people having
blood group O) and toxins like CagA and VacA. It causes
gastritis in two patterns:
a. Antral predominant gastritis: Seen in individuals having lower IL-
1β production and associated with high acid production and
increased risk of duodenal ulcer.
b. Pangastritis followed by multifocal atrophic gastritis—Seen in
individuals having higher IL-1β production and lower gastric acid
production and increased risk of adenocarcinoma.

Intraepithelial neutrophils and subepithelial plasma cells


(meaning plasma cells in the lamina propria) are characteristic
of H. pylori gastritis. H. pylori is also associated with peptic
ulcer disease, gastric cancer and gastric mucosa associated
lymphoma Q.
Fig. 3: Chronic gastritis with H. pylori (H).

2. Autoimmune gastritis (in 10% patients)


It is caused due to formation of autoantibodies against the proton
pump, gastrin receptor and intrinsic factor and is associated
with pernicious anemia, Hashimoto’s thyroiditis, Addison’s
disease, type 1 diabetes, gastric cancer and carcinoid tumor.
This is particularly associated with damage to the mucosa of
the body and fundus with less involvement of the antrum.
Hyperplasia of gastrin producing G cells in the antral mucosa
may result in gastric carcinoid tumor formation.
The histologic features of chronic gastritis include
regenerative change, intestinal metaplasia (columnar absorptive
cells and goblet cells of intestinal type), atrophy and dysplasia. In
autoimmune gastritis, there is presence of inflammatory infiltrate
having lymphocytes, macrophages in the deeper layers. Plasma
cells in the superficial lamina propria are characteristically absent.
Clinical features include nausea, vomiting and epigastric
pain. Autoimmune gastritis maybe associated with symptoms seen
in pernicious anemia (beefy tongue, paresthesia, numbness,
sensory ataxia, loss of vibration and position sense).

PEPTIC ULCER DISEASE

Any breach in the mucosa of the GIT that involves the submucosa
or deeper due to exposure to gastric acid is called peptic ulcer. It is
usually a chronic and solitary lesion less than 4 cm caused due
to imbalance between gastroduodenal protective and damaging
factors:
Damaging factors Protective factors
• Gastric acid • Mucus and bicarbonate secretion
• Pepsin • Mucosal blood flow
• Smoking, alcohol • Prostaglandin production
• Drugs like NSAIDs • Epithelial regenerative capacity
• H. pylori
• Ischemia and shock
• Delayed gastric emptying
• Duodenal gastric reflux or gastric
hyperacidity

The location of the peptic ulcer (in decreasing order of


frequency) is:

• Duodenum (first part)Q


• Stomach (lesser curvature near the junction of body and
antrumQ)
• Gastroesophageal junction in GERD or Barrett’s esophagus
• Margins of jejunostomy
• Stomach, duodenum and/or jejunum of patients with Zollinger-Ellison
syndrome
• In ileal Meckel’s diverticulum containing ectopic gastric mucosa.
Males are more commonly affected than females
Duodenal ulcers are located near the pyloric ring and gastric ulcers
are predominantly located near the lesser curvature and the
antrum. More commonly, there is involvement of the anterior wall of
the duodenum as compared to the posterior wall. Benign peptic
ulcer is classically punched with margins of the ulcer usually at
level with the surrounding mucosa whereas heaping up of the
margins is more frequently associated with malignancy.
Histologically the zones in peptic ulcer are:
i. Base and margins having necrotic fibrinoid debris
ii. Zone of neutrophil predominant infiltrate
iii. Base having active granulation tissue with mononuclear cells
iv. Zone of fibrous or collagenous scar
Clinical features include burning epigastric pain (usually
getting worse at night), nausea, vomiting and bloating.

Complications of Peptic Ulcer

Bleeding

• Most frequent complicationQ


• More common in posterior wall duodenal ulcersQ
Perforation

• Most common cause of death in peptic ulcerQ


• More common in anterior wall duodenal ulcersQ
Gastric outlet obstruction (GOO)
• Results in persistent vomiting leading to fluid and electrolyte imbalance
(metabolic alkalosis due to loss of acid in vomitus)
Malignancy
• Associated with gastric ulcer but never with duodenal ulcer.
Duodenal and Gastric peptic ulcer
Features Duodenal ulcer Gastric ulcer
Site 1st part of duodenum Along lesser curvature
Incidence More common Less common
Age 25 – 50 yrs, M>F Beyond 6th decade, M>F
Etiology Almost all patients have Less stronger association
H. pylori infection
Acid level High Usually normal; ↑ if
hypergastrinemia
Pain after food intake Relieved Aggravated
Clinical features Night pain and melena No night pain,
more common hematemesis more
common
No vomiting/no weight Vomiting common/weight
loss loss is present
Complications No malignant change Malignant change present
(though rarely)

Investigations

• Screening test: Serum ELISA for antibodies against H. pylori


• Urea breath test (radiolabeled urea is broken down to radiolabeled
CO2 by urease enzyme which is detected by breath analyzer, thus
suggesting presence of H. pylori infection)
• Gold standard: Staining of H. pylori with silver stain or Warthin
starry stainQ
• Most specific investigation: Culture of bacteriaQ (done on
Skirrow’s medium)

Treatment: with “triple drug therapy”Q (combination of


lansoprazole, clarithromycin and metronidazole) for 2 weeks.
Fig. 4: Menetrier disease with foveolar cell hyperplasia (F).

GASTRIC CANCER
Gastric cancer is the most common gastric malignancy. The risk
factors for this cancer are:
Environmental factors Genetic factors Host factors
• H. pylori infection • Family history of • Chronic gastritis
• Nitrites in diet gastric cancer • Intestinal metaplasia
• Nutritional (vitamins • Blood group A • Partial gastrectomy
C, E) deficiency • Hereditary • Gastric adenoma
• Smoking nonpolyosis colon • Barrett’s esophagus
cancer syndrome • Menetrier disease
(HNPCC)
• Familial gastric
cancer syndrome
(due to E-cadherin
mutation)

Classification of gastric cancer


1. Based on Lauren’s histological classification
i. Intestinal type: This is localized type of cancer
composed of the neoplastic intestinal glands which
exhibit an expanding sheet pattern of spread due to
cohesion of the cells.
ii. Diffuse type: Poorly differentiated non-cohesive cells
which do not form glands. The appearance of the cells
is “signet ring” appearance Q (because mucin in the
cell pushes the nucleus to the periphery). It is seen
more frequently with E-cadherin mutation.

2. Based on Depth of invasion


i. Early gastric cancer: Characterized by the
involvement of mucosa and the submucosa
irrespective of the involvement of perigastric lymph
nodes and is associated with better prognosis.
ii. Late gastric cancer: Characterized by the involvement
of the muscle layerQ of the stomach and is
associated with poor prognosis.
3. Based on Macroscopic pattern
i. Protruding mass or exophytic (type I lesion)
ii. Flat or depressed: No obvious mass in the mucosa
(type II lesion)
iii. Excavated: Erosion is present in stomach wall (type III
lesion).

Clinical Features
The most common location of the gastric cancer is the antrum of
the stomachQ
Symptoms include postprandial heaviness in the abdomen
(earliest symptom), weight loss (most common symptom), vomiting
and anorexia.

Investigation of choice: Endoscopy with biopsy and brush cytologyQ.

Metastasis occurs to the liver (first organ to be affected)


followed by lungs, bone, ovary (where it is known as
Krukenberg’s tumor), periumbilical lymph nodes (Sister Mary
Joseph nodule), peritoneal cul-de-sac (Blumer’s shelf palpable on
rectal or vaginal examination) and left supraclavicular lymph node
(Virchow’s lymph node Q).
Treatment: Surgical resection is the only curative treatment in
gastric cancer. Chemotherapy may be given in advanced cancers
with ECF regime (Epirubacin, Cisplatin and 5-Fluorouracil).

GASTROINTESTINAL STROMAL TUMOR (GIST)

• Arise from pacemakers of the GIT known as interstitial cells of


CajalQ.
• GIST is the most common mesenchymal tumorQ of the
abdomen
• The most common location of the GIST is the stomachQ
• This is associated with patients having neurofibromatosis-1Q.
• Microscopically the tumor may show either epithelioid cells,
spindle cells or mixed (both the epithelioid cells and spindle
cells).
• The useful diagnostic markers are DOG (detected on GIST)
and c-kit (CD117)Q.
• It is best diagnosed withCT scan with PET scanQ (preferable)
or CT scan.
• Treatmentis done with surgical resection (localized tumors)
and non-excisable tumors are managed with tyrosine kinase
inhibitors called imatinib mesylateQ or sunitinib.

INTESTINE

Infectious Diseases

The important causes of infections in the intestine are as follows:


I. Enteric fever (typhoid)
It is caused because of infection with Salmonella species usually
affecting the ileum and the colon.
– It is associated with ulceration of the Peyer’s patches in
the terminal ileum and presence of longitudinal ulcersQ
(oval ulcers with long axis along the long axis of the
ileum). Microscopic examination reveals the presence of
macrophages having bacteria and red blood cells
(erythrophagocytosisQ)
– In the liver, the hepatocytes are replaced by an
aggregation of macrophages called as “typhoid nodule”Q.
Involvement of gallbladder results in development of
chronic carrier state. Healing in ulcer is uncommonly
associated with fibrosis or stricture formation.
– Clinically, the patient develops step-ladder pyrexia, rose
spots (erythematous macular lesions on chest and
abdomen), abdominal pain, vomiting and diarrhea.
Salmonella osteomyelitis is particularly common in
patients having sickle cell disease.
– Complications include hemorrhage and perforation.
– Blood culture is the mainstay of diagnosis and Widal test
is use for measuring the antibody titer.
– Drug of choice for the treatment is
ciprofloxacin/ceftriaxone and for carriers, it is ampicillin
+ probenecid.

II. Tuberculosis: It can present itself in two of the following forms:


Primary infection

• Caused by infection due to Mycobacterium bovis (due to intake of


infected/non pasteurised milk) and results in the development of
hyperplastic tuberculosis. The infection is present in the lymphoid
follicles of the intestine and associated with thickening and
narrowing of the lumen of the intestine. It usually affects the ileocecal
region and is associated with subacute intestinal obstruction. The
mesenteric lymph nodes are enlarged; matted and caseous. This is
known as tabes mesenterica.
• Clinical features of the patient include acute abdominal pain and
intermittent diarrhea. Investigations show widening of the ileocecal
angle (known as “pulled up cecum”) on barium radiography.

Secondary infection

• Caused by Mycobacterium tuberculosis secondary to swallowing of


infected sputum in a patient of pulmonary tuberculosis. It is
characterized by presence of the transverse ulcers in the intestine
particularly the ileum.
• Clinical features of the patient include weight loss and intermittent
diarrhea. Investigations show “filling defect in the ileum, cecum and
ascending colon” on barium radiography. Complications include
perforation of the intestine and fistula formation.

Treatment is by administration of antitubercular therapy


(conservative management) or surgical resection of the
affected part of the intestine (in case of obstruction or fistula
formation).
III. Amoebiasis
It is caused by infection with an anaerobic protozoa E.
histolytica and results in the development of flask shaped
ulcersQ (ulcer with a broad base but narrow neck). The
disease affects the cecum and ascending colon followed by
sigmoid colon, rectum and appendix. The ulcers usually
involve the mucosa and the submucosa (not the muscle layer)
and have the presence of liquefactive necrosis. Liver is
another important organ affected by the disease resulting in
the development of hepatic abscess having necrotic material
and hemorrhage (called as “anchovy sauce pus”Q). The
invasive disease is diagnosed with ELISA.

Microbiology link!
• Rotavirus is the most common cause of diarrhea in children of age 6-24
months.
• Giardia lamblia is the most common pathogenic parasitic infection in the
humans.
• Cholera is caused by Vibrio cholerae resulting in the passage of “rice
water” stools.

PSEUDOMEMBRANOUS COLITIS (ANTIBIOTIC


ASSOCIATED PSEUDOMEMBRANOUS COLITIS

It is a condition most commonly caused by Clostridium difficile


and is characterized by inflammation in the colon associated with
the formation of a pseudomembrane (layer of inflammatory cells
and necrotic material overlying the sites of mucosal injury). In
normal individuals, the normal flora of the gut is responsible for the
production of chemicals called bacteriocins in the intestine. On
administration of broad spectrum antibiotics (most commonly, IIIrd
generation cephalosporins), the normal bacteria are destroyed
resulting in increased proliferation of the Clostridium bacteria which
then produces large amounts of two toxins, toxin A and toxin B.
These toxins induce cytokine production and host cell apoptosis
resulting in diarrhea. The disease is diagnosed with the
demonstration of C. difficile cytotoxin in stool.

Malabsorption Syndromes

Defective absorption of fats, vitamins, proteins, carbohydrates and


fats is called malabsorption. Its hallmark feature is steatorrhea and
chronic diarrhea is the most common clinical presentation. The
important causes are as follows:
1. Celiac Disease (Celiac Sprue or Gluten Sensitive
Enteropathy or Non-troical sprue)
It is a disease characterized by increased sensitivity to a protein
called gluten or its alcohol soluble fraction α-gliadinQ present
in the grains like wheat, oat, barley and rye resulting in a T-cell
mediated chronic inflammatory reaction in the small intestine
and impaired absorption. It is associated with HLA-DQ2 or
HLA-DQ8. Clinical features include diarrhea, flatulence, weight
loss and fatigue and a characteristic skin lesion called
‘dermatitis herpetiformis’.

Biopsy of the intestine shows the diffuse enteritis (lymphocytes and


plasma cells in lamina propria) with marked atrophy of the villi
and elongated and hyperplastic crypts (overall mucosal
thickness is unalteredQ). The disease also demonstrates the
presence of antigliadin, antiendomysialQ (most useful) or
antitransglutaminase antibodiesQ (useful for screening test)
whereas definitive diagnosis is made by the following three
features:
Fig. 5: Celiac disease having villous atrophy on surface (S) and
presence of intra epithelial lymphocytes (L). ...(AIIMS Image)
• Clinical documentation of malabsorption
• Demonstration of small intestinal lesion by biopsy
• Improvement in clinical features and mucosal histology on gluten
withdrawal from the diet.

Treatment is intake of the gluten-free diet and substitution by


rice, millet, tapioca, potato and maize in the diet.
2. Tropical Sprue (Post Infectious Sprue) is now called as
Environmental Enteropathy
It is a disease similar in features to celiac sprue but present in the
tropical region. Though the exact cause is unknown, but
bacterial overgrowth particularly E. coli and Hemophilus have
been implicated. Biopsy of the intestine shows the diffuse
enteritis with atrophy of the villi. Treatment is done with the
help of antibiotics.

Note:
• There is characteristically involvement of the proximal intestine in
celiac sprue resulting in iron deficiency anemia whereas in tropical
sprue, there is generalized involvement of the small intestine
(resulting in megaloblastic anemia because B12 and folic acid are
absorbed from the terminal ileum).
• Another important difference between the two is that tropical sprue
is not associated with cancer development whereas celiac
sprue is associated with cancers like non Hodgkin’s lymphoma,
small intestine adenocarcinoma and esophageal squamous cell
cancer.

3. Whipple’s Disease
– It is a systemic infectious disease caused by an
actinomycete, Tropheryma whippelii affecting the triad of
small intestine, CNS and jointsQ. The bacteria
characteristically proliferate inside the macrophages
without getting destroyed.
– The hallmark feature of the disease is small intestinal
mucosa having macrophages in the lamina propria and
these macrophages show the presence of PAS positive,
diastase resistant granulesQ and rod shaped bacteria on
electron microscopy. There is mucosal edema, dilation of
the lymphatics and involvement of mesenteric lymph
nodes. The macrophages having the bacteria can also be
found in the joints, brain, cardiac valves etc with absence
of inflammation being a typical feature.
– Clinical features include arthropathy (initial presentation),
diarrhea, weight loss, hyperpigmentation and dementia.
The diarrhea is due to impaired lymphatic transport.
– Diagnosis is confirmed by identification of T. whipplei by
polymerase chain reaction (PCR). Treatment is done with
cotrimoxazole (drug of choice) for one year.
Fig. 6: Whipple’s Disease: Several broad villi (V), abundant foamy
macrophages (M).

Notes:
• Hallmark of Whipple’s disease had been presence of PAS positive
macrophages containing the characteristic small bacilli. But, similar
picture (PAS +ve macrophages with bacilli) can also be seen with
M. avium complex (cause of diarrhea in AIDS). However, these
organisms are acid fast whereas Tropheryma is not.
• The organs in which these foamy macrophages can be seen are
Liver, Small intestine, Lymph nodes, Heart, Eyes, CNS and synovial
membranes of joints.

Disorders Associated with Abnormalities in Small-Bowel


Biopsy Specimens
Biopsy has diagnostic value (Diffuse Lesions)
Whipple’s disease: Lamina propria infiltrated with
macrophages containing PAS-positive
glycoproteins
Abetalipoproteinemia: Villus structure normal; epithelial cells
vacuolated due to excess fat
Agammaglobulinemia: Flattened or absent villi; increased
lymphocytic infiltration; absence of
plasma cells
Mycobacterium avium complex
Biopsy may have diagnostic value (Patchy lesions)
Intestinal lymphoma: Infiltration of lamina propria and
submucosa with malignant cells
Intestinal lymphangiectasia: Dilated lacteals and lymphatics in
lamina propria; clubbed villi
Eosinophilic enteritis: Diffuse or patchy eosinophilic infiltration
in lamina propria and mucosa
Amyloidosis: Presence of amyloid confirmed by
special stains
Regional enteritis: Noncaseating granulomas
Parasitic infestations: Parasitic invasion of mucosa;
adherence of trophozoites to mucosal
surface, as in Giardiasis
Systemic mastocytosis: Mast cell infiltration of lamina propria
Biopsy is abnormal but not diagnostic
Celiac sprue Collagenous sprue Tropical
sprue
Folate deficiency Vitamin B12 deficiency Acute
radiation enteritis
Systemic scleroderma Bacterial overgrowth syndromes

Inflammatory Bowel Disease (IBD)

It is a group of chronic inflammatory conditions as a result of


unregulated and persistent activation of the immune system in
genetically susceptible persons. It is primarily of two types: Crohn’s
disease and ulcerative colitis.

CROHN’S DISEASE
It is a chronic granulomatous disease which can affect any part of
the gut from the esophagus to the large intestine but the most
commonly affected part is small intestine particularly the ileum. So,
it is also called as “terminal ileitis” or “granulomatous colitis”Q. It is
associated with HLA-DR1/DQw5 and NOD2 genes and an
abnormal T-cell response particularly, CD4+ T cells (TH1 cellsQ).

MORPHOLOGY
• The earliest lesion in Crohn’s disease is the aphthous ulcer.
Many such ulcers may fuse together to from serpentine ulcers
arranged longitudinally.
• Grossly, involved bowel segment typically has a rigid, strictured
or thickened wall with creeping fatQ.
• Full thickness of the intestine is affected in the disease i.e.
there is transmural inflammationQ. This causes weakness in
the wall thereby leading to fissure and fistula formation in
Crohn’s disease. Fibrosis is also commoner in this type of IBD.
Perianal fistula is the most common fistula seen.
• There is patchy involvement of the intestine which is known as
presence of “skip lesions”. The intervening area between two
affected portions is absolutely normal. So, the mucosa appears
to be irregular which is known as “cobblestone mucosa”Q
Contd...
Contd...
• There is a presence of non-caseatingQ granulomas.
• Clinical features are intermittent attacks of abdominal pain,
blood in stools, fever, steatorrhea and megaloblastic anemia
(the last two features result because there is impairment in the
absorption of bile acids and vitamin B12 respectively from the
ileum).
• Screening test is presence of ASCA (Anti-Saccharomyces
cerevisae AntibodyQ). Antibody formation is common against
cell wall of yeast, Saccharomyces cerevisae in patients of
Crohn’s disease. The investigation done in these patients to
confirm the diagnosis is endoscopy and colonoscopy so that
direct visualization of the lesions can be done and even a
biopsy can be taken if needed.

Important features of Crohn’s disease

S – Skip lesions
I – Ileum (MC affected site)
S – Saccharomyces cerevisae antibody present
T – Transmural involvement
E – Extra fibrosis and fistula formation (as compared to ulcerative
colitis)
R – Radiological sign- String sign of Kantor, Rectum is usually spared.
ULCERATIVE COLITIS
It is a chronic inflammatory condition affecting the colon. It most
commonly starts from the rectum and affects the superficial layers,
the mucosa and the submucosaQ (muscularis propria is rarely
affected). It is associated with HLA-DR2, polymorphism in IL-10
gene and an abnormal T-cell response particularly of CD4+ T cells
(TH2 cellsQ).

MORPHOLOGY
• The disease involves the entire colon (pancolitis)Q starting
from the rectum (retrograde involvement). There is presence of
regenerating mucosa which projects in the lumen and is called
“pseudopolyps”Q
• In extreme cases, there is involvement of the nerve plexus in
the muscularis layer resulting in decrease in the motility of the
colon and increase in its size over a period of time giving rise
to “toxic megacolon”Q
• The characteristic feature of the disease is mucosal damage
continuously from the rectum and extending proximally. This
may also lead to “backwash ileitis”. This type of IBD is more
commonly associated with progression to the development of
cancer.
• There is absence of granulomasQ.
• Clinical features are intermittent attacks of abdominal pain,
bloody mucoid stools and fever.
• There is presence of p-ANCAQ (perinuclear antineutrophil
cytoplasmic antibodies).
Important features of ulcerative colitis
Ulcerative – Ulcers in mucosa and submucosa (Muscle layer not
effected)
C – Continuous retrograde involvement (No skip lesions)
O – Originates in the rectum
L – Lead pipe appearance
I – Increased chances of cancer (More than that in Crohn’s
disease)
T – Toxic megacolon (Due to involvement of transverse
colon)
I – Increased growth from the mucosa (“Pseudopolyps”)
S – Symptoms are severe (As compared to Crohn’s disease)

• The extraintestinal manifestations in the IBD are uveitis, iritis,


ankylosing spondylitis, clubbing, migratory polyarthritis,
sacroilitis, primary sclerosing cholangitis, pyoderma
gangrenosum, erythema nodosum etc.

• The disease is treated with sulfasalazine (5-aminosalicylic acid is the


principal therapeutic moiety), infliximab (TNF-α antagonist) and
steroids.

• IBD is a precancerous condition and can increase the risk of


development of cancer of the colon.

CARCINOID TUMOR

Carcinoid tumor arises from the endocrine cells called as


argentaffin tissue (also called as Kulchitzsky cells of crypts of
Lieberkuhn) with the GIT and the lungs as the main sites of origin
of this cancer. The clinical features are due to release of peptide
and non-peptide hormones from these cells. The gastrointestinal
carcinoid tumors can be of the following types:
1. Foregut carcinoid tumors: Arise from the esophagus, stomach
and the duodenum proximal to the ligament of Treitz, these are
usually benign.
2. Midgut carcinoid tumors: Arise from the jejunum and ileum;
these are aggressive and metastasize frequently.
3. Hindgut carcinoid tumors: Arise from the appendix, colon and
rectum; usually benign.

Morphology
On section, the tumors show a characteristic solid, yellow tan
appearance and on electron microscopy, the tumor cells show
dense core granules in the cytoplasm which stain positively with
chromogranin A, neuron-specific enolase and synaptophysin Q on
immunocytochemistry.
Carcinoid syndrome is present in 1% patients of carcinoid
tumor and it is due to excessive release of serotonin (5-HT)Q It is
strongly associated with metastatic disease.
Cardiac lesions are present in 50% of the patients with the
carcinoid syndrome. They consist of fibrous intimal thickenings
on the inside surfaces of the cardiac chambers and valvular
leaflets. And are located mainly in the right ventricle, tricuspid
and pulmonic valves, and occasionally in the major blood
vessels. The commonest cardiac manifestation is the tricuspid
regurgitation (tricuspid stenosis is relatively uncommon) followed
by pulmonary regurgitation.

Hepatic metastasis is usually present in this tumor. The most


sensitive screening test for small intestine carcinoids is the
plasma level of chromogranin A. The levels of 5-HT and its
metabolite 5-hydroxyindoleacetic acidQ (5-HIAA) is elevated in the
urine.

POLYPS
Polyps are seen most commonly in colon but can also be seen in
other parts of GIT.
Polyps
Non-inflammatory Neoplastic polyps
• Inflammatory polyp • Adenomatous polyp
• Hyperplastic polyps – Tubular adenoma (most
• Hamartomatous polyps: they of 2 common)
types – Villous adenomas (maximum
– Juvenile polyp (MC in rectum) malignant potential)
– Peutz-jegher polyp (MC in – Tubulovillous adenoma
jejunum)
Adenomatous polyps are usually asymptomatic.

Also Know
Hamartomatous polyps can occur sporadically or as a part of syndromes such
as Juvenile polyposis, Peutz-Jegher syndrome, Cowden syndrome and
Cronkhite-Canada syndrome. All these syndromes have autosomal dominant
inheritance except Cronkhite-Canada syndrome, which is a non-
hereditary disorder.

CARCINOMA OF THE COLON

The cancer of the colon is seen frequently in old age (peak age 60-
79 years). It is an adenocarcinoma in almost all the patients. The
risk factors for the colon cancer are:

A. GENETIC FACTORS
i. Hereditary Non-polyposis Colon Cancer (HNPCC)
syndrome (also called as Lynch syndromeQ)
It is an autosomal dominant condition characterized by the
increased incidence of colon cancer and extraintestinal cancer
particularly the ovarian and endometrial cancer. The hallmark
is the mutation in the DNA repair genes (MSH2 and MLH1)
leading to microsatellite instability. Colon cancers in these
patients affect right or ascending colon and occurs at
younger age (<50 years). The proximal colon tumors in
HNPCC have a better prognosis than sporadic tumors from
patients of similar age.
ii. Familial Adenomatous Polyposis (FAP)
It is caused by the mutation of adenomatous polyposis coli (APC)
gene present on the long arm of chromosome 5 Q (5q21).
Some FAP patients without APC mutation have a mutation in
the nucleotide base excision repair gene called MUTYH.
Colorectal carcinoma develop in 100% of untreated FAP
patients often before age 30. As a result, prophylactic
colectomy is the standard therapy in patients with APC
mutations.

Subtypes of FAP

Classic FAP syndrome

• The patient has a large number of adenomatous polyps and retinal


pigment epithelial hypertrophy. There should be a minimum of 100 polypsQ
to make a diagnosis of this syndrome. Most of the adenomatous polyps
are tubular polyps.

Attenuated FAP syndrome

• The patient has a lower number of adenomatous polyps (around 30) which
are located in proximal colon.

Gardener syndromeQ

• Intestinal polyps + epidermal cysts + fibromatosis + osteomas (of the


mandible, long bones and skull).

Turcot syndromeQ

• Adenomatous colon polyposis + CNS tumors (medulloblastoma in 2/3rd


and gliomas in 1/3rd patients).

iii. Mutations affecting p53 and K-RAS genes


B. ENVIRONMENTAL FACTORS
Factors increasing risk Factors decreasing risk

• Increased calorie intake and obesityQ • Increased intake of dietary


• Decreased intake of micronutrients fiberQ
• Smoking and alcoholQ • Intake of w-3 fatty acids
• Streptococcus bovis septicemia or (fish)Q
endocarditis • NSAID use (especially
• UreterosigmoidostomyQ Aspirin)Q
• Intake of folic acid and
• Inflammatory bowel diseaseQ
calciumQ
• Acromegaly
• Hormone replacement
• Pelvis irradiation therapy

Molecular pathogenesis
1. APC/b-catenin pathway (also called adenoma-carcinoma
sequence): Loss of tumor suppressor APC gene is followed by
increased b-catenin transcriptional activity (normal APC protein
degrades b-catenin) leading to localized colon epithelial
proliferation and formation of small adenoma. This is followed
by dysplastic change due to activating mutation in K-ras and
inhibition of tumor suppressor genes like SMAD2, SMAD4 and
p53 leads ultimately to cancer.
2. Microsatellite instability pathway: Genetic lesions in 90%
cases involve MSH2 and MLH1 genes which are DNA
mismatch repair genes. These genes correct any genetic
disruption which may arise whenever the colonic cells are
multiplying rapidly. Any mutation in these genes causes
activation of BRAF and inhibition of BAX protein and TGF-b
type II gene thereby increasing the chances of development of
colonic cancer. Kras and p53 are not typically mutated.
Colon cancer exemplifies the concept of multi-step
carcinogenesis
MORPHOLOGY
Most of the cancers arise from the rectumQ followed by the
sigmoid colon. Microscopically, it is an adenocarcinoma and
invasive cancers invoke a strong desmoplastic response. Cancers
in the anorectal region are squamous cell cancers.

CLINICAL FEATURES OF COLORECTAL CARCINOMA


Features Right sided/proximal Left sided/distal
Sites in colon Caecum and ascending colon Descending colon and
sigmoid colon
Gross Fungating/ulcerative type Obstructive type
appearance polypoid carcinoma Large Carcinomatous ulcers
cauliflower-like soft friable have Napkin ring
mass projecting into lumen configuration
Infiltration Absent Present
Clinical features Fatigue, weakness, Iron Occult bleeding change in
deficiency anemia, bleed bowel habits carmpy lower
readily left quadrant discomfort,
Melena, diarrhea,
constipation
Diagnosis Later Early stage (theoretically)
due to symptoms
Prognosis Good Poor
Contd...

Contd...

Metastasis occurs in order of preference, to regional lymph


nodes, liver, lungs and bones.

Diagnosis

• Tumor markers: Colonic cancer is associated with the elevated levels of


tumor markers CEA (carcinoembryonic antigen) and CA 19-9.
• Colonoscopy may also be done which helps in the direct visualization of
the cancer and may also be used to take a biopsy. Gross appearance of
the colon is called as “napkin ring”Q appearance (caused by annular
and constricting lesions in distal colon).

Fig. 7: Signet cell (S) in colon cancer.


Most important prognostic indicator of colon cancer is the
stage which means the extent of tumor at the time of diagnosis.
Duke’s staging was used for colon cancer which has now been
replaced by the TNM staging.

TREATMENT
• Right colon cancer is surgically treated with resection and
ileocolic anastomosis whereas for Left sided colon cancer,
Hartman’s procedure (surgical resection of the affected lesion
and proximal diversion with the help of colostomy) is done.
Multiple Choice Questions
ESOPHAGUS

1. Barrett’s esophagus shows:


(AIIMS May 2010)
(a) Intestinal dysplasia
(b) Intestinal metaplasia
(c) Columnar cell metaplasia
(d) Columnar cell dysplasia
2. Predisposing factor for esophageal cancer is all except:
(a) Mediastinal fibrosis
(AIIMS May 2009)
(b) Diverticula
(c) Caustic alkali burn
(d) HPV
3. Most common cause of esophagitis is:
(a) Smoking
(AIIMS May 2009)
(b) Alcohol
(c) Reflux disease
(d) Increased intake of spices
4. Best site for taking biopsy for viral esophagitis is:
(a) Edge of ulcer
(AIIMS Nov 2001)
(b) Base of ulcer
(c) Adjacent indurated area around ulcer
(d) Surrounding normal mucosa
5. Which of the following viruses does not produce viral
esophagitis?
(Delhi PG 2009)
(a) Herpes
(b) Adenovirus
(c) Varicella
(d) Cytomegalovirus
MOST RECENT QUESTIONS
6. Which of the following is true about Barret’s esophagus?
(a) Squamous to columnar metaplasia
(b) Columnar to squamous metaplasia
(c) Does not increase risk of malignancy
(d) None of the above
7. Barrett esophagus can result from:
(a) H. pylori infection
(b) H. simplex infection
(c) Gastroesophageal reflux
(d) Varices
8. Plummer-Vinson syndrome is characterized by all except:
(a) Glossitis
(b) Esophageal webs
(c) Megaloblastic anemia
(d) Esophageal dysphagia
9. Which of the following is true of Barrett esophagus?
(a) A biopsy will show a histologic finding of columnar to
squamous metaplasia
(b) It is a known precursor of carcinoma of the stomach
(c) The most common location is the proximal (upper) third
of the esophagus
(d) It is a known precursor of adenocarcinoma of the
esophagus
10. A patient complains of pain in the upper portion of his
neck on swallowing. He occasionally regurgitates
undigested food shortly after eating. Which of the
following is the most likely etiology of his problems?
(a) Mallory-Weiss tears
(b) Zenker’s diverticulum
(c) Schatzki rings
(d) Traction diverticula
11. A female with chronic dysphagia undergoes an upper
endoscopy that reveals massive dilation of the distal
esophagus. The esophagus is kinked and tortuous and
partly filled with undigested foods. What is the most
likely diagnosis for this patient?
(a) Achalasia
(b) Barrett’s esophagus
(c) Hiatal hernia
(d) Plummer-Vinson syndrome
12. Which of the following locations is most likely for the
development of carcinoma in a man who has chronically
chewed tobacco?
(a) Floor of the mouth
(b) Lower lip
(c) Tongue
(d) Buccal mucosa
13. Most common anatomical location of tongue cancer is:
(a) Anterior third
(b) Lateral margin
(c) Dorsum
(d) Posterior third
14. All are precancerous for carcinoma of esophagus except:
(a) Achalasia
(b) Paterson-Kelly syndrome
(c) Zenker diverticulum
(d) Ectodermal dysplasia
15. Most common antecedent of erythroplakia and
leukoplakia is which of the following?
(a) Diphtheria
(b) Tobacco use
(c) Alcohol
(d) Poor oral hygiene
16. Lymphoid tissue is seen in which parotid tumor?
(a) Pleomorphic adenoma
(b) Warthins tumor
(c) Adenoid cystic carcinoma
(d) Mucoepidermoid cancer
17. False about pleomorphic adenoma is:
(a) Large in size
(b) Encapsulated
(c) Commonly turns malignant
(d) Slow growing

STOMACH: GASTRITIS, PUD, GIST, GASTRIC CANCER

18. A 50 years old male presents with obstructive symptoms.


Biopsy of stomach reveals the likely diagnosis to be
gastrointestinal stromal tumour (GIST). The most
appropriate marker for this tumor would be which of the
following?
(AIIMS May 2011)
(a) CD34
(b) CD117
(c) CD30
(d) CD10
19. Sister Mary Joseph nodule is most commonly seen in
with which of the following?
(AIIMS May 2010)
(a) Ovarian cancer
(b) Stomach cancer
(c) Colon cancer
(d) Pancreatic cancer
20. Most appropriate marker of GIST:
(a) CD117
(AI 2010, AIIMS Nov 09)
(b) CD 34
(c) CK
(d) Vimentin
21. Which of the following is a specific marker for GIST?
(a) CD 117
(b) CD34
(AI 2009)
(c) CD23
(d) S-100
22. Which one of the following is the most significant risk
factor for development of gastric carcinoma?
(AI 2006)
(a) Paneth cell metaplasia
(b) Pyloric metaplasia
(c) Intestinal metaplasia
(d) Ciliated metaplasia
23. When carcinoma of stomach develops secondarily to
pernicious anemia, it is usually situated in the:
(AI 2006)
(a) Prepyloric region
(b) Pylorus
(c) Body
(d) Fundus
24. Sister Mary Joseph nodules are found in:
(a) Gastric carcinoma
(AIIMS May 2009)
(b) Pancreatic carcinoma
(c) Lung carcinoma
(d) Ovary carcinoma
25. Gastrointestinal stromal malignancy arises from which of
the following?
(AIIMS May 2002)
(a) Smooth muscle
(b) Nerve cells
(c) Interstitial cells of Cajal
(d) Vascular endothelium
26. Histologic examination of the lesion in stomach reveal fat-
laden cells, likely cause is:
(a) Lymphoma
(AIIMS Nov 2001)
(b) Postgastrectomy
(c) Signet-cell carcinoma stomach
(d) Atrophic gastritis
27. The following have strong causal association with H.
pylori infection except:
(Karnataka 2008)
(a) Chronic gastritis
(b) Peptic ulcer disease
(c) Gastric carcinoma
(d) Gastric adenoma
28. In early gastric carcinoma malignancy is confined to:
(a) Mucosa
(Karnataka 2004)
(b) Mucosa and submucosa
(c) Gastric wall without lymph node metastasis
(d) Gastric glands
29. In pernicious anemia, antibody is formed against:
(a) G-cell
(Bihar 2006)
(b) Parietal cell
(c) Stem cell
(d) All
30. A 60 years old fashion photographer and smoker Alok
Nath complaints of severe nausea, vomiting, early
satiety, and a 10 kg weight loss over the past 5 months.
His physical examination reveals the presence of mild
muscle wasting. An upper GI endoscopy reveals the
erosion of entire gastric mucosa. In addition, there is
presence of erythematous cobblestone appearance of
the mucosa. The stomach is also found to be shrunken
and reduced in size. Upper gastrointestinal radiographs
show that the stomach is small and shrunken. Which of
the following is the likely microscopic finding in this
man?
(a) Early gastric carcinoma
(b) Gastrointestinal stromal tumor
(c) Signet ring cell adenocarcinoma
(d) Chronic atrophic gastritis
31. A 56-year-old man with a history of glomerulonephritis is
diagnosed with renal failure. The man subsequently
complains of heartburn and nausea, and gives a history
that he has been vomiting each morning for the last few
days. Which of the following forms of gastritis would
most likely be found in this patient?
(a) Acute gastritis
(b) Chronic antral gastritis
(c) Lymphocytic gastritis
(d) Hypertrophic gastritis
32. Which of the following conditions would mostly likely be
associated with chronic gastritis (Type A) resulting from
autoimmune destruction of parietal cells?
(a) Decreased growth of luminal bacteria
(b) Decreased likelihood of developing gastric carcinoma
(c) Decreased plasma concentration of gastrin
(d) Increased production of macrocytic red blood cells
33. Which of the following sites contains striated muscle that
is not under voluntary control?
(a) Bladder
(b) Colon
(c) Esophagus
(d) Gallbladder
34. An old man being evaluated for abdominal pain and
weight loss undergoes endoscopy showing a broad
region of the gastric wall in which the rugae are
flattened. Biopsy of this area shows infiltration by
numerous polygonal tumor cells with small, dark, round
or ovoid nuclei pushed to the margin of the cell by large,
clear, cytoplasmic structures. These cells might be
expected to have which of the following properties?
(a) Keratohyalin granules observed by electron microscopy
(b) Melanosomes and premelanosomes by electron
microscopy
(c) Positive staining for gastrin by light microscopy
(d) Positive staining for mucin by light microscopy

MOST RECENT QUESTIONS

35. Which of the following is the most common site of


mucosa associated lymphoid tissue?
(a) Duodenum
(b) Jejunum
(c) Ileum
(d) Stomach
36. One of the following can have malignant transformation:
(a) Gastric ulcer
(b) Duodenal ulcer
(c) Stomal ulcer
(d) Stress ulcer
37. Gastrointestinal stromal tumor originates in which of the
following?
(a) Parietal cells
(b) Chief cells
(c) Neuroendocrine cells
(d) Interstitial cells of Cajal
38. Most common site of GIST is:
(a) Ileum
(b) Esophagus
(c) Colon
(d) Stomach
39. Which of the following is not true about GIST?
(a) Stomach is the most common site
(b) High propensity of malignant change
(c) Associated with c-KIT mutation
(d) Histology shows spindle shaped cells
40. Which of the following artery is responsible for duodenal
ulcer hemorrhage?
(a) Superior pancreaticoduodenal artery
(b) Inferior pancreaticodudenal artery
(c) Gastroduodenal artery
(d) Left gastric artery
41. Krukenberg tumor associated mostly with which cancer?
(a) Stomach
(b) Brest
(c) Liver
(d) Pancreas
42. The best prognosis is gastric carcinoma is in type:
(a) Linitis plastic
(b) Polypoidal growth
(c) Ulcerative
(d) Superficial spreading
43. Most common association in MEN I is:
(a) Gastrinoma
(b) Insulinoma
(c) Lipoma
(d) Glucagonoma

INTESTINE: INFECTIONS, MALABSORPTION DISEASES

44. Which of the following is a histological feature of


Whipple’s disease?
(AI 2008)
(a) Infiltration of histiocytes in the lamina propria
(b) Granuloma in the lamina
(c) Macrophages with PAS (+) material inside the lamina
propria
(d) Eosinophils in the lamina propria
45. Gluten sensitive enteropathy is most strongly associated
with:
(AI 2003)
(a) HLA-DQ2
(b) HLA-DR4
(c) HLA-DQ3
(d) Blood group ‘B’
46. In the intra-epithelial region of the mucosa of intestine the
predominant cell population is that of:
(AI 2002)
(a) B cell
(b) T-cells
(c) Plasma cells
(d) Basophils
47. Macrophages containing large quantities of undigested
and partial digested bacteria in intestine are seen in:
(a) Whipple’s disease
(AI 2002)
(b) Amyloidosis
(c) Immunoproliferative small intestinal disease
(d) Vibrio cholerae infection
48. The histological features of celiac disease include all of
the following, except:
(AI 2002)
(a) Crypt hyperplasia
(b) Increase in thickness of the mucosa
(c) Increase in intraepithelial lymphocytes
(d) Increase in inflammatory cells in lamina propria
49. Type of anemia caused by Ileocecal TB:
(a) Iron – deficiency
(AIIMS Nov 2009)
(b) Megaloblastic
(c) Sideroblastic
(d) Normocytic Normochromic
50. The following cereals should be avoided in patients with
celiac diseases, except:
(AIIMS Nov 2003)
(a) Wheat
(b) Barley
(c) Maize
(d) Rye
51. Which of the following organs is not involved in
Whipple’s disease?
(Delhi PG 2009)
(a) Heart
(b) CNS
(c) Lungs
(d) GI Tract
52. Morphological features of celiac disease include all
except:
(Delhi PG 2009 RP)
(a) Increase in intraepithelial lymphocytes
(b) Increase in crypt: villous ratio
(c) Distended macrophages with PAS positive granules in
lamina propria
(d) Elongated hyperplastic and tortuous crypts
53. All are true about amoebic ulcer except:
(UP 2002)
(a) Commonest site is ascending colon and cecum
(b) Flask shaped ulcer
(c) Perforation is common
(d) Paucity of inflammatory cells
54. Intestinal biopsy in not diagnostic in:
(UP 2002)
(a) Abetalipoproteinemia
(b) Tropical sprue
(c) Agammaglobulinemia
(d) Intestinal lymphangiectasis
55. Transverse ulcers are seen in:
(UP 2004)
(a) Typhoid
(b) Tuberculosis
(c) Amoebiasis
(d) Ulcerative colitis
56. Aphthous ulcers are also known as:
(UP 2007)
(a) Canker sores
(b) Marjolin’s ulcer
(c) Curling’s ulcer
(d) Cushing’s ulcer
57. All are complication of typhoid ulcers except:
(UP 2008)
(a) Perforation
(b) Stricture formation
(c) Hemorrhage
(d) Sepsis
58. Which one of the following tumors is most commonly
associated with pseudomyxoma peritonei?
(AP 2006)
(a) Appendix
(b) Gall bladder
(c) Stomach
(d) Pancreas
59. All are true about typhoid ulcer except:
(a) Mainly affects ileum
(Kolkata 2005)
(b) Multiple ulcer and transverse
(c) Perforation occurs at 3rd week
(d) Perforation treated by surgery
60. A patient who recently underwent a gastrectomy
procedure complains of nausea, diarrhea, sweating,
palpitations, and flushing soon after eating a meal. This
patient should be instructed to:
(a) Eat less frequent, larger meals that are high in
carbohydrates
(b) Eat more frequent, smaller meals that are high in fat
(c) Eat more frequent, larger meals that are high in protein
(d) Eat more frequent, smaller meals that are high in
carbohydrates
61. A female patient has severe arthritis involving the lower
back. Before making a diagnosis of ankylosing
spondylitis, the patient should be questioned by the
physician about which of the following diseases?
(a) Carcinoid syndrome
(b) Celiac disease
(c) Crohn’s disease
(d) Whipple’s disease
62. A patient with intestinal malabsorption is found to
markedly improve when flour products (bread, noodles,
etc.) are removed from his diet. At the height of the
patient’s disease, marked histologic changes would be
seen at which of the following sites?
(a) Distal large bowel
(b) Distal small bowel
(c) Proximal small bowel
(d) Entire small bowel
63. A 25-year-old man presents to a rheumatologist with
complaints of joint pain involving the large joints of the
legs which exacerbates frequently accompanied by
diarrhea. Which of the following gastrointestinal
diseases is most likely to be implicated as the cause of
the patient’s joint problems?
(a) Amebic colitis
(b) Chronic appendicitis
(c) Diverticulosis
(d) Ulcerative colitis

MOST RECENT QUESTIONS


64. Based on epidemiological studies, which of the following
has been found to be most protective against colon
cancer?
(a) High fiber diet
(b) Low fat diet
(c) Low selenium diet
(d) Low protein diet
65. Purtscher’s retinopathy is seen in:
(a) Meningitis
(b) Pancreatitis
(c) Uncontrolled hypertension
(d) Unilateral carotid artery occlusion
66. Usually, gall stones consists of these types, except:
(a) Oxalate
(b) Bile salts
(c) Bile pigments
(d) Cholesterol
67. Which of the following is the commonest site of intestinal
tuberculosis?
(a) Stomach
(b) Jejunum
(c) Ileum
(d) Colon
68. The most common site for amoebiasis:
(a) Sigmoid colon
(b) Transverse colon
(c) Cecum
(d) Liver
69. Diverticulum most common site is:
(a) Sigmoid colon
(b) Ileum
(c) Ascending colon
(d) Transverse colon
70. Anti-gliadin antibodies are detectable in:
(a) Tropical sprue
(b) Whipple’s disease
(c) Celiac disease
(d) Intestinal lymphoma
71. Which of the following is not associated with celiac
sprue?
(a) Turner syndrome
(b) Down syndrome
(c) Klinefleter syndrome
(d) Type 1 diabetes
72. Paneth cells contain:
(a) Zinc
(b) Copper
(c) Molybdenum
(d) Selenium
73. Which is incorrect of typhoid ulcers?
(a) Hemorrhage is common
(b) Occurs on lymphoid aggregation
(c) Horizontal ulcers
(d) Longitudinal ulcers
74. Which of the following is not considered a pre-
malignant lesion?
(a) Leukoplakia
(b) Erythroplakia
(c) Chronic hyperplastic candidiasis
(d) Oral lichen planus
75. Diagnosis of typhoid in first week is by:
(a) Widal test
(b) Stool culture
(c) Urine culture
(d) Blood culture
76. Perforation of typhoid ulcer usually occurs during which
week?
(a) 1st
(b) 2nd
(c) 3rd
(d) 4th
77. Serum amylase level are raised in all of the following
except:
(a) Duodeneal ulcer perforation
(b) Pancreatitis
(c) Appendicitis
(d) Small Bowel Strangulation
78. What is false about Meckel’s diverticulitis?
(a) Present in 3% of the population
(b) Presents with periumbilical pain
(c) Remnant of proximal part of vitellointestinal duct
(d) Lies on the anti-mesenteric border
79. All are true about celiac disease except:
(a) Crypt hyperplasia
(b) Increase in thickness of the mucosa
(c) Increase in intraepithelial lymphocytes
(d) Increase in inflammatory cells in lamina propria

INTESTINE: IBD, POLYP, TUMORS

80. Most important prognostic factor for colorectal carcinoma


is:
(AIIMS May 2011)
(a) Site of lesion
(b) Tumour size and characteristics
(c) Age of patient
(d) Lymph node status
81. Which of the following is NOT true about FAP?
(a) AR inheritance
(AIIMS May 2011)
(b) Screening done by sigmoidoscopy
(c) Polyps develop in early adulthood
(d) Epidermal cysts and osteomas may occur
82. In Peutz-Jeghers syndrome, polyps are mainly seen in:
(a) Rectum
(b) Colon
(AIIMS May 2010)
(c) Esophagus
(d) Jejunum
83. Which of the following is not true about FAP?
(AIIMS May 2010, May 2011)
(a) Autosomal recessive inheritance
(b) Screening done by sigmoidoscopy
(c) Polyps develop in early adulthood
(d) Epidermal cysts and osteomas may occur
84. Colon carcinoma is associated with all except:
(a) Rb
(AI 2009)
(b) Mismatch repair genes
(c) APC
(d) b-catenin
85. In ulcerative colitis, which of the following is seen?
(a) Cryptitis
(AIIMS May 2008)
(b) Crypt loss
(c) Crypt branching
(d) Proliferating mucosa
86. Which of the following would be the best morphological
feature to distinguish ulcerative colitis from Crohn’s
disease?
(AIIMS May 2004)
(a) Diffuse distribution of pseudopolyps
(b) Mucosal edema
(c) Crypt abscesses
(d) Lymphoid aggregates in the mucosa
87. Which of the following statements about Crohn’s disease
is incorrect?
(Delhi PG 2009)
(a) Granulomas present frequently
(b) It is separate and distinct from ulcerative colitis
(c) Cigarette smoking is a risk factor
(d) Rectum spared in 50% patients with large bowel
involvement
88. Commonest endocrine tumour of pancreas artses from
which of the following cells?
(Karnataka 2009)
(a) a cells
(b) b cells
(c) Delta cells
(d) VIPoma
89. Skin lesions are seen in:
(Karnataka 2006)
(a) Ulcerative colitis
(b) Crohn’s disease
(c) Both (a) and (b)
(d) None of the above
90. Polyps in Peutz-Jegher’s syndrome are:
(a) Adenomatous polyps
(Karnataka 2004, 2008)
(b) Hyperplastic polyps
(c) Hamartomatous polyps
(d) Pseudopolyps
91. All are malignant in nature except:
(UP 2000)
(a) Juvenile polyp
(b) Familial polyp
(c) Carcinoid tumor
(d) Villous adenoma
92. All are true about Crohn’s disease except:
(a) Rectal involvement is common
(UP 2001)
(b) Granuloma formation
(c) Erythema nodosum
(d) Fistula formation
93. Most common tumor of appendix is:
(UP 2005)
(a) Carcinoid tumor
(b) Pseudomyxoma-peritonitis
(c) Adenocarcinoma
(d) Mucocele
94. Skip lesions are seen in:
(UP 2005)
(a) Ulcerative colitis
(b) Crohn’s disease
(c) Carcinoid syndrome
(d) Whipple’s disease
95. Fistula is most common in:
(UP 2007)
(a) Crohn’s disease
(b) Ulcerative colitis
(c) Infective enterocolitis
(d) Celiac sprue
96. Most common site of carcinoid tumor is:
(RJ 2000)
(a) Stomach
(b) Jejunum
(c) Distal ileum
(d) Appendix
97. Most common site of carcinoma pancreas is:
(RJ 2000)
(a) Head
(b) Body
(c) Tail
(d) Equal incidence at all sites
98. Carcinoid tumor produces all except:
(RJ 2003)
(a) Flushing
(b) Diarrhea
(c) Bronchodilation
(d) Raynaud’s phenomenon
99. Which organ is always involved in ulcerative colitis?
(a) Jejunum
(b) Ileum
(RJ 2005)
(c) Rectosigmoid
(d) Duodenum
100. Two identical specimen of the intestine obtained
following colectomy shows on examination hemorrhagic
cobblestone appearance; one of them however, shows
longitudinal grooving. It is likely to be a specimen of:
(a) Ulcerative colitis
(Kolkata 2002)
(b) Ischemic colitis
(c) Multiple polyposis
(d) Crohn’s disease
101. Carcinoma of colon is associated with all except:
(a) High fat diet
(Kolkata 2005)
(b) High fiber diet
(c) Streptococcus bovis infection
(d) Ulcerative colitis
102. Backwash ileitis is seen in:
(Bihar 2006)
(a) Crohn’s disease
(b) Ulcerative colitis
(c) Colonic carcinoma
(d) Ileal polyp
103. Granulomatous inflammation is found in: (Bihar 2006)
(a) Crohn’s disease
(b) Ulcerative colitis
(c) Amoebiasis
(d) Giardiasis
104. Continuous involvement of colonic mucosa is seen in:
(a) Ulcerative colitis
(Bihar 2006)
(b) Crohn’s disease
(c) Carcinoma colon
(d) Colonic polyp
105. All are true about carcinoid syndrome except:
(a) Wheezing
(Jharkhand 2006)
(b) Pulmonary stenosis
(c) Flushing
(d) Splenomegaly
106. A 24-year-old male Anil B. with a 4-year history of
abdominal pain, periodic diarrhea, low-grade fever, and
easy fatigability is found to have an enteroenteric fistula
on contrast radiography. Colonoscopy shows
“cobblestone” mucosa that has linear ulcerations with
“skip areas” of normal bowel wall. Which of the following
is the most likely explanation of fistula formation in this
patient?
(a) Intramural granulomas
(b) Transmural inflammation
(c) Marked lymphoid reaction
(d) Skip lesions of the intestinal wall
107. A 51-year-young man Firdaus had been receiving
antibiotics for severe folliculits. He develops fever,
toxicity, and severe diarrhoea. Which of the following is
the most likely diagnosis?
(a) CMV infection
(b) Pseudomembranous colitis
(c) Ulcerative colitis
(d) Whipple disease
108. A 50-year-old man Bhupi presents to his doctor with
diarrhea, flushing and wheezing. Physical examination is
significant for a grade II/VI diastolic murmur located at
the right sternal border at the 4th intercostal space.
Which of the following substances is most likely to be
elevated in this patient’s urine?
(a) 5-HIAA
(b) HVA
(c) Phenylalanine
(d) Selegiline
109. Biopsy of a small, rounded rectal polyp demonstrates
glands and sawtooth crypts composed of a proliferation
of goblet and columnar epithelial cells. No atypia is seen.
This polyp is best classified as which of the following?
(a) Hyperplastic polyp
(b) Peutz-Jeghers polyp
(c) Tubular adenoma
(d) Tubulovillous adenoma

MOST RECENT QUESTIONS

110. True about ulcerative colitis, all except:


(a) Rectum involved
(b) Pseudopolyps
(c) Pancolitis
(d) Noncaseating granuloma
111. Which of the following is the most common location of
carcinoid tumor?
(a)Pancreas
(b) Lung
(c) Gastrointestinal tract
(d) Gonads
112. Toxic megacolon is seen in:
(a) Chronic nonspecific ulcerative colitis
(b) Crohn’s disease
(c) Colonic diverticulosis
(d) Hamartomatous polyp
113. Carcinoid tumour develops from:
(a) Hematopioetic cells
(b) Kulschitsky cells
(c) Neuroglial cells
(d) Chromaffin cell
114. Which of the following is not an etiological factor for
pancreatitis?
(a) Abdominal trauma
(b) Hyperlipidemia
(c) Islet cell hyperplasia
(d) Germline mutations in the cationic trypsinogen gene
115. Pseudopolyps are seen in:
(a) Crohn’s disease
(b) Ulcerative colitis
(c) Juvenile polyposis
(d) Tuberculosis
116. The highest malignant potential is seen in:
(a) Crohn’s disease
(b) Ulcerative colitis
(c) Familial polyposis
(d) Infantile polyp
117. Commonest malignant small intestinal tumor:
(a) Adenocarcinoma
(b) Lymphosarcoma
(c) Leiomyosarcoma
(d) Carcinoid tumor
118. Most common site of carcinoma pancreas is:
(a) Head
(b) Body
(c) Tail
(d) None
119. Which of the following is the commonest cause of rectal
bleeding in children?
(a) Peutz Jegers polyp
(b) Juvenile polyp
(c) Adenomatous polyp
(d) Inflammatory polyp
120. Which of the following type of anemia would be
associated with carcinoma of the colon?
(a) Megaloblastic anemia
(b) Iron deficiency anemia
(c) Aplastic anemia
(d) Hemolytic anemia
121. Following statements regarding ulcerative colitis is:
(a) Smoking does not have a protective effect
(b) Smoking has a protective effect
(c) No relation with smoking
(d) Smoking causing relapses
122. Zollinger Ellison syndrome is not caused by tumors
from:
(a) Pancreas
(b) Ovary
(c) Colon
(d) Duodenum
123. Crohn’s disease is associated with:
(a) NOD2/CARD15 gene
(b) P53 gene
(c) Philadelphia chromosome
(d) BRCA1 gene
124. Which of the following is inheritance of Gardner
syndrome?
(a)Autosomal recessive
(b) Autosomal dominant
(c) X linked dominant
(d) X linked recessive
125. Osteomas, adenomatous polyps of intestine and
periampullary carcinomas are seen in which oif the
following conditions?
(a) Cowden syndrome
(b) Peutz Jegers syndrome
(c) FAP
(d) Gardener syndrome
126. A highly sensitive and specific marker for detecting
intestinal inflammation in ulcerative colitis is:
(a) CRP
(b) Fecal lactoferrin
(c) Fecal calprotectin
(d) Leukocytosis
127. The minimum number of polyps necessary for a
diagnosis of Familial Adenomatous Polyposis (FAP) is:
(a) 05
(b) 10
(c) 50
(d) 100
128. Antibody suggestive of diagnosis of ulcerative colitis:
(a) p-ANCA
(b) c-ANCA
(c) A.M.A
(d) A.N.A
129. A 32-year-old slum dweller presented with complaints of
diarrhoea and a 2 kg weight loss since last 2 months. On
examination, the patient has a dry tongue and poor skin
turgor. An endoscopic biopsy from the gut reveals the
following microscopic appearance. What is the likely
diagnosis?
(AIIMS Nov 2016)

(a) Whipple’s disease


(b) Entamoeba histolytica
(c) Helicobacter pylori
(d) Giardiasis
130. Which of the following is false regarding carcinoid
tumor?
(a) Neuroendocrine tumor
(b) Most common site lung
(c) Associated with serotonin production
(d) Potentially malignant tumor
131. Most common site for small intestinal carcinoma is:
(a) Duodenum
(b) Jejunum
(c) Ileum
(d) All are affected equally
1. Ans. (b) Intestinal metaplasia
(Ref: Robbins 9/e 757)
Direct quote from Robbins …. ‘Barrett esophagus is a complication
of chronic GERD that is characterized by intestinal
metaplasia within the esophageal squamous mucosa’.
2. Ans. (a) Mediastinal fibrosis
(Ref: Robbins 9/e p758-759)
• External beam irradiation but not mediastinal fibrosis
is a risk factor for esophageal cancer.
• HPV DNA is found frequently in esophageal squmaous
cell carcinoma in high incidence regions..Robbins
7th/807
• Caustic ingestion, achalasia, bulimia, tylosis (an
inherited autosomal dominant trait), Plummer-Vinson
syndrome, external-beam radiation, and esophageal
diverticula all have known associations with squamous
cell cancer...... ...Sabiston textbook of surgery 18th edn
3. Ans. (c) Reflux disease
(Ref: Robbins 8th/769, 9/e p755)
Robbins clearly states that “Reflux of gastric contents into the
lower esophagus is the most important cause of esophagitis”.
4. Ans. (a) Edge of ulcer
(Ref: Robbins 8th/768)
• Herpes viruses typically cause punched-out ulcers; the
nuclear inclusions of herpes virus are found in a narrow
rim of degenerating epithelial cells at the margin of the
ulcer.
• CMV causes linear ulceration of the esophageal
mucosa; the histologic findings of CMV-associated
change with both intranuclear and cytoplasmic
inclusions are found in capillary endothelium and
stromal cells in the base of the ulcer.

For diagnosis, the biopsy should be taken from edge of ulcer in HSV and
base of ulcer in CMV.
So, friends both a and b options are correct. But as we have to
choose only one, we will go for option (a) because Herpes
simplex is the most common virus causing esophagitis.
5. Ans. (b) Adenovirus
(Ref: Robbins 8th/768, 9/e p754, Harrison 17th/1853)
Viruses that can cause esophagitis are: HSV-1, HSV-2, Varicella
zoster virus, Cytomegalovirus and HIV For diagnosis, the
biopsy should be taken from edge of ulcer in HSV and base
of ulcer in CMV.
Findings in biopsy of edge of ulcer in HSV are:
• Ballooning degeneration
• Ground glass changes in nuclei
• Cowdry type A intranuclear inclusion bodies.
6. Ans. (a) Squamous to columnar metaplasia
(Ref: Robbins 7th/804, 9/e p757)
7. Ans. (c) Gastroesophageal reflux
(Ref: Robbins 9/e p757)
8. Ans. (c) Megaloblastic anemia
9. Ans. (d) It is a known precursor of adenocarcinoma of the
esophagus
(Ref: Robbins 9/e p758, 8th/769-772)
Barrett esophagus is columnar metaplasia of the esophageal
squamous epithelium (squamous–to-columnar). The
columnar epithelium is often of the intestinal type with goblet
cells. Barrett esophagus is a complication of long-standing
gastroesophageal reflux disease and is a precursor of
esophageal adenocarcinoma. The most common location is
in the distal (lower) third of the esophagus.
10. Ans. (b) Zenker’s diverticulum
(Ref: Robbins 9/e p753)
This is the classic presentation of Zenker’s diverticulum, which is
a false diverticulum formed by herniation of the mucosa at a
point of weakness at the junction of the pharynx and
esophagus in the posterior hypopharyngeal wall. It is also
associated with halitosis, and if the diverticulum fills
completely with food, it can cause dysphagia or obstruction of
the esophagus.
• Mallory-Weiss tears (option A) are mucosal tears at the
gastroesophageal junction secondary to repeated, forceful
vomiting. They are often seen in alcoholics.
• Schatzki rings (option C) are mucosal rings found in the distal
esophagus at the squamocolumnar junction.
• In contrast to a Zenker’s diverticulum, the usually asymptomatic
traction diverticula (option D) are true diverticula involving all of the
layers of the esophagus. They are typically caused by adherence
of the esophagus to a scarred mediastinal structure.

11. Ans. (a) Achalasia


(Ref: Robbins 8th/768, 9/e p753)
12. Ans. (d) Buccal mucosa
(Ref: Robbins 8th/811, 9/e p721-732)
13. Ans. (b) Lateral margin
The book mentions that ‘60-70% arise from the lateral surface
of the middle third of the tongue’. (Ref:
Oral Cancer: Diagnosis, Management, and Rehabilitation
p 100)
14. Ans. (c) Zenker diverticulum
(Ref: Robbins 9/e p 758-9)
15. Ans. (b) Tobacco use
(Ref: Robbins 9/e p731)
16. Ans. (b) Warthins tumor
(Ref: Robbins 9/e p745)
17. Ans. (c) Commonly turns malignant
(Ref: Robbins 9/e p744)
A carcinoma arising in a pleomorphic adenoma is referred to
variously as a carcinoma ex pleomorphic adenoma or a
malignant mixed tumor. The incidence of malignant
transformation increases with time, being about 2% for
tumors present less than 5 years and almost 10% for those
present for more than 15 years.
18. Ans. (b) CD117
(Ref Robbins 8th/789-790, 9/e p776)
Direct quote Robbins… ‘The most useful diagnostic marker for
GIDT is c-kitQ (also known as CD117Q)’.
19. Ans. (b) Stomach Cancer
(Ref: Robbins 8th/786, 9/e p776)
20. Ans. (a) CD117 (Ref:
Robbins 8th/789-790, 7th 826-827, Harrison 17th/573)
The most useful diagnostic marker is c-kit (CD117) detectable in
95% of the patients. Other markers like CD34 and vimentin
can also be expressed by some tumor cells. CD34 is also
present on pluripotent hematopoetic stem cell.
21. Ans. (a) CD117
(Ref: Robbins 8th/790, 9/e p776)
22. Ans. (c) Intestinal metaplasia see text for details
(Ref: Harrison 17th/572)
23. Ans. (d) Fundus
(Sleisenger &
Fordtrans text book of Gastroinstestinal disease 7th/813)
Pernicious anemia is associated with autoimmune atrophic
gastritis affecting the fundic glands. Intestinal metaplasia
(premalignant for gastric carcinoma), is characteristically
seen in this area of atrophic gastritis. Atrophic glands with
extensive intestinal metaplasia are most characteristically
confined to the fundus in patients with pernicious anemia.
24. Ans. (a) Gastric carcinoma
(Ref: Robbins 8th/786)
25. Ans. (c) Interstitial cells of Cajal
(Ref: Robbins 7th/826, 9/e p775)
26. Ans. (d) Atrophic gastritis > (b) Postgastrectomy
(Ref: Sternberg’s diagnostic surgical pathology,
Volume 2, page 1451, LWW, Biopsy interpretation of the
gastrointestinal tract mucosa by EA Montgomery,
Lippincott William Wilkins; 120, multiple journals)
Sternberg.. ‘Multifocal atrophic gastritis with intestinal metaplasia
is present commonly in postgastrectomy stomach. It is not
clear whether it is pre-existing (for which surgery was done)
or develops after gastrectomy’.
Biopsy interpretation of the gastrointestinal tract
mucosa… ‘some authors have associated gastric xanthoma
with atrophic gastritis’.
Digestive Diseases and Sciences, Vol. 31,1986,page 925-8
mentions… ‘It says xanthomatosis is characterized by
collections of lipid-laden macrophages, or foam cells,
plaques or nodules in many tissues, most commonly the skin.
Involvement can occur in all regions of the gastrointestinal
tract, but is most common in the stomach. It is more
common in patients with gastritis, gastric ulcer, and with
duodeno-gastric reflux after gastric surgery and mucosal
damage has been postulated to play an important role in its
pathogenesis. There is no documented relationship between
degree of hyperlipidemia or hypercholesterolemia and
presence of gastric xanthomatosis. Rather, it is associated
with atrophic gastritis.
Turkish Journal of gastroenterology.. ‘Lipid islands are found in
the stomach only when there are pathological changes such
as chronic gastritis, intestinal metaplasia, atrophic gastritis,
gastric ulcer,and changes caused by bile reflux or partial
gastrectomy’.
Clinical importance of knowing about gastric xanthoma
Atypical xanthoma cells can be easily confused with signet-ring
adenocarcinoma cell. However, xanthoma cells are negative with
periodic acid-Schiff (PAS) stain but show a positive reaction with Oil red
0 and weakly positive reaction with Masson trichrome. (Ref…Acta
Cytol. 2006 Jan-Feb; 50(1):74-9).

Signet-ring adenocarcinoma cells showed a strongly positive reaction


with PAS stain, cytokeratin and mucicarmine.

27. Ans. (d) Gastric adenoma


(Ref: Robbins 7th/817, 823, 826, 9/e p770)
• H. pylori is also associated with peptic ulcer disease,
gastric cancer and gastric mucosa associated
lymphoma (called MALToma or mucosa associated
lymphoid tissue tumor)
• Gastric adenoma are polypoid lesions of the stomach
found in the antrum most commonly.
28. Ans. (b) Mucosa and submucosa
(Ref: Robbins 7th/824, 825; fig/17-25, 9/e p771-772)
GASTRIC CARCINOMA
Classification – On the basis of
Depth of invasion Macroscopic Histologic
pattern subtype
(a) Early-involving mucosa 1. Exophytic (a) Intestinal
and submucosa 2. Flat or depressed type
(b) Advanced-extending into 3. Excavated (b) Diffuse type
muscularis propria and
beyond

29. Ans. (b) Parietal cell


(Ref: Robbins 8th/657; 7th/641, 9/e p765)
30. Ans. (c) Signet ring cell adenocarcinoma
(Ref: Robbins 8th/772)
The description is indicative of the presence of leather bottle
appearance (linitis plastica) of diffuse gastric carcinoma.
Microscopic examination reveals that in this cancer, diffuse
infiltration of the stomach wall by gastric type mucus cells is
present. The tumor cells have a signet ring appearance
because the cytoplasmic mucin pushes the nucleus to one
side.
• Early gastric carcinoma is confined to the mucosa and
submucosa.
• Gastrointestinal stromal tumors tend to be bulky masses.
• In chronic atrophic gastritis, there is no significant scarring or
shrinkage but rugal folds are lost.

Info

In the comparison of the whole GIT, granulomas are rarest in the


stomach.

31. Ans. (a) Acute gastritis


(Ref: Robbins 8th/773, 9/e p763)
Acute gastritis, characterized by patches of erythematous mucosa,
sometimes with petechiae and ulceration, can be seen as a
complication of a variety of other conditions (alcohol use,
aspirin and other NSAIDs use, smoking, shock, steroid use,
and uremia), which usually have in common disruption of the
mucosal barrier of the stomach.
• Chronic antral (type B) gastritis (option B) is associated with
Helicobacter pylori.
• Lymphocytic gastritis (option C) is thought to be a gastric
manifestation of celiac sprue.
• Hypertrophic gastritis (Menetrier’s disease; option D) is an
idiopathic condition characterized by markedly enlarged mucosal
folds.

32. Ans. (d) Increased production of macrocytic red blood


cells
(Ref: Robbins 8th/778-779, 9/e p765)
Autoimmune destruction of parietal cells would lead to decreased
secretion of gastric acid and intrinsic factor followed by poor
absorption of dietary vitamin B12 and then pernicious
anemia. It is characterized by increased production of
macrocytes (megaloblasts) by the bone marrow.
The luminal bacteria (option A) would most likely exhibit increased
(not decreased) growth due to sterilizing action of the acid.
A decrease in acid secretion leads to increased secretion of gastrin
by antral G cells because low gastric pH (less than 3) inhibits
gastrin secretion via paracrine release of somatostatin from
cells in the gastric mucosa that can sense the acidity. With
decreased parietal cells, the pH of the gastric lumen would
rise and remove this inhibitory component.
Because less acid would be delivered to the duodenum with
parietal cell destruction, less secretin would be released into
the blood.
33. Ans. (c) Esophagus (Read explanation below)
Striated (skeletal) muscle not under voluntary control is an unusual
feature of the upper third of the esophagus. The middle third
of the esophagus contains roughly half striated and half
smooth muscle; the lower third contains only smooth muscle.
All the other structures listed in the answer choices contain
smooth muscle.
34. Ans. (d) Positive staining for mucin by light microscopy
(Ref: Robbins 8th/785)
35. Ans. (c) Ileum
(Ref: Robbins 9th/772)
Direct quote… “Although extranodal lymphomas can arise in
virtually any tissue, they do so most commonly in the GI tract,
particularly the stomach”.

However, the question is regarding the most common site for MALT (and
not MALToma) for which the answer is ileum.

36. Ans. (a) Gastric ulcer


(Ref: Bailey 25/e p1055, Robbin 9/e p767)
Chronic duodenal ulcers are not associated with
malignancy but, in contrast, gastric ulcers are.
• It is fundamental that any gastric ulcer should be
regarded as being malignant, no matter how classically
it resembles a benign gastric ulcer.
• Stomal ulcers occur after a gastroenterostomy or a
gastrectomy of the Billroth II type. The ulcer is usually
found on the jejunal side of the stoma.
37. Ans (d) Interstitial cells of Cajal (Ref: Robbins 9/e p 775)
38. Ans. (d) Stomach
(Ref: Robbins 8/e p789-90, 9/e p776)
39. Ans. (b) High propensity of malignant change
(Ref: Robbins 8/e p789-90, 9/e p775-776)
High-yield Imaging: Gastrointestinal p213
Direct quote from High yield.. “90% of stomach GISTs are found to
be benign”.
40. Ans (c) Gastroduodenal artery
(Ref: Robbins 9/e p767; Bailey 25/e p1064)
41. Ans (a) Stomach
(Ref: Robbins 9/e p1034)
42. Ans (d) Superficial spreading (Ref: Robbins 9/e p772-3)
43. Ans. (a) Gastrinoma
(Ref: Robbins 9/e p1136)
Know the following about MEN-1-associated pancreatic
endocrine tumors:
• Pancreatic polypeptide is the most commonly secreted product,
however it is non functional.
• In functional tumors, gastrinomas are the commonest.

44. Ans. (c) Macrophages with PAS (+) material inside the
lamina propria
(Ref: Harrison 18th/2474, Robbins 9/e p792)
• Hallmark of Whipple’s disease had been presence of PAS positive
macrophages containing the characteristic small bacilli.
• Just revise friends that the presence of T. Whipplei outside of
macrophages is more important indicator of active disease than
is their presence inside the macrophages.

45. Ans. (a) HLA–DQ2


(Ref: Harrison’s 17th/2051, Robbins 9/e p782)
Celiac sprue is associated with HLA-DQ2. For other diseases
associated with HLA; refer to the table in the chapter of
Immunity (chap-6)
46. Ans. (b) T-cells
(Ref: Mucosal Immunology Elsevier, 3rd/565)
Direct quote.. ‘IEL are a distinctive population of T cells dispersed
among the luminal epithelial cells. Particularly in the small
intestine, there is a predominance of CD 8+T cells.
• Increase in IEL is defined as > 40 lymphocytes per 100
enterocytes.

47. Ans. (a) Whipple’s disease


(Ref: Robbins 7th/884 9/e p792, Harrison 17th/1884)
The hallmark of Whipple’s disease is a small intestinal mucosa
laden with distended macrophages in the lamia propria. The
macrophages contain periodic acid– Schiff (PAS) positive
granules and small rod shaped bacilli)
48. Ans. (b) Increase in thickness of the mucosa
(Ref: Robbins 7th/843, 9/e p783, Harrison’s 17th/1881)
• Characteristic histological features seen on
duodenal/jejunal biopsy in celiac sprue are:
1. Absence or reduced height of villi, resulting in ‘flat’
appearance.
2. Crypt cell hyperplasia compensate for villous atrophy and
mucosal thickness remain same
3. Cuboidal appearance and nucleus that are no longer basally
oriented and increased intraepithelial lymphocytes.
4. Increased lymphocytes and plasma cells in lamina propria.

• These features are characteristic of celiac sprue but


not diagnostic because similar features can be seen in:
1. Tropical sprue, 2. Eosinophilic enteritis, 3. Milk-protein
intolerance in children

• So, for establishing the diagnosis of celiac sprue, the


characteristic histological picture on small intestinal
biopsy should also revert back to normal on gluten
free diet. Gluten free diet also reverses the symptoms
as well as serological markers (anti-endomysial
antibodies).
49. Ans. (b) Megaloblastic
(Ref: Harsh Mohan 6th/569-571, Harrison 17th/649)
In intestinal tuberculosis the ileocecal junction is the commonest
site of involvement. Ileum is the physiological site for
absorption of vitamin B12. Also, TB is mentioned to be a
cause of folic acid deficiency which is therefore going to
result in megaloblastic anemia.
50. Ans. (c) Maize
(Ref: Robbin’s 8th/796, 9/e p782)
• It is a disease characterized by increased sensitivity to a
protein called gliadin present in the grains like wheat;
oat, barley and rye resulting in a T-cell mediated chronic
inflammatory reaction in the small intestine and impaired
absorption. It is associated with HLA-DQ2 or HLA-DQ8.
51. Ans. (c) Lungs (Ref:
Robbins 9/e p792, 8th/804, Harrison 17th/1884)
The organs in which these foamy macrophages can be seen are
Liver, Small intestine, Lymph nodes, Heart, Eyes, CNS and
synovial membranes of joints.
52. Ans. (c) Distended macrophages with PAS positive
granules in lamina propria
(Ref: Robbins 9/e p792)
It is feature of Whipple’s disease.
53. Ans. (c) Perforation is common
(Ref: Robbins 9/e p795)
54. Ans. (b) Tropical sprue
(Ref: Robbins 9/e p784)
55. Ans. (b) Tuberculosis
(Ref: Robbins 9/e p376)
56. Ans. (a) Canker sores
(Ref: Robbins 9/e p728)
57. Ans. (b) Stricture formation
(Ref: Robbins 9/e p789)
58. Ans. (a) Appendix
(Ref: Robbins 9/e p816)
59. Ans. (b) Multiple ulcer and transverse
(Ref: Robbins 8th/801, 9/e p789)
60. Ans. (b) Eat more frequent, smaller meals that are high in
fat
Read explanation below
The postgastrectomy symptoms in the given question is called the
dumping syndrome. Since all or part of the stomach is
removed, an ingested meal will be delivered to the small
intestine more quickly than normal. The large increase in
tonicity in the small intestine causes an osmotic fluid shift
from the extracellular fluid (plasma) into the lumen of the gut.
The increased distention of the small intestine increases
motility through reflex mechanisms and causes diarrhea. The
blood volume contraction and concomitant release of
vasoactive substances such as bradykinin and/or vasoactive
intestinal peptide can create hypotension and reflex
tachycardia.
These patients should be instructed to eat more frequent, smaller
meals to reduce the osmotic and/or carbohydrate load that is delivered
to the small intestine. Furthermore, since fats are the slowest to be
absorbed, a diet that is higher in fat will also reduce the problem of
rapid absorption.

61. Ans. (c) Crohn’s disease


(Ref: Robbins 9/e p799-800)
62. Ans. (c) Proximal small bowel
(Ref: Robbins 8th/795-796, 9/e p782-783)
The patient has celiac disease, which is apparently an acquired
hypersensitivity to the gluten (such as gliadin) in wheat.
Unlike tropical sprue (which may be related to enterotoxigenic
E. coli infection), which involves the entire small bowel, celiac
sprue is usually limited to the proximal small bowel.
63. Ans. (d) Ulcerative colitis
(Ref: Robbins 9/e p800)
The most frequent GIT disorder which can be associated with
sacroiliitis (related to HLA-B27) or lower limb arthritis is the
chronic inflammatory bowel diseases, ulcerative colitis and
Crohn’s disease. Other GI diseases associated with
arthropathy include bypass surgery, Whipple’s disease,
Behcet’s syndrome, and celiac disease.
Amebic colitis (choice A) is caused by ingestion of infectious
cysts (typically from Entamoeba histolytica). Cecal amebiasis
can resemble acute appendicitis.
Diverticulosis (choice C) is usually a disease of older adults. It is
often asymptomatic unless inflammation supervenes.
64. Ans. (a) High fiber diet
(Ref Robbins 9/e p811)
The dietary factors predisposing to a higher incidence of
cancer are
• Excess dietary caloric intake relative to requirements,
• A low content of unabsorbable vegetable fiber,
• A corresponding high content of refined carbohydrates,
• Intake of red meat, and
• Decreased intake of protective micronutrients.
Concept
• Reduced fiber content leads to decreased stool bulk, increased
fecal transit time in the bowel, and an altered bacterial flora of the
intestine. Potentially toxic oxidative byproducts of carbohydrate
degradation by bacteria are therefore present in higher
concentrations in the stools and are held in contact with the colonic
mucosa for longer periods of time.
• High fat intake (red meat) enhances the synthesis of cholesterol
and bile acids by the liver, which may be converted into potential
carcinogens by intestinal bacteria.
• Refined diets also contain less of vitamins A, C, and E, which may
act as oxygen-radical scavengers.
• NSAIDs like aspirin are protective in colon cancer because they
inhibit the COX-2 enzyme which is responsible for the proliferation
of the colonic mucosa. The COX-2 expression is upregulated by
TLR4 which recognizes lipopolyaccharides and is over-expressed
in adenoma and carcinoma.

65. Ans (b) Pancreatitis


Purtscher’s retinopathy is manifested by a sudden and
severe loss of vision in a patient with acute pancreatitis. It is
caused by occlusion of the posterior retinal artery with
aggregated granulocytes. There are cotton-wool spots and
hemorrhages confined to an area limited by the optic disc
and macula.
66. Ans (a) Oxalate
(Ref: Robbins 9/e p876)
There are two general classes of gallstones: cholesterol stones,
containing more than 50% of crystalline cholesterol
monohydrate, and pigment stones composed predominantly
of bilirubin calcium salts.
67. Ans. (c) Ileum
(Ref: Robbins 8/e p372)
Although any portion of the gastrointestinal tract may be affected,
the terminal ileum and the cecum are the sites most
commonly involved”… Harrison 18th
68. Ans. (c) Cecum
(Ref: Robbins 9/e p795)
Amoebiasis is seen most frequently in the cecum and
ascending colon…….Robbins
69. Ans. (a) sigmoid colon
(Ref: Robbins 8/e p814-5, 9/e p803, Bailey 25/e p1160)
• The condition is found in the sigmoid colon in 90% of cases.
• Interestingly in South-east Asia, right-sided diverticular diseaseQ
is twice as common as the left.
• The main morbidity of the disease is due to sepsisQ.

70. Ans. (c) Celiac disease


(Ref: Robbins 9/e p783)
• The most sensitive tests are the presence of IgA antibodies to
tissue transglutaminase or IgA or IgG antibodies to
deamidated gliadin.
• Anti-endomysial antibodies are highly specific but less sensitive
than other antibodies.

71. Ans. (c) Klinefelter syndrome


(Robbins 9/e p782-783)
Direct line… ‘There is also an association of celiac disease with
other immune diseases including type 1 diabetes,
thyroiditis, and Sjögren syndrome, as well as ataxia,
autism, depression, some forms of epilepsy, IgA
nephropathy, Down syndrome and Turner syndrome’
72. Ans. (a) Zinc
(Ref: Robbins 8/e p804)
Paneth cells contain zinc along with lysozyme. They are involved
in gut defence.
73. Ans. (c) Horizontal ulcers
(Ref: Bailey 25/e p1174)
74. Ans. (d) Oral lichen planus
(Ref: Bailey 25/e p735)

Conditions associated with malignant transformation

High-risk lesions Medium-risk Low-risk/equivocal-


lesions risk lesions

*Erythroplakia *Oral submucous *Oral lichen planus


*Speckled fibrosis *Discoid lupus
erythroplakia *Syphilitic glossitis erythematosus
*Chronic hyperplastic *Sideropenic *Discoid keratosis
candidiasis dysphagia congenita

• In the Indian subcontinent oral submucous fibrosis is


very common. This condition is characterized by limited
opening of mouth and burning sensation on eating of
spicy food.
75. Ans. (d) Blood culture….remember the acronym BASU for
blood, antibody, stool and urine for making the diagnosis
in the 1st/2nd/3rd and 4th week respectively.
76. Ans. (c) 3rd
(Ref: Bailey 25/e p1174)
Perforation of a typhoid ulcer usually occurs during the third
weekQ and is occasionally the first sign of the disease.
77. Ans. (c) Appendicitis
(Ref: Bailey 25/e p1132)
Causes of raised serum amylase level other than acute
pancreatitisQ
• Upper gastrointestinal tract perforationQ
• Mesenteric infarctionQ
• Torsion of an intra-abdominal viscus
• Retroperitoneal haematoma
• Ectopic pregnancy
• Macroamylasaemia
• Renal failure
• Salivary gland inflammation
78. Ans. (a) Present in 3% of the population
(Ref: Robbins 9th/751)
Meckel diverticulum occurs as a result of failed involution of the
vitelline duct, which connects the lumen of the developing gut
to the yolk sac. This solitary diverticulum extends from the
antimesenteric side of the bowel. The “rule of 2s” is often
used to help remember characteristics of Meckel diverticula,
which:

• Occur in approximately 2% of the population


• Are generally present within 2 feet (60 cm) of the ileocecal valve
• Are approximately 2 inches (5 cm) long
• Are twice as common in males
• Are most often symptomatic by age 2 (only approximately 4% are ever
symptomatic).

79. Ans (b) Increase in thickness of the mucosa


(Ref: Robbins 9/e p783)

80. Ans. (d) Lymph node status


(Ref: Robbins 9/e p813)
Direct quote Robbins .. ‘the two most important prognostic factors
are depth of invasion and the presence or absence of lymph
node metastasis,
81. Ans. (a) AR inheritance
(Ref Robbins 8th/820-821, 9/e p809)
Prophylactic colectomy does not decrease risk for cancer due to
adenomas at other sites specially ampulla of Vater and
stomach.
82. Ans. (d) Jejunum
(Ref: Robbins 8th/817, 9/e p806)
The polyps of Peutz-Jeghers syndrome are most common in the
small intestine, although they may occur in the stomach and
colon, and, with much lower frequency, in the bladder and
lungs.
83. Ans. (a) Autosomal recessive inheritance
(Ref: Robbins 8th/820-821, , 9/e p809)
Familial polyposis syndrome (FAP) is an autosomal dominantQ
disorder. It is caused by mutation in the adenomatous
polyposis coli or APC gene on chromosome 5q21. Atleast
100 polyps are necessary for a diagnosis of FAP.
Option ‘c’..Robbins clearly writes that colorectal carcinoma develop
in 100% of untreated FAP patients often before age 30. As a
result, prophylactic colectomy is the standard therapy in
patients with APC mutations.
84. Ans. (a) Rb
(Ref: Robbins 8th/823-824, 9/e p811)
Two distinct genetic pathways are described in
adenocarcinoma of colon:
Adenoma-carcinoma sequence: It accounts for 80% of sporadic
colon cancers. Mutation of APC gene occurs early or is inherited.
Whenever second allele of APC is mutated or inactivated, b-catenin
accumulates [APC protein degrades b-catenin]. Beta–catenin
translocates to nucleus and activates genes like myc and cyclin D1.
Additional mutations occuring later are:
• Kras
• SMAD-2 and SMAD-4
• p53
Microsatellite instability pathway: Mutations in DNA mismatch repair
genes (MSH2 and MLH1) results in expansion of microsatellites. This
microsatellite instability may result in decreased functioning of TGF- b
type II and bax proteins. Mutations in BRAF and epigenetic silencing of
genes by hypermethylation may also occur. Kras and p53 are not
typically mutated.

85. Ans. (a) Cryptitis


(Ref: Robbins 7th/849-850; 9/e 800, Harrison 17th/1888)
The pathology in ulcerative colitis typically involves distortion of
crypt architecture, inflammation of crypts (cryptitis), frank
crypt abscess, and hemorrhage or inflammatory cells in the
lamina propria.
The mnemonic for important features is Ulcerative COLITIS
(described in text).
86. Ans. (a) Diffuse distribution of pseudopolyps
(Ref: Harrison 17th/2077, Robbins 8th/807-13, 9/e p800)
• Pseudopolyps (inflammatory polyps) can be seen in both
Crohn’s disease and ulcerative colitis.
• Even Mucosal edema, crypt abscess and mucosal
lymphoid aggregates are features common to both the
types of inflammatory bowel disease.
• However, diffuse distribution of these polyps is observed
only in ulcerative colitis because in Crohn’s disease,
there is presence of skip lesions in which there is
presence of normal area adjacent to diseased segments
of the intestine. So, patchy distribution of polyps is
observed in Crohn’s disease.
87. Ans. (d) Rectum spared in 50% patients with large bowel
involvement
(Ref: Robbins 8th/810-2, 9/e p799, Harrison 17th/1888)
88. Ans. (b) b cells
(Ref: Robbins 7th/1205)
89. Ans. (c) Both a and b
(Ref: Robbins 7th/849-851, 9/e p800)
Please don’t confuse ‘skin lesions’ with ‘skip lesions’, the latter are
seen only in Crohn’s disease.
90. Ans. (c) Hamartomatous polyp
(Ref: Robbins 9/e p806)
• Peutz-Jegher’s polyps are hamartomatous polyps
that involve mucosal epithelium, lamina propria and
muscularis mucosa. Peutz-Jegher’s polyps are located
usually in small intestine most commonly in jejunum.
• When they occur in multiple numbers, condition is called
Peutz-Jegher’s syndrome. It is rare autosomal
dominant syndrome characterized by multiple
hamartomatous polyps scattered throughout GIT, along
with mucosal and cutaneous melanotic pigmentation
along lips, face, genitalia and palms
Genetic basis of Peutz-Jegher’s syndrome is mutation in gene
STKII (LKB1) located on chromosome 19 which encodes
protein with serine/threonine kinase activity.
91. Ans. (a) Juvenile polyp
(Ref: Robbins 9/e p805)
92. Ans. (a) Rectal involvement is common
(Ref: Robbins 9/e p799, 8th/810-811; 7th/851)
93. Ans. (a) Carcinoid tumor
(Ref: Robbins 9/e p816)
94. Ans. (b) Crohn’s disease
(Ref: Robbins 9/e p799)
95. Ans. (a) Crohn’s disease
(Ref: Robbins 9/e p800)
96. Ans. (c) Distal ileum
(Ref: Robbins 9/e p774)
97. Ans. (a) Head
(Ref: Robbins 9/e p893)
98. Ans. (c) Bronchodilation
(Ref: Robbins 9/e p774)
99. Ans. (c) Rectosigmoid
(Ref: Robbins 9/e p800)
100. Ans. (d) Crohn’s disease
(Ref: Robbins 9/e 799)
101. Ans. (b) High fiber diet
(Ref: Robbins 8th/811)
102. Ans. (b) Ulcerative colitis
(Ref: Robbins 9/e 800)
103. Ans. (a) Crohn’s disease
(Ref: Robbins 9/e p799)
104. Ans. (a) Ulcerative colitis
(Ref: Robbins 9/e p800)
105. Ans. (d) Splenomegaly
(Ref: Robbins 9/e p774)
106. Ans. (b) Transmural inflammation
(Ref: Robbins 9/e p799-800, 8th/810)
The typical presentation of Crohn’s disease is abdominal pain and
diarrhea in a 20-30-year-old patient. Weight loss, fatigability,
low grade fever, and aphthous ulcers of the oral mucosa are
also common.
Transmural inflammation explains the two most common
complications of Crohn’s disease: strictures, and fistulas.
Chronic inflammation causes edema and fibrosis leading to
narrowing of the intestinal lumen (strictures). Necrosis of the
intestinal wall causes ulcer formation. Ulcers can penetrate
the entire thickness of the affected intestinal wall, leading to
the formation of a fistula.
Please contrast with ulcerative colitis in which only the mucosa and
submucosa are inflammed, so, strictures and fistulas are not common.
• (Choices a and d) Intramural granulomas and skip
lesions are commonly found in Crohn’s disease. They do not,
however, predispose to fistula formation.
• (Choice c) A chronic inflammatory infiltration that
consists predominantly of monocytes and lymphocytes is
characteristic of Crohn’s disease. However, it is not the
composition of the inflammatory infiltrate, rather the fistula’s depth
that is responsible for fistula formation.
107. Ans. (b) Pseudomembranous colitis
(Ref: Robbins 9/e p791, 8th/803)
108. Ans. (a) 5-HIAA
(Ref: Robbins 9/e p774, 8th/788-789)
109. Ans. (a) Hyperplastic polyp
(Ref: Robbins 9/e p804)
This is a hyperplastic polyp; these polyps comprise 90% of all
colonic polyps and have no malignant potential.
Peutz-Jeghers polyps (choice ‘b‘) also have no malignant potential,
but tend to be larger and have a complex branching pattern.
Tubular adenomas and tubulovillous adenomas, (choices c and d)
are all true neoplastic polyps containing dysplastic
epithelium; the malignant potential of these polyps increases
with size and the percentage of the polyp which has a villous
configuration.
110. Ans. (d) Noncaseating granuloma
(Ref: Robbins 9/e p799)
111. Ans. (c) Gastrointestinal tract
(Ref: Robbins 9/e p774, 8/e p788 , Harrison 18/e p)
Direct quote from Harrison.. “The GI tract is the most common
site for these tumors, accounting for 64%, with the respiratory
tract a distant second at 28%.”
112. Ans. (a) Chronic nonspecific ulcerative colitis
(Ref: Robbins 9/e p800, 8/e p812)
In ulcerative colitis, inflammation and inflammatory mediators can
damage the muscularis propria and disturb neuromuscular
function leading to colonic dilation and toxic megacolon,
which carries a significant risk of perforation
113. Ans. (b) Kulschitsky cells
(Ref: Robbins 9/e p774)
Carcinoid tumour arises from the neuroendocrine cells called
Kulschitsky cells. These cells are also called as
Enterochromaffin (EC) cellsand are present in
gastrointestinal tract and the respiratory tract.
114. Ans. (c) Islet cell hyperplasia (Ref. Robbins 9th/884)
Etiological factors in acute pancreatitis
Metabolic
Alcoholism
Hyperlipoprotelnemia
Hypercalcemia
Drugs (e.g., azathioprine)
Genetics
Mutations in genes encoding trypsin, trypsin regulators, or proteins that
regulate calium metabolism
Mechanical
Gallstones
Trauma
Iatrogenic injury
• Operative injury
• Endoscopic procedures with dye injection
Vasular
Shock
Atheroembolism
Vasculitis
Infections
Mumps

115. Ans. (b) Ulcerative colitis


(Ref: Robbins 9/e p800)
Though pseudopolyps may be seen in both Crohn’s disease
and ulcerative colitis, it is more common in ulcerative
colitis.
116. Ans. (c) Familial polyposis
(Ref: Robbins 9/e p809)
Direct line.. “Colorectal cancer develops in 100% of untreated FAP
patients often before the age of 30”.
117. Ans. (a) Adenocarcinoma
• Small intestinal adenocarcinomasQ and carcinoids
have roughly equal incidence, followed in order by
lymphomas and sarcomas. However, other texts
mention that adenocarcinomas are common. So, its he
answer of consensus here.
• Most common benign small intestinal tumor is
adenomaQ. It is located most commonly near Ampulla
of VaterQ.
118. Ans. (a) Head
(Ref: Robbins 9/e p893)
Carcinoma of the pancreas
• More than 85% of pancreatic cancers are ductal
adenocarcinomasQ.
• Ductal adenocarcinomas arise most commonly in the head of the
glandQ.
• Painless jaundiceQ secondary to obstruction of the distal bile duct
is the most common symptom.
• The jaundice may be associated with nausea and epigastric
discomfort.
• On examination, there may be evidence of jaundice, weight loss, a
palpable liver and a palpable gallbladderQ
• If there is a genuine suspicion of a tumour in the head of the
pancreas, the preferred test is a contrast-enhanced CT scanQ
• The standard resection for a tumour of the pancreatic head or the
ampulla is a pylorus-preserving pancreatoduodenectomyQ.
This is considered better than the earlier performed Whipple
procedure.

119. Ans. (b) Juvenile polyp


(Ref: Robbin 8/e p 816-7)
• Juvenile polyps are focal malformations of the mucosal
epithelium and lamina propria.
• The vast majority of juvenile polyps occur in children
less than 5 yearsQ of age.
• The majority of juvenile polyps are located in the rectumQ
and most present with rectal bleeding.Q
120. Ans (b) Iron deficiency anemia
(Ref: Robbins 9/e p 813)
The underlying cause of iron deficiency anemia in an older man or
postmenopausal woman is GI cancer until proven otherwise.
121. Ans. (b) Smoking has a protective effect
(Ref: Robbins 9/e p 800)
122. Ans (c) Colon
(Ref: Robbins 9/e p 769; Harrison 18/e p 2455-6)
• Zollinger-Ellison syndrome is caused by gastrin-secreting
tumors. These gastrinomas are most commonly found in
the small intestine or pancreas.
• The extrapancreatic sites of these tumors are
duodenum (most common extrapancreatic site),
stomach, bones, ovaries, heart, liver, and lymph nodes.
Also revise!

Gastrinoma triangle (confluence of the cystic and common bile ducts


superiorly, junction of the second and third portions of the duodenum
inferiorly, and junction of the neck and body of the pancreas medially).

123. Ans (a) NOD2/CARD15 gene


(Ref: Robbins 9/e p 797)
One of genes most strongly associated with Crohn’s disease is
NOD2 (nucleotide oligomerization binding domain 2), which
encodes an intracellular protein that binds to bacterial
peptidoglycans and activates signaling events, including the
NF-κB pathway.
124. Ans. (b) Autosomal dominant
(Ref: Robbins 9/e p809)
Familial Adenomatous Polyposis is caused by the mutation
of adenomatous polyposis coli (APC) gene present on the
long arm of chromosome 5Q (5q21). It is inherited as an
autosomal dominant disorder.
125. Ans. (d) Gardener syndrome
(Ref: Robbins 9/e p809)
126. Ans. (b) Fecal lactoferrin
(Ref: Harrison 18/e)
In ulcerative colitis, active disease can be associated with a
rise in acute-phase reactants C-reactive protein (CRP),
platelet count, erythrocyte sedimentation rate (ESR), and
a decrease in hemoglobin.
Direct line… “Fecal lactoferrin is a highly sensitive and
specific marker for detecting intestinal inflammation”.
Other options:

• Fecal calprotectin levels correlate well with histologic inflammation, predict


relapses, and detect pouchitis.
• Leukocytosis may be present but is not a specific indicator of disease activity.
Fecal lactoferrin is a marker of fecal leukocytes and is more
sensitive and is available in latex agglutination and enzyme-
linked immunosorbent assay formats.
127. Ans. (d) 100
(Ref: Robbins 9/e p809)
128. Ans. (a) p-ANCA
(Ref: Robbins 9/e p860)
p-ANCA is also called now by the name of anti myeloperoxidase
(MPO-ANCA). It is directed against a lysosomal granule
constituent and is seen in the following conditions:

• Ulcerative colitis
• Churg-Strauss syndrome
• Primary sclerosing cholangitis
• Microscopic polyangiitis
• Focal necrotising and crescentic glomerulonephritis
• Rheumatoid arthritis

129. Ans. (d) Giardiasis


(Ref: Robbins 9th/795)
Giardia lamblia are the most common parasitic pathogen in
humans and are spread by fecally contaminated water or
food. It can be identified in duodenal biopsies based on their
characteristic pear shape and the presence of two equally
sized nuclei. Despite large numbers of trophozoites, which
are tightly bound to the brush border of villous enterocytes,
there is no invasion and small intestinal morphology may be
normal (as seen in the figure). However, villous blunting with
increased numbers of intraepithelial lymphocytes and mixed
lamina propria inflammatory infiltrates can develop in patients
with heavy infections.
130. Ans. (b) Most common site lung
(Ref: Robbins 9/e p773-4)
Commonest site for the carcinoid tumor is the gastrointestinal
tract.

131. Ans. (a) Duodenum


(Ref: Robbins 9/e p734)

• 50% of the small bowel adenocarcinomas occur in the duodenum. It


is also good to know that Periampullary region is the location of
more than 50% adenocarcinomas
1. A newborn baby presented with a failure to pass
meconium in the immediate postnatal period. The
pediatrician also notices visible yet ineffective
peristalsis, and abdominal distention. A radiological
contrast enema demonstrated a narrow conical segment
and a dilated proximal bowel. A diagnosis of
Hirschsprung disease was made. Which of the following
is a cause of the condition in the patient?
(NEET 2020 like pattern)
(a) Failure of migration of neural crest cells
(b) Excessive peristalsis of the affected part of gut
(c) Failure of involution of vitelline duct
(d) Obstruction secondary to an infectious agent
Ans. (a) Failure of migration of neural crest cells
(Ref: Robbins 9th/751)
• Hirschsprung disease, also known as congenital aganglionic
megacolon, results when the normal migration of neural crest cells from
cecum to rectum is arrested prematurely or when the ganglion cells
undergo premature death.
• This produces a distal intestinal segment that lacks both the Meissner
submucosal and the Auerbach myenteric plexus (“aganglionosis”).
• Coordinated peristaltic contractions are absent and functional obstruction
occurs, resulting in dilation proximal to the affected segment.
• Patient typically presents with a failure to pass meconium in the immediate
postnatal period. They may also develop obstruction or constipation often
with visible, ineffective peristalsis. Some may have abdominal distention
and bilious vomiting.

2. A patient presented with diarrhea, poor appetite and


malabsoption. His duodenal biopsy was taken which
showed crypt hyperplasia, villi atrophy and infiltration of
CD8+ T cells in the epithelium. What is the likely
diagnosis of the patient?
(NEET 2020 like pattern)
(a) Environmental enteropathy
(b) Celiac disease
(c) Whipple disease
(d) Pancreatitis
Ans. (b) Celiac disease
(Ref: Robbins 9th/ 783)
Celiac disease is an immune-mediated enteropathy triggered by
the ingestion of gluten-containing grains. The malabsorptive
diarrhea in celiac disease is due to loss of brush border
surface area and deficient enterocyte maturation as a result
of immune-mediated epithelial damage.
Presence of crypt hyperplasia, villi atrophy and infiltration of CD8+ T cells in
the epithelium in the duodenal or proximal jejunal biopsy of a patient
presenting with features of malabsorption is a clear pointer towards celiac
sprue.

3. A 5-year-old child presented with rectal bleeding. He has a


polypoidal mass located in the rectum. The biopsy is
shown in the image below. What is the likely diagnosis
for this patient?
(NEET 2020 like pattern)

(a) Serrated adenoma


(b) Villous adenoma
(c) Angiodysplasia
(d) Juvenile polyp
Ans. (d) Juvenile polyp
(Ref: Robbins 9th/ 805)
• Juvenile polyps are focal malformations of the epithelium and lamina
propria.
• Most of them occur in children younger than 5 years of age
• They are located in the rectum and typically present with rectal bleeding.
• Sporadic juvenile polyps (also called as retention polyps) are usually
solitary lesions.
• However, autosomal dominant syndrome of juvenile polyposis have from 3
to as many as 100 hamartomatous polyps and may require colectomy to
limit the chronic and sometimes severe hemorrhage associated with polyp
ulceration.
• Histologically, they are typically pedunculated, smooth-surfaced, reddish
lesions with characteristic cystic spaces (dilated glands filled with mucin
and inflammatory debris). The image shown matches this description.

4. Which of the following feature distinguishes ulcerative


colitis from Crohn’s disease?
(NEET 2019 like pattern)
(a) Crypt abscess
(b) Pseudopolyps
(c) Mucosal edema
(d) Lymphocyte infiltrate in muscle
Ans. (d) Lymphocyte infiltrate in muscle
(Ref: Robbins 9th e/p 799)
• Lymphocyte infiltrate in muscle is seen in Crohn’s disease.
Ulcerative colitis has inflammation limited to only mucosa and
submucosa.
• Rest all features are common to both these conditions.
5. Cushing Ulcer seen in:
(NEET 2019 like pattern)
(a) Stress ulcer in head injury
(b) Stress ulcer in Burns
(c) Ulcer in Hiatus hernia
(d) Ulcer in Crohn’s disease
Ans. (a) Stress ulcer in head injury
(Ref: Robbins 9th e/p 762)
• Ulcers occurring in the proximal duodenum and associated with severe
burns or trauma are called Curling ulcers.
• Gastric, duodenal, and esophageal ulcers arising in persons with
intracranial disease or head injury are termed Cushing ulcers and carry
a high incidence of perforation.

6. Punched out ulcer in esophagus is seen in?


(NEET 2019 like pattern)
(a) Herpetic esophagitis
(b) CMV esophagitis
(c) Eosinophilic esophagitis
(d) Candida esophagitis
Ans. (a) Herpetic esophagitis
(Ref: Robbins 9th e/p 755)
Herpes viruses typically cause punched-out ulcers; the nuclear
inclusions of herpes virus are found in a narrow rim of
degenerating epithelial cells at the margin of the ulcer.
7. Flask shaped ulcers in intestine caused by which of the
following?
(NEET 2019 like pattern)
(a) Typhoid
(b) TB
(c) Entamoeba histolytica
(d) Giardia
Ans. (c) Entamoeba histolytica
(Ref: Robbins 9th e/p 795)
Amebiasis creates a flask-shaped ulcer with a narrow neck and
broad base.
8. The earliest manifestation in the intestine of Crohn’s
disease is:
(AI 2018 Pattern)
(a) Stricture
(b) Aphthous ulcer
(c) Cobblestone mucosa
(d) Perforation
Ans. (b) Aphthous ulcer
(Ref: Robbins 9/e p798)
9. A 42-years-old patient has immunodeficiency and
complaints of dysphagia. His upper GI endoscopy
reveals multiple ulcers in the distal esophagus. The
biopsy picture is shown for the patient as under. What is
the likely diagnosis?
(AI 2018 Pattern)
(a) CMV infection
(b) Herpes infection
(c) Candida
(d) Drug induced esophagitis
Ans. (a) CMV infection
(Ref: Robbins 9/e p755)
The presence of an owl eye appearance of a nucleus (as shown in
the figure) is suggestive of cytomegalovirus infection
10. Which of the following is a histological feature of Crohn
disease?
(AI 2018 Pattern)
(a) Non caseating granulomas are seen
(b) Pseudopolyps are seen
(c) Non involvement of serosa
(d) Backwash ileitis may be seen in some patients
Ans. (a) Non caseating granulomas are seen
(Ref: Robbins 9/e p798)
11. Most common cause of pancreatitis is:
(AI 2018 Pattern)
(a) Alcohol
(b) Gallstones
(c) Trauma
(d) Smoking
Ans. (a) Alcohol
(Ref: Robbins 9/e p888)
12. Most common location of the chronic gastric ulcer is:
(AI 2018 Pattern)
(a) Greater curvature
(b) Lesser curvature near proximal stomach
(c) Lesser curvature near incisura
(d) Pylorus
Ans. (c) Lesser curvature near incisura
(Ref: Robbins 9/e p766)

• Gastric peptic ulcers are predominantly located along the lesser


curvature near the interface of the body and antrum
• Most common site for peptic ulcer is duodenum.

13. Small intestinal biopsy reveals PAS+ macrophages in the


lamina propria as shown below. Which of the following is
the likely diagnosis?
(AI 2018 Pattern)

(a) Gluten sensitive enteropathy


(b) Whipple’s disease
(c) Abetalipoproteinemia
(d) Giardiasis
Ans. (b) Whipple’s disease
(Ref: Robbins 9/e p791)
Presence of blunting of villi with foamy macrophages is suggestive
of Whipple’s disease in the given patient.
14. On endoscopy and barium swallow the following findings
are seen which of the following will be seen on
histopathology.
(AIIMS Nov 2017 Pattern)
Which of the following is an expected finding in this patient?

Ans. (d)
(Ref: Robbins 9/e p759)
• Image A….Normal esophagus with stratified squamous
epithelium
• Image B….esophagus with glandular differentiation
(adenocarcinoma)
• Image C….presence of intestinal epithelium with goblet
cells (Barret’s esophagus)
• Image D….presence of keratin pearls suggestive of
squamous cell cancer
Looking at the lesion given in the question, it is present in the
middle part of esophagus with an exophytic lesion on
endoscopy. That’s the commonest location of the esophageal
cancer in India. The commonest histological subtype of
esophageal cancer is squamous cell cancer an so, the
answer is option D.
15 A 55-year-old patient presented with dysphagia. Identify
the diagnosis from upper GI biopsy of esophagus
showed in the following picture:
(AIIMS May 2017 Pattern)

(a) Squamous cell carcinoma


(b) Barrett’s esophagus
(c) Eosinophilic esophagitis
(d) Adenocarcinorna
Ans. (b) Barrett’s esophagus
(Ref: Robbins 9/e p757)
Barret’s esophagus is signified by the presence of columnar cells
with goblet cells as is seen in the given image.
16. A middle-aged female presented with recurrent bloody
diarrhea. Colonoscopy reveals multiple geographic
ulcers and histopathological examination is shown
below. What is the likely diagnosis?
(AIIMS May 2017 Pattern)

(a) Adenocarcinoma colon


(b) Crohn’s disease
(c) Pseudomembranous colitis
(d) Amoebic colitis
Ans. (b) Crohn’s disease
(Ref: Robbins 9/e p799)
The presence of transmural inflammation in the given image
without the presence of any overlying membrane is
suggestive of the presence of Crohn disease. The question
mentions geographical ulcers which are produced by the
fusion of aphthous ulcers in Crohn’s disease.
Absence of a flask shaped ulcer and pseudomembrane rules out
the presenc of amoebic colitis and pseudomembranous
colitis respectively.
17. Maximum risk of carcinoma pancreas is seen in which of
these?
(AIIMS May 2017 Pattern)
(a) Peutz Jegher syndrome
(b) Hereditary atypical mole syndrome
(c) Hereditary pancreatitis
(d) Familial adenomatosis polyposis
Ans. (a) Peutz Jegher syndrome
(Ref: Robbins 9/e p894)
Increased risk of
Disorder Gene cancer (fold)

Peutz-Jeghers syndrome STK11 130


Hereditary pancreatitis PRSS1, SPINK1 50-80
Hereditary non-polyposis Multiple, including 8-10
colorectal cancer MLH1, MSH2
Familial atypical multiple-mole CDKN2A 20-35
melanoma syndrome
Strong family history Unknown 14-32
Hereditary breast and ovarian Multiple genes 4-10
cancer

Disclaimer
Any resemblance to an actual question is purely coincidental.
• Canals of herring and Von Meyenburg complexes are seen in the liver.
• Hexagonal lobule is a vital anatomical microstructure in the liver.
• Centrilobular necrosis is seen in hypovolemia, carbon tetrachloride and halothane
toxicity whereas yellow fever causes mid zonal necrosis.
• Inherited causes of unconjugated hyperbilirubinemia: Gilbert syndrome, Crigler-Najjar
syndrome.
• Inherited causes of conjugated hyperbilirubinemia: Rotor syndrome, Dubin-Johnson
syndrome.
• Bilirubin has affinity for elastin and so, jaundice is observed in skin and sclera.
• Acute hepatitis has the microscopic findings like focal or centrilobular nerosis,
ballooning degeneration, acidophilic degeneration (Councilman bodies) and interface
hepatitis.
• Chronic (active) hepatitis has the findings like Piecemeal necrosis, bridging necrosis,
portal fibrosis and interface hepatitis.
• Mallory bodies have Cytokeratin/keratin intermediate filament and are seen in causes
Alcoholism, liver cancer, Indian childhood cirrhosis, primary biliary cirrhosis, Wilson’s
disease and alpha 1 antitrypsin deficiency.
• Fatty liver is due to accumulation of: Triglyceride
• Most common cause of fatty liver (triglyceride accumulation) is Alcoholism.
• Major source of collagen in liver in cirrhosis: Ito cells (perisinusoidal stellate cells).
• Non-cirrhotic portal fibrosis is characterised by fibrosis in portal and periportal area
only (bridging fibrosis is absent); it has a young patient with hematemesis (variceal
bleeding) and splenomegaly in absence of hepatomegaly as clinical findings.
• Hemochromatosis is a disorder of iron metabolism characterised by triad of
Micronodular cirrhosis, diabetes mellitus and skin pigmentation.
• Organ not showing iron deposition in hemochromatosis: gonads (Testis/ovary).
• Angiosarcoma of liver is associated with exposure to either Arsenic, vinyl chloride
or thorotrast.
• Cavernous hemangioma is the most common benign tumor of liver.
• Hepatocellular carcinoma is the most common primary malignant tumor of liver.
• Fibrolamellar type of HCC has: best prognosis, equal sex incidence and non
association with HBV/cirrhosis. It has normal AFP levels.
• AFP is the best/ definitive marker for hepatoblastoma.
• Primary biliary cirrhosis has the presence of Antimitochondrial antibodies.
• ‘Comet tail artefact’ with thickening of gallbladder wall: Adenomyomatosis.
• Primary sclerosing cholangitis is associated most commonly with: IBD (UC > CD).
• ‘Onion skin’ fibrosis of bile duct is seen in: Primary sclerosing cholangitis.
• ‘Klastkin’ tumors are: Cholangiocarcinomas located at the junction of right and
left hepatic ducts.
Jaundice is characterized by hyperbilirubinemia and yellowing of the skin and sclera (due to
elastin fibers).

Indirect hyperbilirubinemia is called when unconjugated bilirubin is 85% or more of the


total bilirubin whereas direct hyperbilirubinemia corresponds to conjugated bilirubin more than
15% of total.
Causes of Jaundice

Prehepatic Cause Hepatic Cause Posthepatic Cause


↑ Bilirubin production leading to Due to defect in hepatocyte Due to impaired excretion of conjugated
unconjugated hyperbilirubinemia leading to defective conjugation bilirubin as a result of obstruction e.g.
e.g. hemolytic anemia. or decreased excretion of stone, cancer of pancreas and bile duct
conjugated bilirubin. etc.

Causes of Hyperbilirubinemia
Unconjugated Conjugated
• Physiological jaundice of newborn • Biliary tract obstruction
• Hemolytic anemia • Biliary tract disease like primary biliary cirrhosis and
• Diffuse hepatocellular disease primary sclerosing cholangitis
• Criggler Najjar syndrome • Dubin Johnson syndrome
• Gilbert syndrome • Rotor syndrome

Unconjugated Hyperbilirubinemia

Physiological Jaundice Criggler Najjar Syndrome Gilbert Syndrome


• Due to immaturity of the liver • Due to decreased UGT activity • Decreased bilirubin
• Increased changes with prematurity • Type I → Absence of UGT glucuronidation
and erythroblastosis fetalisQ. enzyme; 100% fatal • Jaundice associated with
• May result in kernicterus • Type II Reduced UGT activity. stress like illness, fasting or
(unconjugated bilirubin crosses BBB exerciseQ.
causing brain damageQ)

CIRRHOSIS

It is the end stage liver disease characterized by disruption of the liver architecture by fibrotic
bands that divide the liver into nodules of regenerating liver parenchyma. It can be
micronodular (if nodule is <3 mm) or macronodular (>3 mm) or mixed.
CAUSES
• Alcoholic liver disease (most common causeQ)
• Viral hepatitis
• Biliary tract disease
• Hemochromatosis
• Cryptogenic/idiopathic (non alcoholic fatty liver disease is its commonest cause)
• Wilson disease
• a-1-antitrypsin deficiency

Fig. 1: Liver cirrhosis.

Non-Cirrhotic Portal Fibrosis (NCPF)/Idiopathic Portal Hypertension (IPH)


It is a condition characterized by portal hypertension and moderate portal fibrosis without
cirrhosis. Though the exact etiology is unknown, it has been associated with the following:
• Infections: sepsis, diarrhea in children, bacterial infections
• Immunological mechanisms: associated with HLA DR-3
• Chemical exposure: arsenic, vinyl chloride, copper sulfate, hypervitaminosis A, drugs (steroid)

Histopathological Features
Intimal fibroelastosis of medium sized portal veins (Obliterative portovenopathy of liver):
characteristic finding. Other findings include: portal fibrosis (intra portal but not bridging
fibrosis), portal vein sclerosis, portal tract edema and lymphocytic infilteration,
pseudolobulation, and atrophy of liver parenchyma with no regenerative capacity.

FIGURE EXPLAINING THE PATHOGENESIS OF NCPF


Sites of resistance to
portal blood flow in
NCPF
Presinusoidal:
caused by
thickening/obstruction
of medium to small
brnches of portal veins
Perisinusoidal:
collagenisation of
Space of Disse

CLINICAL FEATURES
• Presentation in the 3rd-4th decade of life
• Low socioeconomic strata
• Slight sex preponderance M>F (this may vary however depending on geographical areas)
• GI bleeding (commonest symptom), massive splenomegaly with normal liver function
tests.

Clinical significance of NCPF


Massive splenomegaly and emergence of new aberrant blood vessels is most commonly associated with
NCPF out of all causes of portal hypertension.

INFECTIOUS DISORDERS OF LIVER

Hepatitis
ACUTE HEPATITIS
1. Swelling of the hepatocytes called “Ballooning”.
2. Presence of apoptotic hepatocytes giving rise to Councilman bodies. Apoptosis of a single
hepatocyte is called ‘spotty necrosis’.
3. Disruption of lobular architecture of the liver.
4. Necrosis connecting portal to portal, portal to central, central to central regions of adjacent
lobules is called bridging necrosis
5. Infiltration of portal tract with inflammatory cells.
6. Spilling of inflammatory cells in the adjacent parenchyma causing necrosis of adjacent
cells (Interface hepatitis or piecemeal necrosis).

Fig. 2: Active hepatitis having lymphocytic infiltration (L) and apoptotic cells (A).

CHRONIC HEPATITIS
Older classification of chronic hepatitic

Chronic persistant Hepatitis Chronic active hepatitis Chronic lobular


hepatitis
• Inflammation limited to • Inflammation involving both parenchyma and • Inflammation limited
portal tracts portal tracts to the lobules
• Lobular structure of liver is • Associated with “piecemeal necrosis” and
preserved “bridging necrosis”
• Lobular structure not preserved.
Latest classification of hepatitis is based on
1. Cause (Viral hepatitis, autoimmune hepatitis, drug associated hepatitis, etc.)
2. Histological activity or grade
– Grading is based on necrosis and inflammation which include assessment of the
following factors:
– Periportal necrosis including piecemeal necrosis and bridging necrosis
– Portal inflammation
– Intralobular necrosis
– Fibrosis
– 3. Degree of progression or stage
Staging is done based on the degree of fibrosis
Absent Mild Moderate Severe Cirrhosis
0 1 (Fibrosis confined to 2 (Portal and peri- 3 (Bridging fibrosis) 4 (Cirrhosis)
portal tracts with portal portal fibrosis)
expansion)

VIRAL HEPATITIS

1. Hepatitis A virus (Infectious Hepatitis)


– It is a benign and self-limiting disease.
– Incubation period of HAV is 2-6 weeks.
– It does not cause chronic hepatitis or carrier state in hepatitis.
– HAV is an unenveloped, single stranded RNA virus belonging to Picornavirus family.
– Feco-oral route is the predominant route of spread.
– HAV is present in stools 2-3 weeks before and 1 week after onset of jaundice.
– IgM antibody appearing in blood with onset of symptoms is a marker of acute infection
whereas IgG antibody provides life long immunity.

2. Hepatitis B virus (Serum Hepatitis)


– It can cause acute hepatitis, non-progressive chronic hepatitis, progressive chronic
hepatitis leading to cirrhosis, fulminant hepatitis, carrier state and hepatocellular
carcinoma.
– Incubation Period of HBV is 30-180 days.
– HBV is present in all physiologic and pathologic body fluids, except in stools (unlike
HAV).

Mode of Infection

• Blood products, needle sticks etc. (30%)


• Sexual transmission
• Vertical transmission is responsible for development of carrier state
• The virus is a double stranded DNA belonging to Hepadnaviridae family.
• It is a spherical, double layered ‘Dane particle’ 42 mm in size.

Genome
The genome of the virus has several genes coding for different proteins or enzymes. These
include:
‘S’ gene ‘C’ gene ‘P’ gene ‘X’ gene

• Codes for • Codes for 2 nucleocapsid • Codes for DNA • Codes for HBx proteinQ
envelope proteins: polymerase required for viral replication and
protein, HBs • HBc Ag:Q Intracellular having reverse transcriptional activator of viral
AgQ (surface nucleocapsid core antigen transcriptase and host genes.
antigen) • HBe Ag:Q Nucleocapsid activityQ • Particularly important in the
protein with a core and development of hepatocellular
precore region. carcinomaQ.

• The precore region directs the release of HBeAg towards secretion in the blood.
Uncommonly, mutated strains called precore mutants of HBV emerge that do not produce
HBeAg but are replication competent and express HBcAg. In these patients, the HBeAg
may be undetectable despite the presence of HBV viral load. A mutation in the core
promoter region can also lead to an HBeAg negative phenotype. Clinically both these
conditions are characterized by the presence of elevated liver enzymes and active viral
multiplication is indicated only by the high levels of DNA polymerase.
PHASE OF INFECTIONS
There is an initial proliferative phase in which the viral DNA is present in an episomal form
leading to the formation of complete virion with associated antigens. This is followed by
expression of viral antigens with MHC class I molecules resulting in CD8+ T cells activation
and destruction of infected hepatocytes. There is presence of an integrative phase in which the
viral DNA is incorporated into host DNA. This usually occurs in hepatocytes not destroyed by
immune response.

Sequential Appearance of Hepatitis B Markers and Significance

HBs Ag • Present in acute diseaseQ


• Continued presence indicates chronic disease or carrier
state
HBe Ag • Seen with active viral replication and denotes high
HBV DNA
DNA polymerase
infectivityQ

IgM anti HBc • Antibody detectable shortly before onset of symptoms


• Marker of window periodQ
• IgM anti HBc is indicator of recent disease whereas IgG
anti HBc is seen with chronic infection or prior infection
Anti HBe Ab • Detected after HBe Ag disappears and denotes low
infectivityQ
IgG anti HBs • Appears after disappearance of HBs Ag
• Provides protection against Hepatitis B and indicates
immunityQ
• Seen in prior infection and in vaccinated personsQ

NUTSHELL OF HEPATITIS B SEROLOGY

HBsAg IgM anti HBc IgG anti HBc IgG anti HBs
Acute HBV infection + + – –
Window period – + – –
Chronic infection + +/- + –
Prior infection – – + +
Immunization – – – +

In addition: Remember that the presence of HBeAg denotes high infectivity and its
absence denotes low infectivity.
Immunology concept
Immunization for HBV is based on the fact that anti HBs Ab is protective in nature. So, vaccination with non-
infectious HBsAg still retaining its immunogenic potential is done.

3. Hepatitis C virus (Transfusion Associated Hepatitis)


– HCV is a single stranded RNA virus belonging to Flaviviridae family.
– Incubation Period is 2-26 weeks.
– Acute HCV infection is generally undetectable clinically whereas chronic disease
occurs in majority of infected individuals.
– Spread of the virus is through inoculation and blood transfusion (more frequently) and
less commonly through sexual and vertical transmission.
– Acute illness is usually asymptomatic/mild.
– Initially, there is IgM anti-HCV followed by the presence of IgG anti-HCV antibodies.
– Chronic infection is associated with episodic elevations in serum transaminases with
intervening normal period associated with persistence of HCV RNA in the blood.

Note: Antibody against HCV is IgG anti-HCV which does not provide effective immunity because the virus
demonstrates genomic instability and antigenic variability.

4. Hepatitis D virus (Delta Agent)


– It is a single stranded RNA virus.
– Replication is defective and can cause infection only when encapsulated by HBs Ag.
– So, it can cause infection in 2 conditions:
- Acute coinfection: In which there is simultaneous exposure to both HBV and
HDV. However, HBV must establish first. This is associated with 90% chances of
recovery and only rare chances of development of chronic hepatitis.
- Super infection: In which chronic carriers of HBV get infected with HDV. This is
associated with majority developing chronic hepatitis.
The serology shows the presence of IgM Anti HDV which is the most reliable marker
of recent infection.
5. Hepatitis E virus
• It is a single stranded RNA unenveloped enterically transmitted virus accounting for more than
50% of cases of acute hepatitis in India.
• Incubation period is 2-8 weeks.
• It causes sporadic infection in young to middle aged adults (rare in children).
• The disease is self limiting (not associated with chronic disease).
• The serology shows the HEV RNA and presence of virions in stool and the liver even before onset of
clinical illness.
• IgM anti-HEV IgG anti HEV followed by is seen in 2-4 weeks.

6. Hepatitis G virus
It is a single stranded RNA virus transmitted by the parenteral route i.e. by the
contaminated blood or blood products and possibly by the sexual route. In up to 75% of
infections, HGV is cleared from the plasma and the infection becomes chronic in the
remaining 25%. The site of HGV replication is mononuclear cells, so, it does not cause any
rise in serum amino transferases and is non pathogenic. It co-infects patients with HIV
and the dual infection is protective against HIV disease.

Clinicopathologic Syndromes

1. Acute asymptomatic infection with recovery:


This is identified incidentally with the help of elevated serum transaminases or the presence of
antiviral antibodies
2. Acute viral hepatitis: It has got 4 phases:
Incubation period
Peak infectivity during last days of incubation period and early days of acute symptoms.
Symptomatic pre-icteric phase
Nonspecific, constitutional symptoms, malaise, general fatigability, nausea, and loss of appetite.
Symptomatic icteric phase
Caused mainly by conjugated hyperbilirubinemia, dark urine and light stools
Convalescence
Recovery due to T cell activity against infected hepatocytes.

3. Chronic viral hepatitis: Symptomatic, biochemical or serologic evidence of continuing or


relapsing disease for > 6 months with histologic documentation of inflammation and
necrosis. Chronic hepatitis constitutes a “Carrier State”. Healthy carriers are individuals
having the virus without adverse effects. Vertical transmission with HBV produces carrier in
90-95% cases. The most common symptom is fatigue; less common symptoms are
malaise, loss of appetite, and occasionally mild jaundice.
4. Fulminat hepatitis: Progression of hepatic insufficiency from onset of symptoms to
hepatic encephalopathy within 2-3 weeks, is called fulminant hepatic failure. The
progression in upto 3 months is called subfulminant failure.

ALCOHOLIC LIVER DISEASE


Alcoholic liver disease
Hepatic steatosis Alcoholic hepatitis Alcoholic cirrhosis

• Also called fatty liver • Having hepatocyte swelling and • Irreversible form of alcoholic
• Characterized by the presence of necrosis (ballooning liver disease
small (microvesicular) or large degeneration) • Initially, liver is enlarged and
(macrovesicular) lipid droplets • Neutrophilic infiltration in lobule later there is presence of
inside the hepatocytes • Perivenular and periportal fibrosis micronodules and
• Initial centrilobular involvement (due to ito cell in space of sisse) macronodules
followed by entire lobule involved • Some hepatocytes show the • Later, the whole liver has
• Reversible if there is abstinence presence of eosinophilic, tough, pale scar tissue
from alcohol. cytokeratin filaments called (Laennec Cirrhosis).
‘Mallory Hyaline bodies’.

Non Alcoholic Fatty Liver Disease (NAFLD)


• It is a condition that resembles alcohol-induced liver disease but occurs in patients who are not heavy
drinkers. Men and women are equally affected.
Association
• Strong association with obesityQ, dyslipidemiaQ, hyperinsulinemiaQ and insulin resistanceQ.
Spectrum of disease
• NAFLD includes simple hepatic steatosis, steatosis with non specific inflammation and non-alcoholic
steatohepatitis (NASH). Some patients may develop cirrhosis. Patients are largely asymptomatic, with
abnormalities only in biochemical laboratory tests.
Clinical Significance
• NAFL is now the most common cause of “cryptogenic” cirrhosisQ. It also contributes to the progression
of other liver diseases like hepatitis C viral infection.
Diagnosis
• NAFL is a diagnosis of exclusion (excessive alcohol intake has to be excluded).
• Liver biopsyQ is the most important diagnostic tool for NASH. It is also associated with AST/ALT ratio less
than 1 (in alcoholic steatohepattis the same ratio is >2.0).
Clinically
• The patients are asymptomatic with elevated enzyme levels. Cardiovascular diseaseQ is a frequent cause
of death.

HEMOCHROMATOSIS

In normal hepatocytes, HFE protein, hemojuvelin and transferrin receptor 1 and 2 regulate the
formation of hepcidin. As we know hepcidin causes ferroportin degradation, it leads to reduced
iron absorption though intestinal cells. Mutation in the proteins like HFE, HJV and TFR1/2
reduce hepcidin synthesis thereby leading to increased iron absorption and systemic iron
overload.
• It is characterized by excessive accumulation of iron in the body.
• It is an autosomal recessive disorder most commonly caused by mutations in HFE gene located at
6p21.3.
• It is more common in males characterized by the triad of
*Micronodular cirrhosis
*Diabetes mellitus and
*Skin pigmentation
• Most of the cells of the body have increased amounts of hemosiderin in them but skin pigmentation is
primarily due to increased intracellular melanin.
• Inflammation is characteristically absent
• Deposition of hemosiderin in the joint synovial lining can result in acute synovitis and pseudogout.
• Derangement of the hypothalamo-pituitary axis results in hypogonadism (loss of libido and impotence in
male and amenorrhea in the female).
• Treatment is removal of excessive iron and it is accomplished by weekly or twice weekly phlebotomy.
• Chelating agents like desferrioxamine are indicated only when anemia or hypoproteinemia is severe
enough to preclude phlebotomy.
• Cardiac failure and hepatocellular carcinoma are the most common causes of death.

Fig. 3: Hereditary hemochromatosis with Prussian Blue reaction having inclusions.

WILSON DISEASE

Normally in the body, copper is absorbed in the proximal small intestins and it binds to
apoceruloplamin to form ceruloplasmin which is then secreted in the blood. Circulating
ceruloplasmin is degraded in the liver and the released copper is excreted in the bile.

Wilson disease is an autosomal recessive disorder caused by mutation of the ATP7B gene located on
chromosome 13. The deficiency of the ATP7B protein leads to:
• Reduced copper transport in the bile
• Reduced ceruloplasmin formation and its secretion in the blood.
The net result is copper overload in the liver which then spills in the systemic circulation
affecting organs like red cells, liver, brain and eye. The average of onset of symptoms is 11.4
years. Patients may present as a liver disease (mild to cirrhosis like), neurological symptoms
(movement disorders, dystonia, mood liability, psychiatric symptoms and even hemolytic
anemia. Eyes may have the presence of Kayser-Fleischer rings (green to brown deposits of
copper in Descemet membrane in the limbus of the cornea).
Diagnosis is made by the presence of
• Decrease in serum ceruloplasmin,
• An increase in hepatic copper content (the most sensitive and accurate test), and
• Increased urinary excretion of copper (the most specific screening test).
Treatment is done by copper chelation therapy (trientine or penicillamine) or zinc therapy
(zinc inhibits absorption of copper in the intestine).
NODULAR HYPERPLASIAS
These are represented by two conditions: Focal nodular hyperplasia and Nodular
regenerative hyperplasia.

Focal nodular hyperplasia

• Presents as a spontaneous mass lesion


• Most frequently in young to middle-aged adults
• Female preponderance
• Associated with long term use of anabolic hormones or contraceptives
• Typically, there is a central grey-white, depressed stellate scar from which fibrous septa radiate to the
periphery. The central scar contains large vessels, usually arterial, that typically exhibit fibromuscular
hyperplasia with eccentric or concentric narrowing of the lumen.

Nodular regenerative hyperplasia

• Associated with the development of portal hypertension and its clinical manifestations.
• Occurs in conditions affecting intra hepatic blood flow like rheumatoid arthritis (most commonly), Felty
syndrome, myeloproliferative disorders, hyperviscosity syndromes, solid organ (particularly renal and liver)
transplantation, bone marrow transplantation, HIV infection (Robbins), vasculitic conditions and drugs
(anabolic steroids and cytotoxics).
• Characteristically, there is absence of fibrosis in this condition.

Hepatic Tumors
Hepatic tumors
Benign Malignant
• Cavernous hemangioma • Hepatoblastoma
– Most common benign lesion of liver – Most common liver tumor of young children
• Liver cell adenoma (hepatic adenoma) – Activation of Wnt/b catein signaling pathway
– Seen in young females causes carcinogenesis
– Associated with oral contraceptive intake • Angiosarcoma
– Microscopically, the cells have clear cytoplasm – Associated with previous exposure to arsenic
and the portal tracts are absent. vinyl chloride or thorotrast
• Hepatocellular carcinoma
• Cholangiocarcinoma

HEPATOCELLULAR CARCINOMA (HCC)

It is the most common primary malignant tumor of the liver. It usually affects old patients with a
M:F ratio of 3:1.
Risk factors for development of HCC
• Chronic Hepatitis (Hepatitis B, Hepatitis C)
• Alcoholism
• Aflatoxins (due to Aspergillus flavus infection of peanuts, grains)
• Tyrosinemia
• Hereditary hemochromatosis

Pathogenesis: HBV has the presence of HBX protein which causes activation of host cell
proto-oncogenes and disruption of cell cycle control. Aflatoxins cause mutations in proto-
oncogenes or p53 (tumor suppressor gene).

Clinical features include malaise, upper abdominal pain, weight loss and fatigue.
Laboratory investigations show elevation of serum a-fetoprotein (AFP). AFP elevation can also
be seen in yolk sac tumors, cirrhosis, massive liver necrosis, chronic hepatitis, normal
pregnancy, fetal distress or death, fetal neural tube defects (anencephaly and spina bifida).
Histologically, HCC can present as a unifocal mass, multifocal mass or diffuse infiltrative
cancer involving the entire liver. All the three variants have a strong tendency for vascular
invasion and intrahepatic metastasis. These can involve portal vein or inferior vena cava
extending upto right side of the heart. There is presence of Mallory Hyaline bodies (eosinophilic
intracytoplasmic inclusions of keratin filaments).

Distinguishing features of Fibrolamellar cancer (from hepatocellular cancer)


• Seen in young adults (20-40 years).
• Equal incidence in males and females, [In India, however, females > males]
• No association with HBV or cirrhosis risk factors.
• Has better prognosis.
• No elevation of serum AFP levels.
• Microscopic examination shows well differentiated cells separated by dense collagen bands.
• Fibrolamellar variant usually affects left lobe of the liver more commonly.
• It spreads by lymphatic route.

Metastasis from HCC takes place to:


1. Contiguous spread through hepatic vessels – “Satellite nodules”
2. Lungs
3. Perihilar, peri-pancreatic and para-aortic lymph nodes.

Cholangiocarcinoma
Cholangiocarcinoma (CC)

CC typically refers to mucin-producing adenocarcinomas that arise from the bile ducts.
Classification
Extrahepatic- 90%
• Perihilar-60%
• Distal bile duct – 20 to 30%
Intra-hepatic – 10%.
Risk factors (Mnemonic: All have alphabet ‘C’)
Genetic predisposition
There is over expression of IL-6 and K-RAS and reduced expression of p53.
Clinical features
Painless jaundice, often with pruritus or weight loss, and acholic stools.
Microscopically
It is an adenocarcinoma associated with dense collagenous stroma (desmoplastic reaction). The differentiated
bile duct epithelial cells do not produce bile, so, this cancer is rarely bile stained.
Diagnosis

• Percutaneous biopsy for peripheral liver lesions.


• Endoscopic retrograde cholangiopancreatography (ERCP) for central lesions.

Tumor markers

• Tumors stain positively for cytokeratin 7, 8, and 19 and negatively for cytokeratin 20.
• CEA, CA 19-9, and CA-125 are non specific and are useful for following response to therapy.

Metastasis

Spread of the cancer takes place to lungs, vertebrae, adrenals, brain and regional lymph nodes.

METASTATIC TUMORS

Secondary liver cancers are more common than primary liver cancers. The common primary
sites include breast, colon and lung. The metastatic nodules have central necrosis and
umbilication.
BILIRUBIN METABOLISM, HYPERBILIRUBINEMIAS

1. Which of the following condition is associated with unconjugated


hyperbilirubinemia?
(DPG 2011)
(a) Dubin-Johnson syndrome
(b) Rotor syndrome
(c) Gilbert syndrome
(d) Gall stones
2. A patient with unconjugated bilirubinemia has increased excretion of urobilinogen in
his urine. This can be seen in all of the following conditions, except:
(a) G6 PD deficiency
(AI 2010)
(b) Hemolytic anemia
(c) Hereditary spherocytosis
(d) Biliary cirrhosis
3. In post-hepatic jaundice, the concentration of conjugated bilirubin in the blood is
higher than that of unconjugated bilirubin because:
(AIIMS Nov 2002)
(a) There is an increased rate of destruction of red blood cells
(b) The unconjugated bilirubin is trapped by the bile stone produced in the bile duct.
(c) The conjugation process of bilirubin in liver remains operative without any
interference.
(d) The UDP glucuronyl transferase activity is increased manifold in obstructive jaundice
4. Function of hepatic stellate cells is/are:
(a) Formation of sinusoids
(PGI June 2001)
(b) Vitamin A storage
(c) Increases blood perfusion
(d) Phagocytosis
5. Which of the following diseases is not a cause of indirect hyperbilirubinemia?
(Delhi PG 2009)
(a) Rotor’s syndrome
(b) Criggler Najjar syndrome
(c) Gilbert syndrome
(d) Hereditary spherocytosis
6. In unconjugated hyperbilirubinemia, the fraction of unconjugated bilirubin to total
bilirubin exceeds:
(a) 0.65
(b) 0.50
(UP 2002)
(c) 0.35
(d) 0.80
7. A 40 years old woman Hema Thapar presents with generalized pruritus for last 4
months which is not relieved by various lotions available in the market. On
physical examination is unremarkable but her blood sample is sent to the
laboratory. Her reports are as follows:
• Total serum bilirubin of 2.0 mg/dL
• Direct bilirubin of 1.4 mg/dL
• SGOT 58 U/L
• SGPT 52U/L
• Alkaline phosphate is 300U/L
• Total protein is 7.2 g/dL with serum albumin of 3.5 g/dL
• Total cholesterol of 350 mg/dL.
Which of the following serologic test findings is most likely to be positive in this
patient?
(a) Anti-parietal cell antibody
(b) Antimitochondrial antibody
(c) Anti-centromere antibody
(d) Anti ribonucleoprotein antibody

MOST RECENT QUESTIONS


8. Which of the following is not a function of liver?
(a) Production of vitamin K
(b) Production of albumin
(c) Detoxification of ammonia
(d) Metabolism of drugs
9. Primary biliary cirrhosis is positive for:
(a) p-ANCA
(b) Anti nuclear antibody
(c) Anti-microsomal antibody
(d) Anti-mitochondrial antibody
10. Complete deficiency of UDP glucoronyl transferase (UGT) is seen in:
(a) Criggler–Najjar Type I
(b) Criggler–Najjar Type II
(c) Gilbert’s syndrome
(d) Dubin-Jhonson syndrome

CIRRHOSIS, NCPF

11. A 30 years old man Surajmal visits his physician because he noticed the
development of yellowish skin during last 5 days. His physical examination has
absence of abdominal pain or tenderness. His blood reports are as follows:
Haemoglobin 11.5 g/dL, MCV 94 µm3, platelet count 1,80,000/mm3, WBC count
6930/mm3, albumin 3.7 g/dL, total protein 5.6 g/dL, total bilirubin 8.2 mg/dL, direct
bilirubin, 0.5 mg/dL, AST, 45 U/L, ALT 32 U/L, and alkaline phosphatase, 340 U/L.
What of the following is the most likely diagnosis?
(a) Cholelithiasis
(b) HAV infection
(c) Micronodular cirrhosis
(d) Hemolytic anemia
12. Which one of the following is not a feature of liver histology in Non cirrhotic portal
fibrosis?
(AI 05, DPG ‘10)
(a) Fibrosis in and around the portal tracts
(b) Thrombosis of the medium and small portal vein branches
(c) Non specific inflammatory cell infiltrates in the portal tracts
(d) Bridging fibrosis
13. ‘Nutmeg liver’ is seen in:
(Karnataka 2005)
(a) Portal cirrhosis
(b) Biliary cirrhosis
(c) Chronic venous congestion of liver
(d) Fatty liver

MOST RECENT QUESTIONS


14. In cirrhosis of liver collagen is laid down by:
(a) Hepatocytes
(b) Hepatic stellate cells
(c) Biliary epithelial cells
(d) Kupffer cells
15. With the known finding of significantly increased serum ammonia, which of the
following physical findings may be expected in a patient of hepatic failure?
(a) Capillary telangiectasias
(b) Asterixis
(c) Caput medusae
(d) Gynecomastia
16. A 42-year-old woman with polycythemia vera develops progressive severe ascites
and tender hepatomegaly over a period of several months. Liver function tests are
near normal. Which of the following tests would be most likely to establish the
probable diagnosis?
(a) Endoscopic retrograde cholangiopancreatography
(b) Hepatic venography
(c) Serum alpha fetoprotein
(d) Serum iron
17. Micronodular cirrhosis is seen in all except:
(a) Alcoholic cirrhosis
(b) Viral hepatitis
(c) Budd-Chiari syndrome
(d) Indian childhood cirrhosis
18. Commonest site of varices in portal hypertension is:
(a) Esophagus
(b) Anal canal
(c) Periumbilical
(d) Liver
19. A 50-year-old chronic alcoholic with jaundice and ascites secondary to known
cirrhosis becomes disoriented and confused. Asterixis (flapping tremor) can be
demonstrated. Which of the following is not associated with the development of
ascites?
(a) Hypoalbuminemia
(b) Increased hepatic lymph formation
(c) Increased portal venous pressure
(d) Portal-systemic venous shunting
20. Nutmeg liver is seen in which of the following conditions?
(a) Right sided heart failure
(b) Left sided heart failure
(c) Increased pulmonary pressure
(d) Decreased pulmonary pressure
21. Nutmeg liver is seen in:
(a) Right sided heart failure
(b) Left sided heart failure
(c) Increased pulmonary pressure
(d) Decreased pulmonary pressure
22. Fibrosis associated with liver cirrhosis is mediated by:
(a) MCP-1
(b) PDGF
(c) ICAM-1
(d) IFN-gamma

HEPATITIS

23. A 20 year old man with HBs Ag +ve, HbeAg –ve with SGOT and SGPT raised 5 times
the normal value. The HBV DNA copies are 1,00,000/ml. Which is the likely
diagnosis?
(AI 2010)
(a) Wild type HBV
(b) Surface mutant HBV
(c) PreCore mutant HBV
(d) Inactive HBV carrier
24. Which one of the following diseases characteristically causes fatty change in liver?
(AI 2005)
(a) Hepatitis B virus infection
(b) Wilson’s disease
(c) Hepatitis C infection
(d) Chronic alcoholism
25. Councilman bodies are seen in:
(AIIMS Nov 2007)
(a) Wilson disease
(b) Alcoholic hepatitis
(c) Acute viral hepatitis
(d) Auto immune hepatitis
26. In Chronic Viral Hepatitis:
(AIIMS May 2004)
(a) Hepatitis A virus infection is a common cause in children
(b) Morphological classification into Chronic Active Hepatitis and Chronic Persistent
Hepatitis are important
(c) Fatty change is pathognomic of Hepatitis C virus infection
(d) Grading refers to the extent of necrosis and inflammation
27. The liver biopsy in acute hepatitis due to hepatitis B virus is likely to show all of the
following, except:
(a) Ballooning change of hepatocytes
(b) Ground glass hepatocytes
(AIIMS May 2004)
(c) Focal or spotty necrosis
(d) Acidophil bodies
28. All are correctly matched except:
(PGI June 2006)
(a) Hepatitis B - Ground glass hepatocytes
(b) Reye’s syndrome - Ground glass hepatocytes
(c) Alcohol - Mallory bodies
(d) Wilson disease - Mallory bodies
(e) Acute hepatitis - councilman bodies
29. Centrilobular necrosis of liver occurs in:
(UP 2000)
(a) Phosphorus
(b) Phenol
(c) Arsenic
(d) Mercury
30. Most common pathological change seen in acute viral hepatitis is:
(UP 2002)
(a) Ballooning degeneration
(b) Neutrophilic infiltration
(c) Piece meal necrosis
(d) Periportal fatty change
31. Steatosis is NOT seen in:
(UP 2004)
(a) Hepatitis-B infection
(b) Hepatitis-C infection
(c) Alcoholic person
(d) Protein malnutrition
32. Piece meal necrosis is seen in:
(RJ 2001)
(a) Alcoholic hepatitis
(b) Toxic hepatitis
(c) Chronic active hepatitis
(d) Malignancy
33. In pregnancy, which viral infection has maximum mortality?
(RJ 2003)
(a) Hepatitis A Virus
(b) Hepatitis B Virus
(c) Hepatitis C Virus
(d) Hepatitis E Virus.
34. Hepatitis B virus is not associated with:
(RJ 2004)
(a) Fulminant hepatitis
(b) Chronic active hepatitis
(c) Hepatocellular carcinoma
(d) Cholangiocarcinoma
35. Piece meal necrosis is pathognomic of:
(RJ 2006)
(a) Alcoholic Liver disease
(b) Chronic active hepatitis
(c) Toxic hepatitis
(d) Wilson disease
36. Hepatitis E is transmitted by:
(AP 2004)
(a) Blood
(b) Feco-oral
(c) Venereal
(d) All of the above
37. Incubation period of hepatitis B is:
(Bihar 2004)
(a) 6 weeks to 6 months
(b) 6 days to 6 weeks
(c) 6 months to 6 years
(d) More than 6 years
38. Indicator of active multiplication of hepatitis B virus is:
(Bihar 2004)
(a) HBs Ag
(b) HBc Ag
(c) Hbe Ag
(d) Anti-HBs Ab
39. Chronic carrier stage is not found in:
(RJ 2003)
(a) Hepatitis B Virus
(b) Hepatitis C Virus
(c) Both a and b
(d) Hepatitis A Virus
40. A 30 year old woman Aishwarya goes to her gynecologist Dr. Harmeet for a pre-
pregnancy examination. Routine prenatal laboratory testing demonstrates normal
hematological profile with controlled sugar as well negative TORCH infections. She
normal liver function tests with the following profile: HBsAg negative, anti-HBcAg
(-), anti- HBeAg (-), HBV DNA polymerase (-) but anti- HBsAg is positive. Which of
the following likely represents the status of the patient?
(a) Hepatitis B carrier
(b) Recently infected with hepatitis B
(c) Immunized against hepatitis B
(d) Infected with hepatitis B and highly transmissible
41. An eminent hepatobiliary expert Dr. Sarin conducts a study in hepatitis B patients
for which the patients are followed for almost a decade. Their detailed history
regarding the mode of transmission of HBV is taken. A battery of tests including
periodic serologic testing for HBs Ag, anti HBs and anti-HBc, and serum levels of
bilirubin, SOT, SGPT, alkaline phosphatase, and prothrombin time is conducted. Dr.
Sarin finds that a particular group of patients happen to be chronic carriers of HBV.
This finding is most likely to be associated with which of the following modes of
transmission of HBV?
(a) Blood transfusion
(b) Heterosexual transmission
(c) Vertical transmission during childbirth
(d) Needle stick injury
42. A 34-year-old man Bholu presents to his physician with loss of appetite, nausea and
vomiting, and fatigue. Laboratory examination confirms the diagnosis of hepatitis
B, and the man becomes icteric 2 weeks later. This patient may also be particularly
vulnerable to the development of which of the following disorders?
(a) Berry aneurysm
(b) Coronary artery aneurysm
(c) Polyarteritis nodosa
(d) Giant cell arteritis
43. After passing his physical exam, a young army recruit gives urine and blood
samples for further testing. Serum analysis yields elevated ALT, HBsAg, Anti-HBc,
HBeAg, and bilirubin. All other values are normal. Which of the following is the
hepatitis B status of this recruit?
(a) Asymptomatic carrier
(b) Chronic active carrier
(c) Fulminant hepatitis B
(d) Recovered from acute self-limited HBV

MOST RECENT QUESTIONS

44. Which of the following is not a part of HELLP syndrome?


(a) Hemolysis
(b) Elevated liver enzymes
(c) Thrombocytopenia
(d) Retroplacental hemorrhage
45. Nurse got a needle prick injury. Which of the following suggests active phase of
hepatitis?
(a) IgM anti HBc
(b) IgG anti HBc
(c) IgG anti HBs
(d) IgM anti Hbe
46.Maximum ground appearance change is associated with:
(a) Hep A
(b) Hep B
(c) Hep C
(d) Hep E

ALCOHOLIC LIVER DISEASE, NODULAR HYPERPLASIA

47. In a chronic alcoholic all the following may be seen in the liver except:
(AI 2002)
(a) Fatty degeneration
(b) Chronic hepatitis
(c) Granuloma formation
(d) Cholestatic hepatitis
48. Nodular regenerative changes in liver most commonly occur in:
(AIIMS May 2009)
(a) Drugs induced hepatitis
(b) Alcoholic hepatitis
(c) Hepatitis B
(d) Autoimmune hepatitis
49. Mallory hyaline is seen in:
(PGI Dec 2000)
(a) Alcoholic liver disease
(b) Hepatocellular carcinoma
(c) Wilson’s disease
(d) I.C.C. (Indian childhood cirrhosis)
(e) Biliary cirrhosis
50. All of the following are true except:
(DPG &AIIMS Nov 10)
(a) LKM 1 – Autoimmune hepatitis
(b) LKM 2– Drug induced
(c) LKM 1– Chronic hepatitis C
(d) LKM 2– Chronic hepatitis D
51. A chronic alcoholic has an elevated serum alpha fetoprotein levels. Which of the
following neoplasms is most likely seen? (Delhi PG 09 RP)
(a) Prostatic adenocarcinoma
(b) Multiple myeloma
(c) Hepatocellular carcinoma
(d) Glioblastoma multiforme
52. Mallory’s hyaline is seen in: (Delhi PG 2009 RP)
(a) Hepatitis C infection
(b) Amoebic liver abscess
(c) Indian childhood cirrhosis
(d) Autoimmune hepatitis
53. Mallory hyaline body is seen in all except:
(a) Indian childhood cirrhosis
(Delhi PG-2007)
(b) Alcoholism
(c) Secondary biliary cirrhosis
(d) a-1 antitrypsin deficiency
54. Mallory bodies are composed of:
(Karnataka 2009)
(a) Fat droplets
(b) Mitochondria
(c) Lysosomal enzymes
(d) Intermediate filaments
55. Alcoholic hyaline seen in alcoholic liver disease is composed of:
(UP 2007)
(a) Lipofuschin
(b) Eosinophilic intracytoplasmic inclusions
(c) Basophilic intracytoplasmic inclusions
(d) Hemazoin
56. In Alcoholic liver disease, which of the following pigments is deposited in the
hepatocytes?
(UP 2008)
(a) Hemosiderin
(b) Hemoglobin
(c) Lipofuschin
(d) Melanin
57. Mallory bodies are seen in:
(RJ 2006)
(a) Viral hepatitis
(b) Toxic hepatitis
(c) Alcoholic hepatitis
(d) All
58. Mallory bodies are seen in all except:
(AP 2004)
(a) Alcoholic cirrhosis
(b) Biliary cirrhosis
(c) Cardiac cirrhosis
(d) Wilson disease
59. A 46-year-old man, Sushil who has a long history of excessive drinking presents
with signs of alcoholic hepatitis. Microscopic examination of a biopsy of this
patient’s liver reveals irregular eosinophilic hyaline inclusions within the
cytoplasm of the hepatocytes. These eosinophilic inclusions are composed of
which of the following substances?
(a) Immunoglobulin
(b) Excess plasma proteins
(c) Prekeratin intermediate filaments
(d) Basement membrane material
(e) Lipofuscin

MOST RECENT QUESTIONS


60. Which of the following may not cause microvesicular steatosis?
(a) Alcoholic fatty liver
(b) Tetracycline toxicity
(c) Acute fatty liver of pregnancy
(d) Reye’s syndrome
61. The following are true attributes of hepatitis B infection except:
(a) Infants develop chronic infections
(b) HBc Ag in serum is indicative of active infection
(c) Can cause hepatocellular cancer
(d) Interferons are used for treatment
62.Mallory bodies contain:
(a) Vimentin
(b) Cytokeratin
(c) Keratin
(d) Collagen
63. Which of the following is not associated with Mallory hyaline bodies?
(a) Alcoholic liver disease
(b) Primary biliary cirrhosis
(c) Secondary biliary cirrhosis
(d) Indian childhood cirrhosis
64. Given below is the histopathology of liver biopsy of a patient with
hemochromatosis. Which of the following stain has been used?
(AIIMS Nov 2016)

(a) Prussian blue


(b) Alcian blue
(c) Von kossa
(d) Crystal violet

HEPATIC TUMOURS
65. Which of the following most significantly increases the risk of hepatocellular
cancer?
(AIIMS May 2012)
(a) Hep A
(b) Hep B
(c) CMV
(d) EBV
66. True about Fibrolamellar carcinoma of Liver is all, except:
(AIIMS Nov 2001)
(a) Females do not increased incidence than males
(b) Has good prognosis
(c) Not associated with liver cirrhosis
(d) Serum AFP levels are usually > 1000 mg/litre
67. Which of the following is not correct about fibrolamellar variant of hepatocellular
carcinoma? (Delhi PG 2009 RP)
(a) Occurs in young males and females
(b) Hepatitis B virus is an important risk factor
(c) Often has a better prognosis
(d) Is a hard scirrhous tumor
68. Most common primary malignant tumour of liver in adult is:
(UP 2002)
(a) Squamous cell carcinoma
(b) Hepatoblastoma
(c) Hepatocellular carcinoma
(d) Hepatoma
69. Which is not correct about hepatocellular carcinoma?
(a) More in females
(Jharkhand 2004)
(b) Rise of AFP noted
(c) Has stronger propensity to invade vascular channels.
(d) Chronic HBV has high rate of hepatocellular carcinoma
70. A young woman Ms Shaano who is otherwise normal goes for an annual
examination in a nursing home. Her blood investigations reveal hemoglobin is 15
gm/dl, TLC is 7,000/mm3, ESR is 12 mm/hr. Her kidney and liver function tests are
also normal. She undergoes a radiological scanning too. Dr. Sethi, the radiologist
describes her findings to be normal except a mass in the right lobe of the liver. A
biopsy is taken which confirms the diagnosis of a liver adenoma. Which of the
following is likely to be associated with this lesion?
(a) Polycythemia vera
(b) Hepatitis B
(c) Oral contraceptives
(d) Polyvinyl chloride

MOST RECENT QUESTIONS

71. Thorium induced tumor:


(a) Angiosarcoma of liver
(b) Renal cell carcinoma
(c) Lymphoma
(d) Astrocytoma
72. Commonest benign tumor of liver is:
(a) Hamartoma
(b) Hemangioma
(c) Adenoma
(d) Nodular focal hyperplasia
73. Which of the following organ has the presence of infarct of Zahn?
(a) Kidney
(b) Heart
(c) Liver
(d) Spleen
74. Solution currently used for liver preservation for transplant is:
(a) UW solution
(b) IGL solution
(c) Kyoto ET solution
(d) Ross Marshal Citrate solution
75 Most common malignant mesenchymal tumor of liver is:
(a) HCC
(b) Cholangiocarcinoma
(c) Angiosarcoma
(d) Hepatoblastoma

BILIARY TRACT DISORDER

76. Most common site of Cholangiocarcinoma is?


(a) Distal biliary tree
(AIIMS Nov 2008)
(b) Hilum
(c) Intrahepatic biliary duct
(d) Multifocal
77. Cholangiocarcinoma of liver is caused by:
(UP 2008)
(a) Hepatitis B infection
(b) Cirrhosis of liver
(c) Antitrypsin deficiency
(d) Clonorchis sinensis infection
78. Most common bile duct tumor is:
(RJ 2000)
(a) Adenocarcinoma
(b) Squamous cell cancer
(c) Transitional cell carcinoma
(d) All
79. A 50-year-old male film actor Sallu Kahn looses weight rapidly for one of his
forthcoming films. He experiences occasional abdominal discomfort few days after
that and guided by his physician, he undergoes a radiological scan (HIDA scan) is
shown to have slow and incomplete gallbladder emptying in response to
cholecystokinin stimulation. This patient is likely to develop which of the
following?
(a) Black pigment stones
(b) Brown pigment stones
(c) Biliary sludge
(d) Phospholipid stones
80. A middle aged woman comes to the emergency room complaining of severe, right-
sided abdominal pain, fever, and chills for the past several hours. She has a history
of gallstones and her family doctor recommended a cholecystectomy after a
similar episode several months ago. Upon examination, she has a temperature of
102.7°F (39.3°C), is tender in the right upper quadrant, and is visibly jaundiced. Her
white blood count is 18,000/mm3. In which of the following locations is a gallstone
most likely lodged in this patient?
(a) Common bile duct
(b) Cystic duct
(c) Fundus of gallbladder
(d) Proximal duodenum

MOST RECENT QUESTIONS


81. All of the following are risk factors for carcinoma gall-bladder, except:
(a) Typhoid carriers
(b) Adenomatous gall bladder polyps
(c) Choledochal cysts
(d) Oral contraceptives
82. Focal diffuse gall bladder wall thickening with comet tail reverberation artifacts on
USG are seen in:
(a) Adenomyomatosis of gallbladder
(b) Carcinoma gallbladder
(c) Adenomatous Polyps
(d) Xanthogranulomatous gallbladder
83. Onion skin fibrosis of the common bile duct is:
(a) Primary biliary cirrhosis
(b) Primary sclerosing cholangitis
(c) Extrahepatic biliary fibrosis
(d) Congenital hepatic fibrosis
84. Which is risk factor for cholangiocarcinoma:
(a) Obesity
(b) Primary sclerosing cholangitis
(c) Salmonella carrier state
(d) HBV infection
85. Klatskin tumor is:
(a) Nodular type of cholangiocarcinoma
(b) Fibrolamellar hepatocellular carcinoma
(c) Gall bladder carcinoma
(d) Hepatocellular carcinoma

MISCELLANEOUS

86. True about hemochromatosis is:


(AI 2009)
(a) Complete penetrance
(b) Autosomal recessive
(c) Phlebotomy leads to cure
(d) More common in females
87. All are seen in hemochromatosis except:
(AI 2008)
(a) Hypogonadism
(b) Arthropathy
(c) Bronze diabetes
(d) Desferrioxamine is the treatment of choice
88. Liver granulomas may be associated with all of the following except:
(AI 2002)
(a) Candida
(b) Halothane
(c) Sarcoidosis
(d) Hepatic metastasis
89. Histological finding in Reye’s syndrome is:
(a) Budding and branching of mitochondria
(b) Swelling of endoplasmic reticulum
(c) Para-nuclear micro-dense deposits
(d) Glycogen depletion
90. Finding on histopathological examination of liver in case of malaria is:
(AIIMS May 2007)
(a) Microabscess formation
(b) Kupffer’s cell hyperplasia with macrophage infiltration around periportal area laden
with pigments.
(c) Non caseating granuloma
(d) Non specific finding of neutrophilic infiltration
91. Pigmentation in the liver is caused by all except:
(PGI Dec 01)
(a) Lipofuscin
(b) Pseudomelanin
(c) Wilson’s disease
(d) Malarial pigment
(e) Bile pigment
92. True statements about a-l antitrypsin deficiency is:
(a) Autosomal dominant disease
(PGI June 2003)
(b) Emphysema
(c) Fibrosis of portal tract
(d) Diastase resistant positive hepatocytes
(e) Orcein positive granules
93. “Kayser-Fleischer ring” is seen in:
(UP 2007)
(a) Wilson’s disease
(b) a-1 antitrypsin deficiency
(c) Hemochromatosis
(d) Primary biliary cirrhosis
94. All are true about Wilson’s disease except:
(RJ 2000,2004,2006)
(a) ↑Liver Cu
(b) ↑Urine Cu
(c) ↑Ceruloplasmin
(d) ↑Serum Cu
95. α-1 antitrypsin deficiency causes:
(AP 2000)
(a) Congenital cystic fibrosis
(b) Neonatal hepatitis
(c) Pulmonary fibrosis
(d) All of the above
96. Centrilobular necrosis is seen in:
(Kolkata 2005)
(a) CCl4
(b) White phosphorus
(c) Yellow fever
(d) Eclampsia
97. A retired man Pradyuman R complains of vague abdominal pain since last 6
months. One fine day, he experienced acute chest pain with dyspnea. He was
rushed to the emergency of Gangaram Hospital where his chest and abdomen CT
scans demonstrate a pulmonary embolus. The radiologist Dr. Sandeep Goel also
notices a 7.5 cm mass in the body of the pancreas. His blood investigations reveal
elevated levels of CEA and CA 19-9. Which of the following genetic mutations is
likely to be associated with this pancreatic mass?
(a) BRCA -2
(b) K-RAS
(c) PRSS1
(d) SPINK1

MOST RECENT QUESTIONS

98. Copper is mainly transported by:


(a) Albumin
(b) Haptoglobin
(c) Ceruloplasmin
(d) Globulin
99. Liver in hemochromatosis is stained by which of the following stain?
(a)Perls iron stain
(b)Alcian blue
(c) Congo Red
(d) Masson trichome
100. Bronze diabetes is seen in:
(a) Wilson’s disease
(b) Sarcoidosis
(c) Lead intoxication
(d) Hemochromatosis
101. Gene of Wilsons disease is:
(a) ATP 7A
(b) ATP 7B
(c) ADP 7A
(d) ADP 7B
102. True about Wilson’s disease is:
(a) Increased serum ceruloplasmin
(b) Coomb’s positive hemolytic anemia
(c) Sensory deficit
(d) Autosomal recessive
103. Gene for Wilson is present on which of the following chromosome?
(a) 10
(b) 13
(c) 15
(d) 17
1. Ans. (c) Gilbert syndrome
(Ref: Harrison 17th/1929, 9/e p854)
2. Ans. (d) Biliary cirrhosis
(Ref: Robbins 8th/868, 9/e p853-854)
Biliary cirrhosis is characterized by conjugated hyperbilirubinemia. Rest all the mentioned
options cause unconjugated hyperbilirubinemia.
3. Ans. (c) The conjugation process of bilirubin in liver remains operative without any
interference
(Ref: Harrison 17th/262-3, Ganong 22nd/503, 9/e p853)
Post hepatic jaundice is due to impaired excretion of conjugated bilirubin as a result of
obstruction. However, the process of conjugation is not interfered with.
4. Ans. (b) Vitamin A storage
(Ref: Robbins’ 7th/878, 9/e p436)
The hepatic stellate cells (also called Ito cells) are of mesenchymal origin, found in space of Disse.
The stellate cells play a role in the storage and metabolism of vitamin A and are transformed into
collagen producing myofibroblasts when there is inflammation and cause fibrosis of liver.

5. Ans. (a) Rotor’s syndrome


(Ref: Robbins 9/e p854, 8th/841, Harrison 17th/26)
6. Ans. (d) 0.80
(Ref: Robbins 9/e p853, Harrison 17th/262)
Indirect hyperbilirubinemia is called when unconjugated bilirubin is 85% or more of the
total bilirubin whereas direct hyperbilirubinemia corresponds to conjugated
bilirubin more than 15% of total.
7. Ans. (b) Antimitochondrial antibody
(Ref: Robbins 8th/867, 9th/858)
The findings in the stem of the question are suggestive of clinical condition of primary biliary
cirrhosis. It is seen in middle aged women in which the jaundice may progress due to
progressive intrahepatic destruction. In these patients, there is presence of anti
mitochondrial antibodies.
Other options
• Anti parietal antibodies are seen in pernicious anemia.
• Anti-centromere antibody is seen in systemic sclerosis.
• Anti-ribonucleoprotein antibody is seen in different connective tissue disorders.

The following disorders present with insidious onset of features of obstructive jaundice like
pruritus, jaundice, malaise dark urine, light stools and hepatosplenomegaly.
Features of the Bile Duct Disorders
Primary Biliary Cirrhosis Secondary Biliary Primary Sclerosing
Cirrhosis Cholangitis
Cause Possibly autoimmune Biliary atresia, gallstones, Autoimmune; usually
stricture, cancer of associated with inflammatory
pancreatic head bowel disease
Sex predilection Female to male: 6:1 None Female to male: 1:2
Distribution Intrahepatic bile duct Extrahepatic bile duct Extra + intra hepatic duct
obstruction obstruction affected
Lab. findings Same as secondary biliary ↑conjugated bilirubin, Same as secondary biliary
cirrhosis with elevated serum ↑serum alkaline cirrhosis with
IgM antimitochondrial antibody phosphatase with increased hypergammaglobulinemia
cholesterol (↑IgM), atypical p-ANCA (+)
Histological Dense lymphocytic infiltrate in Bile stasis in bile ducts, bile Periductal portal tract fibrosis
findings portal tracts with ductules proliferation with (onion skin fibrosis),
granulomatous destruction of surrounding neutrophils, segmental stenosis of extra
bile ducts portal tract edema and intrahepatic bile ducts
An important feature is Atypical p-ANCA (+) is seen with primary sclerosing cholangitis but that this antibody is
directed against a nuclear envelope protein and not myeloperoxidase seen with typical p-ANCA antibodies.

8. Ans. (a) Production of vitamin K


(Ref: Robbins 9/e p 822)

• Vitamin K is produced by the bacteria of gut and is used by liver for gamma carboxylation of factor
2/7/9/10.
• Liver produces albumin which falls in liver cirrhosis producing ascites/edema
• Ammonia is combined with carbon dioxide to produce urea which in turn is excreted by the liver.
• The cytochrome p450 is responsible for metabolism of drugs.

9. Ans. (d) Anti-mitochondrial antibody


(Ref: Robbins 8/e p867, 9/e p858)
Friends, please do not get confused between options “b” and “d”.
• Antimicrosomal antibodies are autoantibodies, directed against the microsomes (in
particular peroxidase) of the thyroid cells leading to thyroiditis, tissue damage, and
disruption of thyroid function. It is associated with autoimmune conditions like
Hashimoto’s thyroiditis and Grave’s disease.
• Anti-mitochondrial antibody is seen in primary biliary cirrhosis.
Key points for primary biliary cirrhosis

• Non-suppurative, inflammatory destruction of medium-sized intrahepaticQ bile ducts.


• Female to male ratio 6 : 1
• Serum alkaline phosphatase and cholesterol are almost always elevatedQ, even at onset;
hyperbilirubinemia is a late development and usually signifies incipient hepatic decompensation.
• Antimitochondrial antibodiesQ are present in 90% to 95% of patients.
• Associated with increased risk to develop hepatocellular carcinomasQ.
• The major cause of death is liver failureQ,

10. Ans. (a) Criggler–Najjar Type I


(Ref: Robbins 9/e p853)
11. Ans. (d) Hemolytic anemia
(Ref: Robbins 8th/840-1, 9/e p853)
The patient has unconjugated hyperbilirubinemia which can result in the given options from
hemolytic anemia. Option (a), Cholelithiasis results in conjugated hyperbilirubinemia.
Option (c) and (d), hepatitis and micronodular cirrhosis can present with both unconjugated and
conjugated hyperbilirubinemia though conjugated bilirubin predominates.
12. Ans. (d) Bridging fibrosis
(Ref: Tropical Hepato-Gastroenterology by Tandon Elsevier 1st/391, Histopathology of
the liver in non-cirrhotic portal hypertension of unknown etiology Histopathology
1996;28:195–204 by Nakanuma Y et al., Non-cirrhotic portalfibrosis: current
concept and management by Sarin SK, Kapoor D in J Gastroenterol Hepatol 2002;
17:526-534)
The Following is a Figure explaining the pathogenesis of NCPF along with Histologically
findings in NCPF
• Portal fibrosis (intra portal
but not bridging fibrosis)
• Portal vein sclerosis
• Subcapsular scarring
• Pseudolobulation
• Portal tract edema and
lymphocytic infilteration
• Intimal fibroelastosis of
medium sized portal veins;
obliterative portal
venopathy (characteristic
lesion)
• Sclerosis and obliteration of
portal vein radicals
• Atrophy of liver parenchyma
with no regenerative capacity
• Collagen deposition in the
space of Disse

So, the answer is clear from the above mentioned features given collectively in all the
references mentioned above. Bridging fibrosis is not seen in NCPF.
13. Ans. (c) Chronic venous congestion of liver
(Ref: Robbins 7th/122-3, 9/e p864)
Congestion is a passive process resulting from impaired outflow from a tissue. In long standing
or chronic venous congestion, the stasis results in chronic hypoxia resulting in
parenchymal cell death. The central part of hepatic lobule is red brown and slightly
depressed (due to loss of cells) and is accentuated against surrounding zone of
uncongested tan liver. This is called nutmeg liver. Microscopically, there is presence of
hemosiderin laden macrophages.
In severe cases (as with heart failure); there may be presence of hepatic fibrosis which is
called cardiac cirrhosis.
14. Ans. (b) Hepatic stellate cells
(Ref: Robbins 9/e p822, 8th/837; 7th/883)
15. Ans. (b) Asterixis
(Ref: Robbins 8th/836, 9/e p826)
• Asterixis is a flapping tremor of the hands associated with hepatic encephalopathy.
Failure of the liver to detoxify metabolites absorbed from the gastrointestinal tract
results in accumulation of nitrogenous wastes that are neurotoxic.
• Disturbed mental status is also attributed to production of false neurotransmitters,
increased CNS sensitivity to GABA, reduced activity of urea cycle enzymes due to
zinc deficiency and swelling of astrocytes. Ref 2013 (CMDT).
• Caput medusa results from dilation of the periumbilical venous collaterals as a result
of portal hypertension and opening of portal-caval anastomoses. Other findings like
palmar erythema, capillary telangiectasias, and gynecomastia results from the
inability of the liver to metabolize estrogen leading to hyperestrinism.
16. Ans. (b) Hepatic venography
(Ref: Robbins 8th/872-873, 9/e p863-864)
The clinical presentation is most consistent with Budd-Chiari syndrome (hepatic vein
obstruction), which may occur as a complication of thrombogenic and myeloproliferative
disorders including polycythemia vera. The presentation in the question is the most
common. Hepatic venography is the best technique of those listed to demonstrate the
occlusion of the hepatic venous system.
• Endoscopic retrograde cholangiopancreatography (choice A) is most useful in demonstrating
lesions of the biliary tree.
• Serum alpha fetoprotein (choice C) is a marker for hepatocellular carcinoma.
• Serum iron studies (choice D) are useful when considering hemochromatosis as a cause of
cirrhosis.

17. Ans. (b) Viral hepatitis


(Ref: Clinical Hepatology: Principles and Practice of Hepatobiliary Diseases: Volume 2
Springer publications p952)
• Common causes of micronodular cirrhosis (nodules <3 mm in diameter) Alcohol,
Metabolic, Hemachromatosis, Wilson’s Disease, Indian childhood cirrhosis, chronic
venous outflow tract obstruction, bile duct obstruction
• Common causes of macronodular cirrhosis (nodules >3 mm in diameter) Viruses,
Toxins, Poisoning
Budd-Chiari syndrome is characterized by the obstruction of two or more major hepatic veins
produce liver enlargement, pain, and ascites. So, it is a cause of hepatic venous outflow
obstruction.
18. Ans. (a) Esophagus
(Ref: Robbins 9/e p830)
19. Ans. (d) Portal-systemic venous shunting
(Ref: Robbins 8th/839, 9/e p830, Harrison 18th/2600)
Portal-systemic venous shunting leads to encephalopathy in end-stage cirrhosis. It is also
contributing to other features like esophageal varices, rectal haemorrhoids, and distention
of periumbilical venous collaterals. Other factors like hypoalbuminemia, increased hepatic
lymph formation and increased portal venous pressure mentioned as the options
contribute to the development of ascites, but not to encephalopathy.
20. Ans. (a) Right sided heart failure
(Ref: Robbins 9/e p864)
• Right-sided cardiac decompensation leads to passive congestion of the liver. In long
standing or chronic venous congestion, the central part of hepatic lobule is red
brown and slightly depressed (due to loss of cells) and is accentuated against
surrounding zone of uncongested tan liver. This is called nutmeg liver.
• In severe cases this is called as cardiac sclerosis or cardiac cirrhosis.
21. Ans. (a) Right sided heart failure explained earlier
(Ref: Robbins 9/e p864)
22. Ans. (b) PDGF
(Ref: Robbins 9/e p823)
In the pathogenesis of cirrhosis, proliferation of hepatic stellate cells and their activation
into myofibroblasts is initiated by a series of changes that include an increase in the
expression of platelet-derived growth factor receptor β (PDGFR-β) in the stellate cells.
23. Ans. (c) preCore mutant HBV
(Ref: Robbins 8th/846, Harrison 17th/1935-6)
The precore region directs the release of HBeAg towards secretion in the blood. Uncommonly,
mutated strains called precore mutants of HBV emerge that do not produce HBeAg but
are replication competent and express HBcAg. In these patients, the HBeAg may be
undetectable despite the presence of HBV viral load. Another mutation in the core
promoter region can also lead to an HBeAg negative phenotype. Clinically both these
conditions are characterized by the presence of elevated liver enzymes and active viral
multiplication is indicated only by the high levels of DNA polymerase.
Additional info
Another important mutation seen in HBV is “escape mutants” due to amino acid substitution causing
a conformational change in HBs Ag resulting in loss of neutralizing activity by anti-HBs. It is seen
in association with active and passive immunization and in liver transplant patients. In both these
conditions, there is increased concentration of anti HBs leading to mutation in the virus so that it can
escape from the protective effect of anti-HBs.

24. Ans. (d) Chronic alcoholism


(Ref: Robbins 7th/903-4, 9/e p 842, Harrisons 17th/1982)
Alcoholic liver disease (ALD) is the most common cause of fatty liver. So, Fatty liver is
characteristically seen in chronic alcoholism. However, now many other causes of fatty
liver have also been elucidated known as Non-Alcoholic fatty liver disease (NAFLD) or
Non-alcoholic steatohepatitis (NASH). These are explained in the chapter review.
• Sternberg 4th mentions “although fatty change has been reported as a common
feature of hepatits C, the degree of fatty changes is usually minimal and absence is
not unusual.“
25. Ans. (c) Acute viral hepatitis
(Ref: Robbin’s 7th/899, 9/e p823; Harrison 17th/1929)
Councilman bodies are feature of acute hepatitis. These are associated with the cellular
phenomers of opposit.
26. Ans. (d) Grading refers to the extent of necrosis and inflammation
(Ref: Harrison 17th/1955), Sternberg’s diagnostic surgical pathology 4th/1682
The new classification system of hepatitis is based on its etiology, grade or stage. Grading
refers to the assessment of necroinflammatory activity whereas the staging refers to
degree of progression.
Concept
Regarding option ‘c’, Sternberg writes clearly ‘although fatty change has been reported as a common
feature of hepatits C, the degree of fatty changes is usually minimal and absence is not unusual. Fatty
change due to virus is most likely associated with genotype 3. Biopsy in hepatitis B and C is done for
purpose of staging and grading and not for diagnosis’.

27. Ans. (b) Ground glass hepatocytes

(Ref: Harrison 17th/1727; Robbins 7th/899, 8th/851, 9/e p837)


Ground glass hepatocytes are large hepatocytes containing surface antigen. Their cytoplasm is
ground glass in appearance. These cells are a feature of chronic hepatitis and not acute
hepatitis.
28. Ans. (b) Reye’s syndrome—ground glass appearance of hepatocytes.
(Ref: Robbins 9/e p841, H Mohan 6th-602)
In acute hepatitis B, hepatocytes show ground glass appearance.
• In Reye’s syndrome, hepatocytes show microvesicular fatty change.
• Mallory bodies are seen in alcoholic hepatitis, primary biliary cirrhosis, non-alcoholic
fatty liver disease, Wilson’s disease, chronic cholestatic jaundice and hepatocellular
carcinoma.
29. Ans. (b) Phenol
(Ref: Robbins 7th/903)
30. Ans. (a) Ballooning degeneration
(Ref: Robbins 9/e p838)
31. Ans. (a) Hepatitis-B infection
(Ref: Robbins 9/e p841-842)
32. Ans. (c) Chronic active hepatitis
(Ref: Robbins 8th/852-3)
33. Ans. (d) Hepatitis E Virus
(Ref: Robbins 9/e p835)
34. Ans. (d) Cholangiocarcinoma
(Ref: Robbins 9/e p833)
35. Ans. (b) Chronic active hepatitis
(Ref: Robbins 8th/851-2)
36. Ans. (b) Feco-oral
(Ref: Robbins 9/e p835)
37. Ans. (a) 6 weeks to 6 months
(Ref: Robbins 9/e p831-832)
38. Ans. (c) HBeAg
(Ref: Robbins 9/e p832, 8th/846; 7th/893)
39. Ans. (d) Hepatitis A Virus
(Ref: Robbins 9/e p831)
40. Ans. (c) Immunized against hepatitis B
(Ref: Robbins 9/e 832-833, 8th/846, Harrison 18th/2550)
Ms Aishwarya is positive only for the antibody to the hepatitis B antigen. This is suggestive of
her being vaccinated for hepatitis B virus (HBV). The vaccine consists of recombinantly
produced HBV surface antigen (HBsAg) alone and the antibodies to this protein provide
immunity.
HBsAg would be seen in the serum within the first 3-4 months after initial infection. Antibodies
to the core protein (anit-HBcAg) appear during acute illness and between the
disappearance of HBsAg and the appearance of anti-HBsAg, the “window period.” Anti-
HBeAg appears during the window period as well. HBeAg and HBV DNA polymerase are
the indices of infectivity.
Nutshell of Hepatitis B Serology
HBs Ag IgM anti HBc IgG anti HBc IgG anti HBs
Acute HBV infection + + – –
Window period – + – –
Chronic infection + +/- + –
Prior infection – – + +
Immunization – – – +

Also know: The presence of HBe Ag denotes high infectivity and its absence denotes
low infectivity.
41. Ans. (c) Vertical transmission during childbirth
(Ref: Robbins 8th/845, 9/e p832)
Hepatitis B infection is not commonly transmitted through blood transfusion now (due to
mandatory screening of blood and its products), heterosexual transmission and needle
stick injury (much more chances in comparison to HIV and HCV).
Important concept
• In adults, the viral hepatitis develops because they are immunocompetent, so, HBV induced T
cells induce apoptosis of infected liver cells.
• Vertical transmission during childbirth is responsible for HBV chronic carrier stage. This is
attributed to the fact that unlike the adults, the immune responses in the neonatal period are not
fully developed thereby preventing the development of hepatitis.
• Clinical significance of high carrier rate is increased risk of development of hepatocellular
cancer.

42. Ans. (c) Polyarteritis nodosa


(Ref: Robbins 9/e p509)
Thirty percent of patients with polyarteritis nodosa have hepatitis B antigenemia. Polyarteritis is
a systemic necrotizing vasculitis that can be difficult to diagnose, since the vascular
involvement is typically widely scattered, and the specific symptoms depend on the
specific vessels (small- to medium-sized arteries) involved. Patients typically present with
low-grade fever, weakness, and weight loss. Abdominal pain, hematuria, renal failure,
hypertension, and leukocytosis may occur. The disease is frequently fatal if untreated.
43. Ans. (b) Chronic active carrier
(Ref: Robbins 9/e 837)
The presence of elevated ALT, HBsAg, anti-HBc, HBeAg, and bilirubin all point to active
hepatitis B.
• An asymptomatic carrier (choice A) does not have elevated ALT and bilirubin.
• The absence of findings on physical examination rules out fulminant hepatitis B
(choice C).
• Recovery from acute self-limited HBV (choice D) is associated with the presence of
anti-HBs and the decrease in HBsAg and HBeAg.
44. Ans. (d) Retroplacental hemorrhage
(Robbins 9th/ 866)
The subclinical hepatic disease may be the primary manifestation of preeclampsia, as part of a syndrome of
hemolysis, elevated liver enzymes, and low platelets, dubbed the HELLP syndrome.

45. Ans. (b) IgM anti HBc


(Ref: Robbins 9th/833)
IgM anti-HBc antibody becomes detectable in serum shortly before the onset of symptoms,
concurrent with the onset of elevated serum aminotransferase levels (indicative of
hepatocyte destruction).
46. Ans. (b) Hep B
(Ref: Robbins 9/e p837)
In chronic hepatitis B, “ground-glass” hepatocytes (cells with endoplasmic reticulum
swollen by HBsAgQ) are a diagnostic hallmark.
47. Ans. (c) Granuloma formation (Ref: Robbins 7th/94 -
905, 9/e 842 Harrison - 17th/1970)
Spectrum of alcoholic liver disease includes:
• Fatty Liver (Hepatic steatosis)
• Alcoholic hepatitis - Hallmark of alcoholic hepatitis is hepatocyte injury
characterized by ballooning degeneration, spotty necrosis, polymorphonuclear
infiltrate and fibrosis in the perivenular and perisinusoidal space of Disse. Mallory
bodies may also be present.
• Alcoholic Cirrhosis

Granuloma is not seen in alcoholic liver disease.

48. Ans. (a) Drugs induced hepatitis


(Ref: Robbins 8th/876, Diseases of the liver and biliary system by Sheila Sherlock,
Sheila Sherlock (Dame.), James S. Dooley Blackwell Science 11th/530)
Nodular regenerative hyperplasia: summary from Robbins and Sherlock
• Associated with the development of
portal hypertension and its clinical
manifestations.

• Occurs in conditions affecting intra


hepatic blood flow like rheumatoid
arthritis (most commonly), Felty
syndrome, myeloproliferative
disorders, hyperviscosity
syndromes, solid organ (particularly
renal and liver) transplantation,
bone marrow transplantation, HIV
infection (Robbins), vasculitic
conditions and drugs (anabolic
steroids and cytotoxics).

• Characteristically, there is absence


of fibrosis in this condition.

49. Ans. (a) Alcoholic liver disease; (b) Hepatocellular carcinoma; (c) Wilson’s disease;
(d) ICC (Indian childhood cirrhosis); (e) Biliary cirrhosis
(Ref: Robbins’ 7th/905, 9/e p843)
Mallory bodies- scattered hepatocytes accumulate tangled skeins of cytokeratin intermediate
filaments and other proteins, visible as eosinophilic cytoplasmic inclusions in
degenerating hepatocytes.
50. Ans. (d) LKM 2- Chronic hepatitis D
(Ref: Harrison 17th/1956, 1968 CMDT/595-600, Robbin 839-840)
LKM stand for liver kidney microsomal antibodies
Type of LKM antibody Associated conditions

Anti LKM 1 antibodies Chronic hepatitis C, Autoimmune hepatitis type 2

Anti LKM 2 antibodies Drug induced hepatitis

Anti LKM 3 antibodies Chronic hepatitis D, type 2 autoimmune hepatitis (rarely)

51. Ans. (c) Hepatocellular carcinoma


(Ref: Robbins 8th/879-880, 9/e p873)
Elevated levels of serum α-fetoprotein are found in 50 to 75% of patients with HCC.
False-positive results are encountered with yolk-sac tumors and many non-neoplastic
conditions, including cirrhosis, massive liver necrosis, chronic hepatitis, normal
pregnancy, fetal distress or death, and fetal neural tube defects such as anencephaly and
spina bifida.
Concept
The staining for Glypican-3 is used to distinguish early hepatocellular carcinoma from a dysplastic
nodule. Other tests cannot be used because the levels of serum α-fetoprotein are inconclusive in this
condition.

52. Ans. (c) Indian childhood cirrhosis


(Ref: Harsh Mohan 6th/621-622)
53. Ans. (c) Secondary biliary cirrhosis
(Ref: Robbin 7th/904, 9/e p843)
54. Ans. (d) Intermediate filaments
(Ref: Robbins 9/e p843)
Scattered hepatocytes accumulate tangled skeins of cytokeratin intermediate filaments and
other proteins, visible as eosinophilic cytoplasmic inclusions in degenerating hepatocytes
called as Mallory bodies. These inclusions are a characteristic but not specific feature of
alcoholic liver disease.
55. Ans. (b) Eosinophilic intracytoplasmic inclusions
(Ref: Robbins 8th/858; 7th/612, 9/e p843)
56. Ans. (a) Hemosiderin
(Ref: Robbins 8th/857; 7th/905)
57. Ans. (c) Alcoholic hepatitis
(Ref: Robbins 9/e p843, 8th/858, 7th/905)
58. Ans. (c) Cardiac cirrhosis
(Ref: Robbins 9/e p843)
59. Ans. (c) Prekeratin intermediate filaments;
(Ref: Robbins 7th/34, 37- 41, 423, 905 9/e p843)
• Hyaline is a nonspecific term that is used to describe any material, inside or outside
the cell, that stains a red homogeneous color with the routine H&E stain.
• Alcoholic hyaline inclusions (Mallory bodies) are irregular eosinophilic hyaline
inclusions that are found within the cytoplasm of hepatocytes. These are composed
of pre-keratin intermediate filaments. They are a nonspecific finding and can be
found in patients with several diseases other than alcoholic hepatitis, such as
Wilson’s disease, and in patients who have undergone bypass operations for morbid
obesity.
Other options
• Immunoglobulins may form intracytoplasmic or extracellular oval hyaline bodies called Russell
bodies.
• Excess plasma proteins may form hyaline droplets in proximal renal tubular epithelial cells or
hyaline membranes in the alveoli of the lungs.
• The hyaline found in the walls of arterioles of kidneys in patients with benign nephrosclerosis is
composed of basement membranes and precipitated plasma proteins.

60. Ans. (a) Alcoholic fatty liver (Ref: Robbins 9/e p830, 841,
8/e p857-8, Schiffs Diseases of the Liver)
Steatosis is considered to be microvesiuclar when multiple small cytoplasmic vacuoles tend to
leave the nucleus centrally placed. In contrast, macrovesicular steatosis has a single
large fat vacuole which displaces the nucleus to the periphery.
Causes of Microvasicular steatosis

• Reye syndrome, acute fatty liver of pregnancy, drugs (tetracycline, valproate, aspirin, nucleoside
analogs of anti HIV drugs)

Causes of Macrovasicular steatosis

• Malnutrition, diabetes, obesity, malabsorption, steroid therapy, some metabolic diseases


Alcohol intake can be associated with both microvesicular (initially) and macrovesicular
steatosis (later on continued drinking).
61. Ans (b) HBc Ag in serum is indicative of active infection
(Ref: Robbins 9/e p833)
HBe Ag and not HBc Ag in serum is indicative of active infection.

• Persistence of HBeAg is an important indicator of continued viral replication, infectivity, and probable
progression to chronic hepatitis.

62. Ans. (b) Cytokeratin


(Ref: Robbins 8/e p858, 9/e p843)
Mallory bodies are visible as eosinophilic cytoplasmic clumps in hepatocytes. They are
composed of tangled skeins of cytokeratin intermediate filamentsQ such as cytokeratin
8 and 18, in complex with other proteins such as ubiquitin.
63. Ans. (c) Secondary biliary cirrhosis
(Ref: Robbins 9/e 843, 8/e 858, 7/e p905)
Mallory Hyaline bodies are seen in conditions memorized by Mnemonic: New Indian WATCH.
64. Ans. (a) Prussian blue
(Ref: Robbins 9/e p849)
Hemochromatosis is characterized by excess of iron deposition in different tissues. This can be
demonstrated with the help of Prussian blue stain (Perls reaction).
65. Ans. (b) Hep B
(Ref: Robbins 8th/878-9, 9/e p870)
Risk factors for development of hepatocellular cancer (HCC)

• Chronic Hepatitis (Hepatitis B, Hepatitis C)


• Alcoholism
• Aflatoxins (due to Aspergillus flavus infection of peanuts, grains)
• Tyrosinemia
• Hereditary hemochromatosis

66. Ans. (d) Serum AFP levels are usually greater than 1000 mg/liter
(Ref: Robbins 7th/925, 9/e p873)
As discussed in text fibrolammelar cancer is not associated with elevated AFP levels.
67. Ans. (b) Hepatitis B is an important risk factor
(Ref: Robbins 8th/879, 9/e p873)
68. Ans. (c) Hepatocellular carcinoma
(Ref: Robbins 8th/878; 7th/922-4, 9/e p869)
69. Ans. (a) More in females
(Ref: Robbins 9/e p873)
70. Ans. (c) Oral contraceptives
(Ref: Robbins 8th/877, 9/e p868)
Liver adenomas are benign liver tumors commonly associated with oral contraceptive use in
young women (usually 3-4th decade of life). They may resemble hepatocellular
carcinoma. If they are subscapular, they can rupture, causing intra-abdominal
haemorrhage leading to acute abdominal pain.
• Hepatitis B may lead to hepatocellular carcinoma.
• Polycythemia vera is associated with thrombosis of the hepatic veins.
• Polyvinyl chloride, thorotrast (a contrast material) and arsenic are the risk factors
for development of angiosarcoma of the liver. Immunohistochemical staining of
these tumor cells is positive for the CD 31 cell marker.
71. Ans. (a) Angiosarcoma of liver
(Ref: Robbins 8/e p877, 9/e p875)
Angiosarcoma of the liver is a highly aggressive tumor which is associated with exposure
to:

• Vinyl chlorideQ,
• ArsenicQ, or
• ThorotrastQ,

Thorotrast is a suspension containing particles of the radioactive compound thorium


dioxide. It emits alpha particles due to which it has been found to be extremely
carcinogenic.
72. Ans. (b) Hemangioma
(Ref: Robbins 9/e p867, 8/e p876, 7/e p922)
• Cavernous hemangiomas are the most common benign liver tumours.
73. Ans. (c) Liver
(Ref: Robbins 9/e p863)
Intrahepatic portal vein radicles may be obstructed by acute thrombosis. The thrombosis
does not cause ischemic infarction but instead results in a sharply demarcated area of
red-blue discoloration called infarct of Zahn. There is no necrosis, only severe
hepatocellular atrophy and marked stasis in distended sinusoids.
74. Ans. (a) UW solution
(Ref: Transplantation E-Book/99)
University of Wisconsin (UW) solution has been known as the standard solution for liver
graft preservation.
75. Ans. (c) Angiosarcoma
(Ref: Robbins 9/e p875)
Angiosarcoma of the liver resembles those occurring elsewhere and has historical
associations with vinyl chloride, arsenic, or Thorotrast.
76. Ans. (b) Hilum
(Ref: Robbins 9/e 874, 8th/880, Harrison 17th/585)
The commonest location of the cholangiocarcinoma is at the hilum. Klatskin tumors are
located at the junction of the right and the left hepatic ducts.
77. Ans. (d) Clonorchis sinensis infection
(Ref: Robbins 9/e p874, 8th/880; 7th/671)
78. Ans. (a) Adenocarcinoma
(Ref: Robbins 9/e p874, 8th/888, 7th/926-927)
79. Ans. (c) Biliary sludge
(Ref: Robbins 9/e p876, 8th/882-4. Harrison 18th/2617-9)
The demonstration of slow or incomplete gallbladder emptying in response to cholecystokinin
stimulation is called gallbladder hypomotility. This is usually associated with risk factors
like pregnancy, rapid weight loss prolonged use of total parenteral nutrition or
octreotide, and high spinal cord injuries.
Gallbladder hypomotility frequently results in the formation of biliary sludge, which results from
bile precipitation. Biliary sludge typically contains cholesterol monohydrate crystals,
calcium bilirubinate, and mucus and is a known precursor to stone formation.
Complications such as acute cholecystitis occur in up to 20% of patients with biliary
sludge.
• (Choice b) Brown pigment stones are most likely to arise in cases of biliary tract
infection.
• (Choice a) Black pigment stones are most likely to arise in cases of intravascular
hemolysis.
• (Choice d) Gallstones do not typically contain phospholipid as a primary
ingredient.
Concept

Gallbladder hypomotility often results in bile precipitation and the formation of biliary sludge.

80. Ans. (a) Common bile duct


(Ref: Robbins 9/e p877, 8th/887)
The patient is probably suffering from choledocholithiasis, a condition in which a gallstone
becomes lodged in the common bile duct.
She is displaying “Charcot’s triad” (fever, jaundice, and right upper quadrant pain), which
is indicative of cholangitis (infection of the biliary tree proximal to an obstruction such as a
gallstone or malignancy).
The important point in this case is the fact that the patient is jaundiced, eliminating all options
other than a stone in the common bile duct. Stones within the cystic duct (option B) or
gallbladder (option C) or small intestine (options D) do not cause jaundice.
81. Ans. (d) Oral contraceptives
(Ref: Robbins 9/e p879, 8/e p888, Cancer Nursing: Principles and Practice 7/e p1317)
Risk factors of gall bladder cancer
• Gallstones: most important risk factor associated with gallbladder carcinoma.
• Gholedochal cyst
• Carcinogens and chemicals including nitrosamines, rubber and textile industries
• Rubber plant workers
• Obesity
• Estrogen
• Typhoid carrier
• Porcelain gall bladder (calcification of the gallbladder wall)
• Gall bladder polyps
• Anomalous pancreatobiliary duct junction
• Also know
• Only 0.5% of patients with gallstones develop gallbladder cancer after twenty or
more years
• In Asia, where pyogenic and parasitic diseases of the biliary tree are common, the
coexistence of gallstones in gallbladder cancer is much lower.
82. Ans. (a) Adenomyomatosis of gallbladder
(Ref: Learning Ultrasound Imaging p9, Harrison 17/e 1998)
Focal diffuse gall bladder wall thickening with comet tail reverberation artifacts on USG
is a diagnostic finding of Adenomyomatosis of gall bladder.
83. Ans. (b) Primary sclerosing cholangitis
(Ref: Robbins 9/e p860)
84. Ans. (b) Primary sclerosing cholangitis
(Ref: Robbins 8/e p880)
Risk factors (Mnemonic: All have alphabet ‘C’)

• Primary sclerosing Cholangitis


• Liver flukes like Clonorchis sinensis and Opisthorchis viverrini
• Cause of chronic biliary inflammation and injury (Choledocholithiasis)
• Contrat material: thorotrast
• Chronic alcoholic liver disease
• Congenital fibropolycystic diease (Choledochal cysts, Caroli’s disease)

85. Ans. (a) Nodular type of cholangiocarcinoma


(Ref: Robbins 9/e p874, 8/e p880)
• According to their localization, cholangiocarcinomas (CCAs) are classified into
intrahepatic and extrahepatic forms.
• Eighty to 90% of the tumors are extrahepaticQ.
• The extrahepatic forms include perihilar tumors known as Klatskin tumorsQ, which
are located at the junction of the right and left hepatic ductsQ forming the
common hepatic duct, and distal bile duct tumors.
• Most extrahepatic CCAs appear as firm, gray nodulesQ within the bile duct wall
• Klatskin tumors generally have slower growth than other CCAs, show prominent
fibrosis, and infrequently involve distal metastases.
86. Ans. (b) Autosomal recessive
(Ref: Robbins 9/e p847-849, 8th/861, Harrison 18th/3166)
87. Ans. (d) Desferrioxamine is the treatment of choice
(Ref: Harrison 18th/3166; Robbins 7th/615-7, 9/e p849)
• Desferrioxamine is the drug of choice.
• The treatment of choice is phlebotomy at regular intervals.
88. Ans. (d) Hepatic Metastasis
(Ref: Harrison’s 17th/1983; Oxford Texbook of Pathology by McGeel 1312t, Robbins 9/e
p841)
Causes of Hepatic Granulomas
Systemic disease Infections Drugs

• Sarcoidosis - Tuberculosis • Sulfonamides


• Hodgkin’s and Non-Hodgkin’s - MAC, Leprosy • Isoniazid
lymphoma - Brucellosis • Allopurinol
• Primary biliary cirrhosis - EBV, CMV, Chicken pox • Methyldopa
• Berylliosis - Histoplasmosis, Candidiasis • Quinidine
• Crohn’s disease - Schistosomiasis • Phenylbutazone
• Wegener’s granulomatosis - Q fever • Halothane
• Idiopathic - Syphilis

89. Ans. (d) Glycogen depletion


(Ref: OP Ghai 6th/524; 7th/543, CPDT 18th/662-3)
Reye’s Syndrome/Jamshedpur Fever
• Described as a diffuse fatty infiltration of the liver, kidney and cerebral edema with diffuse
mitochondrial injury.
• An acute self-limiting metabolic insult resulting in generalized mitochondrial dysfunction due to
inhibition of fatty acid β-oxidation due to:
– Salicylates
– Inborn error of coenzyme A dehydrogenase
– Varicella or Influenza B viral infections
– Contamination of food with aflatoxin
– Usually observed from 2 month - 15 years of age.

Clinical features
• Child presents with vomiting, anorexia, listlessness followed by altered sensorium, irregular
breathing, seizures and coma.
• Hepatomegaly is present in 50% cases. Jaundice and focal neurological signs are absent.
Diagnosis
• Liver biopsy shows fatty change and glycogen depletion but no NECROSIS of liver cells.
• Liver is showing fatty change, so these lipids can be stained with Oil Red – O.

90. Ans. (b) Kupffer’s cell hyperplasia with macrophage infiltration around periportal
area laden with pigments
(Ref: Robbins 7th/402, 9/e p391-392)
• In severe infections with Plasmodium falciparum, the vital organs are packed with
erythrocytes containing mature form of the parasite. There is abundant intra and
extraerythrocytic pigment and organs such as liver, spleen and placenta may be
grey black in color.

Liver and spleen in severe malaria


Liver
• Liver is generally enlarged and may be black from malarial pigment.
• There is congestion of the centrilobular capillaries with sinusoidal dilatation and Kupffer cell
hyperplasia
• The Kupffer cells are heavily laden with malarial pigment, parasites and cellular debris.
• Sequestration of parasitized erythrocytes is associated with variable cloudy swelling of the
hepatocytes and perivenous ischemic change and sometimes centrizonal necrosis.
• Hepatic glycogen is often present despite hypoglycemia.

Spleen

• The spleen is often dark or black from malarial pigment enlarged, soft and friable.
• It is full of erythrocytes containing mature and immature parasites.
• There is evidence of reticular hyperplasia and architectural reorganization.
• The soft and acutely enlarged spleen of acute lethal infections contrasts with the hard fibrous
enlargement associated with repeated malaria.

Also know

Durck’s granuloma is pathognomic of malignant cerebral malaria.

91. Ans. None


(Ref: Harsh Mohan 6th/628; Robbins’ 39, 910, 914)
Pigmentation in liver is caused by:
1. Lipofuscin: It is an insoluble pigment known as lipochrome and ‘wear and tear’ pigment. It is seen
in cells undergoing low, regressive changes and is particularly prominent in liver and heart of
ageing patient or patients with severe malnutrition and cancer cachexia.
2. Pseudomelanin: After death, a dark greenish or blackish discoloration of the surface of the
abdominal viscera results from the action of sulfated hydrogen upon the iron of disintegrated
hemoglobin. Liver is also pigmented.
3. Wilson’s disease: Copper is usually deposited in periportal hepatocytes in the form of reddish
granules in cytoplasm or reddish cytoplasmic coloration stained by rubeanic acid or rhodamine
stain for copper or orcein stain for copper associated protein. Copper also gets deposited in
chronic obstructive cholestasis.
4. Malarial pigment: Liver colour varies from dark chocolate red to slate-grey even black depending
upon the stage of congestion.
5. In biliary cirrhosis liver is enlarged and greenish-yellow in colour due to cholestasis. So liver is
pigmented due to bile.

92. Ans. (b) Emphysema; (c) Fibrosis of portal tact; (d) Diastase resistance positive
hepatocytes
(Ref: Robbins 7th/911-2, 9/e p850-851)
• This is an autosomal recessive disease characterized by deficiency of α 1-
antitrypsin (important protease inhibitor).
• There is portal tract fibrosisQ in neonatal hepatitis. About 10%-20% of newborn
with α1-antitrypsin deficiency develop neonatal hepatitis and cholestasis.
• Hepatocellular carcinoma develops in 2-3 % a1 - antitrypsin deficiency in adults.
• The treatment and cure, for severe hepatic disease is orthotopic liver transplantation.
• Most important treatment for pulmonary disease is to avoid cigarette smoking
because it accelerates the development of emphysema.
93. Ans. (a) Wilson’s disease
(Ref: Robbins 9/e p850, 8th/864, 7th/911)
94. Ans. (c) ↑Ceruloplasmin (Ref: Robbins 8th/863-4, 9/e p849-850, Harrison 17th/1492)
95. Ans. (b) Neonatal hepatitis
(Ref: Robbins 9/e p851, 8th/865-6; 7th/492,719)
96. Ans. (a) CCl4
(Ref: Robbins 8th/872, 7th/882)
97. Ans. (b) K-RAS
(Ref: Robbins 8th/900-2, 9/e p892)
Presence of pulmonary thromboembolism with a pancreatic mass in an old man suggests a
diagnosis of pancreatic cancer with Trosseau syndrome. This is also supported with
elevated serum levels of tumor markers like CEA and CA 19-9.
The K-RAS gene (chromosome 12pQ) is the most frequently altered oncogene in
pancreatic cancer. This oncogene is activated by point mutation in 80% to 90% of
pancreatic cancers.
Other options
• BRCA 2 mutation may be associated with some pancreatic cancers but usually there is a history
of other cancers like breast cancer, prostate cancer (in males) and breast and ovarian cancers (in
females).
• PRSS1 mutation is also associated with pancreatic cancer but this is usually a cancer starting
early in life secondary hereditary pancreatitis.
• SPINK1 is only associated with hereditary pancreatitis but not pancreatic cancer.

98. Ans. (c) Ceruloplasmin


(Ref: Robbins 8/e p863, 9/e p849-850)
Wilson disease is an autosomal recessive disorder caused by mutation of the ATP7B geneQ,
resulting in impaired copper excretion into bile and a failure to incorporate copper into
ceruloplasminQ.
99. Ans. (a) Perls iron stain
(Ref: Robbins 8/e p862, 9/e p849)
In the liver, iron becomes evident as golden yellow hemosiderin granules in the cytoplasm of
periportal hepatocytes which stain blue with Prussian blue stain. The last mentioned
stain is also called as Perls iron stain.
100. Ans. (d) Hemochromatosis

(Ref: Robbins 8/e p862, 9/e p849)


In hemochromatosis, there is iron deposited in different tissues in the form of hemosiderin. This
deposition along with increased epidermal melanin production leads to a characteristic
slate-gray color to the skin. The development of diabetes in these patients is therefore
termed as bronze diabetes.
101. Ans. (b) ATP 7B.. see above explanation…..
(Ref: Robbins 8/e p863, 9/e p849)
102. Ans. (d) Autosomal recessive
(Ref: Robbins 9/e p849-50)
Wilson disease is an autosomal recessive disorder caused by mutation of the ATP7B gene
located on long arm of chromosome 13, resulting in impaired copper excretion into bile
and a failure to incorporate copper into ceruloplasmin. This causes copper accumulation
in the liver and a decrease in circulating ceruloplasmin.
Spillage of non–ceruloplasmin-bound copper from the liver into the circulation causes
hemolysis and pathologic changes at other sites such as the brain, corneas, kidneys,
bones, joints, and parathyroids. Concomitantly, urinary excretion of copper markedly
increases from its normal miniscule levels.
Diagnosis: It is based on
• A decrease in serum ceruloplasmin
• Increase in hepatic copper content (the most sensitive and accurate test)
• Increased urinary excretion of copper (the most specific screening test)

103. Ans. (b) 13


(Ref: Robbins 9/e p849-50)
ANNEXURE
Precursor Lesions of Hepatocellular and Cholangiocarcinoma

Hepatocellular Cancer Cholangiocarcinoma

Low High Intraductal


Small Large Grade Grade Mucinous Papillary
Hepatocellular Cell Cell Dysplastic Dysplastic Cystic Biliary
Adenoma Change Change Nodule Nodule BiIIN-3 Neoplasm Neoplasia

Focality in Single or Diffuse Diffuse Single or Single or Diffuse or Single Focal or


liver multiple multiple multiple multifocal diffuse
(adenomatosis)

Premalignant Yes Yes In some Uncertain* Yes Yes Yes Yes


HBV*

Association Rare Common Comon Usual Usual Sometimes No No


with cirrhosis

Commonly NAFLD, Sex HBV, HBV, HBV, HBV, PSC, None None
associated hormone HCV, HCV, HCV, HCV, Hepatolithiasis
diseases exposures Alcohol, Alcohol, Alcohol, Alcohol, Liver flukes
Glycogen NAFLD, NAFLD, NAFLD, NAFLD,
storage A1AT, A1AT, A1AT, HH A1AT,
diseases HH, PBC HH PBC HH, PBC
PBC

Occurrence Occasional No No No No Yes Yes Yes


without
identified
predisposing
condition

Need for ± depending on Yes Yes Yes Yes Yes No Yes


surveillance presence of
cancer predisposing
screening condition

*While these are not certain to be directly premalignant, they are always at least an indication of increased risk for
malignancy in the liver as a whole.
BillN-3, Bliary intraepithelial neoplasia, high grade; NAFLD, nonalcoholic fatty liver disease; HBV, hepatitis B
virus; hepatitis C virus; A1AT, α1 -antitrypsin deficiency; HH, hereditary hemochromatosis PBC, primary biliary
cirrhosis, PSC, primary sclerosis cholangitis.

1. Vitamin A is stored in:


(NEET 2019 like pattern)
(a) Hepatocyte
(b) Ito cell
(c) Hepatic endothelial cell
(d) Kupffer cell
Ans. (b) Ito cell
(Ref: Robbins 9th e/p 436)
More than 90% of the body’s vitamin A reserves are stored in the liver, predominantly in the
perisinusoidal stellate (Ito) cells.
2. MRP 2 associated with which of the following?
(NEET 2019 like pattern)
(a) Criggler Najjar syndrome type I
(b) Criggler Najjar syndrome type II
(c) Dubin Johnson syndrome
(d) Rotor syndrome
Ans. (c) Dubin Johnson syndrome
(Ref: Robbins 9th e/p 854)
Dubin-Johnson syndrome is an autosomal recessive disorder leading to impaired biliary
excretion of bilirubin glucuronides due to mutation in canalicular multidrug
resistance protein 2 (MRP2).
3. Hepatitis B infection persists in 3% asymptomatic individuals. Why is there an
increased risk of developing liver cancer in these patients?
(AIIMS Nov 2017 Pattern)
(a) Inability to induce inflammation to remove organism
(b) Increased liver transaminases
(c) High rate of hepatocyte proliferation
(d) Integration of viral DNA to host DNA
Ans. (c) High rate of hepatocyte proliferation
(Ref: Robbins 9/e p328)
The HBV and HCV genomes do not encode anyviral oncoproteins, and although the HBV DNA
is integratedwithin the human genome, there is no consistent pattern of integration in
liver cells. Indeed, while the oncogeniceffects of HBV and HCV are multifactorial,
thedominant effect seems to be immunologically mediated chronic inflammation
and hepatocyte death leading to regeneration and, over time, genomic damage.
Chronicviralinfection leads to the compensatory proliferation of hepatocytes.Onekey
mutagenic step is the activation of the NF-κBpathway in hepatocytes in response to
mediators derivedfrom the activated immune cells. This activation blocks apoptosis,
allowing thedividing hepatocytes to incur genotoxic stress and to accumulatemutations.
Although this seems to be the dominantmechanism in the pathogenesis of virus-
induced hepatocellularcarcinoma, the HBV genome also contains genesthat may
directly promote the development of cancer
Disclaimer
Any resemblance to an actual question is purely coincidental.
• Breast lesions not involving terminal duct lobules unit
(TDLU): Nipple adenoma and syringoma.
Non-proliferative breast lesions (no risk of malignancy):
Duct ectasia, cysts; apocrine metaplasia, adenosis,
fibroadenoma without atypia, mild hyperplasia.
Proliferative lesions without atypia (slight risk of
malignancy): Moderate to florid hyperplasia, sclerosing
adenosis, papilloma, fibroadenoma with complex
features.
Proliferative lesion with atypia (moderate risk of
malignancy): Atypical epithelial hyperplasia.
Most common gene involved in breast cancer (overall):
p53 whereas the commonest gene involved in familial
breast cancer is BRCA-1. Subtypes of breast carcinoma
with BRCA-1 mutation are Medullary and mucinous
carcinomas.
DCIS which can present as palpable mass:
Comedocarcinoma (it is a carcinoma in situ)

• Most common breast cancer: Invasive ductal


carcinoma.

• Bilateral breast cancer: Lobular carcinoma .


Histologic hallmark of lobular carcinoma: Pattern of single
infiltrating tumor cells often only one cell in width or in
loose clusters or sheets.

• Paget cells are: Large cells with clear cytoplasm and


prominent nucleus.
Carcinoma of penis is extremely rare in people having
circumcision (Jews and Muslims)
Most common testicular tumor Seminoma.
Most common testicular tumor in infant and young
children: Yolk sac tumor.\
Most common testicular tumor in elderly is lymphoma.
Microscopic feature of seminoma: Sheets of uniform cells
with lymphocytic infiltration.
Seminoma is positive for: Placental alkaline phosphatase,
keratin, only in 7% cases HCG.
Yolk sac (endodermal sinus) tumor is, positive for: AFP,
α-1 antitrypsin and has presence of Schiller-Duval’ bodies
(glomeruloid structures).
Common sites for extragonadal germ cell tumors:
Mediastinum (M/C,) retroperitoneum (2nd most common),
sacrococcygeal region, pineal gland.

• Sarcoma botryoides is seen in infant and children (< 5


years). It shows the presence of Tennis racket cells
Most common ovarian tumor: Surface epithelial tumors
specially serous tumors like serous cystadenoma/
cystadenocarcinoma. Reinke crystalloids are seen in:
Hilus cell tumor/pure Leydig (interstitial) cell tumor.
Psammoma bodies are seen in: Serous cystadenoma.
Call-Exner bodies are seen in: Granulosa-theca cell
tumor.

• Rokitansky’s protuberance is seen in: Mature (benign)


teratoma/dermoid cyst.
Largest ovarian tumor: Mucinous cystadenoma (also
causes Pseudomyxoma peritonei)

• Krukenberg’s tumor of ovary is produced by:


Carcinoma of breast (most commonly) and GIT, like
stomach, pancreas, colon, gallbladder (not by liver).
MALE GENITAL TRACT

Penis

• Congenital malformations affecting the penis are abnormal


locations of urethral openings and phimosis. These abnormal
locations may produce obstruction of urinary tract infection or
infertility.
• Phimosis occurs when the orifice of the prepuce (foreskin) is
too small to permit normal retraction. It may be due to
abnormal development or more commonly due to inflammatory
scarring. It interferes with cleanliness and favors the
development of secondary infections and possibly carcinoma.
• Paraphimosis is inability to roll back the prepuce after forcible
retraction over glans penis. It is extremely painful and may
cause obstruction of urinary tract (cause of acute urinary
retention) or blood flow (may lead to necrosis of penis).
• Balanoposthitis is a non-specific infection of glans and
prepuce. It is mostly caused by Candida, anerobic bacteria and
Gardernella.
Tumors of penis may be benign [condyloma acuminata] or
malignant [carcinoma in-situ and invasive carcinoma].

• Condyloma acuminatum is a benign tumor caused by human


papilloma virus (HPV), most commonly type 6 and sometimes type
11. Koilocytosis is a characteristic of infection with HPV. It is seen in
condyloma as well as carcinoma.

• Carcinoma in-situ refers to epithelial lesions in which


cytological changes of malignancy are confined to epithelium,
with no evidence of local invasion or metastasis. These are
considered as pre-cancerous lesions. In about 80% of cases,
these lesions are associated with HPV-16. Bowen disease,
Erythroplasia of Queyrat (a variant of Bowen’s disease) and
Bowenoid papulosis are examples of carcinoma in-situ.
Bowen disease may transform into invasive squamous cell
carcinoma in 10% patients and is associated with occurrence
of visceral cancers in about one thirds of patients. In contrast,
bowenoid papulosis never develops into invasive carcinoma
and many times, it spontaneously regresses.
• Squamous cell carcinoma is associated with cigarette
smoking and infection with HPV-16 (more commonly) and
HPV-18. Mostly, squamous cell carcinoma invades tissue as
finger like projections (papillary) of atypical squamous epithelial
cells. These show varying degree of differentiation. A variant of
squamous cell carcinoma is verrucous carcinoma [also
known as Giant condyloma or Buschke-Lowenstein tumor]
which invades the underlying tissue along a broad front (in
contrast, papillary carcinoma invades as finger like
projections).
TESTIS AND EPIDIDYMIS

• Cryptorchidism (undescended testes) is found in 1% of 1-


year-old boys and is mostly unilateral (Right > Left). Testicular
descent has two phases; transabdominal and inguino-scrotal.
Transabdominal phase is controlled by Mullerian-inhibiting
substance whereas inguinoscrotal phase is androgen
dependent (mediated by androgen induced release of CGRP
from genitofemoral nerve). Grossly, testis is small, brown and
atrophic. Microscopically, tubules are atrophic with thickened
basement membranes. Leydig cells are spared and appear
to be prominent. Occasionally, proliferation of Sertoli cells
may also be seen. Smaller but definite risk of malignancy is
present for contralateral, correctly placed testis. Persistently
undescended testes require orchiopexy (placement in scrotal
sac) preferably before 2 years before histological deterioration
sets in. Orchiopexy does not guarantee fertility.

Note: Cryptorchidism is associated with trisomy 13 and genitourinary


malformations like hypospadias or in-utero exposure of DES.
• Ectopic testes is the deviation of testes from normal path of
descent. Gubernaculum testis has five tails (namely scrotal,
pubic, perineal, inguinal and femoral). Normally scrotal tail is
strongest, so testes descend to scrotum. If other accessory
tails become strong, testis may drain toward that tail.
Difference of ectopic testis from undescended testis is that
former is fully developed and hence has normal
spermatogenesis whereas latter lacks spermatogenesis.
• Scrotal swelling may occur due to inflammation, abnormality
of blood vessels, cysts or tumors of testes or epididymis.

• Non-specific inflammations in a sexually active young patient (<


35 years) is mostly caused by Chlamydia trachomatis and
Neisseria gonorrhea whereas most common culprit in men
older than 35 years are E. coli and pseudomonas.
• Twisting of spermatic cord resulting in cut-off of the venous
drainage and arterial supply to testis may result in TORSION.
Neonatal torsion lacks any associated anatomical defect
whereas adult torsion results from a bilateral anatomical defect
in which testes have increased mobility [bell-clapper
abnormality]. If untwisted within 6 hours, testes may remain
viable. To prevent subsequent torsion, contralateral normal
testis is fixed to scrotum [orchiopexy].
Note: Doppler flow studies and testicular scintigraphy are useful if
testicular torsion is expected clinically.

• Benign scrotal cysts may form from abnormalities of tunica


vaginalis. Processus vaginalis is an outpouching of the
peritoneum that enters into the scrotum. When testis reaches
the scrotum, proximal portion of processus vaginalis obliterates
whereas distal portion persists and forms tunica vaginalis.
Cysts involving tunica vaginalis can be hydrocele [contain
clear fluid], hematocele [results from hemorrhage into a
hydrocele], chylocele [accumulation of lymph in tunica due to
elephantiasis) or spermatocele (cystic enlargement of efferent
ducts or rete testis with numerous spermatocytes present].
Varicocele results from dilatation of testicular veins in
pampiniform plexus. It is associated with oligospermia (< 20
million spermatozoa/ml of semen) and is most common cause
of infertility. Left side is affected more commonly.
• Sertoli cell only syndrome also known as Del-Castillo’s
syndrome is a condition in which seminiferous tubules are
lined by only Sertoli cells.
Fig. 1: Sertoli Only syndrome; Reinke crystals (arrows) in Leydig
cells (L).
Testicular Germ Cell Tumors
Most of these arise from intratubular germ cell neoplasia (ITGCN)
except spermatocytic seminoma and teratoma.

Predisposing factors for germ cell tumors are:

• Cryptorchidism [abdominal > inguinal]


• Testicular dysgenesis [feminization and Klinefelter]
• Siblings of affected person (have tenfold higher risk)
• Isochromosome of short arm of chromosome 12, i (12p) is seen in all
germ cell tumors [both testicular as well as ovarian]

Clinically germ cell tumors of testes can be divided into seminoma


and non-seminomatous germ cell tumors (NSGCT).
Fig. 2: Seminoma: Clear cell surrounded by fibrous septa (S).

SEMINOMATOUS GERM CELL TUMORS

• Seminomas are characterized by large cells with distinct cell


membranes and clear cytoplasm, large central nucleus with
prominent one or two nucleoli. Cytoplasm contains glycogen.
Classical seminomas do not contain alpha fetoprotein
(AFP). Human chorionic gonadotropin (hCG) is present in 15%
of seminomas that contain syncytiotrophoblasts.
Spermatocytic seminoma is a distinctive tumor characterized
by being found in old age and having excellent prognosis [do
not metastasize]. Histologically, it is characterized by
maturation of tumor cells, some of which resemble secondary
spermatocytes. Spermatocytic seminoma is seen only in
testis. There is no ovarian counterpart.
NON-SEMINOMATOUS GERM CELL TUMORS
These may be embryonal carcinoma, yolk sac tumors,
choriocarcinoma or teratoma.
• Choriocarcinomas have a mixture of malignant
cytotrophoblasts and syncytiotrophoblasts. These are most
aggressive variants. Malignant teratomas have tissue derived
from all three germ layers with scattered immature neural
elements. In children, differentiated mature teratomas are
considered benign whereas in post-pubertal males, all
teratomas are regarded as malignant.

• Embryonal cell carcinomas present as sheets of


undifferentiated cells. Focal glandular differentiation may be
present. Elevated AFP and hCG is seen in this tumor.
• Yolk sac tumor or infantile embryonal carcinoma or
endodermal sinus tumor are the most common testicular
tumor in infants and children up to 3 years of age. These have
very good prognosis. Half of tumors show Schiller-Duval
bodies or glomeruloid structures [structures resembling
endodermal sinuses]. Presence of AFP is highly characteristic
of yolk sac tumors.
Seminoma NSGCT
1. Radiosensitive Radio resistant
2. Localized to testes for long Early dissemination (60% present
duration (70% present in stage in stage II & III)
1)
3. Metastasis first to lymph nodes Early hematogenous metastasis
4. Tunica albuginea spared Tunica breached mostly

Important points about germ cell tumors

• Painless enlargement of testis is a characteristic feature.

• Lymphatic spread is common to all testicular tumors. Retroperitoneal


nodes are first involved.
• Hematogenous spread is primarily to lungs.

Non-Germinal Tumors

• Leydig cell tumors are derived from stroma and Sertoli cell
tumors from sex cords. Sertoli cell tumors are also known as
Androblastoma. Both these tumors are benign.
• Gonadoblastoma contains a mixture of germ cells and
gonadal stromal elements.

Lymphomas are most common testicular neoplasms in men over the age
of 60 years. The prognosis is extremely poor. These are most common
cause of bilateral testicular tumors.

Fig. 3: Yolk sac tumor with Schiller Duval body (Glomeruloid


structure; G).

PROSTATE
In a normal adult prostate weighs about 20 g. It is divided into
peripheral, central, transitional zones and the region of anterior
fibromuscular stroma. Prostate is a combined tubuloalveolar
organ. Characteristically, the glands are lined by two layer of cells,
basal layer of cuboidal cells covered by a layer of columnar
secretory cells. Three important conditions of prostate are
inflammations, hyperplasia and tumors.
Inflammation of prostate (prostatitis)
It is characterized by finding at least 15 leukocytes per high
power field in prostatic secretions.
Acute prostatitis present with sudden onset of fever, chills and
dysuria. It is mostly caused by E. coli
• Chronic bacterial prostatitis is associated with recurrent UTI.
• Chronic abacterial prostatitis is associated with infections with
Chlamydia or Ureaplasma.
• Granulomatous prostatitis is mostly caused by intravesical
administration of BCG (used for treatment of superficial
bladder carcinoma).

Malacoplakia is a granulomatous disease with defective intracellular


lysosomal digestion of bacteria. It is mostly caused by E. coli. Sharply
demarcated spherical structures with concentric owl eye (known as
Michaelis-Guttmann bodies) are seen histologically.

Nodular hyperplasia
It is also known as benign prostatic hyperplasia (BPH). Clinical
symptoms are urinary frequency, nocturia, difficulty in starting or
stopping urination, dribbling and dysuria. Histologically, nodules
are composed of hyperplastic stromal cells and hyperplastic
glands. Glands consist of two layer of cells; cuboidal and
columnar [in carcinoma single layers of cells are present in
glands] with intervening stroma. Histological signs of malignancy
are absent. Development of BHP is associated with advanced age
and high testosterone levels. Di-hydrotestosterone (DHT) is
produced from testosterone with the help of an enzyme, 5a-
reductase type 2. DHT is the main substance responsible for
prostatic growth. In addition to mechanical effects of enlarged
prostate, clinical symptoms are also due to smooth muscle
mediated contraction of prostate by a1A receptors.

Fig. 4: Prostate showing benign hyperplasia (B) and carcinoma


(C).

Note: In some cases, nodular enlargement may project up into the floor
of urethra as a hemispherical mass, which is termed as “median lobe
hypertrophy”.

Tumors
Adenocarcinoma of prostate is most common form of cancer in
men. Advancing age, race (more in American blacks, least in
Asians), dietary factors (increases with more fat consumption,
decreases with lycopene, vitamin A, vitamin E, selenium and soy
products), androgens and genetic factors are implicated in
pathogenesis of prostate cancer. Genetic factors include germ line
mutations of BRCA2 tumor suppressor gene, chromosomal re-
arrangements that juxtapose ERG or ETV1 next to androgen
regulated TMPRSS2 promoter and epigenetic alterations like
hypermethylation of glutathione 5-transferase (GSTP1) gene
causing down regulation of GSTP1 expression. Local extension
most commonly involves seminal vesicles and later base of
bladder; Fascia of Denonvilliers prevents the backward
extension of the tumor. Hematogenous spread occur chiefly to
bones (osteoblastic secondaries) most commonly to lumbar
spine. Lymphatic spread occurs initially to obturator nodes.
Histologically, most lesions are adenocarcinomas characterized by
small glands that appear “back to back” without intervening
stroma or that appear to be infiltrating beyond the normal prostate
lobules.

Most prostate cancers arise peripherally, away from urethra; therefore urinary
symptoms occur late. Osteoblastic secondaries in bone are virtually diagnostic
of prostate cancer.

Grading of prostate cancer


Grade 1 is well differentiated and Grade 5 shows no glandular
differentiation. Grade 2, 3 and 4 are in-between. Most tumors
contain more than one pattern, so primary grade is assigned to
dominant pattern and secondary grade to subdominant pattern.
Combined Gleason score is derived by addition of these two
grades. Tumors with only one pattern are assumed to have both
primary and secondary grade as same. Thus Gleason score for
these is double the grade. Score of 2-4 are considered well-
differentiated, 5-6 as moderately differentiated, 7 moderate to
poorly differentiated and 8-10 high grade cancers. Grading is of
particular importance in prostatic cancer, as it is the best marker,
along with stage, for predicting prognosis.

Major role of transrectal ultrasonography (TRUS) in prostate cancer is in


guiding the placement of needle biopsies to thoroughly sample the gland.
Transperineal or transrectal biopsy is required for diagnosis.

Prostate specific antigen (PSA): 20-40% patients with prostate


cancer have PSA value of 4 ng/ml or less and so, four different
refinements in PSA value can be utilized.
a. PSA density: It is the ratio of serum PSA value and volume of
prostate. It reflects PSA produced per gram of prostate tissue.
Upper normal limit is 0.15.
b. Age specific reference range
Age Upper normal value
40-49 years 2.5 ng/ml
50-59 years 3.5 ng/ml
60-69 years 3.5 ng/ml
70-79 years 6.5 ng/ml
c. PSA velocity: It is rate of change of PSA with time. At least 3
PSA measurements should be taken over a period of 1.5 to 2
years.
d. Percentage of free PSA: It is calculated as
Free PSA/Total PSA × 100
It is more valuable, when total PSA is in ‘gray zone’ of 4 to 10
ng/ml. Free PSA less than 10% indicates high risk of carcinoma
whereas value > 25% indicates lower risk.

FEMALE GENITAL TRACT

Embryology

The paired genital ducts consist of the mesonephric (Wolffian)


duct, which extends from the mesonephros to the cloaca, and the
paramesonephric (Mullerian) duct, which runs parallel and lateral
to the Wolffian duct.

• The mesonephric ducts in males, if stimulated by


testosterone (secreted by the Leydig cells), develop into the
vas deferens, epididymis, and seminal vesicles. In contrast,
because normal females do not secrete testosterone, the
Wolffian ducts regress and form vestigial structures. They may,
however, form mesonephric cysts in the cervix or vulva, or they
may form Gartner duct cysts in the vagina. The cranial group of
mesonephric tubules (the epoophoron) remains as vestigial
structures in the broad ligament above the ovary, while the
caudal group of mesonephric tubules (the paroophoron) forms
vestigial structures in the broad ligament beside the ovary.
• The paramesonephric ducts in the female form the fallopian
tubes, the uterus, the uppermost vaginal wall, and the hydatid
of Morgagni. The lower portion of the vagina and the vestibule
develop from the urogenital sinus. Males secrete Mullerian-
inhibiting factor (MIF) from the Sertoli cells of the testes, which
causes regression of the Mullerian ducts. This results in the
formation of the vestigial appendix testis.

• Several abnormalities result from abnormal embryonic


development of the Mullerian ducts.
– Uterine agenesis may result from abnormal development
or fusion of these paired paramesonephric ducts.
Developmental failure of the inferior portions of the
Mullerian ducts results in a double uterus, while failure of
the superior portions to fuse (incomplete fusion) may form
a bicornuate uterus.
– Retarded growth of one of the paramesonephric ducts along
with incomplete fusion to the other paramesonephric ducts
results in the formation of a bicornuate uterus with a
rudimentary horn.

GENITAL CYSTS

Obstruction of the ducts of any of the glands found within the


female genitalia may cause the formation of a genital cyst.

• Bartholin’s cyst: The paired Bartholin’s glands, which are analogous


to the bulbourethral glands of the male, are located in the lateral wall
of the vestibule. If these are obstructed, a cyst may form that is
usually lined with transitional epithelium.
• Gartner’s duct cysts: These are derived from Wolffian
(mesonephric) duct remnants and are located in the lateral walls of
the vagina.
• Mesonephric cysts: Cysts derived from the same Wolffian duct may
also be found on the lateral aspect of the vulva and are called
mesonephric cysts.
• Nabothian cysts: Obstruction of the ducts of the mucous glands in
the endocervix may result in small mucous (Nabothian) cysts.
• Epithelial inclusion cysts: Cysts may also be found within the skin
of the vulva. These cysts, which contain white, cheesy material, are
called keratinous (epithelial inclusion) cysts. Clinically they are
referred to as sebaceous cysts, which is a misnomer.
• Follicular cysts: These are benign cysts of the ovary.

DISEASES OF VULVA

1. Leukoplakia
Several pathologic conditions are associated with the formation of
white plaques on the vulva, which are clinically referred to as
leukoplakia.
• Lichen sclerosis is seen histologically as atrophy of the
epidermis with underlying dermal fibrosis.

The four cardinal histologic features are:


Atrophy (thinning) of the epidermis, with disappearance of the rete pegs
Hydropic degeneration of the basal cells
Replacement of the underlying dermis by dense collagenous fibrous tissue
A monoclonal bandlike lymphocytic infiltrate
• Loss of pigment in the epidermis (vitiligo) can also
produce leukoplakia.
• Inflammatory skin diseases, squamous hyperplasia and
vulvar intraepithelial neoplasia can also present with
leukoplakia.
2. Benign Tumors
a. Papillary Hidradenoma
Hidradenomas consist of tubular ducts lined by a single or
double layer of nonciliated columnar cells, with a layer of
flattened “myoepithelial cells” underlying the epithelium. These
myoepithelial elements are characteristic of sweat glands and
sweat gland tumors. It is identical in appearance to intraductal
papillomas of the breast.

b. Condyloma Acuminatum
Condylomata acuminata are sexually transmitted, benign tumors
that have a distinctly verrucous gross appearance.
Condylomata are caused by HPV, principally types 6 and 11. It
is not considered to be precancerous lesions.

3. Premalignant and Malignant Neoplasms


i. Squamous cell carcinoma
It may be associated with high-risk HPV or with squamous cell
hyperplasia and lichen sclerosus.
• Rare variants of squamous cell carcinoma include
verrucous carcinomas, which are fungating tumors resembling
condyloma acuminatum, and basal cell carcinomas, which are
identical to their counterparts on the skin. Neither tumor is
associated with papillomaviruses.

ii. Paget’s disease


• It manifests grossly as pruritic, red, crusted, sharply
demarcated map-like areas.
• Histologically, it reveals single anaplastic tumor cells
infiltrating the epidermis. These cells are characterized by
having clear spaces (“halos”) between them and the
adjacent epithelial cells. These malignant cells stain
positively with PAS or mucicarmine stains.
iii. Malignant melanoma
Malignant melanoma of the vulva may resemble Paget’s
disease, however, these malignant cells stain positively with
a melanin stain or S100 immunoperoxidase stain.

DISEASES OF VAGINA

1. Adenocarcinoma

• The tumors are most often located on the anterior wall of the
vagina, usually in the upper third.
• These are often composed of vacuolated, glycogen-containing
cells, hence the term clear cell carcinoma. These cancers can
also arise in the cervix.
• A probable precursor of the tumor is vaginal adenosis, a
condition in which glandular columnar epithelium of Müllerian
type either appears beneath the squamous epithelium or
replaces it.

2. Embryonal rhabdomyosarcoma or sarcoma botryoides

• It is an uncommon vaginal tumor most frequently found in


infants and in children younger than 5 years of age.
• The tumor consists predominantly of malignant embryonal
rhabdomyoblasts and is thus a type of rhabdomyosarcoma.
• These tumors have the appearance and consistency of
grapelike clusters (hence the designation botryoides,
meaning grapelike).

DISEASES OF CERVIX

1. Cervicitis

• Acute cervicitis is characterized by acute inflammatory cells,


erosion, and reactive or reparative epithelial change.
• Chronic cervicitis includes inflammation, usually
mononuclear, with lymphocytes, macrophages, and plasma
cells.
• HSV is most strongly associated with epithelial ulcers (often
with intranuclear inclusions in epithelial cells) and a
lymphocytic infiltrate, and C. trachomatis with lymphoid
germinal centers and a prominent plasmacytic infiltrate.
Epithelial spongiosis is associated with T. vaginalis infection.

2. Intraepithelial and Invasive Squamous Neoplasia


a. Cervical Intraepithelial Neoplasia (CIN)
CIN can be divided into three grades; CIN I, CIN II and CIN III

• CIN I: These lesions are on the extreme low end of the spectrum and
are often indistinguishable histologically from condylomata
acuminata. These have a low rate of progression to cancer.
• CIN II: These consist of the appearance of atypical cells in the lower
layers of the squamous epithelium but nonetheless with persistent
(but abnormal) differentiation toward the prickle and keratinizing cell
layers.
• CIN III: As the lesion evolves, there is progressive loss of
differentiation accompanied by greater atypia in more layers of the
epithelium, until it is totally replaced by immature atypical cells,
exhibiting no surface differentiation (CIN III).

b. Squamous Cell Carcinoma


• Invasive cervical carcinoma manifests in three somewhat
distinctive patterns: fungating (or exophytic), ulcerating,
and infiltrative cancers. The most common variant is the
fungating tumor, which produces an obviously neoplastic
mass that projects above the surrounding mucosa.
• On histologic examination, a small subset of tumors (less
than 5%) are poorly differentiated small cell squamous or,
more rarely, small cell undifferentiated carcinomas
(neuroendocrine or oat cell carcinomas). The latter closely
resemble oat cell carcinomas of the lung and have an
unusually poor prognosis owing to early spread by
lymphatics and systemic spread. These tumors are also
frequently associated with a specific high-risk HPV, type
18.

Clear cell adenocarcinomas of the cervix in DES-exposed women are


similar to those occurring in the vagina.
Fig. 5: CIN I.

Fig. 6: CIN II.

Fig. 7: CIN III.

DISEASES OF UTERUS

1. Endometritis
• The endometrium and myometrium are relatively resistant to
infections. Therefore, inflammation of the endometrium
(endometritis) is rare.
• Acute endometritis is usually caused by bacterial infection
following delivery or miscarriage and is characterized by the
presence of neutrophils in non-menstrual endometrium.
• The histologic diagnosis of chronic endometritis depends on
finding plasma cells within the endometrium. All it takes is one
plasma cell to make the diagnosis.
• Chronic endometritis may be seen in patients with intrauterine
devices (IUDs), pelvic inflammatory disease (PID), retained
products of conception (postpartum), or tuberculosis.

2. Endometriosis and Adenomyosis

• Adenomyosis is thought to result from the abnormal down


growth of the endometrium into the myometrium. Symptoms
produced by adenomyosis include menorrhagia, colicky
dysmenorrhea, dyspareunia, and pelvic pain.
• Endometriosis is thought to possibly arise from metaplasia of
celomic epithelium into endometrial tissue or implantation of
normal fragments of menstrual endometrium either via the
fallopian tubes or via the blood vessels. Histologically, it
reveals endometrial glands, stroma, and hemosiderin pigment
(from the cyclic bleeding). Repeated cyclic bleeding in patients
with endometriosis can lead to the formation of cysts (3-5 cm
diameter) that contain areas of new and old hemorrhages.

• Sites of endometriosis include the ovary, uterine ligaments


(associated with dyspareunia), the rectovaginal pouch
(associated with pain on defecation and low back pain), the
fallopian tubes (associated with peritubular adhesions,
infertility, and ectopic pregnancies), the urinary bladder
(associated with hematuria), the GI tract (associated with pain,
adhesions, bleeding and obstruction), and the vagina
(associated with bleeding).

3. Menstrual abnormalities
With normal menstruation about 30 to 40 ml of blood is lost.
Amount greater than 80 ml lost on a continued basis are
considered to be abnormal.

• Menorrhagia refers to excessive bleeding at the time of menstruation,


either in the number of days or the amount of blood. A submucosal
leiomyoma could produce menorrhagia.
• Metrorrhagia refers to bleeding that occurs at irregular intervals.
• Menometrorrhagia refers to excessive bleeding that occurs at
irregular intervals. Causes of metro or menometrorrhagia include
cervical polyps, cervical carcinoma, endometrial carcinoma, or
exogenous estrogens.
• Oligomenorrhea refers to infrequent bleeding that occurs at
intervals greater than 35 days. Causes include polycystic ovarian
syndrome and too low a total body weight.
• Polymenorrhea refers to frequent, regular menses that are less
than 22 days apart. It is commonly associated with anovulatory
cycles, which can occur at menarche.

4. Dysfunctional uterine bleeding (DUB)


DUB is defined as abnormal uterine bleeding that is due to a
functional abnormality rather than an organic lesion of the
uterus.
The three main categories of DUB are:
• Anovulatory cycles (the most common form),
• Inadequate luteal phase
• Irregular shedding.

Anovulatory cycles consist of persistence of the Graffian


follicle without ovulation. This results in continous and excess
estrogen production without the normal postovulatory rise in
progesterone levels. With no progesterone production, no
secretory endometrium is formed. Instead, biopsies reveal
proliferative endometrium with mild hyperplasia. The mucosa
becomes too thick and is sloughed off, resulting in the abnormal
bleeding.

If there is ovulation but the functioning of the corpus luteum is


inadequate, then the levels of progesterone are decreased, resulting in
asynchrony between the chronologic dates and the histologic
appearance of the secretory endometrium. This is referred to as an
inadequate luteal phase (luteal phase defect).

The luteal phase defect is an important cause of infertility.


Biopsies are usually performed several days after the predicted
time of ovulation. If the histologic dating of the endometrium lags 4
or more days behind the chronologic date predicted by menstrual
history, the diagnosis of luteal phase defect can be made.
Clinically, these patients exhibit low serum progesterone, FSH, and
LH levels.
Prolonged functioning of the corpus luteum (persistent luteal
phase with continued progesterone production) results in
prolonged heavy bleeding at the time of menses. Histologically,
there is a combination of secretory glands mixed with proliferative
glands (irregular shedding). Clinically, these patients have regular
periods, but the menstrual bleeding is excessive and prolonged
(lasting 10 to 14 days).
5. Endometrial hyperplasia (Endometrial Intraepithelial
Neoplasia)
Fig. 8: Endometrial hyperplasia without atypia.

It is related to excess estrogens and is important clinically because


of its relation to the development of endometrial adenocarcinoma.
The types of endometrial hyperplasia include simple hyperplasia
and complex hyperplasias.

Note: The shift in gland morphology from benign to precancerous is often


highlighted by a loss of PTEN gene expression.
Fig. 9: Endometrial hyperplasia with atypia.

Characteristis of type I and Type II endometerial carcinoma


Charateristics Type I Type II
Age 55-65 year 65-75 year
Clinical setting Unopposed estrogen Atrophy thin physique
obesity hypertension
diabetes
Morphology Endometroid Serous clear cell mixed
mullerian tumor
Precursor Hyperplasia Serous endometrial
intraepithelial carcinomas
Mutated genes/genetic PTEN TP53
abnormalities ARIDA (regulator of Aneuploidy
chromatin) PIK3CA (PI3K)
PIK3CA (PI3K) FBXW7 (regulator of MYC,
KRAS cyclin E)
FGF2 (growth factor) CHD4 (regulator of
MSI* chromatin)
CTNNB1 (Wnt signating) PPP2R1A (PP2A)
TP53
Behavior Indolent Aggressive
Spreads via lymphatics Intraperitoneal and
lymphatic spread
(MSI: Microsatellite instability; CTNNB1: Beta-cetanin gene).

6. Endometrial carcinoma

• Endometrial carcinomas that are associated with hyperplasia


tend either to be well-differentiated, mimicking normal
endometrial glands (endometrioid) in histologic appearance, or
to display altered differentiation (mucinous, tubal, squamous
differentiation).
• Endometrial cancer not associated with pre-existing
hyperplasia are generally more poorly differentiated, including
tumors that resemble subtypes of ovarian carcinomas
(papillary serous carcinomas). Overall, these tumors have a
poorer prognosis than estrogen-related cancers do. In
contrast to endometrioid tumors, serous subtypes infrequently
display microsatellite instability and are linked to mutation of
p53.

• Histologically most of the endometrial carcinomas are


adenocarcinomas.
• If there are areas of squamous differentiation within these tumors,
they are called adenoacanthomas.
• If there are areas of malignant squamous differentiation, they are
called adenosquamous carcinomas.

7. Tumors of the Endometrium with Stromal Differentiation


a. Carcinosarcomas or malignant mixed Müllerian tumors
Carcinosarcomas consist of endometrial adenocarcinomas in
which malignant stromal differentiation takes place. The stroma
tends to differentiate into a variety of malignant mesodermal
components, including muscle, cartilage, and even osteoid. On
histology, the tumors consist of adenocarcinoma mixed with
the stromal (sarcoma) elements Sarcomatous components
may mimic extrauterine tissues (i.e., striated muscle cells,
cartilage, adipose tissue, and bone).
b. Adenosarcomas
It consists of malignant appearing stroma, which coexists with
benign but abnormally shaped endometrial glands.
c. Stromal Tumors
The endometrial stroma occasionally gives rise to neoplasms that
may resemble normal stromal cells. Stromal neoplasms may
be benign stromal nodules or endometrial stromal sarcomas.

8. Tumors of Myometrium

• Fibroids (Leiomyoma) of the uterus arise in the myometrium,


submucosally, subserosally, and mid-wall, both singly and
several at a time. They are benign smooth-muscle tumors that
are sharply circumscribed, firm, gray-white, and whorled on
cut section.
• Their malignant counterpart, leiomyosarcoma of the uterus, is
quite rare in the de novo state and arises even more rarely
from an antecedent leiomyoma.


DISEASES OF OVARIES

1. Stein-Leventhal syndrome/Polycystic ovarian disease


(PCOD)

• The symptoms of patients with this syndrome are related to


increased androgen production, which causes hirsutism, and
decreased ovarian follicle maturation, which can lead to
amenorrhea.
• The cause of this syndrome is thought to be the abnormal
secretion of gonadotropins by the pituitary. Increased secretion
of LH stimulates the thecal cells to secrete excess amounts of
androgens, which are converted to estrone by the peripheral
aromatization of androgens by the adrenal gland. Excess
estrogens in turn increase the levels of gonadotropin-releasing
hormone (GnRH) but decrease the levels of FSH. The GnRH
increases the levels of LH, which then stimulate the thecal cells
of the ovary to secrete more androgens, and the hormonal
cycle begins again.

The ovaries in these patients are enlarged and show thick capsules,
hyperplastic ovarian stroma, and numerous follicular cysts, which are
lined by a hyperplastic theca interna. Because these patients do not
ovulate, there is a markedly decreased number of corpora lutea, which,
in turn, results in decreased progesterone levels.
2. Ovarian tumors
Ovarian neoplasms may be divided into four main categories;
epithelial tumors, sex cord-stromal tumors, Germ cell tumors and
metastases.
Mnemonic:
WHO classification of ovarian tumors

1. Surface Epithelial Tumors 2. Germ Cell Tumors


My Mucinous Doctor Dysgerminoma
Servant Serous Examined Endodermal Sinus
tumor
Began Brenner The Teratoma
Experiencing Endometrioid Ovaries Ovarian
choriocarcinoma
Cancer Clear
3. Sex cord Stromal Tumors 4. Metastatic
She Sertoli-Leydig Killed: Krukenberg
Felt Fibroma-thecoma
Grim Granulosa theca

A. Surface Epithelial Tumors

These are derived from the surface celomic epithelium, which


embryonically gives rise to the Mullerian epithelium.
Therefore, these ovarian epithelial tumors may recapitulate
the histology of organs derived from the Mullerian epithelium.

a. Serous ovarian tumors


These are composed to ciliated columnar serous epithelial cells,
which are similar to the lining cells of the fallopian tubes.
They commonly involve the surface of ovary.
• Bilaterality is common, occurring in 20% of benign
cystadenomas, 30% of borderline tumors, and
approximately 66% of cystadenocarcinomas.
• Concentric calcifications (psammoma bodies) characterize
serous tumors, although they are not specific for neoplasia
when they are found alone.

b. Mucinous ovarian tumors


Benign mucinous tumors are characterized by a lining of tall
columnar epithelial cells with apical mucin and the absence of
cilia, akin to benign cervical or intestinal epithelia. In gross
appearance, the mucinous tumors differ from the serous
variety in several ways:

• They are characterized by more cysts of variable size and a rarity of


surface involvement.
• They are less frequently bilateral.
• Mucinous tumors tend to produce larger cystic masses, and some
have been recorded with weights of more than 25 kg.

• One group of typically benign or borderline mucinous


tumors arises in endometriosis and is termed “Müllerian
mucinous” cystadenoma, resembling endometrial or
cervical epithelium. These tumors are uncommonly
malignant.
• The second, more common group includes tumors
exhibiting abundant gland-like or papillary growth with
nuclear atypia and stratification and is strikingly similar to
tubular adenomas or villous adenomas of the intestine.
These tumors are presumed precursors to most
cystadenocarcinomas. Cystadenocarcinomas contain more
solid growth with conspicuous epithelial cell atypia and
stratification, loss of gland architecture, and necrosis, and
are similar to colonic cancer in appearance.
• Pseudomyxoma peritonei refers to the formation of multiple
mucinous masses within the peritoneum. This condition
results from the spread of mucinous tumors, either from
metastasis or rupture of an ovarian mucinous cyst.

c. Endometrioid ovarian tumors


These are composed of nonciliated columnar cells, which are
similar to the epithelial cells of the endocervical glands.
d. Clear cell carcinoma of the ovary
It is similar histologically to clear cell carcinoma of the kidney, or
more accurately, the clear cell variant of endometrial
adenocarcinoma or the glycogen-rich cells associated with
pregnancy.

e. Brenner tumor
It is similar to the transitional lining of the renal pelvis or bladder.
This ovarian tumor is associated with benign mucinous
cystadenomas of the ovary. Most Brenner tumors are benign,
but borderline (proliferative Brenner tumor) and malignant
counterparts have been reported.
f. Cystadenofibromas
These are variants in which there is more pronounced proliferation
of the fibrous stroma that underlies the columnar lining
epithelium. They may be composed of mucinous, serous,
endometrioid, and transitional (Brenner tumors) epithelium.

Fig. 10: Granulosa cell tumor with Call-exner Body.

B. Sex-Cord Stromal Tumors

Examples of ovarian stromal tumors include thecomas, fibromas,


granulosa cell tumors, and Sertoli-Leydig cell tumors.
Thecomas are composed of spindle-shaped cells with vacuolated
cytoplasm. They are vacuolated because of steroid hormone
(estrogen) production, which can be stained with an Oil Red O
stain.
• Fibromas are also composed of spindle-shaped cells, but they
do not produce steroid hormones and are Oil Red O-negative.
• Granulosa cell tumor: These are the most common type of
ovarian tumor that is composed of cells that stain positively
with inhibin. Histologically, the cells may form Call-Exner
bodies, which are gland-like structures formed by the tumor
cells aligning themselves around a central space that is filled
with acidophilic material. The tumor cells may secrete
estrogens and cause precocious sexual development in girls or
increase the risk for endometrial hyperplasia and carcinoma in
women. Less commonly granulosa cell tumors can secrete
androgens and produce masculinization.

• Sertoli-Leydig tumors (Androblastomas): These also may


secrete androgens and produce virilization in women. The
tumor cells may stain positively with inhibin, but Call-Exner
bodies are not present. Granulosa cell tumors vary in their
clinical behavior, but they are considered to be potentially
malignant.
• Hilus cell tumors (Pure Leydig cell tumor): The ovarian
hilum normally contains clusters of polygonal cells arranged
around vessels (hilar cells). Hilus cell tumors are rare tumors
derived from these cells and are mostly unilateral. These are
characterized histologically by large lipid-laden cells with
distinct borders. Typically, patients with hilus cell tumors
present with evidence of masculinization, hirsutism, voice
changes, and clitoral enlargement. True hilus cell tumors are
almost always benign.


• Small cell carcinoma of the ovary is the another tumor of
possible stromal origin. These malignant tumors occur
predominantly in young women and may be associated with
hypercalcemia.

C. Germ Cell Tumors

a. Teratomas: These are divided into three categories: mature


(benign), immature (malignant), monodermal or highly
specialized.
Mature (Benign) Teratomas:
• Cystic teratomas are usually found in young women during
the active reproductive years.

Characteristically, they are unilocular cysts containing hair and cheesy


sebaceous material.
Within the wall, it is common to find tooth structures and areas of
calcification.
Fig. 11: Dermoid cyst.

• On histologic examination, the cyst wall is composed of


stratified squamous epithelium with underlying sebaceous
glands, hair shafts, and other skin adnexal structures.
• In most cases, structures from other germ layers can be
identified, such as cartilage, bone, thyroid tissue, and other
organoid formations.
• About 1% of the dermoids undergo malignant
transformation of any one of the component elements
(e.g., thyroid carcinoma, melanoma, but most commonly,
squamous cell carcinoma).
• These tumors arise from an ovum after the first meiotic
division.
Monodermal or Specialized Teratomas:
The most common of the specialized teratoma are struma
ovarii and carcinoid. They are always unilateral.
Struma ovarii is composed entirely of mature thyroid tissue.
Interestingly, these thyroidal neoplasms may hyperfunction, causing
hyperthyroidism.

• The ovarian carcinoid presumably arises from intestinal


epithelium in a teratoma and may result in carcinoid
syndrome. Primary ovarian carcinoid can be distinguished
from metastatic intestinal carcinoid, the latter virtually
always bilateral.
• Even more rare is the strumal carcinoid, a combination of
struma ovarii and carcinoid in the same ovary.
Immature Malignant Teratomas:
• These are rare tumors that differ from benign teratomas in
that the component tissue resembles that observed in the
fetus or embryo rather than in the adult.
• The tumor is found chiefly in prepubertal adolescents and
young women, the mean age being 18 years.
• On microscopic examination, there are varying amounts of
immature tissue differentiating toward cartilage, glands,
bone, muscle, nerve, and others.
b. Dysgerminoma
• It is the ovarian counterpart of the seminoma of the
testis.
• Similar to the seminoma, it is composed of large vesicular
cells having a clear cytoplasm, well-defined cell
boundaries, and centrally placed regular nuclei.
• Most of these tumors have no endocrine function. A few
produce elevated levels of chorionic gonadotropin and may
have syncytiotrophoblastic giant cells on histologic
examination.
• These are usually unilateral (80% to 90%) and solid.

• On histologic examination, the dysgerminoma cells are


dispersed in sheets or cords separated by scant fibrous
stroma. As in the seminoma, the fibrous stroma is
infiltrated with mature lymphocytes and occasional
granulomas.
c. Endodermal Sinus (Yolk Sac) Tumor
• It is the second most common malignant tumor of germ cell
origin.
• It is thought to be derived from differentiation of malignant
germ cells toward extraembryonic yolk sac structure.
• Similar to the yolk sac, the tumor is rich in α-fetoprotein and
α1-antitrypsin.

d. Choriocarcinoma
• More commonly of placental origin, the choriocarcinoma,
similar to the endodermal sinus tumor, is an example of
extraembryonic differentiation of malignant germ cells.
• Most ovarian choriocarcinomas exist in combination with
other germ cell tumors, and pure choriocarcinomas are
extremely rare.
• Like all choriocarcinomas, they elaborate high levels of
chorionic gonadotropins that are sometimes helpful in
establishing the diagnosis or detecting recurrences.

e. Other Germ Cell Tumors


These include embryonal carcinoma (another highly malignant
tumor of primitive embryonal elements, histologically similar to
tumors arising in the testes), polyembryoma (a malignant
tumor containing so-called embryoid bodies) and mixed germ
cell tumors (containing various combinations of
dysgerminoma, teratoma, endodermal sinus tumor, and
choriocarcinoma).

Gonadoblastoma is an uncommon tumor thought to be composed


of germ cells and sex cord-stroma derivatives. It occurs in
individuals with abnormal sexual development and in gonads of
indeterminate nature. Eighty per cent of patients are phenotypic
females, and 20% are phenotypic males with undescended
testicles and female internal secondary organs. On microscopic
examination, the tumor consists of nests of a mixture of germ cells
and sex cord derivatives resembling immature Sertoli and
granulosa cells. A coexistent dysgerminoma occurs in 50% of the
cases. The prognosis is excellent.
D. Metastatic tumors of ovary
• The most common “metastatic” tumors of the ovary are
probably derived from tumors of Mullerian origin: the uterus,
fallopian tube, contralateral ovary, or pelvic peritoneum.
• The most common extramullerian primaries are the breast and
gastrointestinal tract, including colon, stomach, biliary tract,
and pancreas.
• Also included in this group are the rare cases of
pseudomyxoma peritonei, derived from appendiceal tumors.
• Krukenberg tumor classically refers to a metastatic ovarian
malignancy whose primary site arose in the gastrointestinal
tract or breast. Microscopically, they are characterized by
appearance of mucin-secreting signet-ring cells in the tissue
of the ovary; when the primary tumor is discovered, the same
signet-ring cells are typically found. Carcinomas of colon,
appendix, and breast (mainly invasive lobular carcinoma) are
the next most common primary sites. Rare cases of
Krukenberg tumor originating from carcinomas of the
gallbladder, biliary tract, pancreas, small intestine, ampulla of
Vater, cervix, and urinary bladder/urachus have been reported

Fig. 12: Krukenberg tumor having signet ring cells.

SECONDARY AMENORRHEA
Secondary amenorrhea refers to absent menses for 3 months in a
woman who had previously had menses. Causes of secondary
amenorrhea include pregnancy (the most common cause),
hypothalamic/pituitary abnormalities, ovarian disorders, and end
organ (uterine) disease. The remainder of the disorders causing
secondary amenorrhea can be differentiated by examining
gonadotropin (FSH and LH) levels along with the results of a
progesterone challenge test.

• Withdrawal bleeding following progesterone administration


indicates that the endometrial mucosa had been primed with
estrogen, which, in turn, indicates that the
hypothalamus/pituitary axis and ovaries are normal.
• Hypothalamic/pituitary disorders, which are characterized by
decreased FSH and LH levels, include functional gonadotropin
deficiencies, such as can be seen in patients with a weight loss
syndrome. In these patients, markedly decreased body weight
(> 15% below ideal weight) causes decreased secretion of
GnRH from the hypothalamus. Decreased gonadotropin levels
decrease estrogen levels, which results in amenorrhea and an
increased risk for osteoporosis. Because of the decreased
estrogen levels, progesterone challenge does not result in
withdrawal bleeding.
• Ovarian conditions, such as surgical removal of the ovaries,
would most likely produce elevated gonadotropin levels due to
the lack of negative feedback from estrogen and progesterone.
Because of the decreased estrogen levels, a progesterone
challenge would not result in withdrawal bleeding.
• Uterine (end organ) disorders are characterized by normal
FSH and LH levels. An example is Asherman’s syndrome.
(Describe alongside) A patient with Asherman’s syndrome
would have no response to progesterone.

GESTATIONAL TROPHOBLASTIC DISEASES

The diseases in this category include; benign hydatidiform mole


(partial and complete), invasive mole, placental site trophoblastic
tumor and choriocarcinoma.
1. Hydatidiform mole: Both partial and complete, are
composed of avascular, grape-like structures that do not invade
the myometrium.
Differentiating features between partial and complete mole

• In complete (classic) moles, all the chorionic villi are abnormal and
fetal parts are not found. In partial moles, only some of the villi are
abnormal and fetal parts may be seen.
• Complete moles have a 46, XX diploid pattern and arise from the
paternal chromosomes of a single sperm by a process called
androgenesis. In contrast, partial moles have a triploid or a tetraploid
karyotype and arise from the fertilization of a single egg by two
sperm.
• Another way to differentiate these two disorders is to use
immunostaining for p57, which is a gene that is paternally imprinted
(inactivated). Because the complete mole arises only from paternal
chromosomes, immunostaining for p57 will be negative.
2. Invasive moles: This is defined as a mole that penetrates and
may even perforate the uterine wall. There is invasion of the
myometrium by hydropic chorionic villi, accompanied by
proliferation of both cytotrophoblast and syncytiotrophoblast.
Hydropic villi may embolize to distant sites, such as lungs and
brain, but do not grow in these organs as true metastases.
3. Placental site trophoblastic tumor: In contrast to syncytial
cytotrophoblast, which is present on the chorionic villi,
intermediate trophoblast is found in the implantation site and
placental membranes. Intermediate trophoblasts may give rise
to placental site trophoblastic tumors (PSTTs). PSTTs comprise
less than 2% of gestational trophoblastic neoplasms and
present as neoplastic polygonal cells infiltrating the
endomyometrium. PSTTs may be preceded by a normal
pregnancy (one-half), spontaneous abortion (one-sixth), or
hydatidiform mole (one-fifth). Distinction of PSTTs from normal
exaggerated placental implantation site trophoblast may be
difficult and can be achieved by using biomarkers (Mel-Cam
and Ki-67) that detect increased proliferation in the
trophoblastic cells.
4. Gestational choriocarcinomas: These are composed of
malignant proliferations of both cytotrophoblasts and
syncytiotrophoblasts without the formation of villi, can arise
from either normal or abnormal pregnancies: 50% arise in
hydatidiform moles, 25% in cases of previous abortion, 22% in
normal pregnancies, and the rest in ectopic pregnancies or
teratomas. Both hydatidiform moles and choriocarcinomas
have high levels of human chorionic gonadotropin (hCG); the
levels are extremely high in choriocarcinoma unless
considerable tumor necrosis is present.
BREAST

Pain (mastalgia or mastodynia) is the most common breast


symptom.

Discrete palpable masses are the second most common breast


symptom. A breast mass usually does not become palpable until it
is about 2 cm in diameter. Approximately 50% of carcinomas arise
in the upper outer quadrant, 10% in each of the remaining
quadrants, and about 20% in the central or subareolar region.
Nipple discharge is a less common presenting symptom but is of
concern when it is spontaneous and unilateral. Bloody or serous
discharges are most commonly associated with benign lesions but,
rarely, can be due to a malignancy. The most common etiologies
for discharge are a solitary large duct papilloma, cysts, or
carcinoma.
The principal mammographic signs of breast carcinoma are
densities and calcifications:

• Densities. Most neoplasms grow as solid masses and are


radiologically denser than the intermingled connective and adipose
tissue of the normal breast. Mammography can detect masses
before they become palpable. The most common lesions that are
detected as densities are invasive carcinomas, fibroadenomas, and
cysts. Ductal carcinoma in situ (DCIS, or carcinoma limited to the
ductal system) rarely presents as a density.
• Calcifications. Calcifications are associated with secretory material,
necrotic debris, and hyalinized stroma. Calcifications associated with
malignancy are commonly small, irregular, numerous, and clustered
or linear and branching.

INFLAMMATIONS

Acute Mastitis
Almost all cases of acute mastitis occur during lactation usually
caused by Staphylococcus aureus.

Periductal Mastitis/Zuska disease


This condition is also known by the names of recurrent subareolar abscess or
squamous metaplasia of lactiferous ducts. Both women, as well as men, present
with a painful erythematous subareolar mass. The main histologic feature is
keratinizing squamous epithelium extending to an abnormal depth into the
orifices of the nipple ducts.

Mammary Duct Ectasia


This disorder tends to occur in the fifth or sixth decade of life,
usually in multiparous women, and, unlike periductal mastitis,
is not associated with cigarette smoking.
Patients present with a poorly defined palpable periareolar mass,
sometimes with skin retraction, often accompanied by thick, white
nipple secretions. This lesion is characterized chiefly by dilation of
ducts, inspissation of breast secretions, and a marked periductal
and interstitial chronic granulomatous inflammatory reaction.

Fat Necrosis

Fat necrosis can present as a painless palpable mass, skin


thickening or retraction, a mammographic density, or
mammographic calcifications.
BENIGN EPITHELIAL LESIONS

NONPROLIFERATIVE BREAST CHANGES (FIBROCYSTIC


CHANGES)

Three principal patterns of morphologic change:


1. Cyst formation, often with apocrine metaplasia;
2. Fibrosis; and
3. Adenosis
Cysts: Small cysts form by the dilation and unfolding of lobules.
When cystic lobules coalesce, larger cysts are formed. Unopened
cysts are brown to blue (blue-dome cysts) owing to the contained
semitranslucent, turbid fluid. Cysts are lined either by a flattened
atrophic epithelium or by cells altered by apocrine metaplasia.
Metaplastic cells have an abundant granular, eosinophilic
cytoplasm, with round nuclei, resembling the apocrine epithelium
of sweat glands.

Fibrosis: Cysts frequently rupture, with release of secretory


material into the adjacent stroma. The resulting chronic
inflammation and fibrous scarring contribute to the palpable
firmness of the breast.
Adenosis: Adenosis is defined as an increase in the number of
acini per lobule.
The acini are often enlarged (blunt duct adenosis) and are not
distorted as is seen in sclerosing adenosis.

Proliferative Breast Disease without Atypia


This group of disorders is characterized by proliferation of ductal
epithelium and/or stroma without cellular abnormalities suggestive
of malignancy. The following entities are included in this category:
(1) moderate or florid epithelial hyperplasia, (2) sclerosing
adenosis, (3) complex sclerosing lesions, (4) papillomas, and (5)
fibroadenoma with complex features.

Epithelial Hyperplasia
In the normal breast, only myoepithelial cells and a single layer of
luminal cells are present above the basement membrane. Epithelial
hyperplasia is defined by the presence of more than two cell layers.
Sclerosing Adenosis
• The number of acini per terminal duct is increased to at least twice the
number found in uninvolved lobules. The normal lobular arrangement
is maintained. The acini are compressed and distorted in the central
portions of the lesion but characteristically dilated at the periphery.
Myoepithelial cells are usually prominent.
Complex Sclerosing Lesion (Radial Scar)
Radial scars are stellate lesions characterized by a central nidus of
entrapped glands in a hyalinized stroma.
• Papillomas
Papillomas are composed of multiple branching fibrovascular cores,
each having a connective tissue axis lined by luminal and
myoepithelial cells. Growth occurs within a dilated duct.
Small duct papillomas have been shown to be a component of
proliferative breast disease and increase the risk of subsequent
carcinoma.

Proliferative Breast Disease with Atypia


Proliferative disease with atypia includes atypical ductal
hyperplasia (ADH) and atypical lobular hyperplasia (ALH).
• ADH is recognized by its histologic resemblance to ductal carcinoma in situ,
including a monomorphic cell population, regular cell placement, and round
lumina. However, the lesions are characteristically limited in extent, and the
cells are not completely monomorphic in type or they fail to completely fill
ductal spaces.
• ALH refers to a proliferation of cells identical to those of LCIS (described
later), but the cells do not fill or distend more than 50% of the acini within a
lobule.

Non-proliferative changes do not increase the risk of cancer.


Proliferative disease is associated with a mild increase in risk.
Proliferative disease with atypia (ADH and ALH) confers a
moderate increase in risk.

CARCINOMA OF THE BREAST

Carcinoma is the most common malignancy of the breast, and


breast cancer is the most common non-skin malignancy in women.
PROGNOSTIC FACTORS
Major

Invasive carcinoma has worse prognosis than in-situ carcinoma


Distant metastasis indicates bad prognosis.
Axillary lymph node involvement is associated with worse prognosis.
Tumor Size: Less than 1 cm good prognosis, more than 2 cm bad
prognosis.
Local invasion into skeletal muscle carries poor prognosis.
Inflammatory carcinoma has poor prognosis.

Minor
• Histological type: Invasive ductal carcinoma (no special type; NST) carries
poor prognosis. Special types have good prognosis except medullary.
• Nottingham histological score (Scarff-Bloom-Richardson grade): Grade
1 good prognosis, grade 3 poor
• Estrogen and Progesterone receptor positivity indicates good response to
antiestrogen therapy.
HER2/neu overexpression: Poor prognosis
• Lymphovascular invasion: poor prognosis
High proliferative rate indicates worse prognosis
• Aneuploidy indicates bad prognosis
• The major risk factors for the development of breast cancer are hormonal
and genetic (family history). Breast carcinomas can, therefore, be divided
into sporadic cases, possibly related to hormonal exposure, and hereditary
cases, associated with family history or germ-line mutations.
Hereditary Breast Cancer
Mutated BRCA1 also markedly increases the risk of developing
ovarian carcinoma, which is as high as 20 to 40%. BRCA2 confers
a smaller risk for ovarian carcinoma (10 to 20%) but is associated
more frequently with male breast cancer. BRCA1 and BRCA2
carriers are also susceptible to other cancers, such as colon,
prostate, and pancreas, but to a lesser extent. BRCA1-associated
breast cancers are commonly poorly differentiated, have medullary
features and do not express hormone receptors or HER2/neu (so
called, triple negative phenotype). Their gene profile signature is
similar to basal-like breast cancers. These are frequently
associated with loss of inactive X-chromosome and reduplication
of active X, resulting in absence of Barr body. BRCA2 are also
poorly differentiated but are more commonly estrogen receptor
positive.

Sporadic Breast Cancer


The major risk factors for sporadic breast cancer are related to
hormone exposure: gender, age at menarche and menopause,
reproductive history, breast-feeding, and exogenous
estrogens. The majority of these cancers occur in
postmenopausal women and overexpress estrogen receptors
(ER).
CLASSIFICATION OF BREAST CARCINOMA

ALMOST ALL BREAST MALIGNANCIES ARE


ADENOCARCINOMAS, ALL OTHER TYPES (I.E., SQUAMOUS
CELL CARCINOMAS, PHYLLODES TUMORS, SARCOMAS,
AND LYMPHOMAS) MAKING UP FEWER THAN 5% OF THE
TOTAL.

HER2-
Positive (ER- ER-Negative
Defining Positive or HER2-
Features ER-positive, HER2-negative Negative) Negative
Frequency -40-55% (Low 10% (High -20% -15%
proliferation) proliferation)
Included Well or Poorly Some Medullary,
special moderately differentiated apocrine adenoid
histologic differentiated lobular cystic,
types lobular, tubular, secretory,
mucinous metaplastic
Typical Older women, BRCA2 Young Young
patient groups men, cancers mutation women, women,
detected by carriers non-white BRCA1
mammographic women, mutation
screening TP53 carriers,
mutation African and
carriers (ER American and
positive) Hispanic
women
Metastatic Bone (70%) Bone (80%) Bone (70%), Bone (40%),
pattern more common more common visceral visceral
than visceral than (30%) and (35%) and
(25%) or brain visceral(30%) brain(30%) brain (25%)
(>10%) or brain are all are all
(>10%) common common
Relapse Late, >10 years, Intermediate Usually short, Usually short,
Pattern long survival <10 years, <5years,
possible with survival with survival with
metastases metastases metastases
rare rare
Complete <10% -10% ER Positive- -30%
response to 15%
chemotherapy ER Negative-
30%

Carcinomas are divided into in situ carcinomas and invasive


carcinomas.
Noninvasive carcinomas (carinoma in situ) may be located
within the ducts (intraductal carcinoma) or within the lobules
(lobular carcinoma in situ). There are several variants of intraductal
carcinoma, including comedocarcinoma, cribriform carcinoma, and
intraductal papillary carcinoma. Comedocarcinoma grows as a
solid intraductal sheet of cells with a central area of necrosis, that
commonly undergoes calcification. It is frequently associated with
the erb B2/neu oncogene and a poor prognosis. Cribriform
carcinoma is characterized by round, ductlike structures within the
solid intraductal sheet of epithelial cells, while intraductal papillary
carcinoma has a predominant papillary pattern.
Infiltration of the nipple by large cells with clear cytoplasm is
diagnostic of Paget’s disease. These cells are usually found both
singly and in small clusters in the epidermis. Paget’s disease is
always associated with (in fact, it begins with) an underlying
intraductal carcinoma that extends to infiltrate the skin of nipple
and areola. Paget cells may resemble the cells of superficial
spreading melanoma, but they are PAS-positive and diastase-
resistant (mucopolysaccharide-or mucin-positive), unlike
melanoma cells.

INVASIVE (INFILTRATING) CARCINOMA


Invasive breast carcinoma is divided into two main types:
• No- special type carcinoma [Intraductal]
• Special carcinoma
– Lobular
– Cribriform
– Colloid
– Medullary
– Papillary
– Metaplastic

Invasive carcinomas of no special type include the majority of


carcinomas (70 to 80%) that cannot be classified as any other
subtype.

Invasive Carcinoma, No Special Type (NST; Invasive Ductal


Carcinoma)
On gross examination, most carcinomas are firm to hard and have
an irregular border. Within the center of the carcinoma, there are
small pinpoint foci or streaks of chalky white elastotic stroma and
occasionally small foci of calcification. There is a characteristic
grating sound (similar to cutting a water chestnut) when cut or
scraped. Five major patterns of gene expression in the NST group
are noted:

• Luminal A (40-55%): ER positive and HER2/neu negative. These are


generally slow growing and respond to hormonal treatments.
• Luminal B (15-20%): Triple positive cancers i.e. ER, PR and
HER2/neu positive. These are of higher grade and more likely to
have lymph node metastasis.
• Normal breast-like (6-10%): Well-differentiated ER positive and
HER2/neu negative.
• Basal-like (13-25%): Triple negative cancers i.e. ER, PR and
HER2/neu negative. Express markers of typical myoepithelial cells
(e.g. basal keratins, P-cadherin, p63 or laminin), progenitor cells or
putative stem cells (cytokeratin 5 and 6). Members of this group
include medullary carcinoma, metaplastic carcinoma (e.g. spindle
cell carcinoma or matrix producing carcinoma) and carcinomas with
a central fibrotic focus. Many cancers in women with BRCA-1
mutations are of this type.
• HER2 positive (7-12%): ER negative but overexpress HER2/neu. In
more than 90%, it is due to amplification of the segment of DNA on
chromosome 17q21.

Lobular Carcinoma
Lobular carcinoma (invasive) of breast is one of the very few
carcinomas which are seen bilaterally. Histologic hallmark is the
presence of dyscohesive infiltrating tumor cells, often arranged in
single file or in loose clusters or sheets. Tubule formation is absent.
Signet ring cells containing intracytoplasmic mucin droplets are
common. It metastasizes frequently to peritoneum,
retroperitoneum, leptomeninges, GIT and ovaries. The incidence of
this carcinoma is increasing among postmenopausal females
presumably because of increasing use of HRT.

Medullary Carcinoma
The tumor has a soft, fleshy consistency (medulla is Latin for
“marrow”) and is well-circumscribed. The carcinoma is
characterized by

• Solid, syncytium-like sheets (occupying more than 75% of the tumor)


of large cells with vesicular, pleomorphic nuclei, containing prominent
nucleoli
• Frequent mitotic figures
• A moderate to marked lymphoplasmacytic infiltrate surrounding and
within the tumor
• A pushing (non-infiltrative) border.

Medullary carcinomas have slightly better prognosis than do


NST carcinomas, despite the almost universal presence of poor
prognostic factors. These show overexpression of E-cadherin and
basal like gene expression.

Mucinous (Colloid) Carcinoma


The tumor cells are seen as clusters and small islands of cells
within large lakes of mucin that push into the adjacent stroma.

Tubular Carcinoma
These tumors consist exclusively of well-formed tubules. However,
a myoepithelial cell layer is absent, and tumor cells are in direct
contact with stroma.

Invasive Papillary Carcinoma


Invasive carcinomas with a papillary architecture are rare and
represent 1% or fewer of all invasive cancers. Papillary
architecture is more commonly seen in DCIS.
Metaplastic Carcinoma
“Metaplastic carcinoma” includes a wide variety of rare types of
breast cancer (<1% of all cases), including conventional
adenocarcinomas with a chondroid stroma, squamous cell
carcinomas, and carcinomas with a prominent spindle cell
component that might be difficult to distinguish from sarcomas.
Some of these carcinomas express genes in common with
myoepithelial cells and likely to arise from this cell type.
Multiple Choice Questions
MALE GENITAL TRACT

1. Alpha fetoprotein is Not raised in which testicular


tumors?
(AI 2010)
(a) Choriocarcinoma
(b) Teratocarcinoma
(c) Yolk sac tumor
(d) Embryonal cell carcinoma
2. Which one of the following is not used as a tumor marker
in testicular tumors?
(AI 2005) (DNB 2007)
(a) AFP
(b) LDH
(Kolkata 2008)
(c) hCG
(d) CEA
3. All of the following statements about Gleason grading
system are true except:
(AIIMS Nov 2008)
(a) Score range from 1 to 10
(b) High score is associated with bad prognosis
(c) Helps in grading of tumor
(d) Helps decide treatment modality
4. Infertility is a common feature in “Sertoli cell only”
syndrome because:
(AIIMS May 2003)
(a) Too many Sertoli cells inhibit spermatogenesis via inhibin
(b) Proper blood testis barrier is not established
(c) There is no germ cell in this condition
(d) Sufficient numbers of spermatozoa are not produced
5. Predisposing factors for germ cell tumor are:
(a) Cryptorchidism
(PGI Dec 2002)
(b) Testicular feminizing syndrome
(c) Klinefelter’s syndrome
(d) Smoking
(e) Right side more common than left side.
6. In the testis intratubular germ cell neoplasia is seen in all,
except:
(Delhi 2009 RP)
(a) Seminomas
(b) Spermatocytic seminoma
(c) Yolk sac tumor of testis
(d) Embryonal carcinoma
7.Condyloma are mostly caused by HPV types:
(a) 11 and 13
(b) 6 and 11
(Delhi PG-2006)
(c) 6 and 13
(d) 30 and 33
8. Which one of the following is not used as a tumor marker
in testicular tumors?
(DNB- 2007)
(a) AFP
(b) LDH
(c) hCG
(d) CEA
9. Gleason’s classification is used for:
(UP 2008)
(a) Carcinoma breast
(b) Carcinoma prostate
(c) Carcinoma pancreas
(d) Carcinoma rectum
10. Which of the following is not a malignant tumor of germ
cell origin?
(AP 2002)
(a) Mature teratoma
(b) Choriocarcinoma
(c) Dysgerminoma
(d) Embryonal carcinoma
11. Metastasis is least common with:
(AP 2007)
(a) Embryonal cell carcinoma
(b) Endodermal sinus tumor
(c) Teratocarcinoma
(d) Spermatocytic seminoma
12. A 25-year-old man, Ramesh presents with a testicular
mass and is found to have high serum levels of α-
fetoprotein (AFP). Microscopic examination of a biopsy
from this mass reveals sheets of undifferentiated cells
along with focal primitive glandular differentiation. The
tumor cells have large and hyperchromatic nuclei.
Further workup fails to reveal the presence of any
metastatic disease as the tumor is confined within the
testis. Based on all of these findings, which of the
following best characterizes this tumor?
Tumor Aggressiveness Grade Stage
(a) Benign Low Low
(b) Benign Low High
(c) Malignant Low Low
(d) Malignant High Low
(e) Malignant High High
13. In which of the following respects do a seminoma
involving the testis and a dysgerminoma involving the
ovary differ most significantly?
(a) Most common age of presentation
(b) Number of mitoses
(c) Potential to contain foci of more aggressive tumors
(d) Ultrastructural appearance
14. A 10-year-old child develops a testicular mass and
undergoes orchiectomy. On cut section, the mass shows
a variety of appearances and colors. Histologically, many
different tissues are seen, including cartilage, thyroid,
and neural tissue. A small focus of clear-cut squamous
cell carcinoma is seen. Which of the following is the
most appropriate classification for this tumor?
(a) Dermoid cyst
(b) Teratoma with malignant transformation
(c) Immature teratoma
(d) Solid mature teratoma

MOST RECENT QUESTIONS

15. Benign hyperplasia of prostate first develops in:


(a) Central zone
(b) Peripharal zone
(c) Periurethral transition zone
(d) Any of the above
16. The commonest site for extragonadal germ cell tumour is:
(a) Pineal gland
(b) Mediastinum
(c) Retroperitoneum
(d) Sacrococyygeal region
17. Which one of the following is not used as a tumor marker
in testicular tumors?
(a) AFP
(b) LDH
(c) HCG
(d) CEA
18. All of the following genes are associated with seminoma
except:
(a) PLAP
(b) C-KIT
(c) OCT-4
(d) CDK-4

FEMALE GENITAL TRACT


19. The cytogenicity of solid tumors is not easily assessed
especially in carcinoma cervix because
(a) Metaphase is distinct
(AIIMS Nov 2010)
(b) Due to contamination with infectious agents
(c) High mitotic rate
(d) Deficient tissue sample
20. With regard to the malignant behavior of leiomyosarcoma,
the most important criterion is:
(a) Blood vessel penetration by tumor cells
(AI 2006)
(b) Tumor cells in lymphatic channels
(c) Lymphocyte infiltration
(d) The number of mitoses per high power field
21. All are true about polycystic ovarian disease except:
(a) Persistently elevated LH
(AIIMS Nov 2008)
(b) Increased LH/FSH ratio
(c) Increased DHEAS
(d) Increased prolactin
22. Sections from a solid-cystic unilateral ovarian tumor in a
30-year old female show a tumor composed of diffuse
sheets of small cells with doubtful nuclear grooving and
scanty cytoplasm. No Call-Exner bodies are seen. The
ideal immunohistochemistry panel would include:
(AIIMS May 2006)
(a) Vimentin, epithelial membrane antigen, inhibin, CD99
(b) Desmin, S-100 protein, smooth muscle antigen,
cytokeratin
(c) Chromogranin, CD45, CD99, CD20
(d) CD3, Chromogranin, CD 45, Synaptophysin
23. An ovarian neoplasm in a 14-year old girl is most likely to
be:
(Delhi 2009)
(a) Germ cell tumor
(b) Epithelial tumor
(c) Sertoli-Leydig cell tumor
(d) Granulosa cell tumor
24.The incidence of bilaterality in a dermoid cyst is
approximately:
(Delhi 2009)
(a) 10%
(b) 30%
(c) 50%
(d) 70%
25. The risk of sarcoma developing in a fibroid uterus is
approximately:
(Delhi 2009)
(a) < 1%
(b) 10%
(c) 30%
(d) 50%
26. Uterine leiomyoma is least likely to undergo:
(a) Malignant change
(Delhi PG-2005)
(b) Hyaline change
(c) Calcification
(d) Red degeneration
27. Carcinosarcoma occurs in:
(UP 2003)
(a) Uterus
(b) Liver
(c) Breast
(d) Lungs
28. Schiller–Duval bodies are seen in:
(UP 2005, 2007)
(a) Teratoma
(b) Seminoma
(c) Yolk-Sac tumor
(d) Choriocarcinoma
29. Call-Exner bodies are seen in:
(UP 2007)
(a) Mature teratoma
(b) Endodermal sinus tumor
(c) Granulosa cell tumor
(d) Sertoli Leydig cell tumor
30. Hormone produced by endodermal sinus tumor is
(a) AFP
(b) Alpha1 antitrypsin
(c) Both
(d) hCG
(RJ 2002)
31. Choriocarcinoma is characterized by all except:
(a) Primarily trophoblastic tumor
(AP 2004)
(b) It can occur following hydatidiform mole
(c) Villi present
(d) It can metastasize to lungs
32. Call-Exner bodies are characteristic feature of (AP
2005)
(a) Granulosa theca cell tumor
(b) Brenner tumor
(c) Dysgerminoma
(d) Endodermal sinus tumor
33. Tennis Racquet cells are seen in :
(AP 2007)
(a) Rhabdomyoma
(b) Rhabdomyosarcoma
(c) Histiocytoma
(d) Eosinophilic granuloma
34. A 30-year-old woman Shagun visits her gynecologist for a
surgery. After laparotomy, a mass is removed which on
microscopic examination demonstrates a cystic cavity
filled with hair and keratin debris, and the wall contains
skin, adnexal tissue, thyroid tissue, and neural tissue. All
of the tissues are similar to those normally found, and no
malignant changes are seen. Which of the following is
the most likely diagnosis?
(a) Immature teratoma
(b) Leiomyoma
(c) Leiomyosarcoma
(d) Mature teratoma
35. Bilateral ovarian masses are identified on pelvic
examination of a 40-year-old woman for which she
undergoes total abdominal hysterectomy. Pathologic
examination demonstrates papillary carcinoma
producing serous fluid. Which of the following tumor
markers would be most useful in monitoring for
recurrence?
(a) Alpha-fetoprotein
(b) Bombesin
(c) CA-125
(d) PSA

MOST RECENT QUESTIONS


36. A patient with chronic pelvic pain undergoes a
hysterectomy. The resected uterus is filled with nodules
composed of benign smooth muscle cells. Which of the
following terms best describes these nodules?
(a) Angiosarcoma
(b) Leiomyoma
(c) Leiomyosarcoma
(d) Rhabdomyoma
37. Endodermal sinus tumor is characterized by:
(a) Call Exner body
(b) Psammoma body
(c) Schiller duval body
(d) Homer wright body
38. An adenofibroma of the ovary in which the epithelial
component consists of the nests of transitional cells is
called:
(a) Thecoma
(b) Brenner tumour
(c) Serous cystadenoma
(d) Granulose cell tumor
39. A lady with abdominal mass was investigated. On
surgery, she was found to have bilateral ovarian masses
with smooth surface. On microscopy they revealed
mucin secreting cells with signet ring shape. What is the
most likely diagnosis?
(a) Dysgerminoma
(b) Krukenberg tumor
(c) Mucinous adenocarcinoma of the ovaries
(d) Dermoid cyst
40. Most common subtype of endometrial carcinoma on
histopathology is:
(a) Clear cell
(b) Adenocarcinoma
(c) Squamous cell carcinoma
(d) Transistional cell carcinoma
41. All are true about condylomata acuminata except which
tumor?
(a) Caused by HPV
(b) Sexually transmitted
(c) Progresses to malignancy
(d) Occurs in genitalia
42. Risk factor for endometrial carcinoma is all except:
(a) Obesity
(b) Smoking
(c) Infertility
(d) Tamoxifen
43. Most common ovarian tumor:
a. Fibroma
b. Teratoma
c. Mucinous cystadenoma
d. Serous cystadenoma

BREAST TISSUE

44. Lesions affecting the teminal duct lobulat unit (TDLU) in


breast are all except
(DPG 2011)
(a) Nipple adenoma
(b) Blunt duct adenosis
(c) Intraductal papilloma
(d) Fibroadenoma
45.Dimorphic carcinoma is:
(a) Follicular carcinoma thyroid
(b) Papillary carcinoma thyroid
(c) Papillary carcinoma breast
(d) Gastric adenocarcinoma
46. The type of mammary ductal carcinoma in situ (DCIS)
most likely to result in a palpable abnormality in the
breast is:
(AI 2006)
(a) Apocrine DCIS
(b) Neuroendocrine DCIS
(c) Will-differentiated DCIS
(d) Comedo DCIS
47. BRCA 1 gene is located on:
(AIIMS Nov 2008)
(a) Chromosome 13
(b) Chromosome 11
(c) Chromosome 17
(d) Chromosome 22
48. Increased susceptibility to breast cancer is likely to be
associated with a mutation in the following gene:
(a) p53
(b) BRCA-1
(AIIMS Nov 2004)
(c) Retinoblastoma (Rb)
(d) H-Ras
49. A female patient presented with a firm mass of 2 × 2 cm in
the upper outer quadrant of the breast. She gives a
family history of ovarian carcinoma. The investigation
that needs to be done to assess for mutations is:
(a) p53
(b) BRCA-2
(AIIMS May 2002)
(c) Her 2/Neu gene
(d) C-myc gene
50. Bilateral breast carcinoma is:
(PGI June 2002)
(a) Scirrhous carcinoma
(b) Medullary carcinoma
(c) Lobular carcinoma
(d) Ductal carcinoma
(e) Paget’s carcinoma
51. Rare histological variants of carcinoma breast with better
prognosis include all except:
(Delhi 2009)
(a) Colloid carcinoma
(b) Medullary carcinoma
(c) Inflammatory carcinoma
(d) Tubular carcinoma
52. Tumor marker useful in the diagnosis of the cancer of the
breast is:
(Karnataka 2005)
(a) CEA
(b) AFP
(c) CA-125
(d) CA-15-3

MOST RECENT QUESTIONS


53. Histologic hallmark of Paget’s disease of nipple is:
(a) Caseous necrosis
(b) Infiltration of the epidermis by malignant cells
(c) Atypical lobular hyperplasia
(d) Desmoplasia
54. BRCA-1 gene lies on chromosome:
(a) 17
(b) 18
(c) 20
(d) 21
55. Commonest carcinoma of the breast with multifocal origin
is:
(UP 2001)
(a) Scirrhous carcinoma
(b) Adenocystic carcinoma
(c) Lobular carcinoma
(d) Ductal carcinoma
56. Which of the following breast tumors is
bilateral?
(a) Colloid carcinoma
(b) Invasive ductal carcinoma
(c) Invasive lobular carcinoma
(d) Medullary carcinoma
57. Indian file pattern is seen in histopathological
examination of:
(a) Infiltrating duct carcinoma
(b) Fibroadenoma
(c) Fibro carcinoma
(d) Lobular carcinoma
58. Fleshy, soft lymphatic infiltration of skin in breast cancer
appears as:
(a) Puckering
(b) Peau ‘d orange
(c) Cancer encurasse
(d) All of the above
59. A 54-year-old female Shanti presents for an annual exam.
Her right breast is swollen, red, and tender. The
physician palpates a firm area in the breast and suspects
inflammatory breast cancer. Which of the following best
describes the histological changes observed in this
disorder?
(a) Acute inflammation in breast carcinoma
(b) Chronic inflammation in breast carcinoma
(c) Dermal lymphatic invasion by cancer cells
(d) Epidermal invasion by cancer cells
60. BRCA-1 gene lies on chromosome:
(a) 17
(b) 18
(c) 20
(d) 21
61. Paget’s disease of the nipple is:
(a) Infection
(b) Dermatisis
(c) Neoplasia
(d) Hypopigmentation
62. Molecular classification of breast cancer is based on
which of the following?
(a) Gene expression profiling
(b) Expression of hormone receptors like ER,PR, and HER-
2 neu
(c) Histology
(d) Response to chemotherapy
63. HER-2/neu gene causes breast carcinoma due to:
(a) Overexpression
(b) Suppression
(c) Mutation
(d) Translocation
64. BRCA 1 responsible which type breast cancer?
(a) Medullary
(b) Lobular
(c) Colloid
(d) Secretory
65. Which antigen is not of prognostic significance in
carcinoma breast?
(a) Her 2 neu receptor
(b) Epithelial membrane antigen
(c) Estrogen receptor
(d) Progesterone receptor
66. Infiltrative lobular breast carcinoma is associated with
which of the following pattern of arrangement of cells?
(a) Pin wheel pattern
(b) Pleomorphic cells in sheets
(c) Cribiform pattern
(d) Single file pattern

CONCEPTUAL QUESTIONS
1-3. Will have two statements, assertion and reason. Read
both of them carefully and answer according to these
options.
(a) Both assertion and reason are true and reason is
correct explanation of assertion.
(b) Both assertion and reason are true and reason is not
the correct explanation of assertion.
(c) Assertion is true and reason is false.
(d) Both assertion and reason are false.
1. Assertion: Stromal cells are responsible for androgen
dependent prostatic growth
Reason: Stromal cells have 5α reductase activity which
produces testosterone in prostate.
2. Assertion: Struma ovarii is mature teratoma
Reason: It is responsible for production of the gonadotropins
3. Assertion: Seminoma is germ cell tumor with good
prognosis.
Reason: The tumor cells rarely have areas of necrosis and
hemorrhage
1. Ans. (a) Choriocarcinoma
(Ref: Robbins 9/e p978, 8th/327, 989-991, 7th/339)
AFP is a marker of hepatocellular cancer and non-seminomatous
germ cell tumors of testes.
Non-seminomatous germ cell tumors may be embryonal
carcinoma, yolk sac tumors, choriocarcinoma or teratoma.
• Embryonal cell carcinomas and Yolk sac tumor have
elevated AFP levels.
• Dorland’s dictionary 27th edition writes that
Teratocarcinoma refers to a germ cell tumor that is a
mixture of teratoma with embryonal carcinoma, or with
choriocarcinoma, or with both. So, it may be having
elevated levels of AFP.
• Choriocarcinomas have elevated levels of hCG which
can be readily demonstrated in the cytoplasm of
syncytiotrophoblastic cells.
2. Ans. (d) CEA
(Ref: Robbins 7th/1045, 9/e p979, Harrison 17th/602)
3. Ans. (a) Score range from 1 to 10
(Ref: Schwartz 8th/1216, Robbins 9/e 987)
• A Gleason score is given to prostate cancer based upon
its microscopic appearance. Cancers with a higher
Gleason score are more aggressive and have a worse
prognosis. Most tumors contain more than 1 pattern.
• The pathologist assigns a grade to the most common
tumor, and a second grade to the next most common
tumor. The two grades are added together to get a
Gleason score. For example, if the most common tumor
was grade 3, and the next most common tumor was
grade 4, the Gleason score would be 3 + 4 = 7.
• The Gleason grade ranges from 1 to 5, with 5 having the
worst prognosis. The Gleason score ranges from 2 to
10, with 10 having the worst prognosis.
• It should be noted that for Gleason score 7, a Gleason 4 +
3 is a more aggressive cancer than a Gleason 3 + 4.
Also, there is not really any difference between the
aggressiveness of a Gleason score 9 or 10 tumour.
• Gleason scores are associated with the following
features:

– Grade 1: The cancerous prostate closely resembles


normal prostate tissue. The glands are small, well-
formed, and closely packed
– Grade 2: The tissue still has well-formed glands, but they
are larger and have more tissue between them.

– Grade 3: The tissue still has recognizable glands, but the


cells are darker. At high magnification, some of these
cells have left the glands and are beginning to invade the
surrounding tissue.
– Grade 4: The tissue has few recognizable glands. Many
cells are invading the surrounding tissue.
– Grade 5: The tissue does not have recognizable glands.
There are often just sheets of cells throughout the
surrounding tissue

The Gleason score is used to help evaluate the prognosis of


men with prostate cancer. Together with other parameters, the
Gleason score is incorporated into a strategy of prostate
cancer staging which predicts prognosis and helps guide
therapy.
4. Ans. (c) There is no germ cells in this condition
(Ref: Anderson 10th 2177)
‘Sertoli cell only’ syndrome also called as Germ cell aplasia has
small seminiferous tubules. In this condition seminiferous
tubules are smaller than normal and are lined by a single
layer of Sertoli cells and no germ cells. Without germ cell,
spermatogenesis does not take place resulting in infertility.
5. Ans. (a) Cryptorchidism ; (b) Testicular feminising
syndrome; (c) Klinefelter syndrome; (e) Rt. side has more
common flow than Lt. side
(Ref: Robbins’ 7th/1041, 9/e p975, Harrison’ 16th/550)
The predisposing factors of germ cell tumors of testes are:
• Cryptorchidism
• Testicular feminization syndrome
• Klinefelter syndrome
• Excess 12P copy number either in the term of i(l2P) or
increased 12P an aberrantly banded marker
chromosome.
• Prior germ cell tumor
• Strong family history of germ cell tumor
6. Ans. (b) Spermatocytic seminoma
(Ref: Robbins 8th/988, 9/e p975,)
Most testicular germ cell tumors originate from intratubular germ
cell neoplasia (ITGCN).
ITGCN is seen adjacent to all germ cell tumors in adults except for
spermatocytic seminoma and epidermoid and dermoid cysts.
With rare exceptions, it is also not seen in pediatric tumors
(teratomas, yolk sac tumors… Robbins 7th edition page 1096)
So, obviously friends, if we have to choose between options b and
c, we would prefer option b as the answer. ITCGN is seen
with a high frequency in the following conditions:
• Cryptorchidism
• Prior germ cell tumors
• Strong family history of germ cell tumor
• Androgen insensitivity syndrome
• Gonadal dysgenesis syndrome.
Untreated ITGCN progresses to invasive germ cell tumor in
approximately 50% of cases over 5 years of follow-up.
7. Ans. (b) 6 and 11
(Ref: Robbins 7th/1035, 9/e p970)
• Condyloma are most commonly caused by HPV types 6
and 11.
• Condyloma acuminatum is a benign tumor caused by
human papilloma virus (HPV).
• It is related to the common wart (verruca vulgaris) and
may occur on any moist mucocutaneous surface of the
external genitalia in either sex.
• Also, HPV and associated diseases are sexually
transmitted.
• On the penis, these lesions occur most often about the
coronal sulcus and inner surface of the prepuce.
• They consist of single or multiple sessile or pedunculated,
red papillary excrescences that vary from 1 mm to
several mm in diameter.
• Histologically a branching, villous, papillary connective
tissue stroma is covered by a thickened hyperplastic
• Epithelium that may have considerable superficial
hyperkeratosis and thickening of the underlying
epidermis (acanthosis).
– The normal orderly maturation of the epithelial cells is
preserved.
– Clear vacuolization of the prickle cells (Koilocytosis),
characteristic of HPV infection, is noted in these
lesions.
– The basement membrane is intact, and there is no
evidence of invasion of the underlying stroma.
– Condyloma acuminata tend to recur but do not evolve
into invasive cancers.
8. Ans. (d) CEA
(Ref: Robbins 8th/327, 9/e p979)
9. Ans. (b) Carcinoma prostate
(Ref: Robbins 9/e p987)
10. Ans. (a) Mature teratoma (Mature teratoma is a benign
tumour)
(Ref: Robbins 9/e p979, 8th/1047; 7th/1093,1096)
11. Ans. (b) Endodermal sinus tumor
(Ref: Robbins 9/e p977)
12. Ans. (d) (Ref: Robbins 7th/335, 1043, 9/e p977,
Chandrasoma/307-308)
• An embryonal carcinoma is testicular malignancy that
secretes alpha-fetoprotein (a-AFP) and is composed of
undifferentiated cells along with primitive glandular
differentiation. Once the diagnosis ofmalignancy is
established, prognosis for the patient is estimated
through the process of grading and staging.
• It is important to understand the difference between
these two terms. First of all, note that these terms are
applied only to malignant neoplasms. Basically, grading
is done histologically, while staging is done
clinically.
• Lower grades, such as grades I and II, are more
differentiated, less aggressive and have a better
prognosis, while higher grades, such as grades III and
IV, are less differentiated, more aggressive and have a
worse prognosis. Tumors composed of malignant cells
that appear primitive or undifferentiated are classified as
high grade tumors.
• In contrast to grading, the staging of cancers is based on
the size of the primary lesion, the presence of lymph
node metastases, and the presence of blood-borne
metastases. Two main staging systems are Union
International Centre le Cancer (UICC) and American
Joint Committee (AJC), UICC classification is called the
TNM classification. AJC staging system generally
divides cancers into stages 0 through IV. Lower stage
tumors are smaller, localized, and have a better
prognosis, while higher stage tumors are larger,
widespread, and have worse prognosis.
13. Ans. (a) Most common age of presentation
(Ref: Robbins 8th/988, 1049, 9/e p976, 1030)
• Seminomas and dysgerminomas are very similar tumors
but differ in two significant respects: the most common
age of presentation in men is in the fourth decade, while
in women, it is in the third decade. Also, seminomas are
relatively common in men (30% of testicular germ cell
tumors), while dysgerminomas are rare in women (1% of
ovarian tumors).
• Both of these tumors are composed of sheets of uniform
polyhedral cells with intervening fibrous septa of
connective tissue, lymphocytes, and multinucleated
giant cells. The number of mitoses (choice B) per high-
power field and ultrastructural appearance (choice D) do
not differ greatly between the two tumors.
• These tumors in pure form are very radiosensitive but
can be much more aggressive (choice C) if foci of other
germ cell tumors (notably embryonal carcinoma,
choriocarcinoma, and yolk sac tumors) are present.
14. Ans. (b) Teratoma with malignant transformation
(Ref: Robbins 8th/991, 9/e p978)
• This is teratoma with malignant transformation. The
possibility of malignant transformation is the reason why
mature teratomas with very well differentiated tissues
should be completely excised. Malignant transformation
is more common in teratomas in adults than in children
or babies.
• Dermoid cyst (choice A) is a cystic form of mature
teratoma, usually found in the ovaries.
• Immature teratoma (choice C), while clinically malignant,
shows embryonal tissues and often displays no clear-cut
cytological evidence of malignancy.
• Solid mature teratoma (choice D) without the added
descriptor “with malignant transformation” is by definition
a benign tumor. Careful extensive sampling is required
to exclude minute foci of cancerous transformation.
15. Ans. (c) Periurethral transition zone
(Ref: Robbins 9/e p982)
16. Ans. (b) Mediastinum
(Ref: Robbins 9/e p475; Harsh Mohan 6/e p703)
17. Ans. (d) CEA
(Ref: Robbins 8/e 327, 9/e p979)
Carcino embryonic antigen is increased in cancers of the pancreas
and colon.
18. Ans. (d) CDK-4
(Ref: Robbins 9/e p976)
Seminomas contain iso chromosome 12p and express
OCT3/4 and NANOG. Approximately 25% of these tumors
have KIT activating mutations.
19. Ans. (b) Due to contamination with infectious agents (Ref:
Enzinger and Weiss Soft Tissue Tumors, 5th/73,
Wintrobes Hematology, 12th/50-60)
• Cytogenetics is the study of chromosome structure which
can be done with techniques like karyotyping or
molecular techniques (FISH, spectral
karyotyping/multicolor FISH, comparative genomic
hybridization). These techniques needs metaphase
(more commonly) but can be applied to interphase also.
Cytogenetics is easy if the metaphase is distinct. So,
culture is often done in solid tumors to get the cells in
metaphase.
• Cytogenetics is also easier if mitotic rate is high.
The problems with cytogenetics are:

• Unpredictable growth of the neoplastic cells in tissue culture


• Overgrowth of neoplastic cells by reactive non-neoplastic cells
• Contamination of tumor cultures by bacteria or fungi
• Predominance of nonviable tumor (necrotic sample)
Deficient tissue sampling like sampling from nonrepresentative
areas or the necrotic areas will impair the results in most of
the solid tumors but the commonest problem with cancer of
the cervix is the high contamination rate. So, option ‘B’ is
better than option ‘D’.
20. Ans. (d) The number of mitoses per high power field
(Ref: Robbins 7th/1090, 9/e p1021)
The most important criterion for distinction of
leiomyosarcoma from leiomyoma (malignant
transformation) is the number of mitoses present.
Ten high power fields (hpf) are examined. If > 10 mitoses
are seen, it signifies malignancy. If cellular atypia is also
present, > 5 mitoses are enough to make a diagnosis of
leiomyosarcoma.
21. Ans. (d) Increased Prolactin
(Ref: Shaw 13th/353, 9/e p1022)
The diagnosis of polycystic ovarian syndrome (PCOS) is
straightforward using the Rotterdam criteria, even when the
syndrome is associated with a wide range of symptoms.
Standard diagnostic assessments
• History-taking, specifically for menstrual pattern, obesity,
hirsutism, and the absence of breast discharge. A
clinical prediction rule found that these four questions
can diagnose PCOS with a sensitivity of 77.1% (95% CI
62.7-88.0%) and a specificity of 93.8% (95% CI 82.8-
98.7%).
• Gynecologic ultrasonography, specifically looking for small
ovarian follicles. These are believed to be the result of
disturbed ovarian function with failed ovulation, reflected
by the infrequent or absent menstruation that is typical
of the condition. In normal menstrual cycle, one egg is
released from a dominant follicle - essentially a cyst that
bursts to release the egg. After ovulation the follicle
remnant is transformed into a progesterone producing
corpus luteum, which shrinks and disappears after
approximately 12-14 days. In PCOS, there is a so called
“follicular arrest”, i.e. several follicles develop to a size of
5-7 mm, but not further. No single follicle reach the
preovulatory size (16 mm or more). According to the
Rotterdam criteria, 12 or more small follicles should be
seen in an ovary on ultrasound examination. The
follicles may be oriented in the periphery, giving the
appearance of a ‘string of pearls’. The numerous follicles
contribute to the increased size of the ovaries, that is,
1.5 to 3 times larger than normal.
• Laparoscopic examination may reveal a thickened,
smooth, pearl-white outer surface of the ovary.
• Serum (blood) levels of androgens (male hormones),
including androstenedione, testosterone and
dehydroepiandrosterone sulfate may be elevated: free
testosterone is more sensitive than total. Free
testosterone is reflected as the ratio of testosterone to
sex hormone-binding globulin (SHBG).
• Some other blood tests are suggestive but not diagnostic.
The ratio of LH (Luteinizing hormone) to FSH (Follicle
stimulating hormone) is greater than 1:1, as tested on
Day 3 of the menstrual cycle. The pattern is not very
specific and was present in less than 50% in one study.
There are often low levels of sex hormone binding
globulin, particularly among obese women.
22. Ans. (a) Vimentin, epithelial membrane antigen, inhibin,
CD99 (Ref.
Sternberg Pathology/2581, 2583, 2543, 2579, 2652
Ackerman’s Pathology/1694, 1675, 1681, 687)
The specimen in the given question is most likely to be of
granulosa cell tumor of ovary:
The features pointing toward this diagnosis are:
• Unilateral tumor
• Solid and cystic areas
• Small cells arranged in sheets
• Nuclear grooving
• Scant cytoplasm
The only point against this diagnosis is absence of Call-Exner
bodies, but note that these structures if found are diagnostic
of Granulosa cell tumors but these are not prerequisite for
diagnosis.
Vimentin, EMA, inhibin and CD99 all are the markers of granulosa
cell tumors.
23. Ans. (a) Germ cell tumor
(Ref: Robbins 8th/1047-1049, 9/e p1029, Harrison 17th/606,
604)
• Epithelial tumors of ovary usually occur in old age
• Germ cell tumors of ovary generally occur in younger
women. About 75% of these occur in women <30 years
old
• Stromal tumors (like Sertoli Leydig tumors and granulosa
cell tumors) occur in all ages.
• Granulosa- theca cell tumors are mostly seen in post-
menopausal women. -
Robbins/1050
•Sertoli-Leydig cell tumors occur in women of all ages,
although the peak incidence is in second and third
decade
(Ref: Robbins/1051)
Thus, we are left with two options: Germ cell tumors and Sertoli
Leydig cell tumors. But, if we see the frequency of tumors;
germ cell tumors have 15-20% whereas all sex-cord stromal
tumors (Sertoli Leydig is one of them) together constitute only
5-10% of ovarian neoplasms
Thus, a 14-year-old girl is most likely to have germ cell tumor.
Important Points about Ovarian Neoplasms

• These may develop from epithelial cells (like serous, mucinous,


Brenner tumor, etc), germ cells (like granulose cell tumor, Sertoli-
Leydig cell tumors, etc).
• Most common ovarian neoplasms are epithelial tumors
• Most common malignant ovarian neoplasms are epithelial tumors.
Two types of autosomal dominant familial cancers have been identified:
– Breast/ovarian cancer syndrome: Due to mutations in BRCA1 or
BRCA2 genes.
– Lynch type II syndrome: Results due to mutations in
mismatch repair genes. This is associated with Non-
polyposis colorectal cancer, endometrial and ovarian cancer.
• CA-125 is marker of epithelial ovarian cancers
• Dysgerminoma is ovarian counterpart of seminoma and is highly
sensitive to radiation therapy

24. Ans. (a) 10%


(Ref: Robbins 8th/1047, 9/e p1029)
• Germ cell tumors constitutes 15-20% of all ovarian tumors
• Most are benign cystic teratoma, also known as dermoid
cyst.
• Benign teratomas are bilateral in 0-15% of cases —
Robbins 1047
25. Ans. (a) < 1%
(Ref: Robbins 8th/1037, 9/e p1020)
• Fibroid is the term used for uterine leiomyoma. Malignant
transformation (leiomyosarcoma) within a leiomyoma is
extremely rare
• -- Robbins/1037
• Fibroids are most common tumor in women
• They have characteristic whorled pattern of smooth
muscle bundles on cut section as well as histologically
• Differences between leiomyoma and leiomyosarcoma is
mainly based on number of mitoses
• If >10 mitoses per high power field are present or > 5
mitoses with nuclear atypia are present, it indicates
malignancy
26. Ans. (a) Malignant change
(Ref: Shaw’s 13th/341, Robbins 9/e p1020)
Secondary changes (degenerations) in uterine leiomyoma:
• Atrophy: It can be:
After menopause
– After delivery
Tumour becomes firmer and more fibrotic. It is due to
diminished blood supply.
• Calcareous degeneration: Phosphates and carbonates
of lime are deposited in the periphery along the course
of the vessels.
– ‘Womb-stones’ in graveyard: In old patients with long-
standing myomas.
• Red (Carneous) degeneration: It is more common
during pregnancy. The myoma becomes tense and
tender and causes severe abdominal pain with
constitutional upset and fever. The tumour itself
assumes a peculiar purple-red color and develops a
fishy odour.
– Although the patient is febrile with moderate
leucocytosis and raised ESR, the condition is an
aseptic one.
– It needs to be differentiated from–
- Appendicitis
- Twisted ovarian cyst
- Pyelitis
- Accidental hemorrhage
• Sarcomatous change: It is extremely rare. Incidence is
no more than 0.5% of all myomas.
– Intramural and submucous tumors have a higher
potential than sub-serous.
– It is rare for malignant change to develop in a women
under the age of 40.
– It is highly malignant and spreads through blood.
• Other complications:
– Torsion
– Inversion
– Capsular hemorrhage
– Infection
– Associated endometrial carcinoma
27. Ans. (a) Uterus
(Ref: Robbins 9/e p1018, 8th/1035; 7th/1088)
28. Ans. (c) Yolk-Sac tumour
(Ref: Robbins 8th/1049, 9/e p1031)
29. Ans. (c) Granulosa cell tumour
(Ref: Robbins 8th/1050, 9/e p1032)
30. Ans. (c) Both
(Ref: Robbins 8th/1049, 9/e p1031)
31. Ans. (c) Villi present
(Ref: Robbins 7th/1113, 9/e p1041)
32. Ans. (a) Granulosa theca cell tumor
(Ref: Robbins 9/e p1032, 8th/1050; 7th/1102)
33. Ans. (b) Rhabdomyosarcoma
(Ref: Robbins 9/e p1001, 8th/1017)
34. Ans. (d) Mature teratoma
(Ref: Robbins 9/e p1029, 8th/1047-1048)
The lesion is a mature teratoma. Teratomas located in the ovary
and containing a hair and keratin filled cyst are sometimes
called dermoid cysts. They contain cells of a variety of types,
often including skin, skin adnexal structures (hair follicles,
sweat glands, sebaceous glands), connective tissues, neural
tissue, muscle, and thyroid tissue. If immature tissues such
as primitive neuroepithelial cells or developing skeletal
muscle cells are seen, the lesion is considered potentially
malignant and classified as an immature teratoma (option A).
35. Ans. (c) CA-125
(Ref: Robbins 9/e p1029, 8th/327)
• The tumors are serous papillary cystadenocarcinomas of
the ovaries. These tumors express CA-125 and are
apparently derived from the surface epithelium of the
ovaries.
• Bombesin (choice B) is a marker for neuroblastoma, small
cell carcinoma, gastric carcinoma, and pancreatic
carcinoma.
36. Ans. (b) Leiomyoma
(Ref: Robbins 8th/327, 9/e p1020)
37. Ans. (c) Schiller duval body
(Ref: Robbins 9/e p977)
38. Ans. (b) Brenner tumour
(Ref: Robbins 9/e p1028)
Transitional cell tumors contain neoplastic epithelial
cells resembling urothelium and are usually benign. They
comprise roughly 10% of ovarian epithelial tumors and are
also referred to as Brenner tumors
39. Ans. (b) Krukenberg tumor
(Ref: Robbins 9th/1034)

• A classic metastatic gastrointestinal carcinoma involving the ovaries


is termed Krukenberg tumor, characterized by bilateral metastases
composed of mucin-producing, signet-ring cancer cells, most often
of gastric origin.

40. Ans. (b) Adenocarcinoma


(Ref: Robbins 9/e p1013)
41. Ans. (c) Progresses to malignancy
(Ref: Robbins 9/e p970)
• Condyloma acuminatum is a benign sexually transmitted wart caused by
human papillomavirus.
• It is related to the common wart and may occur on any moist mucocutaneous
surface of the external genitals in either sex.
• Cytoplasmic vacuolization of the squamous cells called as koilocytosis is
characteristic of HPV infection.
• It tends to recur but only rarely progress into in situ or invasive cancers.

42. Ans. (b) Smoking


(Ref: Robbins 9/e p1014)
43. Ans. (d) Serous cystadenoma
(Ref: Robbins 9/e p1024)
44. Ans. (c) Intraductal papilloma
(Ref: Pathology and Genetics of tumours of breast and female
genital organs/81, Robbin 9/e p1044)
Successive branching of the large ducts eventually leads to the
terminal duct lobular unit (TDLU).
All breast cancers whether ductal or lobular originate from
terminal duct lobular unit (TDLU).
• Most of the benign breast diseases also originate from
TDLU except intraductal papilloma, 90% of which occurs
in large ducts in the central portion of breast.
45. Ans. (c) Papillary carcinoma breast
(Ref: Rossen Breast Pathology 390)
Dimorphic papillary carcinoma is a term that has been
used to refer to papillary carcinomas that have 2 types of
neoplastic cells, with a second population of cells showing
pale cytoplasm. So, morphologically the cells appear to have
diverse origins but expression of immunohistochemical
markers is useful for confirming the diagnosis.
46. Ans. (d) Comedo DCIS
(Ref: Robbins 9/e p1057)
• DCIS most frequently presents as mammographic
calcifications. Less typically DCIS may present as a
vaguely palpable mass. This is most likely with
Comedocarcinomas.
• Most of the breast malignancies are
adenocarcinomas. These can be divided into insitu
carcinomas and invasive carcinomas.
• Carcinoma in situ refers to a neoplastic population of
cells limited to ducts and lobules by the basement
membrane. It does not invade into lymphatics and blood
vessels and cannot metastasize.
• Invasive carcinoma (synonymous with “infiltrating”
carcinoma) has invaded beyond the basement
membrane into stroma. Here, the cells might also invade
into the vasculature and thereby reach regional lymph
nodes and distant sites. Even the smallest invasive
breast carcinomas have some capacity to metastasize.
All carcinomas are thought to arise from the terminal duct
lobular unit, and the terms “ductal” and “lobular” do not
imply a site or cell type of origin.
47. Ans. (c) Chromosome 17
(Ref: Harrison 17th/563, Robbin 9/e p1054)
• Tumor-suppressor gene, BRCA-1, has been identified at
the chromosomal locus 17q21; this gene encodes a zinc
finger protein, and the product therefore may function as
a transcription factor. The gene appears to be involved
in gene repair.
• BRCA-2, which has been localized to chromosome
13q12, is also associated with an increased incidence of
breast cancer in men and women.
48. Ans. (a) p53 (Ref:
Robbins 7th/286, 300, 302, 8th/290-291, 9/e p1054,
Harrison 16th/516, 517; 17th/563)
Breast cancer can be either familial (associated with germline
mutation) or sporadic (associated with somatic mutation).
The familial breast cancer is caused due to mutation in the
following 4 genes:
• p53 tumor suppressor gene (called Li Fraumeni syndrome
having multiple malignancies like breast cancer,
osteogenic sarcoma etc.)
• PTEN gene: Gene on chromosome 10q associated with
epithelial cancers of breast, endometrium and thyroid.
• BRCA1 gene: Located on 17q: females having mutant
gene have increased incidence of breast and ovarian
cancer whereas males having mutant gene have high
incidence of breast and prostate cancer.
• BRCA2 gene: Located on chromosome 13q is associated
with increased incidence of breast cancer in men and
women.
Sporadic breast cancer is associated with mutation in p53 gene
(p53 defect is present in 40% of human breast cancer as an
acquired defect) and PTEN gene.
Since, incidence of sporadic cancer is much more than familial
cancer, so the most important gene mutation increasing
susceptibility to breast cancer should be answered as p53.
49. Ans. (b) BRCA2
(Ref: Harrison 17th/604, 9/e p1054)
In women with hereditary breast/ovarian cancer, there can be two
susceptibility loci:

• BRCA1 located on chromosome 17q12-21, and


• BRCA2 located on 13q12-13.

Both these are tumor-suppressor genes which produce nuclear


proteins that interact with RAD 51 affecting genomic integrity.
The cumulative risk of ovarian cancer with critical mutations of
BRCA1 or - BRCA2 is 25%. Men in such families have an
increased risk of prostate cancer.
50. Ans. (c) Lobular Ca
(Ref: Robbins 9/e p1065)
• Lobular carcinoma (invasive) of breast is one of the very
few carcinomas which are seen bilaterally.
• Lobular carcinoma in situ in 20-40% cases is bilateral.
• Histologic hallmark is pattern of single infiltrating tumor
cells often only one cell in width or in loose clusters or
sheets.
• Signet ring cells common.
• Metastasize frequently to peritoneum, retroperitoneum,
leptomeninges, GIT and ovaries.
• The incidence of this carcinoma is increasing among
postmenopausal females presumably because
ofincreasing use of HRT.
51. Ans. (c) Inflammatory carcinoma
(Ref: Robbins 8th/1083)
52. Ans. (d) CA 15-3
(Ref: Robbins 7th/339, 9/e p337, Anderson pathology 144-152)
53. Ans. (b) Infiltration of the epidermis by malignant cells
(Ref: Robbins 7th/1140, 9/e p1057)
54. Ans. (a) 17
(Ref: Robbins 8th/275-276, 9/e p1054)
55. Ans. (d) Ductal carcinoma
(Ref: Robbins 9/e p1057)
56. Ans. (c) Invasive lobular carcinoma
(Ref: Robbins 8th/1082; 7th/1142, 9/e p1059)
57. Ans. (d) Lobular carcinoma
(Ref: Harsh Mohan 6th/762-763)
58. Ans. (b) Peau ‘d orange
(Ref: Robbins 9/e p1067, 8th/1083; 7th/122,1140,1142)
59. Ans. (c) Dermal lymphatic invasion by cancer cells
(Ref: Robbins 8th/1089, 9/e p1067)
• Inflammatory breast cancer is a pattern of invasive
breast cancer in which the neoplastic cells infiltrate
widely through the breast tissue. The cancer involves
dermal lymphatics and therefore has a high incidence of
systemic metastasis and a poor prognosis. If the
lymphatics become blocked, then the area of skin may
develop lymphedema and “peau d’orange,” or orange
peel appearance. The overlying skin in inflammatory
breast cancer is usually swollen, red, and tender.
• Acute inflammation (option A) is associated with
secondary infection or abscess whereas chronic
inflammation in breast cancer (option B) is a non-
specific finding. In medullary breast cancer, a type of
invasive ductal carcinoma, there are a large number of
lymphocytes around the tumor and a desmoplastic
reaction is often absent in the surrounding tissue.
• Epidermal invasion by cancer cells (option D) is a poor
prognostic indicator. Intraepidermal malignant cells are
called Paget cells. Paget’s disease of the nipple is a
type of ductal carcinoma that arises in large ducts and
spreads intraepidermally to the skin of the nipple and
areola. There is usually an underlying ductal carcinoma.
60. Ans. (a) 17
(Ref: Robbins 8/e p275-276, 9/e p1054)
61. Ans. (c) Neoplasia
(Ref: Robbins 9/e p1057)
Paget disease of the nipple is a rare manifestation of breast
cancer (1% to 4% of cases) that presents as a unilateral
erythematous eruption with a scale crust. Pruritus is common,
and the lesion may be mistaken for eczema.
62. Ans. (a) Gene expression profiling
(Ref: Robbins 9th/1061)
Invasive carcinomas can be divided based on molecular and
morphologic characteristics into several clinically important
subgroups. Breast carcinomas have a wide variety of
morphologic appearances. One third can be classified
morphologically into special histologic types, some of which
are strongly associated with clinically relevant biologic
characteristics. The remainder are grouped together and
called “ductal” or no special type (NST).

Recent detailed description of genomic alterations and gene and


protein expression in large cohorts of breast cancers has provided a
framework for a molecular classification for this group of breast
cancers…..Robbins 9th/ 1060

Gene expression profiling measures relative levels of mRNA


expression…figure 23.20 Robbins
63. Ans. (a) Overexpression
(Ref: Robbins 9/e p284)
HER-2/neu gene is a proto-oncogene whose
amplification/overexpression cause breast carcinoma.
64. Ans. (a) Medullary
(Ref: Robbins 9/e 1055)
BRCA1-associated breast cancers are commonly poorly
differentiated, have “medullary features” (a syncytial growth
pattern with pushing margins and a lymphocytic response).
65. Ans. (b) Epithelial membrane antigen
(Ref: Robbins 9/e 1067)
66. Ans. (d) Single file pattern
(Ref: Robbins 9/e 1065)
The histologic hallmark is the presence of discohesive
infiltrating tumor cells, often including signet-ring cells
containing intracytoplasmic mucin droplets. This is called as
single file pattern.

EXPLANATIONS TO CONCEPTUAL
QUESTIONS
Explanations(1-3): While solving assertion reason type of
questions, we can use a particular method.
1. First of all, read both assertion (A) and reason (R)
carefully and independently analyse whether they are true
or false.
2. If A is false, the answere will directly be (d) i.e. both A and
R are false. You can note that all other options (i.e. a, b or
c) consider A to be true.
3. If A is true, answer can be (a), (b) or (c), Now look at R. If
R is false, answer will be (c)
4. If both A and R are ture, then we have to know whether R
is correctly explaining A [answer is (a)] or it is not the
explanation of assertion [answer is (b)]
1. Ans. (c) Assertion is true and reason is false.
(Ref: Robbins 8th/995, 9/e p983)
Stromal cells are responsible for androgen dependent prostatic
growth. The main hormone responsible for androgen
dependent prostatic growth is dihydrotestosterone (and not
testosterone) because stromal cells have type 2 5α reductase
enzyme which converts testosterone into
dihydrotestosterone. DHT is more potent than testosterone
because it binds more strongly to the androgen receptor.

• FGF-7Q (fibroblast growth factor-7) is the most important factor


mediating paracrine regulation of androgen dependent growth.
• Type 1 Q 5α reductase enzyme is minimally present in prostate and
is mainly located mainly in liver and skinQ.

2. Ans. (c) Assertion is true and reason is false.


(Ref: Robbins 8th/1048, 9/e p1030)
Struma ovarii is mature teratoma which is composed of
functioning thyroid tissue. It is therefore responsible for
production of the thyroid hormones (and not
gonadotropins).
3. Ans. (b) Both assertion and reason are true and reason is
not the correct explanation of assertion.
(Ref: Robbins 8th/988-9, 9/e p976)
Seminoma is a germ cell tumor with a good prognosis because it is
an extremely radiosensitive and chemosensitive tumor.
These tumors histologically have necrosis and hemorrhage
only rarely.
Features Seminoma Non –seminoma
Localization Localized to testes for long Metastasize early
time
Stage 70% patients present in 60% in stage II and III
stage I
Metastasis Mainly lymph node: Hematogenous spread
hematogenous spread later more frequent
Necrosis and Rare Common
hemorrhage
Radiation sensitivity Radiosensitive Radioresistant
General behavior Less aggressive More aggressive
Prognosis Good Bad
1. A middle-aged female presented with a 4 cm mass in upper
outer quadrant of the breast. Biopsy showed densely
packed cells within large lakes of mucin. Which of the
following is the most likely diagnosis of this patient?
(NEET 2020 like pattern)
(a) Medullary carcinoma of breast
(b) Colloid carcinoma of breast
(c) Tubular carcinoma of breast
(d) Papillary carcinoma of breast
Ans. (b) Colloid carcinoma of breast
(Ref: Robbins 9th/1065)
• Middle-aged female has a 4 cm mass in upper outer
quadrant of the breast which is the most common site
for having a breast cancer.
• As the biopsy mentions the presence of densely packed
cells within large lakes of mucin (see image), this is
suggestive of mucinous carcinoma of the breast which is
also called as colloid carcinoma of the breast.
2. Chromosome number of partial hydatidiform mole is:
(NEET 2019 like pattern)
(a) 46XX
(b) 46XY
(c) 69XXX
(d) Any of the above
Ans. (c) 69XXX
(Ref: Robbins 9th e/p 1039-40)
Partial moles result from fertilization of an egg with two sperm. In
these moles the karyotype is triploid (e.g., 69,XXY) or
occasionally tetraploid (92,XXXY).
3. A female has undergone hysterectomy with the following
gross and histological features. Which of the following is
the most likely diagnosis?
(AI 2018 Pattern)
(a) Leiomyosarcoma
(b) Leiomyoma
(c) Mixed Mullerian tumor
(d) Vaginal carcinoma
Ans. (b)Leiomyoma
(Ref: Robbins 9/e p1019)
The uterus is reveals multiple tumors in submucosal (bulging into
the endometrial cavity), intramural, and subserosal locations
that display a firm white appearance on sectioning.
Histologically, leiomyomas show well-differentiated, regular,
spindle-shaped smooth muscle cells associated with
hyalinization.
4. Extramammary Paget’s disease is seen in:
(AI 2018 Pattern)
(a) Ovary
(b) Uterus
(c) Vagina
(d) Vulva
Ans. (d) Vulva
(Ref: Robbins 9/e p999)

• Extra mammary Paget’s disease is seen in the skin of the vulva is


similar in its manifestations to Paget disease of the breast.
• In the vulva, it presents as a pruritic, red, crusted, map like area,
usually on the labia majora.
• In contrast to Paget disease of the nipple, in which 100% of patients
have an underlying ductal breast carcinoma, vulvar Paget is
typically not associated with underlying cancer.
• Histologically, Paget cells are larger than surrounding keratinocytes.
The cells have pale cytoplasm containing mucopolysaccharide that
stains with periodic acid–Schiff (PAS), Alcian blue, or mucicarmine
stains. In addition, the cells express cytokeratin.
• The treatment consists of wide local excision.

5. A middle-aged adult undergoes orchiectomy. Looking at


the gross and the histological findings, which of the
following is the likely diagnosis?
(AI 2018 Pattern)
(a) Lymphoma
(b) Seminoma
(c) Yolk sac tumor
(d) Teratoma
Ans. (b) Seminoma
(Ref: Robbins 9/e p976)
• Seminoma is the commonest germ cell tumor.
• It produces a bulky mass with a homogenous grey white
appearance without hemorrhage and necrosis.
• Microscopically, is characterised by round to polyhedral
cells with distinct cell membrane, central nucleus and
clear cytoplasm.
6. In a known case of breast cancer, Fluorescent In situ
hybridization (FISH) for gene amplification will be done
based on which of the following immunohistochemistry
(IHC) staining for HER2/neu?
(AIIMS Nov 2017 Pattern)
(a) 1
(b) 2
(c) 3
(d) Any of the above
Ans. (b) 2
Try to understand the following: when we do
immunohistochemistry, it is given a score as follows:

• Positive:
– IHC 3+ (strong positive): tumor displays complete, intense
circumferential membranous staining
• Equivocal:
– IHC 2+: incomplete circumferential membrane staining or
weak/moderate and within > 10% of invasive tumor cells; or
complete and circumferential membrane staining that is intense
and within ≤ 10% of invasive tumor cells
• Negative:
– IHC 1+: incomplete faint membrane staining and within > 10% of
invasive tumor cells
– IHC 0: no staining observed or incomplete faint membrane staining
within ≤ 10% of invasive tumor cells

Disclaimer
Any resemblance to an actual question is purely coincidental.
• Functions of different cells in the brain include Gitter cells or microglia
(phagocytic cells), neuroglia or astrocytes (tissue repair in brain) and
oligodendrocytes (myelination).
• Maximum decrease in CSF chloride: TB meningitis.
• Vasculitis is typically absent in HIV encephalitis.
• Rabies encephalitis is characterized by Brainstem encephalitis and
histologicaly by intra-neuronal Negri bodies.
• Key words with Alzheimer disease: Meyernet nucleus involvement,
cortical atrophy, neuritic (senile) plaque, neurofibrillary tangles,
amyloid angiopathy and Hirano bodies. Grossly, there is atrophy of
parietal and temporal lobe.
• Hydrocephalus ex vacuo is seen in: Alzheimer disease, Pick disease.
• Most common cause of intracerebral (intraparenchymal) hemorrhage:
Hypertension.
• Most common cause of subarachnoid hemorrhage: Trauma (most
common) followed by Berry aneurysm
• Most common cause of subdural hematoma: Traumatic rupture of
bridging veins
• Most common cause of epidural hematoma: Injury to middle
meningeal artery.
• Dandy Walker syndrome can be differentiated from Aqueductal
stenosis by: Posterior fossa volume.
• Ash leaf macules are characteristic of: Boumeville’s disease (tuberous
sclerosis).
• Tectal breaking is seen in: Arnold chiary malformation.
• Most common nerve from which schwannomas arise is 8th cranial
nerve whereas commonest nerve from which schwannomas arise in
neck is 10th cranial nerve (vagus).
• Brain tumor arising from arachnoid villi: Meningioma.
• Medulloblastoma (arising mostly from cerebellum) commonly
metastatizes to other parts of brain and spine (via CSF).
• Spongiform degeneration of cerebral cortex is characteristic:
Creutzfeldt Jakob disease.
• Intranuclear inculsions of oligodendrocytes are seen in: Progressive
multifocal leukoencephalopathy.
• Onion bulb appearance on nerve biopsy is seen in: Chronic
inflammatory demyelinating polyneuropathy (CIDP).

CEREBRAL HERNIATION

The cranial cavity is separated into compartments by infoldings of


the dura. The two cerebral hemispheres are separated by the falx,
and the anterior and posterior fossae by the tentorium. Herniation
refers to displacement of brain tissue into a compartment that it
normally does not occupy. These are of three main types:
transtentorial, transfalcine (subfalcine) and tonsillar (foraminal).

Transtentorial Herniation

The most common herniations are from the supratentorial to the


infratentorial compartments through the tentorial opening, hence
transtentorial. These may be divided into temporal (Uncal) or
central herniations.
• Uncal transtentorial herniation refers to impaction of the anterior
medial temporal gyrus (the uncus) into the tentorial opening just
anterior to and adjacent to the midbrain. The displaced brain tissue
compresses the third nerve and results in mydriasis and
ophthalmoplegia (pupil point down and out) of the ipsilateral
pupil.
• Central transtentorial herniation denotes a symmetric downward
movement of the thalamic medial structures through the tentorial
opening with compression of the upper midbrain. Miotic pupils and
drowsiness are the heralding signs.

Transfalcine Herniations

These are caused by herniation of the medial aspect of the


cerebral hemisphere (cingulate gyrus) under the falx, which may
compress the anterior cerebral artery.

Tonsillar Herniation

Masses in the cerebellum may cause tonsillar herniation, in which


the cerebellar tonsils are herniated into the foramen magnum. This
may compress the medulla and respiratory centers, causing death.

DEVELOPMENTAL ABNORMALITIES OF THE BRAIN

Developmental abnormalities of the brain include the Arnold-


Chiari malformation, the Dandy-Walker malformation, and the
Phakomatoses, which include tuberous sclerosis,
neurofibromatosis, von Hippel-Lindau disease, and Sturge-Weber
syndrome.

• Dandy-Walker malformation has severe hypoplasia or


absence of the cerebellar vermis. There is cystic distention of
the roof of the fourth ventricle, hydrocephalus, and possibly
agenesis of the corpus callosum.

• Arnold-Chiari malformation consists of herniation of the


cerebellum and fourth ventricle into the foramen magnum, flattening
of the base of the skull, and spina bifida with meningomyelocele.
Newborns with this disorder are at risk of developing hydrocephalus
within the first few days of delivery secondary to stenosis of the
cerebral aqueduct.

• Tuberous sclerosis may show characteristic firm, white


nodules (tubers) in the cortex and subependymal nodules of
gliosis protruding into the ventricles (“candle drippings”)
Facial angiofibromas (adenoma sebaceum) may also occur.
• von Hippel-Lindau disease shows multiple benign and
malignant neoplasms including hemangioblastomas of the
retina, cerebellum, and medulla oblongata; angiomas of the
kidney and liver; and renal cell carcinomas.
• Sturge-Weber syndrome is a non-familial congenital disorder,
display angiomas of the brain, leptomeninges, and ipsilateral
face, which are called port-wine stains (nevus flammeus).
• Syringomyelia: Bilateral loss of pain and temperature
sensations in both arms is most likely to be caused by
syringomyelia, which is a chronic myelopathy that results from
formation of a cavity (syrinx) involving the central gray matter
of the spinal cord. This is the location where pain fibers cross
to join the contralateral spinothalamic tract. Interruption of the
lateral spinothalamic tracts results in segmental sensory
dissociation with loss of pain and temperature sense, but
preservation of the sense of touch and pressure or vibration,
usually over the neck, shoulders, and arms. Other features of
syringomyelia include wasting of the small intrinsic hand
muscles (claw hand) and thoracic scoliosis. The cause of
syringomyelia is unknown, although one type is associated
with a Chiari malformation with obstruction at the foramen
magnum.

NEURAL TUBE DEFECTS

These are caused by defective closure of the neural tube. These


defects may occur anywhere along the extent of the neural tube
and are classified as either caudal or cranial defects. Failure of
development of the cranial end of the neural tube results in
anencephaly, while failure of development of the caudal end of the
neural tube results in spina bifida.
• Anencephaly, which is not compatible with life, is characterized
by the absence of the forebrain. Instead, there is a mass of
disorganized glial tissue with vessels in this area called a
cerebrovasculosa. Ultrasound examination will reveal an
abnormal shape to the head of the fetus with an absence of
the skull.
• Spina bifida can be spina bifida occulta which results due to failure of
closure of vertebral arches posteriorly. It is a mild disorder with
normal meninges and spinal cord. If meninges also herniate out, it is
known as meningocele whereas protruding out of both meninges as
well as spinal cord is called meningomyelocele.

• Neural tube defects are associated with maternal obesity and


decreased folate during pregnancy (folate supplementation in
diet decreases the incidence of these development defects).

CEREBRAL HEMORRHAGE

It can be epidural, subarachnoid, subdural and intraparenchymal.


• Epidural hemorrhage: It results from hemorrhages into the
potential space between the dura and the bone of the skull.
These hemorrhages result from severe trauma that typically
causes a skull fracture. The hemorrhage results from rupture of
one of the meningeal arteries, as these arteries supply the
dura and run between the dura and the skull. Since the
bleeding is of arterial origin (high pressure), it is rapid and the
symptoms are rapid in onset, although the patient may be
normal for several hours (lucid interval). Bleeding causes
increased intracranial pressure and can lead to tentorial
herniation and death.
• Subarachnoid hemorrhage: It is much less common than
hypertensive intracerebral hemorrhage. It most often results from the
rupture of a berry aneurysm. These aneurysms are Saccular
aneurysms that result from congenital defects in the media of
arteries. They are typically located at the bifurcations of arteries.
They are not the result of atherosclerosis. Instead, berry aneurysms
are called congenital, although the aneurysm itself is not present at
birth. The chance of rupture of berry aneurysms increases with age
(rupture is rare in childhood). Rupture causes marked bleeding into
the subarachnoid space and produces severe headaches, typically
described as the “worst headache ever”. Additional symptoms
include vomiting, pain and stiffness of the neck (due to meningeal
irritation caused by the blood), and papilledema. Death may follow
rapidly.

• Subdural hemorrhage: The space beneath the inner surface


of the dura mater and the outer arachnoid layer of the
leptomeninges is also a potential space.

CEREBRAL ISCHEMIA

Decreased brain perfusion may be generalized (global) or


localized.
Global ischemia results from generalized decreased blood
flow, such as with shock, cardiac arrest, or hypoxic episodes (e.g.
near drowning or carbon monoxide poisoning).
Fig. 1: Neuronal injury showing chromatolysis (C) and injured eosinophilic
neurons (N).

• The gross changes produced by global hypoxia include watershed


(border zone) infarcts, which typically occur at the border of areas
supplied by the anterior and middle cerebral arteries, and laminar
necrosis, which is related to the short, penetrating vessels originating
from pial arteries.
Fig. 2: Red neurons on global ischemia.
• The microscopic changes produced by global hypoxia are
grouped into three categories. The earliest histologic changes,
occurring in the first 24 h, include the formation of red neurons
(acute neuronal injury), characterized by eosinophilia of the
cytoplasm of the neurons, and followed in time by pyknosis
and karyorrhexis. Subacute changes occur at 24 h to 2 weeks.
These include tissue necrosis, vascular proliferation, and
reactive gliosis.

INTRACRANIAL ANEURYSMS

• Charcot-Bouchard aneurysms: It results from weakening of the wall of


cerebral artery by lipohyalinosis (deposition of lipids and hyaline material)
caused by hypertension. Hypertensive hemorrhage shows a predilection
for the distribution of the lenticulostriate arteries (branch of middle cerebral
artery) with small (lacunar) hemorrhages, or large hemorrhages
obliterating the corpus striatum, including the putamen and internal
capsule. Hypertensive hemorrhages also commonly occur in cerebellum
and pons and are often fatal.
• Berry aneurysms (small saccular aneurysms) are the result of congenital
defects in the media of blood vessels and are located at the bifurcations
of arteries.
• Atherosclerotic aneurysms are fusiform (spindle-shaped) aneurysms
usually located in the major cerebral vessels. They rarely rupture, but may
become thrombosed.
• Mycotic (septic) aneurysms result from septic emboli, most commonly
from subacute bacterial endocarditis.

CNS INFECTIONS

• Meningoencephalitis caused by HIV (human immunodeficiency virus)


is characterized by microglial nodules within the brain that are
composed of mononuclear cells, microglia, and scattered
multinucleated giant cells.
• Herpes simplex virus produces characteristic Cowdry type A
intranuclear inclusions in neurons and glial cells.
• Rabies form characteristic inclusions within neurons called Negri
bodies. Rabies, caused by a single-stranded RNA rhabdovirus, is
transmitted by the bite of a rabid animal, usually a dog, and travels to
the brain via peripheral nerves. Symptoms caused by destruction of
neurons in the brainstem include irritability, difficulty in swallowing
and spasms of the throat (these two resulting in “hydrophobia”),
seizures, and delirium. The illness is almost uniformly fatal.
• Enlarged cells (cytomegaly) with intranuclear and intracytoplasmic
inclusions are seen with cytomegalovirus infection.
Fig. 3: Negri body in rabies.

Progressive multifocal leukoencephalopathy (PML): It is a


demyelinating disease of the central nervous system that results
from infection of oligodendrocytes by the JC polyomavirus. Signs
and symptoms of PML are varied but include dementia and ataxia
along with abnormal vision and speech. PML occurs as a terminal
complication in immunosuppressed individuals, especially
individuals with AIDS.

Neurosyphilis: Neurosyphilis, a tertiary stage of syphilis,


includes syphilitic meningitis, paretic neurosyphilis, and tabes
dorsalis.
Syphilitic meningitis is characterized by perivascular infiltrates
of lymphocytes and plasma cells that cause obliterative endarteritis
and meningeal fibrosis.
• Tabes dorsalis is the result of degeneration of the posterior
columns of the spinal cord. This is caused by compression
atrophy of the posterior spinal sensory nerves, which produces
impaired joint position sensation, ataxia, loss of pain sensation
(leading to joint damage, i.e. Charcot joints), and Argyll
Robertson pupils (pupils that react to accommodation but not
to light).
Prion diseases: The spongiform encephalopathies include Creutzfeldt-
Jakob disease (CJD), Gerstmann-Straussler-Scheinker syndrome (GSS),
fatal familial insomnia, and kuru. Microscopically, there is characteristic
spongiform change in the gray matter (“cluster of grapes” vacuolation)
without inflammation.

All of the spongiform encephalopathies are associated with


abnormal forms of a prion protein (PrP). Disease results from
alternate folding of the normal a-helix (called PrPc) to an abnormal
b-pleated sheet form (called PrPsc). This conformational change
can occur spontaneously at a very slow rate. Once formed,
however, PrPsc can combine with PrPc to much more quickly form
many more PrPsc particles, which can “crystallize” and form
plaques. PrPc can also form PrPsc at much higher rates if
mutations are present in PrPc, which can result from mutations in
the gene that codes for PrPc called PRNP. Mutations in this gene
have been identified in patients with the familial forms of CJD,
GSS, and fatal familial insomnia.
Note: SOD1 mutations are seen with amyotrophic lateral
sclerosis (ALS), FGFR3 mutations with achondroplasia, UBE3A
mutations with Angelman’s syndrome, and PTEN mutations with
endometrial and prostate cancers.
CSF Findings in CNS Infections
Parameters Normal Bacterial Tuberculous Viral
values meningitis meningitis meningitis
Pressure 50-180 mm Raised Raised Raised
water

Gross Clear and Turbid Clear (may Clear


appearance colorless clot)

Protein 20 – 50 High Very High Slightly high


mg/dL

Glucose 40-70 mg/dL Very low Low Normal

Chloride 116 – 122 Low Very low Normal


µg/dL

Cells < 5/microlitre Neutrophils Pleocytosis Lymphocytosis

Meningitis [inflammation of the arachnoid and the


cerebrospinal fluid (CSF)] may be classified as acute pyogenic,
aseptic, or chronic. The etiology and CSF findings vary in these
three groups.

• The CSF in acute pyogenic meningitis, which is usually caused


by bacteria, is grossly cloudy (not bloody, which is suggestive
of a subarachnoid hemorrhage) and displays increased
pressure, increased neurophils, increased protein, and
decreased glucose.
• With chronic meningitis, such as that caused by Mycobacterium
tuberculosis, the CSF is clear grossly, with only a slight
increase in leukocytes (either mononuclear cells or a mixed
infiltrate), a markedly increased protein level, increased
pressure, and moderately decreased or normal amounts of
sugar.
• Brain abscesses and subdural empyemas, which are
parameningeal infections rather than direct meningeal
infections, cause increased CSF pressure (more marked with
abscess because of mass effect) along with increased
inflammatory cells (lymphocytes and polymorphonuclear cells)
and increased protein but a normal glucose level. The CSF is
clear.
• Encephalitis, also not a direct infection of the meninges, results
in clear CSF, increased pressure, increased protein, normal
glucose, and possibly increased lymphocytes.

DEMYELINATING DISORDERS

Multiple Sclerosis

In primary CNS demyelination there is loss of myelin sheaths with


relative preservation of axons. Primary demyelination is seen
predominately in multiple sclerosis, in the perivenous
encephalomyelopathies, and in progressive multifocal
leukoencephalopathy (PML). Multiple sclerosis (MS), a disease of
unknown etiology, causes disseminated but focal plaques of
primary demyelination anywhere in the CNS, but often in the white
matter near the angles of the lateral ventricles. It primarily affects
young adults between 20 and 40 years of age, with the onset of
symptoms such as abnormalities of vision, tremors, paresthesias,
and incoordination. The course is typically remitting and relapsing.
Early findings include weakness of the lower extremities and visual
abnormalities with retrobulbar pain.

Apart from the Charcot triad, another pathognomonic feature in MS is


internuclear ophthalmoplegia (INO), also known as the MLF
syndrome, which results from demyelination of the medial longitudinal
fasciculus. It results in medial rectus palsy in attempted lateral gaze and
monocular nystagmus in abducting eye with convergence.

Examination of the CSF in patients with MS reveals increased


T lymphocytes, increased protein, and normal glucose. Protein
electrophoresis of the CSF reveals oligoclonal bands (individual
monoclonal spikes), although this latter finding is not specific for
MS.

Neuromyelitis Optica (also known as Devic disease)

It is characterized by the development of synchronous bilateral


optic neuritis and spinal cord demyelination.

DEGENERATIVE DISORDERS

The degenerative diseases of the CNS are diseases that affect the
gray matter and are characterized by the progressive loss of
neurons in specific areas of the brain.

Alzheimer’s Disease

Alzheimer’s disease (AD) is the most common cause of dementia


in elderly (followed by vascular multi-infact dementia and diffuse
Lewy body disease). AD often begins insidiously with impairment
of memory and progresses to dementia. Histologically, AD is
characterized by numerous neurofibrillary tangles and senile
plaques with a central core of amyloid alpha-protein. Both tangles
and plaques are found to a lesser extent in other conditions, for
example, neurofibrillary tangles in Down syndrome. Silver stains
demonstrate tangles and plaques and Congo red shows amyloid
deposition in plaques and vascular walls (amyloid angiopathy). In
AD there are also numerous Hirano bodies, and granulovacuolar
degeneration is found in more than 10% of the neurons of the
hippocampus. Grossly, brain atrophy (narrowed gyri and widened
sulci) is predominant in the frontal and superior temporal lobes.

Fig. 4: Alzheimer disease, senile plaques.


Fig. 5: Alzheimer disease, Tau protein.

The etiology of AD is not well understood, but it is clear that


there are multiple etiologic pathways to this disease state.
Alzheimer’s disease has been linked to abnormalities involving four
specific genes. Cleavage of the beta-amyloid precursor protein
(beta-APP) by alpha-secretase precludes beta-A formation; but
cleavage of beta-APP by beta-secretase (BACE-1) or gamma-
secretase produces fragments that tend to aggregate into the
pathogenic amyloid fibrils. Beta-amyloid deposition is necessary
but not sufficient for the development of Alzheimer’s disease.
Early-onset familial Alzheimer’s is also related to mutations in
presenilins. The presenilin 1 (PS1) gene is located on
chromosome 14, while the presenilin 2 (PS2) gene is located on
chromosome 1.
Parkinson’s Disease

It is characterized by a mask-like facial expression, coarse tremors,


slowness of voluntary movements, and muscular rigidity, there is
degeneration and loss of pigmented cells in the substantia nigra,
resulting in a decrease in dopamine synthesis. The decreased
synthesis of dopamine by neurons originating in the substantia
nigra leads to decreased amounts and functioning of dopamine in
the striatum. This results in decreased dopamine inhibition and a
relative increase in acetylcholine function, which is excitatory in the
straitum. The effect of this excitation, however, is to increase the
functioning of GABA neurons, which are inhibitory. The result,
therefore, is increased inhibition or decreased movement. The
severity of the motor syndrome correlates with the degree of
dopamine deficiency.

Lewy Body Disorders

Lewy bodies are intracytoplasmic eosinophilic inclusions that are


composed of fine filaments, which are densely packed in the core
but loose at their rim. These filaments are composed of
neurofilament antigens, parkin, and ubiquitin, but the major
component of the Lewy body is alpha-synuclein. The histologic
presence of Lewy bodies can be seen in several disorders (Lewy
body disorders) that differ in the location where the Lewy bodies
are found.

• In classic Parkinson’s disease, Lewy bodies are found in the


nigrostriatal system (producing extrapyramidal movement
disorder).
• In Lewy body dementia, Lewy bodies are found in the cerebral
cortex (producing dementia; this is the third most common cause of
dementia).
• In Shy-Dragger syndrome, Lewy bodies are found in sympathetic
neurons in the spinal cord (causing autonomic dysfunction,
including orthostatic hypotension, impotence, abnormal sweat and
salivary gland secretion, and pupillary abnormalities).

Huntington’s Disease: (HD)

It is characterized by choreiform movements and progressive


dementia that appear after the age of 30. It is an autosomal
dominant disorder that results from an abnormal gene (showing
CAG repeats) on chromosome-4. Choreiform movements and
progressive dementia appear after the age of 30. There is
degeneration of GABA neurons in the striatum, which leads to
decreased function (decreased inhibition) and increased
movement.
Note: Huntington’s disease is one of four diseases that are characterized by
long repeating sequences of three nucleotides (the other diseases being fragile
X syndrome, myotonic dystrophy, and spinal and bulbar muscular atrophy).

Amyotrophic Lateral Sclerosis (ALS)

Amyotrophic lateral sclerosis (ALS), also known as Lou Gehrig’s


disease, is a degenerative disorder of motor neurons, principally
the anterior horn cells of the spinal cord, the motor nuclei of the
brainstem, and the upper motor neurons of the cerebral cortex.
Clinically, this disease is a combination of lower motor neuron
(LMN) disease with weakness and fasciculations and upper motor
neuron (UMN) disease with spasticity and hyperreflexia. Early
symptoms include weakness and cramping and then muscle
atrophy and fasciculations. Reflexes are hyperactive in upper and
lower extremities, and a positive extensor plantar (Babinski) reflex
develops because of the loss of upper motor neurons. The clinical
course is rapid, and death may result from respiratory
complications.

TUMORS

CNS tumors can be gliomas, neuronal tumors, poorly differentiated


neoplasms, meningiomas and metastatic tumors.

1. Gliomas: These are most common group of primary brain


tumors. These include astrocytoma, oligodendroglioma and
ependymoma.
a. Astrocytoma
It is the most common primary brain tumors in adults
and range from low grade to very high grade (glioblastoma
multiforme). These grades of astrocytomas include:

Grade I: The least aggressive and histologically difficult to differentiate


from reactive astrocytosis.
Grade II: Some plemorphism microscopically.
Grade III: Anaplastic astrocytoma, characterized histologically by
increased pleomorphism and prominent mitoses.
Grade IV: Glioblastoma multiforme. A highly malignant tumor
characterized histologically by endothelial proliferation and serpentine
areas of necrosis surrounded by peripheral palisading of tumor cells. It
frequently crosses the midline (“butterfly tumor”).

Progression of low grade astrocytoma to a higher grade


(secondary glioblastoma multiforme) is associated with
several genetic abnormalities, such as disruption of the
p16/CDKNZA gene or overexpression of PDGF-A and its
receptor.
b. Oligodendroglioma
These most commonly involve the cerebrum
(hemispheres) in adults, are slow-growing tumors that
have a high recurrence rate. Some oligodendrogliomas do
proliferate in a rapid and aggressive fashion and may be
associated with a malignant astrocytoma component.
Histologically, oligodendrogliomas consist of sheets of
cells with clear halos (“fried-egg” appearance) and
various amounts of calcification (which can be seen on -
ray). Cytogenetic abnormalities have therapeutic
significance for this type of tumor, as only tumors with
deletion involving 19q or 1p respond to chemotherapy.
Fig. 6: Fried Egg appearance in oligodendroglioma.

c. Ependymoma
These most often arise next to the ependyma-lined
ventricular system, including central canal of spinal cord.
Spinal cord ependymoma frequently occur in setting of
neurofibromatosis type 2.

2. Neuronal tumors
The most common CNS tumor containing mature appearing
neurons (ganglion cells) is ganglioglioma. These are most
commonly found in temporal lobe and frequently contain
ganglion cells with binucleated forms.
3. Poorly differentiated neoplasms
Most common among these is medulloblastoma. Others
include atypical tetroid/rhabdoid tumor.
a. Medulloblastoma
Primitive neuroectodermal tumors (PNETs) are a type of
malignant embryonal tumor that can be found at sites
within or outside of the central nervous system. An
example of a PNET located outside of the CNS is Ewing’s
sarcoma of bone. PNETs of the CNS can be divided into
supratentorial tumors (sPNET) and infratentorial tumors
(iPNET). The latter are also called as medulloblastoma
and they usually arise in the midline of the cerebellum (the
vermis) but in adults, where the incidence is much less
than in children, they are more apt to arise in the
cerebellar hemispheres in a lateral position. In about one-
third of cases, these show rosette formation centered by
neurofibrillary material. Medulloblastomas grow by local
invasive growth and may block cerebrospinal fluid
circulation (CSF block) via compression of the fourth
ventricle.

4. Meningioma
A tumor that is attached to the dura is most likely to be a
meningioma. This type of tumor arises from the arachnoid villi
of the brain or spinal cord. Although they usually occur during
middle or later life, a small number occur in persons 20 to 40
years of age. They commonly arise along the venous sinuses
(parasagittal, sphenoid wings, and olfactory groove).

Note: Both oligodendroglioma and craniopharyngioma show calcification


fairly frequently, oligodendroglioma is often located in the frontal lobe,
whereas craniopharyngioma occurs around the third ventricle and
demonstrates suprasellar calcification.
Fig. 7: Meningioma contain calcified bodies called psammoma bodies.

5. Metastatic tumors
These are most common intra—cranial tumors. Five most
common sites are
Lung
Breast
Skin (melanoma)
Kidney
GIT

CNS Tumors and Age


The location of a tumor and the age of the patient are both very
important in the differential diagnosis of tumors of the central
nervous system. Astrocytomas occur predominantly in the cerebral
hemispheres in adult life and old age, in the cerebellum and pons
in childhood, and in the spinal cord in young adult. The pilocytic
astrocytoma is a subtype that is the most common brain tumor in
children, and therefore it is also called a juvenile pilocytic
astrocytoma. It is characterized by its location in the cerebellum
and better prognosis. Meningiomas, found within the meninges,
have their peak incidence in the fourth and fifth decades. The
highly malignant glioblastoma multiforme is also found primarily in
adults. Oligodendrogliomas also involve the cerebrum in adults.
Ependymomas are found most frequently in the fourth ventricle,
while the choroid plexus papilloma, a variant of the ependymoma,
is found most commonly in the lateral ventricles of young boys.
The medulloblastoma is a tumor that arises exclusively in the
cerebellum and has its highest incidence toward the end of the first
decade. In children medulloblastomas are located in the midline,
while in adults they are found in more lateral locations.

PERIPHERAL NERVE SHEATH TUMOR

Fig. 8: Schwannoma Antoni A and Antoni B areas.

Schwannomas
These are benign tumors that generally appear as extremely
cellular spindle cell neoplasms, sometimes with metaplastic
elements of bone, cartilage, and skeletal muscle. Schwannomas
(neurilemomas) are single, encapsulated tumors of nerve sheaths,
usually benign, occurring on peripheral, spinal, or cranial nerves.
Acoustic neuromas typically located at the cerebellopontine angle
or in the internal acoustic meatus. Initially, when they are small,
these tumors produce symptoms by compressing CN VIII and CN
VII (facial). CN VIII symptoms include unilateral tinnitus (ringing in
the ear), unilateral hearing loss, and vertigo (dizziness).
Involvement of the facial nerve produces facial weakness and loss
of corneal reflex. Histologically, an acoustic neuroma consists of
cellular areas (Antoni A) and loose edematous areas (Antoni B).
Verocay bodies (foci of palisaded nuclei) may be found in the
more cellular areas.

Fig. 9: Schwannoma with Verrocay Body (V).

Familial Tumor Syndromes

a. Tuberous sclerosis
Tuberous sclerosis is an autosomal dominant syndrome
characterized by the clinical triad of angiofibromas
(“adenoma sebaceum”), seizures, and mental retardation.
Patients develop hamartomas in the central nervous system
including “tubers”, which are film areas with haphazardly
arranged neurons and glia with stout processes. The
syndrome is associated with the development of several
different types of tumors, including subepedymal giant cell
tumor, rhabdomyoma of the heart, and angiomyolipoma of the
kidney. Mutations at several loci have been associated with
tuberous sclerosis including the TSC1 locus, which codes for
hamartin, and the TSC2 locus, which codes for tuberin. These
two proteins inhibit mTOR, which is the mammalian target of
rapamycin. mTOR plays a central role in the regulation of cell
growth.

Note: Dysregulation of mTOR activity is associated with several


hamartoma syndromes, including the tuberous sclerosis, von Hippel-
Lindau syndrome, Peutz-Jegher’s syndrome, and the PTEN-related
hamartoma syndromes.

b. von Hippel-Lindau disease


In this rare autosomal dominant disorder, multiple benign, and
malignant neoplasms occur. These include
hemangioblastomas of retina and brain (cerebellum and
medulla oblongata), angiomas of kidney and liver, and renal
cell carcinomas (multiple and bilateral) in 25 to 50% of cases.
c. Neurofibromatosis type 1
Classic neurofibromatosis (NF-1) is an autosomal dominant
disorder. It is characterized by cafe-au-lait skin macules,
axillary freckling, multiple neurofibromas, plexiform
neurofibromas, and Lisch nodules (pigmented iris
hamartomas). Lisch nodules are found in 95% of patients after
age 6. There is increased risk of developing meningiomas or
even pheochromocytoma. A major complication of NF-1 is the
malignant transformation of a neurofibroma to a
neurofibrosarcoma. The gene for the classic form (NF-1) is
lcoated on chromosome 17. It encodes for neurofibromin, a
protein that regulates the function of p21 oncoprotein.

d. Neurofibromatosis type 2
Central neurofibromatosis (NF-2) is an autosomal-dominant
disorder in which patients develop a range of tumors, most
commonly bilateral VIII nerve schwannomas and multiple
meningiomas. Gliomas, typically ependymomas of the spinal
cord, also occur in these patients. Many individuals with NF2
also have non-neoplastic lesions, which include nodular
ingrowth of Schwann cells into the spinal cord (schwannosis),
meningioangiomatosis (a proliferation of meningeal cells and
blood vessels that grows into the brain), and glial hamartia
(microscopic nodular collections of glial cells at abnormal
locations, often in the superficial and deep layers of cerebral
cortex). The NF2 gene is located on chromosome 22 and
encodes for merlin.
DEVELOPMENTAL DEFECTS, CEREBRAL HEMORRHAGE,
ANEURYSM

1. Cervical syringomyelia all are seen except:


(a) Burning sensation in hands
(AIIMS Nov 2012)
(b) Hypertrophy of abductor pollicis brevis
(c) Extensor plantar response is present
(d) Absent biceps reflex.
2. The best described etiology for Berry aneurysm is which of
the following?
(AIIMS May 2011)
(a) Degeneration of internal elastic lamina
(b) Degeneration of tunica media
(c) Defect in muscular layer
(d) Low grade inflammation in the vessel wall

MOST RECENT QUESTIONS

3. The defect in Berry aneurysm is:


(a) Degeneration of internal elastic lamina
(b) Degeneration of media
(c) Deposition of mucoid material in media
(d) Low grade inflammation of vessel wall
4. Which of the following would distinguish hydro-cephalus
due to aqueductal stenosis when compared to that due
to Dandy walker malformation?
(a) Third ventricle size
(b) Posterior fossa volume
(c) Lateral ventricular size
(d) Head circumference
5. Most common site for berry aneurysm is:
(a) Basilar artery
(b) Anterior communicating artery
(c) Posterior communicating artery
(d) Posterior cerebral artery
6. Middle meningeal vessel damage results in:
(a) Subdural hemorrhage
(b) Extradural hemorrhage
(c) Subarachnoid hemorrhage
(d) Intracerebral hemorrhage
7. Hypertensive hemorrhage is most commonly seen in :
(a) Basal ganglia
(b) Thalamus
(c) Brain stem
(d) Cerebrum

CNS INFECTIONS, DEMYELINATING DISEASE

8. Enzymes found in CSF:


(AIIMS Nov 2012)
(a) GGT+ALP
(b) ALP+CK-MB
(c) CK+LDH
(d) Deaminase and Peroxidase
9. A 17-year-old female presents with a history of fever and
headache and now develops altered sensorium. CT scan
shows basal exudates with meningeal enhancement. The
CSF is most likely to show:
(AIIMS Nov 2011)
(a) Lymphocytic pleocytosis, low sugar, low protein
(b) Polymorphonuclear pleocytosis, normal sugar, high
protein
(c) Lymphocytic pleocytosis, low sugar, high protein
(d) Lymphocytic pleocytosis, normal sugar, high protein
10. Inclusion body in oligodendroglia is a feature of which of
the following?
(AIIMS Nov 2010)
(a) Progressive multifocal leucoencephalopathy
(b) Japanese encephalitis
(c) Polio
(d) CJD
11. Which of the following is not a Prion disease?
(a) Creutzfeldt-Jakob disease
(DPG 2011)
(b) Fatal familial insomnia
(c) Gerstmann-Straussler-Scheinker syndrome
(d) Parkinson’s disease
12. All of the following diseases show abnormal folding of
proteins except:
(AIIMS Nov 2008)
(a) Creutzfeldt-Jakob disease
(b) Prion disease
(c) Multiple sclerosis
(d) Amyloidosis
13. Prion includes:
(AIIMS Nov 2007)
(a) DNA and RNA
(b) Only RNA
(c) Proteins
(d) Only DNA
14. Pathologic features of brain in AIDS are all, except:
(a) Perivascular giant cell invasion
(AIIMS Nov 2001)
(b) Microglial nodules
(c) Vasculitis
(d) Temporal lobe infarction
15. Febrile response in CNS is mediated by: (PGI Dec
2003)
(a) Bacterial toxin
(b) IL-l
(c) IL-6
(d) Interferon
(e) Tumor Necrosis Factor
16. Cerebral infarction is caused by:
(PGI June 01)
(a) Toxoplasma
(b) Cryptococcus
(c) Aspergillus
(d) Mucor
(e) Histoplasma
17. Complications of tubercular meningitis are:
(a) Endarteritis
(PGI Dec 2000)
(b) Hydrocephalus
(c) Deafness
(d) Venous sinus infarct
18. The pathogenesis of cerebral malaria includes:
(a) Cytoadhesion
(PGI Dec 2000)
(b) Sequestration of cerebral vessels by RBCs
(c) Reticulocytopenia
(d) Also caused by P. vivax
(e) Sporozoites are sequestrated in blood
19. Brain infarct is seen in:
(PGI Dec 2003)
(a) TB
(b) Cryptococcosis
(c) Aspergillosis
(d) Toxoplasmosis
(e) Rabies

MOST RECENT QUESTIONS

20. Commonest cause of cerebral infarction is:


(a) Arterial thrombosis
(b) Arteritis
(c) Venous thrombosis
(d) Embolism
21. Albumino-cytologic dissociation occurs in cases of:
(a) Guillain-Barré syndrome
(b) TB meningitis
(c) Motor neuron disease
(d) Demyelinating disorder
22. Most common type of pathological changes seen in
Rabies are:
(a) Meningitis
(b) Cranial arteritis
(c) Ventriculitis
(d) Brain stem encephalitis
23. What is the histological appearance of brain in
Creutzfeldt-Jakob disease?
(a) Neuronophagia
(b) Spongiform change in brain
(c) Microabscesses
(d) Demyelination
24. Perivascular lymphocytes and microglial nodules are
seen in:
(a) Multiple sclerosis
(b) CMV meningitis
(c) Bacterial meningitis
(d) HIV encephalitis
25. Albumino-cytologic dissociation occurs in cases of:
(a) Guillain Barre syndrome
(b) TB meningitis
(c) Motor neuron disease
(d) Demyelinating disorder
26. Dissociated sensory loss is seen in:
(a) Syringomyelia
(b) Vitamin B12 deficiency
(c) Transverse myelitis
(d) Pellagra
27. Locomotor ataxia, a late manifestation of syphilis due to
parenchymatous involvement of the spinal cord is called:
(a) General paralysis of insane
(b) Tabes dorsalis
(c) Meningovascular syphilis
(d) Syphilitic amyotrophy
28. All of the following are seen in thymoma except:
(a) Hypogamma globulinemia
(b) Hyperalbuminemia
(c) Red cell aplasia
(d) Myasthenia Gravis
29. Spongiform degeneration of cerebral cortex occurs in
which of the following?
(a) Subacute sclerosing panencephalitis
(b) Fatal familial insomnia
(c) Creutzfeldt-Jakob disease
(d) Cerebral toxoplasmosis
30. Which of the following is true about multiple sclerosis?
(a) Affects white matter
(b) Affects gray matter
(c) Rarest demyelinating disorder
(d) Affects males more than females

ALZHEIMER DISEASE, PARKINSON DISEASE AND OTHER


DEGENERATIVE DISEASE

31. Disease or infarction of neurological tissue causes it to


be replaced by:
(AI 2002)
(a) Fluid
(b) Neuroglia
(c) Proliferation of adjacent nerve cells
(d) Blood vessel
32. Nucleus involved in Alzheimer’s disease is:
(a) Nucleus Basalis of Meynert
(Delhi PG-2005)
(b) Superior salivary nucleus
(c) Ventromedial nucleus of thalamus
(d) All of the above
33. Damage to nervous tissue is repaired by:
(a) Neuroglia
(b) Fibroblasts
(DNB- 2001,2005)
(c) Axons
(d) Microglia

MOST RECENT QUESTIONS

34. The following is not a feature of Alzheimer’s disease:


(a) Neurofibrillary tangles
(DNB- 2007)
(b) Senile (neuritic) plaques
(c) Amyloid angiopathy
(d) Lewy bodies
35. Neurofibrillary tangles are seen in:
(a) Parkinsonism
(b) Alzheimer’s disease
(c) Multiple sclerosis
(d) Perivenous encephalomyelitis
36. Dementia in an old man with senile plaques is usually
associated with:
(a) Alzheimer’s disease
(b) Picks disease
(c) Parkinson’s disease
(d) All of the above
37. The following is not a feature of Alzheimer’s disease:
(a) Neurofibrillary tangles
(b) Senile (neuritic) plaques
(c) Amyloid angiopathy
(d) Lewy bodies
38. Which of the following is incorrect about neuro-
blastoma?
(a) Most common abdominal tumor in infants
(b) X-ray abdomen shows calcification
(c) Can show spontaneous regression
(d) Urine contains 5H.I.A.A
39. Commonest cause of cerebral infarction is:
(a) Arterial thrombosis
(b) Arteritis
(c) Venous thrombosis
(d) Embolism
40. Damage to nervous tissue is repaired by:
(a) Neuroglia
(b) Fibroblasts
(c) Axons
(d) Microglia
41. Tau protein are associated with all except:
(a) Parkinsonism
(b) Alzheimer’s disease
(c) Frontotemporal lobar degeneration
(d) Huntington’s disease
42. Alzheimer’s disease associated chromosome is:
(a) 2
(b) 6
(c) 12
(d) 19

CNS TUMORS

43. Which of the following receptor on neuronal membrane


that induces development of glioma?
(AIIMS Nov 2012)
(a) CD45
(b) CD133
(c) CD33
(d) CD24
44. Pituitary adenomas are regarded as macroadenomas
when their size is:
(AI 2012)
(a) > 1 cm
(b) > 1.5 cm
(c) > 2 cm
(d) > 2.5 cm
45. Which is not a neuronal tumor?
(AI 2011)
(a) Ependymoma
(b) Neuroblastoma
(c) Gangliocytoma
(d) Ganglioglioma
46. Which of the following brain tumors does not spread via
CSF?
(DPG 2011)
(a) Germ cell tumors
(b) Medulloblastoma
(c) CNS Lymphoma
(d) Craniopharyngioma
47. A metastatic carcinoma in the brain of an adult, most
often comes from a primary in the:
(AIIMS Nov 2005)
(a) Stomach
(b) Ovary
(c) Oral cavity
(d) Lung
48. Which of the following is true about Medullo-blastoma?
(PGI Dec 2005)
(a) Radiosensitive tumor
(b) Spreads through CSF
(c) Surgical treatment not done
(d) Occurs in young age group

MOST RECENT QUESTIONS

49. Commonest type of intracranial tumor is:


(a) Astrocytoma
(b) Medulloblastoma
(c) Meningioma
(d) Secondaries
50. Most common CNS tumor is:
(a) Astrocytoma
(b) Meduloblastoma
(c) Meningioma
(d) Oligodendroma
51. Glial fibrillary proteins are present in:
(a) Astrocytoma
(b) Meduloblastoma
(c) Ependymoma
(d) All
52. Enamel like superstructure is seen in which CNS lesion?
(a) Craniopharyngioma
(b) Pituitary tumour
(c) Astrocytoma
(d) Glioma
53. Pseudorosettes are seen in all except:
(a) Neuroblastoma
(b) Retinoblastoma
(c) Medulloblastoma
(d) Thecoma
54. Most common intracranial malignancy is:
(a) Glioblastoma multiforme
(b) Ependymoma
(c) Choroid angioma
(d) Pinealoma
55. Worst prognosis meningioma is:
(a) Syncytial
(b) Fibroblastic
(c) Anaplastic
(d) Atypical
56. Which of the following is affected in patients with
Alzheimer’s disease?
(a) Parietal and frontal lobe
(b) Parietal and temporal lobe
(c) Temporal and occipital lobe
(d) Parietal and occipital lobe
57. Lewy bodies are found in the substantia nigra neurons in:
(a) Alzheimer disease
(b) Parkinson disease
(c) Huntington disease
(d) Pick disease
58. Tuberous sclerosis is associated with all except:
(a) Ash leaf macule
(b) Shagreen patch
(c) Schwannoma
(d) Adenoma sebaceum
59. Which of the following is seen in schwannnoma?
(a) Spindle cells
(AIIMS Nov 14)
(b) Storiform pattern
(c) Target cells
(d) Antoni A and B pattern
60. Most common site for medulloblastoma is:
(a) Medulla
(b) Cerebellum
(c) Cerebrum
(d) Pineal gland
61. Medulloblastoma most common metastasis is to:
(a) Lung
(b) Liver
(c) Spleen
(d) CNS
62. Rosenthal fibres are seen in which of the following
tumors?
(a) Pilocytic astrocytoma
(b) Glioblastoma
(c) Medulloblastoma
(d) Ependymoma
63. Most common cerebellar tumor in children:
(a) Medulloblastoma
(b) Ependymoma
(c) Astrocytoma
(d) PNET
64. Commonest type of intracranial tumor is:
(a) Astrocytoma
(b) Medulloblastoma
(c) Meningioma
(d) Secondaries
65. Verocay bodies are seen in:
(AIIMS May’ 14)
(a) Meningioma
(b) Hemangioma
(c) Glioma (d) Schwannoma
66. Most common tumor of head which undergoes
calcification is:
(a) Ependymoma
(b) Medulloblastoma
(c) Oligodendroglioma
(d) Glioblastoma multiformae
67. Which chromosome mutation is associated with
medulloblastoma?
(a) Chromosome 16
(b) Chromosome 17
(c) Chromosome 18
(d) Chromosome 19

ASSERTION AND REASON QUESTIONS


1-5. Will have two statements, assertion and reason. Read
both of them carefully and answer according to these
options.
(a) Both assertion and reason are true and reason is correct
explanation of assertion.
(b) Both assertion and reason are true and reason is not the
correct explanation of assertion.
(c) Assertion is true and reason is false.
(d) Both assertion and reason are false.
1. Assertion: Berry aneurysm is the commonest cause of
subarachnoid hemorrhage
Reason: Rupture of the aneurysm occurs commonly in childhood
2. Assertion: B12 deficiency causes subacute combined
degeneration of the spinal cord
Reason: B12 deficiency causes degeneration of both the
ascending and descending tracts of the spinal cord
3. Assertion: Alzheimer’s disease is associated with Down
syndrome
Reason: Beta amyloid gene is located on chromosome 21
4. Assertion: Shy-Dragger syndrome is characterized by
autonomic dysfunction
Reason: Lewy bodies are found in nigrostriatal neurons
5. Assertion: Huntington’s disease is characterized by chorea
and dementia
Reason: Degeneration of GABA neurons in the striatum leads to
decreased function (decreased inhibition) and increased
movement.
1. Ans. (b) Hypertrophy of abductor pollicis brevis
(Ref: Robbins 8th/1286, 9/e p1258)
Atrophy and not hypertrophy of the abductor pollicis brevis is a
feature of syringomyelia.
Syringomyelia

• It is a chronic myelopathy that results from formation of a cavity


(syrinx) involving the central gray matter of the spinal cord. The cause
of syringomyelia is unknown, although one type is associated with a
Chiari malformation with obstruction at the foramen magnum.
• Since the gray matter is the location where pain fibers cross to join the
contralateral spinothalamic tract, the interruption of the lateral
spinothalamic tracts results in segmental sensory dissociation with
loss of pain and temperature sense, but preservation of the sense of
touch and pressure or vibration, usually over the neck, shoulders, and
arms.
• The most common location of a syrinx is the cervicothoracic region
and therefore, the loss of pain and temperature sensation affects
both arms.
• Other features of syringomyelia include wasting of the small
intrinsic hand muscles (claw hand) and thoracic scoliosis. This is
accompanied by areflexic weakness in the upper limbs. As the cavity
enlarges, spasticity and weakness of the legs, bladder and bowel
dysfunction as well as Horner syndrome appear due to compression
of the long tracts.
• The diagnosis of syringomyelia is best made with MRI of the
spine (cervical region should be examined first)

2. Ans. (b) Degeneration of tunica media


(Ref: Robbins 8th/1297-8, 9/e p1270)
Direct quote from Robbins… ‘the berry aneurysms develop over
time because of an underlying defect in the media of the
vessel”.
Concept
It is different from other causes of aneurysm (atherosclerosis, trauma,
infections) which cause only cerebral infarction and not subarachnoid
hemorrhage.
Salient points about Berry aneurysm (saccular aneurysm)
Saccular aneurysm is the most common type of intracranial
aneurysm.
Risk factors
Smoking and hypertension are the important risk factors for Berry
aneurysm. They are not the result of atherosclerosis (which is a
disease of intima).
Berry aneurysms are called congenital, although the aneurysm itself is
not present at birth.
Location
Commonest location is in the circle of Willis, typically at the junction of
the anterior communicating artery with the anterior cerebral arteryQ.
It is also present at the junction of the middle cerebral artery and the
posterior communicating artery.
Clinically
It is responsible for a clinically significant subarachnoid hemorrhage. In
fact, excluding trauma, berry aneurysm is the commonest cause of
subarachnoid hemorrhage.
The chance of rupture of berry aneurysms increases with age (rupture
is rare in childhood). Rupture causes marked bleeding into the
subarachnoid space and produces severe headaches, typically
described as the “worst headache ever”.

3. Ans. (b) Degeneration of media


(Ref: Robbins 8th/1297-8, 9/e p1270)
Direct quote from Robbins… ‘the berry aneurysms develop over
time because of an underlying defect in the media of the
vessel”.
Concept
It is different from other causes of aneurysm (atherosclerosis, trauma,
infections) which cause only cerebral infarction and not subarachnoid
hemorrhage.

4. Ans. (b) Posterior fossa volume


(Ref: Robbin’s illustrated 6th/27691, 9/e p1255)
The basics of CSF production and drainage
CSF production (Occurs by choroid plexus of lateral and IIIrd ventricle)

Foramen of Monro

Third ventricle

Aqueduct of Sylvius

Fourth ventricle

Foramen of Magendie and Lushka

Subarachnoid space (It is absorbed here, by arachnoid villi)

Both Aqueductal stenosis and Dandy Walker syndrome cause non-


communicating hydrocephalus but the site of obstruction is
different.
• Let us see the causes of hydrocephalus in aqueductal
stenosis and Dandy-Walker malformation.
Aqueductal stenosis

• In aqueductal stenosis the aqueduct connecting the 3rd and 4th


ventricle is stenosed which leads to hydrocephalus with the dilatation
of ventricular system prior to the aqueduct, i.e. lateral ventricles and
third ventricle.

Dandy Walker Malformation


• In Dandy Walker malformation there is cystic dilatation of the fourth ventricle
in the posterior fossa with obstruction at the formation of Lushka and
Magendie.
• So the ventricular system in this condition will also be dilated as in
aqueductal stenosis but here the posterior fossa is also enlarged (in contrast
to normal posterior fossa size in aqueductal stenosis). i.e.
5. Ans. (b) Anterior communicating artery
(Ref: Robbins 9/e p1270, 8th/1297; 7th/1367)
6. Ans. (b) Extradural hemorrhage
(Ref: Robbins 9th/1261)
7. Ans. (a) Basal ganglia
(Ref: Robbins 9th/1268)
Direct quote … “Hypertensive intraparenchymal hemorrhage may
originate in the putamen (50% to 60% of cases), thalamus,
pons, cerebellar hemispheres (rarely), and other regions of
the brain”. Putamen is a part of basal ganglia, hence, the
answer here.
8. Ans. (c) CK + LDH
(Ref: Chatterjee Shinde 8th/730)
The following enzymes are present in the CSF:
Aspartate transaminase (AST): 5- 12 units/ml. Its value increases in
abscess, cerebral hemorrhage and infarction and in primary or
metastatic malignant disease. It may increase in some patients with
multiple sclerosis.
Lactate dehydrogenase (LDH): normal value is 5- 40IU/l. Its value
increases in abscess, cerebral hemorrhage and infarction and in
metastatic malignant disease.
Increase in LDH4 isoenzyme of CSF is seen in tuberculous meningitis.
Creatine kinase (CK): CK-BB is present in the brain. Its value increases
in associated with meningitis, cerebral hemorrhage and infarction.
>30 units/ml is suggestive of tubercular meningitis and <30 units/ml is
suggestive of pyogenic meningitis.

In MI, lDH1 increases but in heart failure, LDH 5 increases


because right sided heart failure causing hepatic congestion
and release of LDH5 from them. ….Dinesh Puri 3rd/122-3
9. Ans. (c) Lymphocytic pleocytosis, low sugar, high protein
(Ref: Harrison 18th/3426, Harsh Mohan 6th/appendix)
CT scan shows basal exudates with meningeal enhancement is
highly suggestive of tuberculous meningitis. For CSF
changes in CNS infection see text.
10. Ans. (a) Progressive multifocal leukoencephalopathy
(PML)
(Ref: Robbins 8th/1305, 9/e p1278)
Progressive multifocal leukoencephalopathy (PML) is a
demyelinating disease of the central nervous system that
results from infection of oligodendrocytes by the JC
polyomavirus. It occurs almost exclusively in
immunocompromised individuals as in HIV due to reactivation
of the virus.
Microscopic examination shows lesions in the white matter
which is an area of demyelination, in the center of which are
scattered lipid-laden macrophages and a reduced number of
axons. At the edge of the lesion are greatly enlarged
oligodendrocyte nuclei whose chromatin is replaced by
glassy amphophilic viral inclusion…..Robbins
Significance of microscopic findings in PML
In PML, the virus also infects astrocytes, leading to bizarre giant
forms with irregular, hyperchromatic, sometimes multiple nuclei that
can be mistaken for tumor.

OTHER OPTIONS
• In acute cases of Polio, there is mononuclear cell perivascular cuffs
and neuronophagia of the anterior horn motor neurons of the
spinal cord.
• In CJD, microscopically, there is characteristic spongiform change
in the gray matter (“cluster of grapes” vacuolation) without
inflammation.

11. Ans. (d) Parkinson’s disease


(Ref: Harrison 17th/2647, Robbins 8th/1308, 9/e p1281)
Prion’s disease of humans include:
• Iatrogenic Creutzfeldt-Jakob disease
• Sporadic Creutzfeldt-Jakob disease
• Variant Creutzfeldt-Jakob disease
• Fatal Familial Insomnia
• Gerstmann-Straussler-Scheinker syndrome
• Sporadic Fatal Insomnia
• Kuru
12. Ans. (c) Multiple sclerosis
(Ref: Harrison 17th/2647; Robbins 9/e p57, 1283-1284)
Disorders caused by misfolding of proteins are
• Amyloidosis
• Alzheimer’s disease and other neurodegenerative
diseases
• Transmissible prion diseases like CJD
• Some genetic diseases caused by mutations that lead to
misfolding of protein and loss of function, such as
certain of the cystic fibrosis mutations.
Prions are infectious proteins that cause degeneration of the
central nervous system (CNS). Prion diseases are disorders
of protein conformation, the most common of which in
humans is called Creutzfeldt-Jakob disease (CJD). CJD
typically presents with dementia and myoclonus, is
relentlessly progressive, and generally causes death within a
year of onset.
Four new concepts have emerged from studies of prions:
• Prions are the only known infectious pathogens that are devoid of
nucleic acid; all other infectious agents possess genomes
composed of either RNA or DNA that direct the synthesis of their
progeny.
• Prion diseases may manifest as infectious, genetic, and sporadic
disorders; no other group of illnesses with a single etiology
presents with such a wide spectrum of clinical manifestations.

• Prion diseases result from the accumulation of PrPSc, the


conformation of which differs substantially from that of its
precursor, PrPC.
• PrPSc can exist in a variety of different conformations, each of
which seems to specify a particular disease phenotype. How a
specific conformation of a PrPSc molecule is imparted to PrPC
during Prion replication to produce nascent PrPSc with the same
conformation is unknown. Additionally, it is unclear what factors
determine where in the CNS a particular PrPSc molecule will be
deposited.

13. Ans. (c) Proteins


(Ref: Harrison 17th/2646; 16th/2495, 9/e p1281)
• Prions are infectious proteins that cause degeneration of
the central nervous system (CNS). Prion diseases are
disorders of protein conformation, the most common of
which in humans is called Creutzfeldt-Jakob disease
(CJD).
• In mammals, prions reproduce by binding to the normal,
cellular isoform of the prion protein (PrPC) and
stimulating conversion of PrPC into the disease-causing
isoform (PrPSc). PrPC is rich in alpha-helix and has little
beta-structure, while PrPSc has less alpha-helix and a
high amount of beta-structure. This alpha-to-beta
structural transition in the prion protein (PrP) is the
fundamental event underlying prion diseases.
• Prions are the only known infectious pathogens that are devoid of
nucleic acid; all other infectious agents possess genomes
composed of either RNA or DNA that direct the synthesis of their
progeny.
14. Ans. (c) Vasculitis
(Ref: Anderson10th/2728, Robbins 9/e p1278)
Anderson clearly mentioned that “Unlike most other encephalitides,
HIV does not seem to infect neurons and perivasculitis is
conspicuously absent”
“Characteristic multinuclear Giant Cells of Macrophage origin are
seen in white matter of frontal and temporal lobes particularly
in perivascular location”
Feature of CNS involvement in AIDS
• Diffuse and focal spongiform changes
• Vacuolar myelopathy of posterior column of spinal cord
• Major cells affected are macrophages and monocytes
• Most characteristics finding is chronic inflammatory reaction with
widely distributed infiltrates of microglial nodules

15. Correct answer: (a) Bacterial toxin; (b) IL-l; (d) Interferon;
(e) Tumor necrosis factor (TNF).
(Ref: Robbins 7th/84, Harrison 16th/106)
Fever is produced in response to substances called pyrogens that
act by stimulating prostaglandin synthesis in the vascular and
perivascular cells of hypothalamus.
They can be classified as
• Exogenous pyrogens → Lipopolysaccharides (bacterial
toxin) stimulate WBCs to release endogenous pyrogens.
• Endogenous pyrogens → IL-1 (α, β) and TNF-α, IL-6,
Ciliary neurotropic factor and interferons that increase
the enzyme (cyclooxygenase) that converts arachidonic
acid into prostaglandins.
NSAIDs reduce fever by inhibiting cyclooxygenase.
16. Ans. (a) Toxoplasma; (c) Aspergillus; (d) Mucor
(Ref: Robbins 7th/1363, 1378)
Arteritis of small and large vessels causes cerebral infarcts.
Causes of cerebral infarction:
• Syphilis
• Tuberculosis
• Infectious vasculitis in setting of immunosuppression
• Toxoplasmosis
• Aspergillosis
• CMV encephalitis
• Mucormycosis
17. Ans. (a) Endarteritis; (b) Hydrocephalus
(Ref: Robbins 9/e p1274, 7th/1371, OPG’ 6th/520-22)
Most serious complication of chronic tubercular meningitis

• Arachnoid fibrosis → Hydrocephalous


• Obliterative endarteritis → Arterial occlusion and infarction of
underlying brain.
• Spinal roots may also be affected.
• Calcification
• Tuberculomas

18. Ans. (a) Cytoadhesin ; (b) Sequestration of cerebral


vessels by RBCs
(Ref: Harrison 16th/1222, Robbins 9/e p391)
Cerebral malaria is caused by P. falciparum.
Pathophysiology of cerebral malaria

Cytoadhesion: On the surface of infected RBC appears an antigen


called PfEMP1 (P. falciparum erythrocyte membrane protein-I). Due to
this antigen infected RBCs adheres in the blood vessels.
Sequestration: It is the key events of falciparum pathology. Brain
capillaries especially capillaries of white matter of brain are plugged with
parasitized RBCs which terminally leads to thrombosis. Three receptors
for parasitized RBC have been identified (ICAM - 1, CD36,
thrombospondin). These cause focal cerebral damage.
Rosetting occurs in middle of a sexual life cycle. It is seen in cerebral
malaria in brain capillaries.

19. Ans. (a) TB; (c) Aspergillosis; (d) Toxoplasmosis.


(Ref: Robbins’ 7th/1363, 1378)
20. Ans. (a) Arterial thrombosis
(Ref: Robbins 9/e p1263)
21. Ans. (a) Guillain-Barré syndrome
(Ref: Harrison 17th/2667, Robbin 9/e p1231)
22. Ans. (d) Brain stem encephalitis
(Ref: Robbins 9/e p1277, 8th/1304-1305; 7th/1375)
23. Ans. (b) Spongiform change in brain….see earlier
explanation.
(Ref: Robbins 9/e p1282)
24. Ans. (d) HIV encephalitis
(Ref: Robbins 9/e p1278)
• HIV encephalitis is best characterized microscopically
as a chronic inflammatory reaction with widely
distributed infiltrates of microglial nodules.
• The microglial nodules are also found in the vicinity of
small blood vessels, which show abnormally prominent
endothelial cells and perivascular foamy or pigment-
laden macrophages. These changes occur especially
in the subcortical white matter, diencephalon, and
brainstem.
• An important component of the microglial nodule is the
macrophage-derived multinucleated giant cell.
25. Ans. (a) Guillain-Barré Syndrome
(Ref: Robbins 8/e p1262, 9/e p1231, Harrison 17/e p2667)
In patients with Guillanin Barre syndrome, there is elevation of the
CSF protein due to inflammation and altered permeability of
the microcirculation within the spinal roots as they traverse
the subarachnoid space. Inflammatory cells are contained
within the roots, however, and there is little to no CSF
pleocytosis. This is termed as albumin-cytological
dissociation.
26. Ans. (a) Syringomyelia
(Ref: Robbins 9th/1258)
27. Ans. (b) Tabes dorsalis
(Ref: Robbins 9th/1275)
• Tabes dorsalis is the result of damage to the sensory
axons in the dorsal roots. This causes impaired joint
position sense and ataxia (locomotor ataxia); loss of
pain sensation, leading to skin and joint damage
(Charcot joints); other sensory disturbances like the
characteristic “lightning pains”; and absence of deep
tendon reflexes.
• Meningovascular neurosyphilis is chronic meningitis
involving the base of the brain. It may be associated
with obliterative endarteritis (Heubner arteritis)
accompanied by a distinctive perivascular inflammatory
reaction. Cerebral gummas (plasma cell-rich mass
lesions) may also be present.
• General paresis of the insane is caused by the invasion
of the brain by T. pallidum. It manifests as progressive
cognitive impairment associated with mood alterations
(including delusions of grandeur) terminating in severe
dementia.
• Syphilitic amyotrophy presents with painless and
progressive weakness.
28. Ans. (b) Hyperalbuminemia
(Ref: Robbins 9/e p627)
Direct lines…“In addition to myasthenia gravis, other associated
autoimmune disorders with thymoma include
hypogammaglobulinemia, pure red cell aplasia, Graves’
disease, pernicious anemia, dermatomyositis-polymyositis,
and Cushing syndrome”.
29. Ans. (c) Creutzfeldt-Jakob disease (CJD)
(Ref: Robbins 9/e p1282)
• In CJD, on microscopic examination, the pathognomonic
finding is a spongiform transformation of the cerebral
cortex and, often, deep gray-matter structures (caudate,
putamen).
• This multifocal process results in the uneven formation of
small, apparently empty, microscopic vacuoles of
varying sizes within the neuropil and sometimes in the
perikaryon of neurons.
• In advanced cases there is severe neuronal loss,
reactive gliosis, and sometimes expansion of the
vacuolated areas into cystlike spaces (“status
spongiosus”).
• No inflammatory infiltrate is present.
30. Ans. (a) Affects white matter
(Ref: Robbins 9/e p1283)
31. Ans. (b) Neuroglia
(Ref: Robbins 7th/1349, 9/e p1252)
• Neuroglia (astrocytes) are the principal cells in the
central nervous system responsible for reaction to injury
repair and scar formation in the brain. They perform
function similar to fibroblasts in the CNS.
• Gliosis is the most important histopathological indicator
of CNS injury. Astrocytes participate in this process by
undergoing both hypertrophy and hyperplasia.
• Neuroglial cells can be ectodermal in origin (e.g.
astrocytes and oligodendrocytes) or derived from
mesoderm (microglia).
• Microglia resemble macrophages and act as scavenger
cells whereas oligodendrocytes help in myelin formation
[similar to Schwann cells in PNS]
32. Ans. (a) Nucleus basalis of Meynert
(Ref: Harrison 16th/2395, Robbins 9/e p1290)
• The brain of Alzheimer’s disease patients shows severe
neuronal loss in the nucleus basalis of Meynert, the
major source of cholinergic input to cerebral cortex.
• The major microscopic abnormalities of Alzheimer’s
disease are:
– Neurofibrillary tangles, neuropil threads
– Senile (Neuritic) plaques
– Amyloid angiopathy
– Granulovacuolar degeneration
• A dominant component of the plaque core is Aβ, a
peptide of approximately 40-43 amino acid residues
derived from a larger molecule, amyloid precursor
protein (APP).
• Although neurofibrillary tangles are characteristic of
Alzheimer’s disease, they are not specific to this
condition.
• Hirano bodies: Found especially in Alzheimer’s disease,
are elongated, glassy, eosinophilic bodies consisting of
paracrystalline arrays of beaded filaments with actin as
their major component.
33. Ans. (a) Neuroglia
(Ref: Robbins 8th/1282, 9/e p1252)
34. Ans. (d) Lewy bodies
(Ref: Robbins 9/e p1290)
35. Ans. (b) Alzheimer’s disease
(Ref: Robbins 8th/1314)
36. Ans. (a) Alzheimer’s disease
(Ref: Robbins, 9/e p1290)
37. Ans. (d) Lewy bodies
(Ref: Robbins 9/e p1290)
The following are the histopathological features of Alzheimer’s
disease:
• Neuritic plaques : diagnostic feature
• Neurofibrillary tangles : diagnostic feature
• Cerebral amyloid angiopathy (CAA)
• Hirano bodies
38. Ans. (d) Urine contains 5H.I.A.A
(Ref: Robbins 9th/478)
• Neuroblastoma is the most common extracranial solid
cancer in childhood and the most common cancer in
infancy.
• About 90% of neuroblastomas, regardless of location,
produce catecholamines, which are an important
diagnostic feature (i.e., elevated blood levels of
catecholamines and elevated urine levels of the
metabolites vanillylmandelic acid and homovanillic acid.
• Increased urinary 5HIAA is a feature of carcinoid tumour
and not neuroblastoma.
39. Ans. (a) Arterial thrombosis
(Ref: Robbins 8/e p1291-1292)
The majority of thrombotic occlusions are due to atherosclerosis.
The most common sites of primary thrombosis causing
cerebral infarction are the carotid bifurcation, the origin of the
middle cerebral artery, and either end of the basilar artery….
Robbins
40. Ans. (d) Neuroglia
(Ref: Robbins 8/e p1282, 9/e p1252)
“Gliosis is the most important histopathological indicator of CNS
injury regardless of its etiology and is characterized by both
hypertrophy and hyperplasia.” The chief cell involved in this
reaction is an astrocyte. Please remember that the
oligodendrocytes and the ependyma do not participate in
active response to injury.
In contrast, microglia are the fixed macrophage system in the CNS.
41. Ans. (d) Huntington’s disease (Ref: Robbins 9/e p1288)
Relationship between proteins and
neurodegenerative diseases
Protein Diseases with inclusions
Aβ Alzheimer disease
Tau Alzheimer disease
Frontotemporal lobar degeneration
Parkinson disease (with LRRK2 mutations)
Progressive supranuclear palsy
Corticobasal degeneration
TPD-43 Frontotemporal lobar degeneration
Amyotrophic lateral sclerosis
FUS Frontotemporal lobar degeneration
Amyotrophic lateral sclerosis
α-synuclein Parkinson disease
Multiple system atrophy
Polyglutamine Huntington disease
aggregates
Some forms of spinocerebellar ataxia

42. Ans. (d) 19


(Ref: Robbins 9/e p1289)
• In familial Alzheimer disease and Down syndrome, the gene
encoding APP on chromosome 21 is involved.
• The genetic locus on chromosome 19 that encodes apolipoprotein
E (ApoE) has a strong influence on the risk of developing AD.
43. Ans. (b) CD133
(Ref: Wintrobe’s 12th/2559)
• CD 133 is used as a marker for leukemia and
glioblastoma stem cell. It is also used for identifying
immature leukemic stem cell in AML and pro B
leukemia.
• CD 45 is required for lymphocyte activation. Its
deficiency causes Severe Combined Immunodeficiency
disease (SCID).
44. Ans. (a) > 1 cm
(Ref: Robbins 8th/1100, 9/e p1075)
Pituitary adenomas are designated as the following:
• Microadenomas if they are less than 1 cm in diameter
and
• Macroadenomas if they exceed 1 cm in diameter

The most common cause of hyperpituitarism is an adenoma arising


in the anterior lobe.

45. Ans. (a) Ependymoma


(Ref: Robbins 8th/1330, 9/e p1306)
The four major classes of brain tumors are:
1. Gliomas

Astrocytoma
• Pleomorphic xanthoastrocytoma
• Brainstem glioma
• Pilocytic astrocytoma
• Fibrillary (diffuse) astrocytomas
• Glioblastoma
Oligodendroglioma
Ependymoma

2. Neuronal tumors

Ganglion cell tumors


Gangliocytoma
Ganglioglioma
Dysembryoplastic neuroepithelial tumor
Cerebral neuroblastomas

3. Poorly differentiated neoplasms

4. Meningiomas

46. Ans. (d) Craniopharyngioma


(Ref: Robbins 9/e p1082)
Brain tumors spreading via CSF are
• Ependymoma
• Medulloblastoma
• Choroid plexus carcinoma
• Astrocytoma
• Germinoma
• Pineoblastoma
• CNS Lymphoma
47. Ans. (d) Lung
(Ref: Robbins 8th/1339, 9/e p1315)
48. Ans. (a) Radiosensitive tumor; (b) Spreads through CSF;
(d) Occurs in young age group.
(Ref: Robbins 7th/1407; 9/e p1312 Harrison 16th/2455)
Medulloblastoma:
• The tumor occurs predominantly in children and exclusively in
cerebellum (infratentorial tumor).
• Histopathological hallmark is Homer-Wright rosettes.
• Dissemination of tumor occurs through CSF.
• The tumor is highly malignant.
• It is a radiosensitive tumor. Radiotherapy improves the survival in
children.
• Cranio-spinal irradiation (CSI) reduces the recurrence from CSF
dissemination.
49. Ans. (d) Secondaries
(Ref: Robbins 8th/1339, 9/e p1315)
50. Ans. (a) Astrocytoma
(Ref: Robbins 8th/1330, 9/e p1306)
51. Ans. (a) Astrocytoma
(Ref: Robbins 8th/1330, 9/e p1253)
52. Ans. (a) Craniopharyngioma
(Ref: Robbins 9/e p1082)
53. Ans. (d). Thecoma (Ref: Robbins 8th/1051; 7th/1442)
54. Ans. (a) Glioblastoma multiforme
(Ref: Robbins 8th/1330-1331, 9/e p1308)
55. Ans. (c) Anaplastic
(Ref: Robbins 9/e p1315)
56. Ans. (b) Parietal and temporal lobe
(Ref: Robbin 8/e p1314)
Grossly, the brain shows a variable degree of cortical atrophy
marked by widening of the cerebral sulci that is most
pronounced in the frontal, temporal, and parietal lobes..
(Ref: Robbind 8/e p 1314
• Hippocampus and other medial temporal lobe structures
are the earliest and most severely affected in
Alzheimer’s disease.
• Bradley’s Neurology mentions that.. “cortical atrophy is
most pronounced in the temporal and parietal lobe
with the frontal lobe involvement being later in the
disease”.
57. Ans. (b) Parkinson disease
(Ref: Robbins 9th/1294)
58. Ans. (c) Schwannoma
(Ref: Robbins 9th/1316-7)
• Tuberous sclerosis is an autosomal dominant syndrome
characterized by the development of hamartomas and
benign neoplasms involving the brain and other tissues.
Elsewhere in the body, renal angiomyolipomas, retinal
glial hamartomas, pulmonary
lymphangioleiomyomatosis and cardiac rhabdomyomas
develop over childhood and adolescence. Cysts may be
found at various sites, including the liver, kidneys, and
pancreas.
• Cutaneous lesions include angiofibromas, localized
leathery thickenings (shagreen patches),
hypopigmented areas (ash-leaf patches), sebaceous
adenomas and subungual fibromas.
59. Ans. (d) Antoni A and B pattern
(Ref: Robbins 9th/1247)
These are benign tumors that exhibit Schwann cell differentiation
and often arise directly from peripheral nerves. They are a
component of neurofibromatosis -2 (NF2). Microscopically,
they are comprised of an admixture of dense and loose areas
referred to as Antoni A and Antoni B areas, respectively.
The dense eosinophilic Antoni A areas often contain spindle cells
arranged into cellular intersecting fascicles. Palisading of
nuclei is common and “nuclear-free zones” that lie between
the regions of nuclear palisading are termed Verocay
bodies.
Also know!

• The Schwann cell tumors have a uniform immunoreactivity for S-100.


• Schwannomas may recur locally if incompletely resected, but
malignant transformation is extremely rare.

Storiform pattern is seen in dermatofibrosarcoma protuberans.


This is composed of closely packed fibroblasts arranged radially.
60. Ans. (b) Cerebellum
(Ref: Robbins 8/e p1336, 9/e p1312)
Medulloblastoma
• In children, the location is in the midline of the cerebellum, but
lateral locations are more often found in adults.
• Medulloblastomas are the most common malignant brain tumor
of childhood
• 5% of children have it in association with Gorlin syndromeQ (the
most common of the inherited disorders due to mutations in the
patched-1; PTCH-1 gene.
• Histologically, there is presence of Homer-Wright rosettesQ.

• Dissemination through the CSF is a common


complicationQ, presenting as nodular masses
elsewhere in the CNS, including metastases to the
cauda equina that are termed drop metastasesQ.

61. Ans. (d) CNS…………….explained earlier


(Ref: Robbins 9/e p1312)
62. Ans. (a) Pilocytic astrocytoma
(Ref: Robbins 9/e p1309)
On microscopic examination of pilocytic astrocytoma, the tumor
is composed of bipolar cells with long, thin “hairlike”
processes that are GFAP-positive and form dense fibrillary
meshworks; Rosenthal fibers and eosinophilic granular
bodies, are often present.
63. Ans. (c) Astrocytoma
(Ref: Robbins 8/e)
64. Ans. (d) Secondaries
(Ref: Robbins 8/e p1339, 9/e p1315)
• Metastatic lesions, mostly carcinomas, account for
approximately a quarter to half of intra-cranial tumors in
hospitalized patients.
• The meninges are also a frequent site of involvement by
metastatic disease.
65. Ans. (d) Schwannoma
(Robbins 9/e p1247)
Schwannomas are well-circumscribed, encapsulated masses that
abut the associated nerve without invading it. Microscopically,
they are comprised of an admixture of dense and loose areas
referred to as Antoni A and Antoni B areas, respectively.
• The dense eosinophilic Antoni A areas often contain spindle cells
arranged into cellular intersecting fascicles. Palisading of nuclei is
common and “nuclear-free zones” that
lie between the regions of nuclear palisading are termed Verocay
bodies.
• In the loose, hypocellular Antoni B areas the spindle cells are
spread apart by a prominent myxoid extracellular matrix that may
be associated with microcyst formation.

66. Ans. (c) Oligodendroglioma


(Robbins 9/e p1310, Neuroradiology/565)
67. Ans. (b) Chromosome 17
(Robbins 9/e p1312)

EXPLANATIONS TO ASSERTION AND


REASON QUESTIONS
Explanations (1-5): While solving assertion reason type of
questions, we can use a particular method.
• First of all, read both assertion (A) and reason (R)
carefully and independently analyse whether they
are true or false.
• If A is false, the answer will directly be (d) i.e. both A
and R are false. You can note that all other options
(i.e. a, b or c) consider A to be true.
• If A is true, answer can be (a), (b) or (c), Now look at
R. If R is false, answer will be (c)
• If both A and R are ture, then we have to know
whether R is correctly explaining A [answer is (a)] or
it is not the explanation of assertion [answer is (b)]
1. Ans. (d) Both assertion and reason are false.
(Ref: Robbins 8th/1297, 9/e p1270)
Already explained in text and answer 3.
2. Ans. (a) Both assertion and reason are true and reason is
correct explanation of assertion.
(Ref: Robbins 8th/1321, 9/e p1304)
Vitamin B12 deficiency leads to a swelling of myelin layers,
producing vacuoles that begin segmentally at the mid-
thoracic level of the spinal cord in the early stages. With time,
axons in both the ascending tracts of the posterior columns
and the descending pyramidal tracts degenerate. This is
called as subacute combined degeneration of the spinal cord.
Concept
Though isolated involvement of descending or ascending tracts may be
a feature of many spinal cord diseases, the combined degeneration of
both ascending and descending tracts of the spinal cord is
characteristic of vitamin B12 deficiency.
3. Ans. (a) Both assertion and reason are true and reason is
correct explanation of assertion.
(Ref: Robbins 8th/1314, 9/e p1288)
The gene for amyloid precursor protein (APP) is located on
chromosome 21. APP gene mutation causes increased
generation of Aβ. Alzheimer disease is associated with
trisomy 21. This is related to a gene dosage effect with
increased production of APP and subsequently Aβ.
Other genes linked to early-onset familial Alzheimer disease are:
Chromosomes 14Q and 1Q as these two chromosomes encode highly
related intracellular proteins, presenilin-1 (PS1) and presenilin-2
(PS2).

4. Ans. (b) Both assertion and reason are true and reason is
not the correct explanation of assertion.
(Ref:Robbins 8th/1319-1321)
Shy-Dragger syndrome is characterized by autonomic dysfunction
including orthostatic hypotension, impotence, abnormal
sweat and salivary gland secretion, and pupillary
abnormalities. The Lewy bodies are found in sympathetic
neurons in the spinal cord.
• Lewy bodies in the nigrostriatal neurons produce
extrapyramidal symptoms and are a feature of classic
Parkinson’s disease.
• Lewy bodies in the cerebral cortex produce dementia
and are a feature of Lewy body dementia.
• Concept
Unlike Parkinson’s disease, however, no mutations in the gene coding
for alpha-synuclein has been found with the Shy-Drager syndrome.

5. Ans. (a) Both assertion and reason are true and reason is
correct explanation of assertion.
(Ref: Robbins 8th/1322, 9/e p1297)
Huntington’s disease has already been explained in the
text.
1. A 20-year-old presented with swelling in the wrist joint for
2 years duration. Histopathological examination showed
spindle shaped cells and verocay bodies. Which of the
following is the diagnosis?
(NEET 2020 like pattern)
(a) Neurofibroma
(b) Schwannoma
(c) Lipoma
(d) Dermoid cyst
Ans. (b) Schwannoma
(Ref: Robbins 9th/1247)
• Schwannomas are well-circumscribed, encapsulated
masses that abut the associated nerve without invading
it.
• Microscopically, they are comprised of an admixture of
dense and loose areas referred to as Antoni A and
Antoni B areas, respectively. The dense eosinophilic
Antoni A areas often contain spindle cells arranged into
cellular intersecting fascicles. Palisading of nuclei is
common and “nuclear-free zones” that lie between the
regions of nuclear palisading are termed Verocay
bodies.

Schwannoma with Verocay Body (V)


2. Risk factors for Alzheimer’s disease include?
(NEET 2019 like pattern)
(a) Klinefelter syndrome
(b) Noonan syndrome
(c) Down’s syndrome
(d) Turner syndrome
Ans. (c) Down’s syndrome
(Ref: Robbins 9th e/p 1288)
The gene encoding APP, on chromosome 21, lies in the Down
syndrome region; AD pathology is an eventual feature of the
cognitive impairment of these individuals.
3. Which apolipoprotein is responsible for increased risk of
Alzheimer’s disease?
(NEET 2019 like pattern)
(a) APOE1
(b) APOE2
(c) APOE3
(d) APOE4
Ans. (d) APOE4
(Ref: Robbins 9th e/p 1289)
The dosage of the ε4 allele increases the risk of AD and lowers
the age of onset of the disease.
4. Which of the pathognomic feature of Alzheimer’s disease?
(AI 2018 Pattern)
(a) Lewy body
(b) Plaques and tangles
(c) Pick bodies
(d) Red neuronal degeneration
Ans. (b) Plaques and tangles
(Ref: Robbins 9/e p1290)
The major microscopic abnormalities of AD are neuritic (senile)
plaques and neurofibrillary tangles.
5. Patient presented with painless proptosis. Biopsy from the
orbital mass showed the following image.
(AIIMS Nov 2017 Pattern)
(a) Neurofibroma
(b) Schwannoma
(c) Rhabdomyoma
(d) Leiomyoma
Ans. (b) Schwannoma
(Ref: Robbins 9/e p1247)
The presence of Antoni A and Antoni B areas in this often asked
question confirms the diagnosis of schwannoma of this
patient.
Microscopically, schwannomas are comprised of an mixture of
dense and loose areas referred to as Antoni A and Antoni B
areas, respectively. The dense eosinophilic Antoni A areas
often contain spindle cells arranged intocellular intersecting
fascicles. Palisading of nuclei is commonand “nuclear-free
zones” that lie between the regions of nuclear palisading are
termed Verocay bodies.
Disclaimer
Any resemblance to an actual question is purely coincidental.
• Microadenoma of the pituitary is < 1 cm whereas macroadenoma are > 1 cm.
Normal weight of thyroid gland: 15–25 gm.
Commonest cause of hyperthyroidism is Graves’ disease. Hyperthyroidism has
proximal muscular weakness (and not Distal muscle weakness).
The commonest cause of hypothyroidism is Hashimoto thyroiditis. It is also
associated with Type I DM, autoimmune adrenalitis, SLE, myasthenia gravis
and Sjogren syndrome.
• Orphan Annie eye nuclei is characteristic of papillary thyroid carcinoma.

• Hurthle cells are seen in: Follicular carcinoma, follicular adenoma,


Hashmoto thyroiditis.
FNAC is not diagnostic in follicular thyroid carcinoma. The follicular adenoma is
differentiated from follicular carcinoma by the capsular invasion or vascular
invasion.
Struma ovarii is the ectopic mature thyroid tissue in an ovarian tumor.
Crooke’s hyaline change is seen in pituitary gland of individuals having
Cushing’s syndrome.
Most common cause of Adrenocortical insufficiency (Addison’s disease) is
Autoimmune adrenalitis (World) but tuberculosis (in India).
• Zellballen pattern (nest of cells) is seen in Pheochromocytoma and
paraganglioma.
• Malignancy of pheochromocytoma is confirmed by Metastasis.
Ultrastructural finding of paraganglioma: Dense core granule (neurosecretory
granule).
Most common cause of asymptomatic hypercalcemia is Parathyroid adenoma
whereas the symptomatic hypercalcemia is associated with Malignancy.
Diagnostic feature of malignancy in parathyroid tumor: Metastasis.

PANCREAS

The endocrine pancreas consists of the islets of Langerhans, which contain


four major cell types-b, a, d, and PP (pancreatic polypeptide) cells.
Cell Hormone secreted

b cell Insulin, Amylin

a cell Glucagon

d cells Somatostatin

PP cells Pancreatic polypeptide (vasoactive intestinal peptide, VIP)

Diabetes Mellitus

Diabetes mellitus is a group of metabolic disorders having the feature of


hyperglycemia which results from either defect in insulin secretion, insulin
action, or both. The diagnosis of diabetes is established by elevation of
plasma glucose by any one of three criteria:

• A random plasma glucose concentration of 200 mg/dL or higher, with classical


signs and symptoms
• A fasting glucose concentration of 126 mg/dL or higher on more than one
occasion, or
• An abnormal oral glucose tolerance test (OGTT), in which the glucose
concentration is 200 mg/dL or higher 2 hours after a standard carbohydrate
load (75 gm of glucose).
• A level of glycated hemoglobin (HbA1c) > 6.5 g/dL (accepted as an additional
criteria for the diagnosis of DM by American Diabetic Association.

Apart from over diabetics, the following types of individuals are there:
• Euglycemic individuals: serum fasting glucose values less than 110
mg/dL, or less than 140 mg/dL following an OGTT
• Impaired glucose tolerance: serum fasting glucose greater than 110 but
less than 126 mg/dL, or OGTT values of greater than 140 but less than
200 mg/dL. It is associated with increased risk of progressing to
diabetes.

The insulin gene is expressed in the β cells of the pancreatic islets.


Preproinsulin synthesized in the rough endoplasmic reticulum is delivered
to the Golgi apparatus where it is converted to insulin and a cleavage
peptide, C-peptide.

The vast majority of cases of diabetes fall into one of two broad
classes:

• Type 1 diabetes: it is characterized by an absolute deficiency of insulin


secretion caused by pancreatic β-cell destruction, usually resulting from an
autoimmune attack.

• Type 2 diabetes: it is caused by a “relative insulin deficiency” due to


combination of peripheral resistance to insulin action and an inadequate
compensatory response of insulin secretion by the pancreatic β cells.

PATHOGENESIS OF TYPE 1 DIABETES MELLITUS


The following are the risk factors for the development of type 1 DM:

1. Genetic factors: these could affect


HLA genes (commonest locus being affected is on chromosome
6p21(HLA D) like HLA DR3/DR4 with DQ8 haplotype
The non HLA genes like that for insulin or polymorphism in CD25 (normally
regulated the function of T cells)
2. Environmental factors: viral infections like coxsackie B, mumps,
rubella or cytomegalovirus.

The failure of self tolerance in T cells is the main defect in type 1 DM.
The autoreactive Tcells (TH1 cells and CD8+ cytotoxic T cells) get
activated and cause b cell injury resulting in the reduction of β cell mass.
Autoantibodies against a variety of b-cell antigens, including insulin, islet
cell autoantigen 512 and glutamic acid decarboxylase are also found in the
patients.

PATHOGENESIS OF TYPE 2 DIABETES MELLITUS


The disease is characterized by the following metabolic defects:
• Insulin resistance: it is defined as resistance to the effects of insulin
on glucose uptake, metabolism, or storage. It is a characteristic feature
of most individuals with type 2 diabetes.
• β-cell dysfunction: inadequate insulin secretion in the presence of
insulin resistance and hyperglycemia
There is no autoimmune basis of type 2 DM. The insulin resistance is
being contributed maximally by the loss of sensitivity in the
hepatocytes.
*An increase in the number of and size of islets is characteristic of non diabetic infants of
diabetic mothers.

MONOGENIC FORMS OF DIABETES


The monogenic forms of diabetes can be due to the following causes:

Primary defect in β-cell function Defect in insulin-insulin receptor


signaling

Autosomal-dominant inheritance with


high penetrance
• Type A insulin resistance (severe insulin
resistance + hyperinsulinemia + DM)
• Early onset (usually before age 25)
• Lipoatrophic diabetes (insulin resistance
Absence of obesity
+ hypertriglyceridemia + DM +
• Lack of islet cell autoantibodies acanthosis nigricans + hepatic
steatosis)
Pathogenesis of Complications of DM

CLINICAL FEATURES OF DM

Recent information
A family of proteins called sirtuins, identified to be involved in aging are now implicated
in diabetes. Sirt-1 improves glucose tolerance, enhance β cell insulin secretion, and
increase production of adiponectin.

Acute Complications of DM
1. Type 1 DM
Diabetic ketoacidosis is an important complication seen in type 1
diabetics. It is usually precipitated by inadequate insulin therapy,
intercurrent infection, emotional stress and excessive alcohol intake.

The clinical feature of these patients would be nausea, vomiting,


respiratory difficulties, ‘fruity’ breath odour of acetone, and signs of
dehydration (dry skin and poor skin turgor) and altered consciousness
to coma.
2. Type 2 DM
These patients are usually older (> 40 years) and frequently obese. It may
present with polyuria and polydipsia but mostly it is diagnosed after
routine blood or urine testing. A complication that is seen in these
patients is hyperosmolar nonketotic coma due to the severe
dehydration resulting from sustained osmotic diuresis in patients who
do not drink enough water to compensate for urinary losses from
chronic hyperglycemia. It is usually seen in elderly person who are
unable to maintain adequate water intake.
CHRONIC COMPLICATIONS OF DM
The long-standing diabetes may involve:
• Both large- and medium-sized muscular arteries (macrovascular
disease)
• Capillary dysfunction in target organs (microvascular disease): diabetic
retinopathy, nephropathy, and neuropathy.

MACROVASCULAR DISEASE
It includes increased cardiovascular complications like MI, stroke and
gangrene. The hallmark of diabetic macrovascular disease is accelerated
atherosclerosis affecting the aorta and large and medium-sized arteries. It
is having earlier onset and greater severity. Advanced vascular disease
can lead to gangrene of the lower extremities. The LDL cholesterol is kept
under 100 mg/dL in these patients usually with statins. The vascular lesion
in diabetics is Hyaline arteriolosclerosis (amorphous, hyaline thickening
of the wall of the arterioles causing narrowing of the lumen).

Renal atherosclerosis and arteriolosclerosis is due to macrovascular


disease in diabetics.

A characteristic feature of renal involvement in diabetics is Hyaline


arteriolosclerosis affecting both the afferent as well as the efferent arterioles.

MICROVASCULAR DISEASE
The most consistent morphologic feature of diabetic microangiopathy is
diffuse thickening of basement membranes. However, the affected
vessels (diabetic capillaries) are having increased permeability to plasma
proteins. The microangiopathy is responsible for the development of
diabetic nephropathy, retinopathy, and some forms of neuropathy.
1. Diabetic nephropathy
The most important glomerular lesions are capillary basement membrane
thickening; diffuse increase in mesangial matrix, and nodular
glomerulosclerosis (PAS positive nodules called Kimmelsteil Wilson
lesion). These patients also have increased risk of papillary necrosis.
Clinical features include microalbuminuria (urinary excretion of 30-
300 mg/dayQ of albumin). In uncontrolled diabetes, there is presence of
glucosuria resulting in glycogen accumulation in PCT cells (called as
Armani Ebstein cells). Patients with microalbuminuria are managed
with ACE inhibitors.
(See diabetic nephropathy for details in the chapter on kidney).

2. Diabetic retinopathy
The ocular involvement may present as retinopathy, cataract formation, or
glaucoma. Retinopathy is the most common pattern and can be of the
following types: nonproliferative (background) retinopathy and
proliferative retinopathy.
Nonproliferative retinopathy includes intraretinal or pre-retinal
hemorrhages, retinal exudates, microaneurysms (saccular dilations of
retinal choroidal capillaries), venous dilations, edema, and, most
importantly, thickening of the retinal capillaries (microangiopathy). The
retinal exudates can be either “soft” (microinfarcts) or “hard” (deposits
of plasma proteins and lipids).
Proliferative retinopathy includes the process of neovascularization and
fibrosis. Macular involvement can cause blindness whereas vitreous
hemorrhages can result from retinal detachment. It is managed with
laser photocoagulation.
3. Diabetic Neuropathy
DM can affect both the central and peripheral nervous systems. The most
frequent pattern of involvement is a peripheral, symmetric
neuropathy of the lower extremities that affects both motor and
sensory function. It can also manifest as autonomic neuropathy (can
produce disturbances in bowel and bladder function) and diabetic
mononeuropathy (can manifest as sudden foot drop, wrist drop, or
isolated cranial nerve palsies). The neurological changes may be due
to microangiopathy, increased permeability of the capillaries supplying
the nerves and direct axonal damage due to alterations in sorbitol
metabolism. The delayed gastric emptying is called diabetic
gastroparesis and is managed with metoclopramide or erythromycin.

Treatment of DM is done with insulin and/or anti-hyperglycemic agents. The latter


include Sulfonylureas; (Glipizide, Glibenclamide), Biguanides (metformin), Meglitinides
(Repaglinide), Glucosidase inhibitor (Acarbose) and DPP-4 inhibitors (Vildagliptin).

INSULINOMA

β-cell tumors (insulinomas) are the most common of pancreatic endocrine


neoplasms. These benign tumors may be responsible for the elaboration of
sufficient insulin to induce clinically significant hypoglycemia.
There is a characteristic clinical triad resulting from these pancreatic
lesions:

1. Attacks of hypoglycemia occur with blood glucose levels below 50 mg/dl


2. The attacks consist principally of such central nervous system manifestations
as confusion, stupor, and loss of consciousness
3. The attacks are precipitated by fasting or exercise and are promptly relieved
by feeding or parenteral administration of glucose.
Hyperinsulinism may also be caused by diffuse hyperplasia of the
islets which is usually seen in neonates and infants.

The critical laboratory findings in insulinomas are high circulating


levels of insulin and a high insulin-glucose ratio. Surgical removal of the
tumor is usually followed by prompt reversal of the hypoglycemia.

THYROID GLAND

It is a gland (weighing 15-20 g) responsible for the secretion of the thyroid


hormones (T3 and T4) and calcitonin. Thyroid hormones are required for the
development of brain and maintenance of basal metabolic rate whereas
calcitonin is involved in calcium homeostasis. The two types of disorders
associated with this gland are hyperthyroidism and hypothyroidism.

HYPERTHYROIDISM

It is a state of hyperfunctioning of the thyroid gland characterized by


elevated levels of free T3 and T4 and associated with increased sympathetic
activity. It should be differentiated from thyrotoxicosis which is a
hypermetabolic state due to elevated levels of free T3 and T4 (so,
thyrotoxicosis includes hyperthyroidism as well as other causes). The
causes for this condition include
1. Diffuse toxic hyperplasia (Graves’ disease) (Accounts for 85% of
cases)
2. Toxic multinodular goiter
3. Toxic adenoma
4. Uncommon causes:
– Acute or subacute thyroiditis
– Hyperfunctioning thyroid carcinoma
– TSH secreting pituitary adenoma
– Struma ovarii
– Iatrogenic hyperthyroidism
– Thyrotoxicosis factitia

Clinical features: The salient features include tachycardia,


palpitations, diaphoresis (increased sweating), heat intolerance, tremors,
diarrhea and weight loss despite a good appetite.

The diagnosis is made using serum TSH. It is the most useful


screening test as its level may be altered in patients with even subclinical
hyperthyroidism. In primary hyperthyroidism, serum TSH is low and free
T4 is increased whereas in secondary (due to increased TSH secretion
from the pituitary) and tertiary (due to increased thyrotropin releasing
hormone or TRH secretion from the hypothalamus) hyperthyroidism,
serum TSH is high.

HYPOTHYROIDISM

It is caused due to decreased secretion of the thyroid hormones either due


to a primary defect in the thyroid (most commonQ) or a secondary (TSH
deficiency) or rarely a tertiary (TRH deficiency) cause. This can result in
cretinism in children and myxedema (or Gull disease) in adults. The clinical
features of the disease include lethargy, sensitivity to cold, reduced cardiac
output, constipation, myxedema [due to accumulation of
glycoaminoglycans, proteoglycans and water resulting in deep voice,
macroglossia (enlarged tongue) and non- pitting edema of hands and feet]
and menorrhagia (increased menstrual blood loss).
The diagnosis is made using serum TSH. It is the most useful
screening test. Serum TSH is elevated in primary hypothyroidism and it is
reduced in secondary and tertiary hypothyroidism.

THYROIDITIS

It is defined as the inflammation of the thyroid gland which may be


associated with illness and severe thyroid pain (as in infectious thyroiditis
or subacute granulomatous thyroiditis) or can be painless (subacute
lymphocytic thyroiditis or Reidel thyroiditis). The important types of
thyroiditis include:

Hashimoto Thyroiditis (Chronic Lymphocytic Thyroiditis)

It is a chronic inflammation with lymphocytic infiltration of the thyroid gland


(the latter responsible for the term ‘struma lymphomatosa’). It is more
commonly seen in females (F: M ratio is 10:1) of the age group of 45-65
years. This condition is associated with HLA-DR5, HLA-DR3 and
chromosomal defects like Turner and Down syndrome. Increased
susceptibility to Hashimoto’s thyroiditis has been associated with
polymorphisms of cytotoxic T lymphocyte associated antigen-4 (CTLA4)
and protein tyrosine phosphatase-22 (PTPN 22).

Pathogenesis: There is replacement of the thyroid cells with


lymphocytic infiltration and fibrosis. There is presence of antithyroid
antibodies (anti-TSH receptor antibodies, anti-thyroglobulin and anti-thyroid
peroxidase antibodies) in the serum of the affected patients.
Morphology: The thyroid gland is diffusely enlarged with intact capsule. There is
presence of well developed germinal centers and extensive infiltration of parenchyma by
mononuclear inflammatory cells like lymphocytes and plasma cells. The thyroid follicles
are atrophic and lined by epithelial cells having abundant eosinophilic and granular
cytoplasm called Hurthle cells.Q
Clinical features: It is characterized by the presence of painless enlargement of the
thyroid gland and a gradual loss of thyroid function (though initially, thyroid follicular
disruption may cause transient hyperthyroidism). The disorder is associated with
autoimmune diseases (like SLE, Sjögren syndrome, myasthenia gravis) and there is
increased risk of development of B-cell non-Hodgkin lymphoma.

Fig. 1: Hashimoto thyroiditis showing Hurthle cell (H).

Subacute Lymphocytic Thyroiditis (or Silent/Painless Thyroiditis or


Postpartum Thyroiditis)

It is a self limitingQ episode of thyrotoxicosis seen commonly in middle


aged females especially in postpartum period. It is associated with HLA
DR-3 and HLA DR-5 and is autoimmune in etiology. Painless and post-
partum thyroiditis are variants of Hashimoto’s thyroiditis.

Morphology: The thyroid gland has lymphocytic infiltration with


hyperplastic germinal centers and patchy collapse of thyroid follicles.
Clinical features are painless enlargement of the thyroid and transient
hyperthyroidism (lasting about 2-8 weeks). Investigations reveal elevated
levels of T3 and T4 and reduced TSH.
Subacute Thyroiditis (Granulomatous Thyroiditis or De Quervain
Thyroiditis)
It is a disorder seen commonly in females (Female: Male ratio is 3 to 5:1)
of the age group 30-50 years. It is more commonly seen in summer, is
preceded by a viral infection (caused by coxsackie virus, mumps, measles,
adenovirus etc.) and is associated with HLA-B5.
Pathogenesis: It results due to virus induced host tissue damage or direct
viral damage.

Morphology: The thyroid gland is diffusely enlarged with intact capsule. There is
presence of patchy changes. In the initial stages, there is active inflammation
characterized by disruption of follicles by neutrophils (forming micro abscess),
lymphocytes, histiocytes, plasma cells and multi-nucleated giant cells which is followed
by fibrosis.
Clinical features are pain in neck, sore throat, fever, fatigue, anorexia, myalgia,
enlarged thyroid and the presence of transient hyperthyroidism which usually
diminishes in 2-6 weeks. It may be followed by asymptomatic hypothyroidism but
recovery is seen in most of the patients. Almost all patients have high T3 and T4 and low
TSH initially which recovers in 6-8 weeks after the disease completes the course.
Fig. 2: Granulomatous thyroiditis having giant cells (G), lymphocyte (L) and colloid filled
follicles (F).

Reidel’s Thyroiditis (or Fibrous Thyroiditis/Invasive Thyroiditis)

It is an idiopathic rare disorder characterised by the destruction of the


thyroid gland by dense fibrosis. Fibrosis of the surrounding structures like
trachea and esophagus can also occur. It is more commonly seen in
females of middle age and is associated with retroperitoneal and
mediastinal fibrosis.

GRAVES’ DISEASE
It is the commonest causeQ of endogenous hyperthyroidism characterized
by the triad of hyperthyroidism due to hyperfunctional diffuse enlargement
of gland, infiltrative ophthalmopathy and localized, infiltrative dermopathy
(also called as pretibial myxedema). The disorder is more common in
females of the age group of 20-40 years. It is associated with
polymorphisms in HLA B8, HLA DR3, CTLA4 and PTPN-22.

Pathogenesis: It is an autoimmune disease most commonly due to


formation of antibodies to TSH receptors (called TSI or LATS meaning
Thyroid Stimulating Immunoglobulin and Long Acting Thyroid Stimulator
respectively). The other antibodies found in this condition include TGI
(Thyroid Growth stimulating Immunoglobulin) and TBII (TSH Binding
Inhibitor Immunoglobulin), the latter sometimes responsible for paradoxical
hypothyroidism seen in some of these patients. The anti-TSH antibodies
stimulate the TSH receptor in this condition in contrast to Hashimoto’s
thyroiditis in which the antibodies inhibit the receptor.

Morphology: The thyroid gland is symmetrically enlarged with diffuse


hypertrophy and hyperplasia. The capsule is intact.

Clinical features as described above include hyperthyroidism, ophthalmopathy (due


to increased volume of extraocular muscle and retro-orbital connective tissue as a result
of expression of TSH receptor by orbital fibroblasts) and localized, infiltrative
dermopathy (most commonly in skin overlying shin). Investigations reveal increased
levels of T3 and T4 with reduced TSH levels. There is a diffuse increase in the uptake of
radioactive iodine.

DIFFUSE AND MULTINODULAR GOITER

Goiter is enlargement of the thyroid gland. Both diffuse and multinodular


goiter are caused due to impaired synthesis of thyroid hormones most
commonly due to dietary iodine deficiency. This results in increased
secretion of TSH leading to hypertrophy and hyperplasia of the thyroid
gland. The degree of enlargement is proportional to the level and duration
of thyroid hormone deficiency. Usually, the enlargement takes place to
maintain a euthyroid state but may also be associated with hyperthyroid
state.

Diffuse Non-toxic Goiter (Colloid Goiter or Simple Goiter)

In this condition, the thyroid shows no nodules and there are colloid filled
follicles (so, the other name is colloid goiter). It can be endemic (when
>10% of population is affected usually due to low dietary iodine intake) or
sporadic (seen more commonly in females during puberty; usually due to
enzyme defects affecting thyroid hormone synthesis or ingestion of
Goitrogens which are substances interfering with thyroid hormone
synthesis like calcium, cabbage, cauliflower, turnip, cassava, etc.)

Histologically, there can be two stages: initial hyperplastic stage


having diffuse, symmetrically enlarged gland with thyroid follicular
hyperplasia and later, the stage of colloid involution.

MULTINODULAR GOITER (MNG)


This is a condition resulting from recurrent episodes of hyperplasia and
involution resulting in irregular enlargement of thyroid gland. The
differential sensitivity of follicular cells for TSH results in multinodular
goiter. Grossly, there is presence of enlarged multinodular thyroid with
presence of hemorrhage, fibrosis, calcification and cystic change.

Clinical features are due to mass effect (enlarged thyroid causing


compression of esophagus, trachea, etc.) or cosmetic effect.

SOLITARY THYROID NODULE (STN)

It is a clinical entity seen more commonly in females. STN is more likely to


be neoplastic in the presence of certain risk factor (mentioned alongside).
THYROID ADENOMA

These are solitary masses of the thyroid tissue composed of follicular


epithelium and are therefore, called as follicular adenomas. They are
formed due to chronic overactivation of cAMP pathway (due to somatic
mutation of the TSH receptor or the a subunit of Gs receptor). They are
usually asymptomatic and present as ‘cold’ nodules on radio imaging
scans.

Morphologically, these are solitary, spherical lesions having an intact


capsule Usually, the cells form uniform appearing follicles containing colloid
but they can have the following subtypes:
• Follicular or simple colloid
• Microfollicular: Seen in fetal life
• Hurthle cell (oxyphil, oncocytic) adenoma: Cells have eosinophilic,
granular cytoplasm
• Atypical adenoma: Increased variation in cellular and nuclear
morphology
• Clear cell follicular adenoma: Cells have clear cytoplasm.

THYROID CARCINOMAS
It is a cancer seen more commonly in females in early and middle adult life.
The four histological types of thyroid cancers are:
1. Papillary cancer
2. Follicular cancer
3. Medullary cancer
4. Anaplastic cancer
Risk Factors for Thyroid Cancers

• Papillary cancer: It is associated with mutation in either tyrosine kinase receptors


RET or NTRK1 (Neurotrophic Tyrosine Kinase Receptor 1) or BRAF oncogene.
RET is located on chromosome10 and translocation with chromosome 17 causes
formation of a fusion gene ret/PTC (ret/papillary thyroid cancer) which is responsible
for increased tyrosine kinase activity of cells resulting in papillary thyroid cancer.
This cancer is also seen after exposure to ionizing radiation during first two
decades of life.
• Follicular cancer: It is associated with mutation in RAS oncogenes particularly N-
RAS. A specific translocation associated with follicular cancer is t(2;3) resulting in
PAX8-PPARγ1fusion. PPAR is peroxisome proliferator-activated receptor required
for terminal differentiation of the cell whereas PAX8 is a homeobox gene required for
thyroid development.
• Medullary cancer: It is the only thyroid cancer to arise from parafollicular ‘C’ cells.
It is associated with mutation in RET proto-oncogene resulting in constitutional
activation of the receptor.

• Remember that ret/PPTC is NOT seen in medullary carcinoma of thyroid.

• Anaplastic cancer: It is associated with mutation in the p53 tumor suppressor gene.
Fig. 3: Papillary thyroid cancer with characteristic nuclear features Inset: clear nucleus,
nuclear grooving and intranuclear inclusions.

SALIENT FEATURES OF THYROID CANCERS

PAPILLARY CARCINOMA

• It is the commonest type of thyroid cancerQ


• Seen in 20-40 years old age group
• Spread is by lymphatic routeQ
• Carries excellent prognosisQ
• Microscopically there is presence of papillae with fibrovascular
stalk, calcified structures called Psammoma bodiesQ and cancer cells
have diagnostic nuclear features like presence of fine chromatin
leading to ‘ground glass’ or ‘Orphan Annie eye’ nucleiQ, intranuclear
inclusions (called ‘pseudoinclusions’) or intranuclear longitudinal
grooves.
• The variants include encapsulated variant (good prognosis), follicular
variant (poor prognosis) and tall cell variant (poorest prognosis).
FOLLICULAR CARCINOMA

• It is the 2nd most common form of thyroid cancer


• Seen in women of older age (40-50 yrs.)
• Vascular invasion is common (less lymphatic spread) to bone, lung,
liver etc.
• Microscopically, there is presence of cells forming small follicles
having colloid with NO Psammoma bodies. Uncommonly, cells have
abundant , eosinophilic cytoplasm called as Hurthle cellsQ
• Differentiation from follicular adenoma is based on the presence of
capsular invasion preferably and vascular invasion Q (capsular vessel
invasion).

MEDULLARY CARCINOMA

• Arises from parafollicular cells/C cells and secretes calcitoninQ


• Sporadic in 80% of cases
• Associated with multiple endocrine neoplasia II (MEN) syndromesQ
• Only thyroid cancer associated with amyloidosisQ
• Unilateral in sporadic cases and bilateral and multicentric in familial
cases
• Microscopically, there is presence of polygonal, spindle cells in
amyloid stroma. Familial cancers characteristically show the presence
of multicentric C-cell hyperplasia.

Fig. 4: Medullary thyroid cancer with fibrils of pink and waxy amyloid (arrow).

ANAPLASTIC CARCINOMA

• Undifferentiated thyroid cancer


• Have the worst prognosisQ
• Seen mostly in elderly female patients
• 50% patients give a history of presence of multinodular goiterQ
• Microscopically, there is presence of highly anaplastic cells which can
either be giant cells, spindle cells, mixed giant and spindle cells or
small cells.

Quick revision of frequently asked MCQ’s from thyroid cancers:


• Most common thyroid carcinoma -PapillaryQ

• Least common thyroid carcinoma -AnaplasticQ

• Least malignant -PapillaryQ

• Most malignant -AnaplasticQ

• Most common cancer after radiation -PapillaryQ

• Cancer developing in Hashimoto’s thyroiditis -LymphomaQ

• Thyroid cancer developing in long standing multinodular goiter -Follicular,


Anaplastic (rare)

• Type of thyroid cancer in MEN syndrome -MedullaryQ

• Thyroid cancer associated with amyloidosis -MedullaryQ

• Psammoma bodies seen in -PapillaryQ

• Orphan-Annie Eyed Nuclei seen in -PapillaryQ

• Thyroid Ca associated with dystrophic calcification -PapillaryQ

• Carcinoma derived from ‘C’ cell of thyroid -MedullaryQ

• Carcinoma developing in thyroglossal tract -PapillaryQ

PARATHYROID GLAND
These are four glands situated near the thyroid gland and are composed of
chief cells (containing PTH granules) and oxyphil cells (containing
glycogen).
PTH secretion is responsible for elevating serum calcium level and
increasing phosphate excretion in the urine. Malignancy is the most
common cause of clinically apparent hypercalcemia, while primary
hyperparathyroidism is the commonest cause of asymptomatic
hypercalcemia. Increased calcium levels associated with malignancies can
be because of osteolytic metastasis and secretion of a PTH related peptide
(PTHrP).

HYPERPARATHYROIDISM
Hyperparathyroidism can be primary (due to autonomous, spontaneous
overproduction of PTH) or secondary. Rarely it can be tertiary.

Primary Hyperparathyroidism; 1° HPTH

It is the most important cause of asymptomatic hypercalcemia and can be


due to a parathyroid adenoma, primary hyperplasia or parathyroid
malignancy. Hyperparathyroidism can be familial or sporadic. The
important molecular defects associated with sporadic hyperparathyroidism
include:

1. PRAD 1 proto-oncogene on chromosome 11 causes over-expression


of cyclin D1 resulting in proliferation of the parathyroid cells.
2. MEN I suppressor gene on 11 q 13.
The genetic syndromes associated with familial
hyperparathyroidism include

1. Multiple endocrine neoplasia I and II (MEN-I and II), the genes for
which are located on chromosome 11q and 10 q respectively.
2. Familial hypocalciuric hypercalcemia (FHH) gene results in reduced
sensitivity to extracellular calcium and is responsible for increased
secretion of PTH.

Morphology

Adenoma: There is presence of solitary nodule with shrunken glands outside the
adenoma.
Primary hyperplasia: There is asymmetric involvement of all four glands with the
presence of chief cells.
Parathyroid carcinoma: Involvement of a single gland.
Clinical features: Usually asymptomatic, the only indicator for
diagnosis is increased serum calcium and PTH. Symptomatic patients may
have nephrolithiasis (urinary tract stones) or nephrocalcinosis (calcification
of renal interstitium and tubules), osteoporosis, osteitis fibrosa cystica
(bone marrow having foci of fibrosis, hemorrhage and cyst formation),
metastatic calcification (in blood vessels, stomach and myocardium) and
neurological changes like depression, lethargy, etc.

SECONDARY HYPERPARATHYROIDISM
It is seen in renal failure (most common cause), vitamin D insufficiency,
steatorrhea and nutritional deficiency. The hypocalcemia due to any of
these causes stimulates the secretion of PTH.

Morphology shows the presence of hyperplastic parathyroid glands.


Clinical features are similar to primary hyperparathyroidism. There is
also presence of calciphylaxis (vascular calcification causing organ
ischemia). Investigations reveal reduced serum calcium and increased
PTH levels.

TERTIARY HYPERPARATHYROIDISM
Autonomous excessive parathyroid activity even when serum calcium is
increased is called as tertiary hyperparathyroidism which is usually
managed by parathyroidectomy.

HYPOPARATHYROIDISM

It is seen due to surgical removal (commonest causeQ), congenital


absence (as in DiGeorge syndrome; failure of development of 3rd and 4th
pharyngeal pouch leading to absence of thyroid and parathyroid glands) or
is idiopathic.

Clinical features are due to hypocalcemia and the hallmark is tetany


characterised by neuromuscular hyperexcitability, cataract, hypotension, QT
prolongation on ECG, tingling in circumoral region and hands and feet.
Investigations demonstrate the presence of Chvostek sign (percussion of facial
nerve over ear causes contraction of facial muscles and upper lip) and Trousseau
sign (inflation of blood pressure cuff more than the systolic blood pressure for
around 3 minutes causes flexion at metacarpophalangeal joint with extension at
interphalangeal joint). Diagnosis is made by low serum calcium levels.

PITUITARY GLAND

It is a gland weighing 0.5g, present in sella turcica. It has two distinct lobes;
anterior lobe and posterior lobe (stores oxytocin and antidiuretic hormone
or vasopressin).

HYPERPITUITARISM

It can be caused due to adenoma arising from the anterior lobe


(commonest cause), hyperplasia and carcinoma.The majority of the
adenomas are monoclonal in origin or can be associated with MEN I.
Histologically; the adenomas are composed of polygonal cells with little
reticulin or connective tissue. The common pituitary tumors include the
following:

• Prolactinoma: It is the most common pituitary tumor. Small


microadenomas secrete large amount of prolactin responsible for the
clinical features of amenorrhea, galactorrhea and infertility. Since men
will obviously not have amenorrhea and females are detected early
due to menstrual problems, so, microadenomas are commoner in
females.
Any mass in suprasellar compartment may disturb the normal inhibitory influence
of the hypothalamus on prolactin secretion resulting in hyperprolactinemia. This is
called stalk effect.

• Growth hormone adenoma: It is the second most common type of


pituitary adenoma. Almost 40% of the patients have persistent GH
activity resulting in hypersecretion of insulin like growth factor I (or IGF-
I or somatomedin C) causing gigantism in children and acromegaly in
adults. Gigantism is characterized by features of tall stature and long
extremities whereas acromegaly has features of prominent jaw
(prognathism), flat, broad forehead, enlarged hands and feet and
enlargement of internal organs like heart, spleen, kidney etc.
• Other pituitary tumors include corticotroph cell adenoma producing
ACTH (causing Cushing disease), thyrotrope adenoma secreting TSH
(causing hyperthyroidism), gonadotrope adenoma secreting FSH and
LH.

HYPOPITUITARISM

It is usually seen when more than 75% of parenchyma is lost. GH and


gonadotropins (FSH, LH) are typically lost early as compared to other
hormones. The causes of hypopituitarism include:

Compression of the normal pituitary tissue by tumors or cysts


Pituitary surgery or radiation exposure
Pituitary apoplexy (acute hemorrhagic infarction of a pre-existing pituitary
adenoma)
Ischemic necrosis
Sheehan syndrome (postpartum pituitary necrosis due to obstetric hemorrhage
or shock)
Empty sella syndrome.

Clinical features depend upon the hormone whose function is lost for
example, there can be growth failure (due to GH deficiency), loss of libido,
amenorrhea, infertility (due to gonadotropin deficiency), hypothyroidism
and hypoadrenalism. Loss of melanocyte stimulating hormone (MSH) may
cause pallor of the skin.

POSTERIOR PITUITARY SYNDROMES

• Diabetes insipidus: It is caused due to deficiency of ADH or


vasopressin resulting in polyuria, polydipsia, hypernatremia and
hyperosmolality (due to excessive renal loss of free water) and
dehydration.
• Syndrome of inappropriate ADH secretion (SIADH): Excessive
production of ADH can cause oliguria, retention of water, hyponatremia
and cerebral edema. The causes of SIADH include ectopic ADH
secretion by small cell lung cancer (commonest), injury to
hypothalamus or pituitary or both by head trauma and drugs (like
vincristine).

ADRENAL CORTEX
Adrenal gland is divided into adrenal cortex and adrenal medulla. The
cortex is further subdivided into the following three parts from outside to
inside responsible for the secretion of the hormones mentioned in front of
them.
• Zona glomerulosa - Mineralocorticoids
• Zona fasciculata - Glucocorticoids
• Zona reticularis – Sex steroids

So, Hyperadrenalism can have 3 distinctive patterns:


1. Cushing syndrome:Excess of glucocorticoids
2. Hyperaldosteronism: Excess of mineralocorticoids
3. Adrenogenital syndrome: Excess of sex steroids (androgens)

CUSHING SYNDROME

It has four important causes


1. Primary hypersecretion due to increased ACTH (also called Cushing
disease), seen in women of 20–30 years due to an ACTH producing
microadenoma.
2. Adrenal over-secretion due to adenomas or carcinomas (adrenal
Cushing syndrome): There is no effect of ACTH, so, also known as
ACTH independent Cushing syndrome.
3. Secretion of Ectopic ACTH: Small cell cancer of the lung, carcinoid
tumors.
4. Administration of exogenous corticosteroids
Morphology:
In adrenal gland, there can be presence of:
1. Cortical atrophy- Seen with exogenous glucocorticoids which cause
feedback inhibition of ACTH leading to cortical atrophy except in zona
glomerulosa (it functions independent of ACTH).
2. Diffuse hyperplasia
3. Nodular hyperplasia

Diagnosis
There is an increased 24 hour free cortisol level in the urine with loss of
normal diurnal pattern of cortisol secretion. For differentiating between the
causes of Cushing syndrome, we use dexamethasone suppression test.
(See Review of Pharmacology Chapter-6 by the same authors for details)

ADRENOGENITAL SYNDROME

It is an adrenal disorder due to excessive production of androgens resulting


in virilization. It can be caused either by an adrenocortical carcinoma or
more commonly congenital adrenal hyperplasia (CAH). CAH represents a
group of autosomal-recessive inherited metabolic errors in which there is
deficiency of the enzyme/s necessary for synthesis of cortisol.
Steroidogenesis channeled into other pathways lead to increased
production of androgens, which accounts for virilization. Cortisol deficiency
induced increased secretion of ACTH results in adrenal hyperplasia. CAH
can manifest as the following three syndromes:
1. Salt wasting syndrome – Total absence of 21-a hydroxylase.
2. Simple virilising adrenogenitalism – Presents as genital ambiguity due
to partial deficiency of 21 a hydroxylase.
3. Non classic adrenogenitalism – Asymptomatic or may manifest as
hirsutism.
Morphology

Adrenals are hyperplastic bilaterally with nodular cortex that is brown (as
there is absence of lipid).

CLINICAL FEATURES
In 21-a hydroxylase deficiency, excessive androgenic activity causes signs
of masculinization in females which may range from clitoral hypertrophy
and pseudohermaphroditism in infants, to oligomenorrhea, hirsutism, and
acne in post pubertal females. In males, androgen excess is associated
with enlargement of the external genitalia and precocious puberty in
prepubertal patients and oligospermia in older males.

HYPERALDOSTERONISM

The condition is characterized by elevated aldosterone levels leading to


retention of sodium and excretion of potassium and hydrogen ions.

Causes Causes
• Adrenocortical adenoma (Conn syndrome): - • Decreased renal perfusion
Commonest causeQ • Hypovolemia and edema
• Primary adrenocortical hyperplasia → Due to (CHF and cirrhosis)
overactivity of aldosterone synthase gene, • Pregnancy
CYP11B2
• Glucocorticoid remediable hyper aldosteronism due
to fusion between CYP11B1 (11b hydroxylase)
and CYP11B2 (Aldosterone synthetase) genes

Morphology: Adrenal adenomas are usually unilateral (more common


on the left as compared to right). The aldosterone producing adenoma has
the presence of eosinophilic laminated cytoplasmic inclusions called as
‘spironolactone bodies’ seen after treatment with spironolactone.
Clinical features include hypokalemia induced polyuria, polydipsia
and muscle weakness. There may be associated metabolic alkalosis
because of excessive aldosterone secretion. Edema is uncommon in
primary hyperaldosteronism because of ‘escape effect’.

ADRENAL INSUFFICIENCY

It can be due to primary adrenocortical insufficiency (primary


hypoadrenalism) or ACTH deficiency induced reduced adrenal stimulation
(secondary hypoadrenalism).
1. Primary acute adrenocortical insufficiency: It can be seen after
stress, sudden withdrawal of steroids or massive adrenal hemorrhage.
If the acute adrenal insufficiency is associated with bilateral
hemorrhagic infarction of the adrenal glands associated with a
Neisseria infection (septicemia) in a child, it can result in disseminated
intravascular coagulation and rapidly developing hypotension and
shock in the patient which is called Waterhouse-Friedrichsen
Syndrome. The hemorrhage in this condition usually begins in the
medulla and then involves the cortex.
2. Primary chronic adrenocortical insufficiency (Addison disease): It
is a slow and progressive disease resulting from adrenocortical
hypofunction. Idiopathic atrophy (by autoimmune mechanism) is the
commonest cause of adrenal destruction. Autoimmune adrenalitis is
associated with Autoimmune Polyendocrine Syndrome (APS) 1 and 2.
The loss of adrenal cortex can also be due to tuberculosis, sarcoidosis,
AIDS, hemorrhage, trauma and metastatic involvement.
Clinical features include initial manifestations of progressive weakness
and easy fatigability, gastrointestinal disturbances like anorexia,
nausea, vomiting, weight loss and diarrhea. In patients with primary
adrenal disease, increased circulating levels of ACTH precursor
hormone stimulate melanocytes, with resultant hyperpigmentation of
the skin particularly of sun-exposed areas and at pressure points, such
as neck, elbows, knees, and knuckles.
3. Secondary adrenocortical insufficiency: It occurs secondary to any
disorder of the hypothalamus and pituitary, such as metastatic cancer,
infection, infarction, or irradiation. Further, in adrenal insufficiency
secondary to pituitary malfunction, marked hyponatremia and
hyperkalemia are not seen.
Fig. 5: Zellbalen (solid tumor nests; N) in Pheochromocytoma.

ADRENAL MEDULLA

The adrenal medulla is composed of neuroendocrine cells called


chromaffin cells and their supporting cells called sustentacular cells. The
organ is responsible for the secretion of epinephrine and nor-epinephrine
and is controlled by the autonomic nervous system.

PHEOCHROMOCYTOMA

It is a tumor of the adrenal medulla which produces catecholamines. The


patients usually have severe headache, anxiety, increased sweating,
tachycardia, palpitations and hypertensive episodes. The tumor is
associated with a ‘rule of 10’s’ consisting of
• 10% are bilateralQ
• 10% are extra-adrenalQ
• 10% are malignantQ
• 10% occur in childrenQ
• 10% are not associated with hypertensionQ

The tumor morphology shows the presence of small or large tumors that have
yellow tan color that turns brown on incubation. There is presence of nests of chief
or chromaffin cells with sustentacular cells (called zellballenQ) with abundant
cytoplasm which contains catecholamine granules. The nuclei of the cells have
‘salt and pepper’ appearance of the chromatin. The immunomarkers for this
tumor include chromogranin and synaptophysin in chief cells and S-100 for
sustentacular cells.

The definitive

diagnosis of malignancy is based exclusively on the presence of


metastasis.Q
The investigations reveal the presence of elevated urinary excretion of
free catecholamines and their metabolites such as vanillylmandelic acid
(VMA) and metanephrines.
PANCREAS

1. Amylin is secreted by which group of cells of pancreas?


(All India 2012)
(a) Alpha cells
(b) Beta cells
(c) D cells
(d) PP cells (Pancreatic polypeptide)
2. Diabetes is diagnosed by which of the following criteria?
(AIIMS Nov 2011)
(a) The level of fasting glucose is ≥ 100 mg/dL and that of
postprandial glucose is ≥ 140 mg/dL
(b) The level of fasting glucose is > 125 mg/dL and that of post
prandial glucose is > 199 mg/dL
(c) The level of plasma insulin is ≥ 6 IU/dL
(d) The HbA1c level is ≥ 5.5%
3. All statements are true about Nesidioblastosis except?
(a) Hypoglycemic episodes are seen
(AI 2011)
(b) Occurs more commonly in adults than in children
(c) Histopathology shows hyperplasia of Islet cells
(d) Diazoxide is used in treatment
4. Insulin increases glucose entry into skeletal muscle, adipose
tissues and liver cells by:
(DPG 2011)
(a) Increasing the number of glucose transporter GLUT2 in all these
tissues
(b) Increasing the number of GLUT4 in muscle and adipose tissue
and glucokinase in liver cells
(c) Increasing the number of GLUT3 in skeletal muscle and adipose
tissues and GLUT4 in liver cells
(d) Increasing the number of GLUT1 in muscle, GLUT3 in adipose
tissues and GLUT4 in liver cells
5. The term fetal adenoma is used for:
(UP 2003)
(a) Hepatoma liver
(b) Fibroadenoma breast
(c) Follicular adenoma of thyroid
(d) Craniopharyngioma
6. Which of the following is used to measure control of blood sugar
in diabetes mellitus?
(UP 2005)
(a) HbA
(b) HbS
(c) HbA2
(d) HbA 1C
7. Two diabetic patients are seen by an endocrinologist, Dr. Saket.
The first patient is a 16-year-old boy Raju who 2 years previously
had presented with polyuria and polydipsia. The second patient
is a 65-year-old woman Antara whose diabetes was identified by
the presence of hyperglycemia on a routine blood glucose
screen 10 years previously. Compared to Antara, Raju is more
likely to
(a) Not have the HLA-DR3 or HLA-DR4 allele
(b) Become euglycemic with oral hypoglycemic agents
(c) Develop ketoacidosis
(d) Have relatively high endogenous insulin levels
8. A 62 year-old woman Omvati with advanced, metastatic lung
cancer develops profound fatigue and weakness and alternating
diarrhea and constipation. Physical examination demonstrates
hyperpigmentation of skin, even in areas protected from the sun.
Tumor involvement of which endocrine organ is most strongly
suggested by this patient’s presentation?
(a) Adrenal gland
(b) Endocrine pancreas
(c) Ovaries
(d) Pituitary gland

MOST RECENT QUESTIONS

9. Mauriac’s syndrome is characterized by all except


(a) Diabetes
(b) Obesity
(c) Dwarfism
(d) Cardiomegaly
10. Necrobiosis lipoidica is seen in
(a) Diabetes insipidus
(b) Lyme disease
(c) Diabetes mellitus
(d) Symmonds disease
11. Insulin resistance in liver disease is due to:
(a) Decreased insulin release
(b) Steatosis
(c) Hepatocyte dysfunction
(d) Decreased ‘C’ peptide level
12. According to ADA guidelines, the diagnosis of diabetes is made
when the fasting blood glucose is more than
(a) 126 mg/dl
(b) 100 mg/dl
(c) 140 mg/dl
(d) 200 mg/dl
13. Most common association in MEN I is
(a) Gastrinoma
(b) Insulinoma
(c) Lipoma
(d) Glucagonoma
14. True about insulinoma is:
(a) Alpha cell tumor
(b) Most of them are malignant
(c) C-peptide level is elevated
(d) It is not associated with MEN 1 syndrome.
15. Marker for pancreatic non-functional neuro-endocrine tumor
(PNET) is:
(a) Chromogranin-A
(b) CD100
(c) CEA
(d) PSA
16. Amyloidosis is seen in which type of diabetes mellitus?
(a) Maturity onset DM
(b) Type I DM
(c) Type II DM
(d) All of the above
17. Gene not associated with diabetes mellitus:
(a) PPAR-gamma
(b) KCNJ11
(c) CTLA4
(d) PDGF-R
THYROID, PARATHYROID

18. Which of the following term describes hyperthyroidism following


intake of iodine in patients suffering from endemic goiter?
(All India 2012)
(a) Wolff-Chaikoff effect
(b) Jod-Basedow effect
(c) Graves disease
(d) Hashimoto’s thyroiditis
19. A 17 year old girl who was evaluated for short height was found
to have an enlarged pituitary gland. Her T4 was low and TSH was
increased. Which of the following is the most likely diagnosis?
(a) Pituitary adenoma
(AIIMS Nov 2011)
(b) TSH-secreting pituitary tumor
(c) Thyroid target receptor insensitivity
(d) Primary hypothyroidism
20. All are true about Hashimoto’s thyroiditis except:
(a) Follicular destruction
(AIIMS Nov 2011)
(b) Lymphocytic infiltration
(c) Oncocytic metaplasia
(d) Orphan Annie eye nuclei
21. Hypothyroidism is seen in:
(AI 2011)
(a) Hashimoto’s Thyroiditis
(b) Graves’ disease
(c) Toxic Multinodular Goitre
(d) Struma ovarii
22. All are true about Hashimoto’s thyroiditis except:
(a) Follicular destruction
(AIIMS May 2010)
(b) Lymphocytic infiltration
(c) Oncocytic metaplasia
(d) Orphan Annie eye nuclei
23. MC thyroid cancer is:
(AI 2008)
(a) Papillary carcinoma
(b) Follicular carcinoma
(c) Medullary carcinoma
(d) Anaplastic carcinoma
24. Which of the following gene defect is associated with
development of medullary carcinoma of thyroid?
(a) RET Proto-oncogene
(AI 2004)
(b) FAP gene
(c) Rb gene
(d) BRCA 1 gene
25. Medullary carcinoma of the thyroid is associated with which of
the following syndrome:
(AI 2003)
(a) MEN I
(b) MEN II
(c) Li-Fraumeni syndrome
(d) Hashimoto’s thyroiditis
26. The expression of the following oncogene is associated with a
high incidence of medullary carcinoma of thyroid:
(AIIMS Nov 2005)
(a) p 53
(b) Her 2 neu
(c) RET proto oncogene
(d) Rb gene
27. DeQuervain’s thyroiditis is also known as:
(a) Granulomatous thyroiditis
(Delhi PG-2006)
(b) Struma lymphomatosa
(c) Acute thyroiditis
(d) Hashimoto thyroiditis
28. Hurthle cells are seen in:
(Delhi PG-2005)
(a) Granulomatous thyroid disease
(b) Hashimoto’s thyroiditis
(c) Papillary carcinoma of thyroid
(d) Thyroglossal cyst
29. Which of the following histological type of carcinoma thyroid
most commonly metastasizes to lymph nodes?
(a) Medullary
(Delhi PG-2005)
(b) Anaplastic
(c) Papillary
(d) Follicular
30. Struma ovarii is composed entirely of
(a) Mature thyroid tissue
(Karnataka 2006)
(b) Immature-thyroid tissue
(c) Primary ovarian carcinoid tissue
(d) None of the above
31. Oncocytes are found in all of the following except:
(a) Thyroid
(DNB-2000, 2003, 2006, 2007)
(b) Pancreas
(c) Pituitary
(d) Pineal body
(e) None of the above
32. Hurthle cells are seen in:
(DNB- 2000, 2005)
(a) Hashimoto’s thyroiditis
(b) Granulomatous thyroiditis
(c) Carcinoma of thyroid
(d) Acute thyroiditis
33. Calcitonin is a marker of thyroid:
(UP 2001)
(a) Papillary carcinoma
(b) Medullary carcinoma
(c) Anaplastic carcinoma
(d) Adenocarcinoma
34. In Hashimoto’s thyroiditis, there is infiltration of:
(a) Macrophages
(b) Neutrophils
(UP 2003)
(c) Leukocytes
(d) Eosinophils
35. Myasthenia gravis is associated with:
(UP 2006)
(a) Hypergammaglobulinemia
(b) Thymoma
(c) Squamous cell carcinoma
(d) Hepatic adenoma
36. In MEN II B syndrome includes all except:
(a) Hyperparathyroidism
(UP 99, 2007)
(b) Marfanoid features
(c) Medullary thyroid carcinoma
(d) Pheochromocytoma
37. Plunging goiter is
(RJ 2000)
(a) Solitary nodule
(b) Colloid goiter
(c) Retro-sternal goiter
(d) Medullary ca
38. All are parts of MEN-I except:
(RJ 2006)
(a) Pituitary tumor
(b) Parathyroid tumor
(c) Pancreatic tumor
(d) Medullary carcinoma of thyroid

MOST RECENT QUESTIONS

39. Psammoma bodies are seen in all except:


(a) Papillary carcinoma of thyroid
(b) Papillary adenoma of colon
(c) Meningioma
(d) Papillary cancer of the ovary
40. Papillary carcinoma associated with aggressiveness are all
except:
(a) Follicular variant
(b) Unencapsulated
(c) Tall cell variant
(d) Oxyphilic (Hurthle) cell type
41. Which thyroid carcinoma is of C-Cell origin:
(a) Medullary carcinoma
(b) Follicular carcinoma
(c) Papillary carcinoma
(d) Anaplastic carcinoma
42. All of the following regarding thyroid carcinoma are true except:
(a) Prognosis of follicular carcinoma is worse than papillary
carcinoma
(b) Medullary carcinoma is autosomal recessive
(c) Anaplastic carcinoma causes local invasion early
(d) Medullary and papillary carcinoma both spread by lymphatic
route
43. Hurthle cell tumor is:
(a) Papillary carcinoma
(b) Follicular carcinoma
(c) Medullary carcinoma
(d) Colloid carcinoma
(e) Ionizing radiation in early decades is a major risk factor
44. About papillary carcinoma of thyroid, all are true except:
(a) Prognosis better
(b) Psammoma body present in 50% cases
(c) Early metastasis with poor prognosis
(d) Spreads by the lymphatic route
45. A 51-year-old man Sonu with a history of recurrent calcium-
containing renal stones presents to the emergency room with
excruciating flank pain and blood in the urine. This patient is
likely to have which of the following underlying disorders?
(a) Anemia of chronic disease
(b) Chronic Proteus infection
(c) Hyperparathyroidism
(d) Hyperaldosteronism
46. Fine needle aspiration cytology is not able to detect which of the
following?
(a) Papillary carcinoma
(b) Hashimoto thyroiditis
(c) Follicular cancer
(d) Medullary cancer
47. High calcium intake can lead to:
(a) Osteoporosis
(b) Osteopetrosis
(c) Milk alkali syndrome
(d) Renal failure
48. Most common thyroid cancer after radiation exposure is:
(a) Papillary cancer
(b) Medullary cancer
(c) Follicular cancer
(d) Anaplastic cancer
49. Orphan Annie eye nuclei appearance is characteristic of:
(a) Papillary carcinoma thyroid
(b) Carcinoma pituitary
(c) Paraganglioma
(d) Meningioma
50. The laboratory screening test which suggests normal thyroid
function is
(a) TSH
(b) Free T4
(c) T3
(d) Free T3
51. Medullary carcinoma of the thyroid is associated with which of
the following syndrome:
(a) MEN I
(b) MEN II
(c) Fraumeni syndrome
(d) Hashimoto’s thyroiditis
52. Which thyroid carcinoma has amyloid?
(a) Papillary
(b) Follicular
(c) Medullary
(d) Anaplastic
53. Which is not seen in MEN I:
(a) Parathyroid adenoma
(b) Pancreatic cancer
(c) Prolactinoma
(d) Medullary carcinoma thyroid
54. Werner syndrome is:
(a) MEN I
(b) MEN IIA
(c) MEN IIB
(d) AIP
55. Psammoma bodies are seen in?
(a) Papillary carcinoma thyroid
(b) Medullary carcinoma thyroid
(c) Follicular carcinoma thyroid
(d) Anaplastic carcinoma
56. Lymphatic spread is most commonly seen with which type of
thyroid cancer?
(a) Papillary
(b) Medullary
(c) Follicular
(d) Lymphoma
57. Fine needle aspiration cytology is not enough to diagnose which
of the following?
(AIIMS Nov 2016)
(a) Papillary carcinoma of thyroid
(b) Carcinoma breast
(c) Adenocarcinoma lung
(d) Follicular carcinoma of thyroid
58. Which malignancy develops in long standing goiter?
(a) Papillary carcinoma
(b) Follicular carcinoma
(c) Medullary carcinoma
(d) Anaplastic carcinoma
59. All of these are seen in MEN-1 syndrome except:
(a) Foregut carcinoid
(AIIMS Nov 2016)
(b) Posterior pituitary tumors
(c) Pancreatic neuroendocrine tumors
(d) Parathyroid hyperplasia
60. All of the following are feature of granulomatous thyroiditis
except:
(a) Hyperthyroidism
(b) Hypothyroidism
(c) Painless
(d) Giant cell histology
61. Brown tumour is seen in:
(a) Hyperthyroidism
(b) Hypothyroidism
(c) Hyperparathyroidism
(d) Hypoparathyroidism
62. Which of the following is true in parathyroid carcinoma?
(a) 5–10% incidence
(b) Reduced serum calcium levels
(c) Cytology is diagnostic
(d) Metastasis is essential
63. In follicular carcinoma, the most commonly detectable
chromosomal translocation is:
(a) PAX8-PPARγ1
(b) RET-PTC
(c) RET
(d) JAK –TEL

PITUITARY

64. Which of the following is true about pituitary tumor?


(a) It present in 10% of brain tumors
(PGI Dec 2005)
(b) Erodes the sella and extends into surrounding area
(c) Prolactinoma is least common
(d) It is differentiated by reticulin stain
65. In Galactorrhea - amenorrhea syndromes, which is the investi-
gation you should advise (apart from serum prolactin)?
(a) TSH
(b) LH
(c) Urinary ketosteroids
(d) HCG
adrenal gland
66. Addison’s disease was first reported by Thomas Addison. It is
still being widely reported from various parts of the
world/throughout the world. Which of the following is the most
common cause of Addison’s disease in India?
(AIIMS Nov 2011)
(a) Post-partum pituitary insufficiency
(b) Tuberculous adrenalitis
(c) HIV
(d) Autoimmune adrenal insufficiency
67. All are true statements about pheochromocytoma except?
(AI 2011)
(a) 90% are malignant
(b) 95% occur in the abdomen
(c) They secrete catecholamines
(d) They arise from sympathetic ganglia
68. All the following familial syndromes are associated with
development of pheochromocytomas except:
(a) Sturge-Weber syndrome
(AIIMS Nov 2002)
(b) Von Recklinghausen disease
(c) MEN type II
(d) Prader-Willi syndrome
69. The most common cause of Addison’s disease is:
(a) Autoimmune adrenalitis
(AIIMS May 2002)
(b) Meningococcal septicemia
(c) Malignancy
(d) Tuberculosis
70. Most important histopathological indicator of malignancy in
Pheochromocytoma is:
(a) Pleomorphism
(Delhi PG-2005)
(b) High mitotic activity
(c) Vascular invasion
(d) None
71. Which of the following is most often involved in multiple
endocrine neoplasia I:
(DNB- 2007)
(a) Pituitary
(b) Pancreas
(c) Parathyroid
(d) Thyroid
72. All are true about pheochromocytoma except:
(a) 25% are malignant
(UP 2002)
(b) Variety of APUDOMA
(c) Histological type is chromaffin cells
(d) Most common neuroendocrine hormone secreting tumor
73. In Cushing syndrome, the tumor is associated with
(a) Increased level of epinephrine
(UP 2004)
(b) Decreased level of epinephrine
(c) Elevated levels of cortisol
(d) Increased level of norepinephrine
74. Vanillylmandelic acid (VMA) is increased in
(a) Hyperparathyroidism
(UP 2008)
(b) Pheochromocytoma
(c) MEN-I
(d) Addison’s disease
MOST RECENT QUESTIONS
75. Most common cause of Cushing’s syndrome is
(a) Exogenous corticosteroids
(b) Pituitary tumor
(c) Adrenal adenoma
(d) Adrenal carcinoma
76. Submucosal neuroma is associated with:
(a) MEN I
(b) MEN II A
(c) MEN II B
(d) None of the above
77. Most common site of pheochromocytoma after adrenal gland is:
(a) Hilum of kidney
(b) Organs of Zuckerkandl
(c) Neck
(d) Urinary bladder
78. True about adrenal pheochromocytoma is:
(a) Chromaffin negative
(b) Mostly malignant
(c) Bilateral in 10% of cases
(d) Unilateral in 10% of cases
79. Most common cause of Cushing’s syndrome is:
(a) Pituitary adenoma
(b) Adrenal adenoma
(c) Exogenous steroids
(d) Ectopic ACTH
80. Which of the following is not estrogen dependant carcinoma:
(a) Lobular carcinoma breast
(b) Follicular thyroid carcinoma
(c) Endometrial leiomyosarcoma
(d) Carcinoma prostate
81. A 40-year old man with central obesity, “buffalo hump” and
vertical purple striae on the abdomen has fasting blood glucose
is in the high normal range. Plasma levels of ACTH and cortisol
are both increased compared to normal. An overnight high-dose
dexamethasone test produces 75% suppression of cortisol
levels. This patient most likely has
(a) Addison’s disease
(b) an ectopic ACTH-secreting tumor
(c) Conn’s syndrome
(d) Cushing’s disease
82. In Conn’s syndrome, all the following are seen, except
(a) Hypokalemia
(b) Hypernatremia
(c) Hypertension
(d) Edema
83. Dilutional hyponatremia is seen in:
(a) Addison’s disease
(b) Diabetes insipidus
(c) Diuretic therapy
(d) None
84. Tumor that follows rule of 10 is:
(a) Pheochromocytoma
(b) Oncocytoma
(c) Lymphoma
(d) Renal cell carcinoma
85. Ectopic pheochromocytoma may originate from which of the
following?
(a) Organ of Zuckerkandl
(b) Bladder
(c) Filum terminale
(d) Meckel diveticulum
86. Which one of the following is not seen in pheochromocytoma?
(a) Hypertension
(b) Episodic palpitations
(c) Weight loss
(d) Diarrhea
87. Mitotic figures and giant cells are seen in which of the following
tumor?
(a) Benign and malignant thyroid cancers
(b) Benign and malignant pheochromocytoma
(c) Benign and malignant liver tumors
(d) Benign and malignant renal tumors
88.Homer rosette is seen in:
(a) Neuroblastoma
(b) Nephroblastoma
(c) Hepatoma
(d) Ependymoma
89. Which of the following is not part of the “rule of 10” in
pheochromocytoma?
(a) 10% are bilateral
(b) 10% are malignant
(c) 10% are extra adrenal
(d) 10% are symptomatic
90. Which of the following is not a feature of Sipple Syndrome?
(a) Pheochromocytoma
(b) Medullary carcinoma
(c) Hyperthyroidism
(d) Hyperparathyroidism

ASSERTION AND REASON QUESTIONS


1–10. Will have two statements, assertion and reason. Read both of
them carefully and answer according to these options.
(a) Both assertion and reason are true and reason is correct
explanation of assertion.
(b) Both assertion and reason are true and reason is not the correct
explanation of assertion.
(c) Assertion is true and reason is false.
(d) Both assertion and reason are false.
1. Assertion: C peptide levels are used as a surrogate marker for insulin
secretion
Reason: C peptide and insulin are secreted in equal amounts after β cell
stimulation
2. Assertion: Pheochromocytoma is also referred to as ‘rule of 10 tumor’
Reason: Pheochromocytoma is seen in 10% familial cases
3. Assertion: FNAC is not useful for diagnosing follicular thyroid cancer
Reason: Capsular invasion is the definitive feature differentiating follicular
adenoma from follicular carcinoma
4. Assertion: Postpartum thyroiditis presents as a painful enlarged thyroid
gland
Reason: Postpartum thyroiditis is preceded by a viral infection (measles,
mumps etc)
5. Assertion: Graves disease is most common cause of endogenous
hyperthyroidism.
Reason: Autoantibodies like TBIG may lead to reduced thyroid function
6. Assertion: Myocardial infarction is the commonest cause of death in
diabetes
Reason: Uncontrolled blood glucose leads to capillary dysfunction in
target organs
7. Assertion: Skin hyperpigmentation is a feature of Addison’s disease
Reason: ACTH precursor has amino acid sequence similar to
melanocyte stimulating hormone.
8. Assertion: Non ketotic hyperosmolar is a commoner complication than
diabetic ketoacidosis in type 2 diabetes.
Reason: Insulin has a ‘fat sparing effect’ preventing fatty acid oxidation.
9. Assertion: MEN 2A (Sipple syndrome) is characterized by
pheochromocytoma, medullary carcinoma of thyroid and parathyroid
hyperplasia.
Reason: MEN 2A is associated with a loss of function mutation in RET
proto-oncogene
10. Assertion: Amyloid deposition is associated with medullary variant of
thyroid cancer
Reason–T he altered cacitonin secreted by parafollicular cells gets
deposited in the thyroid stroma
1. Ans. (b) Beta cells
(Ref: Robbins 9/e p446, 8th/442, 1130)
• Amylin is secreted by b cells of the pancreas. It reduces food intake
and weight gain by acting on central neurons in the hypothalamus.

2. Ans. (b) The level of fasting glucose is > 125 mg/dL and that of
post prandial glucose is > 199 mg/dL
(Ref: Harrison 18th/2970, Robbin 9/e p1106)
3. Ans. (b) Occurs more commonly in adults than in children (Ref:
Nelson Pediatrics 18th/660-662; Robbins, 9/e p1121)

• Congenital hyperinsulinism was formerly termed as Nesidioblastosis.


It is also known as diffuse Islet cell hyperplasia.
It can occur in adults also but more commonly seen in neonates and
children.
Hypoglycemic episodes occur due hyperinsulinemia.
Medical management includes Frequent feedings, Diazoxide and
Somatostatin.

4. Ans. (b) Increasing the number of GLUT4 in muscle and adipose


tissue and glucokinase in liver cells
(Ref: Harrison 17th/2278-9, 2282 Robbins 8th/1134, 9/e p1112)
Insulin acts by binding to its receptor and stimulating intrinsic tyrosine
kinase activity, leading to receptor autophosphorylation and the
recruitment of intracellular signaling molecules, such as insulin
receptor substrates (IRS). Activation of the phosphatidylinositol-3’-
kinase (PI-3-kinase) pathway stimulates translocation of glucose
transporters (e.g., GLUT4) to the cell surface, an event that is crucial
for glucose uptake by skeletal muscle and fat. Activation of other
insulin receptor signaling pathways induces glycogen synthesis,
protein synthesis, lipogenesis, and regulation of various genes in
insulin-responsive cells.
Glucokinase catalyzes the formation of glucose-6-phosphate from glucose, a
reaction that is important for glucose sensing by the beta cells and for glucose
utilization by the liver. As a result of glucokinase mutations, higher glucose
levels are required to elicit insulin secretory responses, thus altering the set
point for insulin secretion, responsible for Maturity Onset Diabetes of Young-1
(MODY-1).

5. Ans. (c) Follicular adenoma of thyroid


(Ref: Robbins 9/e p1093, 8th/1118; 7th/1175, Harsh Mohan 6th/810)
6. Ans. (d) HbA 1C
(Ref: Robbins 8th/1138, 9/e p1115)
7. Ans. (c) Develop ketoacidosis
(Ref: Robbins 8th/1145, 9/e p1113-1114)
Raju probably has type 1 (juvenile onset) diabetes mellitus, while Antara
probably has type 2 (maturity onset) diabetes mellitus. These two
types of diabetes differ in many respects. Ketoacidosis is more likely
to develop in type 1 diabetes.
Type 1 diabetes has a strong association with HLA-DR3 and HLA-DR4
(option A), while type 2 does not have any strong HLA associations.
Type 1 is usually apparently due to viral or immune destruction of beta
cells, while type 2 is apparently usually due to increased resistance to
insulin; consequently the 65-year-old, rather than the 16-year-old, is
more likely to have relatively high endogenous levels of insulin
(option D).
Type 2 diabetes can often be controlled with oral hypoglycemic agents
(option B), while type 1 diabetics generally require insulin. Note that
some type 2 diabetics also may require insulin as the disease
evolves.

8. Ans. (a) Adrenal gland


(Ref: Robbins 8th/1156-1157, 9/e p1130-11131)
This is Addison disease, in which severe adrenal disease produces
adrenocortical insufficiency. Causes include auto-immune destruction,
congenital adrenal hyperplasia, hemorrhagic necrosis, and
replacement of the glands by either tumor (usually metastatic) or
granulomatous disease (usually tuberculosis). The symptoms can be
subtle and nonspecific (such as those illustrated), so a high clinical
index of suspicion is warranted. Skin hyperpigmentation is a specific
clue that may be present on physical examination, suggesting excess
pituitary ACTH secretion. (The ACTH precursor has an amino acid
sequence similar to MSH, melanocyte stimulating hormone.) Most
patients have symptoms (fatigue, gastrointestinal distress) related
principally to glucocorticoid deficiency. In some cases, however,
mineralocorticoid replacement may also be needed for symptoms of
salt wasting with lower circulating volume.

• Except in the case of primary pancreatic cancer, complete tumor


replacement of the endocrine pancreas (option B) would be uncommon. In
any event, pancreatic involvement would be associated with diabetes
mellitus.
• Involvement of the ovaries (option C) by metastatic tumor (classically gastric
adenocarcinoma) would produce failure of menstruation.
• Involvement of the pituitary gland (option D) could produce Addisonian
symptoms, but the pigmented skin suggests a primary adrenal problem
rather than pituitary involvement.

9. Ans. (d) Cardiomegaly


(Ref: internet)
Mauriac syndrome is a rare complication of type 1 diabetes mellitus in children
associated with hepatomegaly, growth impairment and cushingoid features.

10. Ans. (c) Diabetes mellitus


(Ref: Harrison 18th/chapter 53)
Lesions of necrobiosis lipoidica are found primarily on the shins (90%),
and patients can have diabetes mellitus or develop it subsequently.
Characteristic findings include a central yellow color, atrophy
(transparency), telangiectasias, and a red to red-brown border.
Ulcerations can also develop within the plaques. Biopsy specimens
show necrobiosis of collagen and granulomatous inflammation.
11. Ans. (b) Steatosis
(Ref: Robbin 8/e p1136, Joslin’s Diabetes Mellitus 14/e p436)
Insulin resistance is defined as the failure of target tissues to respond
normally to insulin. It leads to decreased uptake of glucose in muscle,
reduced glycolysis and fatty acid oxidation in the liver, and an inability
to suppress hepatic gluconeogenesis.
• The loss of insulin sensitivity in the hepatocytes is likely to be the
largest contributor to the pathogenesis of insulin resistance in
vivo.
• Obesity is the most important factor in the development of insulin
resistance.
• “In type 2 diabetes patients, the presence of hepatic steatosis is
associated with reduced insulin stimulated glucose uptake.
Increased hepatic fat accumulation results in impaired peripheral
insulin action”….Joslin pg 436
12. Ans. (a) 126 mg/dl
(Ref: Robbins 9th/1106)
13. Ans. (a) gastrinoma
(Ref: Robbins 9/e p1136)
Know the following about MEN-1-associated pancreatic endocrine
tumors:

• Pancreatic polypeptide is the most commonly secreted product, however it is


non functional.
• In functional tumors, gastrinomas are the commonest.

14. Ans. (c) C-peptide level is elevated


(Ref: Robbins 9/e p1121)

• β-cell tumors (insulinomas) are the most common of pancreatic endocrine


neoplasms.
• Most of them are benign.
• It is characterized by hypoglycemic episodes when blood glucose is less than 50
mg/dL.
• Plasma levels of insulin and C peptide are elevated.

15. Ans. (a) Chromogranin-A


(Ref: Robbins 9/e p717)
16. Ans. (c) ) Type II DM
(Ref: Robbins 9/e p259)
17. Ans. (d) ) PDGF-R
(Ref: Robbins 9/e p1116)
18. Ans. (b) Jod-Basedow effect
(Ref: Harrison 18th/2930, 2932)

• Jod Basedow effect is characterized by excessive thyroid hormone


synthesis caused by increased iodine exposure.
• Wolff Chaikoff effect is iodide dependent suppression of the thyroid.

19. Ans. (d) Primary hypothyroidism


(Ref: Robbins 8th/1109-1110, 9/e p1083-5)
Analyzing all options,
• In pituitary adenoma/TSH secreting pituitary tumor increased TSH
with increased T3/4 would be seen. (excludes ‘a’ and ‘b’)
• In thyroid hormone resistance increased T4 as well as T3 with
low TSH will be seen. (option ‘c’ excluded)
• Primary hypothyroidism is due to defect in the thyroid gland
itself. This is associated with high TSH with low T4 will be seen.

20. Ans. (d) Orphan Annie eye nuclei

(Ref: Robbins 8th/1111, 9th/1087)


Robbins writes…‘The nuclei of papillary carcinoma cells contain finely
dispersed chromatin, which imparts an optically clear or empty
appearance, giving rise to the designation ground-glass or Orphan
Annie eye nuclei’.
Salient features of Hashimoto Thyroiditis (Chronic Lymphocytic
Thyroiditis)

Most common type of thyroiditisQ


Most common cause of hypothyroidism in areas having sufficient iodine levels.

Genetic association

Associated with HLA-DR5, HLA-DR3 and chromosomal defects like Turner and
Down syndrome.

Gland morphology

Diffusely enlarged gland with intact capsule.

Microscopic finding

Presence of well developed germinal centers and extensive lymphocytic


infiltration

AtrophiedQ thyroid follicles lined by epithelial cells having abundant eosinophilic


and granular cytoplasm called Hurthle cells.Q (this is a metaplastic response of
epithelium to the ongoing injury)

Chronic inflammation with lymphocytic infiltrationQ of the thyroid gland (the latter
responsible for the term ‘struma lymphomatosa’).

Clinical findings

Painless enlargement of thyroid gland in a middle aged female presenting with


hypothyroidism.
Associated with type 1 diabetes, SLE, Sjogren syndrome, myasthenia gravis,
increased risk of B cell lymphoma
21. Ans. (a) Hashimoto’s thyroiditis

(Ref: Harrison 17th/2230, Robbins 9th/1087)


Hashimoto’s thyroiditis is a cause of hypothyroidism whereas other
diseases mentioned like Graves’ disease, toxic Multinodular goiter
and struma ovarii result in hyperthyroidism.
Causes of hypothyroidism

• Autoimmune hypothyroidism: Hashimoto’s thyroiditis, atrophic thyroiditis


• Iatrogenic: 131I treatment, subtotal or total thyroidectomy, external irradiation
of neck for lymphoma or cancer
• Drugs: iodine excess (including iodine-containing contrast media and
amiodarone), lithium, antithyroid drugs, p-aminosalicyclic acid, interferon-
alpha and other cytokines, aminoglutethimide
• Congenital hypothyroidism: absent or ectopic thyroid gland,
dyshormonogenesis, TSH-R mutation
• Iodine deficiency
• Infiltrative disorders: amyloidosis, sarcoidosis, hemochromatosis,
scleroderma, cystinosis, Riedel’s thyroiditis

Causes of hyperthyroidism

• Graves’ disease
• Toxic multinodular goiter
• Toxic adenoma
• Functioning thyroid carcinoma metastases
• Activating mutation of the TSH receptor
• Activating mutation of Gsa (McCune-Albright syndrome)
• Struma ovarii
• Drugs: iodine excess (Jod-Basedow phenomenon)

22. Ans. (d) Orphan Annie eye nuclei


(Ref: Robbins 8th/1111, 9/e p1096)
Robbins writes…‘The nuclei of papillary carcinoma cells contain finely
dispersed chromatin, which imparts an optically clear or empty
appearance, giving rise to the designation ground-glass or Orphan
Annie eye nuclei’.
23. Ans. (a) Papillary carcinoma
(Ref: Robbins 7th/735-736, 9/e p1095)
24. Ans. (a) RET Proto-oncogene
(Ref: Harrison’s 17th/2361,
Robbins 7th/1182, 9/e p1095)
• Medullary carcinoma of thyroid (MCT) pheochromocytoma and
hyperparathyroidism are present in MEN-2A whereas the
association of MCT, pheochromocytoma, mucosal neuroma and
Marfanoid habitus is designated MEN-2B.
• Most patients of MEN-2 have mutations of RET-proto-oncogene.
• This gene is located on chromosome 10q11.2.
• Most common germline mutation of RET is at codon 634 and is
associated mostly with MEN 2A.
• Most common somatic mutation of RET is at codon 918 and is
mostly associated with MEN 2B.
• Other genes given in the question can be remembered from their
name only:
– FAP: Familial adenomatous polyposis
– Rb: Retinoblastoma
– BRCA1: Breast cancer
25. Ans. (b) MEN II
(Ref: Robbins 7th/1222, 9/e p1099)
Multiple endocrine neoplasia (MEN) syndromes are a group of genetically
inherited diseases resulting in proliferative lesions (hyperplasia,
adenoma and carcinoma) of multiple endocrine glands.
MEN I (Wermer’s Syndrome) MEN II A (Sipple’s MEN II B
syndrome)
Parathyroid hyperplasia/adenoma Parathyroid Medullary
Pancreatic islet cell hyperplasia/adenoma carcinoma of
hyperplasia/adenoma/carcinoma Medullary carcinoma thyroid
Pituitary hyperplasia/adenoma of thyroid Pheochromocytoma
Pheochromocytoma Mucosal and
• Mutant gene is MEN 1
gastrointestinal
• Mutant gene is RET
neuromas
Marfanoid features
• Mutant gene is RET

26. Ans. (c) RET proto-oncogene


(Ref: Robbins 9/e 1099)
RET proto-oncogene is mutated in MEN-2A and MEN-2B syndromes.
These are associated with medullary carcinoma of thyroid.
27. Ans. (a) Granulomatous thyroiditis
(Ref: Robbins 7th/1170, 9/e p1088)
• DeQuervain’s thyroiditis is also referred to as granulomatous
thyroiditis or subacute thyroiditis. (For details, see text)
28. Ans. (b) Hashimoto’s thyroiditis
(Ref: Robbins 9/e p1087)
• Hurthle cells (Oncocytes) are epithelial cells with abundant
eosinophilic, granular cytoplasm.
• Hurthle cells are seen in following conditions:

– Hashimoto’s thyroiditis
– Hurthle cell adenoma of thyroid
– Hurthle cell carcinoma of thyroid

• Hurthle cells are eosinophilic due to abundance of mitochondria.


• Hurthle cells are also called Ashkanazy cells or oxyphil cells.
29. Ans. (c) Papillary
(Ref: Robbins 7th/1180, 9/e p1097)
• Most papillary carcinoma present as asymptomatic thyroid
nodules, but the first manifestation may be a mass in cervical
nodes.
• Follicular carcinoma has little propensity for lymphatics but high
for vascular invasion and spreads to bones, lungs and CNS.
• In anaplastic Ca direct spread (extensive local invasion) is more
common than vascular.
30. Ans. (a) Mature thyroid tissue
(Ref: Robbins 9/e p1030)
Struma ovarii is a specialized teratoma which is always unilateral and is
composed of mature thyroid tissue. It may manifest itself with the
features of hyperthyroidism.
31. Ans. (d) Pineal body
(Ref: Internet)
Oncocytes are large cells with small irregular nuclei and dense acidophilic
granules due to the presence of abundant mitochondria. These are
found in oncocytomas of the kidney, salivary glands, and endocrine
glands (thyroid, parathyroid, pituitary, adrenal cortex and pancreatic
islets).
32. Ans. (a) Hashimoto’s thyroiditis
(Ref: Robbins 9/e p1087)
33. Ans. (b) Medullary carcinoma
(Ref: Robbins 9/e p1099)
34. Ans. (c) Leukocytes
(Ref: Robbins 9/e p1087)
35. Ans. (b) Thymoma
(Ref: Robbins 9/e p1236)
36. Ans. (a) Hyperparathyroidism
(Ref: Robbins 9/e p1136-1137, 8th/1162; 7th/1222)
37. Ans. (c) Retro-sternal goiter
(Ref: Robbins 9/e p1092)
38. Ans. (d) Medullary carcinoma of thyroid
(Ref: Robbins 8th/1162, 9/e p1136)
39. Ans. (b) Papillary adenoma of colon (Ref:
Robbins 9/e p1096, 1314, 1025, 8th/38,1122; 7th/859)
40. Ans. (a) Follicular variant
(Ref: Robbins 9/e p1096-1097)
41. Ans. (a) Medullary carcinoma
(Ref. Robbins 9/e p1099)
42. Ans. (b) Medullary carcinoma is autosomal recessive
(Ref. Robbins 8th/1124-1126, 9/e p1095)
43. Ans. (b) Follicular carcinoma
(Ref: Robbins 9/e p1098)
44. Ans. (c) Early metastasis with poor prognosis
(Ref: Robbins 8th/1121-1122, 9/e 1097)
45. Ans. (c) Hyperparathyroidism
(Ref: Robbins 9/e p1103)
The patient’s history of recurrent urolithiasis with calcium-containing stones
implies a disorder in the regulation of calcium concentration.
Hyperparathyroidism is associated with increased parathormone
(PTH) levels, which can produce hypercalcemia, hypercalciuria, and,
ultimately, renal stones.
Anemia of chronic disease (option A) does not produce calcium stones.
The patient presents with a chronic condition and hematuria but the
urinary blood loss is not usually significant enough to produce an
anemic state.
Hyperaldosteronism (option D) results in potassium depletion, sodium
retention, and hypertension. Primary hyperaldosteronism (Conn’s
syndrome) is associated with adrenocortical adenomas in 90% of
patients and is characterized by decreased renin. Secondary
hyperaldosteronism results from excessive stimulation by
angiotensin II that is caused by excess renin production (plasma
renin-angiotensin levels are high). Neither condition is associated
with renal stones.
46. Ans. (c) Follicular cancer
(Ref: Robbins 9th/1094)
Direct quote… “Because of the need for evaluating capsular integrity, the
definitive diagnosis of adenomas can be made only after careful
histologic examination of the resected specimen. Suspected
adenomas of the thyroid are therefore removed surgically to exclude
malignancy”.
47. Ans. (c) Milk alkali syndrome
(Ref: Robbins 9th/65)
Milk -alkali syndrome is due to excessive ingestion of calcium and
absorbable antacids such as milk or calcium carbonate. This is
associated with the development of metastatic calcification.
48. Ans. (a) Papillary cancer
(Ref: Robbins 9/e p1095)
• There is a marked increase in the incidence of papillary
carcinomas among children exposed to ionizing radiation
(particularly during the first 2 decades of life).
• Deficiency of dietary iodine (and so, goiter) is linked with a
higher frequency of follicular carcinomas.
49. Ans. (a) Papillary carcinoma thyroid
(Ref: Robbins 8/e p1122, 9/e p1096)
• The nuclei of papillary carcinoma cells contain finely dispersed
chromatin, which imparts an optically clear or empty
appearance, giving rise to the designation ground-glass or
Orphan Annie eye nuclei.
50. Ans. (a) TSH
(Ref: Robbins 9th /1083)
51. Ans. (b) MEN II
(Ref: Robbins 8/e p1162, 9/e p1137)
• MEN-2A, or Sipple syndrome, is characterized by
pheochromocytoma, medullary carcinoma, and parathyroid
hyperplasia.
• MEN-2B has significant clinical overlap with MEN-2A. Patients
develop medullary thyroid carcinomas, which are usually
multifocal and more aggressive than in MEN-2A, and
pheochromocytomas. However, unlike in MEN-2A, primary
hyperparathyroidism is not present
52. Ans. (c) Medullary
(Ref: Robbins 8/e p1125, 9/e p1099)
Acellular amyloid deposits, derived from altered calcitonin polypeptides,
are present in the adjacent stroma in many cases of medullary thyroid
cancer.
53. Ans. (d) Medullary carcinoma thyroid
(Ref: Robbins 8/e p1162, 9/e p1136)
MEN-1, or Wermer syndrome, is characterized by abnormalities involving
the parathyroid, pancreas, and pituitary glands; thus the mnemonic
device, the 3Ps

• Parathyroid: Primary hyperparathyroidism is the most common manifestation of


MEN-1
• Pancreas: Endocrine tumors of the pancreas like gastrinomas associated with
Zollinger-Ellison syndrome and insulinomas
• Pituitary: The most frequent anterior pituitary tumor encountered in MEN-1 is a
prolactinoma.

54. Ans. (a) MEN I….see earlier explanation……


(Ref: Robbins 8/e p1162, 9/e p1136)
55. Ans. (a) Papillary carcinoma thyroid
(Ref: Robbins 9th /1096)
56. Ans. (a) Papillary
(Ref: Robbins 9th/1100)
Direct quote… “Papillary carcinomas are recognized based on nuclear features (ground-
glass nuclei, pseudoinclusions) even in the absence of papillae. Psammoma bodies are a
characteristic feature of papillary cancers; these neoplasms often metastasize by way of
lymphatics, but the prognosis is excellent”.

57. Ans. (d) Follicular carcinoma of thyroid


(Ref: Robbins 9th/1098)
There is no reliable cytologic difference between follicular adenomas and
minimally invasive follicular carcinomas. Making this distinction
requires extensive histologic sampling of the tumor-capsule-thyroid
interface to exclude capsular and/or vascular invasion.
58. Ans. (b) Follicular carcinoma
(Ref: Robbins 9/e p1095)
Deficiency of dietary iodine (and by extension, an association with
goiter) is linked with a higher frequency of follicular carcinomas.
59. Ans. (b) Posterior pituitary tumors
(Ref: Robbins 9/e p1136)
MEN-1, or Wermer syndrome is characterized by anterior pituitary
tumours and not posterior pituitary tumours.
MEN-1, or Wermer syndrome is characterized by abnormalities involving
the parathyroid, pancreas, and pituitary glands; (mnemonic: the 3Ps)

• Parathyroid: Primary hyperparathyroidism is the most common manifestation of


MEN-1 (80 – 95% of patients) and is the initial manifestation of the disorder in
most patients. This includes both hyperplasia and adenomas.
• Pancreas: Endocrine tumors of the pancreas are a leading cause of morbidity
and mortality in persons with MEN-1.
• Pituitary: The most frequent anterior pituitary tumor encountered in MEN-1 is a
prolactinoma; some patients develop acromegaly from somatotrophin-secreting
tumors.
• Also know that the spectrum of this disease extends beyond the 3Ps. The
duodenum is the most common site of gastrinomas in individuals with MEN-1.
In addition, carcinoid tumors, thyroid and adrenocortical adenomas, and
lipomas are more frequent than in the general population.

MEN-1 syndrome is caused by germline mutations in the MEN1


tumor suppressor gene, which encodes a protein called menin.
60. Ans. (c) Painless
(Ref: Robbins 9/e p1088)
• Granulomatous thyroiditis is the most common cause of thyroid pain

61. Ans. (c) Hyperparathyroidism

(Ref: Robbins 9/e p1102)


62. Ans. (d) Metastasis is essential for diagnosis
(Ref: Robbins 9/e p1088)
• Diagnosis of parathyroid carcinoma based on cytologic detail is unreliable, and
invasion of surrounding tissues and metastasis are the only reliable criteria
• Parathyroid carcinoma has an incidence of nearly 1%

63. Ans. (a) PAX8-PPARγ1

(Ref: Robbins 9/e p1095)


Papillary thyroid cancer • RET/PTC
• NTRK1
• BRAF
Follicular carcinoma • RAS or the PI-3K/AKT arm
• PAX8- PPARG fusion genes
Medullary cancer • RET
Anaplastic cancer • RAS or PIK3CA mutations, TP53
inactivation, activating mutations of β-
catenin

64. Ans. (a) It is present in 10% of brain tumors; (b) Erodes the sella
and extends into surrounding area; (d) It is differentiated by
reticulin stain; (Ref: Harrison 16th/208I;
Robbins 7th/1l59; 9/e p1075, Brains Neurology 11th/558)
• 10% of all intracranial neoplasms are pituitary tumors.
• Benign adenomas are most common.
• The most common pituitary adenoma is microadenoma and about
70% microadenomas are prolactinoma.
• The tumor can erode the sella and extends into surrounding area
and gives rise to local mass effect.
• Cellular monomorphism and absence of significant reticulin
network distinguishes pituitary adenomas from non-neoplastic
anterior pituitary parenchyma.
65. Ans. (a) TSH
(Ref: Robbins 9th/1079)
Increased serum levels of prolactin, or prolactinemia, cause
amenorrhea, galactorrhea, loss of libido, and infertility. This may be
caused due to the following reasons:
a. Prolactin-secreting pituitary adenomas.
b. Physiologic hyperprolactinemia: occurs in pregnancy
c. Pathologic hyperprolactinemia: can result from damage of the
dopaminergic neurons of the hypothalamus, damage of the pituitary
stalk (e.g., due to head trauma), or exposure to drugs that block
dopamine receptors.

d. Other causes of hyperprolactinemia include renal failure and


hypothyroidism.

Thus, in the given options, estimation of the serum TSH is the most
appropriate answer.
66. Ans. (b) Tuberculous adrenalitis
(Ref: Robbins 9/e p1130, 8th/1155-6, API 7th/1073, Harrison 18th/2955)
The commonest cause of Addison’s disease is as follows:
• In developing countries – TuberculosisQ
• In developed countries – Autoimmune (Idiopathic atrophy)Q

67. Ans. (a) 90% are malignant


(Ref: Robbins 9/e p1134)
Pheochromocytoma is a tumor of the adrenal medulla which produces
catecholamines. The patients usually have severe headache, anxiety,
increased sweating, tachycardia, palpitations and hypertensive
episodes.
Features of ‘rule of 10’s’in pheochromocytoma
10% are bilateralQ
10% are extra-adrenalQ
10% are malignant Q
10% occur in children Q
10% are not associated with hypertension Q

Please note
Earlier, it was mentioned that 10% are pheochromocytoma are familialQ but
latest Robbins says “25% of the individuals with pheochromocytoma and
paraganglioma have a germline mutation”

68. Ans. (d) Prader-Willi syndrome


(Ref: Robbins’s illustrated 7th/1219, 9/e p1134)
Pheochromocytoma is associated with the following familial syndromes.
• MEN syndromes type II and type III
• Von Hippel Lindau syndrome
• Von Recklinghausen disease
• Sturge Weber syndrome Famibial paraganglioma 1/3/4
69. Ans. (a) Autoimmune adrenalitis
(Ref: Harrison, 17th/2263, Robbins 9/e p1134)
Addison’s disease must involve >90% of the glands before adrenal
insufficiency develops.
Idiopathic atrophy due to autoimmune adrenalitis is the most common
cause of Addison’s disease in the worldwhereas tuberculosis is the
most common cause of the same in India.
70. Ans. (d) None
(Ref: Robbins 7th/1221, 9th/1135)
• The histological pattern in pheochromocytoma is quite variable.
The tumors are composed of polygonal to spindle-shaped
chromaffin cells, clustered with their supporting cells into small
nests or alveoli (Zellballen), by a rich vascular network.
• Cellular and nuclear pleomorphism is often present, especially in
the alveolar group of lesions and giant and bizarre cells are
commonly seen.
Mitotic figures are rare and do not imply malignancy.
Both capsular and vascular invasion may be encountered in benign
lesions. Therefore the diagnosis of malignancy in
pheochromocytoma is based exclusively on the presence of
metastases. These may involve regional lymph nodes as well as
more distant sites, including liver, lung and bone.
71. Ans. (c) Parathyroid
(Ref: Robbins 9/e p1136)
72. Ans. (a) 25% are malignant
(Ref: Robbins 9/e p1134)
73. Ans. (c) Elevated levels of cortisol
(Ref: Robbins 9/e p1125)
74. Ans. (b) Pheochromocytoma
(Ref: Robbins 9/e p1136)
75. Ans. (a) Exogenous corticosteroids
(Ref: Robbins 8th/1148, 9/e p1125)
76. Ans. (c) MEN II B
(Ref: Robbins 9/e p1137)
77. Ans. (b) Organs of Zuckerkandl
(Ref: Robbins 9/e p1134, 8th/1159-1161; 7th/1221)
78. Ans. (c) Bilateral in 10% of cases
(Ref: Robbins 9/e p1134)
79. Ans. (c) Exogenous steroids
(Ref: Robbins 9/e p1125)
80. Ans. (b) Follicular thyroid carcinoma
(Ref: Robbins 8th/1120-1121, 9/e p1095)
81. Ans. (d) Cushing’s disease
(Ref: Robbins 9/e p1125)
This patient presents with “Cushingoid” signs and symptoms due to
hypercortisolism. While the acute effect of cortisol is to produce
lipolysis, patients with chronically increased cortisol levels develop a
characteristic central obesity and buffalo hump. The mechanism for
the redistribution of body fat is an interaction between cortisol and
insulin. The weight gain with hypercortisolism usually results from
increased appetite. Cortisol excess causes protein catabolism, which
leads to poor wound healing, decreased connective tissue, and
fragile blood vessels. The combination of thin skin and fragile blood
vessels leads to abdominal stretch marks (striae) the are
characteristically purple in color. Increased gluconeogenesis and
decreased peripheral insulin sensitivity lead to elevated blood
glucose. The hypercortisolism due to a functional tumor in the adrenal
cortex (primary hypercortisolism) has low plasma ACTH level.
The patient in the question has increased cortisol and increased ACTH. This
could result from either a functional ACTH-secreting tumor in the pituitary
(Cushing’s disease) or an ectopic tumor (such as a small cell carcinoma of
the lung, choice B). To distinguish between these two, we administer high
doses of the potent synthetic glucocorticoid, dexamethasone. High-dose
dexamethasone should suppress ACTH secretion from the pituitary by at
least 50%; secretion from an ectopic tumor typically is not suppressed by
dexamethasone.
Addison’s disease (choice A) is primary adrenal insufficiency, with increased
plasma ACTH (producing hyperpigmentation) and decreased plasma
cortisol and aldosterone compared to normal.
Conn’s syndrome (choice C) results from hypersecretion of aldosterone by
the adrenal cortex. In Cushing’s disease, it is due in part to the
mineralocorticoid-like effects of high plasma cortisol.

82. Ans. (b) Hypernatremia; (d) Edema

(Ref: Sircar pg 532, Harrison 18th/2950)


Harrison mentions that…. “The clinical hallmark of mineralocorticoid
excess is hypokalemic hypertension; serum sodium tends to be
normal due to the concurrent fluid retention, which in some cases can
lead to peripheral edema”.
Sircar Physiology writes…. Edema is not seen because of the escape
mechanism: escape from the sodium retaining effects of
hyperaldosteronism.
83. Ans. (b) Diabetes insipidus
Please don’t get confused with the diuretic therapy because it can lead to
increased ADH release and thus, may lead to hyponatremia.
84. Ans. (a) Pheochromocytoma…discussed earlier in detail.
(Ref: Robbins 9/e p1134, 8/e p524-5)
85. Ans. (a) Organ of Zuckerkandl
(Ref: Robbins 9th/1134)
Ten percent of pheochromocytomas are extra-adrenal, occurring in sites
such as the organs of Zuckerkandl and the carotid body.
86. Ans. (d) Diarrhea
(Ref: Robbins 9th/1135)
The dominant clinical manifestation of pheochromocytoma is hypertension,
observed in 90% of patients. Approximately two thirds of patients with
hypertension demonstrate paroxysmal episodes, which are described
as an abrupt, precipitous elevation in blood pressure, associated with
tachycardia, palpitations, headache, sweating, tremor, and a sense of
apprehension.
87. Ans. (b) Benign and malignant pheochromocytoma
(Ref: Robbins 9/e p1135)
Pheochromocytomas are difficult to diagnose exclusively from the
histological appearance. Cellular and nuclear pleomorphism,
including the presence of giant cells, and mitotic figures are often
seen in benign pheochromocytomas. So, the definitive diagnosis of
malignancy in pheochromocytomas is based exclusively on the
presence of metastases.
88. Ans. (a) Neuroblastoma
(Ref: Robbins 9/e p476)
89. Ans. (d) 10% are symptomatic
(Ref: Robbins 9/e p1134)
90. Ans. (c) Hyperthyroidism
(Ref: Robbins 9/e p1137)

EXPLANATIONS TO ASSERTION AND REASON


QUESTIONS
Explanations(1-10): While solving assertion reason type of
questions, we can use a particular method.
1. First of all, read both assertion (A) and reason (R) carefully
and independently analyse whether they are true or false.
2. If A is false, the answere will directly be (d) i.e. both A and R
are false. You can note that all other options (i.e. a, b or c)
consider A to be true.
3. If A is true, answer can be (a), (b) or (c), Now look at R. If R
is false, answer will be (c)
4. If both A and R are ture, then we have to know whether R is
correctly explaining A [answer is (a)] or it is not the
explanation of assertion [answer is (b)]
1. Ans. (a) Both assertion and reason are true and reason is correct
explanation of assertion.
(Ref: Robbins 8th/1133, 9/e p1108)
Preproinsulin produced in rough endoplasmic reticulum is delivered to
the Golgi apparatus where it is cleaed to generate mature insulin
and C peptide. Both are stored in secretory granules and
equimolar quantities are secreted after β cell simulation.
• Most important stimulus for insulin synthesis and release is
glucose itself.
2. Ans. (c) Assertion is true and reason is false.
(Ref: Robbins 8th/1159, 9/e p1134)
Pheochromocytoma is also referred to as ‘rule of 10 tumor’. It is
associated with the following:

• 10% are bilateralQ


10% are extra-adrenalQ
10% are malignant Q
10% occur in children Q
10% are not associated with hypertension Q

Most of the earlier texts mention that 10% of these tumors are familialQ but
Robbins 8th/1159 clearly says that it has been modified.
“As many as 25% of the individuals with pheochromocytoma and
paraganglioma have a germline mutation”.
3. Ans. (c) Assertion is true and reason is false. (Ref:
Robbins 8th/1119, 9/e p1098)
FNAC is not useful for diagnosing follicular thyroid cancer because it can
not distinguish between follicular adenoma from follicular carcinoma.
The most reliable feature of follicular cancer is demonstration of
capsular invasion or vascular invasion. This is best done with careful
histologic examination after specimen resection.
Intact capsule encircling the tumor is the hallmark of the benign tumor. …
Robbins 8th/1119
4. Ans. (d) Both assertion and reason are false. (Ref:
Robbins 8th/1113, 9/e p1088)
Postpartum thyroiditis/subacute lymphocytic thyroiditis is a variant of
Hashimoto’s thyroiditis. It presents as a painless enlargement of the
thyroid and transient hyperthyroidism (lasting about 2-8 weeks).
Investigations reveal elevated levels of T3 and T4 and reduced TSH.
How to differentiate between Hashimoto and subacute lymphocytic
thyroiditis?
• Hurthle cell metaplasia and fibrosis are not prominent as in
Hashimoto’s thyroiditis.
(Granulomatous Thyroiditis (De Quervain Thyroiditis) is more
commonly seen in females preceded by a viral infection (caused by
coxsackie virus, mumps, measles). The thyroid gland is diffusely
enlarged with intact capsule.
Clinical features are pain in neck, sore throat, fever, fatigue, anorexia,
myalgia, enlarged thyroid and the presence of transient
hyperthyroidism lasting for 2-6 weeks. It may be followed by
asymptomatic hypothyroidism but recovery is seen in most of the
patients.
5. Ans. (b) Both assertion and reason are true and reason is not the
correct explanation of assertion.
(Ref: Robbins 9/e p1089, 8th/1114-5)
Graves disease is most common cause of endogenous hyperthyroidism.
This disease is characterized by breakdown in self tolerance to
thyroid auto-antigens (most importantly TSH receptor). The
antibodies seen in these patients are as follows:
• Thyroid stimulating immunoglobulin: lead to hyperthyroidism
• Thyroid growth stimulating immunoglobulin: lead to
hyperthyroidism
• TSH binding inhibitor immunoglobulin (TBIG): lead to
episodes of hypothyroidism in some patients.
6. Ans. (b) Both assertion and reason are true and reason is not the
correct explanation of assertion.
(Ref: Robbins 8th/1138-9, 9/e p1117)
Myocardial infarction is the commonest cause of death in diabetes. It is
caused by atherosclerosis of the coronary arteries. Large and
medium sized vessel involvement is responsible for macrovascular
disease (MI, stroke and lower extremity gangrene).
Capillary dysfunction in target organs leads to microvascular disease (not
macrovascular disease) leading to diabetic retinopathy, neuropathy
and nephropathy.
7. Ans. (a) Both assertion and reason are true and reason is correct
explanation of assertion.
(Ref: Robbins 8th/1157, 9/e p1130)
Hyperpigmentation is quite characteristic of primary adrenal disease
(Addison’s disease) especially at pressure points and sun exposed
areas. This is caused by elevated levels of pro-opiomelanocortin
(POMC), which is derived from anterior pituitary and is a precursor of
ACTH and melanocyte stimulating hormone (MSH).
8. Ans. (a) Both assertion and reason are true and reason is correct
explanation of assertion.
(Ref: Robbins 9/e p1115)
Hyperosmolar nonketotic coma is due to the severe dehydration resulting
from sustained osmotic diuresis in patients (commoner in elderly)
who do not drink enough water to compensate for urinary losses from
chronic hyperglycemia. These patients don’t develop ketoacidosis
and its symptoms (nausea, vomiting, respiratory difficulties) because
of elevated portal insulin levels. The ‘fat sparing’ effect of insulin
prevents the formation of ketone bodies by inhibiting the fatty acid
oxidation in the liver.
9. Ans. (c) Assertion is true and reason is false.
(Ref: Robbins 8th/1162, 9/e p1137)
MEN 2A (Sipple syndrome) is characterized by pheochromocytoma,
medullary thyroid carcinoma and parathyroid hyperplasia. It is
associated with a gain of function (not loss of function mutation) in
RET proto-oncogene.
‘Loss of function’ mutations in RET cause intestinal aganglionosis and
Hirschprung diseaseQ.

10. Ans. (a) Both assertion and reason are true and reason is correct
explanation of assertion.
(Ref: Robbins 9/e p1099)
Amyloid deposition is associated with medullary thyroid cancer. The
chemical nature of the amyloid in this condition is ACal. It is because
of the deposition of the altered form of calcitonin which gets
deposited in the thyroid stroma.
1. A 30-year-old came with complaints of thyroid swelling. On
investigations her TSH levels were found to be elevated.
Postoperative histopathological examination reports showed
lymphocytic infiltration and Hurthle cells. Which of the following
is the most likely diagnosis?
(NEET 2020 like pattern)
(a) Graves disease
(b) Follicular carcinoma
(c) Hashimoto thyroiditis
(d) Medullary carcinoma thyroid
Ans. (c) Hashimoto thyroiditis
(Ref: Robbins 9th/1086)
Presence of thyroid swelling and increased TSH are features suggestive of
hypothyroidism. Biopsy showing lymphocytic infiltration and
Hurthle cells (presence of abundant eosinophilic, granular
cytoplasm) is characteristic of Hashimoto thyroiditis.
Good to know information!

• Hurthle cells can also be seen in patients with follicular adenoma and follicular
carcinoma.

2. A 25-year-old male presented with a 2 cm thyroid nodule. A


thyroidectomy was done. The histology picture is given below.
What could be the diagnosis?
(NEET 2020 like pattern)

(a) Papillary carcinoma thyroid


(b) Follicular adenoma
(c) Graves disease
(d) Adenomatous goiter
Ans. (a) Papillary Carcinoma thyroid Ref: Robbins 9th/1096
The histology shows the presence of optically clear or empty appearance
of the nuclei in the tumor cells. This appearance is also given the
designation of ground glass or Orphan Annie eye nuclei.
Additional nuclear findings can be the invaginations of the cytoplasm
which may give the appearance of intranuclear inclusions (“pseudo-
inclusions”) or intranuclear grooves.
Vital concept!
The diagnosis of papillary carcinoma can be made based on these
nuclear features, even in the absence of papillary architecture.

3. A middle-aged female undergoes a surgery for a neck swelling.


The gross and the microscopic images are shown as under.
What is the most probable diagnosis of this patient?
(AI 2018 Pattern)
(a) Hashimoto thyroiditis
(b) Follicular cancer thyroid
(c) Medullary cancer of the thyroid
(d) Anaplastic cancer of the thyroid
Ans. (c) Medullary cancer of the thyroid
(Ref: Robbins 9/e p1099)
The histology shows the presence of amyloid deposition in the histology
section. Acellular amyloid deposits derived from calcitonin
polypeptides are present in the stroma in patients of medullary thyroid
cancer.
4. All of the following are true about Graves disease except:
(AI 2018 Pattern)
(a) More common in males
(b) Autoimmune in aetiology
(c) May result in hyperthyroidism
(d) Non thyroid manifestations can also be seen
Ans. (a) More common in males
(Ref: Robbins 9/e p1089)
5. VMA is elevated in which of the following condition?
(AI 2018 Pattern)
(a) Conn syndrome
(b) Pheochromocytoma
(c) Tuberous sclerosis
(d) Addison disease
Ans. (b) Pheochromocytoma
(Ref: Robbins 9/e p1135)
Disclaimer
Any resemblance to an actual question is purely coincidental.
• Bone specific alkaline phosphatase, osteocalcin and type I collagen
extension peptide are markers for bone growth whereas
Hydroxyproline, TRAP, Cross linked N and C telopeptides, urine total
free deoxypyridinoline are markers of bone destruction/resorption.
• Tetracycline labeling: Best estimation of mineralization of newly
synthesized osteoid
• Seronegative spondyloarthropathies are: ankylosing spondylitis,
psoriatic arthritis, reactive arthritis (Reiter syndrome), arthritis in IBD
(enteropathic arthritis).
• Paget disease involves: Pelvis (most common), tibia, femur, skull,
spine (vertebrae), humerus, clavicle.
• Most common malignant tumor of bone: Secondaries (metastasis).
• Ewing’s sarcoma arises from primitive neuroectodermal cells. It is
associated with t (11 : 22). The cells contain glycogen in this
condition. MIC-2 (CD 99) is a marker of: Ewing’s sarcoma and
peripheral primitive neuroectodermal tumors (PENT).
• Metastasis is NOT seen in bones of: small bones of hand and feet.
It is however observed in the vertebrae (most common), pelvis,
proximal half of femur and humerus.
• Brown tumor of bone is seen in: Hyperparathyroidism.
• Dystrophin is lacking in: Duchenne’s muscular dystrophy.
• Myasthenia gravis is associated with Thymic hyperplasia (65%)
more commonly than with thymoma (15%).
• Perifascicular atrophy is seen in: Dermatomyositis.
• Cytogenetic abnormality of synovial cell sarcoma is t(X:18). It shows a
‘Biphasic pattern’ on histology. Its prognosis is dependent on the
grade of the tumor. Its markers include vimentin, cytokeratin and S-
100.
• Number of mitoses per high power field is the most distinctive
feature between leiomyosarcoma and leiomyoma.
• Rhabdomyosarcoma is the commonest sarcoma in children and has
the presence of Tadpole cells.
The bone is a type of connective tissue which is composed of
cells (osteoclasts and osteoblasts) and the extracellular matrix.
The extracellular components of bone consist of a solid mineral
phase (consisting of calcium and phosphate) and an organic matrix
consisting of type I collagen (90–95%), serum proteins such as
albumin, cell attachment/signaling proteins such as
thrombospondin, osteopontin and fibronectin, calcium-binding
proteins such as matrix glial protein and osteocalcin and
proteoglycans such as biglycan and decorin.
The mineral phase of bone is deposited initially in intimate
relation to the collagen fibrils and is found in specific locations in
the “holes” between the collagen fibrils. This architectural
arrangement of mineral and matrix results in a two-phase material
well suited to withstand mechanical stresses.

CELLS OF BONE

Osteoblasts synthesize and secrete the organic matrix. They are


derived from cells of mesenchymal origin. As an osteoblast
secretes matrix, which is then mineralized, the cell becomes an
osteocyte. Mineralization is a carefully regulated process
dependent on the activity of osteoblast-derived alkaline
phosphatase, which probably works by hydrolyzing inhibitors of
mineralization. Core-binding factor A1 (CBFA1, also called Runx2)
regulates the expression of several important osteoblast proteins
including osterix, osteopontin, bone sialoprotein, type I collagen,
osteocalcin, and receptor-activator of NFκB (RANK) ligand. Runx2
expression is regulated, in part, by bone morphogenic proteins
(BMPs).

Osteoclasts carry out resorption of bone. Macrophage colony-


stimulating factor (M-CSF) plays a critical role during several steps in
the pathway and ultimately leads to fusion of osteoclast progenitor cells to
form multinucleated, active osteoclasts. RANK ligand, expressed on the
surface of osteoblast progenitors and stromal fibroblasts binds to the
RANK receptor on osteoclast progenitors and stimulates osteoclast
differentiation and activation. Alternatively, a soluble decoy receptor,
referred to as osteoprotegerin, can bind RANK ligand and inhibit
osteoclast differentiation. Several growth factors and cytokines (including
interleukins 1, 6, and 11; TNF; and interferon-gamma modulate osteoclast
differentiation and function.

Remodeling of bone
The cycle of bone remodeling is carried out by the basic
multicellular unit (BMU), comprising a group of osteoclasts and
osteoblasts. In cortical bone, the BMUs tunnel through the tissue,
whereas in cancellous bone, they move across the trabecular
surface. The process of bone remodeling is initiated by contraction
of the lining cells and the recruitment of osteoclast precursors.
These precursors fuse to form multinucleated, active osteoclasts
that mediate bone resorption. Osteoclasts adhere to bone and
subsequently remove it by acidification (protons secreted by type II
carbonic anhydrase) and proteolytic digestion (by cathepsin K). As
the BMU advances, osteoclasts leave the resorption site and
osteoblasts move in to cover the excavated area and begin the
process of new bone formation by secreting osteoid, which is
eventually mineralized into new bone. After osteoid mineralization,
osteoblasts flatten and form a layer of lining cells over new bone.

Remodeling of bone occurs along lines of force generated by mechanical


stress.

Biochemical markers of bone resorption


a. Amino and carboxy terminal crosslinking telopeptide of bone
collagen
b. Pyridinoline
c. Free lysyl-pyridinoline
d. Tartarate-resistant acid phosphatase (TRAP)
e. Hydroxyproline (not very specific)

Biochemical markers of bone formation


a. Bone specific alkaline phosphatase
b. Procollagen type IC and IN propeptide
c. Osteocalcin
d. Alkaline phosphatase (not very specific)

PAGET’S DISEASE (OSTEITIS DEFORMANS)

Paget’s disease (osteitis deformans) can be characterized as a


collage of matrix madness. It is marked by regions of furious
osteoclastic bone resorption, which is followed by a period of
hectic bone formation. The net effect is a gain in bone mass. It has
the following three stages:
i. Initial osteolytic stage
ii. Mixed osteoclastic-osteoblastic stage
iii. Burnt-out quiescent osteosclerotic stage.
Paget’s disease usually begins after the age of 40 years and is
more common in whites. It has been linked to slow virus
infection by paramyxovirus. It can be involving one bone or
monostotic (tibia, ilium, femur, skull, vertebra, humerus) in about
15% of cases and affecting multiple bones or polyostotic (pelvis,
spine, skull) in the remainder. The axial skeleton or proximal femur
is involved in upto 80% of cases.
On radiography, the Pagetic bone is typically enlarged with
thick, coarsened cortices and cancellous bone. There is increased
serum alkaline phosphatase and increased urinary excretion
of hydroxyproline. The most common symptom is pain. Bone
overgrowth in the craniofacial skeleton may produce leontiasis
ossea and the weakened Pagetic bone may lead to invagination of
base of skull (platybasia).

The histologic hallmark is the mosaic pattern of lamellar bone which


is produced by prominent cement lines that anneal haphazardly oriented
units of lamellar bone. The involved bones are weak and fracture easily.

The complications of the disease include arteriovenous


shunts within the marrow resulting in high output cardiac failure
and increased risk of development of sarcomas like osteosarcoma,
chondrosarcoma, etc. Secondary osteoarthritis and chalk-stick
type fractures are the other complications in Paget’s disease.

BENIGN TUMORS OF THE BONE

Osteoma
Subperiosteal osteomas are benign tumors affecting most often the
skull and facial bones. They are usually solitary and are detected
in middle-aged adults. Multiple osteomas are seen in the setting of
Gardner’s syndrome.
Osteoid osteoma and osteoblastoma
Osteoid osteoma and osteoblastoma are terms used to describe
benign bone tumors that have identical histologic features but that
differ in size, sites of origin, and symptoms. Osteoid osteoma is
less than 2 cm in size and usually affects patients in their teens
and twenties. They usually involve the femur or tibia, where they
commonly arise in the cortex and less frequently within the
medullary cavity. Osteoid osteomas are painful lesions. The pain is
characteristically nocturnal, and is dramatically relieved by aspirin.
Microscopically, there is a central nidus of osteoid
surrounded by dense sclerotic rim of reactive cortical bone. X-ray
shows the presence of central radiolucency surrounded by a
sclerotic rim.

Osteochondroma (Exostosis)
It is a benign bony metaphyseal growth capped with cartilage
originating from the epiphyseal growth plate. It is seen in
adolescent males as a firm, solitary growth at the end of long
bones. It may be asymptomatic or may cause pain and deformity.
Rarely, it may undergo malignant transformation.
Multiple osteochondromas occur in multiple hereditary exostosis,
which is an autosomal dominant hereditary disease.
There is inactivation of both copies of the EXT gene in growth
plate chondrocytes in the pathogenesis of osteochondromas.
Multiple osteochondromas become apparent during childhood.

Chondroma
Chondromas are benign tumors of hyaline cartilage that usually
occurs in bones of endochondral origin. These can be

• Enchondroma: When origin is intramedullary


• Subperiosteal or juxtacortical: Originate from the surface of bone

Enchondromas are most common intraosseous cartilage


tumors. Their favored sites are short tubular bones of hand and
feet.
Enchondromas are composed of well-circumscribed nodules of
cytologically benign hyaline cartilage. The center of the nodule can
calcify whereas peripheral portion may undergo enchondral
ossification. The unmineralized nodules of cartilage produce well-
circumscribed oval lucencies that are surrounded by a thin rim of
radiodense bone (O ring sign).
Patients with Ollier’s disease may undergo malignant transformation to
chondrosarcoma whereas those with Maffuci’s syndrome have increased risk
of ovarian cancer and brain gliomas.

MALIGNANT TUMORS OF THE BONE

Osteosarcoma
It is the most common primary malignant tumor of the bone. It
has a bimodal age distribution with almost 75% occurring in
patients younger than age 20. The second peak occurs in the
elderly with conditions like Paget disease, bone infarcts, and prior
irradiation. It is more commonly seen in the males with increased
risk of the development in patients with familial retinoblastoma.
The patients usually have localized pain and swelling. It arises
from the metaphysis of the long bones with the knee being the
most commonly affected site. The tumor is a large, firm white-tan
mass with necrosis and hemorrhage. Microscopically, there is
presence of anaplastic cells producing osteoid and bone.
The tumor frequently breaks through the cortex and lifts the
periosteum, resulting in reactive periosteal bone formation. The
triangular shadow between the cortex and raised ends of
periosteum is known radiographically as Codman’s triangle.

Chondrosarcoma
These are group of tumors that produce neoplastic cartilage.
• Similar to chondroma, chondrosarcoma can be:
– Intramedullary
– Juxtacortical
• Chondrosarcomas are second most common malignant
matrix-producing tumor of bone (Most common is
osteosarcoma)
• Histologically, Chondrosarcomas are composed of malignant
hyaline and myxoid cartilage. Spotty calcifications may be
present and central necrosis may create cystic spaces. Tumors
vary in cellularity and cytological atypia. Malignant cartilage
infiltrates the marrow space and surrounds pre-existing
bony trabeculae.
• Chondrosarcomas commonly arise in central portions of
skeleton (including pelvis, shoulder and ribs). In contrast to
chondroma, chondrosarcoma rarely involve the distal
extremities. The clear cell variant of chondrosarcoma
characteristically originates from Epiphysis of tubular long
bones.
Osteoclastoma or Giant cell tumor of bone
It is a malignant tumor containing multinucleated giant cells mixed
with stromal cells. It is more commonly seen in the females and the
most commonly affected age group is 20-40 years. The tumor
involves the epiphysis of the long bones usually around the knee
(distal femur and proximal tibia). Microscopically, there is presence
of osteoclast like giant cells (having 100 or more nuclei) distributed
in a background of mononuclear stromal cells. Most of the tumors
are solitary.
Radiographically, giant cell tumors are large, purely lytic, and
eccentric, and erode into the subchondral bone plate. The
overlying cortex is frequently destroyed, producing a bulging soft
tissue mass delineated by a thin shell of reactive bone. This gives
rise to the soap bubble appearance. The tumor has high rate of
recurrence after excision.

Ewing’s sarcoma and primitive neuroectodermal tumor


(PNET)
Ewing sarcoma and PNET are primary malignant small round cell
tumors of bone and soft tissue. Both Ewing’s sarcoma and PNET
are the same tumor, differing only in their degree of neural
differentiation. Tumors that demonstrate neural differentiation by
any analysis are called PNETs, and those that are undifferentiated
are known as Ewing’s sarcoma.

Of all bone sarcomas, Ewing’s sarcoma has the youngest average age at
presentation and approximately 80% patients are younger than age 20
years. Boys are affected slightly more frequently than girls. The classic
translocation is t(11;22)(q24;q12) and the most common fusion gene
(EWS-FLI1) generated acts as a dominant oncogenes to stimulate cell
proliferation. The presence of p30/32, a product of mic-2 gene, is a cell
surface marker form Ewing’s sarcoma.

Ewing’s sarcoma and PNET usually arise in the diaphysis of


long tubular bones, especially the femur and the flat bones of the
pelvis.
The tumor is composed of sheets of uniform small, round cells
that are slightly larger than lymphocytes having scant cytoplasm,
which may appear clear because it is rich in glycogen. There is
presence of Homer-Wright rosettes (where the tumor cells are
arranged in a circle about a central fibrillary space) which are
indicative of neural differentiation.
They present as painful enlarging masses, and the affected
site is frequently tender, warm, and swollen with the patients
having systemic findings like fever, increased ESR, anemia and
leukocytosis (all of which results in the tumor resembling infection).
X-ray shows the characteristic periosteal reaction producing layers
of reactive bone deposited in an ‘onion-skin’ pattern.

Note: Metastatic tumors are the most common form of skeletal


malignancy.

SOFT TISSUE TUMORS

SYNOVIAL SARCOMA
These tumors forms about 10% of all soft tissue sarcomas. Less
than 10% of them are intra-articular. 60-70% involve the lower
extremities especially around knee and thigh. The histologic
hallmark of biphasic synovial sarcoma is the dual lining of
differentiation of the tumor cells (e.g. epithelial-like and
spindle cells). The calcified concretions can be present which
help in the diagnosis radiologically.
Immunohistochemically, these tumor cells yield positive
reactions for keratin and epithelial membrane antigen
(differentiating from most other sarcomas). Most synovial
sarcomas show a characteristic chromosomal translocation t(X ;
18) producing SYT-SSX1 or - SSX2 fusion genes. This specific
translocation is associated with poor prognosis.
Architectural Patterns in Soft Tissue Tumors

Pattern Tumor Type

Fascicles of eosinophilic spindle cells intersecting Smooth muscle


at right angles

Short fascicles of spindle cells radiating from a Fibrohistiocytic


central point (like spokes on a wheel)—storiform

Nuclei arranged in columns—palisading Schwann cell

Herringbone Fibrosarcoma

Mixture of fascicles of spindle cells and groups of Synovial sarcoma


epithelioid cells—biphasic
Fig. 1: Biphasic Appearance in Synovial Sarcoma: Spindle cells (long arrow)
and epithelioid cells (short arrows).

MUSCULAR DYSTROPHIES
The muscular dystrophies are a heterogeneous group of inherited
disorders, often beginning in childhood, that are characterized
clinically by progressive muscle weakness and wasting. The two
most common forms of muscular dystrophy are X-linked:
Duchenne muscular dystrophy (DMD) and Becker muscular
dystrophy (BMD). BMD is less common and much less severe
than DMD.

DMD and BMD are caused by abnormalities in a gene encoding a protein


termed dystrophin.

Dystrophin and the dystrophin-associated protein complex


form an interface between the intracellular contractile apparatus
and the extracellular connective tissue matrix. The role of this
complex of proteins is transferring the force of contraction to
connective tissue, so, myocyte degeneration occurs in the absence
of dystrophin.
Muscle biopsy specimens from patients with DMD show minimal evidence of
dystrophin by both staining and Western blot analysis. BMD patients have a
mutation causing a reduced amount of altered dystrophin.
Histopathologic abnormalities common to DMD and BMD
include
• Variation in fiber size due to the presence of both small and
enlarged fibers
• Increased numbers of internalized nuclei
• Degeneration, necrosis, and phagocytosis of muscle fibers
• Regeneration of muscle fibers
• Proliferation of endomysial connective tissue
• DMD cases also often show enlarged, rounded, hyaline fibers
that have lost their normal cross-striations, believed to be
hypercontracted fibers, this finding is rare in BMD. In later
stages, the muscles eventually become almost totally
replaced by fat and connective tissue. Cardiac involvement
consists of interstitial fibrosis, more prominent in the
subendocardial layers.
• Boys with DMD are normal at birth, and early motor milestones
are met on time. Walking is often delayed and weakness
begins in the pelvic girdle muscles and then extends to the
shoulder girdle giving rise to Gower’s sign (child uses his
hands to climb upon himself on getting up from the floor).
Enlargement of the calf muscles associated with weakness, a
phenomenon termed pseudohypertrophy caused initially by an
increase in the size of the muscle fibers later by an increase in
fat and connective tissue. Cognitive impairment is a
component of the disease. Serum creatine kinase is elevated
during the first decade of life but returns to normal in the later
stages of the disease, as muscle mass decreases. Death
results from respiratory insufficiency, pulmonary infection, and
cardiac decompensation.

• Boys with BMD develop symptoms at a later age than those


with DMD. The onset occurs in later childhood or in
adolescence, and it is accompanied by a slower and variable
rate of progression. Cardiac disease is frequently seen in
these patients.
1. A 10-year-old girl presents with a tibial mass.
Histopathological examination reveals a small round cell
tumor. Which of the following molecular findings is most
likely to be present?
(AII India 2012)
(a) 22q translocation
(b) 11q deletion
(c) 7p translocation
(d) n-myc amplification
2. Which of the following is the most specific test for
rheumatoid arthritis?
(AII India 2012)
(a) Anti Ig M antibody
(b) Anti CCP antibody
(c) Anti Ig A antibody
(d) Anti IgG antibody
3. Onion bulb appearance of nerve ending on biopsy is seen
in:
(AIIMS Nov. 2010)
(a) Diabetic neuropathy
(b) Amyloid neuropathy
(c) Leprous neuritis
(d) Chronic inflammatory demyelinating polyneuropathy
(CIDP)
4. The rate of newly synthesized osteoid mineralization is
best estimated by:
(AIIMS Nov. 2010)
(a) Tetracycline labeling
(b) Alizarine red staining
(c) Calceine stain
(d) Von Kossa stain
5. Which of the following is false in relation to
Osteosarcoma?
(DPG 2011)
(a) Paget’s disease and prior irradiation are pre-disposing
factors
(b) Rb gene mutation is associated with hereditary variant
(c) C-myc gene implicated in the genesis
(d) Codman’s triangle is the characteristic X-ray finding
6. Cytogenetics for synovial cell sarcoma is:
(AI 2008)
(a) t (X: 18)
(b) t (17, 9)
(c) t (9, 22)
(d) t (11, 14)
7. MIC-2 mutation associated with:
(AIIMS Nov 2009)
(a) Osteosarcoma
(b) Ewing’s sarcoma
(c) Alveolar soft tissue sarcoma
(d) Dermatofibrosarcoma protuberance
8. A 9-year-old girl has difficulty in combing hairs and
climbing upstairs since 6 months. She has Gower’s sign
positive and maculopapular rash over
metacarpophalangeal joints. What should be the next
appropriate investigation to be done?
(AIIMS Nov 2008)
(a) ESR
(b) RA factor
(c) Creatine kinase
(d) Electromyography
9. Antibody found in myositis is:
(AIIMS Nov 2008)
(a) Anti-Jo 1
(b) Anti scl 70
(c) Anti Sm
(d) Anti Ku
10. CD-99 is for:
(AIIMS May 2008)
(a) Ewing’s sarcoma
(b) SLL
(c) Dermatofibroma protruberans
(d) Malignant histiocytic fibroma
11. Bone resorption markers are all except:
(AIIMS May 2008)
(a) Tartarate resistant alkaline phosphatase (TRAP)
(b) Osteocalcin
(c) Crosslinked-N-telopeptides
(d) Urine total free deoxypyridinoline
12. A 50-year-old lady presented with a 3-month history of
pain in the lower third of the right thigh. There was no
local swelling; tenderness was present on deep
pressure. Plain X-rays showed an ill-defined
intramedullary lesion with blotchy calcification at the
lower end of the right femoral diaphysis, possibly
enchondroma or chondrosarcoma. Sections showed a
cartilaginous tumor. Which of the following histological
features (if seen) would be most helpful to differentiate
the two tumors?
(AIIMS May 2006)
(a) Focal necrosis and lobulation
(b) Tumor permeation between bone trabeculae at periphery
(c) Extensive myxoid change
(d) High cellularity
13. Dystrophic gene mutation leads to:
(AIIMS May 2003)
(a) Myasthenia gravis
(b) Motor neuron disease
(c) Poliomyelitis
(d) Duchenne’s muscular dystrophy
14. Giant cells are seen in:
(PGI Dec 2006)
(a) Osteoclastoma
(b) Chondroblastoma
(c) Chordoma
(d) Osteitis fibrosa cystica
15.“Biphasic pattern” on histology is seen in which tumor?
(Delhi 2010)
(a) Rhabdomyosarcoma
(b) Synovial cell sarcoma
(c) Osteosarcoma
(d) Neurofibroma
16.An epiphyseal bone lesion is: (Delhi
2009 RP)
(a) Osteogenic sarcoma
(b) Chondroblastoma
(c) Ewing’s sarcoma
(d) Chondromyxoid fibroma
17.Mosaic pattern of lamellar bone histology is found in:
(a) Osteopetrosis
(Delhi PG-2006)
(b) Osteoid osteoma
(c) Osteitis deformans
(d) Osteomalacia
18. Which one of the following inflammatory markers of
muscle biopsy is diagnostic of polymyositis?
(a) CD8/MHC-I complex
(Karnataka 2006)
(b) Vascular cell adhesion molecules
(c) Intracellular adhesion molecules
(d) Membrane attack complex
19. Bone tumor arising from epiphysis is:
(DNB 2001)
(a) Osteogenic sarcoma
(b) Ewing’s sarcoma
(c) Chondromyxoid fibroma
(d) Giant cell tumor
20. The commonest malignant bone tumor is:
(DNB 2001)
(a) Multiple myeloma
(b) Osteosarcoma
(c) Ewing’s sarcoma
(d) Giant cell tumor
21. Characteristics microscopic features of osteogenic
sarcoma is:
(UP 2000)
(a) Osteoid formation
(b) Osteoid formation by mesenchymal cells with
pleomorphism
(c) Codman’s triangle
(d) Predominant osteoclast
22. Ground glass appearance is found in:
(UP 2001)
(a) Inverted papilloma
(b) Fibro calcification
(c) Fibrous dysplasia of bones
(d) Chronic osteomyelitis

MOST RECENT QUESTIONS

23. Paget’s disease increases the risk of:


(a) Osteoma
(b) Osteosarcoma
(c) Fibrosarcoma
(d) All
24. Osteoclast are stimulated by:
(a) Thyroxine
(b) PTH
(c) Calcitonin
(d) Estrogen
25. Large intracytoplasmic glycogen storage is seen in which
malignancy?
(a) osteosarcoma
(b) Mesenchymal chondrosarcoma
(c) Ewing’s sarcoma
(d) Leiomyosarcoma
26. Osteoblastoma resembles histologically:
(a) Osteosarcoma
(b) Osteoid osteoma
(c) Chondroblastoma
(d) Chondrosarcoma
27. Syncytial osteoclastic giant cells are seen in All Except:
(a) Osteosarcoma
(b) Ewing’s sarcoma
(c) Chondroblastoma
(d) Aneurysmal bone cyst
28. Hyaline cartilage contains which type of collagen:
(a) Type I
(b) Type II
(c) Type III
(d) Type IV
29. Tophi in gout are found in all regions, except:
(a) Bone
(b) Skin
(c) Muscle
(d) Synovial membrane
30. Which of the following is true about psoriatic arthritis?
(a) Involves distal joints of hand and foot
(b) Pencil in cup deformity
(c) Sacroiliitis
(d) All of the above
31. Ewings sarcoma arises from:
(a) G cells
(b) Totipotent cells
(c) Neuroectodermal cells
(d) Neurons
32. Dystrophin is lacking in:
(a) Polio
(b) Duchenne’s muscular dystrophy
(c) Peroneal muscular atrophy
(d) Spinal muscular atrophy
33.A 42-year-old woman Paro presents with slowly
progressive syndrome comprising of features like pain
and tenderness in multiple joints, with joint stuffiness on
rising in the morning. Joint involvement is symmetric,
with the proximal interphalangeal and
metacarpophalangeal joints especially involved. The
physician finds presence of tenderness in all the
inflamed joints. Which of the following laboratory
abnormalities is most likely associated in this patient?
(a) Antibodies to double-stranded DNA
(b) IgM anti-IgG antibodies
(c) HLA-B27 antigen
(d) Urate crystals and neutrophils in synovial fluid
34. Most common malignant bone tumor:
(a) Osteogenic sarcoma
(b) Secondaries
(c) Osteoma
(d) Enchondroma
35. Paget disease of the bone is also called:
(a) Osteitis fibrosa
(b) Brittle bone disease
(c) Fibrous dysplasia
(d) Osteomalacia
36. In the giant cell tumor of the bone, the cell of origin is:
(a) Fibroblast cells
(b) Osteoclast and precursors
(c) Osteoblast and precursors
(d) Sinusoidal cells
37. Which of these is characteristic of Gout?
(a) Podagra
(b) Anasarca
(c) Cheiroarthropathy
(d) Calcinosis cutis
38. A 27-year-old male presents with low backache, that
occurs early in the morning, associated with stiffness,
and persists for more than 30 minutes. On examination,
his chest expansion is also restricted. The most probable
diagnosis is:
(a) Rheumatoid arthritis
(b) Oteoarthritis
(c) Gouty arthritis
(d) Ankylosing spondylitis
39. Polyarticular rheumatoid arthritis is diagnosed when
more than _ joints are involved?
(a) Two
(b) Three
(c) Four
(d) Five
40. All of the following are true about Paget’s disease of the
bone except:
(a) Association with virus
(b) Progress to chondrosarcoma
(c) May turn onto a malignant lesion
(e) Osteosclerotic phase seen
41. Which is rheumatoid arthritis autoantibody?
(a) Anti DLE
(b) Anti CCP
(c) Anti ds DNA
(d) Anti histone
42. Potts puffy tumor is:
(a) Osteomyelitis of frontal bone
(b) Osteomyelitis of ethmoid
(c) Osteomyelitis of maxilla
(d) Osteomyelitis of mandible
43. Fibrinoid necrosis is seen in:
(a) Diabetes
(b) Rheumatoid arthritis
(c) Pancreatitis
(d) Alzheimer’s disease
44 All is true about fibrous dysplasia except:
(a) Benign
(b) Trabeculae mimic Chinese characters
(c) Can be polyostotic
(d) Extramedulllary
45. Spindle cell tumour is:
(a) Leiomyoma
(b) Schwannoma
(c) Fibrous histiocytoma
(d) Alveolar soft tissue sarcoma
46. Colloid bodies are seen in:
(a) Psoriasis
(b) Lichen planus
(c) Leprosy
(d) Tuberculosis
47. Koenon tumor is seen in:
(a) NF
(b) Tuberous sclerosis
(c) Turners syndrome
(e) Sturge Weber syndrome
48. Secondary allograft rejection is
mediated by:
(a) Memory cells
(b) Antibodies
(c) Immune complexes
(d) None of the above
49. Dystrophin is absent:
(a) Duchenne muscular dystrophy
(b) Becker’s muscular dystrophy
(c) Myotonic dystrophy
(d) Limb-girdle dystrophy

ASSERTION AND REASON QUESTIONS


1–4. Will have two statements, assertion and reason. Read
both of them carefully and answer according to these
options.
(a) Both assertion and reason are true and reason is correct
explanation of assertion.
(b) Both assertion and reason are true and reason is not the
correct explanation of assertion.
(c) Assertion is true and reason is false.
(d) Both assertion and reason are false.
1. Assertion: Patients with Duchene’s muscular dystrophy have
difficulty in walking.
Reason: Altered dystrophin is responsible for muscular
weakness in muscular dystrophy.
2. Assertion: Osteosarcoma is associated with the radiological
appearance of Codman‘s triangle.
Reason: The tumor results in reactive periosteal bone formation.
3. Assertion: Great toe is the most commonly affected joint in
gout.
Reason: Uric acid is deposited in less temperature
4. Assertion: Reiter syndrome is a n example of seronegative
spondyloarthropathy
Reason: Reiter syndrome is associated with HLA B-27
1. Ans. (a) 22q translocation
(Ref: Robbins 8th/1232)
Ewing sarcoma and primitive neuroectodermal tumor are primary
malignant small round-cell tumors of bone and soft tissue.
Both differ in their degree of differentiation. PNETs
demonstrate neural differentiation whereas Ewing sarcomas
are undifferentiated. Analyzing some features of Ewing
sarcoma with the data in stem of the question:
• Arises in diaphysis and metaphysis (mass in the tibia
in question)
• Most patients are 10 to 15 years old (10 year old girl)
• Approximately 95% of patients with Ewing tumor have
t(11;22)Q (q24;q12) or t(21;22)Q (q22;q12)
• Microscopically there are sheets of small round cells
that contain glycogen.

Please note that the option ‘b’ (11q deletion) given in the question should
not be confused with the answer because in Ewing sarcoma we find 11q
translocation and not deletion

2. Ans. (b) Anti CCP antibody


(Ref: Robbins 8th/1237-1240, Harrison 17th/2088)
• Rheumatoid factor is an IgM antibody reactive with the
Fc portions of the patients’ own IgG. However, it is not
specific for rheumatoid arthritis as it can also be seen in
a wide range of autoimmune disorders, inflammatory
disease and chronic infection.
• Anti cyclic citrullinated peptide antibody (anti CCP
antibody) test is more specific than rheumatoid factor for
diagnosis of rheumatoid arthritis. It is more commonly
seen in the aggressive disease.
3. Ans. (d) Chronic inflammatory demyelinating
polyneuropathy (CIDP)
(Ref: Robbins 8th/1259)
• CIDP can result from altered triggering of T cells by antigen-presenting
cells.
• Most cases occur in adults, and males are affected slightly more
often than females.
• Onset is usually gradual, sometimes subacute; in a few, the initial
attack is indistinguishable from that of GBS. An acute-onset form of
CIDP should be considered when GBS deteriorates >9 weeks after
onset or relapses at least three times.
• Symptoms are both motor and sensory in most cases. Weakness of
the limbs is usually symmetric but can be strikingly asymmetric.
• Death from CIDP is uncommon.

• Biopsy typically reveals little inflammation and onion-bulb changes


(imbricated layers of attenuated Schwann cell processes surrounding
an axon) that result from recurrent demyelination and remyelination
• The diagnosis rests on characteristic clinical, CSF, and
electrophysiologic findings. The CSF is usually acellular with an
elevated protein level. Electrodiagnostically, variable degrees of
conduction slowing, prolonged distal latencies, temporal dispersion of
compound action potentials, and conduction block are the principal
features.
• Treatment: If the disorder is mild, management can be awaiting
spontaneous remission otherwise high-dose IVIg, PE, and
glucocorticoids are all effective.

4. Ans. (a) Tetracycline labeling


(Ref: Bancroft 6th/358)

Tetracycline is absorbed into bone and so, it is used as a marker of bone


growth for biopsies in humans. Tetracycline binds to newly formed bone
at the bone/osteoid (unmineralized bone) interface where it shows as a
linear fluorescence. Tetracycline labeling is used to determine the amount
of bone growth within a certain period of time, usually a period of
approximately 21 days. Tetracycline is incorporated into mineralizing bone
and can be detected by its fluorescence. In double tetracycline labeling, a
second dose is given 11–14 days after the first dose, and the amount of
bone formed during that interval can be calculated by measuring the
distance between the two fluorescent labels.

5. Ans. (c) C-myc gene implicated in the genesis


(Ref: Robbins 8th/1225)

• Osteosarcoma is defined as a malignant mesenchymal tumor in which


the cancerous cells produce bone matrix. It is the most common
primary malignant tumor of bone, exclusive of myeloma and
lymphoma.
• It has a bimodal age distribution; 75% occur in patients younger than
age 20. The smaller second peak occurs in the elderly, who frequently
suffer from conditions like Paget disease, bone infarcts, and prior
irradiation.
• The tumors usually arise in the metaphyseal region of the long bones
of the extremities, and almost 60% occur about the knee
• Genetic mutations seen with osteosarcoma are that of RB gene, p53,
CDK4, p16, INK4A, CYCLIN D1, and MDM2.
• The formation of bone by the tumor cells is characteristic of
Osteosarcoma

• Osteosarcoma typically present as painful and progressively enlarging


masses.
• Radiographs of the primary tumor usually show a large, destructive,
mixed lytic and blastic mass that has permeative margins. The tumor
frequently breaks through the cortex and lifts the periosteum, resulting
in reactive periosteal bone formation.
• The triangular shadow between the cortex and raised ends of
periosteum is known radiographically as Codman triangle

6. Ans. (a) t (X: 18)


(Ref: Robbins 7th/1323)
7. Ans. (b) Ewing’s sarcoma
(Ref: Harrison 17th/613)
8. Ans. (c) Creatine kinase
(Ref: Harrison 17th/2699)
• The diagnosis of the patient is most likely to be
dermatomyositis (DM) as suggested by proximal muscle
weakness (Gower’s sign positive) and skin rash.
• The clinical picture of the typical skin rash and proximal
or diffuse muscle weakness has few causes other than
DM. However, proximal muscle weakness without skin
involvement can be due to many conditions other than
PM or IBM.
DM usually occurs alone but may overlap with scleroderma and
mixed connective tissue disease. Fasciitis and thickening of
the skin, similar to that seen in chronic cases of DM, have
occurred in patients with the eosinophilia-myalgia syndrome
associated with the ingestion of contaminated L-tryptophan.

The CK level usually parallels disease activity, it can be normal in some


patients with active IBM or DM, especially when associated with a
connective tissue disease. The CK is always elevated in patients with
active PM. Along with the CK, the serum glutamic-oxaloacetic and
glutamate pyruvate transaminases, lactate dehydrogenase, and aldolase
may be elevated.

Muscle biopsy is the definitive test for establishing the diagnosis of


inflammatory myopathy and for excluding other
neuromuscular diseases. Inflammation is the histologic
hallmark for these diseases; however, additional features are
characteristic of each subtype.
As biopsy is not given in the options, we will mark CK as the
answer.
9. Ans. (a) Anti-Jo 1
(Ref: Harrison 17th/2038)
10. Ans. (a) Ewing’s sarcoma
(Ref: Harrison 17th/614, 615)

• CD 99 is a marker associated with the diagnosis of Ewing’s


sarcoma.
• Granulosa cell tumour is also often found to be associated with CD
99.
11. Ans. (b) Osteocalcin
(Ref: Harrison 17th/2368)
Osteocalcin is a protein secreted by osteoblasts. It is made
virtually only by osteoblasts. Thus, it is a bone formation
marker, i.e. osteoblastic marker.
Biochemical markers of bone Biochemical markers of bone
resorption resorption

a. Amino and carboxy terminal a. Amino and carboxy terminal


crosslinking telopeptide of crosslinking telopeptide of
bone collagen bone collagen
b. Pyridinoline b. Pyridinoline
c. Free Lysyl-pyridinoline c. Free Lysyl-pyridinoline
d. Tartarate-resistant acid d. Tartarate-resistant acid
phosphatase (TRAP) phosphatase (TRAP)
e. Hydroxyproline (not very e. Hydroxyproline (not very
specific) specific)

12. Ans. (b) Tumor permeation between bony trabeculae at


periphery (Ref:
Robbins 8th/1227-1230; Sternberg’s pathology 4th/276,
281)
• Chondromas are benign tumors of hyaline cartilage that
usually occurs in bones of endochondral origin. These
can be:
• Enchondroma: when origin is intramedullary
• Subperiosteal or juxtacortical: Origin from the surface of
bone
• Ollier disease is a syndrome of multiple enchondromas
(or enchondromatosis)
• Maffucis syndrome is associated of soft-tissue
hemangiomas with enchondromatosis.
• Enchondromas are composed of well-circumscribed
nodules of cytologically benign hyaline cartilage. Centre
of the nodule can calcify whereas peripheral portion may
undergo enchondral ossification.
• Chondrosarcoma – These are group of tumors that
produce neoplastic cartilage
• Similar to chondroma, chondrosarcoma can be:
– Intramedullary
– Juxtacortical
• Chondrosarcomas are second most common malignant
matrix-producing tumor of bone. (Most common is
osteosarcoma).
• Histologically, chondrosarcomas are composed of
malignant hyaline and myxoid cartilage. Spotty
calcifications may be present and central necrosis may
create cystic spaces. Tumors vary in cellularity and
cytological atypia.
• Malignant cartilage infiltrates the marrow space and
surrounds pre-existing bony trabeculae.
• Chondrosarcomas commonly arise in central portions of
skeleton (including pelvis, shoulder and ribs). In contrast
to chondroma, chondrosarcoma rarely involve the distal
extremities.
• Clear cell variant of chondrosarcoma characteristically
originate from epiphysis of tubular long bones.
13. Ans. (d) Duchenne’s muscular Dystrophy
(Ref: Robbins 8th/1268-1269)
14. Ans. (a) Osteoclastoma.
(Ref: Robbins 7th/1302)
Osteoclastoma (giant cell tumor) is so named because it contains
a profusion of multinucleated osteoclast-type giant cells. This
tumor is supposed to have a monocyte lineage and the giant
cells are believed to form via fusion of the mononuclear cells.
Other giant cell lesions include:

• Brown tumor seen in hyperparathyroidism


• Giant cell reparative granuloma
• Chondroblastoma
• Pigmented villonodular synovitis.

15. Ans. (b) Synovial cell sarcoma


(Ref: Harrison 17th/2184; Robbins 7th/1317)
Synovial cell sarcoma arise from primitive mesenchymal tissue that
differentiate into epithelial and spindle cells.
16. Ans. (b) Chondroblastoma
(Ref: Robbins 8th/1228)
• Chondroblastoma is a rare benign tumor occurring in
young patients in their teens with a male-to-female ratio
of 2:1.
• Most arise near the knee.
• Chondroblastoma has a striking predilection for
epiphyses and apophyses (epiphyseal equivalents, i.e.,
iliac crest). Chondroblastomas are usually painful.
• Radiographically, they produce a well-defined
geographic lucency that commonly has spotty
calcifications.
• The tumor cells are surrounded by scant amounts of
hyaline matrix that is deposited in a lacelike
configuration; nodules of well-formed hyaline cartilage
are distinctly uncommon.
• When the matrix calcifies, it produces a characteristic
chicken-wire pattern of mineralization.
Osteogenic sarcoma and chondromyxoid fibroma arises from
metaphysis whereas Ewing sarcoma arises from medullary
cavity.
17. Ans. (c) Osteitis deformans
(Ref: Robbins 7th/1285)
• The histologic hallmark in osteitis deformans (Paget’s
disease) is mosaic pattern of lamellar bone.
• This pattern which is likened to a jig saw puzzle, is
produced by prominent cement lines that anneal
haphazardly oriented units of lamellar bone.
18. Ans. (a) CD8/MHC-I complex
(Ref: Robbins 7th/1343)
Polymyositis is an example of inflammatory myopathy which is
characterized by the presence of symmetrical proximal
muscle weakness and elevated muscle enzymes. It is
diagnosed with the help of T cell infiltrates within the muscle
fascicles (endomysial involvement). The presence of
CD8/MHC I complex is required for the diagnosis of this
condition.

Note: It differs from dermatomyositis by the absence of rash (no


cutaneous involvement) and its occurrence mainly in the elderly. In DM,
there is presence of perifascicular atrophy (defined by the presence of
groups of atrophic fibers at the periphery of the fascicles).

In another inflammatory myopathy called inclusion body myositis,


there is presence of endomysial inflammation, basophilic
granular deposits around the edge of slit-like vacuoles
(rimmed vacuoles) and loss of fibers (being replaced with fat
cells and connective tissue). There is characteristically
presence of β amyloid deposits and cytochrome oxygenase
negative fibers are seen.
19. Ans. (d) Giant cell tumor
(Ref: Robbins 8th/1233)
20. Ans. (a) Multiple myeloma
(Ref: Robbins 8th/609-611)
21. Ans. (b) Osteoid formation by mesenchymal cells with
pleomorphism
(Ref: Robbins 8th/1226; 7th/1294-1296)
22. Ans. (c) Fibrous dysplasia of bones
(Ref: Robbins 8th/1231; 7th/1300-1301)
23. Ans. (b) Osteosarcoma
(Ref: Robbins 8th/1216)
24. Ans. (b) PTH
(Ref: Robbins 8th/1207-1208)
25. Ans. (c) Ewing’s sarcoma
(Ref: Robbins 8th/1232 ; 7th/1295,1298,1301)
26. Ans. (b) Osteoid osteoma
(Ref: Robbins 8th/1224)
27. Ans. (a) Osteosarcoma (Ref: Robbins 8th/1226,1228,1234)
28. Ans. (b) Type II
(Ref: Robbins 8th/1235)
29. Ans. (a) Bone
(Ref: Robbins 8th/1243-1244 ; 7th/1312)
30. Ans. (d) All of the above
(Ref: Robbins 8th/1241; 7th/1310)
31. Ans. (c) Neuroectodermal cells….see text for details ….
(Ref: Robbins 8/e p1232)
32. Ans. (b) Duchenne’s muscular dystrophy
(Ref: Robbins 8/e p1268)
Mnemonic: DMD
• Duchenne’s muscular dystrophy: Does not Make
Dystrophin
Mnemonic: BMD
• Becker’s muscular dystrophy: Badly Made Dystrophin
33. Ans. (b) IgM anti-IgG antibodies
(Ref: Robbins 8th/1238-9)
Symmetric polyarthritis with involvement of the proximal
interphalangeal and metacarpophalangeal joints in a female
patient are characteristics of rheumatoid arthritis. Rheumatoid
factor, an IgM antibody directed against the Fc portion of IgG,
is found in about 80% of affected individuals.
The most specific antibody for rheumatoid arthritis is anti-CCP
(cyclic citrullinated peptide) antibody.
34. Ans. (b) Secondaries…
(Ref: Robbins 8/e p1235)
Metastatic tumours are the most common form of skeletal
malignancy..(Ref: Robbins)
35. Ans. (a) Osteitis fibrosa
(Ref: Robbins 9/e p1189)
Paget disease is a disorder of increased, but disordered and
structurally unsound, bone mass.
36. Ans. (c) Osteoblast and precursors
(Ref: Robbins 9/e p1203)
The neoplastic cells of giant cell tumor are primitive osteoblast
precursors but they represent only a minority of the tumor
cells. The bulk of the tumor consists of non-neoplastic
osteoclasts and their precursors.
37. Ans. (a) Podagra
(Ref: Robbins 9/e p1216)
Podagra is the involvement of the great toe in a patient with gout.
As the book mentions; most first attacks are monoarticular;
50% occur in the first metatarsophalangeal joint.
38. Ans. (d) Ankylosing spondylitis
(Robbins 9th/1213)
Ankylosing spondylitis causes destruction of articular
cartilage and bony ankylosis, especially of the sacroiliac
and apophyseal joints (between tuberosities and
processes). It is also known as rheumatoid spondylitis and
Marie- Strümpell disease. Disease involving the sacroiliac
joints and vertebrae becomes symptomatic in the second
and third decades of life as lower back pain and spinal
immobility. Involvement of peripheral joints, such as the hips,
knees, and shoulders, occurs in at least one third of affected
individuals. Approximately 90% of patients are HLA-B27
positive; associations have also been found with the IL-23
receptor gene.
39. Ans. (d) Five
(Ref: Robbins 9/e p1209; Harrison 18/e p)
• Rheumatoid arthritis is characterized by is a chronic
inflammatory disorder of autoimmune origin that may
affect many tissues and organs but principally attacks
the joints, producing a non suppurative proliferative and
inflammatory synovitis. The earliest involved joints are
typically the small joints of the hands and feet.
• The initial pattern of joint involvement may be
monoarticular, oligoarticular (less than 4 joints), or poly-
articular (>5 joints), usually in a symmetric distribution.
40. Ans. (b) Progress to chondrosarcoma
(Ref: Robbins 9/e p1189-91)
Paget’s disease

• Paget disease is a disorder of locally increased but disordered bone.


• Typically asymptomatic, it is usually discovered incidentally.
• A mosaic pattern of mineralization is the histologic hallmark at the
late stage of the disease.
• Genetic and possibly viral infectious etiologies have been proposed.
• The most dreaded complication is sarcoma (osteosarcoma or
fibrosarcoma), which occurs in less than 1% of all individuals with
Paget disease, and in 5% to 10% of those with severe polyostotic
disease.

41. Ans. (b) Anti CCP


(Ref: Robbins 9/e p1210)
42. Ans. (a) Osteomyelitis of frontal bone
Pots puffy tumor is the osteomyelitis of frontal bone usually
associated with subperiosteal abscess.
43. Ans. (c) Rheumatoid arthritis
(Ref: Robbins 9/e p1211)
44. Ans. (d) Extramedulllary
(Ref: Robbins 9/e p1206)
45. Ans. (c) Fibrous histiocytoma
(Ref: Robbins 9/e p1158)
46. Ans. (b) Lichen planus
(Ref: Robbins 9/e p1167)
Lichen planus is characterized by the presence of colloid or
Civatte bodies.
47. Ans. (b) Tuberous sclerosis
(Ref: Robbins 9/e p1316)
48. Ans. (a) Memory cells
(Ref: Immunology/134)
49. Ans. (a) Duchenne muscular dystrophy
(Ref: Robbins 9/e p1242)
EXPLANATIONS TO ASSERTION AND
REASON QUESTIONS
Explanations (1-4): While solving assertion reason type
of questions, we can use a particular method.
1. First of all, read both assertion (A) and reason (R)
carefully and independently analyse whether they are true
or false.
2. If A is false, the answere will directly be (d) i.e. both A and
R are false. You can note that all other options (i.e. a, b or
c) consider A to be true.
3. If A is true, answer can be (a), (b) or (c), Now look at R. If
R is false, answer will be (c)
4. If both A and R are ture, then we have to know whether R
is correctly explaining A [answer is (a)] or it is not the
explanation of assertion [answer is (b)]
1. Ans. (b) Both assertion and reason are true and reason is
not the correct explanation of assertion.
(Ref: Robbins 8th/1268-9)
Patients with Duchene’s muscular dystrophy have difficulty in
walking. This is attributed to the absence of the skeletal
muscle contractile protein dystrophin in these patients.
Altered dystrophin is responsible for muscular weakness in
patients of Becker’s muscular dystrophy.

Mnemonic:
Duchenne Muscular Dystrophy (DMD) Doesn’t Make Dystrophin (no
formation)
Becker Muscular Dystrophy (BMD) Badly Made Dystrophin (reduced
formation of an altered protein)

2. Ans. (a) Both assertion and reason are true and reason is
correct explanation of assertion.
(Ref: Robbins 8th/1226)
Osteosarcoma is associated with the radiological appearance of
Codman‘s triangle because the tumor frequently breaks
through the cortex and lifts the periosteum, resulting in
reactive periosteal bone formation.

The formation of bone by the tumor cells is characteristic of


osteosarcoma.

3. Ans. (a) Both assertion and reason are true and reason is
correct explanation of assertion.
(Ref: Robbins 8th/1243)
• Great toe is the most commonly affected joint in gout
because uric acid gets supersaturated in the peripheral
joints (ankle and toes) especially so in the lower
temperatures.
4. Ans. (b) Both assertion and reason are true and reason is
not the correct explanation of assertion.
(Ref: Robbins 8th/1241)
The seronegative spondyloarthropathies include the following:
Mnemonic: PAIR

P : Psoriatic arthritis
A : Ankylosing spondylitis
I : Inflammatory bowel disease (Crohn’ disease and ulcerative colitis)
associated arthritis
R : Reiter syndrome

These are called seronegative because they are not associated


with specific autoantibodies although they are associated
with HLA B-27 as well as a triggering infection.

ANNEXURE
Some medically important autoantibodies:
Anti-actin antibodies coeliac disease, autoimmune hepatitis, gastric
cancer

Anti-ganglioside
antibodies

Anti-GD3 Guillain-Barré syndrome

Anti-GM1 Traveler’s diarrhea

Anti-GQ1b Miller-Fisher syndrome

Anti-glomerular Good pasture syndrome


basement membrane
antibody (Anti-GBM
antibody)

Anti-Hu antibody Small cell lung carcinoma

Anti-Jo 1 antibody Polymyositis


(anti-histidyl-tRNA
synthetase)

Anti-liver/kidney Autoimmune hepatitis


microsomal 1 antibody
(anti-LKM 1
antibodies)

Anti-mitochondrial Primary biliary cirrhosis


antibody

Anti-neutrophil Ulcerative colitis


cytoplasmic antibody
(ANCA)

Antinuclear antibody
(ANA)

Anti-p62 antibodies in Primary biliary cirrhosis


Anti-sp100 antibodies Primary biliary cirrhosis
in Primary biliary cirrhosis
Anti-glycoprotein210 SLE
antibodies in Anti-ds
DNA antibody

Anti-extractable
nuclear antigen
antibodies (Anti-ENA
antibodies)-

Anti-Ro antibody Sjögren syndrome

Anti-La antibody Sjögren syndrome

Anti-PM/Scl (anti- scleroderma + polymyositis/dermatomyositis.


exosome) antibody

Anti-Scl 70 antibody Sclerosis and scleroderma

Anti-topoisomerase
antibodies

Anti-transglutaminase
antibodies

Anti-centromere
antibodies

Anti-tTG Coeliac disease

Anti-eTG Dermatitis herpetiformis

IMMUNOHISTOLOGICAL MARKERS FOR SOME


CANCERS
CD-99 Ewing’s/PNET, ovarian granulose cell
tumors

LCA [CD-45] Lymphoma

CD15 Hodgkin’s lymphoma

Adenocarcinoma

Desmin Sarcoma

Vimentin Sarcoma

CD-31 Kaposi’s sarcoma

Angiosarcoma

Thyroid transcription Thyroid carcinoma, lung adenocarcinoma


factor-1

CD-68 and HAM 56 Malignant fibrous histiocytoma

CD117 or DOG-1 Gastrointestinal stromal tumors (GIST)

HMB-45 Melanoma

CD-103 Hairy cell leukemia


1. A middle-aged patient presented with a swelling in the
retroperitoneum. Detailed investigations lead to a biopsy
finding as shown below in the image. Further, molecular
studies demonstrated t(12;16). Which of the following is
the most likely diagnosis for this patient?
(NEET 2020 like pattern)

(a) Lipoma
(b) Myxoid liposarcoma
(c) Synovial sarcoma
(d) Pleomorphic sarcoma
Ans. (b) Myxoid liposarcoma
(Ref: Robbins 9th/1220)
Presence of a retroperitoneal swelling with t(12;16) is a
characteristic feature associated with myxoid liposarcoma.
Salient points about liposarcoma
• Liposarcoma is one of the most common sarcomas of adulthood.
• It occurs mainly in people in their 50s to 60s in the deep soft tissues of the
proximal extremities and in the retroperitoneum.
• Amplification of 12q13-q15 and t(12;16) are characteristic of well-
differentiated and myxoid liposarcomas, respectively.
• Histologically, well-differentiated liposarcoma contains adipocytes with
scattered atypical spindle cells whereas Myxoid liposarcoma contains
abundant basophilic extracellular matrix, arborizing capillaries and primitive
cells at various stages of adipocyte differentiation (as was given in the
image in the examination)

2. Which of the following is a most important infiltrate in


rheumatoid arthritis?
(AI 2018 Pattern)
(a) Dendritic cells
(b) Macrophages
(c) Neutrophils
(d) CD4+ Helper cells
Ans. (d) CD4+ Helper cells
(Ref: Robbins 9/e p1209)

3. Biopsy of a long bone was taken and the histology is


shown below. Which of the following is the likely
diagnosis?
(AI 2018 Pattern)
(a) Osteoporosis
(b) Osteosclerosis
(c) Osteomalacia
(d) Paget’s disease

Ans. (d) Paget’s disease


(Ref: Robbins 9/e p1189)
The histology is classical picture of the mosaic pattern which is
seen in patients with Paget’s disease of the bone.

4. Pathogenesis of myasthenia gravis is which of the


following?
(AI 2018 Pattern)
(a) Mutations in ryanodine receptors
(b) Auto antibodies against acetylcholine receptors
(c) Auto antibodies against synaptobrevin
(d) Auto antibodies against presynaptic calcium channels
Ans. (b) Auto antibodies against acetylcholine receptors
(Ref: Robbins 9/e p1235-6)
Disclaimer
Any resemblance to an actual question is purely coincidental.
1. SALIVARY GLAND TUMORS

About 65 to 80% of the salivary gland tumors arise within the


parotid, 10% in the submandibular gland, and the remainder in the
minor salivary glands, including the sublingual glands. The
likelihood of a salivary gland tumor being malignant is inversely
proportional to the size of the gland which means the tumors in
minor salivary glands are more likely to be malignant and those in
parotid are mostly benign.
These tumors usually occur in adults, with a slight female
predominance except Warthin tumor which occur more often in
males than in females.

Pleomorphic Adenoma or Mixed Tumors

• They are the most common benign tumors that are derived
from a mixture of ductal (epithelial) and myoepithelial cells, and
therefore they show both epithelial and mesenchymal
differentiation. About 60% of tumors in the parotid are mixed
tumors.
• Radiation exposure increases the risk. Most pleomorphic
adenomas present as rounded, well-demarcated masses rarely
exceeding 6 cm.
• The epithelial elements resemble ductal cells or myoepithelial
cells and are typically dispersed within a mesenchyme-like
background of loose myxoid tissue containing islands of
chondroid and, rarely, foci of bone.
• Tumors present as painless, slow-growing, mobile discrete
masses.
• A carcinoma arising in a pleomorphic adenoma is referred to as
a carcinoma ex pleomorphic adenoma or a malignant mixed
tumor. The incidence of malignant transformation increases
with the duration of the tumor.

Warthin’s Tumor (Papillary Cystadenoma Lymphomatosum)

• It is the second most common benign salivary gland neoplasm.


It arises almost always in the parotid gland and occurs more
commonly in males than in females, usually in the fifth to
seventh decades of life. About 10% are multifocal and 10%
bilateral. It is more common in smokers.
• Most Warthin’s tumors are round to oval, encapsulated masses,
2 to 5 cm in diameter.
• On microscopic examination, the cystic spaces are lined by a
double layer of neoplastic epithelial cells resting on a dense
lymphoid stroma. The double layer of lining cells is
distinctive, with a surface palisade of columnar cells having
an abundant, finely granular, eosinophilic cytoplasm, imparting
an oncocytic appearance, resting on a layer of cuboidal to
polygonal cells.

Mucoepidermoid Carcinoma

They occur mainly (60 to 70%) in the parotids. They are the most
common form of primary malignant tumor of the salivary
glands. Mucoepidermoid carcinomas can grow up to 8 cm in
diameter, lack well-defined capsules and are often infiltrative at the
margins. It is associated with a balanced translocation, t(11;19)
producing a fusion gene made-up of MECT1 and MAML2.
The basic histologic pattern is that of cords, sheets, or cystic
configurations of squamous, mucous, or intermediate cells. The
hybrid cell types often have squamous features, with small to large
mucus-filled vacuoles, best seen with mucin stains.
Two other salivary gland tumors include: Adenoid cystic
carcinoma and acinic cell tumor.
Adenoid cystic carcinoma is a relatively uncommon tumor. In 50% of
cases, it is found in the minor salivary glands (in particular; the palate).
These tumors have a tendency to invade perineural spaces. They
have high chances of recurrence and eventually, 50% or more
disseminate widely to distant sites such as bone, liver, and brain.

The acinic cell tumor is composed of cells resembling the


normal serous acinar cells of salivary glands. Most arise in the
parotids and the small remainder arises in the submandibular
glands. They rarely involve the minor glands. Most
characteristically, the cells have apparently clear cytoplasm, but
the cells are sometimes solid or at other times vacuolated. The
cells are disposed in sheets or microcystic, glandular, follicular, or
papillary patterns. There is usually little anaplasia and few mitoses.
On histologic evaluation, they are composed of small cells having
dark, compact nuclei and scant cytoplasm. These cells tend to be
disposed in tubular, solid, or cribriform patterns. The spaces
between the tumor cells are often filled with a hyaline material
thought to represent excess basement membrane.

2. OTHER IMPORTANT TUMORS

a. Neuroblastoma

It is the most common extracranial solid tumor of childhood


and the most frequently diagnosed tumor of infancy.
• Most cases are sporadic. Familial cases (1-2%) result from
germline mutation in anaplastic lymphoma kinase (ALK) gene.
• Most common site of origin is adrenal medulla (40%) followed
by para-vertebral sympathetic chain in abdomen (25%) and
posterior mediastinum (15%).
• Adrenal neuroblastomas are malignant neoplasms arising from
the sympathetic neuroblasts in the medulla of the adrenal
gland. There are two clinical types, based on the differences in
distribution of metastasis. First (Pepper type) occurs in the
stillborn and in young infants and metastasizes to the liver and
regional lymph nodes, then the lungs, and late in the course,
the calvarium and other flat bones. The second (Hutchinson)
type is characterized clinically by secondary growth in the orbit,
meninges, skull and long bones and occurs in children up to 15
years of age.
• Histologically, it is one of the small round blue cell tumor. It
shows central space filled with eosinophilic fibrillary material
called neuropil surrounded by concentrically arranged tumor
cells (Homer-Wright pseudorosettes).

Fig. 1: Neuroblastoma showing Homer-Wright pseudorosettes.

Note: Other small round cell tumors can be remembered as (Sofia


LOREN)
Sophia Small round cell tumors
L Lymphoma
O Oat cell carcinoma
R Rhabdomyosarcoma
E Ewing’s sarcoma
N Neuroblastoma

• Tumor cells are positive for neuron specific enolase and contain
dense core granules.
• Maturation of some of the cells (to form ganglion cells) along
with presence of primitive neuroblasts is called
ganglioneuroblastoma. Even better differentiation with few
neuroblasts is designated ganglioneuroma.
• Only presence of ganglion cells is not enough for designation of
maturation. Presence of Schwannian stroma, mature
Schwann cells and fibroblasts is a histologic pre-requisite
for the designation of ganglioneuroblastoma and
ganglioneuroma.
• Metastasis develops early and widely. Hematogenous spread
may occur to liver, lungs, bone marrow and bones.
• About 60-80% children present with stage 3 or 4 tumors.
• Apart from stage 1, 2, 3 and 4, stage 4S (special stage) is
present in neuroblastoma. It signifies localized primary tumor
with dissemination limited to skin, liver and/or bone marrow.
Stage 4S is limited to infants < 1 year.

Fig. 2: Ganglioneuroma.

Prognostic Factors

Factor Favorable Unfavorable

1. Stage 1, 2A, 2B, 4S 3, 4

2. Age <18 months >18 months

3. Histology Present Absent


Schwannian stroma Present Absent
Gangliocytic differentiation Low High
Mitotic rate <200/5000 cells (≤ >200/5000 (> 4%)
Mitotic karyorrhexis index 4%) Absent
Intramural calcification Present

4. DNA Ploidy Hyperdiploid or Near diploid


near triploid

5. N-Myc Not amplified Amplified

6. Genetics Absent Present


17q gain Absent Present
1p loss Absent Present
11q loss Present Absent
TRKA expression Absent Present
TRKB expression Absent Present
MRP expression Present Absent
CD44 expression Absent Highly expressed
Telomerase expression

Note:
• Age for prognosis in 7th edition of Robbins was 1 year, in 8th edition,
this has been changed to 18 months.
• Chromosome 11q loss, TRKB expression are included in prognostic
factors in 8th edition of Robbins.

b. Retinoblastoma

• Retinoblastoma is the most common malignant eye tumor of


childhood.
• 90% cases are diagnosed before 7 years. Hereditary
retinoblastoma when affecting both eyes is called bilateral
retinoblastoma.
• Bilateral retinoblastoma + pineal gland tumor in suprasellar or
parasellar region is called Trilateral retinoblastoma
• These tumors arise from neuroepithelium of the retina
– A characteristic feature is Flexner Wintersteiner rosettes
consisting of clusters of cuboidal or short columnar cells
arranged around a central lumen. The nuclei are displaced
away from the lumen and photoreceptor like elements are
produced from it. Retinoblastoma also shows Homer
Wright rosettes which are radial arrangement of cells
around a central tangle of fibrils.
• Fleurettes representing photoreceptor differentiation of tumor
cells is also seen.

THYMOMA

• The thymus is derived from the third and fourth pharyngeal


pouches and is located in the anterior mediastinum. The
thymus is composed of epithelial and stromal cells and
lymphoid precursors.
• If a lymphoid cell within the thymus becomes neoplastic,
the disease that develops is a lymphoma which is a T cell
lymphoblastic lymphomas in most of the cases. If the
epithelial cells of the thymus become neoplastic, the
tumor that develops is a thymoma.
• Thymoma is the most common cause of an anterior
mediastinal mass in adults. Thymomas are most common in
the fifth and sixth decade. Majority (90%) of thymomas are in
the anterior mediastinum. Thymomas are epithelial tumors and
all of them have malignant potential. They may have a variable
percentage of lymphocytes within the tumor. The epithelial
component of the tumor may consist primarily of round or oval
cells derived mainly from the cortex or spindle-shaped cells
derived mainly from the medulla or combinations thereof.
• These tumors present clinically in 40% patients as causing
symptoms due to compression on the mediastinal structures or
due to their association with myasthenia gravis. In addition to
myasthenia gravis, other paraneoplastic syndromes, such as
acquired hypogammaglobulinemia, pure red cell aplasia,
Graves disease, pernicious anemia, dermatomyositis-
polymyositis, and Cushing syndrome, can be seen.
• Epstein-Barr virus may be associated with thymomas.

WHO Histologic Classification of Thymus Tumors


Type Histologic Description
A Medullary thymoma
AB Mixed thymoma
B1 Predominantly cortical thymoma
B2 Cortical thymoma
B3 Well-differentiated thymic carcinoma
C Thymic carcinoma

The genetic lesions in thymomas are not well characterized.


Some data suggest that Epstein-Barr virus may be associated with
thymomas.

3. SCLERODERMA

There are three major forms of scleroderma: diffuse, limited


(CREST syndrome) and morphea/linear. Diffuse and limited
scleroderma are both a systemic disease, whereas the
linear/morphea form is localized to the skin.

Diffuse Scleroderma
Diffuse scleroderma (progressive systemic sclerosis) is the most
severe form. It has a rapid onset, involves more widespread skin
hardening, will generally cause much internal organ damage
(especifically the lungs and gastrointestinal tract), and is generally
more life-threatening.
Limited Scleroderma/CREST Syndrome

The limited form is much milder. It has a slow onset and


progression. Skin hardening is usually confined to the hands and
face, internal organ involvement is less severe, and a much better
prognosis is expected. In typical cases of limited scleroderma,
Raynaud’s phenomenon may precede scleroderma by several
years. The scleroderma may be limited to the fingers—known as
sclerodactyly.
The limited form is often referred to as CREST syndrome
characterized by:

• Calcinosis
• Raynaud’s syndrome
• Esophageal dysmotility
• Sclerodactyly
• Telangiectasia
CREST is a limited form associated with antibodies against
centromeres and usually spares the lungs and kidneys.

Morphea/Linear Scleroderma
Morphea/linear scleroderma involves isolated patches of hardened
skin. There generally is no internal organ involvement.
Diagnosis is by clinical suspicion, presence of autoantibodies
(especifically anti-centromere and anti-scl70/anti-topoisomerase
antibodies) and occasionally by biopsy. Of the antibodies, 90%
have a detectable anti-nuclear antibody. Anti-centromere antibody
is more common in the limited form (80-90%) than in the systemic
form (10%), and anti-scl70 is more common in the diffuse form (30-
40%).
1. The immune-cytochemical feature of Langerhans cell
histiocytosis is positivity for which of the following?
(a) CD1a
(AI 2012)
(b) CD99 (mic-2)
(c) HMB-45
(d) CD117
2. In the congenital dystrophic variety of epidermolysis
bullosa, mutation is seen in the gene coding for
(a) Laminin 4
(AI 2012)
(b) Collagen type 7
(c) Alpha 6 integerin
(d) Keratin 14
3. The marker for Langerhans cell histiocytosis is:
(a) CD 1a
(AI 2010)
(b) CD 20
(c) CD 3
(d) CD 30
4. For examination of fungus from a sample, uniformly stain
by:
(AI 2010)
(a) Alizarin
(b) PAS
(c) MassonTrichome
(d) Giemsa
5. All are components of photoreceptor matrix, except:
(a) MMP
(AI 2008)
(b) TIMP
(c) SPARC
(d) MIMECAN
6. Acinic cell carcinomas of the salivary gland arise most
often in the:
(AI 2006)
(a) Parotid salivary gland
(b) Minor salivary glands
(c) Submandibular salivary gland
(d) Sublingual salivary gland
7.In familial Mediterranean fever, the gene encoding the
following protein undergoes mutation
(AI 2005)
(a) Pyrin
(b) Perforin
(c) Atrial natriuretic factor
(d) Immunoglobulin light chain
8. To which of the following events is ‘good’ outcome in
Neuroblastoma associated
(AI 2004)
(a) Diploidy
(b) N-myc amplification
(c) Chromosome/p depletion
(d) Trk A expression
9. Splenic macrophages in Gaucher’s disease differ from
those in ceroid histiocytosis by staining positive for:
(a) Lipids
(AI 2004)
(b) Phospholipids
(c) Acid-fast stain
(d) Iron
10. A 70 years old male who has been chewing tobacco for
the past 50 years presents with a six months history of a
large, fungating, soft papillary lesions in the oral cavity.
The lesion has penetrated into the mandible. Lymph
nodes are not palpable. Two biopsies take from the
lesion proper show benign appearing papillomatosis
with hyperkeratosis and acanthosis infiltrating the
subjacent tissues. The most likely diagnosis is:
(a) Squamous cell papilloma
(AI 2004)
(b) Squamous cell carcinoma
(c) Verrucous carcinoma
(d) Malignant mixed tumour
11. Hereditary retinoblastoma develop the following
chromosomal deletion:
(AI 2003)
(a) 13q14
(b) 11p13
(c) 14q13
(d) 22q11
12. All the statement about lactoferin are true, except:
(a)It is present in secondary granules of neutrophil
(b) It is present in exocrine secretions of body
(AI 2003)
(c) It has great affinity for iron
(d) It transports iron for erythropoiesis
13. Protein involved in intercellular connections is:
(a) Connexins
(AI 2001)
(b) Integrins
(c) Adhesins
(d) None of the above
14. Which of the following stains is used to detect lipid in
frozen section biopsy in histopathology laboratory?
(a) PAS
(AIIMS Nov 2009)
(b) Oil Red O
(c) NSE
(d) Silver Methanamine
15. Young boy presented with multiple flaccid bullae and oral
lesions. Diagnostic finding in skin biopsy
immunofluorescence test would be:
(AIIMS Nov 2009)
(a) Fish net IgG in dermoepidermal junction
(b) Linear IgG in dermoepidermal junction
(c) Linear IgG in dermal papillae
(d) Granular IgA in reticular dermis
16. A 14 years old girl on exposure to cold develop pallor of
extremities followed by pain and cyanosis. In later ages
of life she is prone to develop?
(AIIMS Nov 2008)
(a) Systemic lupus erythematosus
(b) Scleroderma
(c) Rheumatoid arthritis
(d) Histiocytosis
17. Which is false about acrodermatitis? enteropathica?
(AIIMS Nov 2008)
(a) Triad of diarrhea, dementia and dermatitis
(b) Low serum zinc levels
(c) Symptoms improve with zinc supplementation
(d) Autosomal recessive
18.Which of the following statement is incorrect?
(AIIMS Nov 2008)
(a) Selenium deficiency causes cardiomyopathy
(b) Zinc deficiency causes pulmonary fibrosis
(c) Increased calcium intake cause iron deficiency
(d) Vitamin A deficiency occurs after 6 months to 1year
of low vitamin A diet
19.A patient presents with mediastinal mass with sheets of
epithelial cells giving arborizing pattern of keratin
reactivity along with interspersed lymphoid cells. The apt
diagnosis would be:
(AIIMS May 2008)
(a) Thymoma
(b) Thymic carcinoid
(c) Primary mediastinal lymphoma
(d) Non-Hodgkin lymphoma
20. Ultrastructural finding in case of Paraganglioma:
(a) Deposition of glycogen
(AIIMS May 2008)
(b) Enlarged mitochondria
(c) Shrunken mitochondria
(d) Dense core granules
21. Brain natriuretic peptide is degraded by:
(a) Neutral endopeptidase
(AIIMS May 2007)
(b) Elastase
(c) Collagenase
(d) Ompatrilat
22. Why fetal cells continue to divide but terminally
differentiated adult cells do not divide:
(AIIMS Nov 2006)
(a) There are many cyclin inhibitors which prevent cell to
enter into S phase in adult
(b) Phosphatase absent in fetal cells
(c) Proteinase is absent in fetus
(d) Absence of CD kinase
23. All of the following are examples of a round cell tumor,
except:
(AIIMS Nov 2005)
(a) Neuroblastoma
(b) Ewing’s sarcoma
(c) Non-Hodgkin’s lymphoma
(d) Osteosarcoma
24. The tissue of origin of the Kaposi’s sarcoma is:
(a) Lymphoid
(c) Neural
(AIIMS May 2005)
(b) Vascular
(d) Muscular
25. “Tophus” is the pathognomic lesion of which of the
following condition:
(AIIMS May 2003)
(a) Multiple myeloma
(b) Cystinosis
(c) Gout
(d) Eale’s disease
26. Which of the following diseases have an underlying
mitochondrial abnormality? (PGI Dec 01)
(a) Krabbe’s disease
(b) Fabry’s disease
(c) Mitochondrial myopathy
(d) Oncocytoma
(e) Fanconi’s syndrome
27. Foam cells seen in:
(PGI Dec 2005)
(a) Alport’s syndrome
(b) Niemann-Pick disease
(c) Atherosclerosis
(d) Pneumonia
28.Which among the following is the best tissue fixative?
(a) Formalin
(Delhi PG-2007)
(b) Alcohol
(c) Normal saline
(d) Methylene blue
29. All of the following are forms of panniculitis except:
(Delhi PG-2006)
(a) Weber-Christian disease
(b) Erythema induratum
(c) Erythema nodosum
(d) All of the above
30. Warthin-Finkeldey cells are seen in:
(a) Measles
(b) Rubella
(c) Influenza
(d) Rickettsial pox
31. Pathogenesis is sequence of events in response to:
(Delhi PG-2004)
(a) Expression of disease upto clinical manifestation
(b) Expression of disease upto non-clinical manifestation
(c) The etiological agent for the initial stimulus to the
ultimate expression of disease
(d) None
32. All of the following characteristics are true of liposarcoma
except that it:
(Karnataka 2009)
(a) Is commonly found in the retroperitoneum
(b) Frequently gives rise to embolization in lymphatics
(c) Is the most common soft tissue sarcoma
(d) Arises very rarely in subcutaneous tissue
33. All of the following are correctly matched except:
(a) Russell bodies—Multiple myeloma
(Karnataka 2008)
(b) Russell bodies—Alcoholic liver disease
(c) Michaelis Gutmann bodies—Langerhans histio-cytosis
(d) Civatte bodies—Lichen planus
34. Most common second malignancy in patients with
familial retinoblastoma is:
(Karnataka 2004)
(a) Teratoma
(b) Medullary carcinoma
(c) Osteosarcoma
(d) Malignant melanoma
35. Hutchison’s secondaries in skull are due to tumors in:
(a) Lung
(DNB-2000, 2003)
(b) Breast
(c) Liver
(d) Adrenals
(e) Testes
36. Rosette shaped arrangement of cells are seen in:
(a) Thecoma of ovary
(DNB- 2000, 2006, 2007)
(b) Ependymoma
(c) Neurofibroma
(d) Lymphoma
37. Spontaneous regression though rare is seen in:
(a) Burkitt’s lymphoma
(DNB- 2000)
(b) Wilms’ tumor
(c) Neuroblastoma
(d) Melanoma
38. Perioral pallor and Dennie’s lines are seen in:
(a) Atopic dermatitis
(DNB- 2008)
(b) Chronic actinic dermatitis
(c) Blood dyscrasias
(d) Perioral contact dermatitis
39. Most common tumor of parotid gland is:
(a) Pleomorphic adenoma
(UP 2001)
(b) Warthin’s adenoma
(c) Mucoepidermoid carcinoma
(d) Mixed tumor
40. MC malignant tumor of parotid glands is:
(a) Pleomorphic adenoma
(UP 2001)
(b) Mucoepidermoid carcinoma
(c) Warthin’s tumor
(d) Mixed tumor of salivary gland
41. Punctate basophilia is found in:
(UP 2001)
(a) DDT poisoning
(b) Mercury vapors inhalation
(c) Cyanide poisoning
(d) Lead poisoning
42. Epulis is
(UP-98, 2004)
(a) Tumor of gingiva
(b) Tumor of enamel of tooth
(c) Disarrangement of tooth
(d) Dysplastic leukoplakia
43. Most common tumor of infancy is
(UP 2005)
(a) Lymphangioma
(b) Rhabdomyoma
(c) Hemangioma
(d) Lipoma
44. Triad of biotin deficiency is
(UP 2005)
(a) Dermatitis, glossitis, steatorrhea
(b) Dermatitis, glossitis, alopecia
(c) Mental changes, diarrhea, alopecia
(d) Dermatitis, dementia, diarrhea
45. Basophilic stippling is seen in:
(UP 2006)
(a) Cadmium poisoning
(b) Lead poisoning
(c) Chromium poisoning
(d) Iron poisoning
46. Most common tumor of infancy is
(UP 2005, 2007)
(a) Lymphangioma
(b) Rhabdomyoma
(c) Hemangioma
(d) Lipoma
47. Pleomorphic adenoma usually arises from
(a) Parotid gland
(UP 2007)
(b) Submandibular gland
(c) Minor salivary gland
(d) Superficial lobe
48. Direct Coomb’s test detects:
(UP 2008)
(a) Antigen in serum
(b) Antibodies on RBC surface
(c) Antigen on RBC surface
(d) Antibodies in serum
49. In vitamin deficiencies, patient is vulnerable to infection
with:
(RJ 2000)
(a) Measles
(b) Mumps
(c) Rubella
(d) Whooping cough
50. Paralytic food poisoning is caused by:
(RJ 2000)
(a) Staphylococci
(b) E. coli
(c) B. cereus
(d) Clostridia
51. Which is not present in anterior mediastinum?
(a) Lymphoma
(RJ 2000)
(b) Thymoma
(c) Teratoma
(d) Neurofibroma
52. Nonbacterial verrucous endocarditis is associated with
(a) Rheumatic carditis
(RJ 2001)
(b) Rheumatoid arthritis
(c) SLE
(d) Infective endocarditis
53. Frozen section biopsy in not used for:
(Bihar 2005)
(a) Enzyme
(b) Amyloid
(c) Fat
(d) Proteins
54. Rodent ulcer is due to:
(RJ 2002)
(a) Syphilis
(b) Burns
(c) Basal cell carcinoma
(d) TB

MOST RECENT QUESTIONS


55. Most common salivary gland tumor in adult is:
(a) Mucoepidermoid carcinoma
(b) Lymphoma
(c) Pleomorphic adenoma
(d) None
56. Soft chancre is caused by:
(a) Syphilis
(b) TB
(c) Chancroid
(d) L. donovani
57. Kobner’s phenomena is seen in:
(a) Psoriasis
(b) Lichen planus
(c) Toxic epidermal necrolysis
(d) All
58. Pellagra is characterized by all except:
(a) Diarrhea
(b) Dementia
(c) Dermatitis
(d) Diplopia
59. Smoking causes all cancers except:
(a) Liver
(b) Pancreas
(c) Bladder
(d) Lung
60. Endothelial cells have:
(a) Weibel-Palade bodies
(b) Gamma-Gandy bodies
(c) Both
(d) None
61. Which one of the following conditions is NOT associated
with occurrence of pellagra?
(a) People eating mainly corn-based diet
(b) Carcinoid syndrome
(c) Phototherapy
(d) Hartnup disease
1. Ans. (a) CD1a
(Ref: Robbins 8th/631-2)
Entities in Langerhans cell histiocytosis

• Letterer-Siwe syndrome (multifocal multisystem LCH)


• Pulmonary Langerhans’ cell histiocytosis: seen in adult smokers and
can regress on cessation of smoking.
• Eosinophilic granuloma.

The tumor cells in Langerhans cell histiocytosis express HLA-


DR, S-100Q, and CD1aQ.
2. Ans. (b) Collagen type 7
(Ref: Robbins 8th/1196)
Epidermolysis bullosa are a group of non inflammatory disorders
caused by defects in structural proteins which lend stability to
the skin. It can be of the following types:
• Simplex type: defect in basal layer of epidermis due to
mutation in gene for keratin 14 or 5
• Junctional type: blisters occur at the level of lamina
lucida
• Dystrophic type: blisters beneath the lamina densa
due to defect in COL7A1 gene for collagen type VIIQ
Clinical importance

Squamous cell cancersQ can arise in these chronic blisters.


Non-Herlitz junctional epidermolysis bullosa is caused by defect in LAMB3
gene encoding laminin Vβ3.
3. Ans. (a) CD 1a
(Ref: Robbins 8th/631-632)
Langerhans cell histiocytosis (LCH) has the following entities:

• The tumor cells express HLA-DR, S-100, and CD1a.


• The presence of Birbeck’s granules in the cytoplasm is characteristic
which have a tennis-racket appearance under the electron
microscope.

4. Ans. (b) PAS


(Ref: Robbins 8th/336, Harsh Mohan 6th/12-13)
• PAS (Periodic Acid Schiff) stain is used for
Carbohydrates particularly glycogen and all mucins,
amoebae and fungi
• Other frequently asked stains:
Stains Substance
Congo red with polarizing light Amyloid
Ziehl Neilson stain Tubercle bacilli
Masson’s Trichrome Extracellular collagen
Perl’s stain Hemosiderin, iron
Masson-Fontana Melanin, argentaffin cells
Alizarin calcium
Feulgen reaction DNA
Giemsa Campylobacter, leishmaniae,
malaria parasites

• Silver methanamine is a better stain for fungi and stains


Pneumocystis and the fungi black in color.

• Mucicarmine is for staining cryptococci


5. Ans. (d) MIMECAN
(Ref: Robbins 7th/105, 109-111; Retina; Stephen J.Ryan
4th/140)
• Ryan says the retinal pigment epithelial (RPE) cells
actively synthesize and degrade extracellular matrix
(ECM) components. Deposition of ECM molecules is
polarized with different components secreted apically
and basally. The apical domain of the RPE cells is
embedded in the interphotoreceptor matrix which is
produced by RPE and inner segments of the
photoreceptors. Degradation of ECM is regulated by the
equilibrium between matrix metalloproteinases [(MMP)
and their tissue inhibitors (TIMPs)].
• The normal RPE expresses membrane bound type I
(MT1-MMP) and type 2 (MT2 MMP) metalloproteinase
as well as the metalloproteinase inhibitors TIMP-1 and
TIMP-3. TIMP-3 accumulates in Brusch membrane and
is seen in age related macular degeneration (ARMD).
• Patients with SPARC (ostionectin) (Secreted Protein
Acidic and Rich in Cysteine) contributes to tissue
remodeling in response to injury and functions as an
angiogenesis inhibitor. SPACR (sialyprotein associated
with cones and rodes) is a glycoprotein identified in
human interphotoreceptor matrix.
• MIMECAN is a member of small leucine rich
proteoglycans (SLRP) gene family. They are essential
for normal collagen fibrillogenesis in various connective
tissues like cornea. It is not present in photorceptor
matrix. It is also known as osteoglycin.
6. Ans. (a) Parotid salivary gland
(Ref: Robbins 7th/794)
• Acinic cell tumors of salivary glands are uncommon
tumors representing 2 to 3% of salivary gland tumors.
• These are composed of cells resembling the normal
serous acinar cells of salivary glands.
• Most of these arise in the parotids. The remainder arises
in submandibular glands.
• Most parotid tumors are benign but half of
submandibular and sublingual and most minor salivary
gland tumors are malignant.
7. Ans. (a) Pyrin (Ref: Robbins 7th/261, Harrison 17th/2144)
8. Ans. (d) Trk A expression
(Ref: Robbins 7th/503)
• Neuroblastoma has good prognosis in infants (< 1 year
old) regardless of the stage. These tumors are
hyperdiploid or near triploid.
Prognostic factors in Neuroblastoma
• Age and stage: Good prognosis in infants regardless of
stage. In children > 1 year, stage III and IV poor
prognosis as compared to stage I or II.
• Genetics: Hyperdiploid or near triploid and high
expression of Trk-A have good prognosis whereas near-
diploidly, deletion of chromosome 1p or 14, gain of
chromosome 17q and N-myc amplification is associated
with unfavorable outcome.
• 3. Tum
or markers: Telomerase and MRP expression has poor
prognosis whereas CD 44 expression associated with
good prognosis.
• Histology: Differentiation (into Schwann cells and
gangliocytes), low mitotic rate and intramural
calcification has good prognosis.

Note: Most characteristic cytogenetic abnormality in neuroblastoma is 1p


deletion.

9. Ans. (a) Lipids


(Ref: Harrison 17th/2548, 2455)
• Ceroid histiocytosis also known as neuronal
cerebrolipofuschinosis is a group of diseases where
lipofuschin, a yellow brown cytoplasmic pigment is
deposited in the neurons. Lipofuschin is a lipid.
• In Gaucher’s disease, all patients have a nonuniform
infiltration of bone marrow by lipid laden macrophages
termed Gaucher cells’s.
Thus both these disorders have ‘lipids’ in the macrophages.
GAUCHER’S DISEASE
• It is autosomal recessive lysosomal storage disorder due
to deficiency of enzyme α-glucosidase resulting in
accumulation of glucosylceramide.
• Decreased activity (0-20%) of a-glucosidase in nucleated
cells is required for diagnosis.
• Type 1 disease do not involve CNS and present as
hepatosplenomegaly with skeletal dysplasia whereas
Type 2 Gaucher’s disease is a severe CNS disease
leading to death by 2 year of age. Type 3 disease has
highly variable manifestations in CNS and viscera.
10. Ans. (c) Verrucous carcinoma
(Ref: Robbins 7th/1037; Ackerman’s surgical pathology
8th/235)
Verrucous carcinomas also referred to as giant condyloma
accuminatum or Buschke-Lowenstein tumour is considered
an intermediate lesion between condyloma acuminata and
invasive squamous cell carcinoma. It is important to
distinguish verrucuous carcinomas from squamous cell
carcinoma as these tend to remain localized and are cured
by wide excision, however they may undergo malignant
transformation to invasive squamous cell carcinomas.
Features of verrucuous carcinomas
• Predilection for males > 50 years
• Predisposed in tobacco users, poor oral hygiene
• Grossly, it is a soft, large, wart like (papillomatous)
lesion which may show fungation
• Microscopically:
– Cytological features of malignancy are absent or
minimal and rare
– Epithelium is thickened and thrown into papillary
folds
– The folds project both above and below the level of
surrounding mucosa and crypt like surface grooves
exhibit marked, pre-keratin plugging.
– The deep border of epithelial projections is
‘pushing’ and not infiltrative.
11. Ans. (a) 13q14
(Ref: Harrison’s 17th/413)
The term contiguous gene syndrome refers to genetic disorders
that mimic single gene disorders. They result from deletion of
a small number of tightly clustered genes. Because some are
too small to be detected cytogenetically, they are termed as
microdeletion syndromes
The important microdeletion syndromes are:
1. Wilms’ tumor – Aniridia complex (WAGR 11p 13
syndrome)
2. Retinoblastoma 13q 14.11
3. Prader-Willi syndrome 15q11-13
4. Angelman’s syndrome 15q11-13
5. DiGeorge’s syndrome/Velo-cardiofacial 22q 11
syndrome
6. Miller-Dieker syndrome 17p 13

Deletions involving the long arm of chromosome 22 (22q 11) are the most
common microdeletions identified to date.

Note: Important microduplication syndromes include Beckwith-


Wiedemann syndrome (11p 15) and Charcot- Marie-Tooth syndrome type
IA (17 p 11.2)

12. Ans. (d) It transports iron for erythropoiesis


(Ref: Harper 25th/775, Harrison 17th/378,815,847)
• Transport of iron for erythropoiesis is done by transferin
and not by lactoferin.
• Lactoferin is found in specific/secondary granules in
neutrophils and in many exocrine secretions and
exudates (milk, tears, mucus, saliva, bile, etc.)
13. Ans. (a) Connexins
(Ref: Harrison 17th/2479)
• Connexins are complex protein assemblies that traverse
the lipid bilayer of the plasma membrane and form a
continuous channel. A pair of connexins from adjacent
cells joins to form a gap junction that bridges the 2-4 mm
gap between the cells. These are important for
communications in neurons and glial cells.
• Adhesions are microbial surface antigens that
frequently exist in the form of filamentous projection (pili
or fimbria) and bind to specific receptors on epithelial
cell membranes.
• Integrins are a family of cell membrane glycoproteins.
These are involved in cell adhesion.
14. Ans. (b) Oil Red O
(Ref: Harsh Mohan 6th/12-13)
Lipids are detected in histopathology by the use of the following
stains:
• Oil red O: Mineral oils stain red and unsaturated fats
stain pink
• Sudan Black B: Unsaturated fats stain blue black
• Osmium tetroxide: Unsaturated fats stain brown-black
whereas saturated fats are unstained.
• Regarding other options:
PAS (Periodic Acid Schiff) stain is for carbohydrates
particularly glycogen and all mucins
• Silver Methanamine is for fungi
• Non-specific esterase (NSE) is for staining myeloblast in
patients of Acute myeloid leukemia (AML)
15. Ans. (a) Fish net IgG in dermoepidermal junction
(Ref: Robbins 8th/1192-1193, Harrison 17th/336-339)
• The inflammatory bullous lesions may be Pemphigus
vulgaris, Bullous pemphigoid and dermatitis
herpetiformis. The presentation of multiple flaccid bullae
and oral lesions in a young boy is suggestive of
Pemphigus vulgaris. An important histological finding in
pemphigus is acantholysis which is dissolution, or lysis,
of the intercellular adhesion sites within a squamous
epithelial surface. The suprabasal acantholytic blister
that forms is characteristic of pemphigus vulgaris. The
antibody in pemphigus vulgaris reacts with desmoglein 1
and 3, a component of the desmosomes that appear to
bind keratinocytes together. By direct
immunofluorescence, lesional sites show a
characteristic netlike pattern of intercellular IgG
deposits.
About other options:
• Bullous pemphigoid generally affects elderly individuals.
The bullae are tense and oral lesions are present in 10-
15% of affected individuals. The subepithelial
acantholytic blister is characteristic of bullous
pemphigoid. The antibody in bullous pemphigoid reacts
with bullous pemphigoid antigens 1 and 2 (BPAG 1 and
2) present in dermoepidermal junction. Linear IgG in
dermoepidermal junction are seen by direct
immunofluorescence. (option ‘B’).
• Dermatitis herpetiformis is characterized by urticaria and
grouped vesicles. The disease results from formation of
antibodies against gliadin and is associated with celiac
disease. By direct immunofluorescence, dermatitis
herpetiformis shows granular deposits of IgA selectively
localized in the tips of dermal papillae. (option ‘D’).
• Pemphigus foliaceous is having the antibody reacting
with desmoglein 1 alone. There is selective involvement
of superficial epidermis at the level of the stratum
granulosum. It usually affects the scalp, face, chest, and
back, and the mucous membranes are only rarely
affected. Linear IgG in dermal papillae is a feature of
Pemphigus foliaceous. (option ‘C’).
16. Ans. (b) Scleroderma
(Ref: Harrison 17th/2096)
• The girl in this case is showing Raynaud’s phenomenon
and is likely to suffer later from systemic sclerosis.
• Raynaud’s phenomenon is characterized by episodic
digital ischemia, manifested clinically by the sequential
development of digital blanching, cyanosis, and rubor of
the fingers or toes following cold exposure and
subsequent rewarming.
• Raynaud’s phenomenon is broadly separated into two
categories: The idiopathic variety, termed Raynaud’s
disease, and the secondary variety, which is associated
with other disease like scleroderma.
17. Ans. (a) Triad of diarrhea, dementia and dermatitis
(Ref: Harrison 17th/449)
• Acrodermatitis enteropathica also known as Brandt
Syndrome or Danbolt-Cross syndrome is an
autosomal recessive metabolic disorder affecting the
uptake of zinc, characterized by periorificial and acral
dermatitis, alopecia and diarrhea.
18. Ans. (b) Zinc deficiency causes pulmonary fibrosis
(Ref: Harrison 17th/449)
Element Deficiency Toxicity
Calcium Reduced bone mass, Renal insufficiency (milk-
osteoporosis alkali syndrome),
nephrolithiasis, impaired iron
absorption
Selenium Cardiomyopathy, heart General: Alopecia, nausea,
failure, striated muscle vomiting, abnormal nails,
degeneration emotional lability, peripheral
neuropathy, lassitude, garlic
odor to breath, dermatitis
Occupational: Lung and
nasal carcinomas, liver
necrosis, pulmonary
inflammation
Zinc Growth retardation, General: Reduced copper
altered taste and smell, absorption, gastritis,
alopecia, dermatitis, sweating, fever, nausea,
diarrhea, immune vomiting
dysfunction, failure to Occupational: Respiratory
thrive, gonadal atrophy, distress, pulmonary fibrosis
congenital malformations

19. Ans. (a) Thymoma


(Ref: Harrison 17th/89, Devita 6th/1023)
• Tumors made up of two different lineage of cells, i.e.
lymphocytes and epithelial cells suggests the diagnosis
of thymoma.
• Thymoma is the most common cause of an anterior
mediastinal mass in adults, accounting for ~40% of all
mediastinal masses. Thymomas are most common in
the fifth and sixth decade. Some 90% of thymomas are
in the anterior mediastinum. Thymomas are epithelial
tumors and all of them have malignant potential. They
may have a variable percentage of lymphocytes within
the tumor, but genetic studies suggest that the
lymphocytes are benign polyclonal cells. The epithelial
component of the tumor may consist primarily of round
or oval cells derived mainly from the cortex or spindle-
shaped cells derived mainly from the medulla or
combinations thereof.
20. Ans. (d) Dense core granules
(Ref: Robbin’s 7th/769; Devita 6th/900)
• Paraganglioma is a neuroendocrine tumor and like other
neuroendocrine tumors, the ultrastructure shows
dense core granules (neurosecretory granules)
• The tumor cells are separated by fibrovascular stroma
and surrounded by sustentacular cells.
• Chief cells are neuroendocrine cells and are positive
for regular neuroendocrine markers, e.g.
– Chromogranin
– Synaptophysin
– euron specific enolase
– Serotonin
– Neurofilament
• The chief cells are S-100 protein negative but the
sustentacular cells are S-100 positive and are focally
positive for glial fibrillary acid protein.
• Paraganglioma cells are never positive for
cytokeratin like other neuroendocrine tumors.
21. Ans. (a) Neutral endopeptidase
(Ref: Ganong 22nd/462, , Harrison’s 17th/233,2146,2103)
• Brain natriuretic peptide (BND) or B type natriuretic
peptide is a hormone produced by the ventricles of the
heart. It has been shown to increase in response to
ventricular volume expansion and pressure overload.
• BNP is a marker of ventricular systolic and diastolic
function.
– BNP also has a prognostic significance in systemic
sclerosis (with pulmonary artery hypertension) and
in amyloidosis involving the heart.
• Atrial natriuretic peptide (ANP) is a hormone released
by atrial walls of the heart when they become stretched.
Because in heart failure, there is almost always
excessive increase in both the right and left atrial
pressures that stretch the atrial walls the circulating
levels of ANP in the blood increase fivefold to tenfold in
severe heart failure. The ANF in turn has a direct effect
on the kidneys to increase greatly their excretion of salt
and water. Therefore ANP plays a natural role to prevent
the extreme congestive symptoms of cardiac failure.
• Both ANP as well as BNP are metabolized by neutral
endopeptidases and the inhibitors of this enzyme
(omapatrilat and sampatrilat) are used for the
management of CHF.
22. Ans. (a) There are many cyclin inhibitors which prevent
cell to enter into S phase in adult
(Ref: Robbins. 7th/42, 43, 308, 309)
• After a fixed number of divisions, normal cells become
arrested in a terminally non dividing state known as
replicative senescence. It occurs due to shortening of
telomeres.
• Telomeres are short repeated sequences of DNA
present at the linear ends of chromosomes that are
important for ensuring the complete replication of
chromosomal ends and protecting chromosomal
terminals from fusion and degradation. When somatic
cells replicate a small section of the telomere is not
duplicated and telomeres become progressively
shortened. The loss of telomere function leads to
activation of p53 dependent cell cycle checkpoints
causing proliferative arrest or apoptosis.
• Germ cells, some stem cells and cancer cells
continue to divide because in these cells telomere
shortening is prevented by sustained function of the
enzyme telomerase that maintains the length of the
telomere by nucleotide addition.
23. Ans. (d) Osteosarcoma
(Ref: Robbin’s 7th/500, 8th/475)
• Most of the malignant pediatric neoplasms are unique in
many respects:
– They tend to have a more primitive (embryonal)
rather than pleomorphic-anaplastic microscopic
appearance, are often characterized by sheets of
cells with small, round nuclei, and frequently exhibit
features of organogenesis specific to the site of
tumor origin. Because of this latter characteristic,
these tumors are frequently designated by the
suffix -blastoma, for example, nephroblastoma
(Wilms’ tumor), hepatoblastoma, and
neuroblastoma.
– Owing to their primitive histologic appearance,
many childhood tumors have been collectively
referred to as small round blue cell tumors.
– The differential diagnosis of such tumors includes
– Neuroblastoma
- Wilms’ tumor
- Lymphoma
- Rhabdomyosarcoma
- Ewing sarcoma/Primitive neuroectodermal tumor.
24. Ans. (b) Vascular
(Ref: Harrison 17th/1186-1187, Robbins 7th/548, 550)
25. Ans. (c) Gout
(Ref: Robbins 8th/1243-1246)
Tophi are formed by large aggregations of urate crystals. They are
surrounded by macrophages, lymphocytes and foreign body
giant cells. They are characteristic of gout.
They are seen in the
• Articular cartilage of joints*
• Periarticular ligaments*
• Tendons and soft tissues*
• Achilles tendon*
• Ear lobes*
Other important points
Most common joint involved in Gout is Big Toe (First
metatarsophalangeal joint)
• The diagnosis is made by presence of monosodium
urate crystal in polarized light which are needle shaped
and strongly negative birefringent crystal.
26. Ans. (c) Mitochondrial myopathy
(Ref: Robbins 7th/33, 1342, Harrison’ 16th/2534, 374)
27. Ans. (a) Alport’s syndrome; (b) Niemann-Pick disease; (c)
Atherosclerosis
(Ref: Robbin’s 7th/523, 988 ,163)
• Foam cells are lipid laden phagocytes. In Niemann-Pick
disease, they are widely distributed in spleen, liver,
lymph nodes, bone marrow, and tonsils.
• During atherosclerosis, oxidized LDL is ingested by
macrophages forming foam cells.
• In Alport’s syndrome, interstitial cells of kidney may
acquire a foamy appearance owing to accumulation of
neutral fats and mucopolysaccharides forming foam
cells.
28. Ans. (a) Formalin
(Ref: Harsh Mohan 6th/276)
Formalin is the best tissue fixative.
29. Ans. (d) All of the above
(Ref: Robbins 7th/1265, 8th/1199)
• Panniculitis is an inflammatory reaction in the
subcutaneous fat that may affect principally the
connective tissue septa separating lobules of fat or
predominantly the lobules of fat themselves.
• The various forms of panniculitis are:
– Erythema nodosum: Most common form of
panniculitis and usually has an acute
presentation. Its occurrence is often associated
with infections (β-hemolytic streptococci, TB and
less commonly, coccidiodomycosis, histoplasmosis
and leprosy), drug administration (sulfonamides,
oral contraceptives), sarcoidosis, inflammatory
bowel disease, and certain malignant neoplasms.
– Erythema induratum: Uncommon type of
panniculitis that affects primarily adolescents and
menopausal women. It is a primary vasculitis
affecting deep vessels with subsequent necrosis
and inflammation within the fat. There is no
associated underlying disease.
– Weber-Christian disease (relapsing febrile
nodular panniculitis): It is a rare form of lobular,
nonvascular panniculitis seen in children and
adults.
– Factitial panniculitis: It is a result of self-inflicted
trauma or injection of foreign or toxic substances, is
a form of secondary panniculitis.
– Lupus erythematosus may occasionally have deep
inflammatory components with associated
panniculitis.
30. Ans. (a) Measles
(Ref: Robbins 7th/364)
31. Ans. (c) The etiological agent for the initial stimulus to the
ultimate expression of disease
(Ref: Robbins 7th/4)
Pathogenesis refers to the sequence of events in the response of
cells or tissues to the etiological agent, from the initial
stimulus to the ultimate expression of the disease”.
32. Ans. (b) Frequently gives rise to embolization in the
lymphatics
(Ref: Robbins 7th/1318)
Liposarcomas are one of the most common sarcomas of adulthood
and are uncommon in children. They usually arise in the
deep soft tissues of the proximal extremities and
retroperitoneum and are notorious for developing into large
tumors. Histologically, liposarcomas can be divided into well-
differentiated, myxoid, round cell, and pleomorphic variants.
The cells in well-differentiated liposarcomas are readily
recognized as lipocytes. In the other variants, some cells
indicative of fatty differentiation called lipoblasts are almost
always present. The myxoid and round cell variant of
liposarcoma has a t(12;16) chromosomal abnormality.
33. Ans. (c) Michaelis Gutmann bodies—Langerhans
histiocytosis
(Ref: Robbins 7th/1258, 680-681, 905, 1027-1028, 701-702)
Russell bodies
Inclusions containing immunoglobulins present in the cytoplasm of
patients of multiple myeloma; similar inclusions in the nucleus are
called Dutcher bodies.
Mallory bodies
Eosinophilic cytokeratin inclusions seen in alcoholic liver disease (can
also be seen in Wilson’s disease, Indian childhood cirrhosis, chronic
cholestatic conditions, hepatocellular cancer and primary biliary
cirrhosis)
Civatte bodies
Lichen planus is a disease characterized by “purple, pruritic, polygonal
papules” and characterized histologically by dense lymphocytic
infiltrates along dermoepidermal junction. The lymphocytes are
intimately associated with basal keratinocytes which show
degeneration and necrosis contributing to saw-toothing of dermo-
epidermal junction. Anucleate, necrotic basal cells may get
incorporated into the inflamed papillary epidermis where they are called
colloid or Civatte bodies.
Michaelis Gutmann bodies
Seen in Malacoplakia (vesicle inflammatory reaction associated with E.
coli infection characterized by raised mucosal plaques and
histologically by infiltration with large, foamy macrophages having
laminated mineralized concretions of calcium inside lysosomes called
Michaelis Gutmann bodies).
Langerhan’s histiocytosis
It is a term used for proliferative disorders of dendritic cells which has
three disorders namely Letterer-Siwe syndrome, Hand-Schuller-
Christian disease and eosinophilic granuloma. The presence of
Birbeck’s granules in the cytoplasm is a characteristic feature. These
granules have a rod-like structure and terminal dilated ends (Tennis
racket appearance)

34. Ans. (c) Osteosarcoma


(Ref: Robbins 7th/299)
• Patients with familial retinoblastoma are also at greatly
increased risk of developing osteosarcoma and some
other soft tissue sarcomas (Robbins pg 299)
• Alterations in ‘RB pathway’ involving INK 4a proteins,
cyclin D-dependent kinases and RB family proteins
which are present in normal cells lead on to inactivation
of tumor suppressor gene (CRB gene) and associated
somatic/inherited mutations cause the increased risk of
other tumors.
35. Ans. (d) Adrenals
(Ref: Arch Ophthal. 1939; 22(4):575-580)
Adrenal neuroblastomas are malignant neoplasms arising from the
sympathetic neuroblasts in the medulla of the adrenal gland.
There are two clinical types, based on the differences in
distribution of metastasis. First (Pepper type) occurs in the
stillborn and in young infants and metastasizes to the liver
and regional lymph nodes, then the lungs, and late in the
course, the calvarium and other flat bones. The second
(Hutchinson) type is characterized clinically by secondary
growth in the orbit, meninges, skull and long bones and
occurs in children up to 15 years of age.
36. Ans. (b) Ependymoma
(Ref: Robbins 8th/1334)
37. Ans. (c) Neuroblastoma
(Ref: Robbins 8th/477)
38. Ans. (a) Atopic dermatitis
(Ref: Robbins 8th/1187-1189)
39. Ans. (a) Pleomorphic adenoma
(Ref: Robbins 8th/757)
40. Ans. (b) Mucoepidermoid carcinoma
(Ref: Robbins 8th/759; 7th/791)
41. Ans. (d) Lead poisoning
(Ref: Robbins 8th/407)
42. Ans. (a) Tumor of gingiva
(Ref: Robbins 8th/748)
43. Ans. (b) Rhabdomyoma
(Ref: Robbins 8th/584)
44. Ans. (b) Dermatitis, glossitis, alopecia
(Ref: Harsh Mohan 6th/254)
45. Ans. (b) Lead poisoning
(Ref: Robbins 8th/406-407)
46. Ans. (b) Rhabdomyoma
(Ref: Robbins 8th/584)
47. Ans. (a) Parotid gland
(Ref: Robbins 8th/757)
48. Ans. (d) Antibodies in serum
(Ref: Robbins 8th/653)
49. Ans. (a) Measles
(Ref: Robbins 8th/432-433)
50. Ans. (d) Clostridia
(Ref: Robbins 8th/378-379)
51. Ans. (d) Neurofibroma
(Ref: Robbins 8th/730)
52. Ans. (c) SLE
(Ref: Robbins 8th/220)
53. Ans. (b) Amyloid
(Ref: Robbins 8th/253)
54. Ans. (c) Basal cell carcinoma
(Ref: Robbins 8th/1180)
55. Ans. (c) Pleomorphic adenoma
(Ref: Robbins 8th/261)
56. Ans. (c) Chancroid
(Ref: Robbins 8th/366)
57. Ans. (d) All
(Ref: Robbins 8th/1191-1192)
58. Ans. (d) Diplopia
(Ref: Robbins 8th/438)
59. Ans. (a) Liver
(Ref: Robbins 8th/273; 7th/923)
60. Ans. (a) Weibel-Palade bodies
(Ref: Robbins 8th/990)
61. Ans. (c) Phototherapy

(Ref: Harrison, Chapter 74. Vitamin and Trace Mineral


Deficiency and Excess)
Niacin deficiency causes pellagra which can be due to the
following reasons:

People eating corn-based diets


• In alcoholics
• In congenital defects of intestinal and kidney absorption
of tryptophan like Hartnup disease
• In patients with carcinoid syndrome
Contd...

HEMATOXYLIN AND EOSIN (H & E)

This is the most commonly used stain in routine pathology.


Hematoxylin, a basic dye stains acidic structures a purplish blue.
Nuclei (DNA), ribosomes and rough endoplasmic reticulum (with
their RNA) are therefore stained blue with H&E. Eosin, in contrast
is an acidic dye which stains basic structures red or pink. Most
cytoplasmic proteins are basic and therefore stained pink or
pinkish red. In summary, H&E stains nuclei blue and cytoplasm
pink or red.

PERIODIC ACID-SCHIFF (PAS)

This stain is versatile and has been used to stain many structures
including glycogen, mucin, mucoprotein, glycoprotein, as well as
fungi. PAS is useful for outlining tissue structures—basement
membranes, glomeruli, blood vessels and glycogen—in the liver.

ROMANOWSKY STAINS

These histology stains are used for blood and bone marrow.
Examples of Romanowsky histology stains include Wright’s stain,
Giemsa stain and Jenner’s stain. These histology stains are based
on a combination of eosin and methylene blue.

SILVER STAINS
These histology stains use silver. Argyrphilic tissue has an affinity
for silver salts. The silver salts will be seen in argyrphilic tissues.
Silver histology stains are used to show melanin and reticular
fibers.

SUDAN STAINS

Sudan histology stains are used for staining of lipids and


phospholipids. Examples of such histology stains are Sudan black,
Sudan IV, and oil red O.
TYPE OF STAIN Used for staining
Acid Fast Stain Mycobacterial Organisms and other
Acid Fast Organisms
Aldehyde Fuchsin Pancreatic Islet Beta Cell Granules
Alician Blue Mucins and Muco-substances
Alizarin Red S Calcium
Bielschowsky Stain (Uses Silver) Reticular Fibers, Neurofibrillary Tangles
and Senile Plaques
Cajal Stain Nervous Tissue
Congo Red Amyloid
Cresyl Violet (Nissl Stain) Neurons and Glia
Fontana Masson’s Melanin and Argentaffin Cells
Giemsa Bone Marrow
Golgi Stain Neurons
Gomori Methenamine Silver Fungi
(GMS)
Gram Stain (Taylor’s) Bacteria
Hematoxylin & Eosin (H&E) General Stain Used in Routine
Pathology
Luna Stain Elastin and Mast Cells
Luxol Fast Blue (LFB) Myelin
Masson’s Trichrome Connective Tissue, Collagen
Mucicarmine Epithelial Mucin
Oil-Red-O (On Frozen Sections) Lipid
Orcien Stain Elastin fibers
Osmium Tetroxide Lipids
Periodic Acid-Schiff (PAS) Glycogen, Fungi
Phosphotungstic Acid- Fibrin, Cross Striations of Skeletal
Haematoxylin Muscle Fibres
Picrosirius Red (polarized) Collagen
Reticulum Silver Reticulum Fibres
Safranin O Mucin, Cartilage and Mast cells
Toluidine Blue Mast Cell Granules
Verhoeff Vangieson (VVG) Elastic Fibres
Von Kossa Calcium Salts

Inclusion Bodies

A. INTRA-CYTOPLASMIC

Rabies Negri bodies


Small pox Guarnieri bodies
Molluscum Contagiosum Henderson Peterson bodies
Fowl pox Bollinger bodies
Trachoma Halberstaedter- Prowazeki bodies
Contd...

B. INTRA-NUCLEAR

Cowdrey Type A
Herpes Virus Lipschutz Inclusions
Yellow fever Torres Bodies
Cowdrey Type B
Adenovirus (Basophilic)
Poliovirus (acidophilic)

C. BOTH INTRANUCLEAR AND INTRACYTOPLASMIC


Measles Virus

Other Important Bodies

Asteroid body Sarcoidosis and Sporotrichosis


Ferruginous body Asbestosis
Torres body Yellow fever
Lafora body Myoclonic epilepsy
Michaelis Gutmann body Malacoplakia
Mallory bodies Primary biliary cirrhosis
Alcoholic hepatitis
Wilson’s disease
Chronic cholestasis
Hepatocellular carcinoma
Miyagawa’s Corpuscles Buboes from LGV
Leishman Donovan Bodies Kala –Azar
Babes- Ernst Granules Corynebacterium diphtheriae
Donovan Bodies Granuloma Inguinale
Lewis Bodies Parkinsonism
Russell Bodies Multiple Myeloma
Warthin- Finkedely Giant Cells Measles
Owl-Eye Inclusions CMV and Herpes
Keratin Pearls Squamous Cell Carcinoma
Pick Body Pick’s Disease
Aschoff Bodies Rheumatic Fever
Bodies of Arantius Aortic Valve Nodules
Body of Highmore Mediastinum Testis
Bollinger Bodies Fowlpox
Brassy Body Dark Shrunken Blood Corpuscle in
Malaria
Call Exner Bodies Granulosa Theca Cell Tumor
Chromatid Bodies Entamoeba histolytica Pre-cyst
Citron Bodies Clostridium septicum
Civatte Bodies Lichen Planus
Councilman Bodies Hepatitis
Coccoid X Bodies Psittacosis
Creola Bodies Asthma
Gamma Gandy Bodies Congestive Splenomegaly
Guarnieri Bodies Vaccinia
Henderson Peterson Bodies Molluscum contagiosum
Heinz Bodies G 6 PD Deficiency
Hirano Bodies Alzheimer’s Disease
Levinthal Coles Lille Bodies Psittacosis
Mooser Bodies Endemic Typhus
Moot Bodies Multiple Myeloma
Psammoma Bodies Papillary Carcinoma of Thyroid, Ovary
and Salivary Glands,
Meningioma, Mesothelioma
Reilly Bodies Hurler’s Syndrome
Rokitansky Bodies Teratoma
Ross’s Bodies Syphilis
Rushton Bodies Odontogenic Cyst
Sclerotic Bodies Chromoblastomycosis
Sandstorm Bodies Parathyroid Glands
Schiller Duval Bodies Yolk Sac Tumor
Schaumann Bodies Sarcoidosis
Verocay Bodies Schwannoma
Winkler Bodies Syphilis
Zebra Bodies Metachromatic Leukodystrophy
NOTES
...........................................................................................................
...........................................................................................................
..........
...........................................................................................................
...........................................................................................................
..........
...........................................................................................................
...........................................................................................................
..........
...........................................................................................................
...........................................................................................................
..........
...........................................................................................................
...........................................................................................................
..........
...........................................................................................................
...........................................................................................................
..........
...........................................................................................................
...........................................................................................................
..........
...........................................................................................................
...........................................................................................................
..........
...........................................................................................................
...........................................................................................................
..........
...........................................................................................................
...........................................................................................................
..........
...........................................................................................................
...........................................................................................................
..........
...........................................................................................................
...........................................................................................................
..........
...........................................................................................................
...........................................................................................................
..........
...........................................................................................................
...........................................................................................................
..........
...........................................................................................................
...........................................................................................................
..........
...........................................................................................................
...........................................................................................................
..........
...........................................................................................................
...........................................................................................................
..........
...........................................................................................................
...........................................................................................................
..........
...........................................................................................................
...........................................................................................................
..........
...........................................................................................................
...........................................................................................................
..........
...........................................................................................................
...........................................................................................................
..........
...........................................................................................................
...........................................................................................................
..........
...........................................................................................................
...........................................................................................................
..........
...........................................................................................................
...........................................................................................................
..........
...........................................................................................................
...........................................................................................................
..........
...........................................................................................................
...........................................................................................................
..........
...........................................................................................................
...........................................................................................................
..........
...........................................................................................................
...........................................................................................................
..........
...........................................................................................................
...........................................................................................................
..........
...........................................................................................................
...........................................................................................................
..........
...........................................................................................................
...........................................................................................................
..........
...........................................................................................................
...........................................................................................................
..........
...........................................................................................................
...........................................................................................................
..........
...........................................................................................................
...........................................................................................................
..........
...........................................................................................................
...........................................................................................................
..........
...........................................................................................................
...........................................................................................................
..........
...........................................................................................................
...........................................................................................................
..........
...........................................................................................................
...........................................................................................................
..........
...........................................................................................................
...........................................................................................................
..........
...........................................................................................................
...........................................................................................................
..........
...........................................................................................................
...........................................................................................................
..........
...........................................................................................................
...........................................................................................................
..........
...........................................................................................................
...........................................................................................................
..........
...........................................................................................................
...........................................................................................................
..........
...........................................................................................................
...........................................................................................................
..........
...........................................................................................................
...........................................................................................................
..........
...........................................................................................................
...........................................................................................................
..........
...........................................................................................................
...........................................................................................................
..........
...........................................................................................................
...........................................................................................................
..........
...........................................................................................................
...........................................................................................................
..........
...........................................................................................................
...........................................................................................................
..........

You might also like